PA Easy Question Bank
Q 118.14: You are reviewing the labs of a 78-year-old male with a 4-month history of fatigue. You suspect primary hypothyroidism as your diagnosis. What laboratory findings would support your conclusions? A elevated thyroid-stimulating hormone (TSH) B low thyroid-stimulating hormone (TSH) C low parathyroid hormone (PTH) D high total triiodothyronine (T3) E high free triiodothyronine (FT4)
A The correct answer is (A). An elevated TSH and low FT 4 is characteristic of primary hypothyroidism. A low TSH and elevated T 3 or FT 4 would suggest hyperthyroidism. Low PTH suggests hypoparathyroidism. (McPhee and Papadakis, 2011, Chapter 26)
Q 47.10: You suspect that a 5-year-old patient has allergic conjunctivitis. Which of the following symptoms or signs would best support this diagnosis? A Itching B Fever C Preauricular adenopathy D Profuse discharge E Sore throat
The Correct Answer is: A Allergic conjunctivitis is characterized by itching, tearing, redness, and chemosis, with itching being uncommon in other common forms of conjunctivitis (A). Fever (B) and sore throat (E) are more likely to occur in viral or bacterial conjunctivitis. Preauricular adneopathy (C) typically occurs in viral or chlamydial conjunctivitis.
Q 105.1: What is the most common electrolyte that can effect the initiation of ventricular tachycardia? A Magnesium B Sodium C Chloride D Phosphorus E Calcium
The Correct Answer is: A Hypomagnesia and hypokalemia are the two electrolyte disorders for ventricular tachycardia. (Bashore et al., Current Medical Diagnosis and Treatment, Chapter 10)
Q 69.8: A 59-year-old female with history of non-insulin-dependent diabetes mellitus (NIDDM), hypertension, and chronic kidney disease (CKD) returns for follow-up of her labs. You note that she her GFR has decreased from 40 to 36 ml/min/1.73m 2 . What stage of chronic kidney disease is she currently? A 1 B 2 C 3 D 4 E 5
The Correct Answer is: C There are five stages of CKD. Stage 3 Chronic kidney disease is referred to as a moderately decreased GRF between 30-59 ml/min/1.73m 2 . All other choices reflect different ranges of GRF above or below stage 3. (McPhee and Papadakis, 2011, Chapter 22)
Q 97.45: You are caring for a 29-year-old G1P0 at who is pregnant with twins. She has received routine obstetrical care and her pregnancy has been uneventful to date. What is the average gestation age for twins at delivery? A 33 to 34 B 39 to 40 C 35 to 36 D 36 to 37 E 37 to 38
The Correct Answer is: D The average length of gestation for a single fetus is 40 weeks; the average age of gestation decreases with increasing number of fetuses.
Q 49.11: Which of the following is the first sign of puberty in a normal male? A appearance of axillary hair B appearance of pubic hair C deepening of the voice D enlargement of the testes
The Correct Answer is: D The first sign of pubertal development in boys is the enlargement of testicular size and occurs at the mean age of 11.6 years. Genital stages accelerate before pubic hair development, which occurs, on average, at 13.4 years of age. The deepening of the voice and the development of chest and axillary hair usually occurs in midpuberty or 2 years after the growth of pubic hair.
Q 17.4: A 73 year-old woman with a past medical history of diabetes and hypertension presents to the clinic complaining of progressive fatigue. An initial CBC reveals a hemoglobin 10.1 mg/dL and MCV of 91 fL. Which of the following best describes the cause of the abnormality pictured in the patient's peripheral smear? (note photo taken from figure 57-5- in Harrison's) A Absent or non-functioning spleen B Failure of nuclear maturation C Intravascular hemolysis D Lead intoxication E Presence of uremia
The Correct Answer is: E Burr cells are present on the peripheral smear and are associated with uremia. Howell-Jolly bodies result from the lack of removal of nuclear material due to an absent or non-functioning spleen (A). Macrocytosis results from failure of nuclear maturation (B) commonly secondary to vitamin B 12 or folate deficiency. Intravascular hemolysis (C) creates schizocytes, and lead intoxication (D) or thalassemia cause basophilic stippling.
Q 44.10: What is the most serious complication of the influenza B virus? A Multi-lobar pneumonia B Encephalitis C Reye's syndrome D Renal failure E Hepatitis
The Correct Answer is: C Reye's syndrome, while rare, is a very serious complication related to influenza B virus infections. There is a 30% mortality rate, which primarily affects the pediatric population. Hepatic failure and encephalopathy are events that occur in this fatal disease.
A 78-year-old woman with a medical history of diabetes and hypertension presents to the emergency department complaining of left hand weakness and slurred speech. Which of the following tests is most likely to determine the source of an arterial thrombus? A carotid ultrasound B CT of the brain C erythrocyte sedimentation rate D magnetic resonance angiography (MRA) of the vertebral arteries
*The Correct Answer is: A The patient's symptoms are consistent with pathology arising from the anterior cerebral circulation including the carotid arteries.* A CT should be ordered to rule out acute hemorrhage and an erythrocyte sedimentation rate may be useful if giant cell arteritis were suspected. An MRA of the vertebral arteries would likely show deficits but is not likely to demonstrate the etiologic location of this stroke.
What is the initial treatment for a patient who is having an acute episode of supraventricular tachycardia? A Caffeine B Beta blockers C Valsava maneuver D No treatment E Synchronized cardioversion
The Correct Answer is: C The initial treatment that should be attempted is a simple vagal maneuver to break the reentry tachycardia. If this is unsuccessful, pharmacologic therapy is indicated.
Q 106.18: Which of the following psychiatric conditions has the highest mortality rate? A anorexia nervosa B bulimia nervosa C depression D generalized anxiety disorder E obsessive-compulsive disorder
The Correct Answer is: A Complex biologic, psychological, and social factors determine onset and severity of eating disorders, along with mortality and morbidity. Patients with eating disorders have some of the highest rates of psychiatric comorbidities, including depression, affective disorders, anxiety disorders, obsessive-compulsive personality traits or disorder, and substance abuse. Eating disorders are risk factors for both attempted and completed suicide. Suicide attempts occur in up to 35% of patients with bulimia, but completed suicides do not appear to be elevated. *In contrast, women with anorexia have been shown to have a fifty-fold increased risk for completed suicide. Suicide is one of the leading causes of death for patients with anorexia.* (Tintinalli, et al., 2011, Chapter 286)
Q 35.2: A 1-day-old infant being examined in the newborn nursery is noted to have a central, 4 mm cataract affecting his right eye. Which of the following is the most appropriate management for this patient? A Cataract surgery within the next 6 weeks B Cataract surgery within the next year C Observation every 3 months D Observation every 6 months E Observation every year
The Correct Answer is: A Congenital cataracts that are large and affect visual acuity (e.g., central) must be surgically corrected within the first two months of life (A) to avoid the development of deprivation amblyopia. Observation (C, D, and E) or delayed surgery (B) may result in permanent deprivation amblyopia.
Q 98.65: Which of the following etiologies is the most common cause of nontraumatic subarachnoid hemorrhage (SAH)? A Cerebral aneurysm B Poorly controlled hypertension C Anticoagulant use D Arteriovenous malformation
The Correct Answer is: A Congenital cerebral aneurysms or Berry aneurysms account for 75% to 80% of nontraumatic subarachnoid hemorrhages (SAHs). Poorly controlled hypertension and anticoagulant use are more commonly associated with intracerebral hemorrhages (ICH). AVMs can cause either SAH or ICH.
Q 29.4: A father brings his son to the family practice clinic for a concern for coarctation of the aorta. The father was recently diagnosed with this disease, and he is concerned that his 5-year-old son may have it as well. What is the cardinal physical finding in coarctation of the aorta? A Decreased or absent femoral pulses B Systolic machinery-type murmur C Holosystolic murmur at the lower left sternal border D Thrill in the carotid arteries E Cyanosis at birth
The Correct Answer is: A Decreased or absent femoral pulses is the correct answer. A systolic machinery-type murmur is classic for a patent ductus arteriosus. A holosystolic murmur at the lower left sternal border is most commonly seen with a ventricular septal defect. A thrill in the carotid arteries is associated with aortic stenosis. Cyanosis at birth can occur with many different congenital anomalies, but is not seen with coarctation of the aorta.
Q 35.9: Which of the following is the most common etiologic agent of bacterial meningitis in the pediatric population of the United States? A Streptococcus pneumoniae B Haemophilus influenzae type B C Listeria monocytogenes D Neisseria meningitides
The Correct Answer is: A Despite the increase in vaccination of infants in the United States, Streptococcus pneumoniae remains the most common etiologic agent for bacterial meningitis in the pediatric population. Haemophilus influenzae type B is the second most common, but has gone down significantly due to the widespread vaccination of children. Neisseria meningitides has approximately 2,400 to 3,000 cases a year. Meningitis due to Listeria monocytogenes is typically seen in the neonatal period due to transmission from the mother. It is present in normal fecal matter in around 10% of the population. Its rates have gone down due to strict guidelines for the food industry, resulting in less than 1,000 cases per year.
Q 46.4: Which of the following are predictive or diagnostic of anorexia nervosa? A having a weight that is 85% less than predicted normal weight B homosexual orientation in females C promiscuity D menorrhagia
The Correct Answer is: A Diagnostic criteria for anorexia nervosa include a weight loss to 85% of the required body weight. Homosexual orientation in men, not women, is considered a predisposing factor. Anorexia is associated with amenorrhea and decreased interest in sex.
Patients in which of the following age groups are least likely to experience a dislocation or sprain when a significant stress is placed on their joints? A 5-10 years old B 15-20 years old C 35-40 years old D 55-60 years old E 75-80 years old
The Correct Answer is: A Dislocations and ligamentous injuries are uncommon in prepubertal children as the ligaments and joints are quite strong as compared to the adjoining growth plates. Excessive force applied to a child's joint is more likely to cause a fracture through the growth plate than a dislocation or sprain.
Q 97.106: Septic arthritis in adults younger than 30 years is usually caused by A Neisseria gonorrhea B Staphylococcus aureus C Pseudomonas aeruginosa D Streptococcus pyogenes E Salmonella species
The Correct Answer is: A In patients younger than 30 years, gonococcus is the most common cause of septic arthritis. When all patients are considered, Staphylococcus aureus is the most common cause. Patients with prevalent joint disease and intravenous drug users are especially susceptible to Staphylococcus. Pseudomonas is also a common cause of septic arthritis in intravenous drug users. Salmonella is not a common cause of joint infection
Q 97.117: A 33-year-old returns to clinic for reevaluation of her dysmenorrhea. She is a G3P3 who menstruates every 27 days. She has had a tubal ligation. She is a non-smoker. She has tried Tylenol (acetaminophen) and just completed 3 months of ibuprofen 800 mg TID x 7 days, starting 2 days before her menstrual cycle. She had only mild improvement of her symptoms. What should be the next step in management of her symptoms? A Norgestimate/ethinyl estradiol B Acetaminophen/ tramodol C Acetaminophen/ codeine D Cyclobenzaprine E Paroxetine
The Correct Answer is: A Primary dysmenorrhea is associated with ovulation, so suppression of ovulation in a woman who no longer desires fertility usually improves symptoms. Narcotics are not indicated as primary treatment in the absence of pathologic disease. Combination oral contraception will suppress ovulation.
Q 38.17: A 15-year-old obese male presents with annularly configured lesions in a generalized distribution. They are asymptomatic. What can these lesions be associated with? A Diabetes mellitus B Hyperlipidemia C Hypertension D Systemic lupus erythematosus
The Correct Answer is: A This patient presents with a generalized form of granuloma annulare (GA). Generalized GA can be associated with diabetes mellitus.
Q 97.95: A patient presents to the office at 5 weeks gestation. She has been spotting for several days, and the flow is increasing slightly. She has mild, crampy pain; no fetal heart tones are auscultated. Ultrasound reveals an intrauterine gestational sac with a fetal pole. What is the most likely diagnosis? A Threatened abortion B Spontaneous abortion C Incomplete abortion D Septic abortion E Ectopic pregnancy
The Correct Answer is: A While she is at risk for a spontaneous abortion, one cannot determine fetal viability with one exam. Serial exams are necessary. Fetal heart tones are not always present at 5 weeks.
Q 98.5: A 66-year-old woman presents to the emergency department with a complaint of abdominal pain and distension for the past 3 days. Examination reveals a protuberant abdomen with diminished bowel sounds and tympany to percussion. Flat and upright abdominal radiographs reveal distended loops of bowel with prominent haustral markings. Which of the following etiologies is the most likely cause of the patient's condition? A Volvulus B Adenocarcinoma C Diverticular disease D Strangulated hernia E Adhesions
The Correct Answer is: B Large bowel obstructions are most commonly caused by an adenocarcinoma (65%). This is followed in decreasing incidence by diverticular scarring and volvulus. Adhesions are the most common cause of small-bowel obstruction but are rare as a cause of large bowel obstruction. The presence of haustral markings on radiographic evaluation helps differentiate between small and large bowel involvement.
Q 97.62: A 25-year-old female presents for an ultrasound after having a positive home pregnancy test. She has an unremarkable past medical history and physical exam. She states she has been feeling fine without any abdominal discomfort or vaginal bleeding noted. On ultrasound you determine she is 10 weeks pregnant. You note a noncomplex unilateral mass on her left ovary measuring 2 cm in diameter. Which additional history would support your suspected diagnosis? A History of herpes simplex virus B She used clomiphene to conceive C Two previous Caesarian sections D 20-pound unintentional weight loss E Previous miscarriage at 7 weeks gestation
The Correct Answer is: B Patients who used assisted reproduction, such as clomiphene, present a special subgroup as their ovaries frequently have ovarian cysts. This is common during the first trimester due to ovarian hyperstimulation. Herpes (A), Cesarean sections (C), and miscarriages (E) are not proven to increase the risk of functional ovarian cysts. A 20-pound unintentional weight loss (D) would have you consider a malignant cause, which is much rarer than a functional ovarian cyst.
Q 97.122: A 23-year-old G1P0 presents to the office complaining of headache, nausea, swelling, and generally not feeling well. She is at 33 weeks gestation. A physical exam reveals a 5-pound weight gain in 2 weeks, BP 148/90, P 84, T 98.1°F, and UA concentrated with 1+ protein. What is the most likely diagnosis? A Pregnancy induced hypertension B Preeclampsia C Eclampsia D Gastroenteritis E Third space fluid retention
The Correct Answer is: B Preeclampsia has a classic triad of weight gain, elevated blood pressure, and proteinuria. A woman with this triad of symptoms has preeclampsia until proven otherwise.
Q 39.17: What is the peak incidence of age for a patient who presents with acute rheumatic heart disease? A <5 years old B 5 to 15 years C 20 to 35 years D 40 to 55 years E >55 years old
The Correct Answer is: B The bulk of the cases of acute rheumatic fever are within the pediatric population. It is rare in younger children, as well as in adults over the age of 40.
Q 97.8: A 29-year-old G2 P1 who is term requests induction. Her pregnancy has been uncomplicated. She has been bothered by significant, poor quality contractions, which have caused her pain and interfered with her sleep. An elective induction is considered safe when the Bishops score is greater than what number? A 6 B 7 C 8 D 9 E 10
The Correct Answer is: C A Bishop score greater than 8 is considered a positive predictor for safe delivery in a term pregnancy.
Q 9.6: A 23 year-old woman seeks guidance on foods, medications, and supplements that can help increase the amount of iron supplement. Which of the following is the best advice to provide this patient? A Co-administer with a calcium supplement B Co-administer with proton pump inhibitors C Co-administer with vitamin C D Taking with a basic beverage E Taking with food
The Correct Answer is: C An acidic achieved through presence of additional acids (C) (e.g., ascorbic acid) increases absorption of iron. Food (E), other heavy metals (A), and basic stomach environments (B, D) negatively affect the absorption of iron.
Q 38.1: What is a commonly shared feature of bipolar disorder and ADHD in pediatric patients? A disruptive B obsessed with ideas C behavior problems D impaired concentration E insomnia
The Correct Answer is: C Behavior problems are a commonly shared feature of pediatric ADHD and bipolar disease. Disruptive and impulsive behaviors are a shared feature of conduct disorder. Disruptive behavior and being annoying to others can be found with oppositional defiant disorder; impaired attention and concentration can be found in major depression. Bipolar patients are obsessed with ideas while ADHD and conduct disorder patients are not.
Q 28.5: A 53 year-old man with a past medical history of coronary artery disease, hypertension, diabetes, and allergic rhinitis is admitted to the hospital secondary to a non-ST elevation MI. On his second hospital day he develops severe thrombocytopenia with a platelet count of 17,000. Which of the following medications most likely caused his thrombocytopenia? A Atenolol B Erythromycin C Heparin D Loratadine E Metformin
The Correct Answer is: C Heparin (C) directly causes the production of heparin/PF4 antibodies that affect platelets, which leads to decreased platelet numbers, but an increased thrombosis risk. Atenolol (A), erythromycin (B), loratadine (D) and metformin (E) are not associated with thrombocytopenia.
Q 17.7: A diet high in nitrates is a significant risk factor for cancer of which of the following? A oropharynx B esophagus C stomach D pancreas E liver
The Correct Answer is: C In addition to chronic H pylori infections, dietary nitrates are a significant risk factor for gastric cancer.
Q 118.8: What is the most common pathogen that causes atypical pneumonia? A Chlamydia pneumoniae B Legionella pneumophila C Mycoplasma pneumoniae D Streptococcus pneumoniae E Hemophilus influenzae
The Correct Answer is: C Mycoplasma is the third most common pathogen of all pneumonias, and the most common atypical. Streptococcus pneumoniae is considered a typical bacterium, while Legionella and Chlamydia are not as common when compared to Mycoplasma. (Lee et al., Current Diagnosis and Treatment in Pulmonary Medicine, Section X, Pulmonary Lung Disease, Chapter 37, Viral and Atypical Pneumonia)
Q 49.17: Which of the following is indicated for treatment of oral thrush? A Prednisone B Penicillin C Nystatin D Cephalexin E Acyclovir
The Correct Answer is: C Oral candidiasis, also referred to as oral thrush, is most commonly seen in infants and immunocompromised patients. It can also occur in asthma patients being treated with a metered dose steroid inhaler. The antifungal treatment of choices is nystatin suspension. Other agents that can be used are fluconazole and clotrimazole oral troches.
Q 73.6: What is the most common bacterial pathogen isolated in otitis externa? A Group A streptococcus B Strep pneumonae C Pseudomonas aeruginosa D Hemophilus influenza E Aspergillus
The Correct Answer is: C Staphylococcus aureus and pseudomonas are the most common pathogens isolated in otitis externa. Proteus, diphtheroids, and E. coli are also causes. (Lalwani A.K., 2008, Chapter 47)
Q 118.16: What is the initial treatment for a patient who is having an acute episode of supraventricular tachycardia? A Caffeine B Beta blockers C Valsava maneuver D No treatment E Synchronized cardioversion
The Correct Answer is: C The initial treatment that should be attempted is a simple vagal maneuver to break the reentry tachycardia. If this is unsuccessful, pharmacologic therapy is indicated. (Piktel JS., Cardiac Rhythm Disturbances, Section 3, Chapter 22. From Tintinalli J et al.,Tintinalli's Emergency Medicine)
Q 83.4: A 55-year-old man patient presents with tachycardia and heart palpitations. Physical exam shows a multinodular goiter. He does not have obstructive symptoms. He has suppressed TSH and elevated T 3 and T 4 , and a thyroid scan shows multiple functioning nodules. What is the treatment of choice for this patient? A propylthiouracil B beta-blockers C 131I ablation D surgical resection
The Correct Answer is: C The treatment of choice for multinodular goiter is 131 I ablation. In patients with very large thyroid glands with obstructive symptoms, surgical resection may be the best option. (Herman-Bonert, 2007, p. 651) Herman-Bonert VS. Hypothalamic-pituitary axis. In: Andreoli TA , Carpenter CC , Griggs RC, et al., eds. Cecil Essentials of Medicine. 7th ed. Philadelphia, PA: WB Saunders; 2007.
Q 97.116: A 40-year-old female is status post a dilatation and curettage for hydatidiform mole. On week 3 post surgery, her follow-up quantitative hCG level has elevated slightly. What is the most likely diagnosis? A Adenocarcinoma of the ovary B Adenocarcinoma of the uterus C Retained hydatidiform mole D Choriocarcinoma E Corpus luteum cyst
The Correct Answer is: D 5% of hydatidiform mole progress to choriocarcinoma; the longer the mole in intrauterine the higher the risk. Pre-surgical evaluation for mole removal includes a chest x-ray to rule out distant metastasis. hCg that either plateuas or elevates is choriocarcinoma until proven otherwise, and requires prompt evaluation.
Q 98.25: Following a total thyroidectomy for papillary carcinoma, a 72-year-old man develops a heart rate of 140 and a temperature of 104.8F. He vomits almost continuously and has severe diarrhea. He is disoriented and mildly combative. Electrocardiography demonstrates sinus tachycardia. What is the most likely diagnosis? A pulmonary embolism B pneumonia C sepsis D thyroid storm E wound infection
The Correct Answer is: D Although rare, thyroid storm or crisis can occur following thyroid surgery, administration of radioactive iodine, or a stressful illness. It is characterized by extreme tachycardia, vomiting, diarrhea, dehydration, delirium, and high fever. Pulmonary embolism (A) may include tachycardia, along with chest pain and shortness of breath, but is unlikely to include GI symptoms. Pneumonia (B) and sepsis (C) are certainly possible, although the extreme nature of her findings more strongly suggest thyrotoxicosis. Wound infection (E) would more likely to present with localized tenderness and a less marked fever.
A patient who is intoxicated presents to the emergency department. On ocular exam, you notice mydriasis. Which substance could he have been using? A sedatives B PCP C opioids D cocaine
The Correct Answer is: D Patients intoxicated with cocaine present with mydriasis. In opioid intoxication, the pupils are constricted. PCP intoxication is associated with nystagmus.
Q 97.3: An 18-year-old G1P0 presents for evaluation of her amenorrhea. Her LMP was 2 months ago. Her HCG is positive, and she has been having vaginal spotting x 2 weeks. Her ultrasound reveals an intrauterine heterogeneous echogenic mass, without fetus or placenta. What is the most likely diagnosis? A Threatened abortion B Missed abortion C Ectopic pregnancy D Hydatidiform mole E Early intrauterine pregnancy
The Correct Answer is: D The classic signs for a mole are a heterogenous mass without the placenta or fetus, and vaginal spotting is present all of the time. The presence of the mass intrauterine without fetus or placenta rules out ectopic and IUP, and therefore threatened abortion. Missed abortion is not echogenic.
Q 34.10: A 12-year-old girl presents with complaints of intermittent pain and stiffness involving her hands. This pain has been progressively worsening over the past 3 years. She relates that for the past 2 months she has been feeling increasingly tired and has experienced swelling and stiffness of her hands, which appears worse in the morning and is relieved as the day progresses. The physical examination shows she has a low-grade fever. There are multiple symmetrical joint swelling of the proximal interphalangeal and metacarpophalangeal joints with associated warmth, tenderness, and effusion. Initial laboratory findings include a CBC that reveals mild anemia, an elevated erythrocyte sedimentation rate, a positive rheumatoid factor, and a negative antinuclear antibody (ANA) test. X-rays of the hands and wrists show soft tissue swelling and periarticular osteopenia. Which of the following is the most likely diagnosis? A reactive arthritis B infectious arthritis C systemic juvenile rheumatoid arthritis D polyarticular juvenile rheumatoid arthritis E pauciarticular juvenile rheumatoid arthritis
The Correct Answer is: D The most likely diagnosis in this patient is polyarticular juvenile rheumatoid arthritis (JRA). This form of JRA is seen in approximately 35% of patients with JRA. It is characterized by symmetrical involvement of five or more joints. Two subsets of the disease exist that are distinguished by the presence or absence of rheumatoid factor. A positive rheumatoid factor is most commonly seen in girls with later disease onset (at least 8 years old). An antinuclear antibody (ANA) test may be positive but is more likely to be positive with the pauciarticular form. In the early stage of the disease, the x-ray may be normal or show soft tissue swelling and periarticular osteopenia. In addition to the positive ANA of pauciarticular JRA patients, the arthritis must be present in four or fewer joints. Early onset disease is commonly seen in girls aged 1 to 5 years and has a positive ANA; up to 30% of patients will also have eye involvement. Late onset disease is more common in male patients, with involvement of the large joints. Systemic JRA, also known as "Still disease," is seen in about 10% to 15% of children with JRA. It is characterized by daily intermittent fever spikes and a transient, nonpruritic, pale pink, blanching macular, or maculopapular rash found on the trunk. A positive rheumatoid factor is rare in this form of JRA. Reactive arthritis is usually associated with a recent viral or bacterial infection. Infectious arthritis more commonly presents as monarticular and is usually acute in onset.
Q 16.1: A 17-year-old female presents to your family practice office for the annual physical examination required by her cheerleading coach. Upon examination you note that her joints are more flexible than anticipated. You also note her long thin fingers. You listen to her heart and hear no murmurs. Her blood pressure is 105/65 mmHg, pulse 60 beats/min and regular, respirations of 15 breaths/min, and temperature 98.7˚F. As you are examining her she tells you that her "joints sprain and strain easily." Furthermore, you obtain family history and she tells you that some connective tissue disorder runs in her family. Before you can medically clear her you should do which of the following? A This patient does not require any further evaluation. B Obtain an echocardiogram. C Perform and EKG. D Perform a chest radiograph. E Refer her to a rheumatologist.
The Correct Answer is: B This patient has clear signs and symptoms that are suspicious for Marfan syndrome. The complications of Marfan syndrome include cardiovascular issues, especially valvular and aortic disease. An echocardiogram is an appropriate, non-invasive initial first-step to begin your investigation to rule out significant valvular and/or aortic root abnormalities.
Q 39.9: An 18 month infant with congenital heart disease is diagnosed with acute bronchiolitis secondary to respiratory syncytial virus. Which of the following therapies should be initiated? A Albuterol B Amoxicillin C Azithromycin D Prednisone E Ribavirin
The Correct Answer is: E High-risk infants (i.e., congenital heart disease) who develop RSV are eligible for treatment with ribavirin. Antibiotics (B, C) are indicated if secondary bacterial infections develop. Albuterol (A) and prednisone (D) haven't been shown to improve RSV-related bronchiolitis.
Q 64.10: A 57-year-old woman underwent a hemicolectomy for adenocarcinoma of the colon. Which of the following recommendations is part of postoperative monitoring for a potential recurrence? A Annual fecal occult blood testing B Annual chest radiograph C Annual CA 19-9 testing D Annual flexible sigmoidoscopy E Annual colonoscopy
The Correct Answer is: E Routine follow-up after surgical resection of a colon cancer includes annual colonoscopy not sigmoidoscopy, which only assesses the distal colon. The tumor marker for colon cancer is carcinoembryonic antigen (CEA) not carbohydrate antigen 19-9 (CA 19-9), which is used for pancreatic cancer. There is no role for annual chest films or fecal occult blood testing to monitor for a recurrence.
Q 56.9: A 22-year-old woman with a history of scoliosis presents to the office with complaints of retrosternal chest discomfort, occurring frequently at rest and lasting for several hours at a time. She is currently experiencing this chest discomfort during the office visit, but has never experienced this while working out three days per week. On physical exam, a mid-systolic click is noted. With standing, the click moves closer to S1. An EKG demonstrates normal sinus rhythm at 76 bpm, with no acute ST or T wave changes. A transthoracic echocardiogram reveals mild prolapse of the posterior leaflet of the mitral valve. Which of the following would be the most appropriate next step in the management of this patient? A Reassurance and monitoring with periodic transthoracic echocardiogram B Cardiac catheterization C Infectious endocarditis prophylaxis D Mitral valve replacement E Transesophageal echocardiogram
The Correct Answer is: A Choice A, reassurance and monitoring with periodic transthoracic echocardiogram, is the most appropriate choice given the patient's findings on echocardiogram. Most patients with mitral valve prolapse are asymptomatic, and do not demonstrate significant progression of their valvulopathy over their lifetime. Periodic transthoracic echocardiogram allows a noninvasive, highly sensitive method of monitoring. Choice B, cardiac catheterization, is useful for evaluation of coronary artery anatomy and for evaluation of valvulopathy; however, it is invasive, and usually reserved for investigation of serious valvular dysfunction, and/or following a stress test suggestive of myocardial ischemia. The chest pain experienced by patients with mitral valve prolapse is varied in presentation, and in this setting, with a young patient with no cardiac risk factors, unlikely to be secondary to coronary artery disease. According to the American Heart Association's most recent guidelines, patients with mitral valve prolapse alone do not require infectious endocarditis prophylaxis, so choice C is inappropriate. Choice D, mitral valve replacement, is only indicated with severe mitral valve prolapse, resulting in severe mitral regurgitation. Choice E, transesophageal echocardiogram, while an excellent diagnostic tool for the evaluation of mitral valve disease, would be more invasive than monitoring via transthoracic echocardiogram, and thus would not be an appropriate choice in this patient with only mild mitral valve prolapse.
Q 62.6: What type of pharmacological agent would be used as a first-line medication to treat obsessive compulsive disorder? A risperidone B lonazepam C fluvoxamine D trazadone E venlafaxine
The Correct Answer is: C Fluvoxamine, paroxetine, and sertraline are all approved for the treatment of obsessive compulsive disorder. Use of an SSRI in combination with behavioral therapy is recommended.
Q 55.2: A patient who is an Ashkenazi Jew and is homozygous for Factor XI deficiency requires abdominal surgery. Which hemostatic laboratory testing outcomes would you expect to find? A Decreased platelets, normal aPTT, prolonged PT B Decreased platelets, prolonged aPTT, normal PT C Normal platelets, normal aPTT, prolonged PT D Normal platelets, prolonged aPTT, normal PT E Normal platelets, prolonged aPTT, prolonged PT
The Correct Answer is: D Patients with factor XI deficiency have a prolonged aPTT, normal PT, and normal platelets count.
Q 55.6: A 26-year-old woman has decreased appetite, weight gain, cold intolerance, hoarse voice, constipation, and arthralgias. What is the most likely etiology of her condition? A autoimmune thyroiditis B congenital hypothyroidism C dietary iodine deficiency D surgical resection of the thyroid gland
The Correct Answer is: A Auto immune thyroiditis is the most common cause of hypothyroidism in the United States. Dietary iodine deficiency is the most common cause in underdeveloped regions of the world.
Q 54.19: A 22-year-old recent immigrant from Vietnam, who is 28 weeks pregnant with her first child, presents to the emergency department with complaints of worsening dyspnea and lower extremity edema. She is unable to answer definitively whether or not she has a history of rheumatic fever. On physical examination, a possible opening snap, loud S 1 , and a very soft diastolic rumbling murmur is auscultated. When the patient is placed in the left lateral decubitus position, the murmur is accentuated, and heard best at the apex. With inspiration, the murmur does not increase in amplitude. On echocardiogram, mitral stenosis is noted. Which of the following is the most appropriate next step in management of this patient? A Beta blockade and support stockings B ACE inhibitor C Mitral valve replacement D Cardiac catheterization E Increased sodium intake to maintain fluid volume
The Correct Answer is: A Choice A is the most appropriate next step in the management of this pregnant patient with mitral stenosis. In pregnancy, blood volume, cardiac output, and heart rate are increased. In pregnant patients with mitral stenosis, this increases the pressure across the mitral valve and can lead to pulmonary edema. The use of appropriate beta blockade is helpful for decreasing the heart rate, and may be used in conjunction with digoxin if the patient develops atrial fibrillation, a common dysrhythmia in this patient population. Support stockings are helpful in preventing venous pooling in the lower extremities, which can lead to large fluctuations in hemodynamics. Choice B, ACE inhibitors, are contraindicated in pregnancy, secondary to the teratogenic effects. Choice C, mitral valve replacement, is indicated only if medical management is insufficient to prevent congestive heart failure, a serious consideration in this valvulopathy, which is the most likely to cause death in pregnancy. Choice D, cardiac catheterization, is used for evaluation of coronary artery disease, but is not indicated in this young patient with few risk factors for coronary artery stenosis. Choice E, increased sodium intake to maintain fluid volume, would worsen the pressure across the mitral valve, and thus is incorrect, as avoidance of fluid overload is one of the keys to management of a pregnant patient with mitral stenosis, especially in the latter stages of pregnancy. Patients who are well-controlled throughout pregnancy, with medical management, are still at risk during labor and delivery, when large fluctuations in hemodynamics will occur.
Q 59.5: You are providing care to a woman who is at 33 weeks gestation. Her pregnancy is complicated by gestational diabetes. She is being provided education by the dietician, and has weekly obstetrical appointments. What fasting blood sugar (FBS) readings should necessitate switching from diet control to insulin therapy? A FBS > 70 mg/dL B FBS > 90 mg/dL C FBS > 95 mg/dL D FBS > 110 mg/dL E FBS > 126 mg/dL
The Correct Answer is: C Gestational diabetes has different risks associated with it, in particular for the fetus. Stricter glycemic controls are recommended for pregnant versus non pregnant women by the ACOG and the ADA.
Q 12.9: Chronic NSAID use is a known cause of peptic ulcer disease. Which of the following NSAIDs is the least likely to lead to ulcer formation? A Aspirin B Ibuprofen C Naproxen D Celecoxib E Diclofenac
The Correct Answer is: D Celecoxib is the only selective NSAID listed in the choices. Coxibs preferentially inhibit COX-2, the principle enzyme involved at sites of inflammation, while sparing COX-1. COX-1 is involved in mucosal cytoprotection in the stomach and duodenum. All the other choices listed are nonselective NSAIDs, and inhibit both COX-1 and COX-2 enzymes.
Q 61.10: You are evaluating a 59-year-old male with history of heavy alcohol use for the past 5 years who appears acutely ill. Which of the following findings is most likely to be an indicator of increased mortality in this patient with severe alcoholic hepatitis? A fatty liver B INR of 7 C AST > ALT by a factor of 2:1, but less than 300 units/L D serum bilirubin 1.8mg/dl E folic acid deficiency
B The correct answer is choice (B). A significantly elevated PT/INR as well as bilirubin > 10 mg/dl are indicators of severe alcoholic hepatitis and increased mortality requiring hospitalization. Choices (A), (C), and (E) are all common findings in alcoholic hepatitis. Mild elevation of serum bilirubin, choice (D), is also common.
Q 32.27: Out of all cervical vertebrae, which two are responsible for the greatest amount of rotation? A C1 & C2 B C2 & C3 C C3 & C4 D C4 & C5 E C5 & C6
The Correct Answer is: A Approximately 50% of cervical rotation takes place between the C1 (atlas) and C2 (axis) vertebrae. These first two cervical vertebrae have a different shape from the other cervical vertebrae that allow for this greater range of motion. The remaining 50 % of cervical rotation is split fairly evenly between the remaining vertebrae. Approximately 50 % of flexion and extension occurs between the occiput at the base of the skull and C1 with the remaining 50% distributed fairly evenly between the remaining vertebrae with a slightly higher percentage occurring at the C5 & C6 level.
Q 43.10: A 9-year-old child, who was diagnosed with a viral upper respiratory infection 2 weeks ago, returns to the clinic with a complaint of a 2-day history of drooping of one side of her mouth. She is afebrile with a blood pressure of 110/60 mm Hg. Her physical examination reveals an inability to completely close her left eye, inability to wrinkle her forehead, and the drooping of her mouth on the left side. Her smile is asymmetric. The remainder of her examination is otherwise normal. Which of the following is the MOST likely diagnosis? A Bell's palsy B botulism C brainstem glioma D Guillain-Barré syndrome
The Correct Answer is: A Bell's palsy is the acquired peripheral facial weakness (cranial nerve VII) of sudden onset and unknown etiology. It often follows a viral illness with notable improvement within 2 weeks and near complete recovery within 2 months. Prednisone therapy may promote recovery of facial strength. Guillain-Barré syndrome (acute idiopathic polyneuritis) generally presents with symmetrical weakness of the lower extremities, which may ascend rapidly to the arms, trunk, and face. Nonspecific respiratory or gastrointestinal symptoms may occur 5 to 14 days preceding the infection. Physical examination will yield symmetric flaccid weakness, which is usually proximal in distribution. Rarely, there is cranial nerve (III-VI, IX-XI) involvement. Botulism is most often caused by the ingestion of food containing the Clostridium botulinum toxin or rarely from an infected wound. Children will present with blurred or double vision, ptosis, or choking. Physical findings include a weak swallow paralysis of accommodation and eye movements. In this case, there was not a history of food ingestion or wound infection to support this diagnosis. Children with a brain stem tumor may present with facial and extraocular muscle palsies, hemiparesis, gait disturbances, and hydrocephalus (25%). Changes in personality such as lethargy, irritability, and aggressive behavior are particularly common findings. Speech and swallowing difficulties are not unusual. Later in the illness, patients will develop vomiting and headaches.
Q 15.1: A 64-year-old female presents for an annual examination. She is 5'0" and weighs 92 pounds; compared to her examination 3 years ago she has lost an inch in height. After performing a dual-energy x-ray absorptiometry (DXA) scan the diagnosis of osteoporosis is confirmed. Which of the following supplements should be recommended? A Calcium carbonate 1,000 mg and vitamin D 800 international units daily B Calcium carbonate 500 mg and vitamin D 2,500 international units daily C Calcium carbonate 2,000 mg and vitamin D 500 international units daily D Calcium carbonate 800 mg and vitamin D 100 international units daily
The Correct Answer is: A Calcium supplementation provides beneficial effects on bone mass throughout postmenopausal life and may reduce fracture rates up to 50%. Postmenopausal women receiving supplemental calcium over a 3-year period in a placebo-controlled, randomized clinical trial had stable total body calcium and BMD in the lumbar spine, femoral neck, and trochanter compared with the placebo group. Recommendations for calcium carbonate are 1,000-1,500 mg once daily orally. Vitamin D increases calcium absorption in the gastrointestinal tract, making calcium more available for reabsorption and circulation. Recommendations for Vitamin D are 800-2,000 international units once daily orally. Calcium Carbonate less than 1,000 mg per day is not sufficient (B and D). Vitamin D levels less than 800 IU per day is not sufficient (C and D).
Q 101.10: What is the treatment of choice for a patient who is HIV positive and has a confirmed pneumonia caused by Pneumocystosis jiroveci (as shown above)? A trimethoprim-sulfamethoxazole B Azithromycin C Aztreonam D Cephalexin E Erythromycin
The Correct Answer is: A For a confirmed bacteria or suspected case of pneumonia caused by Pneumocystosis jiroveci, TMP-SMX is the antibiotic of choice for this patient. (Shelburne SA, Hamill RJ. Medical Diagnosis and Treatment, 2011, Chapter 36, Mycotic Infections)
Q 82.6: A 45-year-old female presents with a rash that started on her ankles two days ago and is spreading up her leg. The lesions are asymptomatic; however, she has abdominal pain and joint pain. The lesions are palpable and non-blanchable when compressed. What is the most likely diagnosis? A Henoch-Schonlein purpura B Polyarteritis nodosa C Pityriasis rosea D Psoriasis vulgaris
The Correct Answer is: A Henoch-Schonlein Purpura (HSP) is a hypersensitivity vasculitis that occurs most frequently after an infection with group A streptococcus. This rash consists of the classic palpable purpura, and can be accompanied by abdominal pain that is worse after meals secondary, bowel ischemia, bloody diarrhea, and arthritis. (Wolff et al., 2009, Fig. 14-34A, pg. 399-400)
Q 109.18: Which of the following drugs can cause syndrome of inappropriate antidiuretic hormone (SIADH)? A carbamazepine B glyburide C lithium carbonate D metoprolol
The Correct Answer is: A Many medications can enhance the release or potentiate the effects of ADH. Carbamazepine may increase ADH release. (Andreoli, 2007, pp. 289-291)
Q 97.1: A 34-year-old woman presents for her obstetrical checkup. She is a G2P1 and her pregnancy is uncomplicated to date. During her visit, what is the most important predictor of fetal well-being? A Normal maternal vitals and fetal activity B Maternal weight gain and fetal activity C Absence of contractions and fetal activity D Normal maternal vitals and absence of contractions E Maternal weight gain and normal maternal vitals
The Correct Answer is: A Maternal vitals are a sign of maternal well-being; combined with normal fetal activity, this gives the best predictive value for fetal well-being without direct fetal surveillance.
Q 106.11: A 47-year-old female patient is diagnosed with a duodenal ulcer. She was determined to have H. pylori infection. Which of the following antibiotics is indicated for the eradication of H. pylori in the treatment of peptic ulcer disease? A Metronidazole B Cephalexin C Trimethoprim-sulfamethoxazole D Gentamycin E Ofloxacin
The Correct Answer is: A Of the antibiotics listed, metronidazole is the only one that has the indication in the treatment of H. pylori induced peptic ulcers. (McPhee SJ, Papadakis MA. Current Medical Diagnosis & Treatment, 2010, p. 551)
Q 98.66: A 56 year-old woman develops a fever of 102.1 F on her first post-operative surgery day following cholecystectomy. Which of the following is the most likely diagnosis? A Atelectasis B Catheter-related phlebitis C Drug reaction D Urinary tract infection E Wound infection
The Correct Answer is: A Post-operative fever that occurs in the first 24 hours is most likely caused by atelectasis (A). Catheter-related phlebitis (B), drug reactions (C) and urinary tract infection (D) typically present between post-operative days 2 and 4, while wound infection (E) typically presents on or after post-operative day 5.
What is the most common blood gas abnormality in patients with a pulmonary embolism? A Respiratory alkalosis B Metabolic acidosis C Metabolic alkalosis D Respiratory acidosis E Compensated respiratory acidosis
The Correct Answer is: A Respiratory alkalosis occurs as a *result of the hyperventilation.*
Q 113.2: Which of the following is the third component of the atopic triad, besides allergic rhinitis and asthma? A Psoriasis B Dermatitis C Arthritis D Urticaria E Pruritus
The Correct Answer is: B Atopic dermatits, or eczema, is the third chronic finding, along with asthma and allergic rhinitis, in patients who are atopic. Urticaria are common in acute and chronic allergies. (Lalwani A.K., 2008, Chapter 13)
Q 72.10: In Western society, diverticulosis most often occurs in which portion of the colon? A transverse B sigmoid C descending D ascending E equally common in all parts of the colon
The Correct Answer is: B Diverticulosis may arise anywhere in the large intestine, from the cecum to the end of the sigmoid colon. In Western societies, diverticula most often occur in the sigmoid colon where there is greatest intraluminal pressure. (McQuaid, 2009, p. 572) McQuaid KR. Gastrointestinal disorders. In: McPhee SJ , Papadakis MA, eds. Current Medical Diagnosis and Treatment. 48th ed. New York, NY: McGraw-Hill; 2009.
Q 1.2: A 6-week-old male with sickle cell disease presents to the pediatric office for his well-child visit. When should this child begin taking daily prophylactic penicillin? A Now B 2 months C 6 months D 12 months E 5 years
The Correct Answer is: B Patients with sickle cell disease develop functional asplenia as early as 3 months of age and should begin treatment with prophylactic penicillin at 2 months of age to prevent infection by encapsulated organisms (i.e., pneumococcus). Discontinuation of therapy can be considered beginning about age 5 (E).
Q 4.10: A 43 year-old woman recently diagnosed with sarcoidosis returns to the clinic for follow-up. Her chest x-ray demonstrates bilateral hilar lymphadenopathy. What stage of disease is this finding most consistent with? A Stage 0 B Stage 1 C Stage 2 D Stage 3 E Stage 4
The Correct Answer is: B Stage 1 (B) disease consists of bilateral hilar and/or mediastinal adenopathy without pulmonary parenchymal abnormalities. Stage 0 (A) is a normal chest x-ray. Stage 2 (C) includes adenopathy and lung parenchymal abnormalities. Stage 3 (D) includes diffuse lung parenchymal disease without nodal enlargement, and Stage 4 (E) is end-stage lung disease with pulmonary fibrosis and honeycombing.
A 7-year-old male presents to the emergency department with right leg pain after falling off a swing at the school playground. Imaging of the right lower extremity shows the following fracture pattern: Which type of Salter-Harris Classification is observed? A Type I B Type II C Type III D Type IV E Type V
The Correct Answer is: C A Salter-Harris Type III fracture (C) involves a portion of the epiphysis only. Type I (A) is the entire epiphysis, Type II (B) is the entire epiphysis along with a portion of the metaphysis, Type IV (D) involves a portion of the epiphysis along with a portion of the metaphysis, and Type V (E) is a compression injury of the epiphyseal plate (nothing is "broken off").
Q 81.7: Which pharmacologic agents have phototoxicity as a side effect? A Tetracycline, penicillin, and metoprolol B Penicillin, ketoconazole, and metrogel C Doxycycline, hydrochlorothiazide, and naproxen D Cephalexin, acetaminophen, and metoprolol
The Correct Answer is: C A phototoxic reaction is one that results from exposure to the drug and exposure to UV light. The reaction occurs in exposed sites. (Wolff & Johnson, Table 10-4, pg 239) (Fig, 10-22, Wolff K, Johnson RA: Fitzpatrick's Color Atlas & Synopsis of Clinical Dermatology, 6 th Ed: http://www/accessmedicine.com )
Q 98.52: A 29-year-old female has a long history of supraventricular tachycardia, for which she has been treated with long-term flecanide, as well as prior therapy with verapamil. She continues to have repeated episodes, sometimes two to three times a week, along with shortness of breath and at times hypotension that has been recorded. What is the next best therapy for this patient? A Synchronized cardioversion B Cardiac catheterization C Ablation therapy D Pacemaker insertion E Long-term telemetry monitoring
The Correct Answer is: C After exhaustion of non-invasive therapies, ablation therapy can be used to try to negate the aberrant pathway for SVT. Pacemakers will not allow for an override of the pathway, and cardioversion is only a temporary solution to an acute event. Implantable telemetry monitoring is only diagnostic and not therapeutic to treat.
Q 102.6: Which nonnicotine related agent has been approved by the FDA as a first-line medication in the treatment of smoking cessation? A nortriptyline B clonidine C bupropion D fluoxetine
The Correct Answer is: C Bupropion SR has been approved by the U.S. Food and Drug Administration (FDA) for smoking cessation. The drug has been successful in doubling cessation rates. Side effects include dry mouth, agitation, insomnia, and headache. (Rigotti, 2008, p. 166) Rigotti NA. Smoking. In: Feldman MD , Christensen JF, eds. Behavioral Medicine in Primary Care: A Practical Guide, 3rd ed. New York: McGraw-Hill; 2008.
Q 104.1: Which of the following is indicated to confirm the diagnosis of celiac sprue in a patient with positive serologic testing? A stool for fecal fat B barium enema C intestinal biopsy D antimitochondrial antibodies E food challenge
The Correct Answer is: C Intestinal biopsy is the most specific test in establishing the diagnosis of celiac sprue in a patient who has a positive test for IgA endomysial antibody. Classic symptoms of malabsorption are more common in infants but less common in adults. Stool for fecal fat would be a nonspecific finding. Antimitochondrial antibodies are seen in patients with primary biliary cirrhosis. (McQuaid, 2009, pp. 543-544) McQuaid KR. Gastrointestinal disorders. In: McPhee SJ , Papadakis MA, eds. Current Medical Diagnosis and Treatment. 48th ed. New York, NY: McGraw-Hill; 2009.
Q 15.5: Which type(s) of Salter-Harris fractures can generally be treated with closed reduction and cast immobilization? A Type I B Types I and II C Types I, II, and III D Types I, II, III, and IV E Types I, II, III, IV, and V
The Correct Answer is: C Minimally displaced Salter-Harris types I, II, and III fractures generally can be treated with immobilization only. Types IV and V involve the cartilage of both the articular surface and the growth plate. To ensure proper alignment and a congruous joint surfaces open reduction and internal fixation is usually necessary.
Q 119.19: A 55-year-old male presents with severe swelling to his left eye. He denies injury or allergies. He states that he has had a severe sinus infection for the past two weeks. What is the most likely diagnosis for the following physical finding? A Allergic reaction B Eyelid abcess C Orbital cellulitis D Erysipelas E Zoster ophtalmicus
The Correct Answer is: C The image shows a severe orbital cellulitis. These infections often spread from paranasal sinus infections. Multiple pathogens may be involved, such as s.aureus, s. pheumoniae, and anaerobes. An eyelid abscess would be more localized. Allergic reactions that cause angioedema around the eye are usually bilateral, and also pruritic. (Tintinalli et al., 2011, Chapter 236)
Q 65.1: A 27-year-old woman presents with 3 days of fever, chills, headache, and a deep dry cough. She has been working at a pet store for the past month and thinks that one of the parakeets that came in 10 days ago may be sick. On examination, she has dullness to percussion of the right lung base and right-sided coarse crackles. The most likely diagnosis is A sarcoidosis B tularemia C psittacosis D brucellosis E listeriosis
The Correct Answer is: C The key piece of history in this question is the new exposure to parakeets. The symptoms and signs, including atypical pneumonia, are consistent with psittacosis but are not pathognomonic. Sarcoidosis is an illness of unknown cause. Listeriosis has been linked to exposures to contaminated food, particularly dairy products and hot dogs. Brucellosis can be caused by exposure to hogs, cattle, or goats. Tularemia is associated with contact with rabbits, other rodents, and biting arthropods.
Q 97.92: A 24-year-old gravid 3 para 0 ab 2 presents complaining of vaginal pressure every 2 hours, but no pain. She is at 26 weeks gestation. This pregnancy is uncomplicated to date. On physical exam she is in no acute distress and her vitals are normal. She is placed on a fetal monitor and no contractions are noted; fetal heart tones are 138. A sterile speculum exam reveals her cervix to be dilated to 4 cm. What is the most likely diagnosis? A Preterm labor B Group B Streptococcus C Incompetent cervix D Braxton Hicks contraction E Chlamydia trachomatis
The Correct Answer is: C While infection is a significant cause of premature labor, it is not a cause of incompetent cervix. Incompetent cervix is most likely found in the presence of recurrent pregnancy loss and painless dilatation.
Q 35.3: A 12 year old male presents with lesions on the palmar surface. They have been present for less than a week. Prior to the lesion appearing there was a small lesion that was assumed to be a bug bite. The lesion bleeds profusely with minimal provocation. What is the most likely diagnosis? A Glomus tumor B Hemangioma C Spider Angioma D Pyogenic granuloma
The Correct Answer is: D A pyogenic granuloma (PG) occurs at the site of minor trauma such as a bug bite or scratch. The PG grows rapidly forming a popular lesion with a collarette of scale. It will bleed profusely with minimal provocation. The only effective treatment is shave excision with curetting and ablation of the blood vessels that form the base of the PG.
Q 48.3: An 8-year-old girl is brought in to the emergency department with abdominal cramps, nausea, and vomiting since early this morning. She has had two loose stools but denies melena or hematochezia. She has had a low-grade fever. In the past hour, her vision has become blurry and she feels increasingly weak. Her mother has had similar but milder symptoms. Twenty-four hour dietary recall includes only chicken broth today. Last night for dinner they had meatloaf (fully cooked), mashed potatoes, and green beans. Her mother cans all their vegetables. Her medical history is unremarkable. She takes no medications. No known drug allergies. Examination reveals a temperature of 99°F, clear lungs, and mildly tachycardic heart with no murmur audible. Abdomen-bowel sounds present, soft with mild diffuse tenderness, no guarding. Neurologic examination is significant for decreased visual acuity and decreased motor strength (2/5) in the upper and lower extremities. The most likely etiology is A enterotoxic E coli B cholera species C pinworms D Clostridium botulinum
The Correct Answer is: D Clostridium botulinum produces a neurotoxin that can lead to life-threatening illness including respiratory paralysis. Botulism infection is caused by the spore-forming bacteria that lives in soil and can be foodborne. In the latter case, home-canned foods are often the cause. After a 12-hour to 3-day incubation period, botulism begins with classic symptoms of abdominal pain, nausea, vomiting, and mild diarrhea and, if unchecked, evolves into a progressive neurologic disorder marked by double vision, motor weakness, and ptosis. Respiratory muscle involvement may occur ultimately and result in death. Because of the virulence of the neurotoxin it has been used as an agent of bioterrorism. Cholera and enterotoxigenic E. coli cause a foodborne diarrheal illness that can result in significant morbidity and mortality, but they do not have neurologic manifestations. Pinworm infection is usually found among younger children, is marked by severe anal itching, and fecal-oral transmission.
Q 119.1: In a patient whom you suspect has giant cell arteritis, which of the following medication and dosage combinations is indicated? A Naproxen 500 mg twice daily for 1 month. B Prednisone 20mg daily for 1 month C Methylprednisolone dose-pack D Prednisone 60 mg daily for 1 month E Infliximab 5mg/kg daily for 1 month
The Correct Answer is: D Giant cell arteritis (GCA) or temporal arteritis is a large vessel vasculitis that can cause ischemic optic neuropathy and blindness. The first line treatment of high dose prednisone 60 mg/day for one month is sight saving. The lower dose of prednisone 20mg per day is effective in treating polymyalgia rheumatic (PMR). Methylprednisolone dose-pack is a very low dose of steroid and would be ineffective in treating GCA. Infliximab is a tumor necrosis factor inhibitor used for treating ankylosing spondylitis, rheumatoid arthritis and crohn's disease. (McPhee et al., 2011, Chapter 20)
A 2-year-old baby girl is brought to the ED with a history of abdominal pain and diarrhea. Mother states that the child was playing normally and then "doubled over" with what appears to be abdominal pain. The abdomen appears slightly distended and is tender to palpation. While in the ED the child has a bloody, diarrheal bowel movement. Which of the following is the most likely diagnosis? A pyloric stenosis B mesenteric ischemia C Crohn disease D intussusception E Hirschsprung disease
The Correct Answer is: D Intussusception is the most frequent cause of intestinal obstruction in the first 2 years of life. The patient develops paroxysms of pain followed by bloody bowel movements. Pyloric stenosis typically presents prior to the age of 6 months with vomiting but not with diarrhea. Hirschsprung disease results from an absence of ganglion cells in the colon and typically presents early in life with failure to pass meconium, followed by vomiting and abdominal distension. The typical age of onset is later in adolescence in Crohn disease and in the elderly in mesenteric ischemia.
Q 17.2: A 33 year-old man with cystic fibrosis is admitted to the hospital with pneumonia. You order a sputum culture and sensitivity. What is the most likely organism causing this patient's pneumonia? A Klebsiella pneumoniae B Legionella C Mycoplasma pneumoniae D Pseudomonas aeruginosa E Streptococcus pneumoniae
The Correct Answer is: D Patients with Cystic Fibrosis commonly develop chronic infection and subsequent pneumonia from pseudomonous aeruginosa (D).
Q 98.9: A patient who is an Ashkenazi Jew and is homozygous for Factor XI deficiency requires abdominal surgery. Which hemostatic laboratory testing outcomes would you expect to find? A Decreased platelets, normal aPTT, prolonged PT B Decreased platelets, prolonged aPTT, normal PT C Normal platelets, normal aPTT, prolonged PT D Normal platelets, prolonged aPTT, normal PT E Normal platelets, prolonged aPTT, prolonged PT
The Correct Answer is: D Patients with factor XI deficiency have a prolonged aPTT, normal PT, and normal platelets count.
Q 116.10: Mrs. Jones was referred for screening colonoscopy at the age of 50. She has no personal or family history of colorectal cancer. No polyps or lesions were found during the exam. She should be advised that colonoscopy should be repeated in how many years? A 1 year B 2 years C 3 years D 5 years E 10 years
The Correct Answer is: E In average-risk individuals aged 50 or greater than 50, screening colonoscopy should be repeated every 10 years following an initial normal exam. If the individual has a first-degree relative with a history of adenomas or colorectal cancer, screening should begin earlier, generally at age 40 or 10 years younger than the age at diagnosis of the youngest affected relative. (Rugo, 2009, pp. 1452-1453) Rugo HS. Cancer. In: McPhee SJ , Papadakis MA, eds. Current Medical Diagnosis and Treatment. 48th ed. New York, NY: McGraw-Hill; 2009.
Q 9.1: A 38 year-old woman with severe-persistent asthma presents to the clinic complaining of nightly nighttime awakens and frequent use of her inhaled albuterol. Which of the following is an antibody that prevents IgE from binding to its receptor on mast cells and basophils and may be considered as a treatment for this patient? A adlimumab B daclizumab C etanercept D infliximab E omalizumab
The Correct Answer is: E Omalizumab (E), is an antibody that prevents IgE from binding to its receptor on mast cells and is used in the treatment of allergic disease. Daclizumab (B) is anti-IL-2 antibody used to prevent acute rejection of organ transplants. Adlimumab (A), etanercept (C), and infliximab (D) bind TNF, thus inhibiting the action of TNF and are used in the treatment of disorders such as psoriasis and rheumatoid arthritis.
Q 118.20: A 63-year-old male presents with an asymptomatic lesion in his mouth that was discovered by his dentist at a check-up. It is ill marginated with pigment ranging from medium brown to black. Parts of the lesion are raised. What is the next appropriate step in management? A Biopsy of the lesion B Inject the lesion with triamcinolone C Observe for changes over the next six months D Prescribe keflex for 10 days
The Correct Answer is: A Oropharyngeal melanoma is characterized by varying pigment occurring in an irregularly shaped lesion. Although this is a rarely occurring melanoma, a biopsy should be done and any pigmented oral lesion should be excised. Areas which are raised within the lesion usually indicate sites of invasion. (Wolff et al., 2009, Fig. 34-13, pg. 1040 & 327)
Q 97.5: A woman who is pregnant suffered a spontaneous abortion at 12 weeks gestation. She is now a G2P1Ab1 and is Rh negative. When should she receive her next Rhogam (Rho D immune globulin) shot? A Now B In one month C At conception of her next pregnancy D 28 weeks gestation of next pregnancy E After delivery of her next viable infant
The Correct Answer is: A Placental implantation occurred and separated with the spontaneous miscarriage. Therefore, there is a slight chance of isoimmunization, so Rhogam should be given now so that the mother does not develop antigens that can cross the placenta during the first half of the next pregnancy.
Q 101.4: Taking vitamin C, E, zinc, and beta carotene, and stopping smoking, have a preventative effect on the progression of which of the following diseases? A Macular degeneration B Retinal detachment C Central retinal artery occlusion D Diabetic retinopathy E Central retinal vein occlusion
The Correct Answer is: A Smoking cessation and taking supplements, including vitamin C, E, zinc, and beta carotene, have shown an eight percent decrease in progression of late stage macular degeneration. However, smokers or previous smokers should not take beta carotene, due to its link with lung cancer in smokers. (Riordan et al., 2008, Chapter 6)
Q 119.12: A 45-year-old woman with a known seizure disorder has been noncompliant with her anticonvulsant medication due to side effects she has been experiencing. While in your office, she starts convulsing at a frequency that does not allow consciousness. Which of the following is the most appropriate initial drug treatment? A lorazepam B phenytoin C phenobarbital D valproic acid
The Correct Answer is: A Status epilepticus, defined as a continuous seizure or repeated seizures in which interval consciousness is not obtained, is a medical emergency. An intravenous infusion of a longer acting benzodiazepine, such as lorazepam, has been shown to be effective in terminating a seizure. If the seizures persist, then other potential agents to use after the initial lorazepam infusion include fosphenytoin (phenytoin) or valproic acid (phenobarbital). (Lowenstein, 2008, p. 2511) Lowenstein DH. Seizures and epilepsy. In: Fauci AS , Braunwald E , Kasper DL, et al., eds. Harrison's Textbook of Medicine. 17th ed. New York, NY: McGraw-Hill; 2008.
Q 76.4: A 7-year-old male is suspected to have a Wilms' tumor (nephroblastoma). What is the most common symptom at presentation? A Abdominal mass B Hypertension C Hematuria D Coagulopathy E Fever
The Correct Answer is: A The diagnosis of a Wilms' tumor is most commonly made after the discovery of an asymptomatic mass, by a family member or a physician, during a routine physical examination. The most common sign is an abdominal mass (A). Other symptoms at presentation include abdominal pain and distention, anorexia, nausea and vomiting, fever (E) and hematuria. Hypertension (B) is seen in 25-60% of cases and is caused by elevated renin levels. As many as 30% of patients demonstrate hematuria (C) and coagulopathy (D) can occur in up to 10%. Konety BR, Williams RD. Chapter 21. Renal Parenchymal Neoplasms. In: Tanagho EA, McAninch JW, eds. Smith's General Urology. 17th ed. New York: McGraw-Hill; 2008. http://www.accessmedicine.com/content.aspx?aID=3128159 . Accessed March 6, 2013.
Q 108.5: A 25-year-old female presents with a complaint of dry, stinging hands for the past two months. She has never had any rashes or similar problems. She does not work. The patient stays home to care for her six-month-old baby. She has tried treating her hands with over-the-counter lotions, but reports that they sting upon application. What should appropriate management of this condition include? A triamcinolone 0.025% ointment bid, moisturize with petrolatum frequently, use gloves when hands in water B withhold all treatments for one week and have patient undergo patch testing to determine allergen C punch biopsy at periphery of outbreak, and treat with ketoconazole cream for two weeks D KOH in the office, treat with ketoconazole cream alternating with bactroban cream until resolved
The Correct Answer is: A The patient is experiencing an irritant contact dermatitis, secondary to having her hands in water frequently and using diaper wipes, which can be very irritating due to the alcohol content. The appropriate treatment would consist of reducing the irritant (water and wipes) by using barrier protection (gloves). A mid-potency topical steroid, such as triamcinolone 0.025% ointment twice daily, until the irritation has improved is appropriate treatment. Petrolatum and petrolatum based emollients are best for frequent moisturization. The history given is classic for irritant dermatitis, and withholding treatment for one week along with patch testing is not necessary, unless the patient does not respond to conservative therapy. A biopsy is not indicated, as this is classic irritant dermatitis. Ketoconazole cream is an antifungal medications and bactroban is a topical antibiotic. This patient presentation is not typical for a fungal infection, and should not be treated with an antifungal unless a positive KOH or fungal culture has been done. The bactroban ointment can help prevent a secondary bacterial infection if fissures are present. (Wolff et al., 2009, Page 24)
Q 8.1: A 36-year-old man presents to the emergency department with a tight bandage around his chest to help reduce pain from a chest wall injury on his right side that occurred during mixed martial arts sparing. Physical exam reveals dullness to percussion, dry crackles and diminished breath sounds over the right lower lobe. Chest x-ray shows elevation of the right hemi-diaphragm. What is the most likely diagnosis? A Atelectasis B Bronchiectasis C Pleural Effusion D Pneumothorax E Pulmonary edema
The Correct Answer is: A The patient's injury places him at risk of atelectasis, pneumothorax, or other traumatic injuries. The physical exam and chest x-ray findings are classic for atelectasis (A). Pleural effusion (C) would present with fluid in the costophrenic angle on chest x-ray. Pneumothroax (D) would typically present with findings in the upper lung fields including hyperresonance to percussion. Pulmonary edema (E) would present with increased vascular markings and evidence of fluid within the alveolar space on chest x-ray.
Q 38.12: A two-year-old male presents with a nodule on the side of his index finger. His mother states that he has had this nodule on one prior occurrence during infancy, and it resolved on its own. What is the most likely diagnosis? A Digital fibroma B Juvenile xanthogranuloma C Molluscum contagiosum D Verruca vulgaris
The Correct Answer is: A This child has a recurrent digital fibroma. It is a smooth, firm, pink nodule that occurs on the fingers and toes up through early childhood. Surgical excision is recommended so that the function of the digit is not impaired.
Q 39.10: A 1-year-old boy is brought to the emergency department by his parents, who state that the child refuses to walk or crawl and begins crying when they stand him. Swelling to his right knee is noted; it is also warm to the touch and pain response is noted. His parents state that it seemed to start a couple of days ago and has gotten worse. They don't recall a trauma, but state that he seems to bruise easily. The child's mother states that she also bruises easily. Vitals are as follows: Temp: 38.0°C, HR: 70, RR: 15. What laboratory finding would you expect? A Prolonged aPTT (activated partial thromboplastin time) B Prolonged bleeding time C Prolonged PT (prothrombin time) D Prolonged thrombin clotting time E Thrombocytopenia
The Correct Answer is: A This patient has hemophilia A. Patients with severe hemophilia A have a prolonged aPTT; all of the other tests should be within the normal range.
Q 71.7: A patient who is suspected of having pneumonia produces a sputum sample that grows gram-positive diplococci in chains (see image). What is the best choice of antibiotics for this type of patient? A Penicillins B Cephalosporins C Fluroquinolones D Aminoglycosides E Macrolides
The Correct Answer is: A This patient is having an infection that is consistent with the bacterium Streptococcus pneumoniae. The best and most appropriate treatment for this pathogen is penicillins. (Chesnutt MS, Prendergast TJ. Current Medical Diagnosis and Treatment, 2011, Chapter 9, Pulmonary Disorders)
Q 113.5: Upon examination of a 24-year-old male, you note painless lymphadenopathy along the cervical and femoral chains. He notes that he has had to change his bedding daily because he soaks his sheets from sweat. He has also lost 35 pounds without trying to over the last three months. He thinks that he has had fevers at night but has never checked. This is a classic description of which of the following? A "B" symptoms B Cat scratch fever C Neutropenia D Sickle cell crisis E Sideroblastic anemia
The Correct Answer is: A This patient is presenting with signs and symptoms of non-Hodgkin's lymphoma. Many patients with lymphoma exhibit fever, drenching night sweats, and weight loss—these symptoms are referred to as "B" symptoms. Cat scratch fever usually has painful lymphadenopathy. Sickle cell crisis and sideroblastic anemia do not produce non-painful lymphadenopathy. Neutropenia involves a low white cell count. (McPhee SJ, Papadakis MA, Tierney LM. Current Medical Diagnosis and Treatment, 2010, Chapter 13, Blood Disorders)
A 48-year-old man presents to the emergency department with acute right upper quadrant tenderness, fever, and mild jaundice. Which of the following is most likely to be elevated in the blood? A bilirubin B creatinine C glucose D ketones E uric acid
The Correct Answer is: A This patient's signs and symptoms correlate with a suspected case of cholecystitis. Jaundice is associated with hyperbilirubinemia, in which the excess bilirubin can deposit in tissues such as the skin, sclera, and nails, causing a yellowish discoloration. Bilirubin is the waste product generated from the metabolism of hemoglobin.
Q 63.8: What is the most common cause of restrictive cardiomyopathy? A Amyloidosis B Pericarditis C Marfans syndrome D Fatty infiltrative disease E Sarcoidosis
The Correct Answer is: A While restrictive cardiomyopathy is seen in such cases as hemochromatosis, glycogen deposition, endomyocardial fibrosis, sarcoidosis, hypereosinophilic disease, and scleroderma, amyloidosis is the most common cause among the choices provided.
A 35-year-old male presents with pain and decreased range of motion after sustaining a fall in which the patient tried to grab onto a bar which pulled his entire arm in the process. Given this clinical scenario, at what cervical motor neuron level would the biceps reflex be testing? A C4 B C5 C C6 D C7 E C8
The Correct Answer is: B C5 is the primary motor neuron being tested in a biceps reflex. C6 contributes to the brachioradialis reflex primarily, but does have a small role in the biceps reflex. The C7 motor neuron is primarily involved in the triceps reflex. C4 and C8 do not contribute to any primary reflexes.
Q 84.1: A 22-year-old woman with a history of scoliosis presents to the office with complaints of a retrosternal chest discomfort, occurring frequently at rest and lasting for several hours at a time. On physical exam, a mid-systolic click is noted. Which of the following maneuvers would be the most appropriate to choose for better identification of the murmur? A Left lateral decubitus position listening with the bell of the stethoscope B Standing C Seated leaning forward D Inspiration E Expiration
The Correct Answer is: B Choice B, standing, will cause the mid-systolic click associated with mitral valve prolapsed to move toward S1 or become more audible. Choice A, rolling the patient to the left lateral decubitus position, is most useful in identification of the murmur of mitral stenosis. Choices C, D, and E have no effect on the mid-systolic click. Inspiration increases the intensity of the murmurs associated with tricuspid stenosis and regurgitation. (Fauci et al., 2008, Chapter 230)
Q 4.1: A 52-year-old female with a past medical history of dyslipidemia, GERD, hypothyroidism, and osteoarthritis presents with a gradual progression of fatigue and pallor over the last few months. Initial CBC results show a hemoglobin of 10.7 mg/dL, hematocrit of 33%, an MCV of 117 fL, and a reticulocyte count of 0.0%. Which of the following medications should be considered as a potential cause of her anemia? A Celecoxib B Cholestyramine C Levothyroxine D Omeprazole E Ranitidine
The Correct Answer is: B Cholestyramine is associated with folate malabsorption and may rarely be a medication cause for macrocytic anemia. Celecoxib (A), levothyroxine (C), omeprazole (D), and ranitidine (E) are not strongly associated with vitamin B 12 or folate deficiency leading to macrocytic anemia
Q 44.3: A couple presents to the office seeking genetic counseling advice regarding their child's potential risk of being born with Cystic Fibrosis. Both parents are identified as carriers for the cystic fibrosis gene mutation. Which of the following genetic inheritance patterns should be used to predict the probability of their child being affected by Cystic Fibrosis? A Autosomal dominant B Autosomal recessive C Mitochondrial inherited D X-linked dominant E X-linked recessive
The Correct Answer is: B Cystic fibrosis has an autosomal recessive (B) genetic inheritance pattern. In this case their children have a 25% chance of being affected, 50% chance of being a carrier, and 25% chance of lacking inheritance of the genetic trait.
Q 62.11: A 29-year-old female complains of a two-month history of easy bruising. She describes that the bruising is located primarily on her shins, but has noted them on other areas as well. She also describes red freckles on her lower extremities. On exam, you note non-blanching and non-palpable purpura to both legs and petechiae. She denies recent illnesses and states that she has essentially been feeling fine. The PE is normal other than the skin findings. A platelet count of 118,000 is noted. What is the most likely diagnosis? A Antiphospholipid syndrome B Idiopathic thrombocytopenic purpura C Systemic lupus eruthematosus D Thrombotic thrombocytopenic purpura E Von Willebrand's disease
The Correct Answer is: B Diagnosis is based on clinical manifestations including history, physical exam, blood count, and blood film. Typically, these patients have felt fine until they notice development of purpura and petechiae. No additional physical findings are typically found. Blood counts note a thrombocytopenia, with other tests usually in the normal range.
Q 119.9: You suspect that a 44-year-old female with a history of fatigue, anxiety, and weight loss has hyperthyroidism. What physical examination findings would support hyperthyroidism as your diagnosis? A dry skin B exophthalmos C decreased tendon reflexes D thinning hair E bradycardia
The Correct Answer is: B Exophthalmos is a physical examination finding that is found in some patients with hyperthyroidism due to Grave's disease. Other potential physical examination findings may be a tremor, moist skin, and tachycardia. The other choices may be found in patients with hypothyroidism. (McPhee and Papadakis, 2011, Chapter 26)
Q 21.6: A 22-year-old non-obese female presents to the clinic with sign and symptoms of deep vein thrombosis (DVT). She denies recent trauma, prolonged immobilization, smoking, use of oral contraceptives, or changes in her health. Her last menstrual period was one week ago. Which of the following is the most likely diagnosis that contributed to the development of her DVT? A Antithrombin deficiency B Factor V Leiden C Protein C deficiency D Protein S deficiency E Hyperhomocysteinemia
The Correct Answer is: B Factor V Leiden mutation is one of the most common genetically inherited prothrombotic states. Deficiencies of antithrombin (A), protein C (C) and protein S (D) when homozygous in nature typically present as fetal death or severe thrombosis at birth. When heterozygous, they carry a much lesser risk of developing thrombosis as compared to Factor V Leidin. Hyperhomocysteinemia (E) also carries a much lesser risk of developing thrombosis.
In addition to insulin and fluid replacement with 0.9% saline, which electrolyte is commonly infused in the type 2 diabetic patient who arrives in the emergency department in a hyperglycemic, hyperosmolar, nonketotic state? A bicarbonate B potassium C calcium D magnesium E sulfate
The Correct Answer is: B Insulin not only causes cellular uptake of glucose but also of potassium. Hypokalemia may develop when insulin is infused to correct either a hyperglycemic hyperosmolar state or a diabetic ketoacidosis. Hence,* in order to avoid hypokalemia, potassium chloride can be added to a saline solution, as long as the serum potassium is not elevated.*
Q 40.9: The Centers for Disease Control and Prevention recommend the first lead screening for children living in high risk areas in the United States at which age? A 6 months B 9 months C 15 months D 24 months
The Correct Answer is: B The CDC recommends that there are two age ranges for testing lead in children in the United States: 9 to 12 months and again at 24 months. These high-risk areas include poverty-stricken areas, use of lead paint pottery, lead painted homes (peeling or cracking), industrial exposures, and use of diarrhea remedies in Mexico. The CDC recommends using questions to screen all children between 6 months and 6 years of age.
Q 98.13: A 5-year-old male is being evaluated for an acute injury to the right ankle. On the x-ray of the ankle there is a distal tibia fracture that involves the separation of the epiphysis, as well as a small non-displaced chip fracture of the metaphysis of the tibia. Based on these findings, what type of Salter-Harris fracture does this child have? A I B II C III D IV E V
The Correct Answer is: B The growth plate is the most fragile part of the bone prior to bone maturation and thus is usually the first structure disrupted when force is applied. Statistically, Type II fractures are most common - those that involve both the growth plate and a chip fracture of the metaphysis.
Q 116.11: What is the chief complaint associated with bladder cancer? A pyuria B hematuria C dysuria D urinary frequency
The Correct Answer is: B The most common complaint of bladder cancer is painless hematuria, which occurs in 85% to 90% of patients. Additional symptoms of bladder irritability, and urinary frequency, urgency, and dysuria are the second most common presentation and are usually associated with invasive bladder cancer. (Rugo, 2009, p. 1461)
A 63 year-old woman presents to the clinic complaining of increasing dyspnea over the last two weeks. Which of the following diagnoses is supported by the chest x-ray below? A Dissecting aortic aneurysm B Pleural Effusion C Pneumonia D Pneumothorax E Pulmonary Embolism
The Correct Answer is: B The patient has a left-sided pleural effusion (B). The mediastinum is not widened as seen in dissection aortic aneurysms (A) and although the lung markings are absent in the left lower lobe region this is due to the accumulation of fluid (B) and not air as seen in pneumothorax (D). Clear signs of pneumonia (D) or pulmonary embolism (E) aren't present on the chest x-ray, but are potential underlying causes of effusion.
A 3 year-old boy is brought to the emergency department due to acute onset of cough and wheezing. Physical exam reveals focal wheezing in the right lower lobe. Which of the following should be ordered to confirm the suspected diagnosis? A Arterial blood gas B Inspiratory and forced expiratory chest x-rays C PA and lateral chest x-ray D Peak expiratory flow rate E Spirometry
The Correct Answer is: B The patient most likely has aspirated a foreign body. This is best evaluated through the demonstration of inspiratory localized hyperinflation and expiratory mediastinal shift (B) on chest x-ray. ABG (A) results will vary depending on the severity of airway obstruction. PA and lateral chest x-rays (C) are typically normal. PEFR (D) and Spirometry (E) are not typically able to accurately assess this localized airway obstruction.
Q 38.2: An 8-year-old male with hair loss, pruritus, and posterior cervical lymphadenopathy has a + culture on dermatophyte test medium (DTM). Which of the following is the most appropriate treatment? A ketoconazole shampoo daily x 2 weeks B oral griseofulvin x 8 weeks C oral Keflex x 2 weeks D desonide foam qd x 6 weeks
The Correct Answer is: B The positive DTM confirms the diagnosis of tinea capitis. The only approved treatment for tinea capitis is griseofulvin. In tinea capitis, the dermatophyte invades the hair shaft and topical treatment is not effective. An antifungal shampoo is often used as an adjunct to treatment. Keflex is an oral antibiotic, which will not help treat the fungal infection. Desonide is a topical steroid that will worsen the fungal infection.
Q 92.4: A person with an exaggerated sense of entitlement and uniqueness and who believes they can only be understood by people of significance is described to you by a colleague. They go on to state the person is arrogant, is lacking in empathy, and can be manipulative with relationships. What personality disorder best fits this scenario? A histrionic B narcissistic C antisocial D borderline
The Correct Answer is: B The scenario represents a typical "snapshot" of this diagnosis. These persons typically have fantasies of unlimited success and have a strong need for admiration from others. They can be jealous of others but commonly assume that others are extremely jealous of them. Treatment is made difficult as they do not accept criticism or any attack on their "narcissistic supply." (Eisendrath and Lichtmacher, 2009, p. 925-926; Sadock and Sadock, 2008, p. 384) Eisendrath SJ , Lichtmacher JE. Psychiatric disorders. In: McPhee SJ , Papdakis MA, eds. Current Medical Diagnosis and Treatment, 48th ed. New York: McGraw-Hill; 2009. Sadock BJ , Sadock VA. Concise Textbook of Clinical Psychiatry, 3rd ed. Philadelphia, PA: Lippincott, Williams & Wilkins; 2008.
Q 66.6: An 18-year-old male presents with a rash consisting of erythematous target-like lesions on his arms. Physical exam shows a healing cold sore on his lips, but no other skin or mucosal lesions are present. What is the most likely cause? A Borrelia burgdorferi B Herpes simplex virus C Rickettsia rickettsia D Staphylococcus aureus
The Correct Answer is: B This patient is exhibiting the classic rash of erythema multiforme minor. Etiology of this disorder can be traced to herpes simplex outbreaks or other viral or bacterial infections. Recurrent disease is most often associated with herpes simplex outbreaks. (Kane et al., 2002, Fig. 15-5, pg. 328-329)
Q 41.4: An 8-year-old male child presents with brown, nonpruritic, annular lesions on the back of his hands and feet. Intradermal nodules are seen on the extensor surfaces of the elbows and knees that have been present for several months. At today's visit, the lesions are essentially unchanged since his last visit about a month ago. What is the best treatment for this suspected disorder? A excision and biopsy B no treatment C topical steroids D wet to dry dressings
The Correct Answer is: B This presentation is typical for granuloma annulare, which is a benign skin disorder, and treatment is not warranted. It is most commonly seen in children aged 6 to 10. The red to brown lesions are annular or circinate. These asymptomatic lesions are often confused with tinea corporis. The lesions will disappear on their own over a couple of years.
Q 68.7: Which of the following pulmonary function test results demonstrates emphysema, a form of obstructive pulmonary dysfunction? A Decreased total lung capacity, decreased residual volume B Increased total lung capacity, increased residual volume C Decreased total lung capacity, decreased FEV1 D Increased total lung capacity, decreased residual volume E Increased total lung capacity, increased FEV1
The Correct Answer is: B Total lung capacity represents the vital capacity, defined as the amount of gas exhaled after a maximal inhalation, plus the residual volume within the lung after maximum exhalation. With emphysema, the lung parenchymal damage and decreased elasticity results in all flow rates being reduced, including FEV1, FVC, and FEV1/FVC levels. Expiratory time is increased and gas trapping occurs, thus increasing the total lung capacity. (McPhee SJ, Hammer GD. Pathophysiology of Disease, 6e, Chapter 9, Pulmonary Disease)
Q 38.20: The majority of cases of halitosis in young children can be traced to which of the following causes? A dental caries B nasal foreign body C poor dietary habits D upper respiratory tract infection
The Correct Answer is: B While halitosis can be caused by pharyngitis, sinusitis, and poor hygiene, the most common cause of halitosis in children is a nasal foreign body. Seeds and beads are the leading objects inserted into the nose. If not promptly removed, they can cause nasal obstruction, infection, rhinorrhea, bleeding, halitosis, or a foul smell. They are usually easy to remove, but if there is difficulty in removing the foreign body, the child should be referred to an otolaryngologist for definitive care. Tobacco use in adolescents is a common cause of halitosis. Dental disease is the most common cause of halitosis in adults.
What types of connective tissue are injured in a sprain? A Bones and muscles B Fascia and joint capsules C Ligaments and joint capsules D Muscles and tendons E Tendons and bones
The Correct Answer is: C A sprain involves injury to those tissues that give support to joints - ligaments and joint capsules. Injury to muscles, tendons, and fascia would all be classified as a strain. Injuries to bone would be classified as a fracture.
Q 97.126: A 30-year-old female G2 P2, who delivered via normal spontaneous vaginal delivery, presents complaining of increasing vaginal pressure, low back pain, and stress incontinence. What is the mostly likely cause of her condition? A Damage to the levator muscles B Increased intra abdominal pressure C Widening of the levator gap D Widening of the AP pelvis diameter E Endopelvic fascia remodeling and cervical elongation
The Correct Answer is: C Damage to the levator and increased abdominal pressure are known risk factors, but the widening of the gap with the associated risk factors is what allows the defect to occur. Fascia remodeling can occur instead of a prolapse defect.
Q 45.9: A 2-year-old child presents to the pediatric office for a well child visit. The presence of which of the following risk factors places this child at high-risk for dental caries? A Child receives fluoride varnishes B Maternal smoking C Parents are of low socioeconomic status D Single parent home E Use of fluoridated water
The Correct Answer is: C Factors that place children at high-risk for dental caries include the presence of white spots, cavities, or fillings on exam; mother/primary care giver having cavities; mother/primary caregiver being of low socioeconomic status (C); frequent between-meal sugar-containing snacks/beverages; or being put to bed with a bottle that contains sugary beverage. Fluoride (A) and fluoridated water (E) are protective. Maternal smoking (B) and single parenting (D) are not validated risk factors for dental caries.
Q 79.6: Anticoagulation for atrial fibrillation in a 65-year-old with hypertension aims for an international normalized ratio (INR) of: A < 1.0 B 1.0 to 2.0 C 2.0 to 3.0 D 3.0 to 4.0 E > 5.0
The Correct Answer is: C For patients with atrial fibrillation, most clinicians use a CHAD-score to determine if anti-coagulation is necessary. For patients with no contraindication to anticoagulation and at least one risk factor for stroke, anti-coagulation is recommended. Once that determination is made, anticoagulation with warfarin to an INR target of 2.0 to 3.0 should be established and maintained indefinitely. (McPhee and Papadakis, 2011, Chapter 10)
Q 47.5: Which agent is most responsible for croup infections? A Parvovirus B Adenovirus C Parainfluenza D Bocavirus E Streptococcus
The Correct Answer is: C Laryngotracheobronchitis, or croup, is mostly caused by the parainfluenza virus. Other agents that can cause this are respiratory syncytial virus (RSV), human metapneumovirus, influenza virus, rubeola virus, adenovirus, and Mycoplasma pneumoniae.
Q 11.3: What absolute tissue pressure generally is used as a guideline for diagnosing compartment syndrome? A 10 mm Hg B 20 mm Hg C 30 mm Hg D 40 mm Hg E 50 mm Hg
The Correct Answer is: C Many trauma surgery services use an absolute tissue pressure of approximately 30 mm Hg as the threshold for diagnosing compartment syndrome. Based on the entire clinical picture, patients with numbers in that range or higher will likely require surgical decompression with a fasciotomy, while lower numbers will probably be managed with a more conservative approach.
Q 98.75: Which type(s) of Salter-Harris fractures can generally be treated with closed reduction and cast immobilization? A Type I B Types I and II C Types I, II, and III D Types I, II, III, and IV E Types I, II, III, IV, and V
The Correct Answer is: C Minimally displaced Salter-Harris types I, II, and III fractures generally can be treated with immobilization only. Types IV and V involve the cartilage of both the articular surface and the growth plate. To ensure proper alignment and a congruous joint surfaces open reduction and internal fixation is usually necessary.
Q 92.5: Which of the following medications for type 2 diabetes mellitus is thought to best preserve beta cell function in the pancreas? A acarbose B glimepiride C pioglitazone D sitagliptin E insulin glargine
The Correct Answer is: C Pioglitazone is a thiazolidinedione (TZD). TZDs are thought to help preserve beta-cell function. (Fauci et al., 2008, Chapter 338)
Q 79.10: A 65-year-old male presents to you with a growth on the inner aspect of his left eye. He states that it has been getting slightly larger. On physical exam, you note a fleshy triangle shaped protrusion on the inner bulbar conjunctiva, abutting and slightly crossing the limbic border. Which of the following is the correct diagnosis? A Charazion B Hordeolum C Pterygium D Pinguecula E Cataract
The Correct Answer is: C Pterygium is a complication of exposure to ultraviolet light and wind. It consists of hyaline and elastin tissue. If it encroaches on the cornea, surgical removal is indicated. (Riordan et al., 2008, Chapter 5)
Q 80.2: Which of the following is the most prevalent cause of chronic respiratory acidosis? A Anemia B Cerebrovascular accident C Chronic obstructive pulmonary disease D High altitude E Pneumonia
The Correct Answer is: C Respiratory acidosis is associated with elevated PaCO 2 levels, due to the inability of elimination to keep pace with production. This may be due to a decreased rate of ventilation due to control alteration, decreased ventilatory muscle strength, underlying lung disease, or a systemic insult such as infection or medication. Chronic respiratory acidosis is generally due to underlying lung disease, with the most prevalent cause being chronic obstructive pulmonary disease (COPD). High altitude, pneumonia, and severe anemia are associated with respiratory alkalosis. Cerebrovascular accidents may be associated with either respiratory acidosis or alkalosis, depending on the location, extent, and impact. (Fauci et al., Harrison's Principles of Internal Medicine, 17e, Chapter 48, Acidosis and Alkalosis)
Q 97.26: A 19-year-old G1P0 presents for her routine obstetric exam. She is at 34 weeks gestation. When completing the physical assessment you perform a Leopold maneuver. What does this maneuver assess? A Fetal lie and station B Fetal position and presentation C Fetal lie and position D Fetal lie and presentation E Fetal position and station
The Correct Answer is: C The maneuver is used to examine the abdomen and determine the lie (first) and position (second). While the abdominal exam can be used to determine breech versus cephalic, it does not differentiate between, face, brow, and footling presentations. A vaginal exam must be done to determine the presenting part and decent into the pelvis.
Q 37.4: A 3-week-old male infant is brought in by his mother due to his vomiting. The mother notes that a few days ago her son started vomiting after feeding, and it has become projectile in nature. The vomitus is non-bilious and contains no blood. The child seems hungry and nurses regularly, but the vomiting has become more frequent and is occurring with every feeding now. On physical examination, an oval mass is palpated in the right upper quadrant. What imaging study is the best initial test to obtain in this patient? A Abdominal plain film B Barium enema C Barium upper GI series D CT scan of the abdomen E Upper GI endoscopy
The Correct Answer is: C The suspected diagnosis is pyloric stenosis. An upper GI series with barium is the best test to obtain in this instance. A barium enema is a good choice in suspected intussusception cases. An upper GI endoscopy is best used when a gastric or duodenal ulcer is suspected. An abdominal plain film or CT could be ordered, but aren't the best initial tests to obtain in this patient's presentation.
Q 32.15: You are treating a 60-year-old man with a history of angina. He has been on the therapeutic lifestyle change (TLC) diet for 12 weeks (with solid effort). This patient has no other medical conditions and takes nitroglycerin as needed and daily enteric-coated aspirin. His fasting lipid panel from last week demonstrates the following: What is the most appropriate treatment at this time? A prescribe colestipol B prescribe ezetimibe C prescribe simvastatin D prescribe niacin E no pharmacological treatment
The Correct Answer is: C This patient's coronary heart disease risk factors and failed TLC diet warrant pharmacological treatment based on his LDL level. Although there are no absolute guidelines for the selection of lipid-modifying medications, an HMG-CoA reductase inhibitor is preferred.
Q 81.1: Which of the following examinations is a major component of routine monitoring of chronic, open angle glaucoma? A Pupillary response B Corneal reflex testing C Visual field testing D Accommodation E Visual acuity
The Correct Answer is: C Tonometry, gonioscopy, monitoring of the disc-to-cup ratio, and visual field examination are the routine exams done when monitoring primary open angle glaucoma. (Riordan et al., 2008, Chapter 11)
Q 119.17: A 45-year-old presents with a markedly tender nodule protruding from the edge of his upper eyelid. He states that this has been present for 12 hours. No discharge is seen. He denies visual problems. What is the most likely diagnosis? A Blepharitis B Chalazion C Dacryocyctitis D Hordeolum E Conjunctivitis
The Correct Answer is: D A hordeolum (sty) is caused by an acute infection of the Zeis or Moll's glands of the eyelid. Symptoms include pain and tenderness. An "internal hordeolum" points to the inner conjunctiva of the lid and an "external hordeolum" points to the skin surface of the eyelid. (Riordan et al., 2008, Chapter 4)
Q 120.1: A nonpenetrating tear of the gastroesophageal junction in association with a history of vomiting is known as which of the following? A Boerhaave syndrome B Plummer-Vinson syndrome C Peutz-Jeghers syndrome D Mallory-Weiss syndrome E Zollinger-Ellison syndrome
The Correct Answer is: D A mucosal tear of the gastroesophageal junction with a history of prolonged vomiting is known as Mallory-Weiss tear syndrome. Plummer-Vinson is a congenital syndrome associated with anemia and webbing of the esophagus. Boerhaave syndrome is a rare life-threatening problem characterized by a full-thickness tear of the esophageal wall. Zollinger-Ellison syndrome is caused by gastrin-secreting neuroendocrine tumors resulting in acid hypersecretion. (McQuaid, 2009, pp. 489, 521-522, 541, 576) McQuaid KR. Gastrointestinal disorders. In: McPhee SJ , Papadakis MA, eds. Current Medical Diagnosis and Treatment. 48th ed. New York, NY: McGraw-Hill; 2009.
Q 55.5: Which of the following diagnostic tests is the imaging study of choice in patients suspected of having Zollinger-Ellison syndrome? A Transabdominal ultrasound B Computed tomography of the abdomen C Magnetic resonance imaging D Somatostatin receptor scintography E Endoscopic ultrasonography
The Correct Answer is: D Although imaging tests such as CT scan, MRI, transabdominal ultrasound, and endoscopic ultrasound can be utilized with varying frequencies of sensitivity and specificity, the test of choice is the somatostatin receptor scintigraphy. This test has close to a 100% sensitivity and specificity.
Q 112.7: A 20-year-old male presents with cough, nasal congestion, and a low grade fever for one week. His cough seems to be getting worse, which is the reason for his visit. His past medical history includes asthma and nasal polyps. On physical exam, his temperature is 101˚F, his pharynx is erythemic, and there is grey nasal discharge with a few nasal polyps seen using a nasal speculum. His lungs have a few expiratory wheezes bilaterally. What medication is to be avoided in this patient? A Penicillin B Acetaminophen C Erythomycin D Aspirin E Ciprofloxacin
The Correct Answer is: D Aspirin should be avoided in patients with asthma and nasal polyps. Aspirin can precipitate bronchospasm in these patients, due to immunologic salicylate sensitivity. (McPhee et al., 2011, Chapter 8)
Q 24.4: A 43 year-old man presents to the clinic seeking counseling and medication to help quit smoking. Which of the following factors would be a contraindication to prescribing him bupropion? A Allergy to sulfa B Depression C Decreased libido D Seizure disorder E Use of nicotine replacement therapy
The Correct Answer is: D Bupropion is contraindicated in situations where patients are at risk of seizures (D). Bupropion is indicated for the treatment of depression (B) and doesn't typically cause libido concerns (C). Bupropion is commonly prescribed with nicotine replacement therapy (E) during tobacco cessation efforts.
Q 7.9: A 35-year-old nulliparous female has had increasing and heavy flow with her menstrual cycles over the last six months. She is being followed by her gynecologist, who has diagnosed several small uterine fibroids. She presents in clinic today with complaints of tachypnea on exertion, tachycardia, and occasional palpitations. There is no family history of cardiac problems or hypertension. She denies symptoms of infectious etiology and has no fever. On physical examination, you note a mild tachycardia of 110 after she has been sitting for a few minutes. No other physical findings are noted. What would be the next most useful diagnostic study to help you confirm the diagnosis? A CBC B Chest x-ray C EKG D Serum ferritin E Transthoracic ECHO
The Correct Answer is: D Iron deficiency develops in stages. The first is depletion of iron stores. There is no anemia or change in RBC. The serum ferritin will become abnormally low and lead to symptoms of anemia, such as easy fatigability, tachycardia, palpitations, and tachypnea on exertion. Iron deficiency anemia is more frequently seen in women, especially those with heavy menstrual cycles, as they are unable to absorb enough iron in their diet to maintain stores—leading to depletion, signs of anemia, and progression if not treated.
Q 2.8: A 63 year-old woman developed severe thrombocytopenia after undergoing her first treatment of a cisplatin-based chemotherapy regimen. Which of the following agents may be administered to prevent thrombocytopenia if the patient continues her cisplatin-based regimen? A Darbepoetin alfa (DARB) B Erythropoietin (EPO) C Filgrastim (G-CSF) D Oprelvekin (IL-11) E Sargramostim (GM-CSF)
The Correct Answer is: D Oprelvekin, or recombinant interleukin-11, is a megakaryocyte growth factor that promotes platelet production. Erythropoietin (B) and darbepoetin alfa (A) are both stimulators of red blood cell production, while filgrastim (C) and sargramostim (E) stimulate granulocyte production.
Q 20.4: An 8-month-old male presents to the clinic due to irritability, fatigue and parental concerns about developmental delays. He is fifth percentile for weight and 33rd percentile for height. An office based hemoglobin level is 8.4 mg/dL. Which of the following is the most common side effect the infant may experience after starting oral iron supplementation therapy? A Anaphylaxis B Clay-colored stools C Flushing D GI upset E Melena
The Correct Answer is: D Oral iron commonly causes GI upset. Anaphylaxis (A) commonly occurs with parenteral iron therapy. Stools will darken and not be clay colored (B), which occurs in the absence of bile production. Flushing (C) is a common side effect of niacin. Dark stools will test falsely positive for blood on hemoccult tests and not be true melena (E) with blood present in the stool.
Q 59.3: You are caring for a 29-year-old G1P0 at who is pregnant with twins. She has received routine obstetrical care and her pregnancy has been uneventful to date. What is the average gestation age for twins at delivery? A 33 to 34 B 39 to 40 C 35 to 36 D 36 to 37 E 37 to 38
The Correct Answer is: D The average length of gestation for a single fetus is 40 weeks; the average age of gestation decreases with increasing number of fetuses.
Q 105.9: What is the radiographic pattern of a pneumonia that is caused by Pneumocystosis jiroveci in an HIV patient? A Lobar consolidation B Consolidation with effusion C Cavitary lesion with infiltrate D Diffuse interstitial infiltration E Lingular infiltrate
The Correct Answer is: D The presentation of diffuse interstitial infiltrates is what differentiates the patient from a typical pneumonia patient, including the presentation of the history, followed by the chest x-ray results. (Shelburne SA, Hamill RJ. Medical Diagnosis and Treatment, 2011, Chapter 36, Mycotic Infections)
Q 26.3: A 39-year-old male patient presents with low back pain with radiation to the right leg. On examination you place the right hip in a flexed position, and as you palpate between the iscial tuberosity and the greater tronchanter of the femur the patient complains of radiation of pain down his right leg. Based on this history and exam, which nerve was affected by this part of the examination maneuver? A Femoral B Peroneal C Saphenous D Sciatic E Sural
The Correct Answer is: D The sciatic nerve does lie midway between the ischial tuberosity and greater trochanter and it can be palpated when the patient is in a hip flexed position. The gluteus maximus obscures the nerve from being effectively palpated when the leg is in an extended position. Tenderness of the sciatic nerve can be caused by a lumbar disk herniation, direct trauma, or spasm of the nearby pyriformis muscle. The femoral nerve is a deep structure that lies lateral to the femoral artery and is not considered to be palpable. The femoral nerve is responsible for the L1-3 dermatomes and for supplying motor function to the iliopsoas muscle. The peroneal nerve originates from the sciatic nerve and splits into the superficial and deep peroneal nerves, which are responsible for much of the sensory and motor nerve function in the lower leg. The saphenous nerve originates from the femoral nerve in the femoral triangle and runs down the medial aspect of the leg. The sural nerve has medial and lateral components that are found in the lower leg. The medial cutaneous sural nerve arises from the tibial nerve just below the knee and eventually connects with peroneal nerve to form the sural nerve. On the lateral side of the lower leg, the sural nerve arises from the common peroneal nerve just above the knee and eventual connects with the previously discussed medial branch to form the sural nerve.
Q 37.2: A 5-year-old boy presents with a tense, fluid filled blister on his fingertip. What organism is most likely to be found when the lesion is cultured? A Herpes simplex virus B Parvovirus B19 C Pseudomonas aeruginosa D Staphylococcus aureus
The Correct Answer is: D This child has blistering distal dactylitis. This is a form of impetigo, and is caused by either streptococcus pyogenes or staphylococcus aureus.
Q 121.1: A 22-year-old woman, with no previous medical problems, suddenly cried out, fell to the ground, extended her legs, flexed her arms, and jerked her extremities for 30 seconds. There was associated tongue biting and urinary incontinence. She awoke slowly over a 10-minute period and recalled nothing about the episode. She remained lethargic for several hours but the rest of her neurologic examination was normal. What is the most likely etiology for this episode? A epilepsy B hyperventilation C cardiac arrhythmia D seizure E stroke
The Correct Answer is: D This event represents a well-demarcated episode affecting some combination of consciousness, motor, and/or sensory function consequent to abnormal electrical discharges in the brain. This is consistent with the definition of a seizure. Epilepsy refers to multiple, recurrent seizures. This history is not consistent with hyperventilation, stroke, or cardiac arrhythmia, which would typically include chest pain, shortness of breath, dyspnea on exertion, or focal neurological deficits. (Aminoff et al., 2005, p. 265) Aminoff MJ , Greenberg DA , Simon RP. Clinical Neurology. 6th ed. New York, NY: McGraw-Hill; 2005.
Q 39.11: The parents of a four-month-old child present with concern regarding a birthmark on the child's scalp and left side of face. The lesion has been present since birth and is growing. It has an orange, pebbly appearance. What is the appropriate treatment? A CT scan with contrast to rule out underlying brain malformation B laser therapy C no treatment, as the lesion will spontaneously resolve D excision prior to puberty due to increased risk of basal cell carcinoma
The Correct Answer is: D This lesion is consistent with a nevus sebaceous. It will grow as the child grows. After puberty, the lesion becomes thicker with a warty appearance. There is a slight increase in incidence of basal cell carcinoma as the patient ages. It is recommended that the lesion be excised prior to puberty, when these changes occur in response to hormone secretion. It is confined entirely to the skin; therefore, a CT is not necessary. Laser therapy is not an effective treatment for a nevus sebaceous. It requires complete excision. It also has an increased risk of basal cell carcinoma. It will not spontaneously resolve.
Q 80.6: A 24-year-old male is brought to the emergency department by his girlfriend. She states that he began having a seizure in the car approximately seven minutes ago. She is not sure of his medications, but states he has a known seizure history and has seizures a few times a year. The patient is currently actively seizing. Which of the following is the first-line agent to give this patient? A Carbamazepine B Gabapentin C Levetiracetam D Lorazepam E Phenytoin
The Correct Answer is: D This patient has a known history of seizures, with current ongoing seizure activity and the concern of status epilepticus, a true emergency. Prolonged seizure activity is associated with hyperthermia, metabolic disturbances, cardiopulmonary dysfunction, and irreversible damage. Lorazepam, a benzodiazepine, increases the activity of gamma-aminobutyric acid (GABA) in the brain, thereby calming the excessive electrical nerve activity related to the seizure and slowing the central nervous system. It is the preferred first line agent for temporary control of seizure activity due to the duration of action of approximately 12 to 24 hours. This allows additional therapeutic agents to be administered while gaining control of the seizure activity. The other agents are antiepileptic medications, which are used for long-term seizure control. They are differentiated by efficacy, convenience, side effects, and drug-drug interactions. (Fauci et al., Harrison's Principles of Internal Medicine, 17e, Chapter 363, Seizures and Epilepsy)
Q 17.10: A 12-year-old boy is being seen for concerns of development of breast tissue. Upon physical exam, he is noted to have a firm, slightly tender mass under the left areola. What is the most appropriate action at this time? A referral to pediatric surgery for resection B measurement of serum hCG C measurement of testosterone and estrogen levels D reassurance and observation
The Correct Answer is: D Type 1 idiopathic gynecomastia in adolescent men presents with a firm mass under the areola ("breast bud") typically during sexual maturation stages (SMR), stages II to III. This is a result of normal estrogen and androgen activity at the breast tissue level. Appropriate action is observation and to reassure the patient that the condition will likely resolve in 1 to 2 years.
Q 97.7: You are providing care for a 21-year-old G1P1 who delivered a 3990 gram infant by normal spontaneous vaginal delivery. After delivery, she continues to bleed vaginally, more than is expected for routine delivery. What is the most likely cause of her hemorrhage? A cervical laceration B lateral vaginal wall laceration C retained placenta D uterine atony E placenta acreata
The Correct Answer is: D Uterine atony continues to be the most common cause of postpartum hemorrhage, even though all of the answer choices may cause it.
Q 46.10: What is the most common pathogen in pediatric bronchiolitis? A Mycoplasma B Chlamydia C Parainfluenza D Respiratory syncytial virus (RSV) E Streptococcus
The Correct Answer is: D Viruses are by far the most common reasons for bronchiolitis to occur. RSV is the clear leader in causative agents.
Q 14.3: A 67 year-old man with chronic obstructive pulmonary disease is prescribed ipratropium bromide inhaler. Which of the following is a potential side effect associated with ipratropium? A Bradycardia B Diarrhea C Salivation D Sweating E Urinary retention
The Correct Answer is: E Ipratropium is an anticholinergic agent that can directly cause tachycardia, ileus, dry mouth, flushed/warm/dry skin, delirium, and urinary retention (E). Answer choices A, B, C and D result from cholinergic agents.
Q 117.17: Which of the following is the most prevalent fatal cancer in the United States? A Cervical B Colon C Esophageal D Liver E Lung
The Correct Answer is: E Lung cancer continues to lead as the major cause of cancer deaths in both men and women in the United States, with cigarette smoking causing greater than 90% of cases. Despite educational campaigns highlighting the risks of smoking, lung cancer continues to kill more individuals that colorectal, breast, and prostate cancers combined. (McPhee SJ, Papadakis MA. Current Medical Diagnosis & Treatment 2011, Chapter 39, Cancer)
Q 104.5: Which of the following antibiotics is the most appropriate treatment for antibiotic-associated colitis? A oral ciprofloxacin B intravenous vancomycin C oral sulfasalazine D intravenous penicillin E oral metronidazole
The Correct Answer is: E Oral metronidazole is the drug of choice. Both vancomycin and metronidazole are effective; however, metronidazole is less expensive and there is less of a concern for vancomycin resistance. (Dipiro, 2008, p. 1863) Dipiro JT , Talbert RL , Yee GC , et al. , Pharmacotherapy: A Pathophysiologic Approach. 7th ed. New York, NY: McGraw-Hill; 2008.
Q 98.47: A 59-year-old woman presents to the emergency department with an acute upper gastrointestinal hemorrhage. Her medical history is pertinent for peptic ulcer disease for the past 5 years and hypertension. A nasogastric tube is inserted and bright red blood is seen. Her vital signs are BP: 110/70 mm Hg, P: 94/min, R: 14/min, oxygen saturation: 97%, T: 99°F. Which of the following diagnostic studies would be the most appropriate next step to determine the site of bleeding? A Abdominal and pelvic computed tomography B Abdominal ultrasound C Upper gastrointestinal series with barium D Bleeding scan E Upper endoscopy
The Correct Answer is: E Patients who present with upper gastrointestinal bleeding and shock requiring multiple transfusions in 24 hours are at high risk for mortality from gastrointestinal bleeding. The upper gastrointestinal bleeding in this patient warrants further investigation with upper gastrointestinal endoscopy to both determine the site of bleeding and provide potential therapy by endoscopic electrocautery or injection.
Q 49.1: Which of the following is a common adverse effect associated with the use of stimulants such as methylphenidate for attention-deficit hyperactivity disorder (ADHD)? A diarrhea B hypoglycemia C hypotension D paresthesias E reduced appetite
The Correct Answer is: E Stimulants (eg, amphetamines, methyl-phenidate) are considered first-line therapy in the majority of cases of ADHD. Both amphetamines and methylphenidate block dopamine and norepinephrine reuptake, while amphetamines also stimulate norepinephrine release. Elevated levels of CNS norepinephrine have been associated with an anorexigenic effect, leading to reduce caloric intake.
Q 111.5: A 50-year-old male states that his eye is bothering him since yesterday. He complains of pain and redness. He states that he mowed his lawn yesterday and that it was windy outside. He attempted to irrigate the eye but still has significant irritation. He notes that it hurts to blink his eyes. What is the correct sequence of steps to treat this condition? A Anesthetic drops, irrigate the eye, and perform tonometry B Prescribe antibiotic cream and pain medication C Fluorescein stain, irrigate the eye, and prescribe antibiotic cream D Fluorescein stain and lid eversion E Anesthetic drops, fluorescein stain, and lid eversion
The Correct Answer is: E The history suggests a retained foreign body to the upper eyelid. A fluorescein stain will reveal significant superficial vertical scratches on the cornea. An upper eyelid eversion must be done, to inspect for and remove the foreign body. If the practitioner is successful in removing the foreign body, relief of the irritation will be immediate. (Tintinalli et al., Chapter 236)
You are evaluating a patient who is having decreased sensation to his arm after he sustained a head on injury while playing football. He is stating that his right arm has decreased sensation that goes into the hand, but he is slightly vague on being descriptive. As you perform the physical examination, what part of the body would exhibit sensation for the C7 component of the brachial plexus? A Lateral forearm B Lateral upper arm C Medial forearm D Medial upper arm E Third Finger
The Correct Answer is: E The sensory nerve emanating from the C7 level is responsible for sensation of the third finger. The lateral forearm is supplied by C6. The lateral upper arm is supplied by C5. The medial upper arm is supplied T1 and the medial forearm is supplied by C8.
Q 119.4: Which medication is the treatment of choice for symptomatic patients with hypertrophic cardiomyopathy? A Calcium channel blockers B Nitrates C Thiazide diuretics D Alpha antagonists E Beta-blockers
The Correct Answer is: E The use of beta-blockers in symptomatic hypertrophic cardiomyopathy is useful for gaining rate control. This will allow for the optimal amount of filling in order to maintain enough of an ejection fraction. (Bashore et al., Current Medical Diagnosis and Treatment, Chapter 10)
Q 107.6: A 28-year-old female presents complaining of progressive vision loss and mild eye pain in the right eye. She states that her vision is most impacted centrally. She denies any injury, chemical exposure, contact lens use, recent illness, or history of similar symptoms. She has had some diplopia and dysequilibrium for the past two weeks. Her visual acuity reveals 20/50 in her right eye, 20/20 in her left eye, and 20/30 together. She has no conjunctival, scleral, or fundoscopic changes. Her intraocular pressure is 14 bilaterally. What is the most likely diagnosis? A Conjunctivitis B Glaucoma C Keratitis D Macular degeneration E Optic neuritis
The Correct Answer is: E This patient presents with classic symptoms for optic neuritis. Symptoms may include sudden or progressive unilateral vision loss, often with central field defect (scotoma). Pain may be present with eye movement. Vision changes will vary in severity, based on the level of inflammation. The majority of optic neuritis cases are retrobulbar, resulting in a normal fundoscopic exam. There is often a loss of color differentiation, and the affected eye may have decreased papillary light reactivity. The patient also exhibits additional symptoms that are suggestive of possible demyelinating disease (such as Multiple Sclerosis), which are strongly associated with optic neuritis. Conjunctivitis would have associated conjunctival injection, a clear cornea, no visual acuity changes, and would likely be associated with allergic, viral, or bacterial symptomology. Acute glaucoma would likely be associated with marked vision loss, headache, a fixed pupil, corneal cloudiness, and conjunctival injection. In addition, the intraocular pressure would be increased. Keratitis often presents with vision loss, pain, photophobia, conjunctival injection, stromal infiltrate, and corneal disruption. Macular degeneration develops gradually with increasing age, causing bilateral central visual loss. (McPhee SJ, Papadakis MA. Current Medical Diagnosis & Treatment 2011, Chapter 7, Disorders of the Eyes & Lids)
Q 109.6: A patient with chronic cluster headaches is on verapamil for preventive therapy. Which of the following is a potentially harmful complication of this medication? A Cardiovascular thrombotic events B Coronary artery vasospasm C Hyperkalemia D Increased neuromuscular transmission E Slowed conduction at the AV node leading to heart block
The Correct Answer is: E Verapamil, a calcium channel blocker, has been shown to benefit patients with chronic cluster headaches. The patient should have a baseline EKG and EKG monitoring while on this medication, due to an increased risk of developing heart block, which is based on the medication's effect on the atrioventricular node. The other complications are not consistent with this medication. (Fauci et al., Harrison's Principles of Internal Medicine, 17e, Chapter 15, Headache)
Q 54.4: A 24-year-old male has an eight-month history of loose thought associations, social withdrawal, auditory hallucinations, and deterioration in his personal appearance and hygiene. Upon examination, he is noted to have a flat affect, perceptual distortions, and behaves like he is detached from his own actions. Which of the following medications used in treatment of this disease has the risk of prolonging the QTc interval? A Aripiprazole (Abilify) B Chlorpromazine (Thorazine) C Loxapine (Loxitane) D Quetiapine (Seroquel) E Ziprasidone (Geodon)
The Correct Answer is: E Ziprasidone is the only medication of the ones listed that is known to prolong the QTc interval. An ECG is recommended for patients at risk for cardiac sequelae.
Q 11.9: A 36-year-old nurse states she will be traveling to Honduras for a medical mission and wants to ensure that she has adequate protection against hepatitis B. Which of the following laboratory values is consistent with immunity against hepatitis B from prior immunization? A +HBsAg, +anti-HBc IgM B +HBsAg, +anti-Hbc IgG C +anti-HBs D +anti-HBs, +anti-HBIgG E +anti-HCV
The correct answer is (C). The presence of detectable anti-HBs indicates that she is immune from a previous hepatitis B vaccine. Booster immunizations are not indicated in her case. Choice (A) suggests acute hepatitis B. Choice (B) suggests she has highly infective chronic hepatitis B. Choice (D) suggests she had previous hepatitis B infection, recovered. Choice (E) suggests acute hepatitis C.
Q 67.1: Which of the following lab test is considered the "gold standard" for detecting/diagnosing an inherited thrombophilia? A anticardiolipen deficiency B factor V Leiden deficiency C hyperhomocysteinemia D protein C and S deficiency E There is no single lab test.
The correct answer is (E). There is no single "gold standard" lab test for diagnosis of acquired/hereditary thrombophilia.
Q 24.8: Patients with suspected familial hypercholesterolemia have serum cholesterol levels > 300 mg/dL and are at increased risk of atherosclerosis. Which of the following physical exam findings are nearly pathognomonic for familial hypercholesterolemia? A Tendon xanthomas B Lipomas C Bouchard's nodes D Carotid bruits E Visceral obesity
The correct choice is A, tendon xanthomas. These are depositions of cholesterol rich substances that can present in any tendon as a mass-like lesion. They are most commonly found in the Achilles, patellar, and hand extensor tendons. Choice B, lipomas, are benign, soft, moveable subcutaneous tumors made from fat cells. Choice C, Bouchard's nodes, are painless nodules on the PIP joints, commonly seen in patients with osteoarthritis. Choice D, carotid bruits, may be heard with auscultation of the neck during the physical exam in patients with artherosclerosis of the carotid arteries. Choice E, visceral obesity, is a risk factor for diabetes and atherosclerosis, but is not pathognomonic of familial hypercholesterolemia.
Q 44.4: A 4-year-old child is being worked up for type 1 diabetes mellitus. In addition to ordering a fasting plasma glucose and hemoglobin A1c, you would like to determine the presence or absence of any immunologic markers. Which of the following autoantibodies are common in patients with type 1 diabetes mellitus? A Anti-double stranded DNA antibodies B Antinuclear antibodies C Islet beta cell autoantibodies D Pancreatic polypeptide producing cell antibodies E Thyroid stimulating antibodies
The correct choice is C, islet beta cell autoantibodies. Islet cell autoantibodies are seen in greater than 75% of patients with type 1 diabetes. These include several different antibodies and serve as a marker of the autoimmune process in the disease. Choice A, anti-double stranded DNA antibodies, is a marker for certain connective tissue disorders, such as systemic lupus erythematosis. Choice B, antinuclear antibodies, are used as a screening test for many different connective tissue disorders, such as systemic lupus erythematosis and scleroderma. Choice D, pancreatic polypeptide producing cell antibodies, are similar to beta cells, but are not involved in the autoimmune process. Choice E, thyroid stimulating antibodies, are found in patients with hyperthyroidism caused by Graves' disease.
Q 23.7: Which of the following pairs of conditions constitute 90% of the causes of adult hypercalcemia? A Vitamin D intoxication and renal failure B Lithium ingestion and hyperthyroidism C Prolonged immobilization and thiazide use D Hypothyroidism and hypoparathyroidism E Primary hyperparathyroidism and malignancy
The correct choice is E, primary hyperparathyroidism and malignancy. Together, they constitute 90% of the causes of hypercalcemia. Choices A, B, and C can cause hypercalcemia, but at a lower rate. Choice D, hypothyroidism and hypoparathyroidism, do not cause hypercalcemia.
Q 48.8: An 11-year-old boy has a history of early growth failure and cherubic features. His height velocity chart is shown. What is the most likely cause of the sharp increase in the growth curve at age six? (Figure 7-10, Greenspan's Basic and Clinical Endocrinology, 2007) A Pituitary adenoma resection B Glucocorticoid replacement C Insulin therapy D Ensure supplementation E Recombinant human growth hormone
The correct choice is E, recombinant human growth hormone. This patient had a classic presentation of growth hormone deficiency. Replacement of growth hormone will result in marked improvement in growth, especially during the first year of treatment. Replacement therapy in most cases can allow for heights reaching genetic potential. Choice A, pituitary adenoma resection, is the treatment of choice for acromegaly. Choice B, glucocorticoid replacement, is not indicated in this patient. Choice C, insulin therapy, could possibly worsen this patient's condition. Patients with growth hormone deficiency are at risk of hypoglycemia. Choice D, ensure supplementation, is helpful for patients needing nutritional supplementation. This patient's short stature is not nutritional in nature.
What is the most likely reason for a patient to have cardiogenic shock? A Acute myocardial infarction B Sepsis C Trauma D Aortic dissection E Pericardial effusion
*The Correct Answer is: A While trauma can account for a patient being subjected to cardiogenic shock, the overwhelming etiology for the shock comes from an MI.*
A 24-year-old HIV-positive man comes to the emergency department complaining of severe left-sided chest discomfort, which radiates through to the left trapezius region. On coming into the room, you note that he is sitting up and hunched forward. Prior to examining him, you have reviewed his chart. Laboratory findings demonstrate troponins x 3, which are negative for myocardial ischemia. His EKG demonstrates diffuse ST segment elevations throughout. Which of the following physical exam findings would be most likely in this patient? A Roth spots B Splenic enlargement C Janeway lesions D Pericardial friction rub E Splinter hemorrhages
*The Correct Answer is: D Choice D is the most likely finding, as this patient is exhibiting signs, symptoms, and EKG findings pathognomonic for acute pericarditis, which is likely infectious in the setting of a patient with HIV.* A pericardial friction rub is heard best with the patient in a seated position, during expiration, and is frequently found in patients with pericarditis. Choice A B, C, and E are physical exam findings seen in acute bacterial endocarditis.
Which medication is the treatment of choice for symptomatic patients with hypertrophic cardiomyopathy? A Calcium channel blockers B Nitrates C Thiazide diuretics D Alpha antagonists E Beta-blockers
*The Correct Answer is: E The use of beta-blockers in symptomatic hypertrophic cardiomyopathy is useful for gaining rate control.* This will allow for the optimal amount of filling in order to maintain enough of an ejection fraction.
Q 82.9: A 55-year-old patient presents with symptoms of polyuria and is worried that he may have diabetes mellitus. His past medical history includes hypertension and lung cancer. His family history is negative for diabetes mellitus. He works as an accountant and has no history of head trauma. He has an average diet and fluid intake. His fasting plasma glucose is 110 mg/dL and his hemoglobin A1c is 5%. What is the most likely cause of his polyuria? A Cancer related diabetes insipidus B Type 2 diabetes mellitus C Nephrogenic diabetes insipidus D Familial hypothalamic diabetes insipidus E Primary polydipsia
A The correct choice is A, cancer related diabetes insipidus. Solid tumors, such as cranipharyngiomas and metastatic lung and breast cancer, are known to cause diabetes insipidus. This patient has a history of lung cancer. Choice B, diabetes mellitus, is not present in this patient. His pasting glucose and hemoglobin A1c are within the reference range. Choices C, nephrogenic diabetes insipidus, and choice D, familial hypothalamic diabetes indipidus, are causes of polyuria. Both are noted in young patients. Choice E, primary polydipsia, is associated with excessive drinking of liquids, which leads to excessive urination. There is no history of this occurring in this patient. (Robinson et al., 2007, Chapter 6)
Q 99.10: A 40-year-old female presents to your office with symptoms of weight gain, hirsuitism, and easy bruising. Past medical and surgical history is noncontributory. She drinks one glass of wine on weekends and does not smoke cigarettes. She takes one multivitamin daily. Upon physical exam, you note facial fullness, central obesity, and thin skin. Which of the following is a valuable biochemical screening test for this patient that will aide in the diagnosis? A Dexamethasone suppression test B Radioactive iodine uptake C Glucose tolerance test D Cosyntropin stimulating test E Plasma fractionated free metanephrines
A The correct choice is A, dexamethasone suppression test. This patient is presenting with classic signs and symptoms of Cushing's syndrome. The dexamethasone suppression test is a simple test of the hypothalamic-pituitary-adrenal axis, and requires ingestion of oral dexamethasone at nighttime and a blood test in the morning hour, to measure the amount of plasma cortisol. Most patients with Cushing's syndrome demonstrate a lack of normal axis suppression and present with a morning plasma cortisol level >5 mcg/dL. Choice B, radioactive iodine uptake, is used in patients with suspected thyroid disorders. Choice C, glucose tolerance test, is used in patients with suspected diabetes mellitus and in prenatal testing, to investigate gestational diabetes. Choice D, cosyntropin stimulating test, is used to investigate possible adrenal insufficiency. Choice E, plasma fractionated free metanephrines, is used in the diagnostic workup of pheochromocytoma. (Aron et al., 2007, Chapter 10) (Fitzgerald et al., 2011, Chapter 26)
Q 89.3: Which of the following types of thyroid cancer involve the parafollicular cells (or C cells) of the thyroid? A Medullary B Papillary C Follicular D Lymphoma E Anaplastic
A The correct choice is A, medullary thyroid carcinoma. Unlike papillary (choice B) and follicular (choice C) thyroid cancers, which arise from thyroid epithelial cells, medullary thyroid cancer arises from the parafollicular cells of the thyroid. Anaplastic thyroid cancer, choice E, arises from undifferentiated cells. Thyroid lymphoma, choice D, is most commonly of B cell follicle center cell origin. (Lee et al., 2008, Chapter 41)
Q 120.6: Your patient is taking atorvastatin 40 mg daily for her history of hyperlipidemia. Based on the potential side effects of this medication what labs are indicated for periodic monitoring? A HGB/HCT B AST/ALT C WBC count D TSH/T4 E B12
B The correct answer is (B). Atorvastatin is an HMG-CoA reductase inhibitor, which has a potential to cause liver injury. It is recommended that liver enzymes are monitored regularly due to the potential for liver injury. The remaining choices are incorrect because statins are not known to cause changes in these laboratory values and monitoring based on potential adverse reactions to atorvastatin is not recommended. (McPhee and Papadakis, 2011, Chapter 28)
Q 61.4: A patient was recently diagnosed with acromegaly. He was reading on the Internet that acromegaly is associated with diabetes mellitus. He is especially concerned, since his father was recently diagnosed with diabetes as well. How would you reply to the patient's following question: "What percentage of people with acromegaly develop diabetes mellitus?" A 10% B 25% C 50% D 65% E 80%
B The correct choice is B, 25%. Growth hormone is a counter-regulatory hormone of insulin, and therefore acts against insulin. This can lead to hyperglycemia, glucose intolerance, and diabetes mellitus in 25% of patients with acromegaly.
Q 58.4: Patients prescribed which of the following medications may present with a dorsocervical fat pad, thin extremities, and central obesity, which may mimic Cushing's syndrome? A ACE inhibitors B HIV antiretrovial medications C Loop diuretics D Opioid analgesics E Aminoglycosides
B The correct choice is B, HIV antiretroviral medications. Patients using highly active antiretroviral therapy (HAART) can develop partial lipodystrophy and changes in body fat distribution. This complication occurs more commonly with long-term use, and affects 1/3 to 2/3 of patients on therapy for longer than one year. The other medication classes noted are not associated with lipodystrophy. Opioids, choice D, are associated with a decline of cortisol secretion.
Q 110.10: A patient is admitted to the hospital for an acute adrenal crisis. He has a history of chronic adrenal insufficiency and was admitted with severe weakness, nausea and vomiting while fighting a pulmonary infection. Which of the following suggested plans might this patient have forgotten or been unable to do? A Restrict fluid intake during times of metabolic stress B Increase the daily dose of hydrocortisone during times of metabolic stress C Hold the daily dose of hydrocortisone during times of metabolic stress D Add levothyroxine to the daily dose of hydrocortisone during times of metabolic stress E Increase the daily ingestion of proteins during times of metabolic stress
B The correct choice is B, increase the daily dose of hydrocortisone during times of metabolic stress. The cortisol dose should be increased to between 60 and 80 mg/day, to mimic the normal physiologic response of the body. Increased mineralocorticoid therapy is generally not required. Choice A, restrict fluid, would aggravate the potential for the development of dehydration in this patient. Choice C would lead to further cortisol depletion. Choices D and E would not benefit this patient. (Aron et al., 2007, Chapter 10)
Q 105.4: A new patient is seen in your internal medicine office today. She is coming in to request the removal of several skin tags. She is a 55-year-old woman with a history of untreated acromegaly. A health maintenance plan is set up with the patient, and includes a colonoscopy. This patient is at increased risk for which of the following findings on colonoscopy? A Anal fissures B Ulcerative colitis C Colon polyps D Pseudomembranous colitis E Colonic fistulas
C The correct choice is C, colon polyps. Approximately 30% of patients with acromegaly have been found to have colon polyps. These patients also have an increased risk of colon cancer. Patients with acromegaly have not been found to be at increased risk for the other response choices listed here. (Melmed et al., 2008, Chapter 333)
Q 82.7: Which of the following cardiac medications is known to cause clinically significant hypothyroidism? A Furosemide B Captopril C Amiodarone D Digoxin E Dopamine
C The correct choice is C; Amiodarone is an antiarrythmic medication used to treat patients with recurrent ventricular tachycardia or fibrillation. Its structure is similar to thyroid hormone, and its metabolites antagonize thyroid hormone function in approximately 13% of patients treated with amiodrone in the United States. Choice A, furosemide, choice B, captopril, choice D, digoxin, and choice E, dopamine, have no effect on thyroid function. (Fitzgerald et al., 2011, Chapter 26) (Katzung et al., 2009, Chapter 13) (Roden et al., 2006, Chapter 34; 2008, Chapter 335)
Q 19.7: A 48 year-old male presents to the clinic with a history of productive cough for the past 3 days. Which of the following factors if present best predicts the diagnosis of pneumonia in this patient? A Adenopathy B Double sickening C Dyspnea D Fever E Hoarseness
Cough, fever (D) and sputum production are classic symptoms of pneumonia. Adenopathy (A) and hoarseness (E) may coexist in some cases of viral pneumonia. Dyspnea (C) may develop as pneumonia progresses. Double sickening (B) is a clinical consideration in the differentiation of acute sinusitis from upper respiratory infection that typically occurs after approximately 7 days of illness.
Q 91.5: A 32-year-old female returns to the office following an emergency department visit for abdominal pain that has since resolved. A computed tomography (CT) scan performed in the emergency department reveals a 1 cm cavernous hemangioma in the right lobe of the liver. What are your recommendations to the patient regarding the CT findings? A immediate referral to oncologist B referral to GI for liver biopsy C referral to surgeon for resection D reassurance that no treatment is necessary E repeat CT scan in 3 months
D The correct answer is (D). Cavernous hemangiomas are common benign lesions of the liver that rarely require surgical intervention unless they are very large (>10 cm) and symptomatic. No further imaging follow-up is necessary. (Greenberger et al, 2009, Chapter 49)
Q 67.3: A 38-year-old male with a history of frequent urinary tract infections (UTIs) and kidney stones is following up with the urology physician assistant. His urine today continues to reveal persistent hematuria, which he had the last few times he saw his primary care physician. His vitals are 120/80, P = 72, T = 96.7˚F. He denies any discomfort at this time. GFR is normal. You review a urogram, which reveals a paint-brush appearance of the papillae. The kidneys are normal in size. What is the most likely diagnosis? A chronic renal insufficiency B renal cell carcinoma C end stage renal disease D medullary sponge disease E polycystic kidney disease
D The correct answer is (D). The patient's history of hematuria, recurrent UTIs, and kidney stones are a common presentation of this inherited disease with medullary cysts. The findings on the urogram of a paint-brush appearance of the papillae are pathognomonic of the disease. Choices (A) and (C) are unlikely due to the normal GFR. The patient's history is not suggestive of choice (B), renal cell carcinoma. A solitary solid renal mass would suggest renal cell carcinoma. (Lerma, et al., 2008, Chapter 46)
Q 73.5: One of your patients is requesting your help. He has seen three physicians in the past 3 months and is still having symptoms. He states, "I keep having these episodes of feeling like I am going to die. Out of the blue I feel real nervous, I get a splitting headache, break out in a sweat, and even feel like I am trembling. Sometimes I feel my heart beating out of my chest but don't have any pain or shortness of breath." He has had numerous tests including a cardiac stress test, multiple EKGs, complete blood count (CBC), basic metabolic panel (BMP), thyroid tests, and computed tomography (CT) scans of his head, which were normal. His last physician told him that he had an anxiety disorder and should try medications; he declined. His only medical problem is hypertension, which has worsened recently. He is currently taking lisinopril 20 mg QD, amlodipine 10 mg QD, and HCTZ 25mg QD. His physical examination is unremarkable except a BP of 190/92, P = 74. What is the best test to order to confirm your suspected diagnosis? A dexamethasone suppression test B sleep study C renal artery ultrasound D plasma fractionated free metanephrines E thyroid uptake scan
D The correct answer is (D). The patient's symptoms and uncontrolled hypertension with a previous negative evaluation for cardiac, electrolyte, or thyroid causes highly suggests a pheochromocytoma as the possible cause. Plasma fractionated free metanephrines is a very sensitive test for the diagnosis of pheochromocytoma. Another test to consider would be evaluation of the total urinary metanephrines. A dexamethasone suppression test, choice (A), is used to r/o Cushing's syndrome, which is unlikely due to the normal physical examination findings. A sleep study, choice (B), (suggesting sleep apnea) and renal artery ultrasound, choice (C), (suggesting renal artery stenosis) are used to diagnose secondary hypertension, but would unlikely explain all the symptoms in the scenario. A thyroid uptake scan, choice (E), is not indicated since there is no indication of abnormal thyroid testing suggesting hyperthyroidism, or examination stated findings suggestive of a thyroid nodule. (McPhee and Papadakis, 2011, Chapter 26)
Q 97.48: A 33-year-old G1P0 presents for evaluation of her inability to conceive a pregnancy for six months. She menstruates monthly. Her past medical history is significant for PID x 2, for which she was hospitalized for IV antibiotics. Her Chlamydia, GC, and pap smear are normal. When should a more comprehensive evaluation for her infertility begin? A Now B 6 months C 12 months D 18 months E 24 months
The Correct Answer is: A A comprehensive work up should begin now, due to her advancing age and history of significant PID, which may require surgical treatment.
Q 39.15: A newborn male is diagnosed with Christmas factor deficiency. What is the likelihood that he inherited this disorder from his father? A 0% B 25% C 50% D 75% E 100%
The Correct Answer is: A All daughters of a hemophilic male are carriers of hemophilia, whereas all sons are normal. Hemophilia B (or Christmas factor deficiency) is one of only two sex-linked pattern-bleeding disorders, and as such the disease occurs almost exclusively in males. Sons of carriers have a 50% chance of being affected and daughters of carriers have a 50% chance of being carriers themselves.
Q 35.1: A 3-year-old white male with low grade fever, lethargy, and fatigue for several weeks is seen on repeat visit. His mother states that nothing has helped. He has been trialed on a course of antibiotics without improvement. He has petechiae to his lower extremities and pallor throughout. What other physical finding is likely to be found in this patient? A Bone pain B Cranial nerve palsy C Epidural spinal cord compression D Painless enlargement of the scrotum E Subcutaneous nodules
The Correct Answer is: A All physical findings are possible in this patient, who has acute lymphoblastic leukemia (ALL). However, of those listed, bone pain is the most common. All other choices are extremely rare findings (usually less than 1%). In children presenting with ALL, the frequency of bone pain or limp can be more than 25% of cases.
Q 119.7: A patient describes a desire for close relationships and to be more successful at work. However, she views herself as being undesirable and inferior. Because of these feelings she avoids social activities and extra occupational projects out of fear of criticism, rejection, and embarrassment. Which diagnosis would best fit this description? A avoidant personality disorder B borderline personality disorder C histrionic personality disorder D schizoid personality disorder
The Correct Answer is: A An individual with avoidant personality disorder differs from schizoid in that they desire interaction and closeness but are unable to overcome their deep seated self-beliefs and fears. They tend to be less impulsive and more stable than borderline personality disorder patients and have less of a need to be the center of attention than those with histrionic personality disorders. (Sadock and Sadock, 2008, p. 385) Sadock BJ , Sadock VA. Concise Textbook of Clinical Psychiatry, 3rd ed. Philadelphia, PA: Lippincott, Williams & Wilkins; 2008.
Q 108.8: A 26-year-old female is returning for a follow-up visit for gastroesophageal reflux disease. She has been seen in the office frequently over the past three months for symptoms associated with reflux, but she has also exhibited signs and symptoms that are consistent with a personality disorder. In particular, she is withdrawn, shy, introverted, and avoids close relationships. These signs are most consistent with which personality disorder? A Avoidant B Dependent C Histrionic D Schizoid E Schizotypal
The Correct Answer is: A Avoidant personality disorder presents clinically as someone who fears rejection, overreacts to rejection and failure, and has poor social endeavors and low self-esteem. Signs of a dependant personality disorder include someone who lacks confidence and self-esteem, has difficulty making decisions, and is passive and overaccepting. Clinical findings of histrionic personality disorder include being dependent, immature, seductive, egocentric, vain, and emotionally labile. Schizotypal disorder is characterized by being superstitious, socially isolated, and suspicious, and having limited interpersonal ability, odd speech, and eccentric behaviors. (McPhee SJ, Papadakis MA. Current Medical Diagnosis & Treatment, 2010, p. 951)
Q 71.6: A patient presents to the clinic with a family member. Upon obtaining history from the patient, he responds with excessive details of his symptoms and the reason for his visit. He is unable to answer a question directly without signification elaboration. What problem does this patient have? A circumstantiality B derailment C incoherence D tangentiality
The Correct Answer is: A Circumstantiality is seen in someone who eventually gets to the point after a delay in the thought process. Tangentiality is a disturbance in thought causing the person to start a train of thought, but never getting to the point. Derailment is when a patient skips to another subject. This mainly occurs if a topic is brought up that the patient does not wish to discuss. (Nurcombe and Ebert, 2008, p. 48) Nurcombe B , Ebert MH. The psychiatric interview. In: Ebert MH , Loosen PT , Nurcombe B , Leckman JF, eds. Current Diagnosis and Treatment in Psychiatry. New York: McGraw-Hill; 2008.
Q 76.3: A 24-year-old male has an eight-month history of loose thought associations, social withdrawal, auditory hallucinations, and deterioration in his personal appearance and hygiene. Upon examination, he is noted to have a flat affect, perceptual distortions, and behaves like he is detached from his own actions. If chosen for treatment, which of the following medications would require weekly white blood cell count monitoring for the first six months? A Clozapine (Clozaril) B Haloperidol (Haldol) C Olanzapine (Zyprexa) D Risperidone (Risperdal) E Quetiapine (Seroquel)
The Correct Answer is: A Clozapine has a risk of agranulocytosis. While the risk is only 1%, weekly monitoring of the white blood cell count for the first six months, followed by monitoring of the white blood cell count every other week thereafter, is required. The other medications listed do not have the risk of agranulocytosis. (McPhee SJ, Papadakis MA. Current Medical Diagnosis & Treatment, 2010, p. 955)
Q 110.20: An 18-year-old female presents with a history of large open comedones and painful abscesses in both axillae. She has been treated by her primary care provider with oral antibiotics. A thorough physical exam may show which other affected sites? A breasts, groin, and perineum B face and scalp C only the axillae are affected D upper back, ears, and lower legs
The Correct Answer is: A Common areas of involvement in hydradenitis suppurtiva include the axillae, breasts, anogenital region, perineum, and scalp. (Wolff et al., 2009, Page 16)
Q 111.4: A 28-year-old male presents with a rash on his hands that is unresponsive to topical steroids. On physical exam you notice periocular violaceous erythema and edema. He also exhibits significant proximal muscle weakness. What is the most likely diagnosis? A Dermatomyositis B Psoriasis C Seborrheic dermatitis D Systemic lupus erythematosus
The Correct Answer is: A Dermatomyositis is an autoimmune disease that targets the skin and skeletal muscle. Skin lesions usually consist of a periorbital heliotrope rash, which can have associated edema. There can also be flat topped violaceous papules (Gottron papules) located on the neck, shoulders, and knuckles. Periungual erythema with telangiectasias may also occur. Possible muscular symptoms include muscle tenderness, muscle atrophy, and progressive proximal muscle weakness. (Wolff et al., 2009, Fig. 14-17B, pg. 370-373)
Q 97.85: A 45-year-old woman presents with weight gain, fatigue, dry skin, constipation, and oligomenorrhea. On physical exam, bradycardia and slow deep tendon reflexes are noted. Her free T 4 is low and TSH is elevated. Which of the following medications may be responsible for her condition? A amiodarone B beta-blockers C levadopa D hydrocholorthiazide
The Correct Answer is: A Hypothyroidism is reported in up to 10% of patients taking amiodarone, an antiarrhythmic medication. With the high iodine content of the medication and the structural similarities to thyroxine, thyroid abnormalities occur. Common side effects of amiodarone include bradycardia and constipation, so laboratory evaluation for thyroid dysfunction must be used.
Q 107.1: A 57-year-old man with a history of HTN, hyperlipidemia, and chronic tobacco use presents to the emergency department with complaints of worsening chest tightness over the last 2 months. He initially noticed that every time he raked leaves, he had a few minutes of chest tightness, which was relieved within 5 minutes if he rested. He now notices that raking will precipitate severe chest discomfort, diaphoresis, and dyspnea, which lasts for 20 minutes even if he rests. Last night, while watching football, he again noticed chest tightness, which began suddenly and slowly dissipated over 15 minutes. His physical examination is normal. An EKG performed during an episode of chest discomfort demonstrates normal sinus rhythm at 90 bpm with ST-segment depression. Beta blockers, IV nitroglycerin, aspirin, and oxygen are started. Serial troponin levels are negative. A repeat EKG continues to demonstrate ST segment depression, along with t-wave inversion. The patient's chest discomfort is decreased in intensity and duration, but returns periodically. Which of the following is the most appropriate next step in the management of this patient? A Cardiac catheterization B Exercise nuclear stress test C Holter monitor D Tilt table test E Transesophageal echocardiogram
The Correct Answer is: A In patients with unstable angina pectoris, if symptoms and EKG changes are not stabilized with appropriate medical therapy, including beta blockade, aspirin, oxygen, and nitroglycerin, then choice A, cardiac catheterization, with likely percutaneous coronary intervention, would be recommended, as this patient is at high risk for acute myocardial infarction. Choice B, exercise nuclear stress testing, would provide information regarding exercise tolerance and exercise-induced dysrhythmias, as well as regarding myocardial ischemia, but in this unstable patient, it would not be recommended. In patients whose conditions stabilize with medical therapy, nuclear stress testing is a viable option. Choice C, Holter monitor, is a useful diagnostic tool for evaluation of patients with palpitations occurring on a daily basis. Choice D, tilt table testing, is utilized in the evaluation of patients suffering from near-syncope or syncope. Choice E, transesophageal echocardiogram, is helpful in more direct visualization of heart valves, especially when transthoracic echocardiogram is unclear. (Fauci et al., 2008, Chapter 238)
Q 31.1: A 28-year-old male is concerned because he has a friend who was diagnosed with chronic hepatitis C and wants to know if he is at risk for this disease as well. Which of the following answers elicited in your history would make you the most suspicious? A previous intravenous drug use B blood transfusion last year C monogamous relationship D recent negative HIV test E previous history of hepatitis
The Correct Answer is: A Intravenous drug abuse accounts for over 50% of hepatitis C cases. Recent blood transfusions are unlikely to cause hepatitis C. Hepatitis C is more likely to be contracted in patients with multiple sexual partners, although the incidence of transmission via sexual intercourse is considered low. Hepatitis A is transmitted via the fecal-oral route and does not increase the chances of hepatitis C.
Q 119.13: Women who suffer from migraine with aura are at increased risk for which of the following? A deep vein thrombosis B cerebrovascular accidents C pulmonary emboli D thrombophilias E dissecting aortic aneurysms
The Correct Answer is: B Relatively recent studies show that women who have migraine with aura are at a small but statistically significant increased risk of stroke. Patients in this population should be treated even more aggressively with stroke risk-reduction strategies. (Fuster et al., 2008, Chapter 3)
Q 114.4: When initially screening for CKD, which of the following would be ordered? A 24-hour urine collection B blood pressure measurement, serum creatinine level, spot urine protein measurement C renal ultrasound D abdominal CT scan E renal angiogram
The Correct Answer is: B Screening for the presence of chronic kidney disease involves checking a serum creatinine level, checking blood pressure for the presence of hypertension, checking urinary protein for evidence of glomerular injury, and obtaining a history to check for the presence of risk factors, such as hypertension, diabetes mellitus, autoimmune disease, infection, or family history. Initial screening would not include a 24-hour urine collection. This is a cumbersome, inconvenient, more expensive test than the spot urinary protein reading and would not provide additional information. Renal ultrasound and abdominal CT scan would not be indicated in the initial stages of the work-up. These would be done only after laboratory studies were done and only if indicated. (NKF-K/DOQI Guidelines, 2002, p. 31) NKF-K/DOQI Clinical Practice Guidelines for Chronic Kidney Disease: Executive Summary. New York, NY: National Kidney Foundation; 2002.
Q 97.90: You are taking care of a 32-year-old G2P1 at 39 weeks gestation in active labor. Her pregnancy is complicated by gestation diabetes. The fetal head delivered, but the anterior shoulder did not deliver with gentle downward traction. What would be the next most appropriate action? A More forceful traction and fundal pressure B Call for assistance and McRoberts maneuver C Call for assistance and more forceful traction D Call for help and fundal pressure
The Correct Answer is: B Shoulder dystocia is an obstetrical emergency, and help should always be summoned. The McRoberts maneuver increased the AP diameter, thus accommodating a large head; subrapubic pressure can help dislodge the anterior shoulder, but simple fundal pressure continues to impact it against the pelvic bone.
Q 30.7: A 14-month-old African American boy is diagnosed with sickle cell anemia. Neither parent has the disease. If you are counseling the parents of this child on the likelihood of another one of their children having sickle cell disease, what percentage would you tell them? A 0% B 25% C 50% D 75% E 100%
The Correct Answer is: B Sickle cell anemia is an autosomal recessive genetic disorder. The presence of two defective genes is required for sickle cell anemia to occur. If each parent carries one sickle hemoglobin gene and one normal gene, each child has a 25% chance of inheriting two defective genes and developing sickle cell, a 25% chance of inheriting two normal genes and not having the disease, and a 50% chance of being an unaffected carrier.
What is the most common cause of pneumothorax in a healthy patient? A Traumatic B Infectious C Ruptured bleb D Surfactant abnormality E Malignancy
*The Correct Answer is: C Rupture of a bleb is thought to be more relevant to young, thin men, and also patients who have a family history of them, and smoking.*
Q 92.10: You are examining a 48-year-old male with a history of liver cirrhosis from chronic hepatitis C. Which of the following physical examination findings is a result of the effects of portal hypertension? A gynecomastia B caput medusae C testicular atrophy D jaundice E ecchymosis
B The correct answer is (B). Caput medusa, or dilated abdominal veins, is a direct result of portal hypertension. Other findings could include splenomegaly, hemorrhoids, and esophageal varices. The other choices are effects of liver cell failure. (McPhee and Papadakis, 2011, Chapter 16) Source: (McPhee and Papadakis, 2011, Chapter 16)
Q 121.17: Release of which of the following substances is triggered by pituitary growth hormone and promotes growth of other tissues in the body? A C-pepide B IL-I C IGF-I D Thyroxine E Catecholamines
C The correct choice is C, IGF-I or insulin like growth factor I. This growth factor leads to increased DNA, RNA, and protein synthesis, which leads to overgrowth of bone, soft tissue, and cartilage. Choice A, c-peptide, is a part of the prohormone of insulin. Choice B, IL-I or interleukin I, is an important cytokine that promotes cell activation. Choice D, thyroxine, potentiates the actions of growth hormone on tissues. (Fitzgerald et al., 2011, Chapter 26) (Aron et al., 2007, Chapter 5)
Q 27.8: A 20-year-old college football player presents with a chief complaint of a dull ache in his scrotum after prolonged standing on the sideline. It seems to get worse with vigorous activity and is relieved by lying down. Dilated veins in the left scrotum are observed on inspection, and both testicles are palpable and without masses. What is the most likely diagnosis? A varicocele B spermatocele C hydrocele D testicular mass
The Correct Answer is: A A varicocele can be recognized by the presence of scrotal enlargement caused by dilation of the pampiniform venous plexus. Varicoceles present as a "bag of worms" in the spermatic cord and are more prominent when the patient stands. More than 80% of the time, varicoceles occur on the left side. Hydroceles and spermatoceles are caused by fluid collection and are usually asymptomatic. Testicular masses must always be included in the differential diagnosis of scrotal masses, as they generally present as painless.
Q 97.17: You are treating a 14-year-old female for gonorrhea. She has a past history of Chlamydia. Her LMP was 3 weeks ago. She is allergic to penicillin. Her only medication is oral combination birth control pills. Why should Levofloxin be avoided in this patient? A Her age B Her LMP status C Her history of Chlamydia D Her oral contraceptives E Her penicillin allergy
The Correct Answer is: A Fluoroquinolones are contraindicated in pediatric patients. The risk-benefit assessment indicates that levaquin is only appropriate in pediatric patients at least 6 months of age, for treatment of inhalational anthrax (post-exposure). There are other medications, both oral and injectable, with better safety profiles that are available.
Q 105.8: A patient presents with a cerumen impaction. Which of the following is true when performing the Rinne- Weber test? A Weber lateralizes to the affected ear B Weber is equal in both ears C Weber lateralizes to the unaffected ear D Air conduction > Bone conduction in the affected ear E Air conduction = Bone conduction in the affected ear
The Correct Answer is: A For conductive hearing loss, the Weber will lateralize to the affected ear and bone conduction will be greater than air conduction. Conductive hear loss prevents sound from entering the inner ear due to obstruction in the external auditor canal and middle ear. Examples would be cerumen impaction and otitis media. Sensory neural hearing loss affects the inner ear and cranial nerve VIII. Weber will lateralize to the unaffected ear (normal ear) and Rinne will reveal Air conduction > Bone conduction. (Le Blond et al., 2009, Chapter 20)
Q 8.4: A 12 year-old African-American male with a history of sickle cell disease presents to the clinic for routine follow up. A peripheral smear is performed. Which of the following best describes the cause of the abnormality pictured in the patient's peripheral smear? (note photo taken from figure 29-12 in Harrison's) A Absent or non-functioning spleen B Failure of nuclear maturation C Intravascular hemolysis D Lead intoxication E Presence of uremia
The Correct Answer is: A Howell-Jolly bodies are noted in the peripheral smear and result from the lack of removal of nuclear material due to an absent or non-functioning spleen. Failure of nuclear maturation (B) produces macrocytosis, intravascular hemolysis (C) leads to the creation of schizocytes, lead intoxication (D) and thalassemia cause basophilic stippling, and uremia (E) is associated with Burr cells.
Q 31.10: Shoulder dislocation commonly involves injury to which nerve? A Axillary B Median C Peroneal D Radial E Ulnar
The Correct Answer is: A The axillary nerve is in close proximity to the glenohumeral joint, thus making it vulnerable to injury during a shoulder dislocation. The median, radial, ulnar nerves are more distal in the upper extremity and and the peroneal nerve is in the leg.
Q 118.2: A 29-year-old woman comes to the office because she "just keeps gaining weight and can't stop." Since she was last seen in the office at age 24, she has gained nearly 100#. Physical examination is remarkable for blood pressure of 140/92. She appears depressed. Her trunk and abdomen are heavy with normal-sized extremities. Her facial hair is dark and in a "male" distribution. Large dark violaceous striae are present on her abdomen and proximal extremities. Which of the following additional findings is most consistent with this presentation? A Buffalo hump B Doughy, thickened skin C Exophthalmos D Lid lag E Thickened tongue
The Correct Answer is: A This woman appears to have Cushing Syndrome (hypercortisolism) which is characterized, in addition to the signs listed above, by plethoric facies, supraclavicular fatpads, and the so-called "buffalo hump." Doughy, thickened skin (B) and thickened tongue (E) may be found in hypothyroidism, exophthalmos (C) in Graves disease, and lid lag (D) in hyperthyroidism from any cause. Fitzgerald PA, Endocrine Disorders, in Current Medical Diagnosis and Treatment, 52 nd ed. 2013.
Q 86.7: As a diver descends for a deep water dive, at about 10 feet of depth he begins to feel nausea, severe ear pain, and develops vertigo and vomiting. What is the most likely cause of his symptoms? A Decompression sickness B Decreasing pressure in the middle ear C Benign paroxysmal positional vertigo D Increasing pressure in the middle ear E Equalization of pressure between the middle ear and eustachian tube
The Correct Answer is: B Boyle's law states that as a diver descends, the increasing external pressure causes an equal decrease in pressure in the middle ear, which must be equalized during the descent. If the middle ear pressure is not equalized, the tympanic membrane becomes severely retracted, due to the negative middle ear pressure. This can result in hemotympanum, hemorrhage, or tympanic membrane perforation. Ascent causes increased pressure in the middle ear as the external pressure is decreased. Equalization techniques must also be used to prevent a tympanic membrane perforation. Decompression sickness occurs on ascent, when nitrogen gas bubbles are forced into the middle ear, and vascular and lymphatic spaces. (Lalwani A.K., 2008, Chapter 57)
Q 18.9: A 76 year-old woman with steroid dependent chronic obstructive pulmonary disease is hospitalized with fever, chills, and a productive cough. The sputum gram stain shows many WBCs and small, pleomorphic gram-negative rods. Which of the following is the most likely causative agent? A Chlamydia pneumoniae B Haemophilus influenzae C Mycoplasma pneumoniae D Staph aureus E Strep pneumoniae
The Correct Answer is: B Haemophilus influenzae (B) is a gram-negative pleomorphic coccobacillus. Strep pneumonia (E) and Staph aureus (D) are gram positive organisms. Mycoplasma pneumonia (C) and Chlamydia pneumoniae (A) aren't visible on gram stain.
Q 98.43: A 28-year-old woman presents with nervousness and palpitations associated with heat intolerance. On examination, there is no evidence of thyromegaly, but there is a palpable nodule that is "hot" on a thyroid scan. The TSH was low and T3 and T4 were both elevated. Which of the following is the recommended treatment for this patient? A Propylthiouracil (PTU) B Thyroid lobectomy C Total thyroidectomy D Radioiodine ablation
The Correct Answer is: B In Graves' disease, the thyroid is diffusely enlarged in contrast to a toxic adenoma in which the thyroid is normal sized but with a palpable nodule. Surgery is the treatment of choice for a toxic adenoma. Surgical treatment of a toxic adenoma is a thyroid lobectomy and isthmusectomy. A subtotal or total thyroidectomy is indicated for toxic multinodular goiters or Plummer disease. Thionamides and radioiodine ablation are not effective therapies for toxic adenomas.
Q 22.1: What is the most common joint dislocation in children? A Ankle B Elbow C Finger D Knee E Shoulder
The Correct Answer is: B In children the elbow is the most commonly dislocated joint and it is the third most common joint dislocation for adults. The shoulder and finger are dislocated more frequently than the elbow in adults. Knee dislocations at the femur-tibia joint are rare (patellofemoral dislocations are more common) and ankle dislocations are also relatively rare.
Q 50.3: A 4-year-old boy presents to the outpatient clinic for a well child visit. Developmental assessment is normal. Upon visual acuity testing his vision is noted to be 20/30 right eye, 20/40 left eye, and 20/30 with both eyes. Which of the following is the most appropriate management of this child's vision? A Re-assess his vision in 3 months B Re-assess his vision in one year C Re-assess his vision prior to enrolling in kindergarten D Refer to an optometrist E Refer to a pediatric ophthalmologist
The Correct Answer is: B Pediatric ophthalmology referral criteria for 3- to 5-year-old children include visual acuity of less than 20/40 in either eye or greater than a two line difference between eyes, so referral to an optometrist (D) or pediatric ophthalmologist (E), which is preferred by the American Academy of Pediatrics, is not warranted and his vision can be re-assessed at his next well child visit in one year (B).
Q 12.3: A 48-year-old man presents to the clinic for a routine employment physical. The patient is asymptomatic. A pre-employment CBC reveals hemoglobin of 13.2g/dl, hematocrit of 39.5%, and MCV of 60.6 fL. Subsequent iron studies, hemoglobin electrophoresis, and sickle cell screening are "normal." After the labs are reviewed, the patient states "they are always normal." What is the most likely diagnosis? A Anemia of chronic disease B Alpha thalassemia minor C Beta thalassemia major D Iron deficiency anemia E Sickle cell disease
The Correct Answer is: B The patient has a mild anemia with pronounced microcytosis consistent with alpha thalassemia minor. Alpha thalassemia minor hemoglobin electrophoresis reveals normal results. Patients with anemia of chronic disease (A) and iron deficiency anemia (D) have abnormal iron studies; while patients with beta thalassemia major (C) and sickle cell disease (E) have abnormal hemoglobin electrophoresis results.
Q 120.18: Which of the following lists the common adverse effects caused by nitroglycerin when administered sublingually at high doses? A constipation, blurred vision, tinnitus B dyspepsia, abdominal distention, vomiting C elevated pulse, facial flushing, headache D photophobia, excessive salivation, excessive tearing E wheezing, cough, heartburn
The Correct Answer is: C Sublingual nitroglycerin produces venodilation and vasodilation, which causes secondary responses of flushing and headache. The elevated pulse or tachycardia is reflexive in nature, and the heart tries to compensate for the drop in blood pressure by raising its rate. (Talbert, 2008, pp. 237-239)
Q 98.70: What types of connective tissue are injured in a strain? A Bones and muscles B Fascia and joint capsules C Ligaments and joint capsules D Muscles and tendons E Tendons and bones
The Correct Answer is: D A strain involves injury to the muscles and tendons that are responsible for active movement of various body parts. Fascia is a part of the muscle-tendon unit, so injury to fascia would be considered a strain as well. Injury to ligaments and joint capsules would be considered a sprain and damage to bone would be classified as a fracture.
Q 97.12: A 65-year-old female presents to clinic for her annual pap smear. She is in good health but has mild hyperlipidemia, which is controlled with diet. She had a hysterectomy more than 10 years ago for dysfunctional uterine bleeding. How often should she have a pap smear? A Annually B Every 2 years C Every 3 years D Symptomatically
The Correct Answer is: D According to the new 2010 ACOG guidelines, women who have no high-grade lesions or cervical cancer history, and are over 65, may discontinue cervical cancer screening due to the decrease risk and slow progression if disease does occur.
Q 117.18: Following a gunshot wound to the lower abdomen, a 29-year-old man is hospitalized and treated with clindamycin for a potential anaerobic infection. After 3 days of clindamycin therapy, while recuperating in the hospital, he develops severe diarrhea, dehydration, and lower abdominal cramping. A stool culture is ordered and later discovered to contain Clostridium difficile. After discontinuing the clindamycin, which of the following would be the most appropriate treatment? A cefaclor B doxycycline C amoxicillin D metronidazole E cephalexin
The Correct Answer is: D Clostridium difficile is a gram-positive, anaerobic, spore-forming bacillus that is responsible for the development of antibiotic-associated diarrhea and colitis. C difficile colitis results from a disturbance of the normal bacterial flora of the colon, colonization with C difficile, and release of toxins that cause mucosal inflammation and damage. Antibiotic therapy is the key factor that alters the colonic flora. Specific therapy aimed at eradicating C difficile is indicated if symptoms are persistent or severe. The drug of choice is metronidazole, 500 mg orally three times daily or 250 mg orally four times daily. Oral metronidazole and vancomycin are equally effective in treating diarrhea caused by C difficile. Despite the isolation of metronidazole-resistant strains of C difficile, metronidazole is the drug of first choice because of its lower cost and the fact that it can promote vancomycin-resistant nosocomial infections. (Martin and Jung, 2008, p. 1863; McQuaid, 2008, pp. 543-544) Martin S , Jung R. Gastrointestinal infections and enterotoxigenic poisonings. In: DiPiro JT , Talbert RL , Yee GC, et al., eds. Pharmacotherapy: A Pathophysiologic Approach. 7th ed. New York: McGraw-Hill; 2008. McQuaid KR. Gastrointestinal disorders. In: Tierney LM Jr , McPhee SJ , Papadakis MA, eds. Current Medical Diagnosis & Treatment. 47th ed. New York: McGraw-Hill; 2008.
Q 2.5: A toddler became infected with Giardia lamblia at day care and his two elder siblings and both parents have also developed foul-smelling diarrheal stools. Of the following treatments, which is most appropriate for his mother, who is 24 weeks pregnant? A Albendazole B Furazolidone C Metronidazole D Paromomycin E Tinidazole
The Correct Answer is: D First-line treatments for giardiasis are albendazole (A), metronidazole (C), and tinidazole (E). Furazolidone (B) is also effective, but causes gastrointestinal side effects. Paramomycin has lower efficacy, but is considered safe in pregnancy, unlike the others.
Q 47.12: What is the appropriate first line treatment of lyme disease in a non-pregnant adult female with erythema migrans and no other symptoms of lyme disease and no known drug allergies? A Erythromycin 150 mg qid B Ceftriaxone 1g IM C Amoxicillin 500 mg q 8 hours D Doxycycline 100 mg bid
The Correct Answer is: D In patients over the age of 9 exhibiting skin or joint manifestations of lyme disease, the first line treatment is Doxycycline 100 mg bid. Patients less than 9 or those who are allergic to Doxycycline should be treated with amoxicillin. Erythromycin is fourth line treatment for all age groups. Ceftriaxone is first line for patients with nervous system involvement.
Q 97.63: A 37-year-old female presents to the clinic for her obstetrical appointment. She is a G5P4 African American female. On physical exam her uterus is larger than expected for dates by 5 cm. What is her risk of having twins increased by? A Her increasing age B Her increasing parity C Her race D Her use of clomifine E Her late age of menarche
The Correct Answer is: D Incidence of multiples is not related to age, parity, or menarche; the increase seen in the last few decades is solely related to the increased use of fertility drugs.
Q 4.3: A 68-year-old female with a past medical history of diabetes, dyslipidemia, hypertension, and gout presents with a gradual progression of fatigue and pallor over the last few months. Initial CBC results show a hemoglobin of 10.4 mg/dL, hematocrit of 32%, an MCV of 112 fL, and a reticulocyte count of 0.1%. Which of the following medications should be considered as a potential cause of her anemia? A Allopurinol B Atorvastatin C Lisinopril D Metformin E Nicotinic acid
The Correct Answer is: D Metformin is associated with vitamin B 12 deficiency and may rarely be a medication cause for macrocytic anemia. Nicotinic acid (E) administration may help reduce the risk of vitamin B 12 deficiency secondary to nicotinic acid deficiency. Allopurinol (A), atorvastatin (B), and lisinopril (C) are not strongly associated with vitamin B 12 or folate deficiency.
Q 110.8: A surgical physician assistant suffers a deep puncture wound during surgery on an HIV-positive patient. The patient, who is on a multidrug regimen, has a viral load of 120,000 copies. Which of the following drugs is contraindicated for the physician assistant because of its potential for hepatotoxicity in the setting of HIV prophylaxis? A abacavir B indinavir C lamivudine D nevirapine E zidovudine
The Correct Answer is: D Nevirapine should be avoided for HIV prophylaxis as reports have linked it to hepatotoxicity in the prophylactic setting. Abacavir may cause rash and fever, indinavir kidney stones, lamivudine rash and peripheral neuropathy, and zidovudine anemia, neutropenia, nausea, malaise, headache, insomnia, and myopathy. (Katz and Zolopa, 2009, p. 1192) Katz MH , Zolopa AR. HIV infection. In: McPhee SJ , Papadakis MA, eds. Current Medical Diagnosis and Treatment. 48th ed. New York, NY: McGraw-Hill; 2009.
Q 64.8: At what stage of hospitalization can active tuberculosis patients be placed in a non-negative pressure room? A When patient is afebrile B When patient is on a minimum of three days of antibiotics C When there is a clear chest x-ray D When there is a clear sputum gram stain E When patient is absent of leukocytosis
The Correct Answer is: D Once the patient has had a documented clear sputum gram stain that shows no evidence of the tuberculin bacteria, then the patient can be cleared. Chest x-rays can lag on clearing and would not be an effective measure of clearance. Leukocytosis does not rule in or rule out the infection and is not a reliable indicator.
Q 25.7: The rotator cuff is comprised of which four muscles? A Scalenes, infraspinatus, teres minor, subscapularis B Supraspinatus, infraspinatus, teres major, subscapularis C Supraspinatus, infraspinatus, teres minor, soleus D Supraspinatus, infraspinatus, teres minor, subscapularis E Supraspinatus, intercostals, teres minor, subscapularis
The Correct Answer is: D Only option D lists the correct muscles of the rotator cuff located in the shoulder area that contribute to arm elevation, internal rotation and external rotation. Of the teres muscle group, it is the teres minor, not teres major that is part of the rotator cuff. The scalene muscles are in the neck and cause rib elevation and neck movement. The soleus muscles cause plantar flexion of the feet and are located in the calf area of the posterior lower leg.
Q 11.5: A 5-year-old boy presents to urgent care complaining of painful lesions in his mouth that have made eating difficult the past 2 days. The mother confirms he has been unable to eat for 48 hours, but has been able to sip water. On physical exam he has a temperature of 102.6º F; numerous small vesicles and ulcers on the buccal mucosa and tongue; inflamed gingiva; and tender anterior cervical adenopathy. Which of the following is the most likely causative organism? A Coronavirus B Coxsackie virus A16 C Group A beta-hemolytic strep D Herpes simplex 1 E Rhinovirus
The Correct Answer is: D The classic presentation of initial infection with herpes simplex virus 1 (D) includes multiple small, painful vesicles or ulcers on the mucousa with gingival involvement, fever, and adenopathy. Coxsakie virus A16 (B) causes hand, foot, and mouth disease. Coronavirus (A) and rhinovirus (E) cause viral pharyngitis.
Q 16.6: A 15-year-old female comes into your office with multiple symptoms of an eating disorder. What is the single feature that would lead you to diagnose anorexia rather than bulimia or binge eating disorder? A Binge eating behaviors B Compensatory measures such as purging C Preoccupation with body weight D Refusal to maintain normal body weight E Strict focus on dietary intake
The Correct Answer is: D The diagnosis of anorexia requires that the patient refuse to maintain a normal body weight (D). All the other behaviors mentioned (A, B, C, E) may be shared between anorexia and bulimia. People with binge eating disorder (not fully recognized in the DSM IV-TR) display binge eating without compensatory measures, resulting in obesity.
Q 54.3: The best initial diagnostic study for a suspected perforated peptic ulcer is which of the following? A abdominal ultrasound B upper GI barium swallow C esophagogastroduodenoscopy (EGD) D upright/decubitus abdominal plain film E colonoscopy
The Correct Answer is: D The presence of free intraperitoneal air on an upright or decubitus film in the majority of patients with peptic ulcer perforation. This finding along with a classic history of sudden onset of severe abdominal pain and a rigid, quiet abdomen should establish the diagnosis in most cases without the need for further studies. Barium studies are contraindicated in patients with a possible perforation.
Q 98.61: A 58-year-old man presents with the acute onset of abdominal pain associated with fever and shaking chills. The patient is hypotensive and febrile with a temperature of 102.2°F. Although he is confused and disoriented, he complains of right upper quadrant pain during palpation of the abdomen. His sclerae are icteric and the skin is jaundiced. Which of the following is the most likely diagnosis? A Acute cholecystitis B Choledocholithiasis C Acute pancreatitis D Ascending cholangitis
The Correct Answer is: D The presenting symptoms associated with ascending cholangitis include fever, chills, right upper quadrant pain, and jaundice (Charcot's triad); the symptoms are secondary to an infected obstruction of the common bile duct. With spread of the infection, the patient may also develop hypotension and mental status changes; these additional symptoms in conjunction with Charcot's triad are known as Reynolds' pentad. Additional symptoms of common bile duct obstruction include light-colored stools and dark, tea-colored urine.
A 44-year-old female presents to the emergency department with a right-sided headache. She states the headache is located on the right temple region, is non-radiating, and does not cause photophobia. She is otherwise healthy and has no reported medical problems. She only takes acetaminophen for the pain, which has minimal relief. On physical exam she is alert, awake, and oriented. Her vitals are T 98.8, P 78, R 18, and BP 128/76. Her head is normocephalic, atraumatic, and pupils are equal and reactive. She has tenderness to the right temporal area of the temporal artery, and there is no noted swelling, redness, or abnormalities noted. There is no tenderness to the cervical muscles. She exhibits a non-focal neurological exam. Based on the description, what is the most likely diagnosis of this patient? A Migraine headache B Tension headache C Cluster headache D Temporal arteritis E Takayasu arteritis
The Correct Answer is: D This patient is exhibiting a case of temporal arteritis (D). She has the signs and symptoms that are classic in nature and do not fit into the realm of the other headaches (A, B, and C), all of which usually have different characteristics on history and physical exam. Takayasu arteritis (E) will typically not present in the temporal artery.
Q 39.7: What is the most common clinical cardiac abnormality that is associated with acute rheumatic heart disease? A Hypotension B Arrhythmia C Ischemia D Ventricular aneurysm E Carditis
The Correct Answer is: E Carditis is the most common finding in rheumatic heart disease. This can present with the sequelae of pericarditis, cardiomegaly, heart failure (either right or left sided), and either a mitral or aortic murmur.
Q 72.7: A 50-year-old woman with a history of hypertension complains of chest tightness and dyspnea while walking up one flight of stairs. She recently experienced an episode of near-syncope while walking her dog. She denies a history of rheumatic fever. On auscultation, a crescendo-decrescendo systolic ejection murmur is heard at the upper right sternal border, radiating to the carotids bilaterally. Troponin levels are negative at 0, 3, and 6 hours. Her EKG demonstrates evidence of left ventricular hypertrophy. Given the patient's physical exam findings and recent symptoms, which of the following is the most appropriate next diagnostic study? A Chest X-ray B Transesophageal echocardiogram C Holter monitor D Treadmill exercise stress test E Transthoracic echocardiogram
The Correct Answer is: E Choice E, transthoracic echocardiogram, is a simple, sensitive, and non-invasive diagnostic tool which can evaluate for the presence of valvulopathy in a patient in this age group, who is likely demonstrating severe aortic stenosis secondary to a congenital bicuspid valve. Patients with a congenital bicuspid aortic valve typically develop symptoms once the valve leaflets have become calcified and thickened, secondary to the undue stress over many years on a structurally abnormal aortic valve. Choice A might be able to give evidence of cardiomegaly or calcification of heart valves, but would not be sensitive enough to detect the degree of valvulopathy, if present. Choice B, transesophageal echocardiogram, would give information regarding valvulopathy, but is a more invasive test; therefore, choice E is more appropriate. Choice C is a useful diagnostic tool for evaluation of patients complaining of palpitations, but incorrect for this patient, who has no symptoms of palpitations. Choice D, although a useful diagnostic tool for the evaluation of exercise tolerance and in patients complaining of chest pain, does not allow direct visualization of the heart valves to evaluate the degree of aortic stenosis; as the patient is likely demonstrating severe aortic stenosis, cardiac catheterization to evaluate for coronary artery disease prior to surgery will need to be performed. (McPhee et al., 2011, Chapter 10)
Q 29.10: A 44-year-old female presents to the emergency department with a right sided headache. She states the headache is located on the right temple region, is non-radiating, and does not cause photophobia. She is otherwise healthy and has no reported medical problems and has only taken acetaminophen for the pain, which has minimal relief. On physical exam she is alert, awake, and oriented. Her vitals are T 98.8, P 78, R 18, and BP 128/76. Her head is normocephalic, atraumatic, and pupils are equal and reactive. She has tenderness to the right temporal area of the temporal artery, and there is no noted swelling, redness, or abnormalities noted. There is no tenderness to the cervical muscles. She exhibits a non-focal neurological exam. A biopsy of the temporal artery reveals inflammatory cells of the artery wall. Based on the history and biopsy results, what is the best initial treatment for this patient? A Cyclosporine B Azathioprine C Methotrexate D Ibuprofen E Prednisone
The Correct Answer is: E This patient is exhibiting a case of temporal arteritis. She has the signs and symptoms that are classic in nature and do not fit into the realm of the other headaches, all of which usually have different characteristics on history and physical exam. In this instance, the use of oral steroids will be the best choice in management of the condition. Anti-inflammatories (C, D) and immunosuppressant agents (A, B) will not be as effective as oral prednisone (E), and the long-term outcomes are improved with steroids.
Q 89.5: A 24-year-old female patient develops her second deep vein thrombosis (DVT) in two years. After a hypercoaguability work up, you discover that she has a factor V Leiden abnormality. How long should she remain on oral anticoagulation therapy? A She does not need anticoagulation therapy. B You may stop her anticoagulation therapy when her current DVT has resolved. C You may stop her anticoagulation therapy in six months. D You may stop her anticoagulation therapy in one year. E She needs lifetime anticoagulation therapy.
The Correct Answer is: E When patients have a deep vein thrombosis, it usually warrants a hypercoaguability work up to rule out genetic causes. While complicated and sometimes requiring hematology consultation, patients with hypercoaguable genetic disorders such as a factor V Leiden abnormality generally require lifetime anticoagulation. (Lichtman, et al., 2010, Chapter 131)
Q 49.4: A young child and his parents have been adhering to the treatment plan for type 1 diabetes, as discussed with their health care provider. It includes a change in diet, as well as blood glucose and ketone monitoring. They noticed that the amount of insulin needed decreased after the first two weeks. What is this time period commonly called? A Postprandial control phase B Glucose tolerance time C Pre-diabetic period D Mature onset diabetes of youth E Honeymoon phase
The correct choice is E, honeymoon phase. During this time, some pancreatic beta cell function may recover, although within eight weeks to two years most patients will show absent or negligible pancreatic beta cell function. Choice A, postprandial control phase, choice B, glucose tolerance time, and choice C, pre-diabetic period, are not true time periods. Choice D, mature onset diabetes of youth, is a subgroup of autosomal dominant inherited disorders, characterized by diabetes in non-obese older children that are not ketosis prone and generally do not need insulin therapy to control their disease. This type of diabetes accounts for up to 5% of diabetes in North America and Europe.
Q 25.9: An 11-year-old boy is being seen in the clinic for well-child care. His father inquires whether his son is starting to show physical signs of puberty. Which of the following is the first sign of puberty in males? A change of voice B scrotal and testicular enlargement C gynecomastia D pubic hair development
The Correct Answer is: B s crotal and testicular enlargement is the first sign of puberty in boys. This typically occurs at the average age of 10 to 12 years.
Q 65.10: A patient has a 2-day history of an itchy red left eye and marked tearing. There is no history of injury. On physical exam the conjunctiva appear markedly erythemic. What physical finding would help most in differentiating this as a viral conjunctivitis? A Copious discharge B Subconjunctival hemorrhage C Pre-auricular lymph node D Vesicular rash around the eye E Punctate keratitis
The Correct Answer is: C A palpable pre-auricular lymph node is most often seen with viral conjunctivitis and rarely seen in bacterial conjunctivitis.
Q 97.114: You are evaluating a 53-year-old post-menopausal patient during her routine annual examination in a primary care office. You notice that the patient has not been properly assessed for risk for osteoporosis, and does have some complaints that are of concern for osteoporosis. Based on this history, and the standard of care, what test is used to confirm osteoporosis in this patient? A Bone scan B CT scan C DEXA scan (dual energy x-ray absorptionmetry) D MRI scan E X-ray
The Correct Answer is: C A DEXA scan is the best test to determine bone density and is commonly used as a screening tool for those at risk of developing osteoporosis. This test is fast, reproducible and exposes patients to a relatively low dose of radiation. The results are reported as Z and T scores. Z scores indicate how the patients bone density compares to peers and T scores compare the patient to young, health individuals. Z and T scores are presented in the form of standard deviations below the comparison group. If the T score is 0 to greater than -1 the test is interpreted as normal. T scores of -1 to -2.5 are indicative of someone with osteopenia and a T score lower than -2.5 is consistent with osteoporosis. None of the other imaging options presented are ideal for determining bone density, but they each play important roles in other aspects of patient care. Bone scans are best suited for imaging studies designed to look for occult fractures, tumors, inflammatory or infectious processes within the bones, or metabolic bone diseases. CT is useful for a detailed examination of bone when looking for fractures or lesions and intra-articular pathology. MRI is helpful in the evaluation of spinal column pathology and various soft tissue injuries involving the muscles, tendons, ligaments, and cartilage. MRI is also useful in the evaluation of stress and occult fractures, osteomyelitis and early osteo necrosis. Radiographs (x-ray) are useful for most initial evaluation of musculoskeletal pathology, but are somewhat limited in their usefulness in visualizing some conditions. Low bone density can be perceived on x-ray, but only after the bone loss has been rather extensive. It is not a great early screening tool for osteoporosis.
Q 48.6: A 2-year-old child presents to the emergency department via ambulance due to a seizure lasting approximately 2 minutes with jerking and somnolence. En route in the ambulance her vital signs are: temperature 39°C rectal; pulse 120/min; respirations 32/min; blood pressure 110/64 mm Hg. Upon further questioning, her mother claimed she had a runny nose yesterday. On physical examination, she is sleepy but arousable with negative Kernig and Brudzinski signs. Which of the following seizures is the MOST likely diagnosis? A absence seizure B complex partial seizure C febrile seizure D simple partial seizure
The Correct Answer is: C A febrile seizure is a brief (less than 15 minutes), generalized, symmetric, tonic-clonic seizure associated with a febrile illness (temperature greater than 38.8°C) without any central nervous system infection or neurologic cause. An absence (petit mal) seizure is a brief (2 to 25 seconds) loss of consciousness that can occur multiple times per day. There is no loss of tone, and frequently the only observable behaviors are staring or minor movements such as lip smacking and semipurposeful movements of the hands. There is no postictal period. Complex partial seizures (psychomotor) have varied symptoms including alterations in consciousness, unresponsiveness, and repetitive complex motor activities that are purposeless. Often, at the beginning of the attack, there is a psychoillusory phenomenon such as hallucinations, visual distortions, visceral sensations, or feelings of intense emotions. Simple partial seizures include focal motor, adversive, and somatosensory seizures. Manifestations of these seizures are varied including hallucinatory, psychoillusory, or complex emotional phenomena. Children will interact normally with their environment, with the exception of those limitations imposed by the seizure. Following the seizure (minutes to hours), there may be transient paralysis of the affected body part.
Q 64.3: A 37-year-old female, who is two weeks post caesarean section, develops acute dyspnea and chest pain. A chest CT reveals a left segmental perfusion defect. Which of the following is the next step in the management of this patient? A Aspirin B Embolectomy C Heparin D Pulmonary angiography E Streptokinase
The Correct Answer is: C Heparin is indicated as initial therapy for acute pulmonary thromboembolism, followed by oral anticoagulation with warfarin. Heparin promotes the effect of antithrombin, which inhibits factors Xa, IXa, Xia, and XIIa, and has been shown to decrease mortality and recurrent pulmonary embolism. Streptokinase, a thrombolytic agent, is recommended for hemodynamically unstable patients being treated with heparin, but with continued risk of death. Embolectomy, although associated with increased mortality, is another alternative for these patients. Pulmonary angiography, the gold standard for pulmonary embolus diagnosis, is being replaced with helical contrasted CT, due to angiography's invasiveness, time involvement, and cost. Aspirin, an antithrombotic agent, inhibits platelet aggregation and is effective for preventing platelet thrombosis. It also has a role in thrombosis prevention. However, anticoagulation with heparin remains the mainstay of therapy for pulmonary embolus.
Q 118.1: Which of the following is a potential adverse effect associated with unfractionated heparin (UFH)? A hyperglycemia B hypothyroidism C thrombocytopenia D excessive cough E muscle cramps
The Correct Answer is: C Heparin-induced thrombocytopenia (HIT) is a potentially serious complication of unfractionated heparin therapy, usually occurring within 4 to 10 days after heparin treatment has started. Fortunately, current estimates show that it is infrequent, occurring in approximately 0.3% to 3.0% of patients receiving UFH for more than 4 days. HIT should immediately be suspected in a patient who develops deep vein thrombosis or pulmonary embolism while receiving UFH. (Whitby and Johns, 2008, pp. 1710-1712) Whitby DH , Johns TE. Drug-induced hematologic disorders. In: DiPiro JT , Talbert RL , Yee GC, et al., eds. Pharmacotherapy: A Pathophysiologic Approach. 7th ed. New York: McGraw-Hill; 2008.
Q 57.2: Jane, a 21-year-old female, was seen in the office 10 days ago and was diagnosed with perennial allergic rhinitis and sent home with instructions for increased fluids, decongestants, and nasal steroids. She returns today with worsened symptoms of malaise, low-grade fever, nasal discharge, cough that is worse at night, mouth breathing, early morning unilateral pain over sinuses, and congestion. Physical examination reveals thick purulent nasal discharge, postnasal discharge visible in the posterior pharynx, periorbital swelling, and tenderness of sinuses upon palpation. She is 36-weeks pregnant and allergic to penicillin. Of the following, what is the most appropriate antibiotic? A amoxicillin B trimethoprim-sulfamethoxazole C clindamycin D levofloxacin
The Correct Answer is: C Most patients with a diagnosis of acute rhinosinusitis based on clinical grounds improve without antibiotic therapy. The preferred initial approach in patients with mild to moderate symptoms of short duration is therapy aimed at facilitating sinus drainage, such as oral and topical decongestants, nasal saline lavage, and—in patients with a history of chronic sinusitis or allergies—nasal glucocorticoids. Adult patients who do not improve after seven days, children who do not improve after 10 to 14 days, and patients with more severe symptoms (regardless of duration) should be treated with antibiotics. Empirical therapy should consist of the narrowest-spectrum agent active against the most common bacterial pathogens, including S. pneumoniae and H. influenzae—e.g., amoxicillin. But amoxicillin is contraindicated in patients with urticarial reactions to penicillins, and quinolones are similarly contraindicated in pregnancy. trimethoprim-sulfamethoxazole is contraindicated in the third trimester of pregnancy. The best choice is clindamycin.
Q 121.3: An 18-year-old woman is transferred to your emergency department from a local college infirmary. She presented yesterday with a complaint of headache but became confused and is now febrile. You notice a petechial rash on physical examination and her cerebrospinal fluid comes back with increased WBCs, increased protein, and decreased glucose. What is the most likely organism responsible for her meningitis? A Haemophilus influenzae B cytomegalovirus C Neisseria meningitidis D Mycobacterium tuberculosis E coxsackievirus B
The Correct Answer is: C Neisseria meningitidis and Streptococcus pneumoniae are the most common etiologic agents for bacterial meningitis in this patient's age group. So much so that many colleges and universities require a vaccine for students who live in dormitories. Her fever and the cerebrospinal fluid values are consistent with a bacterial and not a viral infectious source for the meningeal irritation. (Aminoff et al., 2005, pp. 20-30) Aminoff MJ , Greenberg DA , Simon RP. Clinical Neurology. 6th ed. New York, NY: McGraw-Hill; 2005.
A 62-year-old man with a history of hypertension, diabetes mellitus type 2, hyperlipidemia, and chronic tobacco use presents to the office with complaints of a retrosternal chest pressure radiating down his left arm, associated with diaphoresis, nausea, and dyspnea, for the last 45 minutes after mowing his lawn. The patient's vital signs are stable, and on physical examination a new systolic murmur is appreciated. According to the most recent American College of Cardiology/American Heart Association Guidelines for the Management of Patients with Unstable Angina/Non-ST-Elevation MI recommendations, an EKG should be performed on patients with a clinical suspicion for acute coronary syndrome within how many minutes of their arrival to the emergency department? A 1 minute B 5 minutes C 10 minutes D 15 minutes E 20 minutes.
*Choice C is correct*, as the 2007 American College of Cardiology/American Heart Association Guidelines for Management of Patients with Unstable Angina/Non-ST-Elevation MI recommend that an ECG *"be performed and shown to an experienced emergency physician as soon as possible after ED arrival, with a goal of within 10 minutes of ED arrival for all patients with chest discomfort or other symptoms suggestive of ACS."* Although choices A and B would be optimal, the question asks for the goal time for which busy emergency departments should aim for in obtaining an EKG in at-risk patients. Choices D and E are less desirable, as times greater than 10 minutes increase both morbidity and mortality rates.
A 76-year-old man with a history of HTN and diabetes mellitus, type 2, presents to the emergency department with complaints of palpitations, tachypnea, and chest pain. He denies history of CAD, stroke, TIA, or congestive heart failure. He is afebrile, with vital signs as follows: BP 145/98, HR 138, and RR 22. His EKG is shown (Figure 1). Troponins are negative X 1. His echocardiogram demonstrates normal LV systolic function and normal valvular function. He states that his symptoms began 3 days ago, and had gotten worse in the last 2 hours. What would be the most appropriate next step in management? A Morphine, oxygen, nitroglycerin sublingually, aspirin 81 mg PO X 4 B Reassurance and anxiolytics C Diltiazem 20 mg bolus IVP over 2 minutes, then diltiazem 10 mg/h IV infusion D Aspirin 81 mg two tablets PO E Dopamine 5 mcg/kg/min
*Choice C is correct; as the patient is demonstrating atrial fibrillation with RVR, the appropriate therapy now that acute myocardial infarction has been ruled out, and as the patient is beyond the therapeutic window for immediate direct current electrocardioversion, is heart rate control.* Choice A is inappropriate, as this is standard therapy for acute MI. Choice B can be used in patients suffering palpitations associated with panic attacks. Choice D is insufficient anticoagulation therapy. Choice E is inappropriate as the patient is not in need of pressor support.
A 66-year-old man with a history of HTN and diabetes mellitus, type 2, presents to the emergency department with complaints of palpitations for over 2 weeks, tachypnea, and chest pain. He denies history of CAD, stroke, TIA, or congestive heart failure. He is afebrile, with vital signs as follows: BP 145/98, HR 138, and RR 22. His EKG is shown (Figure 1). Troponins are negative X 3. Which of the following choices is the most appropriate next diagnostic study for this patient? A Transthoracic echocardiogram B Cardiac catheterization C Nuclear stress test D Holter monitor E Event recorder
*The Correct Answer is: A Choice A, transthoracic echocardiogram, is correct, as it can demonstrate the presence of valvular heart disease. The presence of valvular heart disease can change the recommendations for embolism prophylaxis.* Choice B, cardiac catheterization, is useful in patients suspected to have unstable angina, or who have sustained a myocardial infarction. Choice C, nuclear stress test, is useful in patients suspected to have angina pectoris, and may be a useful diagnostic study in this patient with cardiac risk factors (once the issue of atrial fibrillation has been treated). Choices D and E would be useful tests if the EKG had not established a diagnosis for this patient, with the Holter monitor indicated in patients experiencing symptoms on a daily basis, and the event recorder indicated in patients demonstrating more sporadic symptoms.
A 6-year-old female presents to the emergency department with left wrist pain after falling off the monkey bars at the school playground. Imaging of the left upper extremity shows the following fracture pattern: Which type of Salter-Harris Classification is observed? A Type I B Type II C Type III D Type IV E Type V
*The Correct Answer is: A A Salter-Harris Type I (A) involves the entire epiphysis.* Type II (B) is the entire epiphysis along with a portion of the metaphysis, Type III (C) involves a portion of the epiphysis only, Type IV (D) involves a portion of the epiphysis along with a portion of the metaphysis, and Type V (E) is a compression injury of the epiphyseal plate (nothing is "broken off").
Two days following an uneventful 4-vessel CABG, a 57-year-old man develops a sudden onset of lightheadedness and palpitations. His vital signs are stable, and physical examination demonstrates no abnormalities. Given the results of his EKG, as shown (Figure 2), which of the following is the most appropriate next step in management? A Direct-current cardioversion B Nitroglycerin patch C Digoxin 0.125 mg PO daily D Neurology consult E Meclizine 25 mg PO Q6H
*The Correct Answer is: A Among the choices offered here, choice A is the most appropriate next step in management of a patient with new onset atrial flutter, as determined by EKG; it most effectively converts most patients to normal sinus rhythm.* Choice B is inappropriate, as the patient is not demonstrating angina pectoris, and the EKG does not demonstrate evidence of ischemia or infarction. Choice C is inappropriate, as it is the least effective agent for slowing the ventricular response when compared to beta blockade or calcium channel blockers, all of which act by blocking the AV node (digixon may occasionally convert atrial flutter to atrial fibrillation). Choice D is inappropriate, as the patient's symptoms of lightheadedness do not stem from neurologic changes. Choice E is inappropriate, as the patient's symptoms do not stem from vertigo.
A patient is brought to the Emergency Department by ambulance. He is a 27-year-old male who is well known to the paramedic team as a heroin addict. He is arousable and does not remain alert when aroused. Which of the following physical signs would help to confirm the diagnosis of opioid intoxication? A Bradycardia B Diaphoresis C Mydriasis D Rhinorrhea E Tachypnea
*The Correct Answer is: A Bradycardia (A), lowered respiratory rate, miosis and somnolence are the main effects of opiates.* As with most medications, withdrawal effects are the opposite of the effects of overdose. Tachycardia, tachypnea, rhinorrhea and diaphoresis (B, D, & E) can all occur in opiate withdrawal. Mydriasis (C) is common with other substances of abuse, such as cocaine and LSD.
A 48-year-old man is brought to the emergency department by his sister after suffering from loss of consciousness, followed by muscle rigidity and rhythmic contractions, and then a return to a normal state. When asked about medication use, the patient states he is currently being treated with a drug for depression but cannot remember the name. He claims that he has never had a seizure or seizure-like activity prior to this event. Approximately 6 hours after the first episode, the patient suffers a second one while still in the ED. Which of the following medications is the patient most likely taking? A bupropion B duloxetine C fluoxetine D nortriptyline E phenelzine
*The Correct Answer is: A Bupropion has been shown in some patients to cause seizures in a dose-dependent fashion, particularly in those with a history of head trauma or electrolyte abnormalities.* Tricyclic antidepressants (eg, nortriptyline), selective serotonin reuptake inhibitors (eg, fluoxetine), serotonin-norepinephrine reuptake inhibitors (eg, duloxetine), and monoamine oxidase inhibitors (eg, phenelzine) have not been associated with seizures.
During a hospitalization for acute exacerbation of COPD, troponin levels are drawn on a 62-year-old man with a history of hypertension, hyperlipidemia, and chronic tobacco use, and found to be elevated above the 99 th percentile of normal. Which of the following choices would qualify this patient for the most recent ACC/AHA consensus guideline's definition of myocardial infarction? A Ischemic symptoms B New right bundle branch-block on EKG C J wave on EKG D Pulmonary vascular congestion on CXR E Elevated WBC count
*The Correct Answer is: A Choice A is the most appropriate choice, as troponin elevation may occur in the setting of patients who do not suffer from acute coronary syndrome.* Therefore, the 2007 consensus guidelines recommended that the definition of myocardial infarction be applied to those patients who not only had troponin elevation above the 99 th percentile, but also met one of the following criteria: "ischemic symptoms, new left bundle branch block (not right bundle branch-block as in choice B), new ST and T-wave changes, new Q waves, or imaging evidence of a new loss of viable myocardium or new regional wall-motion abnormality." Choice C, J wave, is characteristic of patients with hypothermia. Choice D, pulmonary vascular congestion, is frequently noted on CXR of patients with congestive heart failure. Choice E, an elevated WBC count, is indicative of an infectious process.
A 22-year-old recent immigrant from Vietnam, who is 28 weeks pregnant with her first child, presents to the emergency department with complaints of worsening dyspnea and lower extremity edema. She is unable to answer definitively whether or not she has a history of rheumatic fever. On physical examination, a possible opening snap, loud S 1 , and a very soft diastolic rumbling murmur is auscultated. When the patient is placed in the left lateral decubitus position, the murmur is accentuated, and heard best at the apex. With inspiration, the murmur does not increase in amplitude. On echocardiogram, mitral stenosis is noted. Which of the following is the most appropriate next step in management of this patient? A Beta blockade and support stockings B ACE inhibitor C Mitral valve replacement D Cardiac catheterization E Increased sodium intake to maintain fluid volume
*The Correct Answer is: A Choice A is the most appropriate next step in the management of this pregnant patient with mitral stenosis. In pregnancy, blood volume, cardiac output, and heart rate are increased. In pregnant patients with mitral stenosis, this increases the pressure across the mitral valve and can lead to pulmonary edema. The use of appropriate beta blockade is helpful for decreasing the heart rate, and may be used in conjunction with digoxin if the patient develops atrial fibrillation, a common dysrhythmia in this patient population. Support stockings are helpful in preventing venous pooling in the lower extremities, which can lead to large fluctuations in hemodynamics.* Choice B, ACE inhibitors, are contraindicated in pregnancy, secondary to the teratogenic effects. Choice C, mitral valve replacement, is indicated only if medical management is insufficient to prevent congestive heart failure, a serious consideration in this valvulopathy, which is the most likely to cause death in pregnancy. Choice D, cardiac catheterization, is used for evaluation of coronary artery disease, but is not indicated in this young patient with few risk factors for coronary artery stenosis. Choice E, increased sodium intake to maintain fluid volume, would worsen the pressure across the mitral valve, and thus is incorrect, as avoidance of fluid overload is one of the keys to management of a pregnant patient with mitral stenosis, especially in the latter stages of pregnancy. Patients who are well-controlled throughout pregnancy, with medical management, are still at risk during labor and delivery, when large fluctuations in hemodynamics will occur.
A 24-year-old HIV-positive man comes to the emergency department complaining of severe left-sided chest discomfort, which radiates through to the left trapezius region. On coming into the room, you note that he is sitting up and hunched forward. On physical examination, the patient's blood pressure is 135/78, with a pulse of 85 bpm, and a pericardial friction rub is noted. Laboratory findings demonstrate elevated serum creatine kinase levels and normal serial troponin levels. His EKG demonstrates peaked T waves. His CXR demonstrates a "water bottle" cardiac silhouette. Which of the following are serious consequences of acute pericarditis, which require careful monitoring? A Pericardial effusion B Aortic dissection C Myxedema D Chylopericardium E Acute myocardial infarction
*The Correct Answer is: A Choice A, pericardial effusion, is a serious consequence of acute pericarditis, which requires careful monitoring to ensure that progression of the pericardial effusion does not lead to cardiac tamponade, which can be fatal if not treated promptly.* Choice B is unlikely in a patient with acute pericarditis. Choices C, D, and E are noninfectious causes, not consequences of pericarditis.
A 55-year-old woman with a history of hypertension and 2 vessel CABG presents to the emergency department with increasing dyspnea while walking up one flight of stairs. She denies chest pain and discomfort, but states that for the last 2 weeks she has also noticed palpitations. On physical examination, her vital signs are stable, with a normal physical exam. On EKG, she demonstrates atrial flutter with 2:1 AV block. Which of the following is the most appropriate next diagnostic study for this patient? A Transthoracic echocardiogram B Cardiac catheterization C Nuclear stress test D Holter monitor E Event recorder
*The Correct Answer is: A Choice A, transthoracic echocardiogram, is the most appropriate next diagnostic study in this patient with atrial flutter, as it can demonstrate the presence of valvular heart disease.* The presence of valvular heart disease can change the recommendations for embolism prophylaxis. Atrial flutter is treated similarly to atrial fibrillation in terms of embolism prophylaxis. Choice B, cardiac catheterization, is useful in patients suspected to have unstable angina, or who have sustained a myocardial infarction. Choice C, nuclear stress test, is useful in patients suspected to have angina pectoris, and may be a useful diagnostic study in this patient with cardiac risk factors once the issue of atrial fibrillation has been treated. Choices D and E would be useful tests if the EKG had not established a diagnosis for this patient, with the Holter monitor indicated in patients experiencing symptoms on a daily basis, and the event recorder indicated in patients demonstrating more sporadic symptoms.
A 27-year-old female presents to the emergency department with severe RUQ pain. Ultrasonography reveals gallstones. In preparation for a potential cholecystectomy, a CBC is obtained that reveals a normocytic anemia with an increased mean corpuscular hemoglobin concentration (MCHC). She is slightly jaundiced and you are able to palpate her spleen on examination. What is her underlying diagnosis? A Hereditary spherocytosis B Iron deficiency anemia C Sickle cell anemia D Thalassemia E Von Willebrand's disease
*The Correct Answer is: A Hereditary Spherocytosis (HS) is characterized by jaundice, an enlarged spleen, and often gallstones; gallstones are more frequently seen in young people, triggering the investigation into HS. An increased MCHC is a characteristic feature of HS and is almost the only condition in which this finding is seen.* Iron deficiency anemia does not have an increased MCHC. Sickle Cell has characteristic findings different than the presenting findings. Thalassemia is noted for a microcytosis and Von Willebrand's is a coagulation disorder.
A 24-year-old man with a recent history of a viral illness comes to the emergency room complaining of severe left-sided chest discomfort, which radiates through to the left trapezius region. On coming into the room, you note that he is sitting up and hunched forward. On physical examination, the patient's temperature is 39°C, blood pressure is 135/78, with a pulse of 85 bpm, and a pericardial friction rub is noted. Laboratory findings demonstrate elevated serum creatine kinase levels and normal serial troponin levels. Which of the following would be the most likely electrocardiographic findings? A Diffuse ST segment elevation B Peaked T waves C Inferior Q waves D Loss of R-wave amplitude E U waves
*The Correct Answer is: A In a patient with these signs, symptoms, and lab findings, acute pericarditis is the most likely diagnosis. In patients with acute pericarditis, EKG changes occur secondary to inflammation of the subepicardium, leading to widespread elevation of the ST segments, often with upward concavity, which returns to normal after several days, followed by T wave inversion.* No significant QRS complex changes are noted, so choice C, the development of inferior Q waves (frequently associated with an inferior myocardial infarction), is incorrect. Choice B is frequently noted with severe hyperkalemia. Loss of R-wave amplitude, choice D, is associated with myocardial infarction. Choice E, U waves, are associated with hypokalemia.
A 22-year-old male hurts his right knee while playing football on artificial turf. He states that he planted his foot and went to turn, but his leg didn't turn with his body. He instantly felt a popping sensation in the knee. A few hours later he develops an effusion. Based on the history of the injury, which knee structure was likely injured? A Anterior cruciate ligament (ACL) B Lateral collateral ligament (LCL) C Medial collateral ligament (MCL) D Posterior cruciate ligament (PCL) E Quadriceps tendon
*The Correct Answer is: A Non-impact rotational or hyperextension forces are the most common mechanisms for sustaining a tear of the ACL. One third of patients report hearing an audible popping sound as their ACL tear occurred. Because the ACL is a vascular structure, when it tears a rapid bloody effusion (hemarthrosis) usually develops which effects mobility of the joint.* Lateral collateral ligaments are the least likely to be injured as the type of force necessary to cause injury would be a varus stress which is unlikely to occur in typical circumstances. Medial collateral ligament injuries are fairly common and produced by a valgus force that stresses the ligament. This can occur in many sporting events including those in which another competitor might fall on or dive into the lateral aspect of the knee. Trauma to a knee can result in tears of both the ACL and MCL in certain situations. A tear of the quadriceps tendon usually occurs when a person falls on a knee that is partially flexed. As the quadriceps muscle contract to prevent excessive flexion, the force and momentum of the fall may overwhelm the knee extension mechanism and cause the rupture. No such mechanism occurred in our scenario. Posterior cruciate injuries occur when the tibia is driven posterior in relation to the femur as may happen when a car dashboard is driven into the tibias during a major front impact collision. A powerful hyperextension force can result in both ACL and PCL tears (usually in that order). PCL tears are much more uncommon than ACL tears and don't generally occur with basic rotational forces as described in our patient scenario.
A 22 year-old male is involved in a motor vehicle crash resulting in fracture of the left femur and left ribs 3 through 6. Approximately 24 to 36 hours after admission he becomes mildly confused and his RR increases to 40. Chest x-ray reveals diffuse pulmonary opacities. ABG shows pH 7.39, PCO2=34, PO2= 55. What is the most likely diagnosis? A ARDS B Cardiac contusion C Pleural effusion D Pneumothorax E Pulmonary thromboembolism
*The Correct Answer is: A The acute onset of respiratory distress after trauma is consistent with ARDS (A)*. The chest x-ray findings are inconsistent with cardiac contusion (B), pleural effusion (C), pneumothorax (D) and pulmonary thromboembolism (E).
A 76-year-old man, is brought to the emergency department by his niece after she found him wandering around his yard in the cold wearing only a tee shirt and jeans. When she set up his pill container about 36 hours earlier, he seemed his usual self but, in retrospect, possibly a little more confused than usual. The niece says that he has "high blood," treated with a "white fluid pill," "sugar diabetes," treated with an oral medication, and early "old timer's" dementia treated with "a memory pill." Vital signs include an oral temperature of 100.8F, pulse 100 beats per minute, respirations 24 and somewhat shallow, and blood pressure of 88/52. Initial examination reveals a slightly dehydrated, stuporous man appearing older than his stated age, who smells strongly of urine. He has no lateralizing signs. What is the most likely cause of the mental status changes? A hyperglycemic hyperosmolar state B lactic acidosis C stroke D urinary tract infection E worsening dementia
*The Correct Answer is: A The combination of confusion and dehydration in a patient with diabetes type 2 who is taking a diuretic strongly suggest hyperosmolar state.* Patients with lactic acidosis (B) have marked hyperventilation and, usually, signs and symptoms of a serious illness. The lack of lateralizing signs makes a stroke (C) less likely. Urinary tract infection (D) could certainly cause confusion and incontinence in an elderly man and should be investigated. Alzheimer dementia (E) progresses slowly; sudden decompensation is usually due to delirium.
A 32 year-old male presents to the emergency department in Acute Renal Insufficiency (AKI). Which of the following conditions would be most likely observed in intrinsic AKI? A Septic shock B Congestive heart failure C Benign prostatic hypertrophy D NSAID overdose E Chronic liver failure
*The Correct Answer is: A The most common causes of intrinsic AKI are sepsis, ischemia, and nephrotoxins, both endogenous and exogenous. Prerenal acute kidney injury can be caused from hypovolemia, decreased cardiac output, decreased circulation of blood volume (CHF, liver failure), and impaired renal autoregulation* (NSAIDs, ACE-I/ARB, cyclosporine)---(E), (D), and (B). Postrenal causes include bladder outlet obstruction including bladder stones and BPH (C).
A 32-year-old female is brought into the emergency department by her partner. His report indicates that she had been in her usual state of good health until a couple of days ago. At that time she started to complain of feeling fatigued. She now appears jaundiced and lethargic, and is complaining of chest pain. On exam, her spleen is palpable. Hemoglobin is 6 g/dl and she is Coombs positive. What is the most likely diagnosis? A Autoimmune hemolytic anemia B Glucose-6-phosphate dehydrogenase deficiency C Hereditary spherocytosis D Pyruvate kinase deficiency E Thalassemia
*The Correct Answer is: A The onset of autoimmune hemolytic anemia (AHA) is often abrupt and dramatic. Anemia can develop in days, along with jaundice and splenic enlargement. When this triad is present, the suspicion for AHA must be high. The diagnostic test for AHA is the Coombs test. If positive, it confirms the presence of the antibody on the red cells.* All other diseases listed are Coombs negative hemolytic anemias.
A 23-year-old man presents to the outpatient clinic for follow-up from a recent urgent care visit. He complains of sore throat, fever, fatigue, myalgias, and a rash that started 5 days ago, and have worsened since he was seen in the urgent care 3 days ago. The patient appears non-toxic with a temperature of 39.4 degrees Celsius. Physical exam reveals pharyngeal and tonsillar erythema without exudates, generalized lymphadenopathy, a morbilliform rash on his trunk, and no hepatosplenomegaly. A rapid strep screen and Monospot performed at the local urgent care were reportedly negative. Which of the following prevention strategies should be recommended to this patient? A Abstain from sexual activity B Avoid aspirin C Avoid contact sports and rest D Bedrest and increased fluids E Take the full course of antibiotics
*The Correct Answer is: A The patient presentation is consistent with acute retroviral syndrome. The patient is highly contagious and should be counseled on strategies to prevent transmission of HIV to others (A).* Aspirin use in viral syndromes (B) is associated with Reye's syndrome, but most often occurs in children with influenza or varicella. Avoiding contact sports (C) is appropriate patient education for a patient with infectious mononucleosis, and patients with group A strep pharyngitis should be instructed to take the full course of their antibiotics (E).
A 36-year-old man presents to the emergency department with a tight bandage around his chest to help reduce pain from a chest wall injury on his right side that occurred during mixed martial arts sparing. Physical exam reveals dullness to percussion, dry crackles and diminished breath sounds over the right lower lobe. Chest x-ray shows elevation of the right hemi-diaphragm. What is the most likely diagnosis? A Atelectasis B Bronchiectasis C Pleural Effusion D Pneumothorax E Pulmonary edema
*The Correct Answer is: A The patient's injury places him at risk of atelectasis, pneumothorax, or other traumatic injuries. The physical exam and chest x-ray findings are classic for atelectasis* (A). Pleural effusion (C) would present with fluid in the costophrenic angle on chest x-ray. Pneumothroax (D) would typically present with findings in the upper lung fields including hyperresonance to percussion. Pulmonary edema (E) would present with increased vascular markings and evidence of fluid within the alveolar space on chest x-ray.
A 23-year-old patient who has recently been on a ski trip presents with pain to the right hand after sustaining a fall. It is difficult to move, and there is pain on flexion of the digit. Based on this history what ligament would the patient most likely have injured? A 1st MCP joint ulnar collateral ligament B 2nd MCP joint ulnar collateral ligament C 3rd MCP joint ulnar collateral ligament D 4th MCP joint ulnar collateral ligament E 5th MCP joint ulnar collateral ligament
*The Correct Answer is: A The ulnar collateral ligament at the base of the thumb, or 1st MCP joint, is often injured in forced abduction, such as a fall while skiing or during other sporting activities. An injury to this ligament has traditionally been called Gamekeeper's Thumb, but the origin of this term referred to a more chronic injury sustained by English gamekeepers as a result of the way they killed rabbits using their hands. *Any of the MCP joint ulnar collateral ligaments could be injured in a fall if the mechanism of injury creates significant forces on the ligaments, but the 1st MCP joint is far more commonly injured than the others mentioned above.
An 18-year-old female comes to the emergency room stating she was raped just a few hours ago. She is not otherwise harmed, but does admit to a history of physical abuse between the ages of six and eight at the hands of a family member. Given this history, you know she is at higher than average risk for post-traumatic stress syndrome. You care for her immediate medical needs and are ready to release her. In counseling her on next steps, which of the following is most critical in order to improve her prognosis? A Begin therapy as soon as possible B Begin treatment with a serotonin uptake inhibitor C Begin treatment with a beta blocker to reduce symptoms D Begin treatment with prazosin to prevent sleep disturbance E Pursue legal procedures against the perpetrator as soon as possible
*The Correct Answer is: A Therapy to aid in working through the traumatic experience (A), instituted as soon as possible after the event, has proven to be the most helpful way to avert or minimize post-traumatic stress disorder.* This patient does not currently have any symptoms of PTSD, so pharmacologic treatment is not indicated. In a patient who is diagnosed with PTSD, SSRIs (B) may be helpful in reducing panic and improving sleep, beta-blockers (C) may reduce symptoms of anxiety, and prazosin (D) may help with sleep. Legal procedures (E) may help a patient in dealing with the event, but there are no data supporting this as a therapeutic intervention.
A 47-year-old male presents to the hospital with complaints of palpitations. He has a history of hypertension, for which he takes diltiazem. On exam he is alert, awake, and oriented. His blood pressure is 144/76, his pulse rate is 52, and his respiratory rate is 18. Lung sounds are clear, and cardiac is a regularly irregular rhythm. There is no peripheral edema noted, and a neurological exam is non-focal. The rhythm strip shown is produced. Based on these findings, what is the next step in the treatment of this patient? A Stop diltiazem B Cardiac catheterization C Permanent pacemaker insertion D Immediate cardiac surgery E Intravenous digoxin
*The Correct Answer is: A This patient has a second-degree AV block, Mobitz Type I, or Wenckebach. In this case, putting the patient on a calcium channel blocker can increase the vagal tone and slow the rate down. The best intervention for this patient is to stop the diltiazem.* If there is a suspicion of ischemia, then cardiac catheterization is warranted. There is no indication for a pacer or surgery.
A 1-year-old boy is brought to the emergency department by his parents, who state that the child refuses to walk or crawl and begins crying when they stand him. Swelling to his right knee is noted; it is also warm to the touch and pain response is noted. His parents state that it seemed to start a couple of days ago and has gotten worse. They don't recall a trauma, but state that he seems to bruise easily. The child's mother states that she also bruises easily. Vitals are as follows: Temp: 38.0°C, HR: 70, RR: 15. What laboratory finding would you expect? A Prolonged aPTT (activated partial thromboplastin time) B Prolonged bleeding time C Prolonged PT (prothrombin time) D Prolonged thrombin clotting time E Thrombocytopenia
*The Correct Answer is: A This patient has hemophilia A. Patients with severe hemophilia A have a prolonged aPTT;* all of the other tests should be within the normal range.
A 77-year-old female who was admitted to the hospital for acute coronary syndrome was found to have the rhythm strip shown when she arrived to the floor. Her initial vital signs were as follows: temperature is 99.0, pulse rate is 140, blood pressure is 100/65, and respiratory rate is 16. An initial bolus and infusion of amiodarone was started, with little success of slowing the rate or converting the patient's rhythm. About an hour later, the blood pressure dropped to 80/55 and she was becoming diaphoretic, with increased dyspnea and some mild chest discomfort. What is the next step in treating this patient? A Synchronized cardioversion B Increase dose of amiodarone C Infusion of magnesium sulfate D Intravenous metoprolol E Adenosine
*The Correct Answer is: A This patient has unstable, sustained ventricular tachycardia. Because of the symptoms, and most importantly the blood pressure, it is critical to intervene immediately to prevent a cardiac arrest.* The treatment option of choice in this case is synchronized cardioversion. Adenosine is not a viable option and may put the patient into a lethal arrhythmia.
Upon testing a patient for function of the hip extensors, which muscle is considered the primary muscle responsible for most extension? A Gluteus maximus B Pectineus C Semimembranosus D Semitendinosus E Vastus lateralis
*The Correct Answer is: A* The gluteus maximus is a large muscle that is partially responsible for giving shape to the buttocks. *It is the dominant muscle responsible for hip extension. It is easily palpable with a patient in the prone position with buttocks squeezed together or with the hip extended and the knee flexed.* The pectineus muscle is considered a secondary hip adductor. The Semimembranosus and Semitendinosus are two of the three hamstring muscles. They are primary movers in knee flexion, but only secondary contributors to hip extension. Vastus lateralis is one of the four quadriceps muscles and plays a role in knee extension, but not hip extension.
A 62-year-old female with a known history of hypertension presents due to the abrupt onset of a severe headache, marked vertigo, nausea, vomiting, and ataxia. On physical exam, you note nystagmus, sensory loss, and weakness of the right face. What is the most likely diagnosis? A Cervical artery dissection B Cerebellar hemorrhage C Internal auditory artery occlusion D Lacunar infarction E Spinal cord compression
*The Correct Answer is: B A cerebellar hemorrhage can result from multiple causes, similar to other intracerebral hemorrhages, such as hypertension, blood dyscrasias, trauma, and arteriovenous malformations. Patient presentation may range from an abrupt onset of headache to full coma, depending on the severity of the hemorrhage. Presentation is often similar to cerebellar infarction, with CT findings assisting in the differentiation. Symptoms may include all of those listed above as well as nuchal pain, altered consciousness, altered respiratory rate, abnormal eye movements, and impaired papillary responses.* Cervical artery dissection may present with similar symptoms, including the abrupt onset of headache, but is not typically associated with ataxia, vertigo, or vomiting. Internal auditory artery occlusion is associated with vertigo and unilateral hearing loss. Lacunar infarction occurs within small resistance vasculature, with motor and sensory symptoms related to the impacted brain region. Spinal cord compression may be acute or progressive, with motor weakness and sensory loss caudad to the lesion.
A 75-year-old man with type 2 diabetes presents to the emergency department with a 2-day history of confusion and lethargy. On physical exam, notable dehydration, tachycardia, and confused mental state is noted. Serum sodium, potassium, magnesium, and chloride levels are normal. The arterial blood gases are normal and serum ketones are negative. The abnormal laboratory findings are as follows: Given this information, what is the most likely diagnosis? A diabetic ketoacidosis B hyperglycemic hyperosmolar state C hypoglycemia D dehydration
*The Correct Answer is: B A hyperglycemic hyperosmolar state is characterized by dehydration, significant hyperglycemia, and an elevated serum osmolality with an insignificant or negative ketosis.* Because of the lack of ketosis, the patient may present with a gradual onset of symptoms, and it can go unnoticed until the dehydration becomes more severe than in ketoacidosis.
What type of fracture is not related to an acute bony trauma? A Greenstick B Stress C Oblique D Comminuted E Spiral
*The Correct Answer is: B A stress or fatigue fracture is caused by small, repetitive forces that usually involve the metatarsal shafts, the distal tibia, and the femoral neck (though many other bones may be affected). These fractures may not be seen on initial radiographs.* A greenstick fracture is an incomplete traumatic fracture with angular deformity seen in children. An oblique fracture is a traumatic fracture with an angulated fracture line. A comminuted fracture is a traumatic fracture in which there are more than two fracture segments. A spiral fracture is a traumatic fracture that has a multiplanar and complex fracture line usually caused by an excessive rotational force on a bone.
A 22-year-old man is brought to the emergency department by paramedics after having sustained a single stab wound along the left sternal border at the fourth intercostal space. Upon arrival to the emergency department, he was hypotensive and tachycardic. The neck veins were distended and heart sounds were muffled. Which of the following interventions is the most appropriate first-line management of this patient? A Left tube thoracostomy B Pericardiocentesis C Fluid resuscitation D Immediate intubation
*The Correct Answer is: B Cardiac tamponade is classically described by the triad of jugular venous distension (JVD), arterial hypotension, and muffled heart sounds. In the emergency department, suspicion of this clinically entity is usually confirmed by ultrasonography and is acutely treated by pericardiocentesis, which will be diagnostic, therapeutic, and buy time until a definitive procedure can be done.* A left tube thoracostomy may be indicated in this patient but would not relieve symptoms. Fluid resuscitation though applied to all trauma patients would help stabilize the patient until more therapeutic interventions could be completed. Immediate intubation, even if indicated, would require a prophylactic tube thoracostomy to prevent the development of tension pneumothorax in the event of an unrecognized lung injury. Emergency thoracotomy will relieve the signs and symptoms associated with cardiac tamponade and allow for repair of any underlying cardiac injuries.
A 55-year-old woman with a history of mitral valve replacement and mitral stenosis (secondary to rheumatic heart disease) presents to the emergency department with increasing dyspnea while walking up one flight of stairs. She denies chest pain and discomfort, but states that for the past few weeks she has noticed palpitations. She also admits to lower extremity edema, which is new within the last week. On EKG, she demonstrates atrial flutter with 2:1 AV block. Her INRs have been therapeutic for the past 4 weeks. Which of the following is the most appropriate next step in treating this patient? A IV quinidine B IV ibutilide C IV vasotec D IV amiodarone E IV dopamine
*The Correct Answer is: B Choice B, IV ibutilide, has been found to be most effective in converting atrial flutter to sinus rhythm out of all the choices listed.* Choice A is contraindicated, as the atrial conduction may decrease to the point that 1:1 atrial to ventricular conduction can occur with the administration of class I antiarrhythmics. The ventricular rate can then increase to rates greater than 200 bpm, and hemodynamic collapse may occur. Choice C is useful for blood pressure control, but not for heart rate control. Choice D is useful for chronic atrial flutter heart rate management, or for helping to maintain sinus rhythm after cardioversion has occurred. Choice E is useful for pressor support, and not for heart rate control or conversion to normal sinus rhythm.
A 67-year-old female presents to the emergency department with abdominal pain, bloating, inability to pass gas or stool, and vomiting. Which of the following, if present, would be a contraindication to placing a nasogastric tube? A Esophagitis B Esophageal strictures C Esophageal varices D Peptic ulcer disease E Gastroesophageal refulx disease
*The Correct Answer is: B Contraindications to nasogastric tube placement include choanal atresia, significant facial trauma, basilar skull fracture, esophageal stricture or atresia, esophageal burn, zenker's diverticulum, recent surgery on the esophagus or stomach, or a history of gastrectomy or bariatric surgery.*
A 33-year-old IV drug user presents to the emergency department with pleuritic chest pain, cough, chills, diaphoresis, anorexia, and malaise. On physical exam, her temperature is 40°C, BP 98/55, P 115 bpm, and RR 22. No murmur could be appreciated. Two separate blood cultures are positive for S.aureus. An EKG, CXR, and transesophageal echocardiogram are ordered. Which of the following lesions is most likely to be seen on TEE in this patient? A Aortic valve vegetation B Tricuspid valve vegetation C Mitral valve vegetation D Left ventricular hypertrophy E Ventricular septal defect
*The Correct Answer is: B In almost 50% of cases involving IV drug users, the only site of infection is the tricuspid valve, and most lesions are right-sided, so choice B is the most appropriate answer.* Left ventricular hypertrophy, choice D, is seen in patients with a history of hypertension. Choice E, ventricular septal defect, is frequently associated with a holosystolic murmur.
A 22-year-old patient with sickle cell disease presents to the emergency department complaining of chest pain, fever, and non-productive cough. On physical exam his temperature is 100.6˚F, BP is 144/88, pulse is 110, respiratory rate is 24, and pulse oximetry is 84%. CBC shows a WBC of 11,500, hemoglobin of 8.3%, and hematocrit of 28%. What is the most likely diagnosis? A Acute bronchitis B Acute chest syndrome C Asthma D Bronchiectasis E Pneumothorax
*The Correct Answer is: B Patients with sickle cell disease are prone to acute chest syndrome, resulting from sickling of cells within the lung that typically presents with chest pain, tachypnea, cough, fever, and oxygen desaturation.* Acute bronchitis (A) and bronchiectasis (D) typically present with a productive cough without significant oxygen desaturation and anemia. Patients with asthma (C) and spontaneous pneumothorax (E) will be afebrile and have normal hemoglobin and hematocrit.
What term is used to describe the forward movement of one vertebral body on the vertebra below it, as shown in this figure? A Spina bifida B Spondylolisthesis C Spondylolysis D Sprengel's deformity E Sustentaculum tali
*The Correct Answer is: B Spondylolisthesis is the forward movement of one vertebral body on the vertebra below. This most commonly occurs with L5 on S1 (more than 85% of cases) or L4 on L5. It is often due to a spondylosis (a bony defect) in the pars articularis as pictured above which is an acquired condition that may develop as a result of a stress fracture and it is not uncommon in children or adolescents.* A small number of spondylolithesis cases are congenital. Spina bifida refers to a non-union of the vertebral arch at the spinous process, which is a congenital condition. Sprengel's deformity refers to a scapula that only partially descends from the neck to the thorax. The higher than normal position of the scapula results in a shortened appearance of the neck. The sustentaculum tali refers to an anatomical landmark in the foot that supports the talus and serves as an attachment for the spring ligament. It is part of the calcaneus bone.
Which of the following is the initial treatment step in an adolescent who presents to the emergency department with status epilepticus? A IV glucose B stabilize airway C arterial blood gas D IV diazepam therapy
*The Correct Answer is: B Status epilepticus is a medical emergency and is defined as seizure activity that lasts a minimum of 30 minutes. This results in hypoxia, acidosis, cerebral edema, and structural damage. In addition, fever, respiratory depression, hypotension, and death may occur. There are both convulsive and nonconvulsive types of status epilepticus.* Because of its emergency status and potential complications, the clinician needs to initiate the ABCs (airway, breathing, circulation). Therefore, the first line of treatment is to establish and maintain an airway, oxygen is next, and then circulation, which encompasses pulse, blood pressure, and IV access. Once the IV is established, the orders should be for administering glucose-containing fluids and IV drug therapy with diazepam, lorazepam, or midazolam as well as administer phenytoin and phenobarbital. Arterial blood gases should be ordered and any abnormalities should be corrected appropriately. Finally, the clinician should determine the underlying cause: trauma, structural disorder, infection, lactic acidosis, toxins, and uremia. Maintenance drug therapy is necessary until the underlying cause is determined and rectified.
A disk herniation that is putting pressure on the L5 nerve root may present with weakness of what muscle(s)? A Anterior tibialis B Extensor hallucis longus C Gastrocnemius-soleus D Iliopsoas E Peroneus longus and brevis
*The Correct Answer is: B The extensor hallucis longus muscle's motor function is associated the L5 motor neuron, which also supplies the gluteus medius and extensor digitorum longus and brevis muscles.* The anterior tibialis muscle is supplied by the L4 motor neuron. Nerves emanating from T12, L1, L2 and L3 supply the iliopsoas. Gastrocnemius, soleus and peroneus longus and brevis are all supplied by nerves coming from the S1 area. The plantar flexing gastrocnemius and soleus muscles also are supplied by S2.
Upon testing a patient for function of the hip flexors, which muscle is considered the primary muscle responsible for most flexion? A Gracilis B Iliopsoas C Rectus femoris D Sartorius E Vastus intermedius
*The Correct Answer is: B The iliopsoas muscle is the primary hip flexor muscle. It originates at T12 and L1-5 vertebrae and intervertebral disks as well as the iliac fossa of the pelvis and connects to the femur at the lesser trochanter.* The gracilis muscle is considered a secondary hip adductor. Rectus femoris does help with hip flexion, but in a secondary role to the iliopsoas. Rectus femoris is also involved in knee extension. Sartorius is also involved in hip flexion, but in a secondary role. The vastus intermedius muscle is one of the four quadriceps muscles and is involved with knee extension and is not involved in hip flexion.
A soft tissue neck x-ray of a patient who complains of a progressively worsening sore throat reveals this lateral film (see image). Based on these findings, what is the initial treatment of choice for this patient? A Endotracheal intubation B Intravenous steroids C Ribovirin injection D Incision and drainage E Cricoidotomy
*The Correct Answer is: B This case of acute epiglottitis is treated with immediate intravenous steroids. Provided that the patient is able to maintain the airway and also keep oxygen saturation rates above 92%, the patient can improve with steroids and supportive care.* Antiviral medications have little effect on the overall illness.
A 78-year-old woman with known diabetes mellitus type 2 is brought to the emergency room after a neighbor became concerned when newspapers began piling up on her doorstep and called the police. In the emergency department, she is found to be lethargic and disoriented, with tenting of the skin, sunken eyes, and dry mucous membranes. She is hypotensive and has a rapid pulse. She is wearing a diaper that apparently has been in place for several days, but is barely moist. What is the most appropriate fluid therapy for them to initiate? A 0.45% saline B 0.9% saline C 5% dextrose in water D 5% dextrose in 0.45% saline E Lactated Ringers
*The Correct Answer is: B This woman is severely dehydrated so 0.9% saline is indicated.* If she were less dehydrated, 0.45% saline (A) would be appropriate because of likely hyperosmolality. Once her glucose has dropped to 250 mg/dL, she should receive dextrose in water (C) or 0.45% saline (D) to prevent her glucose level from dropping too low. Lactated Ringers ı is contraindicated in patients who are likely to have severe acidosis or alkalosis.
A 5-year-old male is being evaluated for an acute injury to the right ankle. On the x-ray of the ankle there is a distal tibia fracture that involves the separation of the epiphysis, as well as a small non-displaced chip fracture of the metaphysis of the tibia. Based on these findings, what type of Salter-Harris fracture does this child have? A I B II C III D IV E V
*The Correct Answer is: B* The growth plate is the most fragile part of the bone prior to bone maturation and thus is usually the first structure disrupted when force is applied. *Statistically, Type II fractures are most common - those that involve both the growth plate and a chip fracture of the metaphysis.*
A 64-year-old man has been experiencing signs and symptoms compatible with diverticular disease for the past 3 weeks. He now presents to the emergency department malnourished with severe left-sided lower abdominal pain. After appropriate workup and hydration, he is taken to the operating room where a perforated sigmoid colon is discovered with gross contamination. What is the most appropriate surgical intervention at this time? A Left colectomy with primary anastomosis B Hartmann procedure C Proctocolectomy D Abdominoperineal resection E Low anterior resection
*The Correct Answer is: B* This vignette is consistent with an emergent resection in an unprepared patient. The most appropriate therapy for an acute perforation is a Hartmann procedure, which *includes resection of the affected portion of the bowel, a temporary diverting colostomy, and oversewing of the distal rectal stump;* the second stage of the procedure will involve taking down the colostomy with anastomosis to the rectal stump. A colectomy with a primary anastomosis should not be done when the bowel is unprepared due to the significant risk of infection and leakage of the bowel at the site of the anastomosis. Abdominoperineal resection is used in the treatment of malignant disease of the lower rectum. In this procedure, a permanent colostomy is created and the entire rectum, anal canal, and anus are removed. In the management of benign disease of the lower rectum, a proctocolectomy is appropriate to preserve anal function.
A 24-year-old intoxicated male presents to the emergency department after being in a fight. He was punched in the nose, and now has mild deformity of the nose and some epistaxis. An x-ray reveals a fractured nasal bone. During his physical exam, what must you look for in order to prevent permanent destruction of his nasal septum? A Orbital fracture B Posterior epistaxis C Septal hematoma D Facial fracture E Deviated septum
*The Correct Answer is: C A septal hematoma can cause ischemic necrosis of the nasal septal cartilage if not identified and drained.* A deviated septum can be expected with a nasal bone fracture, and must be addressed by the otolaryngologist. Excessive epistaxis that does not resolve with direct pressure and anterior packing may indicate a posterior bleed.
An 80-year-old male nursing home patient is brought to the emergency department with abdominal distension. A plain film of the abdomen is pictured below. Which of the following is the most likely diagnosis? A Small bowel obstruction B Cecal volvulus C Sigmoid volvulus D Toxic megacolon
*The Correct Answer is: C A volvulus is an obstruction of the colon due to a loop of bowel that has rotated more than 180 degrees on its axis with the mesentery. The most common site for a volvulus is the sigmoid colon (65%). A sigmoid volvulus is associated with abdominal pain and distension. Plain films of the abdomen would show a characteristic "bent inner tube" appearance. Sigmoidoscopy can be used to decompress the bowel by gently releasing the area of obstruction.* Following decompression, a rectal tube is inserted to act as a stent to prevent the bowel from twisting upon itself again.
A 68-year-old female presents to the emergency department with signs and symptoms of an acute ischemic stroke. The initial CT scan is normal. Her blood pressure is 164/105. What is the most appropriate treatment for the blood pressure of this patient? A Atenolol PO B Clonidine PO C Close monitoring D Labetolol IV E Nicardipine IV
*The Correct Answer is: C Aggressively lowering blood pressure may decrease blood flow to the ischemic tissue, thus decreasing the chances of recovery or increasing the risk of further infarction. In the setting of an acute ischemic stroke, blood pressure elevation should be monitored closely, with some elevation expected.* This elevation is expected to decline without medication in the first few hours to days, but if elevation continues to a systolic blood pressure greater than 220mmHg, or mean arterial pressure greater than 120mmHg, medication is advised. Medications may include intravenous labetolol or nicardipine, with close monitoring of the patient. After the acute phase following a stroke, appropriate oral medications may be considered for outpatient hypertension management.
A 20-month-old boy is brought into the emergency department by his parents. They state he has not been feeling well for 2 days and this morning noted he was "shaking all over" and was not responding to commands. This went on for less than 10 minutes and has never happened before. His current rectal temperature is 100.7°F. The seizures are characteristic of A absence seizures B Lennox-Gastaut syndrome C febrile seizures D infantile spasms E juvenile myoclonic epilepsy
*The Correct Answer is: C Febrile seizures can occur in children younger than 5 years when accompanied by a fever. They are characterized by a brief generalized motor seizure. Absence seizures are generalized seizures characterized by a loss of consciousness without motor involvement, typically seen in older children.* Lenox-Gastaut syndrome presents in childhood as well but is usually associated with developmental delay and seizures of akinetic and myoclonic nature (referred to as drop attacks). Infantile spasms occur without relation to systemic illness and are massive myoclonic events with bending at the waist. Juvenile myoclonic epilepsy evolves in the teenage years and is characterized by repeated episodes of myoclonic seizure activity.
A 46-year-old female presents with pain to her left wrist. She complains that it is painful and swollen as she points to the volar aspect of the wrist on the radial side. On examination, there is a small, soft bump on the dorsum of her wrist with a jelly-like consistency. What is the most likely diagnosis? A Cancerous tumor B Fracture C Ganglion cyst D Hematoma E Lipoma
*The Correct Answer is: C Ganglion cysts commonly occur on the dorsal or volar aspect of the wrist. They result when a joint capsule or tendon sheath is damaged, allowing synovial fluid to escape producing a one-way valve, which allows fluid into the cyst, but not back out. The accumulating fluid forms the ganglion cyst. These cysts may or may not be tender and can fluctuate in size depending on activity level of the affected extremity.* Cancerous tumors would tend to be much more firm, but also may be relatively pain free. Fractures would generally be exquisitely tender and if the bump is due to a displaced bone, it would be much more firm than a ganglion cyst. Hematomas are generally associated with acute trauma and would be tender and ecchymotic in many situations. Lipomas are benign fatty tumors that are more commonly seen on the thenar eminence than the dorsum of the wrist and their size does not change based on activity level.
What is the most common clinical finding on vital signs in a patient with cardiogenic shock? A Fever B Hypertension C Hypotension D Tachypnea E Bradycardia
*The Correct Answer is: C Hypoperfusion to the body is what generally causes the end organ failure associated with shock.* The patient may also be tachycardic, but may not have fever or an increased respiratory rate as part of the abnormality. In most cases patients will have tachycardia unless they suffer a significant pump failure, which may cause bradycardia.
A 70-year-old woman who was found barely responsive at home by her daughter is brought to the emergency department. Evaluation reveals that she is in a hyperglycemic hyperosmolar state with a severe fluid deficit. Treatment is initiated with vigorous saline rehydration and a continuous infusion of insulin. At what point should her glucose be added to her treatment? A when her condition becomes stable B when her urine output reaches 50 mL/hour C when her blood glucose reaches 250 mg/dL D if she develops hypokalemia E if she begins to spill ketones in her urine
*The Correct Answer is: C In hyperglycemic hyperosmolar states, the serum glucose rapidly corrects with fluid administration alone. However, with vigorous rehydration, glucose may fall precipitously and lead to severe hypoglycemia. To avoid this, glucose should be added to water, half-normal, or normal saline as soon as the patient's blood glucose is less than or equal to 250mg/dL. She should continue to receive insulin IV until she is stabilized* (A) when it can be switched to subcutaneous administration. The goal of fluid therapy in this patient is restoring her urine output to 50 mL per hour (B) or more. Because insulin drives potassium into the cells and can cause hypokalemia (D), potassium chloride should be given unless the patient has chronic kidney disease or oliguria. Persons in a hyperglycemic hyperosmolar state typically do not spill ketones (E) the way persons with diabetic ketoacidosis do.
What absolute tissue pressure generally is used as a guideline for diagnosing compartment syndrome? A 10 mm Hg B 20 mm Hg C 30 mm Hg D 40 mm Hg E 50 mm Hg
*The Correct Answer is: C Many trauma surgery services use an absolute tissue pressure of approximately 30 mm Hg as the threshold for diagnosing compartment syndrome.* Based on the entire clinical picture, patients with numbers in that range or higher will likely require surgical decompression with a fasciotomy, while lower numbers will probably be managed with a more conservative approach.
A 4 year-old male presents to the emergency department with vomiting, severe abdominal pain, and diarrhea that occurred 2 hours after "getting candy from Grandma's purse". Which of the following medications did the child most likely ingest? A Cisplatin B Erythropoietin C Ferrous sulfate D Lisinopril E Vincristine
*The Correct Answer is: C Stage 1 of iron toxicity secondary to ferrous sulfate ingestion (C) is characterized by acute GI irritation; this is followed by a latent phase (Stage 2) that can then progress to systemic iron toxicity (Stage 3), that can progress to hepatic failure (Stage 4) or delayed sequelae (Stage 5).* Cisplatin (A), erythropoietin (B), and vincristine (E) are parenterally administered and unlikely to be present in a patient's purse. Lisinopril (D) toxicity consists of cardiovascular symptomatology (e.g., hypotension and tachycardia).
A 27-year-old African American with sickle cell anemia presents to the emergency department with acute onset intractable pain. She is taking quick, shallow breaths and her oxygen saturation is 84% on room air. She appears desiccated, states she hasn't eaten in the last 24 hours, and says that she "just doesn't feel well." She is also afebrile. What should your next course of action be? A Start morphine, hydrate, and start antibiotics B Start morphine, oxygen, and start antibiotics C Start oxygen, hydrate, and exchange transfusion D Start oxygen, hydrate, and give pneumococcal vaccination E Start oxygen, hydrate, and start antibiotics
*The Correct Answer is: C Start oxygen, hydrate, and exchange transfusion* Exchange transfusions are primarily indicated for the treatment of intractable pain crises, priapism, and stroke. Patients should be kept well hydrated, and oxygen should be given if the patient is hypoxic. Antibiotics would be used if there was an infection identified, but are not part of initial treatment in a sickle crises.
A 14-year-old boy presents to the emergency department with acute scrotal pain and vomiting for the past 2 hours. His left testicle is in extreme pain and he states the pain started while playing basketball in gym class. Which diagnosis is highest on your differential in regards to his clinical presentation? A Epididymitis B Hydrocele C Testicular torsion D Varicocele E Intrascrotal tumor
*The Correct Answer is: C Testicular torsion (C) is most common between ages 12-18 with the classic presentation of abrupt and severe onset of pain with nausea/vomiting.* The most common cause of painless scrotal swelling in children is a hydrocele (B). Epididymitis (A) usually has an insidious onset, commonly presenting with urinary frequency, dysuria, or fever. Varicoceles (D) and intrascrotal tumors (E) usually present as painless scrotal edema.
Radiculopathy due to nerve root compression occurs most commonly at which nerve root within the brachial plexus? A C5 B C6 C C7 D C8 E T1
*The Correct Answer is: C The C7 nerve root is affected the most often (approximately 45-60%). This radiculopathy can result from foraminal encroachment of the spinal nerve, cervical disk herniation, tumor, and multiple sclerosis. C7 radiculopathy can present with weakness in the triceps, which cause elbow extension, and finger flexion and extension.* C6 is another common site of radiculopathy. C6 radiculopathy can present with weakness in the biceps, brachioradialis, and wrist extensor muscles. Cervical radiculopathy at the C5, C8, and T1 are less common, but still possible. C5 radiculopathy can present with deltoid and biceps muscle weakness. C8 radiculopathy can present with finger flexor weakness and T1 radiculopathy with finger abduction weakness.
A 21-year-old man presents to the emergency department complaining of a swollen left ear (see image) after he experienced blunt trauma in a collegiate wrestling match. Which of the following is the most appropriate management for this condition? A Evacuation B Evacuation followed by antibiotics C Evacuation with subsequent antibiotics and splinting D Ice and compression E Reevaluation in 24 hours
*The Correct Answer is: C The patient has an auricular hematoma that must be evacuated to prevent cartilage necrosis or infection. Evacuation, antibiotics, and splinting* (C) remove the hematoma, minimize the risk of infection, and help prevent the re-accumulation of blood.
A 17-year-old boy high school wrestler is brought into the emergency department after he collapsed at a wrestling match. He spent time fully clothed in a hot sauna prior to the match to try to "make weight." Labs are ordered, and results come back as follows: Which IV fluid regimen would most effectively treat this patient's hypernatremia? A quarter normal (hypotonic) saline B half-normal saline C isotonic (normal) saline D dextrose 5% in water E lactated Ringer's
*The Correct Answer is: C The patient presents with a combination of inadequate fluid intake and excessive losses due to perspiration, resulting in hypovolemia and hypernatremia.* The most common causes of hypernatremia are inadequate fluid intake resulting in hemoconcentration and diabetes insipidus (DI), resulting in excessive renal fluid losses. Normal urine osmolality is 500 to 850 mOsm/kg but can range from 50 to 1,200 mOsm/kg depending on the patient's fluid intake. Urine osmolality >400 mOsm/kg indicates that the renal fluid-conserving mechanism is intact, as the kidneys are working to preserve volume. A lower urine osmolality would be consistent with DI, characterized by a lack of response to anti-diuretic hormone (ADH), resulting in excessive urinary losses of water with worsening hypernatremia. Treatment is directed at the cause. If the patient is dehydrated, restoring fluid volume is the goal. If the patient has DI, treating the underlying disease will lower the serum sodium level. For this dehydrated patient, the treatment would be to administer isotonic (normal) saline, which contains 0.9% sodium, because of the large free water deficit. Quarter-normal saline contains 0.25% sodium, half-normal saline contains 0.45% sodium, and lactated Ringer's solution is similar to half-normal saline in its sodium content. Dextrose 5% in water (D5W) contains no electrolytes. Isotonic saline is the appropriate choice because it treats not only the volume deficit but the serum osmolality as well. Its osmolality (308 mOsm/kg) is often lower than the plasma osmolality because of the hypovolemic state and, therefore, helps restore normal serum osmolality. Once serum osmolality becomes more normal, the isotonic saline can be replaced by D5W to replace the remaining free water deficit. If the free water deficit were less dramatic, initial IV fluid treatment could be half-normal saline, followed by D5W.
A 43-year-old male presents to the Emergency Department complaining of right eye pain after treating his yard with fertilizer and lime. He attempted to flush his eye at home without relief of pain. Which of the following is the most appropriate initial step in managing this patient's symptoms? A Double evert his eyelids to look for remaining foreign bodies B Fluorescein stain his eye C Instill proparacaine 0.5% ophthalmic solution D Irrigate his eye until the pH is between 6.8 and 7.4 E Refer to ophthalomogist
*The Correct Answer is: C The patient requires all of the above steps and should be given pain relief (C) prior to thoroughly flushing the eye* (D), removing foreign bodies (A), assessing for corneal injuries (B), and referring to ophthalmology (E).
Upon testing a patient for function of the hip abductors, which muscle is considered the primary muscle responsible for most abduction? A Biceps femoris B Gluteus maximus C Gluteus medius D Gluteus minimus E Vastus medialis
*The Correct Answer is: C The primary mover in the motion of hip abduction is the gluteus medius muscle.* Gluteus minimus does play a supporting role in that motion. Biceps femoris is one of the three hamstring muscles and contributes to the motions of knee flexion (primary muscle) and hip extension (secondary muscle). Gluteus maximus is the primary mover for hip extension and vastus medialis is one of the four quadriceps muscles responsible for knee extension, but no hip movements.
A 27-year-old man presents to the emergency department with a five-day complaint of substernal pleuritic chest pain, which worsens while lying supine. He is in no distress. A friction rub is noted over the precordium. The patient's vital signs are as follows: temperature is 100.4˚F, pulse rate is 94, respiratory rate is 20, and blood pressure is 136/84. An ECG reveals widespread diffuse ST elevations with PR interval depressions. He was recently treated for a viral respiratory infection. Which of the following is the most appropriate initial management for this patient? A Administer a broad spectrum antibiotic B Administer intravenous tissue plasminogen activator (T-PA) C Begin a nonsteroidal anti-inflammatory agent D Perform needle thoracentesis E Refer for immediate cardiac catheterization
*The Correct Answer is: C This patient's diagnosis is acute inflammatory pericarditis. Viral infections are the most common cause of acute pericarditis, and males are the most commonly affected. A pericardial friction rub and EKG changes are characteristic of this diagnosis. Treatment is focused on the underlying inflammation, with NSAIDS being first-line and short course corticosteroids also being appropriate.* Antibiotics are not indicated unless a bacterial etiology is confirmed or there are significant risk factors. Choices B, D, and E are not appropriate for this condition and could be harmful.
A 17-year-old male is brought to your Emergency Department by his girlfriend. She states that he has been behaving strangely for the last three days, with rapidly fluctuating moods ranging from euphoric to irritable and paranoid. The patient states that he is fine, just a little nervous about an upcoming test in school. His pulse is 126 beats per minute, BP 182/106, pupils are widely dilated, and he is diaphoretic. What is his most likely diagnosis? A Acute anxiety B Bipolar disorder C Cocaine intoxication D Heroin intoxication E Thyrotoxicosis
*The Correct Answer is: C This patient's presentation with tachycardia, hypertension, diaphoresis, and mydriasis along with the behavioral changes is consistent with cocaine intoxication (C).* His physical symptoms could be explained by thyrotoxicosis (E), but he has no history of hyperthyroid symptoms such as weight loss and it would not explain the mydriasis. Acute anxiety (A) or bipolar disorder (B) might explain some of the mood changes but would not account for all of his physical signs. Heroin withdrawal (D) would cause some similar symptoms, but intoxication would cause somnolence and pinpoint pupils.
A 16-year-old girl is brought to the emergency department by ambulance after reportedly ingesting "a bottle of aspirin." Vital signs are temperature 37.8°C oral; pulse 94/min; respirations 30/min; blood pressure 100/68 mm Hg. What would you expect the blood gases to show that would confirm she had swallowed the aspirin? A anion gap metabolic acidosis with respiratory acidosis B nonanion gap metabolic acidosis with respiratory alkalosis C anion gap metabolic acidosis with respiratory alkalosis D nonanion gap metabolic acidosis with respiratory acidosis
*The Correct Answer is: C* An acute salicylate overdose (greater than 150 mg/kg) will produce symptoms of salicylate intoxication. Chronic salicylate intoxication occurs with ingestion of greater than 100 mg/kg/day for at least 2 days. Salicylates affect most organ systems, leading to various metabolic abnormalities. Because salicylates are a gastric irritant, symptoms of vomiting and diarrhea occur soon after the overdose, which may contribute to the development of dehydration. *Salicylates stimulate the respiratory center leading to hyperventilation and hyperpnea resulting in respiratory alkalosis and compensatory alkaluria. A characteristic feature of salicylate intoxication is the coexistence of a respiratory alkalosis with a widened anion gap metabolic acidosis.*
An 8-year-old girl is rushed to the emergency department by her parents because she has become delirious. The child was diagnosed with influenza three days prior. Her parents say that she had begun vomiting yesterday, almost nonstop, and has not been able to hold down fluids. They also note that she has been breathing rapidly. Your exam reveals a tachypneic, disoriented female with hyperreflexia, a positive babinski reflex, and liver enlargement. CSF analysis reveals a normal protein and cell count. What is the most likely diagnosis? A Bacterial meningitis B Guillain Barre syndrome C Measles encephalitis D Reye's syndrome E Viral meningitis
*The Correct Answer is: D Although rare, Reye's syndrome is associated with viral infections, salicylate use during illness, and metabolic disorders. Illness is associated with liver fat deposition and degeneration, intractable vomiting, and mental status changes, which may progress to seizures, delirium, and coma. Cerebral edema contributes to these changes and other neurologic findings.* Meningeal signs are more consistent with meningitis. Measles encephalitis typically presents days to weeks after the pathognomic measles exanthem and clinical findings. Guillain Barre has been associated with influenza infection, and signs and symptoms would include evolving weakness with ascending paralysis and extremity dysesthesias.
An avulsion fracture at the base of the fifth metatarsal is commonly called which of the following? A Bennett fracture B Boxer's fracture C Chauffer's fracture D Jones fracture E Lisfranc fracture
*The Correct Answer is: D An avulsion fracture at the base of the fifth metatarsal, usually secondary to plantar flexion and inversion is called a Jones fracture.* Also called a ballet or dancer's fracture, it is the most common metatarsal fracture. The fracture occurs at the proximal diaphysis. A Bennett fracture is an oblique fracture of the first metacarpal near the carpometacarpal joint. A boxer's fracture is a fracture of the fifth metacarpal. This is the most common fracture of the hand. A chauffer's fracture is an oblique fracture through the base of the radial styloid in the forearm. A Lisfranc fracture is actually a fracture and dislocation involving the tarsometatarsal joints.
A 16-year-old male soccer player is complaining of pain to the right foot that has been getting progressively worse for the last 2 months. He states it hurts the most when he has all of his weight on his right foot as he plants to kick the ball. Most of the pain appears to be on weight bearing. You are concerned that this patient may be developing a stress fracture. Based on the patient's history and patient presentations, which bone is the most affected by stress fractures in the foot? A Calcaneus B Fifth metatarsal C First metatarsal D Second metatarsal E Talus
*The Correct Answer is: D Any bone that is exposed to repetitive stress can have a stress fracture, but the long and thin metatarsal bones of the foot are the most commonly affected bones. Of the metatarsals, the second metatarsal has the highest number of stress fractures.* These weight bearing bones can be particularly vulnerable to stress fracture if the patient is involved in long distance running, especially if he/she is wearing improper footwear for that activity or footwear that has lost most of its shock absorbing abilities. Some young female athletes may be training so hard that they become amenorrheic which can contribute to osteopenia resulting in weaker bones. Older patients with osteoporosis will also have a higher risk of stress fracture. Initially stress fractures of the metatarsals may present with a small area of localized pain and the dorsal forefoot may demonstrate a fairly diffuse area of swelling. If the stress fracture is not treated early, some patients will experience an audible pop or crack as the incomplete stress fractures progresses to a complete break. All types of fractures occur more easily in long thin bones like the metatarsals, than thicker bones like the calcaneus and talus.
A 68-year-old woman with a history of hypertension and diabetes mellitus type 2 comes to the emergency department with her son, who noticed that while decorating for Christmas she seemed more dyspneic than normal, and had to sit down frequently. In addition, he noticed that she was pale and diaphoretic, and insisted on driving her to the emergency department. On questioning, she denies chest pain, but admits to being more fatigued than usual, with frequent jaw discomfort during activity. Activities such as vacuuming her house cause dyspnea, and she now has to stop several times while carrying laundry up from the basement. On physical examination, the patient's blood pressure is 90/50, pulse 99 bpm, respirations 22, and she is afebrile. Auscultation of the chest demonstrates a new systolic murmur. An EKG demonstrates normal sinus rhythm with nonspecific ST and T wave changes. Which of the following would be the most appropriate next step in the management of this patient? A Transesophageal echocardiogram B Nuclear stress test C Cardiac catheterization D Serial serum troponin levels E CXR
*The Correct Answer is: D Choice D, checking serial serum troponin levels, is the most appropriate next step in the management of this patient. Women and diabetics may present with atypical symptoms with acute non-ST-segment myocardial infarction, including dyspnea, jaw discomfort, and epigastric discomfort.* Frequently, women present much later than men with these symptoms. Therefore, a high level of suspicion should be maintained when women present with symptoms of dyspnea, even in the setting of nonspecific EKG changes, and drawing serum troponin levels before any other testing is recommended. Once non-ST-segment myocardial infarction has been ruled out, choices E and B, and also transthoracic echocardiogram, would likely be evaluated. Transesophageal echocardiogram may be required if better visualization of the heart valves is required, but not as the next step. Choice C, cardiac catheterization, would likely occur if an abnormal stress test demonstrating symptoms of myocardial ischemia is found.
A 70-year-old man, with a history of pulmonary hypertension and obstructive sleep apnea, presents with complaints of increasing dyspnea while walking his dog. He has also recently noted increased lower extremity edema. On physical examination, jugular venous distension is noted. Auscultation of the chest demonstrates a high-pitched blowing diastolic murmur. The murmur is heard over the second and third left intercostal spaces. An S 3 is appreciated. Abdominal exam reveals hepatomegaly and splenomegaly. Which of the following maneuvers would be the most appropriate to choose for better identification of the murmur? A Left lateral decubitus position listening with the bell of the stethoscope B Standing C Seated leaning forward D Inspiration E Expiration
*The Correct Answer is: D Choice D, inspiration, will increase the intensity of the murmur of pulmonic regurgitation/insufficiency. The Valsava maneuver will diminish the intensity of the murmur.* Choice B, standing, will cause the mid-systolic click associated with mitral valve prolapsed to move toward S1 or become more audible. Choice A, rolling the patient to the left lateral decubitus position, is most useful in identification of the murmur of mitral stenosis. Choices B, C, and E, have no effect on the Graham Steell murmur of pulmonic regurgitation.
During a hospitalization for pneumonia, troponin levels are drawn on a 62-year old-man with a history of hypertension, hyperlipidemia, and chronic tobacco use, and found to be elevated above the 99 th percentile of normal. If acute myocardial infarction is ruled out, which of the following disease entities could also cause troponin elevation? A Mitral regurgitation B Gout C Parkinson's disease D Sepsis E Herpes zoster
*The Correct Answer is: D Choice D, sepsis, is one of a long list of disease entities that can cause troponin elevation, including arrhythmias (both tachycardic and bradycardic), aortic valve disease, hypertrophic cardiomyopathy, invasive cardiac surgeries and procedures, severe pulmonary hypertension, pulmonary embolism, myocardial infiltrative diseases (such as amyloidosis, sarcoidosis, scleroderma, and hemochromatosis), acute respiratory failure, burns, pericarditis, endocarditis, myocarditis, and even occasionally due to extreme athletic activities such as marathon running.* Not included on this long list, however, are choices A, B, C, and E.
A 46-year-old man with a history of EtOH abuse is brought to the emergency department in the morning by his wife. She has noted that he has developed tremors in both arms, and he seems mildly confused to her. He complains of feeling weak, with some cramping in the legs. On physical examination, his blood pressure is noted to be 162/95 mm Hg, and his heart rate is 108 beats/min. There is no asterixis. Which of the following electrolyte disorders are you likely to find in this patient? A hypercalcemia B hypocalcemia C hypermagnesemia D hypomagnesemia E hyperphosphatemia
*The Correct Answer is: D Hypomagnesemia is a common finding in the patient who abuses alcohol. Other leading causes include diarrhea, diuretics, aminoglycosides, and amphotericin B.* The etiology of hypomagnesemia in the patient with a history of alcohol abuse is thought to be a combination of malabsorption and inadequate dietary intake, possibly with alcohol exerting an antagonistic effect on absorption. Signs and symptoms are those of neuromuscular and central nervous system hyperirritability, including weakness and muscle cramps, tremors, nystagmus, a positive Babinski response, confusion, and disorientation. Hypertension, tachycardia, and ventricular arrhythmias may develop.
A 17-year-old female distance runner with no significant PMH complains that she has diffuse, aching anterior knee pain that is worsened when she walks up or down stairs or when she squats down. There has been no acute trauma, but she has been increasing her running mileage. No effusion is present. What is the probable diagnosis? A Anterior cruciate ligament tear B Medial meniscal tear C Osteoarthritis of the knee joint D Patellofemoral syndrome E Posterior cruciate ligament tear
*The Correct Answer is: D Patellofemoral syndrome is a common condition in active adolescents due to repetitive stresses on the patellofemoral joint. This can be exacerbated by altered patellofemoral tracking due to growth and development in adolescents.* Other biomechanical issues that may contribute to this problem include poor flexibility, weakness of the vastus medialis muscle, which results in a more lateral tracking of the patella, and excessive foot pronation. This condition does not produce an effusion. Anterior and posterior ligament tears would be highly unlikely given that there was no history of an acutely traumatic event and because ligaments are fairly vascular structures, an effusion would be expected with any cruciate ligament tear. Meniscal tears in a young patient are generally associated with a traumatic twisting event and while not as vascular as a ligament, meniscal tears generally lead to the gradual development of an effusion. Older patients may experience a degenerative tear with minimal trauma, but that is unlikely in our 17-year-old patient. Osteoarthritis is generally associated with gradual wear and tear after many years or the process can be accelerated after a significant acute trauma to a joint. Neither scenario is applicable to this patient.
A 55-year-old woman with a history of mitral stenosis (secondary to rheumatic heart disease) presents to the emergency department with increasing dyspnea while walking up one flight of stairs. She denies chest pain and discomfort, but states that for last 24 hours she has also noticed palpitations. She also admits to lower extremity edema, which is new within the last week. On EKG, she demonstrates atrial flutter with 2:1 AV block. Which of the following is the most appropriate therapy for this patient? A Aspirin 325 mg B Plavix 75 mg C Plavix 75 mg and aspirin 81 mg D Warfarin, dosed to INRs between 2.0 and 3.0 E Dipyridamole 200 mg and aspirin 25 mg
*The Correct Answer is: D Patients with atrial flutter of less than 48 hour duration may be cardioverted without anticoagulation, unless they have mitral valve disease, in which case they should be treated with warfarin;* therefore, choice D is the most appropriate choice. Choices A, B, C and E are inappropriate, as there is no data to support that any of these therapies offer adequate anticoagulation for embolism prophylaxis, for either atrial fibrillation or atrial flutter. Choice A is recommended for cardiac risk factor modification for men, according to the Framingham Risk Trial. Choice C is appropriate therapy following PTCA and stent placement.
A 56-year-old male who works in construction climbing ladders has developed pain to the right foot for several days. You have seen and examined the patient a few days after the patient started complaining of pain to the foot. Your initial x-rays of the foot are negative for fracture. He continues to have pain, and decreased ability to bear weight. Based on this clinical scenario, how many days after the initial examination should another x-ray be ordered to look for a stress fracture? A 1-2 days B 5-7 days C 10-12 days D 14-30 days E 180 days
*The Correct Answer is: D Radiographic evidence of stress fractures is not immediately apparent after the initial onset of symptoms. Estimates of anywhere from 14 to 42 days before visible signs of a stress fracture can be detected on X-ray have been put forth with a commonly utilized range of 14 to 30 days. Any answer choice above that included numbers less than 14 days would simply be inaccurate due to the gradual appearance of the classic x-ray findings of a stress fracture.* After 180 days from the time of stress fracture onset, the fracture should be nearly healed if managed properly. If a stress fracture is highly suspected in light of negative x-rays, imaging with a bone scan should help make the diagnosis. MRI scans may help confirm the diagnosis, but they are rarely utilized in the work-up of suspected stress fractures.
A 76-year-old male presents to the hospital with a hot, swollen, and painful right knee that appears to have an effusion. After your examination, you decide to perform an arthrocentesis. The fluid aspirate is turbid, cloudy, and most definitely looks like it is infectious material. Based on this clinical scenario, what would be most likely bacterial organism present in an elderly septic arthritis? A Hemophilus influenzae B Neisseria gonorrhoeae C Pseudomonas aeruginosa D Staphylococcus aureus E Streptococcus pneumonia
*The Correct Answer is: D Septic arthritis is an infection within a joint space. The infection can be caused by multiple pathogens including bacteria, viruses, fungi and mycobacteria, but in older adults the most common organism implicated is Staphylococcus aureus. These bacteria can get into the joint space through multiple mechanisms including direct inoculation after a penetrating injury, hematogenous spread from an infection elsewhere in the body, or from extension from a contiguous bone infection.* All of the bacteria mentioned as possible answers are possible causes of septic arthritis, but there prevalence varies within different patient populations. Hemophilus influenza and Streptococcus pneumonia are commonly seen in children less than five years of age that develop septic arthritis. Neisseria gonorrhoeae is most common in sexually active adolescents and young adults. Pseudomonas aeruginosa is a common cause of septic arthritis in IV drug abusers.
A 14-year-old boy presents to the emergency department with acute scrotal pain and vomiting for the past 2 hours. His left testicle is in extreme pain and he states the pain started while playing basketball in gym class. On physical exam of the affected testicle, which of the following findings would suggest testicular torsion? A Transillumination B Positive Prehn's sign C Positive cremasteric reflex D Abnormal transverse lie E Mass of enlarged veins palpated
*The Correct Answer is: D Testicular torsion is most common between ages 12-18 with the classic presentation of abrupt and severe onset of pain with nausea/vomiting. The testicle on physical examination is painful, swollen, high-riding, tender, and has an abnormal transverse lie (D).* Transillumination (A) is when light is placed behind the scrotum and fluid is illuminated in cases of hydroceles. Prehn's sign (B) is pain relief with elevating the scrotum and is positive in cases of epididymitis. The cremasteric reflex (C) is a normal finding that causes elevation of the testis on the ipsilateral side when the inner aspect of the inner thigh is stroked. The absence of cremasteric reflex on the affected side is often found in acute torsion. A mass of enlarged veins (E), or "bag of worms," is a finding associated with a varicocele.
A 23-year-old man, unrestrained driver, is brought to the emergency department by ambulance after having been involved in an automobile accident. His vitals are BP: 99/54 mm Hg, P: 112/min, R: 18/min, oxygen saturation: 99%, T: 99.8°F. Examination reveals mild abdominal tenderness with pain radiating to the right shoulder. What is the most appropriate diagnostic test to order initially? A Computed tomography of the abdomen and pelvis B Diagnostic peritoneal lavage C Flat and upright abdominal radiographs D Diagnostic ultrasound
*The Correct Answer is: D The initial evaluation of blunt abdominal trauma is by the performance of a FAST (focused assessment with sonography for trauma) ultrasound, which is performed by an emergency department physician or surgeon.* CT scan remains an adjunct test in hemodynamically stable patients or in patients in whom further assessment of solid intra-abdominal organs is required.
A 56-year-old right hand dominant male presents with swelling to the right elbow. He explains to you that he has had this type of swelling before and was diagnosed with bursitis. Based on this description, where would the most likely site for the elbow bursitis be located? A Capitellum B Lateral epicondyle C Medial epicondyle D Olecranon E Radial head
*The Correct Answer is: D The olecranon bursa is very vulnerable to injury and chronic inflammation due to its superficial location on the extensor side of the elbow.* Acute falls, chronic pressure, and other inflammatory processes such as rheumatoid arthritis or gout may contribute to olecranon bursitis. The medial and lateral epicondyles are known to be associated with chronic tendonopathies. The radial head is a common location for subluxation or fracture, while the capitellum at the distal end of the humerus is sometimes associated with fractures and articular cartilage damage.
A 23-year-old man presents to the outpatient clinic for follow-up from a recent urgent care visit. He complains of sore throat, fever, fatigue, myalgias, and a rash that started 5 days ago, and have worsened since he was seen in the urgent care 3 days ago. The patient appears non-toxic with a temperature of 39.4 degrees Celsius. Physical exam reveals pharyngeal and tonsillar erythema without exudates, generalized lymphadenopathy, a morbilliform rash on his trunk, and no hepatosplenomegaly. A rapid strep screen and Monospot performed at the local urgent care were reportedly negative. Which of the following laboratory tests is most likely to confirm the expected diagnosis? A Complete blood count B Cytomegalovirus titer C Group A beta-hemolytic strep culture D HIV viral load E Epstein-Barr virus titer
*The Correct Answer is: D The patient presentation is consistent with acute retroviral syndrome, which is best confirmed during this initial presentation phase through direct testing for the HIV virus, such as an HIV viral load (D).* The lack of tonsillar exudates, a negative monospot, and presence of generalized adenopathy make infectious mononucleosis (B and E) less likely. A CBC (A) may show lymphopenia and support the diagnosis, but it doesn't confirm the diagnosis.
A 22-year-old woman, with no previous medical problems, suddenly cried out, fell to the ground, extended her legs, flexed her arms, and jerked her extremities for 30 seconds. There was associated tongue biting and urinary incontinence. She awoke slowly over a 10-minute period and recalled nothing about the episode. She remained lethargic for several hours but the rest of her neurologic examination was normal. What is the most likely etiology for this episode? A epilepsy B hyperventilation C cardiac arrhythmia D seizure E stroke
*The Correct Answer is: D This event represents a well-demarcated episode affecting some combination of consciousness, motor, and/or sensory function consequent to abnormal electrical discharges in the brain. This is consistent with the definition of a seizure. Epilepsy refers to multiple, recurrent seizures.* This history is not consistent with hyperventilation, stroke, or cardiac arrhythmia, which would typically include chest pain, shortness of breath, dyspnea on exertion, or focal neurological deficits.
A 22-year-old African American male presents to the emergency department with shortness of breath, which started 2 hours prior to arrival. He does not have a history of pulmonary disease that he is aware of, and he states that in the past at random events he has had similar episodes. He does nothing to get the episodes to stop, and he also states that he feels his chest pounding at the same time of the shortness of breath. He has no medical history that he is aware of, and he takes no medications or any illicit drugs. On examination he is alert, awake, and oriented. His vital signs show T 99.0, P 142, R 18, and BP 132/82. His chest x-ray is negative for any acute cardiopulmonary disease, and his electrocardiogram has an irregularly irregular rate of 142 with visible delta waves. Given the clinical situation above, what is the best medication for managing this patient's condition with a long-term approach? A Amiodarone B Atropine C Hydralazine D Flecainide E Digoxin
*The Correct Answer is: D This patient has Wolff-Parkinson-White (WPW) syndrome along with atrial fibrillation and a rapid ventricular response. Of the choices given in managing this patient's tachycardia, oral flecainide* (D) will serve to slow the process within the accessory pathway and prolong the refractory period. Amiodarone (A) has been shown to not be effective in managing the patient's tachycardia with respect to WPW, and the other medications (B, C, and E) would not have an effect on the condition and may actually worsen the patient's symptoms.
A 42-year-old healthy male presents to the emergency department with the complaint of a progressively worsening sore throat, and difficulty swallowing over the past 48 hours. He also complains of a subjective fever, but denies any headaches, nausea, or vomiting. On exam, the patient is afebrile and in mild distress, with a presentation of leaning forward on the exam table. His TM examination is normal, there is no rhinorrhea, and the oropharynx is patent without signs of stridor. His lungs are clear, and he has a regular rhythm on cardiac exam. What diagnostic test is indicated for a definitive diagnosis? A Chest x-ray B Complete blood count (CBC) C Nasal bacterial culture D Soft tissue neck x-ray E CT scan of the neck
*The Correct Answer is: D This patient has a presentation that is consistent with acute epiglottitis. While ensuring that the airway is patent and the patient can maintain the airway, the first step in determining the diagnosis is a soft tissue neck x-ray, to determine inflammation to the epiglottis.* While rare, epiglottitis can be from a bacterial infection, and can be quite serious and sometimes fatal.
A 70-year-old man with a history of pulmonary hypertension and obstructive sleep apnea presents with complaints of increasing dyspnea while walking his dog. He has also recently noted increased lower extremity edema. On physical examination, jugular venous distension is noted. Auscultation of the chest demonstrates a high-pitched blowing diastolic murmur. With inspiration, the murmur increases in intensity and is heard over the second and third left intercostal spaces. An S 3 is appreciated. Palpation of the precordium reveals a hyperdynamic right ventricle, and both a systolic and diastolic thrill. The abdominal exam reveals hepatomegaly and splenomegaly. Based on this patient's history and physical exam findings, which of the following is the most likely finding on echocardiogram? A Tricuspid regurgitation B Aortic stenosis C Atrial septal defect D Pulmonic regurgitation/insufficiency E Mitral stenosis
*The Correct Answer is: D This patient is demonstrating signs and symptoms of right heart failure, and with a history of pulmonary hypertension and a high-pitched diastolic blowing murmur (Graham Steell murmur), Choice D is the most likely of the choices offered.* A blowing holosystolic murmur at the left lower sternal border is characteristic for tricuspid regurgitation, Choice A. Choice B, aortic stenosis, presents with a systolic ejection murmur. An atrial septal defect, Choice C, if large, could present with similar symptoms of exertional dyspnea secondary to a large shunt, but auscultation would reveal a moderately loud systolic ejection murmur that is heard best in the second and third interspaces. This is secondary to increased pulmonary arterial flow. Choice E, mitral stenosis, presents with a diastolic murmur heard best in the left lateral decubitus position, with the bell of the stethoscope at the apex.
A 29-year-old female who is only taking oral contraceptives presents to the emergency department with a 1-day history of worsening shortness of breath. On examination, the patient is afebrile, her pulse rate is 105, respiratory rate is 24, and blood pressure is 122/78. She has wheezing to all lung fields and appears to be in mild to moderate distress. There is no swelling or edema to the lower extremities. Based on these findings, what is the best test to order to determine the diagnosis in this patient? A CT chest B Chest x-ray C Ventilation to perfusion scan D Pulmonary arteriography E MRI chest
*The Correct Answer is: D This patient presents with a history and physical exam that is consistent with an acute pulmonary embolus (PE). The prognosis for this type of illness can be serious, and in some cases death can result. The test that is still the gold standard for determination of a PE is the pulmonary arteriogram.* CT of the chest with contrast tends to be the more utilized testing modality, but the arteriogram remains the test of choice.
A 49-year-old female with a known history of hypertension presents to the emergency department with a generalized headache that is throbbing. She states she had run out of her normal blood pressure medication about a week ago and since then she has noticed that her headache came about and has been getting worse. She denies any nausea, vomiting, visual changes, chest pain, or other symptoms. On exam the patient has a BP 227/120, P 78, R 18. Her HEENT exam is essentially normal, lungs are clear to auscultation, and heart is a regular rhythm without murmur or gallop. Given this clinical situation, which medication would be the most appropriate to address the patients condition? A IV nitroprusside B Oral furosemide C Spironolactone D IV labetalol E Oral hydralazine
*The Correct Answer is: D This patient's clinical situation is one of a hypertensive emergency. In this situation the goal is to bring down the systolic pressure to prevent end organ damage. Given the possible choices, the best choice would be intravenous labetalol (D) due to its effective quick onset, and its ability to be tolerated with most patients.* While oral furosemide (B) and hydralazine (E) can both be effective in managing hypertension, the IV dosing of labetalol would be the better choice. Nitroprusside (A) is no longer a treatment option. Spironolactone (C) would not have strong enough effects to appropriately lower the blood pressure in an efficient manner.
A 88-year-old female is found lying on the ground in a supine position after sustaining a fall in her house from tripping on the rug.. Her leg is shortened, abducted and externally rotated, and she is complaining of pain to the right leg with radiation to the knee. Based on the history of the patient, what type of injury has this patient sustained? A Non-displaced pelvic fracture B Hip dislocation C Femoral shaft fracture D Femoral neck fracture E Distal femur fracture
*The Correct Answer is: D When a fracture occurs in the femoral neck that includes displacement, the leg appears shortened, abducted and externally rotated.* The tension that exists from muscles attached above and below the fracture site results in shortening of leg length after the fracture as the muscles contract. If there is no displacement of the bone, no leg length changes should occur. Generally, hip fractures would not result in a lengthening of the leg, so all answers that include this option would be wrong. *When the leg appears shortened, flexed, adducted, and internally rotated, a posterior hip dislocation should be suspected. A leg that is flexed, abducted and externally rotated suggests an anterior hip dislocation. Posterior hip dislocations are far more common than anterior dislocations, accounting for more than 90% of all hip dislocations.*
Which of the following indicates EKG changes consistent with a suspected pulmonary embolus? A Increased QRS amplitude with tall R waves in limb leads and deep S waves in V1 and V2 B Notched P wave in leads I and II with an increased duration C Prolonged PR interval with peaked T waves D Prolonged QRS duration with QS complex in V1, monophasic R wave in leads I and V6 E Sinus tachycardia with a right ventricular strain pattern, prominent S in lead I, Q wave and inverted T in lead III
*The Correct Answer is: E Approximately 70% of patients with a pulmonary embolus will demonstrate EKG changes. The changes present may range from sinus tachycardia to findings consistent with marked right heart strain and dilatation. The correct answer also includes the classic "S1Q3T3" pattern of changes associated with pulmonary embolism.* The findings in choice A are consistent with left ventricular hypertrophy. Choice B is indicative of left atrial enlargement. Choice C is consistent with hyperkalemia. Choice D indicates left bundle branch block. These changes are not commonly associated with pulmonary embolism.
A fracture involving the medial epicondyle will most likely cause damage to which nerve? A Axillary B Median C Peroneal D Radial E Ulnar
*The Correct Answer is: E Because the ulnar nerve passes through the cubital tunnel, which is a groove on the posterior aspect of the medial epicondyle, any fractures involving the medial epicondyle can also cause damage to the ulnar nerve.* The median nerve is most susceptible to injury at the carpal tunnel. Branches of the radial nerve can become entrapped on the lateral side of the elbow and the associated symptoms are often confused with lateral epicondylitis. Radial nerve injuries are more commonly associated with humeral shaft fractures. The axillary nerve is significantly proximal to the medial epicondyle and the peroneal nerve is in the leg.
A 65-year-old man presents to the emergency department with an acute ischemic stroke. His CT scan is normal. His blood pressure is 180/100 mm Hg. What is the most appropriate treatment for his hypertension? A labetalol (Normodyne) 20 mg IV B nifedipine (Procardia) 10 mg po C nitroprusside (Nipride) drip at 1 mg/kg/min D clonidine (Catapres) 0.1 mg po E no antihypertensive at this time
*The Correct Answer is: E Blood pressure is typically elevated at the time of presentation in acute ischemic stroke. It will decline without medication in the first few hours to days. Aggressively lowering blood pressure in an acute ischemic stroke may decrease the blood flow to the ischemic but salvageable brain tissue. This potentially salvageable brain tissue is referred to as the penumbra. Decreasing blood flow to the ischemic penumbra by acutely lowering blood pressure may result in eventual infarction of this brain tissue.* Treatment of previously undiagnosed hypertension should be deferred for several days. Blood pressure should be treated if there are other indications, such as angina or heart failure. Control of blood pressure is appropriate in patients who are receiving tissue plasminogen activator (t-Pa) for their stroke. Blood pressure should be lowered cautiously to a systolic of less than 185 mm Hg and a diastolic of less than 110 mm Hg. This is thought to decrease the incidence of intracerebral hemorrhage in these patients.
A 70-year-old man, with a history of HTN and aortic valve replacement 3 months ago, presents with complaints of arthralgia, myalgia, anorexia, fatigue, and weight loss over the last month, with recent dyspnea on exertion and lower extremity edema. Vital signs are as follows: Temperature 38°C, BP 102/64, P 98, RR 20. On physical exam, a new high-pitched, blowing, decrescendo diastolic murmur is noted along the left lower sternal border. Two separate blood cultures are positive for S. aureus. Which of the following is the most appropriate next diagnostic study? A EKG B CXR C TTE D ESR E TEE
*The Correct Answer is: E Choice E, TEE or transesophageal echocardiogram, would be most useful in establishing a diagnosis of infective endocarditis, as a positive echocardiogram demonstrating presence of a vegetation would satisfy one of the Duke criteria's major criteria, as well as determine the extent of the prosthetic valvular dysfunction, if present. TEE is more sensitive than TTE, transthoracic echocardiogram, for detecting vegetations, so choice C is incorrect.* Choices A and B, EKG and CXR, should be performed as part of this patient's evaluation, but would be less useful than TEE in establishing a diagnosis of infective endocarditis. Choice D, erythrocyte sedimentation rates, are frequently elevated in patients with endocarditis, but are not specific to the diagnosis of endocarditis.
A 65-year-old recent alcoholic comes to the emergency department with recent onset of dyspnea, with exertion, 3 pillow orthopnea, lower extremity edema, and palpitations, in which he describes his heart as racing. Which of the following is most likely to be the cause of his high-output congestive heart failure? A Mitral regurgitation B Aortic stenosis C Uncontrolled hypertension D Ruptured chordae tendinae E Beriberi
*The Correct Answer is: E Choice E, beriberi, also known as thiamine deficiency, is common among alcoholics, and the only high-output cause of congestive heart failure among the choices offered. *Other causes include severe anemia, thyrotoxicosis, and arteriovenouis shunting (for example, in hemodialysis patients). Choice A, mitral regurgitation, is a cause of excessive preload, leading to heart failure. Choice D, ruptured chordate tendinae associated with mitral regurgitation, would also be a cause of excessive preload, leading to heart failure. Choices B and C, aortic stenosis and uncontrolled hypertension, are causes in which too much afterload leads to heart failure.
A 24-year-old man with a recent history of a viral illness comes to the emergency department complaining of severe left-sided chest discomfort, which radiates through to the left trapezius region. On coming into the room, you note that he is sitting up and hunched forward. On physical examination, the patient's temperature is 39°C, blood pressure is 135/78, with a pulse of 85 bpm, and a pericardial friction rub is noted. Laboratory findings demonstrate elevated serum creatine kinase levels and normal serial troponin levels. His EKG demonstrates peaked T waves. His CXR demonstrates no acute process. Which of the following is the most appropriate treatment for this patient? A Morphine B Enoxaparin C Nitroglycerin D Penicillin V E Indomethacin
*The Correct Answer is: E Choice E, indomethacin 25-75 mg QID, and bed rest would be the most appropriate treatment in a patient with acute viral pericarditis, as a nonsteroidal anti-inflammatory agent will ameliorate the inflammatory process.* Choices A and C are appropriate in a patient suspected of acute coronary syndrome. Choice B, enoxaparin, is contraindicated in patients with pericarditis, as anticoagulants could lead to worsening of pericardial effusion and cardiac tamponade, especially if it is secondary to bleeding into the pericardial space, such as with trauma or postoperatively.
A 22-year-old recent immigrant from Vietnam, who is 28 weeks pregnant with her first child, presents to the emergency department with complaints of worsening dyspnea and lower extremity edema. She is unable to answer definitively whether or not she has a history of rheumatic fever. On physical examination, a possible opening snap, loud S 1 , and a very soft diastolic rumbling murmur is auscultated. When the patient is placed in the left lateral decubitus position, the murmur is accentuated, and heard best at the apex. With inspiration, the murmur does not increase in amplitude. Which of the following is the most likely finding on echocardiogram? A Tricuspid regurgitation B Tricuspid stenosis C Atrial septal defect D Aortic regurgitation/insufficiency E Mitral stenosis
*The Correct Answer is: E Choice E, mitral stenosis, is the most likely finding in this patient, who presents with physical exam findings including a possible opening snap, loud S 1 , and a very soft diastolic rumbling murmur which is heard best at the cardiac apex and accentuated by placing the patient in the left lateral decubitus position. Although rheumatic fever was not positively confirmed, the patient likely did have a history, given that the majority of cases of mitral stenosis are secondary to rheumatic heart disease. Patients from Asia, Central America, and South America are exposed more frequently than their counterparts in more developed countries, where antibiotic use is more common.* Choices A and B, tricuspid regurgitation and tricuspid stenosis, are also linked with patients with rheumatic heart disease. The murmur of tricuspid regurgitation, however, is a systolic murmur, which increases with inspiration and is heard best at the left lower sternal border. Tricuspid stenosis presents with a diastolic murmur, and with inspiration the murmur increases. It, too, is heard best at the left lower sternal border. Choice C, an atrial septal defect, if large, could present with similar symptoms of exertional dyspnea secondary to a large shunt, but auscultation would reveal a moderately loud systolic ejection murmur that is heard best in the second and third interspaces. This is secondary to increased pulmonary arterial flow. Choice D, aortic regurgitation/insufficiency, is also a diastolic murmur; however, it is usually a diastolic decrescendo murmur that is heard best at the left sternal border.
A 50-year-old woman with a history of hypertension complains of chest tightness and dyspnea while walking up one flight of stairs. She recently experienced an episode of near-syncope while walking her dog. She denies a history of rheumatic fever. On auscultation, a crescendo-decrescendo systolic ejection murmur is heard at the upper right sternal border, radiating to the carotids bilaterally. Troponin levels are negative at 0, 3, and 6 hours. Her EKG demonstrates evidence of left ventricular hypertrophy. Given the patient's physical exam findings and recent symptoms, which of the following is the most appropriate next diagnostic study? A Chest X-ray B Transesophageal echocardiogram C Holter monitor D Treadmill exercise stress test E Transthoracic echocardiogram
*The Correct Answer is: E Choice E, transthoracic echocardiogram, is a simple, sensitive, and non-invasive diagnostic tool which can evaluate for the presence of valvulopathy in a patient in this age group, who is likely demonstrating severe aortic stenosis secondary to a congenital bicuspid valve.* Patients with a congenital bicuspid aortic valve typically develop symptoms once the valve leaflets have become calcified and thickened, secondary to the undue stress over many years on a structurally abnormal aortic valve. Choice A might be able to give evidence of cardiomegaly or calcification of heart valves, but would not be sensitive enough to detect the degree of valvulopathy, if present. Choice B, transesophageal echocardiogram, would give information regarding valvulopathy, but is a more invasive test; therefore, choice E is more appropriate. Choice C is a useful diagnostic tool for evaluation of patients complaining of palpitations, but incorrect for this patient, who has no symptoms of palpitations. Choice D, although a useful diagnostic tool for the evaluation of exercise tolerance and in patients complaining of chest pain, does not allow direct visualization of the heart valves to evaluate the degree of aortic stenosis; as the patient is likely demonstrating severe aortic stenosis, cardiac catheterization to evaluate for coronary artery disease prior to surgery will need to be performed.
Approximately what percentage of patients with a solid primary tumor elsewhere will end up with metastatic disease of the vertebrae during the clinical course of their cancer? A 10% B 20% C 30% D 40% E 50%
*The Correct Answer is: E Fifty percent of cancer patients will develop metastatic disease of the vertebrae at some point during the course of their illness. The highest percentages of cancers that lead to such spinal lesions are carcinomas of the breast, lung, prostate, colon, thyroid and kidney.* This likely occurs through hematogenous spread. Pain is a common presenting symptom, but it may be found while still asymptomatic if routine screenings are done with bone scans, MRI or CT in patients with a known primary tumor elsewhere in the body. The pain is usually worse with weight bearing activities and better when lying down, but pain that persists through the night and prevents sleep needs to be evaluated for possible neoplasm. Some metastatic disease of the spine may present with neurological symptoms such as sensory or motor deficits following a spinal nerve root distribution or more generalized neurological deficits due to spinal cord compression or cauda equina syndrome.
An elderly patient is brought in to the emergency department (ED) complaining of incontinence of liquid "like tea water" stool. He is complaining of rectal pressure and lower abdominal pain. The pain is cramping in quality and the patient's abdomen is "bloated." Digital rectal exam reveals hard stool in the rectum. Which of the following should be selected as the initial treatment for this patient? A passing a nasogastric tube B milk of magnesia C opiate analgesics for pain D oral sodium phosphate E manual disimpaction
*The Correct Answer is: E Mechanical bowel obstruction in the rectum does not usually respond to oral laxatives.* A nasogastric tube would not be used for an obstruction in the distal colon/rectum. One would avoid opiates in fecal impactions and other constipation problems because they tend to be more constipating. This patient needs to be disimpacted. Oral agents are unlikely to be effective against the fecal impaction and may cause complications.
A 32-year-old man who is HIV positive has a seizure. On presentation to the emergency department (ED) he is confused and unsure of what happened. His partner reports that he had been complaining of headache in the days preceding the event. CT scanning of the head demonstrates five peripheral contrast-enhancing lesions. What is the most likely diagnosis? A AIDS dementia complex B central nervous system lymphoma C cryptococcal meningitis D progressive multifocal leukoencephalopathy E toxoplasmosis
*The Correct Answer is: E The most common space-occupying CNS lesion in patients with HIV is toxoplasmosis. This condition may present with headache, focal neurologic deficits, seizures, and/or mental status changes.* The typical appearance on brain imaging is that of multiple contrast-enhancing lesions in the periphery, particularly the basal ganglia. CNS lymphoma is more typically a single lesion. AIDS dementia complex presents a diagnosis of exclusion, without a characteristic appearance on imaging. The diagnosis of cryptococcal meningitis is made by examination of the spinal fluid, while PML imaging shows nonenhancing white matter lesions without mass effect.
A 36 year-old woman with no significant past medical history presents with gradual onset of dyspnea and fatigue leading to an episode of "fainting" this morning. Physical exam reveals increased jugular venous pressure, weak carotid pulses, clear lungs, and a loud S2. What is the most likely diagnosis? A Aortic stenosis B Cardiac tamponade C Mitral Regurgitation D Pulmonary fibrosis E Pulmonary hypertension
*The Correct Answer is: E The patients symptoms are due to decreased cardiac output resulting from decreased preload associated with pulmonary hypertension (E).* Aortic stenosis (A) presents more commonly in geriatric patients who present with a murmur. Cardiac tamponade (B) can decrease cardiac output, but would lead to decreased heart sounds. Mitral regurgitation (C) would cause pulmonary edema and rales in conjunction with increased jugular venous pressure. Pulmonary fibrosis (E) is unlikely in this patient with normal lung sounds.
A 46-year-old male is evaluated for a fall from approximately 15 feet, landing on his legs. There is intense pain and swelling to the lower extremity, with decreased sensation. What bone is fractured and is most commonly associated with acute compartment syndrome? A Femur B Patella C Navicular D Talus E Tibia
*The Correct Answer is: E The tibia is the most common bone fracture that leads to acute compartment syndrome. It can also occur after other long bone fractures of the arms and legs.* While it does occur in the hands and feet, it is unlikely that the fracture of a small carpal bone (scaphoid) or tarsal bone (talus) would result in compartment syndrome. Damage to the hyoid bone in the neck and the patella in the knee are not associated with compartment syndrome.
The most commonly fractured long bone in both adults and children is which of the following? A Femur B Fibula C Humerus D Radius E Tibia
*The Correct Answer is: E The tibia is the most commonly fractured long bone in the body for both adults and children. The fractures are often the result of sporting activities in the young and may occur from a simple fall in the elderly - especially those with osteoporosis. Motor vehicle accidents are another common cause of tibial fractures. Open or complex tibial fractures are sometimes associated with compartment syndromes, infection and neurovascular compromise.* The femur is the strongest of the long bones and generally only sustains fractures when exposed to extreme stress, such as that experienced in a motor vehicle collision or industrial accident. Fibular fractures commonly occur with a direct below to the lateral lower leg or with extreme ankle rotational forces or excessive inversion. A high percentage of ankle fractures involve the fibula, especially in older women. Humerus fractures are relatively rare in adults, but are the second most common fractures to occur at birth - behind only the clavicle in frequency. The radius is the most commonly fractured bone in the upper extremity, but still less common in frequency than the tibia. Falls on an outstretched arm are a common mechanism for the injury.
A 52-year-old male with chronic alcoholism is brought to the emergency department by his family, due to his acting differently for several days. A physical exam reveals nystagmus, eye muscle weakness, global confusion, retrograde amnesia, and ataxia. Which of the following is the most likely etiology of the diagnosis? A Cerebrovascular accident B Hypoxemia C Uremia D Vitamin B12 deficiency E Vitamin B1 (thiamine) deficiency
*The Correct Answer is: E This patient exhibits classic symptoms associated with Wernicke encephalopathy. Wernicke encephalopathy is due to vitamin B1 (thiamine) deficiency.* In the United States, this condition occurs most frequently in chronic alcoholics, but it may be seen in any condition affecting thiamine levels. Each of the other etiologies listed may also cause neurologic findings, and should be considered in the differential diagnosis and evaluation of this patient.
A 76-year-old man with a history of HTN and diabetes mellitus, type 2, presents to the emergency department with complaints of palpitations, tachypnea, and chest pain. He denies history of CAD, stroke, TIA, or congestive heart failure. He is afebrile, with vital signs as follows: BP 145/98, HR 138, and RR 22. His EKG is shown (Figure 1). Troponins are negative X 3. His echocardiogram demonstrates normal LV systolic function and normal valvular function. Which of the following questions, when answered, will help to determine the next course of therapy? A How long has the patient been a diabetic? B Has the patient ever had an allergic reaction to aspirin? C Does the patient have a family history of dysrhythmia? D Is the patient a smoker? E How long has the patient been experiencing palpitations?
*The Correct Answer is: E This patient has atrial fibrillation. Choice E is the most important question to ask to determine the next course of therapy. If the patient has been experiencing definitive symptoms for less than 48 hours, direct current cardioversion can be performed, as the risk of atrial clot formation and thromboembolism is low.* Choice A is incorrect because the CHADS2 scoring system does not take into account how long the patient has been diabetic. Choice B is incorrect because aspirin therapy is not utilized. Choice C is incorrect because family history is not a factor in determining the patient's treatment plan. Choice D is incorrect because smoking status is used in calculating Framingham risk score, not a CHADS2 score.
A 1-year-old boy is brought to the emergency department by his parents, who state that the child refuses to walk or crawl and begins crying when they stand him. He seems calm while lying on the examination table. Vitals are as follows: Temp: 38°C, HR: 70, RR: 15. Bruising is noted in several places. His parents deny trauma, but have noticed that he bruises easily. What other physical finding would you expect? A Conjunctival hemorrhages secondary to shaken baby syndrome B Pain response over the wrists secondary to passive range of motion C Pain response with passive range of motion to the hip secondary to slipped epiphysis D Pharyngitis and sand paper rash secondary to a staph infection E Swelling and warmth over the knee secondary to hemarthroses
*The Correct Answer is: E This patient has hemophilia A. Hemarthroses usually occur when an affected child begins to walk. Due to his hemophilia, easy bruising can occur. Hemarthroses can cause low-grade fevers without infection being present*, so choice D is incorrect. Wrist joints are less involved then knees, ankles, and elbows.
A 66-year-old male with a history of hypertension, diabetes mellitus, and hypercholesterolemia presents by emergency medical services (EMS) to the emergency department complaining of severe chest pain with radiation into his back. The patient states that he was feeling well in the morning, but while performing some light activity he felt a "ripping" sensation in his back, which he initially thought was a pulled muscle. The pain continued and the patient started to have chest pain, shortness of breath, and lightheadedness. On initial examination the patient is still in pain, pale, diaphoretic, and has a blood pressure of 85/40. His chest is clear to auscultation, and he has a 3/6 diastolic murmur best appreciated at the base of the heart. Given this clinical scenario, what is the best test to definitively diagnose this medical problem? A Chest x-ray B Transthoracic echocardiography C Transesophageal echocardiography D Cardiac catheterization E Computed tomography
*The Correct Answer is: E This patient is exhibiting a history and physical examination that is consistent with a thoracic aneurysm. The patient's history of hypertension, along with the "ripping" sensation in his back and hypotension give a clinical presentation that is suggestive of a thoracic aneurysm dissection. Given this clinical situation, the best test to evaluate for a potential dissection is by computed tomography (E).* This test is sensitive enough to determine if there is a luminal irregularity. While echocardiography (B and C) may be able to show evidence of an aneurysm, it is not specific enough to show all the areas of an aneurysm. In this situation a chest x-ray (A) would not give enough specificity to appropriately diagnose an aneurysm, as well as a cardiac catheterization (D).
A 68-year-old male presents with the complaint of palpitations in the center of his chest over the last few hours. The symptoms come and go, and last anywhere from 30 seconds to a few minutes. There is some associated lightheadedness with these episodes, and occasionally there is mild shortness of breath. While examining the patient, he has another episode. During this time, the patient's vital signs reveal a pulse of 170 and a blood pressure of 118/69. Based on this history and the findings on the exam and rhythm strip shown, what is the best treatment for this patient? A Diltiazem B Digoxin C Lisinopril D Metoprolol E Amiodarone
*The Correct Answer is: E This patient is having non-sustained ventricular tachycardia (NSVT). Based on the clinical presentation, this patient can benefit from an infusion of amiodarone.* The dose is normally a 150mg bolus, followed by a drip of 1mg/min for the first 6 hours, then followed by 0.5mg/min for the next 18 to 24 hours. The other agents would not be able to maintain a rhythm for this type of abnormality.
An adult male, not previously vaccinated for rabies, presents to the emergency department after being bitten by an aggressive stray dog. The dog was captured, and declared "probably rabid" by a local veterinarian. Which of the following treatment options should you select for this patient? A administer human rabies immune globulin only B administer equine rabies antiserum only C administer human rabies immune globulin and equine rabies antiserum D administer human rabies immune globulin and equine rabies antiserum and human diploid cell rabies vaccine E administer human rabies immune globulin and human diploid cell rabies vaccine
*The Correct Answer is: E Transmission of rabies to this patient must be seriously considered, and postexposure immunization should begin immediately by the administration of human rabies immune globulin (HRIG; 20 IU/kg). About half the HRIG should be infiltrated around the bite wound, and the remainder injected intramuscularly. Human diploid cell rabies vaccine (HDCV) should also be given (1 mL IM in the deltoid), and again on days 3, 7, 14, and 28. HDCV should be delivered in a different syringe and administered at a different site than HRIG.*
A 77-year-old male is admitted to the ICU with community acquired pneumonia and sepsis. There is a concern for the possible development of stress gastritis. Which of the following is an important risk factor in the development of stress gastritis? A Platelets < 150,000 per microliter B INR < 1.5 C Patient remaining NPO for > 24 hours D Hematocrit < 35% E Respiratory failure requiring mechanical ventilation > 48 hours
*The Correct Answer is: E Two of the most important risk factors in the development of stress gastritis are coagulopathy (platelets < 50,000/mcl or INR > 1.5) and respiratory failure, requiring mechanical ventilation for > 48 hours. *Hematocrit and amount of time that a patient receives nothing by mouth are not important risk factors.
In a boxer's fracture, the presence of how much angulation of the fifth metacarpal neck would require referral for reduction as opposed to treatment with a simple ulnar gutter splint? A Less than 10 degrees B 10-19 degrees C 20-29 degrees D 30-39 degrees E Greater than 40 degrees
*The Correct Answer is: E When there is greater than 40 degrees of angulation or an extensor lag (the patient cannot fully extend the affected finger) then a referral is required.* Lesser degrees of angulation without an extensor lag can generally be handled conservatively with an ulnar gutter.
A 56-year-old insulin dependent diabetic has been under your evaluation for his diabetes for several years. The patient has a 3-year history of diabetic neuropathy to the right foot, and may have suffered an injury to the foot without knowing due to loss of sensation. The patient now presents with a tender, reddened, and swollen right foot for the last 10 days that is also warm to the touch. You suspect that this patient may have an acute case of osteomyelitis. Based on this history, what bacterial organism is most commonly the cause of osteomyelitis? A Group A beta-hemolytic streptococci B Hemophilus influenzae C Mycoplasma D Pseudomonas aeruginosa E Staphylococcus aureus
*The Correct Answer is: E * Osteomyelitis is an infection in a bone and can occur in patients of all ages. The most common organism implicated in osteomyelitis across all age groups is Staphylococcus aureus. It can enter the bone through multiple mechanisms including by direct inoculation during an open fracture or during surgical intervention following a fracture (most common mechanisms for adults)* or by hematogenous spread from another source (the usual cause in children). Hemophilus influenzae was a much more common organism in pediatric osteomyelitis in the past, but its prevalence is decreasing due to routine immunizations. Group A beta-hemolytic streptococci is the second most common organism found in osteomyelitis in children, while Pseudomonas aeruginosa is the second most common pathogen in adults. Mycoplasma induced osteomyelitis is relatively rare and usually confined to immunocompromised patients.
Which of the following is the most reliable clinical tool for confirming endotracheal intubation in an emergency situation? A Auscultation over the stomach B Endotracheal tube condensation C Pulse oximetry monitoring D Sellick maneuver E Visualizing the tube passing through the vocal cords
*The Correct Answer is: E* Clinical assessments and practices used to assess tube placement, and help with placement, such as auscultating for breath sounds and noise within the stomach, have not had a confirmation rate comparable to directly visualizing the tube passing through the vocal cords. Tube condensation may occur with esophageal intubation as well. The Sellick maneuver may help with correct positioning, but is not a confirmatory test. Once placement is suspected, confirmation with an end-tidal CO 2 detector and chest x-ray is recommended. Pulse oximetry measurement should be performed throughout the intubation, with decreased saturations representing a worsening clinical condition and/or esophageal intubation.
A 64-year-old female who has a history of injectable drug use presents with blood work that reveals leukocytosis with a left shift, and there is suspicion of osteomyelitis based on the patient's prior history. Based on this history, what bone would be most affected by hematogenous osteomyelitis in adults? A Feet B Long bones C Pelvis D Sternoclavicular bones E Vertebrae
*The Correct Answer is: E* Hematogenous osteomyelitis accounts for about 20% of all cases of osteomyelitis in adults. It is more common in males and the prevalence is higher amongst those who are IV drug abusers, patients being treated with dialysis or who have sickle cell disease. Other conditions which may lead to sepsis (i.e. patients with central lines, urinary infections, and urethral catheterization) increase the risk of hematogenous osteomyelitis. Unlike children, the long bones are rarely affected in adults with the vertebrae being the most likely location for the bone infection to occur. *Lumbar vertebrae are most often affected, followed by thoracic and cervical vertebrae.* Osteomyelitis of the sternoclavicular bones and pelvic bones are not uncommon sites, but these tend to be most frequent amongst IV drug abusers. The feet do not tend to be significantly affected by hematogenous osteomyelitis as frequently as osteomyelitis caused by infected foot ulcers as are often seen in those with diabetes or peripheral vascular disease.
A 66-year-old female with a history of nephrotic syndrome presents to the emergency department complaining of a non-productive cough and dyspnea on exertion following a recent vacation to Orlando. She is currently taking prednisone. What diagnosis is highly suspected in this patient due to her history of nephrotic syndrome? A pneumonia B pulmonary embolism C chronic obstructive pulmonary disease (COPD) exacerbation D Cushing's syndrome E asthma exacerbation
*The correct answer is (B). Patient's with nephrotic syndrome commonly have a hypercoagulable state and are at risk for deep venous thrombosis (DVT) with resultant PE.* Patients on chronic prednisone may have a risk of developing Cushing's syndrome, but this is not consistent with the patient's history. Pneumonia is possible due to chronic prednisone use but not directly caused by her history of nephrotic syndrome. COPD and asthma are not linked to nephrotic syndrome.
You are evaluating a 67-year-old male with known cirrhosis of the liver secondary to alcoholic liver disease, although he has been sober for the past year. He is brought in to the emergency department by his daughter, who notes that for the past few days he has seemed to be more confused. On examination you note the patient to be mildly confused but alert to person and place. He has noticeable asterixis. He is not currently taking any medications and his blood alcohol level is undetectable. What is the treatment of choice in this case based on your physical examination findings? A amoxicillin B prednisone C lactulose D folic acid E thiamine
*The correct answer is (C). * This patient most likely has hepatic encephalopathy due to end-stage liver disease. *Asterixis indicates an increase in serum ammonia. The treatment of choice is lactulose. Both folic acid and thiamine are used in the treatment of alcoholic liver disease, but do not treat elevated ammonia levels.* Antibiotics may be used secondarily in patients nonresponsive to lactulose, but amoxicillin is not preferred. Prednisone is not a treatment for hepatic encephalopathy.
You are doing your usual morning rounds with your supervising physician when you are called to the emergency department with a new admission. In room 7 of the emergency department you note an ill-appearing 72-year-old male with a diagnosis of acute renal failure due to postrenal azotemia. Which of the following would be the most likely cause of his postrenal azotemia? A renal artery stenosis B nonsteroidal anti-inflammatory drug (NSAID) use C massive bleeding D dehydration E bladder outlet obstruction
*The correct answer is (E). Postrenal azotemia is caused by an obstructive process of the bladder, urethra, or ureters. In this case the patient is likely to have bladder outlet obstruction as a cause of his postrenal azotemia.* In older men with benign prostatic hypertrophy (BPH), the patient may present with n/v, abdominal pain, and bladder distension. Relief of the obstruction should relieve the patient's symptoms and potentially the acute renal failure. All other choices are suggestive of prerenal azotemia.
A 21-year-old male presents to the emergency department due to vomiting blood. He is a college student and was initiated into a fraternity earlier in the week, and admits to drinking enough alcohol to make him vomit that night. There was no blood in the vomit at that time. The hematemesis started today, about 20 minutes after he finished a workout at the gym. He notes that today was a particularly tough workout, as he increased the amount of weight he was lifting significantly compared to previous workouts. His vital signs are as follows: heart rate 86 bpm, respirations 14, BP 126/82, temp 98.8°F. What is his most likely diagnosis? A Mallory-Weiss syndrome B Peptic ulcer disease C Arteriovenous malformation D Esophageal varices E Erosive gastritis
*The correct answer is A Mallory-Weiss syndrome. This is characterized by a nonpenetrating mucosal tear at the gastroesophageal junction, which may arise from a sudden increase in transabdominal pressure, such as lifting, retching, or vomiting.* Peptic ulcer disease usually presents with a history of dyspepsia. Arteriovenous malformations usually present with melena or hematochezia. Esophageal varices usually develop secondary to portal hypertension, and are most often found in patients with cirrhosis. Erosive gastritis is often due to NSAIDs, alcohol, or severe medical or surgical illness, and is more often associated with chronic blood loss.
Q 97.4: A 50-year-old woman with a history of hypertension complains of chest tightness and dyspnea while walking up one flight of stairs. She recently experienced an episode of near-syncope. She denies a history or rheumatic fever. On auscultation, a crescendo-decrescendo systolic ejection murmur is heard at the upper right sternal border radiating to the carotids bilaterally. Given the patient's physical exam findings, which of the following is the most likely diagnosis? A Severe aortic stenosis secondary to congenital bicuspid aortic valve B Aortic regurgitation/insufficiency C Mitral stenosis D Aortic regurgitation/insufficiency E Tricuspid regurgitation/insufficiency
A Choice A is correct, as the murmur of aortic stenosis is usually described as a crescendo-decrescendo or systolic ejection murmur heard best at the right upper sternal border. In addition, the murmur of aortic stenosis is frequently transmitted to the carotid arteries. In a patient of this age, with symptoms suggestive of severe aortic stenosis and with these physical exam findings, a congenital bicuspid aortic valve is the best choice, especially as the patient denies a history or rheumatic fever, which is also a cause of developing aortic valve stenosis in individuals under the age of 65. Patients with a congenital bicuspid aortic valve typically develop symptoms once the valve leaflets have become calcified and thickened, secondary to the undue stress over many years on a structurally abnormal aortic valve. Choice B is incorrect, as the murmur of aortic regurgitation is usually described as a high-frequency decrescendo early diastolic murmur heard best at the left upper sternal border or at the right upper sternal border. Choice C is incorrect, as the murmur of mitral stenosis is described as a low-frequency rumbling diastolic murmur that is decrescendo in early diastole, but may become crescendo up to the first heart sound with moderately severe mitral stenosis and sinus rhythm. Choice E is incorrect, as the murmur of tricuspid regurgitation is described as a holosystolic descrescendo murmur. (McPhee et al., 2011, Chapter 10)
Q 98.57: A 75-year-old woman with a history of hypertension is seen for preoperative evaluation prior to bladder suspension. On auscultation, a crescendo-decrescendo systolic ejection murmur is heard at the upper right sternal border, radiating to the carotids bilaterally. Given the patient's physical exam findings, which of the following is the most likely diagnosis? A Aortic stenosis B Aortic regurgitation/insufficiency C Mitral stenosis D Tricuspid regurgitation/insufficiency
A Choice A is correct, as the murmur of aortic stenosis is usually described as a crescendo-decrescendo or systolic ejection murmur that is heard best at the right upper sternal border. In addition, the murmur of aortic stenosis is frequently transmitted to the carotid arteries. Choice B is incorrect, as the murmur of aortic regurgitation is usually described as a high-frequency decrescendo early diastolic murmur that is heard best at the left upper sternal border or at the right upper sternal border. Choice C is incorrect, as the murmur of mitral stenosis is described as a low-frequency rumbling diastolic murmur that is decrescendo in early diastole, but may become crescendo up to the first heart sound, with moderately severe mitral stenosis and sinus rhythm. Choice D is incorrect, as the murmur of tricuspid regurgitation is described as a holosystolic descrescendo murmur.
Q 118.17: A 75-year-old woman with a history of hypertension is seen for preoperative evaluation prior to bladder suspension. On auscultation, a crescendo-decrescendo systolic ejection murmur is heard at the upper right sternal border, radiating to the carotids bilaterally. Given the patient's physical exam findings, which of the following is the most likely diagnosis? A Aortic stenosis B Aortic regurgitation/insufficiency C Mitral stenosis D Tricuspid regurgitation/insufficiency
A Choice A is correct, as the murmur of aortic stenosis is usually described as a crescendo-decrescendo or systolic ejection murmur that is heard best at the right upper sternal border. In addition, the murmur of aortic stenosis is frequently transmitted to the carotid arteries. Choice B is incorrect, as the murmur of aortic regurgitation is usually described as a high-frequency decrescendo early diastolic murmur that is heard best at the left upper sternal border or at the right upper sternal border. Choice C is incorrect, as the murmur of mitral stenosis is described as a low-frequency rumbling diastolic murmur that is decrescendo in early diastole, but may become crescendo up to the first heart sound, with moderately severe mitral stenosis and sinus rhythm. Choice D is incorrect, as the murmur of tricuspid regurgitation is described as a holosystolic descrescendo murmur. (Crawford et al., 2009, Chapter 1)
Q 116.19: You are evaluating a 69-year-old female who complains of an intermittent sensation of hot flashes, flushing of her face/chest, and pruritus after starting a new medication for her cholesterol. Which of the following medications is the most likely cause of her symptoms? A niacin B lovastatin C gemfibrozil D ezetimibe E fenofibrate
A The correct answer is (A). Niacin has a characteristic side effect of hot flashes, flushing, and pruritus. These symptoms can be reduced by addition of ASA or a nonsteroidal anti-inflammatory drug (NSAID) if there are no contraindications. The other choices are unlikely to cause this combination of symptoms. (McPhee and Papadakis, 2011, Chapter 28)
Q 117.14: You are evaluating an 80-year-old female for the first time. She has a history of mild Alzheimer's disease, for which she takes Aricept. She states that she feels fine but her daughter feels she is depressed and has been complaining of not feeling well. Her daughter admits that the patient has a history of primary hyperparathyroidism. What laboratory results would be most consistent with her diagnosis of hyperparathyroidism? A high serum calcium B low intact PTH C low cortisol D low urine calcium E high cortisol
A The correct answer is (A). The hallmark of primary hyperparathyroidism is a high serum calcium and high intact PTH. A low intact PTH is consistent with hypoparathyroidism. The urine serum calcium is usually high in primary hyperparathyroidism. Cortisol is related to endocrine conditions affecting the adrenal cortex. (McPhee and Papadakis, 2011, Chapter 26)
Q 119.6: A 55-year-old female presents to the emergency department with acute tetany and carpal pedal spasms. She has a positive Trousseau's phenomenon on exam. Her calcium and PTH is low. What is the most appropriate treatment at this time? A calcium gluconate IV B thiamine C magnesium oxide D vitamin D E calcium carbonate
A The correct answer is (A). The patient has symptoms of acute hypocalcemia of hypoparathyroidism, with a low calcium and PTH. Acute treatment of hypocalcemic tetany due to hypoparathyroidism is calcium gluconate IV to maintain the calcium between 8-9 mg/dl. Once the patient is stable p.o. calcium such as calcium carbonate and p.o. vitamin D is added. If magnesium is low, it should also be corrected using magnesium sulfate IV. Once corrected, the patient may need to continue supplementation with p.o. magnesium oxide. Thiamine is not indicated at this time for treatment of acute tetany, due to hypoparathyroidism. (McPhee and Papadakis, 2011, Chapter 26)
Q 97.1: A 55-year-old generally healthy male returns from a vacation to Mexico with a 5-day history of nausea, vomiting, fever of 100.5°F, and vague RUQ abdominal discomfort. Which of the following physical examination findings would be most consistent with your suspected diagnosis? A jaundice B ascites C macular diffuse rash D white tonsillar exudate E petechial rash extremities
A The correct answer is (A). The patient's history is most consistent with a diagnosis of hepatitis A. physical examination findings in hepatitis A include jaundice and hepatomegaly. Ascites, choice (B), is likely to occur with chronic hepatitis, not acute hepatitis A. Exudative tonsillitis, choice (D), is more indicative of strep A as a cause, but is not consistent with the history above. A macular or petechial rash, choices (C) and (E) would be unlikely as a direct result of hepatitis A. (Tintinalli, et al., 2010, Chapter 83)
Q 108.6: A 78-year-old Caucasian female has a 3-year history of stiffness and achiness of bilateral shoulders and hips. She has been tested for rheumatoid arthritis in the past and has been found negative. Multiple radiographs of her hips and shoulders are unremarkable. She admits that she was placed on prednisone for an allergic reaction and noted a temporary resolution of her symptoms. For the past two weeks she complains of increasing symptoms now involving her neck and pain in her jaw with chewing. Today she noticed that her scalp is sore when she brushed her hair on the right side. What is the most feared complication of this condition that may be prevented with prompt diagnosis and treatment? A blindness B costovertebral angle (CVA) C aneurysm D arm claudication E polymyalgia rheumatica (PMR)
A The correct answer is (A). This patient has long standing symptoms of PMR with current symptoms suggestive of giant cell (temporal) arteritis. Visual loss is the most feared complication of temporal arteritis, but it can be prevented by prompt initiation of high-dose prednisone. PMR often occurs with or prior to development of temporal arteritis and is not considered a complication. Large vessel involvement--which may result in choices (B), (C), and (D)--is less common than temporal artery involvement in GCA. The patient does not have symptoms of large vessel involvement. (Imboden et al., 2007, Chapter 31), (Fauci, et al, 2008, Chapter 319)
Q 120.10: A generally healthy, 20-year-old male complains of malaise, mild flank discomfort, and cola-colored urine for the past 24 hours. He denies any history of kidney stones. He does admit to having a sore throat and fever about a week ago that has since resolved. His BP is 160/94 P-92 and T is 97.2˚F. His urinalysis is positive for protein, RBCs, leukocytes, and RBC casts. His BUN (blood urea nitrogen) and CR are also elevated. A post-streptococcus glomerulonephritis B infectious mononucleosis C urinary tract infection (UTI) caused by resistant e-coli D pyelonephritis E metabolic syndrome
A The correct answer is (A). This patient's history of sore throat and acute nephritic presentation lead to the most likely diagnosis of post-streptococcus glomerulonephritis . Infectious mononucleosis can cause a sore throat, but is not typically associated with the nephritic presentation above. UTIs and pyelonephritis may present with pyuria and hematuria, but would unlikely cause hypertension and elevated BUN/CR. Pyelonephritis would likely present with fever. Metabolic syndrome is not associated with a sore throat or any of these renal symptoms. (Fauci et al., 2008, Chapter 277)
Q 76.8: A 58-year-old female patient has a history of cirrhosis. She undergoes a diagnostic endoscopy, which shows large varices. Which of the following medications is the most appropriate to reduce the risk of a first variceal hemorrhage? A Beta blocker B ACE inhibitor C Calcium channel blocker D Angiotensin receptor blocker E Alpha blocker
A The correct answer is a beta blocker. This is the recommended medication to reduce the risk of first variceal hemorrhage in patients with large or small varices, who either have variceal red wale marks or advanced cirrhosis. (McPhee SJ, Papadakis MA. Current Medical Diagnosis & Treatment, 2010, p. 541)
Q 30.6: A 22-year-old female presents to the primary care office with a complaint of heartburn. She has symptoms once every few weeks, and the heartburn occurs about 30 minutes after eating. The symptoms are mostly present when eating foods that contain tomato sauce. She has not tried any medication. She denies dysphagia, odynophagia, or weight loss. You are going to recommend a medication that works by inhibiting the proton pump of the gastric parietal cells. Which of the following medications has the mechanism of action listed above? A Omeprazole B Cimetidine C Calcium carbonate D Sucralfate E Magnesium hydroxide
A The correct answer is omeprazole, which is a proton pump inhibitor. Cimetidine is a H2 receptor antagonis, which works by blocking the action of histamine on parietal cells in the stomach. Calcium carbonate and magnesium hydroxide are both antacids, which work by neutralizing the acid in the stomach. Sucralfate works by creating a paste-like material when it comes in contact with stomach acid, and serves as a buffer from the acid.
Q 80.3: Which of the following sets of lab values is most consistent with the diagnosis of Hashimoto's thyroiditis? A High serum TSH, low serum total T4, and high thyroidal peroxidase antibodies B Low serum TRH, low serum TSH, and low serum free T3 levels C Normal TSH, normal serum total T4, and normal radioactive iodine uptake D Low serum TSH, high serum total T4, and high thyroid stimulating antibodies E High serum TSH, high radioactive iodine uptake, and high serum free T3
A The correct choice is A, high serum TSH, low serum total T 4 , and high thyroidal peroxidase antibodies. Hashimoto's thyroiditis is the most common cause of primary hypothyroidism and is autoimmune in nature. The serum thyroid hormone levels are low, secondary to the destruction occurring in the thyroid gland. The negative feedback loop causes the pituitary to respond by increasing production and secretion of TSH. Thyroidal peroxidase, thyroglobulin, and TSH receptor blocking autoantibodies can be found in these patients. Choice B, low serum TRH, low serum TSH, and low serum free T 3 levels, is seen in patients with secondary hypothyroidism relating to pathology in the hypothalamus. Choice C, normal TSH, normal serum total T 4 , and normal radioactive iodine uptake, is seen in patients who are euthyroid. Choice D, low serum TSH, high serum total T 4 , and high thyroid stimulating antibodies, are findings in patients with primary hyperthyroidism (e.g. Graves' disease). Choice E, high serum TSH, high radioactive iodine uptake, and high serum free T 3 can be seen in patients with secondary hyperthyroidism, as a result of anterior pituitary pathology. (Bauer et al., 2010, Chapter 20)
Q 109.9: A patient was prompted to visit his health care provider after his wife started to notice that he was not interested in eating, has lost weight, and has been suffering from nausea for the last few weeks. The practitioner notes hyperpigmentation of the patient's skin, although the patient denies any recent sun or tanning salon exposure. Routine non-fasting blood work reveals the following: Sodium = 130 meq/L Potassium = 5.2 meq/L Chloride = 105 meq/L Glucose = 135 mg/dL Hemoglobin = 13.0 g/dL Hematocrit = 39.0 WBC count = 8,000/mm 3 Which of the following physical exam findings would you expect to see in this patient? A Orthostatic hypotension B Wide, purple striae C Central obesity D Full facial features E Exophthalmous
A The correct choice is A, orthostatic hypotension. The first step in the discussion of this patient is the suspected diagnosis of adrenal insufficiency. Patients with this disorder will have an excess of ACTH, which will act like melanocyte stimulating factor on the skin and cause hyperpigmentation. In adrenal insufficiency, aldosterone is deficient, thereby causing a decrease in sodium retention and potassium excretion. Hypotension is found in approximately 90% of these patients, sometimes associated with syncope as well. Choices B, C, and D are found in patients with cortisol excess. Choice E can be found in patients with Graves' disease. (Aron et al., 2007, Chapter 10)
Q 89.2: A 40-year-old female presents to your office with symptoms of weight gain, hirsuitism, and easy bruising. Past medical and surgical history is noncontributory. She drinks one glass of wine on weekends and does not smoke cigarettes. She takes one multivitamin daily. Upon physical exam, you note facial fullness, central obesity, and thin skin. Which of the following is the most common cause of her symptoms? A Pituitary adenoma B Iatrogenic C C Adrenal micronodular hyperplasia D Adrenocortical adenoma E Ectopic ACTH syndrome
A The correct choice is A, pituitary adenoma. This patient's clinical presentation is typical in Cushing's syndrome. The most common cause of Cushing's syndrome (other than ingestion of oral steroid medications) is Cushing's disease. This disease is caused by a benign, ACTH secreting pituitary adenoma. Choice B, iatrogenic, refers to the ingestion of prescribed (or non-prescribed) oral corticosteroid medications. This is frequently seen in patients requiring long-term oral steroid medications. This patient does not have this type of history. Choice C, adrenal micronodular hyperplasia, and choice D, adrenocortical adenoma, can cause Cushing's syndrome at less frequent incidence. Choice E, ectopic ACTH syndrome, presents more commonly in males with extremely elevated levels of plasma cortical and ACTH. These patients commonly have a positive history of an ectopic source of the ACTH, such as in small cell carcinoma of the lung. (Aron et al., 2007, Chapter 10) (Else et al., 2010, Chapter 21)
Q 85.2: A young man presents with difficulty breathing at times. Upon exam you note evidence of a firm, fixed thyroid nodule with extension toward the trachea and surrounding muscles. The patient has a family history of thyroid cancer. You are concerned that the patient may have medullary thyroid cancer. Which of the following lab tests would you monitor in this patient after treatment? A Serum calcitonin B Plasma alkaline phosphatase C Serum anti-thyroglobulin antibodies D Serum T3 resin uptake E Urine CA-125 levels
A The correct choice is A, serum calcitonin. Both calcitonin and CEA are secreted by medullary thyroid cancer cells, and are used both in diagnosis and monitoring of patients after treatment. Choice B, alkaline phosphatase, is elevated in disorders of the bone and biliary tract. Choice C, serum anti-thyroglobulin antibodies, are most commonly associated with autoimmune disorders of the thyroid, such as Hashimoto's thyroiditis. Choice D, serum T 3 resin uptake, is an indirect test of thyroid function. It measures the amount of unoccupied binding sites for thyroid hormone. Choice E, urine CA-125 levels, are not commonly used for monitoring any disorder. Serum CA-125 has been used to investigate and follow patients with malignancies, such as ovarian cancer. (Greenspan et al., 2007, Chapter 8)
Q 111.7: A 40-year-old female presents to the clinic with symptoms of a 10 pound unintentional weight loss, diarrhea, and palpitations. She has a positive family history for thyroid disease and diabetes mellitus. Her screening TSH level is 0.15mIU/L, and her fasting plasma glucose is 105mg/dL. Which physical exam finding would you expect to find in this patient? A Tremor B Thin hair C Puffy face D Delayed reflexes E Bradycardia
A The correct choice is A, tremor. This question first requires the identification of hyperthyroidism in the patient. The symptoms of weight loss, diarrhea, and palpitations, with a low TSH level, are characteristic of the disorder. Hyperthyroidism also presents most commonly in women in the third and fourth decade, as well as in the elderly. Common physical exam findings of hyperthyroidism include tachycardia, warm moist skin, tremor, goiter, muscle weakness, lid retraction or lag, and gynecomastia. Choices B, thin hair, C, puffy face, D, delayed reflexes and E, bradycardia, are all signs of hypothyroidism. (Jameson et al., 2008, Chapter 335) (Fitzgerald et al., 2011, Chapter 26)
Q 117.19: A 24-year-old man presenting to the clinic 1 week ago was diagnosed with depression and subsequently prescribed 10 mg/day of fluoxetine. He unexpectedly shows up today and states that he is not experiencing any improvement since starting the medication. What is the best treatment option at this time? A double the dose of fluoxetine to 20 mg/day B maintain the current dose of fluoxetine and comfort the patient that the medication may still take at least 1 to 2 more weeks to work C discontinue the fluoxetine and start sertraline D discontinue the fluoxetine and start amitriptyline E maintain the current dose of fluoxetine and add phenelzine to the medication regimen
Alleviation of symptoms associated with depression is typically slow in onset following initiation with SSRIs. Fluoxetine, for instance, can take anywhere between 2 to 6 weeks to achieve substantial benefit when used for depression. After just 1 week of therapy, there is little justification to increase the current dose or switch to another SSRI such as sertraline. Switching the patient to a TCA such as amitriptyline at this point would further delay symptom relief, as TCAs can take several weeks to produce improvement. Compared to SSRIs, TCAs are also more likely to create unwanted side effects such as weight gain, orthostatic hypotension, and constipation. Combining an SSRI with a monoamine oxidase inhibitor (MAOI) such as phenelzine can cause serotonin syndrome that can be lethal. In order to avoid interaction between SSRIs and MAOIs, it is recommended that at least 4 to 5 weeks pass after discontinuing one and starting the other. (Eisendrath and Lichtmacher, 2008, pp. 923-924) Eisendrath SJ , Lichtmacher JE. Psychiatric disorders. In: Tierney LM Jr , McPhee SJ , Papadakis MA, eds. Current Medical Diagnosis & Treatment. 47th ed. New York: McGraw-Hill; 2008.
Q 77.8: A 32-year-old female with history of non-insulin-dependent diabetes mellitus (NIDDM) returns to the clinic for a routine visit. She reports that she her blood sugars have been controlled. However, her vitals today show a blood pressure (BP) of 136/78 P = 72. You note the past three office visits that her blood pressures were in the mid 130s to 140 systolic. She is not currently taking anything for her blood pressure but admits that she has not been exercising as much as usual. What do you advise the patient? A Advise lifestyle modification and continue to monitor BP next visit. B Initiate antihypertensive therapy to target BP < 130/80. C Initiate DASH diet and recheck blood pressure in 6 months. D Monitor blood pressure next visit, and if > 140/90, initiate antihypertensive therapy. E Her blood pressure indicates prehypertension, there is no need for treatment other than exercise at this time.
B The correct answer is (B). According to the JNC-7 guidelines, a patient with diabetes mellitus (DM) or chronic kidney disease (CKD) with this patient's blood pressures should be treated to achieve a goal BP of < 130/80. Although she would be classified as prehypertensive, due to her compelling indication of DM initiation of antihypertensive treatment should begin because she is at higher risk for cardiovascular disease. Lifestyle modifications such as increased exercise, low salt diet, and weight loss are recommended, but in addition to drug therapy at this time due to her increased risk of cardiovascular disease. (McPhee and Papadakis, 2011, Chapter 11)
Q 109.4: Your patient is a 70-year-old female who has a history of non-insulin-dependent diabetes mellitus (NIDDM), hypertension, chronic kidney disease (CKD), and gastroesophageal reflux disease (GERD). You receive a call that her potassium is elevated at 5.8 mEq/L. Last week you treated her for potassium of 6 mEq/L. Her current medications are listed in the following choices. Which of the following medications should you consider discontinuing due to persistent hyperkalemia? A acetaminophen B lisinopril C amlodipine D glyburide E omeprazole
B The correct answer is (B). A potential complication of CKD, usually in the later stages, is hyperkalemia. Hyperkalemia can also be induced by ACE inhibitors such as lisinopril or ARBs and may need to be discontinued. Another antihypertensive that does not cause hyperkalemia may be substituted. The remaining choices are unlikely to cause hyperkalemia. (McPhee and Papadakis, 2011, Chapter 22)
Q 99.3: It is a busy day in the internal medicine office. A 49-year-old noncompliant male with history of non-insulin-dependent diabetes mellitus (NIDDM) and hypertension returns for his 3-month follow-up, now 1 year later. His blood pressure is 156/92, P = 88, BS 250 mg/dl fasting, urine +microalbumin, +glucose, -ketones. He is not taking any medications at this time. You request that he resume his metformin from his last visit, but want to give him something else based on your current findings. What single medication would be best choice for this patient to treat both his blood pressure and microalbuminuria? A atenolol B lisinopril C amlodipine D hydrochlorothiazide (HCTZ) E terazosin
B The correct answer is (B). ACE inhibitors, such as lisinopril, or ARBs are both ideal choices for treating patients with diabetes that have hypertension and microalbuminuria. Although beta blockers (atenolol), calcium channel blockers (amlodipine), and diuretics (HCTZ) are recommended for use in diabetes to control blood pressure, they are not the first choice for patients with chronic kidney disease and will not reduce microalbuminuria. Terazosin (alpha blocker) would not be recommended first-line for a diabetic with hypertension and has no effect on microalbuminuria. (McPhee and Papadakis, 2011, Chapter 11)
Q 95.9: Your patient comes to the office for a follow up of her atrial fibrillation and hypertension. She has noted that she have been more tired than usual. Laboratory findings include a thyroid-stimulating hormone (TSH) < 0.05 mU/L. What medication is the most likely cause of her laboratory findings? A cardizem B amiodarone C warfarin D dotalol E labetalol
B The correct answer is (B). Amiodarone is an antiarrhythmic medication containing iodine that is commonly used in treatment of atrial fibrillation. The use of amiodarone can cause thyrotoxicosis by several mechanisms and may also cause hypothyroidism. In this case the patients suppressed TSH would suggest the presence of amiodarone induced thyrotoxicosis. A high T 3 and FT 4 would support your diagnosis. All the other choices used in the treatment of atrial fibrillation would not cause thyroid dysfunction. (McPhee and Papadakis, 2011, Chapter 26)
Q 6.9: 42-year-old female with history of severe rheumatoid arthritis requiring multiple medications follows up with her primary care physician assistant. She has been doing well on her current medication regimen but has been having some mild headaches. Her blood pressure today is 170/92 P = 82, T = 96.2˚F. She denies any previous history of hypertension. Which of the following medications that she is taking is most likely to be A methotrexate B cyclosporine C acetaminophen D hydrocodone E fexofenadine
B The correct answer is (B). Cyclosporine, an immunosuppressant, is known to cause hypertension as a potential side effect and should be considered as a potential secondary cause of secondary hypertension. Methotrexate, acetaminophen, and fexofenadine are not known to cause secondary hypertension. Hydrocodone may actually cause hypotension as a potential adverse reaction.
Q 97.75: A 34-year-old diabetic female complains of amenorrhea for the past 2 months. Last week she tested positive for strep pharyngitis and is currently being treated. She admits that she has not been using birth control. Her urine HCG is positive. Her current medications are listed in the choices below. Which of the following medications should you discontinue? A amoxicillin B lisinopril C acetaminophen D humalog insulin E methyldopa
B The correct answer is (B). Lisinopril, an ACE inhibitor, is contraindicated in pregnancy due to known problems with fetal toxicity and should be stopped as soon as possible once pregnancy is confirmed. If a patient is planning on becoming pregnant the ACE inhibitor should also be discontinued. ARBs should also be avoided. Choices (A), (C), and (D), and (E) can be used safely in pregnancy and are considered category B. Methyldopa is preferred in the treatment of hypertension in pregnancy and its safety is supported by evidence.
Q 94.1: A 63 -year-old male returns to the office for follow-up complaining of increasing abdominal girth and weight gain of 10 pounds in the past month, despite his usual caloric intake. His vital signs are normal. He continues to drink at least a six-pack of beer daily despite your previous recommendations. On examination you note ascites, palmer erythema, and gynecomastia. The patient finds his increasing abdominal girth to be uncomfortable and is requesting treatment. He admits that his diet is low in salt. What treatment is indicated at this time? A increase in salt intake B spironolactone C hydrochlorothiazide (HCTZ) D large volume paracentesis E transjugular intrahepatic portosystemic shunt (TIPS)
B The correct answer is (B). Spironolactone is the treatment of choice initially and may be combined with furosemide. Salt intake restriction, not increase, is indicated initially. HCTZ is not indicated and will likely be ineffective. Paracentesis and TIPS are treatment options for refractory ascites and due to increased risks are not initial treatments. (McPhee and Papadakis, 2011, Chapter 16)
Q 120.19: A 72-year-old female is a new patient in your office who is not taking any medications other than cranberry pills. She has been healthy most of her life but recently has had several episodes of kidney stones. Furthermore, in the past 6 months she has felt somewhat tired and depressed but can't figure out why. She had a bone density during a health screening and was told she has osteoporosis but is not taking anything for this at this time. She wonders if osteoporosis would cause her joint and bone pains. She has also had some abdominal discomfort and constipation recently, which does not seem to be attributed to what she eats. Her labs in the office today reveal serum calcium of 11.4 mg/dl with normal renal function. What is the most likely cause of her symptoms? A Cushing's syndrome B hyperparathyroidism C hypothyroidism D hypoparathyroidism E hypopituitarism
B The correct answer is (B). The classic symptoms of "bone, stones, abdominal groans, psychotic moans, and fatigue overtones" fits this patient's history. These symptoms, combined with an elevated serum calcium over 10.5 mg/dl, primary indicate that hyperparathyroidism is the most likely diagnosis of those listed. An elevated PTH would confirm the diagnosis. The other choices do not explain all the symptoms and the elevated calcium. (McPhee and Papadakis, 2011, Chapter 26)
Q 104.3: A 78-year-old Caucasian female has a 3-year history of stiffness and achiness of bilateral shoulders and hips. She has been tested for rheumatoid arthritis in the past and has been found negative. Multiple radiographs of her hips and shoulders are unremarkable. She admits that she was placed on prednisone for an allergic reaction and noted a temporary resolution of her symptoms. For the past two weeks she complains of increasing symptoms now involving her neck and pain in her jaw with chewing. Today she noticed that her scalp is sore when she brushed her hair on the right side. What test is the gold standard for diagnosis of this patient's current symptoms? A ESR 60 mm/h B temporal artery biopsy C Elevated CRP D ultrasound of temporal artery E polymyalgia rheumatica (PMR) of temporal artery
B The correct answer is (B). This patient has long standing symptoms of polymyalgia rheumatica (PMR) with current symptoms suggestive of giant cell (temporal) arteritis. Temporal artery biopsy is considered the gold standard for diagnosis of giant cell (temporal) arteritis. Patients with temporal arteritis may have an elevated erythrocyte sedimentation rate (ESR) or CRP, but this is not required for diagnosis. A color ultrasound of the temporal artery will sometimes show edema or stenosis of the affected artery but is not very sensitive for giant cell arteritis. MRA is used for diagnosis of larger arteries with vasculitis and not routinely used in the diagnosis of temporal arteritis. (Imboden et al., 2007, Chapter 31)
Q 66.1: Your patient is asked to see you in followup for his fasting labs. His total cholesterol = 230 mg/dl, triglycerides = 1200 mg/dl, unable to calculate LDL. Your patient should be advised that due to these findings he is at risk for which of the following? A diabetes mellitus B pancreatitis C gout D diabetes insipidus E hypertension
B The correct answer is (B). Very elevated triglycerides, especially over 1000 mg/dl, increase the patient's risk of acute pancreatitis. This patient requires prompt treatment. Hypertriglyceridemia may be a component of metabolic syndrome but does not directly cause diabetes mellitus or hypertension. Hypertriglyceridemia does not increase the risk of gout and diabetes insipidus.
Q 105.10: A 39-year-old male patient, who underwent a kidney transplant four months ago, reports to the clinic complaining of pain on swallowing and difficulty swallowing solids. On physical exam, you note a few white plaques present on his tongue. You order an upper endoscopy, which reveals small, yellow-white plaques with surrounding erythema present along the esophagus. What is your treatment of choice for this patient? A Amphotericin lozenges B Diflucan (fluconazole) C Septra (TMP/SMX) D Nexium (esomeprazole) E Nystatin swish and swallow
B The correct answer is Diflucan (fluconazole). This patient has candidal esophagitis, and the recommended initial therapy is with fluconazole, 100mg/d for 14 to 21 days. Amphotericin lozenges can be used for oropharyngeal candidiasis, but not esophageal. Septra is an antibiotic and will not treat candidal infections. Nexium is a proton pump inhibitor and will not treat candidal infections. *Nystatin can be used for oropharyngeal candidiasis, but not esophageal.* (McPhee SJ, Papadakis MA. Current Medical Diagnosis & Treatment, 2010, p. 535)
Q 108.10: A 64-year-old female presents to the office with a complaint of retrosternal chest pain, painful swallowing, and difficulty swallowing. The symptoms occur about three hours after she takes a daily medication. Which of the following medications is most likely to be the cause of her symptoms? A Lisinopril (Zestril) B Alendronate (Fosamax) C Levothyroxine (Synthroid) D Drospirenone, ethinyl estradiol (Yaz) E Rosuvastatin (Crestor)
B The correct answer is alendronate (fosamax), which is known to cause esophagitis. The other choices are very unlikely to cause pill-induced esophagitis. Lisinopril is an ACE inhibitor that is used to treat hypertension. Levothyroxine is a syntethic T4 and is used to treat hypothyroidism. Drospirenone, ehtinyl estradiol is an oral contraceptive. Rosuvastatin is a statin and is used to treat hypercholesterolemia. (McPhee SJ, Papadakis MA. Current Medical Diagnosis & Treatment, 2010, p. 536)
Q 121.19: A 50-year-old male presents to your office with symptoms of weight gain and easy bruising. He is currently being treated for hypertension. No surgical history is noted and family history is noncontributory. He does not drink alcohol. Upon questioning, he also notes some difficulty with climbing stairs. He has had to use the railing more often than in the past. A photo of the patient is shown below. Which of the following blood hormones would most likely be found to be in excess? A Renin B Cortisol C Testosterone D Antidiuretic hormone E Thyroid stimulating hormone
B The correct choice is B, Cortisol. The reader must first consider Cushing's syndrome as the diagnosis for this patient. The clinical manifestations of Cushing's syndrome are noted in this patient, and include easy bruising, proximal muscle weakness, pinkish-purple wide striae, weight gain, and central obesity. These findings are related to the excess Cortisol present in the plasma of these patients. In addition, hypertension is found in 75 to 85% of patients with Cushing's syndrome. Choices A, rennin, and C, testosterone, are noted to be low or normal in patients with Cushing's syndrome. Choice D, Antidiuretic hormone, is lacking in patients with diabetes insipidus. Choice E, thyroid stimulating hormone, is an anterior pituitary hormone that is essential in thyroid stimulation. (Else et al., 2010, Chapter 21)
Q 118.12: A 40-year-old female presents to the clinic with symptoms of a 10 pound unintentional weight loss, diarrhea, and palpitations. She has a positive family history for thyroid disease and diabetes mellitus. Her screening TSH level is 0.15mIU/L, and her fasting plasma glucose is 105mg/dL. What is the most common cause of this patient's symptoms? A Pituitary tumor B Graves' disease C Thyroid carcinoma D Hashimoto's thyroiditis E Euthyroid sick syndrome
B The correct choice is B, Graves' disease. Graves' disease is an autoimmune disorder, resulting in an increase in synthesis and release of thyroid hormone. It is the most common cause of hyperthyroidism. Choice A, pituitary tumor, and choice C, thyroid carcinoma, are rare causes of hyperthyroidism. Choice D, Hashimoto's thyroiditis, is a common cause of hypothyroidism. Choice E, euthyroid sick syndrome, is a condition of normal thyroid function that occurs after major surgery or an acute illness. This patient is not acutely ill and does have symptoms of true hyperthyroidism. (Fitzgerald et al., 2011, Chapter 26)
Q 115.5: A 60-year-old female patient living in New York City presents for a routine office visit prior to travelling to Europe with her husband for a second honeymoon. Her medical history includes pernicious anemia, for which she is being treated with vitamin B12 supplementation. She uses no other medications. Past surgical history includes an appendectomy at age five. Upon reviewing her symptoms, you find that she has been trying to self treat for chronic constipation, without positive results. She also notes that she is easily tired and has gained 10 pounds in the last few months. On exam you note bradycardia and cool, dry skin. What is the most likely cause of her recent symptoms? A Major depression B Hashimoto's thyroiditis C Hypothalamic destruction D Toxic thyroid nodule E Iodine deficiency
B The correct choice is B, Hashimoto's thyroiditis. This autoimmune disorder is by far the most common cause of hypothyroidism in adults. This patient is presenting with classic signs and symptoms of hypothyroidism including bradycardia, cool, dry skin, and modest weight gain. She also has a history of another autoimmune disorder, pernicious anemia, which is commonly associated with hypothyroidism. Choice A, major depression, seems unlikely since the patient is planning a second honeymoon. Although many of her symptoms can be attributed to depression, her cool, dry skin is not characteristic of the disorder. Choice C, hypothalamic destruction, can cause hypothyroidism, but it is rare. Patients with choice D, toxic thyroid nodule, would typically present with symptoms of hyperthyroidism. Choice E, iodine deficiency, is a cause of hypothyroidism, but it is unlikely in patients living in developed countries with access to a variety of food sources. (Cooper et al., 2007, Chapter 8)
Q 83.5: Which of the following is the most common cause of short stature worldwide? A Acromegaly B Malnutrition C Prader-Willi syndrome D Congenital growth hormone deficiency E IGF-I receptor deficiency
B The correct choice is B, malnutrition. When not associated with chronic diseases, this is the most common cause of short stature worldwide. Children with malnutrition commonly present with failure of weight gain before growth rate decreases. A dietary history is key to the diagnosis, as well as a history of any parasites in the local area. Choice A, acromegaly, is a disorder of growth hormone excess. Choice C, Prader-Willi syndrome, choice D, congenital growth hormone deficiency, and choice E, IGF-I receptor deficiency, have been found to cause short stature, but are not seen as commonly as malnutrition. (Styne et al., 2007, Chapter 7)
Q 110.2: A 25-year-old woman is seen today in your office for vague abdominal pain, nausea, anorexia, weight loss, anxiety, and dizziness. Her past medical history is significant for type 1 diabetes mellitus, and her family history is significant for hypothyroidism in several family members. A review of systems reveal a history of amenorrhea. Upon exam, you note hyperpigmentation of her skin and areas of vitiligo, but no mucocutaneous candidiasis. You are not surprised to find that her serum ACTH level is elevated and her serum cotisol is low. Which of the following syndromes should be investigated in this patient? A Type 1 polyglandular autoimmune syndrome type 1 B Type 2 polyglandular autoimmune syndrome type 2 C Multiple endocrine neoplasia type 2A D Multiple endocrine neoplasia type 1 E Metabolic syndrome
B The correct choice is B, type 2 polyglandular autoimmune syndrome. This patient is presenting with signs and symptoms of adrenal insufficiency including abdominal pain, nausea, anorexia, vomiting, weight loss, anxiety, and hyperpigmentation. Type 2 polyglandular autoimmune syndrome presents most commonly in young women between 20 and 40 years old, with evidence of adrenal insufficiency, type 1 diabetes mellitus, and autoimmune thyroid disease. Because of the strong family history of thyroid disease, it would be prudent to conduct an investigation into this disorder, starting with a serum TSH level. Primary ovarian failure and vitiligo may be symptoms of the autoimmune polyglandular syndrome as well. Choice A, type 1 autoimmune polyglandular syndrome, presents more commonly in childhood with adrenal insufficiency, hypoparathyroidism, and mucocutaneous candidiasis. Choices C and D are inherited disorders, characterized by the development of several different types of endocrine organ neoplasias. Choice E, metabolic syndrome, includes a constellation of several metabolic disorders that increase the risk of cardiovascular disease and diabetes mellitus in the patient. (Fitzgerald et al., 2011, Chapter 26)
Q 77.1: An 84-year-old female is taking prednisone for the past year, secondary to a diagnosis of biopsy-confirmed giant cell arteritis. Her dose was tapered to 20 mg/day, but her symptoms returned several months ago and her dose was increased. In addition she is taking hydrochlorothiazide (HCTZ) for hypertension, but denies any other medications or medical problems. What treatment should you recommend to this patient at this time? A glucophage B simvastatin C calcium and vitamin D D fosamax E naprosyn
C The correct answer is (C). This patient requires chronic prednisone treatment. Common complications of chronic glucocorticoid treatment include osteoporosis and diabetes. At this point with this patient there is no information stating that she has diabetes or osteoporosis. Before initiation of treatment with glucophage, the patient should have a diagnosis of diabetes mellitus. Before initiation of fosamax the patient should have a bone density scan to look for osteopenia/osteoporosis. In the meantime, calcium supplementation with vitamin D is warranted for prevention of osteoporosis. Simvastatin is not indicated, and naprosyn may increase risk of gastrointestinal bleeding with chronic prednisone. (Imboden et al., 2007, Chapter 31)
Q 116.18: You are evaluating a 55-year-old female with a history of hypertension. Her blood pressures have been in the 120s to 130s systolic and 60s to 70s diastolic until recently. She notices some days her blood pressure is normal and other days her systolic readings are in the 150s to 160s and wonders if a medication she could be taking might be contributing to the elevation. Her blood pressure today is 142/66. Which of the following medications is most likely to result in her blood pressure being elevated? A loratadine B simvastatin C pseudoephedrine D acetaminophen E lisinopril
C The correct answer is (B). Decongestants, such as pseudoephedrine, are known to increase blood pressure. Discontinuing pseudoephedrine and rechecking the blood pressure off of this medication may provide further information on the need for additional antihypertensive drug therapy. Loratadine, simvastatin, and acetaminophen are not known to cause secondary hypertension. Lisinopril is an ACE inhibitor used to treat blood pressure. (McPhee and Papadakis, 2011, Chapter 11)
Q 105.3: Your supervising physician asks you to advise him which finding is least likely to be suggestive of a thyroid malignancy in your 49-year-old female with a small palpable thyroid nodule. Which of the following choices would be least likely to suggest malignancy in this patient? A ultrasound showing lesion with microcalcifications B ultrasound showing a lesion of > 1 cm C hot nodule on 123I uptake scan D ultrasound showing a solid lesion E cold nodule on 123I uptake scan
C The correct answer is (C). A hot nodule, which is a hyperfunctioning thyroid nodule, suggests a benign etiology. The other choices, including ultrasound findings of microcalcifications, solid lesions, and lesions > 1 cm, should increase your index of suspicion for possible malignancy. Cold nodules are nonfunctioning thyroid nodules, which should increase your suspicion, especially in combination with suspicious ultrasound and/or clinical examination findings. (Fauci et al., 2008, Chapter 335), (McPhee and Papadakis, 2011, Chapter 26)
Q 99.5: Your patient states she has been gaining weight for no apparent reason over the past year and is concerned that she might have hypothyroidism. What other historical information would support a diagnosis of hypothyroidism? A anxiety B diarrhea C depression D palpitations E heat intolerance
C The correct answer is (C). Depression is a common presenting symptom of hypothyroidism. Weight gain can occur with hypothyroidism. Other symptoms may be weakness, fatigue, and menorrhagia. Hoarseness may also be a presenting symptom. The other choices are common symptoms of hyperthyroidism. (McPhee and Papadakis, 2011, Chapter 26)
Q 117.20: Your patient is a 77-year-old male with a history of hypertension. For about the past 50 years has smoked a pipe daily. He feels great but admits that his cholesterol was elevated for the past 5 years, and has declined treatment. His best friend died of an myocardial infarction last week and the patient now agrees to treatment for his hyperlipidemia. His LDL is 285 mg/dl and HDL is 30 mg/dl. You decide to put him on simvastatin 80 mg QD. Prior to initiation, you advise the patient to notify you about which of the following potential side effects of this medication? A cough B double vision C myalgias D elevated blood pressure E restlessness
C The correct answer is (C). Myalgias are common side effects of statins, which may result in a patient discontinuing the medication. If the patient develops significant myalgias a CPK may be ordered to rule out myositis and if elevated the statin may need to be discontinued. Elevated liver enzymes may occur, which may result in discontinuation of the medication. The other options are unlikely side effects of statins. (McPhee and Papadakis, 2011, Chapter 28)
Q 102.5: Your patient has a history of primary hyperparathyroidism. Recently she has been hospitalized due to obstructing kidney stones. She has had several fractures including her hip, sacrum, and forearm in the past year, all on separate occasions. She is constantly complaining of a lack of energy. What is the recommended treatment for her symptomatic hyperparathyroidism? A surgical removal of the pituitary B high-dose calcium supplementation C parathyroidectomy D thyroidectomy E thiazide diuretics
C The correct answer is (C). Primary hyperparathyroidism is most commonly secondary to a single parathyroid adenoma. The recommended treatment for symptomatic primary hyperparathyroidism is parathyroidectomy. High-dose calcium supplementation and thiazide diuretics, choices (B) and (E), can worsen the hypercalcemia associated with hyperparathyroidism. Neither surgical removal of the pituitary, choice (A), nor or a thyroidectomy, choice (D), is an indicated treatment for this condition. (McPhee and Papadakis, 2011, Chapter 26)
Q 115.2: A patient presents to for a pre-op physical secondary to a right rotator cuff injury. He is a 77-year-old male with a history of chronic obstructive pulmonary disease (COPD), non-insulin-dependent diabetes mellitus (NIDDM), hypertension, and hyperlipidemia. His medications include albuterol MDI, glipizide, candesartan, and simvastatin. A routine EKG is taken. What laboratory abnormality would you expect to find in this patient based on the portion of the EKG revealed below? Source: (Knoop et al., 2010, Chapter 23) A hypernatremia B hypercalcemia C hyperkalemia D hypocalcemia E hypokalemia
C The correct answer is (C). The classic EKG findings in the figure, displaying peaked T waves and widening of the ST segment, are consistent with hyperkalemia. The patient is taking candesartan, an ARB that is a frequent cause of hyperkalemia alone or when combined with other medications that may cause hyperkalemia, such as spironolactone. Flattened T waves, U waves, and ST depression is characteristic of hypokalemia. Prolongation of the QT interval is characteristic of hypocalcemia while shortening of the QT interval is characteristic of hypercalcemia. The patient is not on any medications that would expect to prolong or shorten the QT interval or cause hypocalcemia or hypercalcemia. Changes associated with hyponatremia are not found on an EKG. (Fauci, et al., 2008, Chapter 221)
Q 115.13: The patient below comes to your office for a routine physical exam. She has no complaints. You note the following physical examination findings. There is an absence of tenderness with palpation. What is the most likely diagnosis? Source: (Wolff and Johnson, 2009, Section 15) A gout B cellulitis C hyperlipidemia D rheumatoid arthritis E lacrimal duct obstruction
C The correct answer is (C). The following finding represents xanthelasma, which is commonly associated with hyperlipidemia. In some familial hyperlipidemias, skin eruptions or nodules may form (xanthomas). Gout and rheumatoid arthritis are types of inflammatory arthritis and manifestations are more likely to be found around joints. Gouty tophi may also be found around the helix of the ear and are usually whitish in color in contrast to the yellow xanthelasma. There is no evidence of erythema consistent with cellulitis and the patient is asymptomatic. Lacrimal duct obstruction would likely be symptomatic and there would be tender swelling over the lacrimal duct.
Q 121.5: A 41-year-old female comes in for her annual exam. She generally feels well and has no complaints other than some general fatigue. On examination you note a normal size thyroid with a palpable firm 1.5 cm nodule in the left lobe of her thyroid. What is the best test to order that will provide a definitive diagnosis? A thyroid ultrasound B thyroid uptake scan C fine needle biopsy of nodule D MRI thyroid E TSH
C The correct answer is (C). The patient has a solitary, > 1cm, firm nodule, which is suspicious for a thyroid malignancy. A fine needle biopsy of the nodule would provide you with a definitive diagnosis. A thyroid ultrasound, thyroid uptake scan, and TSH would be helpful in determining the etiology of the nodule but will not provide you with a definitive diagnosis. Since this patient has a suspicious clinical examination a biopsy is indicated. MRI of the thyroid is not a preferred test. (McPhee and Papadakis, 2011, Chapter 26)
Q 80.13: 42-year-old female complains of weight gain (especially in her abdomen) over the past 8 months. She also has noted that her skin bruises easily. Her husband has noted she seems to be very moody lately and she is worried about their relationship. Furthermore, her hair seems to be getting thinner and she is now getting acne like she had in her teenage years. She wonders if this is due to menopause since her periods have stopped suddenly about a year ago. On physical examination her BP = 170/50, P = 82, T = 98.2˚F. You note the following findings on examination (see picture). What diagnostic test is indicated initially to confirm your suspected diagnosis? A cosyntropin stimulation test B MRI pituitary C dexamethasone suppression test D 24-hour urine for protein E serum protein electrophoresis
C The correct answer is (C). The patient's symptoms are consistent with a diagnosis of Cushing's syndrome (or disease). Her physical examination findings of hypertension and abdominal obesity with the classic purple striae also support the diagnosis. The initial diagnostic test of choice would be the dexamethasone suppression test. If the test is positive, further confirmatory testing is done which would also help to identify the cause. An MRI of the pituitary is appropriate if further testing suggests the possibility of a pituitary adenoma as the cause of the Cushing's syndrome, but is not used as an initial diagnostic test for Cushing's disease. A cosyntropin stimulation test, choice (A), is indicated for the diagnosis of Addison's disease. (McPhee and Papadakis, 2011, Chapter 26) Source: (McPhee and Papadakis, 2011, Chapter 26)
Q 63.3: You are evaluating a 67-year-old male with known cirrhosis of the liver secondary to alcoholic liver disease, although he has been sober for the past year. He is brought in to the emergency department by his daughter, who notes that for the past few days he has seemed to be more confused. On examination you note the patient to be mildly confused but alert to person and place. He has noticeable asterixis. He is not currently taking any medications and his blood alcohol level is undetectable. What is the treatment of choice in this case based on your physical examination findings? A amoxicillin B prednisone C lactulose D folic acid E thiamine
C The correct answer is (C). This patient most likely has hepatic encephalopathy due to end-stage liver disease. Asterixis indicates an increase in serum ammonia. The treatment of choice is lactulose. Both folic acid and thiamine are used in the treatment of alcoholic liver disease, but do not treat elevated ammonia levels. Antibiotics may be used secondarily in patients nonresponsive to lactulose, but amoxicillin is not preferred. Prednisone is not a treatment for hepatic encephalopathy.
Q 119.3: A 42-year-old male presents with a three year history of progressive dysphagia, weight loss, and nocturnal aspiration. An upper GI study (barium swallow) shows a moderately dilated esophagus with a smooth, tapered, distal "bird's beak" deformity. What is the most likely diagnosis? A Diffuse esophageal spasm B Barrett's esophagus C Achalasia D Gastroesophageal reflux disease E Esophagitis
C The correct answer is achalasia. Diffuse esophageal spasm typically presents with dysphagia and chest pain, and has a corkscrew or nutcracker appearance on barium swallow. Barrett's esophagus does not provoke particular symptoms and does not have a characteristic appearance on a barium swallow. Gastroesophageal reflux typically presents with heartburn and regurgitation, and a barium swallow is not used to diagnose this condition. Esophagitis presents with odynophagia, dysphagia, and chest pain, and a barium swallow is not used to diagnose this condition. (McPhee SJ, Papadakis MA. Current Medical Diagnosis & Treatment, 2010, p. 541)
Q 92.6: A 64-year-old caucasian male presents to the office with a complaint of worsening difficulty swallowing over the last two months. He also has a long history of gastroesophageal reflux disease. He is now becoming concerned because he has lost 15 pounds in the last two weeks. What is the most likely diagnosis? A Leiomyosarcoma B Esophageal stricture C Adenocarcinoma D Achalasia E Squamous cell carcinoma
C The correct answer is adenocarcinoma. Adenocarcinoma is more common in caucasians, and the majority develop as a complication of Barret metaplase, due to chronic gastroesophageal reflux. Leiomyosarcoma is a rare smooth muscle malignancy. Esophageal stricture would present with the gradual development of solid food dysphagia over many months to years. Achalasia is a motility disorder, in which there is loss of peristalsis in the distal two-thirds of the esophagus and impaired relaxation of the lower esophageal sphincter. Squamous cell carcinoma is more common in African Americans, and is associated with chronic alcohol and tobacco use. (McPhee SJ, Papadakis MA. Current Medical Diagnosis & Treatment, 2010, p. 1468)
Q 83.7: A 52-year-old male presents to the office with a complaint of intermittent dysphagia. His symptoms occur mostly when he eats steak, and have been ongoing for about six months. The symptoms have not worsened and he denies weight loss. You are concerned that he may have an esophageal web or ring. The diagnostic test that best visualizes an esophageal web or ring is which of the following? A Endoscopy B Esophageal manometry C Barium esophagram D Esophageal pH monitoring E CT scan of the abdomen
C The correct answer is barium esophagram, which provides the best visualization of an esopahgeal web or ring. Endoscopy and CT scan of the abdomen are less sensitive than barium esophagography. Esophageal manometry is used to measure and record esophageal pressures. pH monitoring is used to assess for reflux into the esophagus. (McPhee SJ, Papadakis MA. Current Medical Diagnosis & Treatment, 2010, p. 538)
Q 9.10: A 60-year-old female presents for a routine physical exam. She has not seen a health care provider in over 30 years. She has no significant past medical history and is taking no medications. Her surgical history includes only carpal tunnel repair five years ago. Upon review of symptoms, she notes feeling somewhat tired. She attributes that to old age. Vital signs include BP 110/55, pulse 55 bpm, and respirations 16 per minute. Upon exam you note an enlarged, firm thyroid, thickening of her skin, and puffy facial features and pallor. A portion of blood tests that you ordered is shown below: Serum TSH = 4.4 uIU/mL Hemoglobin = 10.0 g/dL Hematocrit = 30% MCV = 101 Fasting plasma glucose = 105 mg/dL BUN = 10 mg/dL Creatinie = 0.6 ng/mL Which of the following is the most appropriate intervention? A Lithium carbonate 300 mg PO twice daily B Resection of the anterior pituitary C Levothyroxin 50 to 100 ug PO daily D Ferrous sulfate 325 mg PO three times daily E Radioactive iodine protocol
C The correct choice is C, levothyroxin 50 to 100 ug PO daily. This patient has classic signs and symptoms of hypothyroidism and required thyroid hormone supplementation. The most common form of hypothyroidism is primary hypothyroidism (e.g. Hashimoto's thyroiditis), and the most common thyroid hormone supplementation is levothyroxine. Choice A, lithium carbonate 300 mg PO twice daily, is a medication used in patients with psychiatric disorders and is known to cause hypothyroidism. Choice B, resection of the anterior pituitary, is not indicated without evidence of a tumor or other pituitary pathology. Secondary hypothyroidism related to the anterior pituitary is quite rare. Choice D, ferrous sulfate 325 mg PO three times daily, is a common treatment protocol for patients with iron deficiency anemia. This patient's MCV is elevated, indicating large red blood cells, as seen in disorders such as vitamin B12 deficiency or folic acid deficiency. Patients with iron deficiency anemia present with microcytic hypochromic anemia. Choice E, radioactive iodine protocol, is a treatment used in patients with hyperthyroidism.
Q 99.6: A patient describes an insidious onset of vague gastrointestinal symptoms including nausea, vomiting, and abdominal pain. She has been losing weight over the last few weeks as well. Past medical history reveals a new diagnosis of anemia. The patient is not currently taking any medications. Physical exam findings include hypotension and dark freckles with dark, bluish black pigmentation of the mucosal membranes. Serum sodium is decreased, while the potassium and plasma ACTH levels are elevated. What is the most likely diagnosis for this patient? A Acute abdomen B Cushing's syndrome C Primary adrenal insufficiency D Disseminated intravascular coagulation E Diabetic ketoacidosis
C The correct choice is C, primary adrenal insufficiency. This disorder presents with symptoms resulting from progressive destruction of the adrenal glands and resultant decrease in aldosterone and cortisol secretion. The presentation can range from subtle to fulminant adrenal crisis. Common clinical findings include hyperpigmentation, weakness, anorexia, nausea, vomiting, diarrhea, and hypotension. Some patients with adrenal insufficiency may initially be treated for an acute abdomen, choice A. In this patient, ACTH levels are known to be elevated, pointing the cause to the pituitary-adrenal axis. Choice B, Cushing's syndrome, presents with signs and symptoms related to an excess in plasma cortisol, such as truncal obesity, hyperglycemia, hypertension, skin changes, and weight gain. Choice D, disseminated intravascular coagulation, is a coagulation disorder presenting with bleeding and thrombosis. Choice E, diabetic ketoacidosis, is an acute complication seen in patients with type 1 diabetes mellitus. (Williams, Harrison's Online, Chapter 336)
Q 75.3: What is the definitive treatment for the majority of patients presenting with mild symptoms of hyperthyroidism secondary to subacute thyroiditis? A Subtotal thyroidectomy B Oral methimazole C Symptomatic treatment D Radioactive iodine E Antibiotics
C The correct choice is C, symptomatic treatment. Most patients with subacute thyroiditis and symptoms of hyperthyroidism require only symptomatic treatment, with non-steroidal anti-inflammatory medications and/or beta blockers, for any cardiac symptoms including palpitations and tachycardia. Occasionally, patients may require a course of prednisone for this acute inflammatory condition. Most patients will recover spontaneously within a few months. Choices A, B, D, and E are not necessary in this condition. Most cases of subacute thyroiditis are associated with viral infections, and resolve without additional thyroid medications. (Cooper et al., 2007, Chapter 8)
Q 114.9: A 37-year-old woman presents with symptoms of weight gain, weakness, and poor concentration. Her past medical history includes a vertebral fracture two years ago. Vital signs include the following: BP = 145/102, pulse = 80, and respirations = 16. A physical exam reveals truncal obesity, hirsuitism, and thin skin. Laboratory results include an elevated 11 p.m. salivary cortisol and lack of suppression with an overnight dexamethasone suppression test. Gadolinium enhanced MRI of the pituitary reveals a microadenoma. What is the treatment of choice for this patient? A Adrenal radiation therapy B Bilateral adrenalectomy C Transsphenoidal resection D Chemotherapy regimen E Glucocorticoid replacement
C The correct choice is C, transsphenoidal resection of the pituitary microadenoma. This is the treatment of choice for this patient. This patient has developed Cushing's syndrome secondary to an ACTH secreting pituitary adenoma (Cushing's disease). This more commonly occurs in women who are between 20 and 40 years old. It is the most common cause of ACTH dependent Cushing's syndrome. The remission rate for patients with pituitary microadenomas is approximately 80%. Since the pathology lies in the pituitary gland, choice A, adrenal radiation, and choice B, bilateral adrenalectomy, would not address the cause. Cushing's syndrome is not treated with chemotherapy, choice D. Choice E, glucocorticoid replacement, would not be advised as this patient already has an elevated level of plasma cortisol. (Melmed et al., 2008, Chapter 333)
Q 83.10: A patient presents with symptoms of polyuria for several months. There is no history of diabetes mellitus or intrinsic kidney disease. Which of the following tests can best determine whether the polyuria is resulting from primary polydipsia or some form of diabetes insipidus? A 24-hour urine volume test B Routine urinalysis C Water deprivation test D Urine culture and sensitivity E Vasopression suppression test
C The correct choice is C, water deprivation test. This test helps the practitioner determine if the patient can concentrate his/her urine with or without becoming hyperosmolar. The patient is restricted from liquids and food during the test, which must be done in a controlled environment. A patient with primary polydipsia will be able to concentrate his/her urine without becoming hyperosmolar. A patient with diabetes insipidus will become hyperosmolar without concentrating the urine. Choice A, 24-hour urine volume test, can provide quantification of the polyuria, but will not determine its cause. Choice B, routine urinalysis, may provide evidence of dilute urine, but will not provide evidence of its cause. Choice D, urine culture and sensitivity, is used when trying to determine the cause of a urinary tract infection. Choice E, vasopression suppression test, is not a test that is performed. It could be dangerous to try to suppress a hormone that is already thought to be deficient. (Robinson et al., 2007, Chapter 6)
A 50-year-old woman with a history of hypertension complains of chest tightness and dyspnea while walking up one flight of stairs. She recently experienced an episode of near-syncope. She denies a history or rheumatic fever. On auscultation, a crescendo-decrescendo systolic ejection murmur is heard at the upper right sternal border radiating to the carotids bilaterally. Given the patient's physical exam findings, which of the following is the most likely diagnosis? A Severe aortic stenosis secondary to congenital bicuspid aortic valve B Aortic regurgitation/insufficiency C Mitral stenosis D Aortic regurgitation/insufficiency E Tricuspid regurgitation/insufficiency
Choice A is correct, as the murmur of aortic stenosis is usually described as a crescendo-decrescendo or systolic ejection murmur heard best at the right upper sternal border. In addition, the murmur of aortic stenosis is frequently transmitted to the carotid arteries. *In a patient of this age, with symptoms suggestive of severe aortic stenosis and with these physical exam findings, a congenital bicuspid aortic valve is the best choice, especially as the patient denies a history or rheumatic fever, which is also a cause of developing aortic valve stenosis in individuals under the age of 65. Patients with a congenital bicuspid aortic valve typically develop symptoms once the valve leaflets have become calcified and thickened, secondary to the undue stress over many years on a structurally abnormal aortic valve.* Choice B is incorrect, as the murmur of aortic regurgitation is usually described as a high-frequency decrescendo early diastolic murmur heard best at the left upper sternal border or at the right upper sternal border. Choice C is incorrect, as the murmur of mitral stenosis is described as a low-frequency rumbling diastolic murmur that is decrescendo in early diastole, but may become crescendo up to the first heart sound with moderately severe mitral stenosis and sinus rhythm. Choice E is incorrect, as the murmur of tricuspid regurgitation is described as a holosystolic descrescendo murmur.
A 55-year-old woman with a history of mitral stenosis, secondary to rheumatic heart disease, presents to the emergency department with increasing dyspnea while walking up one flight of stairs. She denies chest pain and discomfort, but states that recently she has also noticed palpitations. She also admits to lower extremity edema, which is new within the last week. Which of the following tachyarrhythmias is she most likely to demonstrate on EKG? A Ventricular tachycardia B Atrial flutter C Ventricular fibrillation D Ventricular bigeminy E Torsades de pointes
Choice B is correct. As patients with mitral stenosis age, and their mitral stenosis progresses to moderate or moderately severe mitral stenosis (most commonly after their fourth decade), the incidence of atrial arrhythmias—including premature atrial contractions, paroxysmal tachycardia, atrial flutter, and atrial fibrillation—increases. Choices A, C, D, and E are less likely, given that they are ventricular arrhythmias.
A 76-year-old man with a history of HTN and diabetes mellitus, type 2, presents to the emergency department with complaints of palpitations, tachypnea, and chest pain. He denies a history of CAD, stroke, TIA, or congestive heart failure. He is afebrile, with vital signs as follows: BP 145/98, HR 138, and RR 22. His EKG is shown (Figure 1). Troponins are negative X 1. His echocardiogram demonstrates normal LV systolic function and normal valvular function. Based on his electrocardiogram, which of the following is his most likely diagnosis? A Normal sinus rhythm at 65 bpm with premature atrial complexes B Atrial fibrillation with a rapid ventricular rate of approximately 140 bpm C Atrial flutter with 2:1 conduction D Sinus tachycardia at a rate of approximately 120 bpm E Ventricular fibrillation
Choice B is correct. This patient's EKG demonstrates atrial fibrillation, with disorganized atrial activity, as evidenced by the lack of discrete p waves and irregularly irregular ventricular activity. Choice A, normal sinus rhythm with PACs, would demonstrate a somewhat irregular rhythm, but discrete p waves would be noted. Choice C, atrial flutter, would be characterized by a regular ventricular rate with a sawtooth morphology. Choice D, sinus tachycardia, would demonstrate a regular, though fast, ventricular rate with discrete p waves. Choice E, ventricular fibrillation, would be an irregularly irregular rhythm; however, QRS complexes would not be clearly defined.
A 55-year-old woman with a history of emphysema, who is undergoing chemotherapy for lung cancer, comes to the emergency room complaining of a sudden increase in dyspnea, with exertion and fatigue. On physical exam, hypotension, pulsus paradoxus, and muffled heart sounds are noted. On transthoracic echocardiography, cardiac tamponade is noted with over 200 mL of pericardial fluid described. Which of the following is the most appropriate next step in management? A Enoxaparin 1 mg/kg subcutaneously Q 12 H B EKG C Cardiac catheterization D Penicillin V 500 mg PO BID x 10 days E Emergent pericardiocentesis
Choice E, emergent pericardiocentesis, should be considered when patients exhibit symptoms suggestive of severe cardiac tamponade, such as described above, with confirmatory echocardiographic findings of a large pericardial effusion, as this can be fatal if not treated promptly. Choice A is inappropriate, as this could lead to worsening of cardiac tamponade if it is secondary to bleeding into the pericardial space, such as with trauma or postoperatively. Choice B could provide additional confirmation of the diagnosis if a reduction in amplitude of QRS complexes is demonstrated, and electrical alternans of the P, QRS, and T waves, but would not provide any therapeutic benefit. Choice C could be the next appropriate step in management of myocardial infarction, but not in cardiac tamponade. Choice D represents appropriate therapy for patients with acute rheumatic fever, but not for patients with cardiac tamponade.
Q 115.11: A patient with long-standing, untreated acromegaly is seen in your office with symptoms of severe headaches. After completing a thorough history and physical exam, you order a set of x-rays including a skull series. Which of the following findings would you expect in this patient? A Punched out lesions B Basilar skull fracture C Metastatic bone lesions D Enlarged sella tursica E Thinning of the skull
D *The correct choice is D, enlarged sella tursica. This finding is seen in 90% of patients with acromegaly. Other findings on skull radiographs include thickened calvarium (upper portion of the skull), enlarged mandible, and sinuses.* Bony growth is a hallmark of the disease. The pituitary adenoma, which typically causes the disease, can be found in the sella tursica. This disorder doesn't typically metastasize, and is not associated with metastatic bone cancer, as noted in choice C. Choice A, punched out lesions, are commonly associated with Paget's disease. There is no history of head trauma, as would be the case in choice B, basilar skull fracture. As noted earlier, the skull may be thickened. Therefore, choice E, thinning of the skull, would not fit this patient's presentation. (Aron et al., 2007, Chapter 7)
Q 121.7: An 88-year-old female patient has been advised by her primary care physican that she needs a computed tomography (CT) scan of her abdomen and pelvis due to persistent abdominal pain, bloating, and weight loss. She was told that she needs to hold one of her medications the day of the procedure and that she may resume the medication 48 hours later. She can't remember which medication she should discontinue. Which medication listed below should she discontinue temporarily as specified above due to the diagnostic test ordered? A glyburide B glipizide C pioglytizone D metformin E acarbose
D The correct answer is (D). A CT of the abdomen and pelvis requires p.o. and IV iodinated contrast unless ordered specifically without IV contrast. There is an increased risk of acute renal failure with IV iodinated contrast. The risk to the patient may increase with metformin and therefore should ideally be held prior to and for 48 hours after any radiocontrast IV study to avoid the added possibility of lactic acidosis. The other treatments for non-insulin-dependent diabetes mellitus (NIDDM) do not require discontinuation with IV contrast studies for these time periods. However, the am dose of a sulfonylurea (glyburide or glipizide) may be held until after the study that day when the patient resumes eating to avoid hypoglycemia in some patients. (McPhee and Papadakis, 2011, Chapter 27)
Q 121.2: A 44-year-old female complains of nonproductive cough for the past 6 months. She denies rhinorrhea, wheezing, dyspnea, chest pain, or hemoptysis. Her medical problems include hypertension. Medications include benazepril 10 QD, Amlodipine 5 mg QD, and HCTZ 25 mg QD. She is a nonsmoker and denies any foreign travel. The following CXR PA/LAT is taken. What is the most likely cause of her cough? Source: (Hanley and Welsh, 2003, Chapter 3) A amlodipine B pneumonia C bronchiectasis D benazepril E asthmatic bronchitis
D The correct answer is (D). ACE inhibitors such as benazepril have a potential adverse reaction of a chronic cough. Discontinuing the ACE inhibitor is appropriate in this case while substituting this for another antihypertensive. The CXR is normal and there are no findings suggestive of pneumonia, bronchiectasis, or asthmatic bronchitis. A chronic cough is not a significant side effect of amlodipine. (McPhee and Papadakis, 2011, Chapter 11)
Q 105.2: Your patient returns to your office for a follow up for non-insulin-dependent diabetes mellitus (NIDDM). Her HgA1c in the office is 6.4%. She is concerned about developing kidney disease from her diabetes and requests that you test her for this. What initial screening test should you order that would provide clues to potential diabetic nephropathy allowing for treatment to slow the disease progression? A 24-hour urine for protein B serum BUN/CR C urine microscopic D urine microalbumin E serum protein
D The correct answer is (D). An easy office dipstick or laboratory test for urine microalbumin should be done initially and periodically on diabetic patients who are at risk for diabetic nephropathy. Treatment should be initiated if microalbuminuria is found to slow disease progression. A urine microscopic for renal casts may be helpful if the patient has symptoms of kidney disease, but is not an initial screening test. Serum BUN/CR and GFR are useful tests for patients with known diabetic nephropathy to indicate the stage of chronic renal failure but is not elevated early in the disease progression, before urine microalbumin. A 24-hour protein is not indicated in this case as an initial screening test. (McPhee and Papadakis, 2011, Chapter 22)
Q 116.8: Your patient is a 44-year-old female who was advised by her endocrinologist that she has thyroid cancer and seeks your opinion. You inquire about the type of thyroid cancer but the patient is unsure. She does state that she was advised that it is the most common and she has a good prognosis. What is the most likely cause of her thyroid malignancy? A lymphoma B follicular thyroid carcinoma C anaplastic thyroid carcinoma D papillary thyroid carcinoma E medullary thyroid carcinoma
D The correct answer is (D). Papillary thyroid carcinoma is the most common and least aggressive of the thyroid carcinomas. The usual age of presentation is in the early 40s. This is followed in frequency by follicular thyroid carcinoma. Anaplastic thyroid cancer is very aggressive and has a high mortality rate. Lymphoma affecting the thyroid can occur, but is not common, and usually presents in the elderly. (McPhee and Papadakis, 2011, Chapter 26)
Q 105.16: A 42-year-old male comes to the office to discuss his total cholesterol of 215 mg/dl, which was obtained last week at a health fair. He is generally healthy, but smokes one pack of cigarettes per day for the past 10 years. He is unsure of his family history. What is your next step? A prescribe simvastatin B advise dietary changes C repeat the total cholesterol today, fasting D obtain a fasting lipid profile E prescribe gemfibrozil
D The correct answer is (D). The patient comes in only with total cholesterol. With this information a fasting lipid profile is indicated to evaluate the patient's cardiovascular risk and should be the next step. Repeating the total cholesterol is not very helpful. It is premature to recommend a specific treatment at this point until you fully assess the patient's risk. (McPhee and Papadakis, 2011, Chapter 28)
Q 118.13: A 55 -year-old male returns for a routine follow up. He has a history of hypothyroidism and is taking levothyroxine 100 mcg QD for the past 2 years. He has not had a TSH since his dose was changed 2 years ago. He states that he has felt very nervous and tired in the past 3 months. He has lost 6 pounds but has not changed his diet and states he is always hungry. His TSH is 0.5 mU/L. His FT 4 is elevated. What treatment would you recommend at this time? A increase levothyroxine to 125 mcg QD B change to methimazole C change to propylthiouracil D decrease levothyroxine to 50 mcg E Radioa
D The correct answer is (D). The patient has symptoms of hyperthyroidism but has a history of hypothyroidism. He is currently taking thyroid replacement. The most likely cause of his low TSH is too much thyroid replacement. A reasonable treatment is to decrease the dose of his levothyroxine. Increasing the dose would likely cause worsening symptoms of hyperthyroidism. Although the patient has symptoms of hyperthyroidism and a suppressed TSH treatment for hyperthyroidism, choices (B), (C), and (E) are not recommended and can be dangerous. (McPhee and Papadakis, 2011, Chapter 26)
Q 118.5: A 63-year-old female complains of a 5-day history of a persistent left-sided headache, which she has not experienced before. She also notes a tender swollen area around her left temple, which appeared around the same time. On examination you note tenderness and prominence of the left temporal artery. You order an ESR, which is 75 mm/h. What is your best course of action at this time? A Repeat the ESR in 72 hours. B Begin prednisone 20 mg/d and increase if symptoms persist. C Refer to a rheumatologist for appointment next month, with a trial of nonsteroidal anti-inflammatory drugs (NSAIDs). D Begin prednisone 60 mg/d immediately. E Refer for a temporal artery biopsy next week, with a trial of hydrocodone for analgesia.
D The correct answer is (D). The patient's history and physical examination findings point to giant cell arteritis (temporal arteritis) as the most likely cause, prompting immediate treatment with high-dose prednisone to prevent visual loss. The patient meets the criteria for clinical diagnosis of giant cell arteritis without a temporal artery biopsy, but it is recommended for definitive diagnosis due to the complications associated with long-term corticosteroid treatment. Treatment with prednisone should not be withheld while waiting for a temporal artery biopsy. NSAIDs and hydrocodone do not prevent the complications of temporal arteritis. (Imboden et al., 2007, Chapter 31)
Q 113.9: You are examining n 42-year-old male with the following physical examination findings (see picture below). What symptoms would most likely correspond to his diagnosis? A anxiety B palpitations C excessive snoring D polyuria and polydipsia E weight gain
D The correct answer is (D). The patient's physical examination findings suggest acanthosis nigricans, a condition associated with diabetes mellitus. This patient would most likely present with polydipsia and polyuria at diagnosis. He may have weight loss and fatigue. Anxiety and palpitaions are not as likely to present symptoms of diabetes. (McPhee and Papadakis, 2011, Chapter 27)
Q 79.7: A 39-year-old male complains of intermittent episodes of hematuria over the past 6 months, which resolved spontaneously without treatment. He also has noted some dull bilateral flank discomfort recently that he attributed to a strain after moving. He denies any dysuria or penile discharge. He further denies any history of kidney stones. His mother died in her 60s of kidney disease that required dialysis. On examination his BP is 148/92, P = 68, T = 98.2˚F. His examination is unremarkable except for bilateral palpable enlarged kidneys. Urine dip is positive for +1protein. What diagnostic test would be most helpful to confirm your suspected diagnosis? A complete blood count (CBC) with diff B acute abdominal series C 24-hour urine protein D renal ultrasound E kidney-ureter-bladder (KUB)
D The correct answer is (D). This patient's history and physical examination findings are suggestive of polycystic kidney disease. His family history also suggests the potential for polycystic kidney disease, which is a common hereditary disease that may lead to end-stage renal disease (ESRD). A renal ultrasound is the preferred test to confirm the diagnosis revealing multiple renal cysts. Radiographs and labs, choices (A) and (C), can help in your differential diagnosis but are not diagnostic of polycystic kidney disease. (Fauci et al., 2008, Chapter 278)
Q 108.4: A 52-year-old male undergoes an upper endoscopy for evaluation of GERD, and is diagnosed with Barrett's esophagus. There is no evidence of dysplasia at this time. In addition to placing him on a PPI, the most appropriate management includes which of the following? A Balloon dilation of the lower esophageal sphincter B Barium swallow every two years C Nissen fundoplication D Endoscopy every three years E Esophagectomy
D The correct answer is endoscopy every three years. Balloon dilation of the lower esophageal sphincter is performed for achalasia. A barium swallow will show motility disorders, but cannot be used to detect dysplasia. Nissen fundoplication is a surgical treatment used in gastroesophageal reflux disease. Esophagectomy is used for patients with high grade dysplasia or adenocarcinoma of the esophagus. (McPhee SJ, Papadakis MA. Current Medical Diagnosis & Treatment, 2010, p. 532)
Q 79.3: A 32-year-old male presents with odynophagia, dysphagia, and chest pain. His past medical history consists of him being HIV positive. He is currently not taking any medications, as he cannot afford to pay for them. An endoscopy is ordered and the results show several large, deep ulcerations. Initial treatment consists of which of the following medications? A Nystatin suspension B Acyclovir C Famciclovir D Ganciclovir E Fluconazole
D The correct answer is ganciclovir. This patient has cytomegalovirus esophagitis, and the initial treatment is ganciclovir 5 mg/kg IV every 12 hours for 3 to 6 weeks. Nystatin suspension is used to treat oropharyngeal candidiasis. Acyclovir and famciclovir are used to treat herpetic esophagitis. Fluconazole is used to treat candidal esophagitis.
Q 24.6: A 65-year-old female presents to the primary care office with epigastric pain and nausea. Which of the following is a spiral gram-negative rod that resides beneath the gastric mucosa layer and causes gastric mucosal inflammation, with polymorphonuclear cells and lymphocytes? A Treponema pallidum B Borrelia burgdorferi C Corynebacterium diphtheriae D Helicobacter pylori E Giardia lamblia
D The correct answer is helicobacter pylori. Treponema pallidum is a spirochete that is capable of infection in almost any organ or tissue in the body, and causes protean clinical manifestations. It is most commonly associated with syphilis. Borrelia burgdorferi is a spirochete and is associated with Lyme disease. Corynebacterium diphtheriae is a bacterium that infects the respiratory tract and is associated with diphtheria. Giardia lamblia is a protozoal infection of the upper small intestine and is associated with giardiasis.
Q 6.2: An adult patient was recently diagnosed with type 2 diabetes mellitus. She met with her diabetic educator to discuss suggested changes to her diet and exercise regimens. What percent of her total daily calories should be in the form of carbohydrates? A 5% to 15% B 20% to 25% C 35% to 40% D 45% to 65% E 70% to 80%
D The correct choice is D, 45% to 65%. The American Diabetes Association recommends a diet with 45% to 65% carbohydrates, 25% to 35% fats, and 10% to 35% protein.
Q 76.10: A 30-year-old female presents to your office for a routine physical exam. She has not seen a health care provider in many years. Upon talking with the patient, you find out that she had been diagnosed with hypertension several years ago, but was unable to afford the antihypertensive medications that were prescribed to her. She has no complaints at this time. Upon exam of the head and neck, you note widened spaces between her lower incisor teeth and a large, fleshy nose. Her skin is oily and she demonstrates mild proximal muscle weakness. Her EKG reveals a left axis deviation and widened QRS. What is the most likely rationale for her clinical presentation? A Diabetes mellitus B Cushing's syndrome C Hypothyroidism D Acromegaly E Clinical depression
D The correct choice is D, acromegaly. Patients with acromegaly have an abundance of growth hormone secretion. This leads to excessive growth of many areas of the body including soft tissue. Patients with acromegaly also have an increased incidence of hypertension and left ventricular hypertrophy. None of the other choices will cause this patient's constellation of symptoms. Patients with many endocrine disorders may develop weaknesses as seen in this patient, but the large nose and widely spaced teeth are characteristic of acromegaly. (Melmed et al., 2008, Chapter 333)
Q 112.2: Which of the following is the most common cause of primary adrenal insufficiency in the United States? A Tuberculosis B Adrenal hemorrhage C Lymphoma D Autoimmune destruction E Metastatic carcinoma
D The correct choice is D, autoimmune destruction. This is responsible for 80% of cases of primary adrenal insufficiency in the United States. All of the other choices can cause adrenal insufficiency, but they are less common. Tuberculosis, choice A, is a common cause of adrenal insufficiency in other areas of the world, where the infection is more common. Bilateral adrenal hemorrhage, choice B, can occur as a complication of sepsis, heparin use, anti-phospholipid syndrome, and after major trauma or surgery. Lymphoma, choice C, and metastatic carcinoma, choice E, are rare causes of adrenal insufficiency. (Fitzgerald et al., 2011, Chapter 26)
Q 83.2: To which of the following areas does follicular thyroid cancer most commonly first spread? A Intraglandular metastasis B Local spread into regional vocal cords C Distant lymph nodes D Bone and lung via bloodstream E Local extension into the muscle and trachea
D The correct choice is D, bone and lung via bloodstream. Follicular thyroid cancer can spread to regional lymph nodes and distant sites via the blood stream. Choice A, intraglandular metastasis, is seen more commonly in papillary thyroid carcinoma, and choice B, local spread into regional vocal cords, occurs in anaplastic thyroid carcinoma. Choice C, distant lymph nodes, and choice E, local extension into the muscle and trachea, are more commonly seen in patients with medullary thyroid cancer. (Bauer et al., 2010, Chapter 20) (Lee et al., 2008, Chapter 41)
Q 97.78: A woman presents for a routine post-partum checkup four weeks after delivery of her child. She is currently breast feeding without difficulty. She tells you that she has been feeling anxious and very warm, despite the change of season into winter. She is happy as a mother and has not had problems caring for her baby. What is the next best step in the investigation of her symptoms? A Radioactive iodine uptake test B Serum total T4 level C Thyroid fine needle aspiration D Serum TSH level E MRI of the anterior pituitary
D The correct choice is D, serum TSH level. This patient is presenting with post-partum thyroiditis. Thyroid dysfunction occurs in 2 to 5% of women after giving birth. It can recur with subsequent pregnancies and develop into long-term thyroid disease as well. Typically, women with this disorder first develop signs and symptoms of hyperthyroidism, which then later changes to hypothyroidism. Most symptoms resolve spontaneously within a few months. In the hyperthyroid stage of this disorder, blood tests will reveal a suppressed TSH level with elevated serum thyroid hormone levels, as is common in all forms of primary hyperthyroidism. Choice A, a radioactive iodine uptake test, would reveal little or no uptake, but this test should not be ordered in a woman who is breastfeeding. For choice B, a serum total T 4 level can be elevated, but is not as sensitive or specific as TSH as a screening test. Any protein status changes in the woman can cause an elevated total T 4 without any thyroid dysfunction. Choice C, a thyroid fine needle aspiration, can be performed, but is not likely necessary and would not be the best next step for this patient. Choice E, an MRI of the anterior pituitary, would only be suggested if there is a suspicion of a pituitary tumor. This is a rare cause of hyperthyroidism and would not be the best next step.
Q 116.12: What is the most sensitive test available for the screening and detection of early thyroid dysfunction? A Radioactive iodine uptake B Serum T3 resin uptake C Serum total T4 level D Serum TSH level E Thyroid scan
D The correct choice is D, serum TSH level. Very small changes in serum TSH level can provide clues that there are changes in the functioning of the hypothalamic-anterior pituitary-thyroid axis. The test is easier and less expensive than any thyroid imaging tests. All of the other choices can be used in the work up of patients for thyroid dysfunction, but they are less sensitive, and many are more expensive and more invasive. (Bauer et al., 2010, Chapter 20)
Q 29.8: A patient is being treated for hypothyroidism. His condition has been stable for the past year. What blood test should be ordered and monitored yearly in this patient? A Total T4 B T3 resin uptake C Thyroid releasing hormone D Thyroid stimulating hormone E Free T3
D The correct choice is D, thyroid stimulating hormone. This test will help to monitor patient adherence with thyroid hormone supplementation, as well as to fine tune the dose so that the TSH remains within the reference range. Choices A, B, and E can be used in the work up of patients for primary hypothyroidism, but alone each test is not helpful to monitor chronic disease in patients. Choice C, serum thyroid releasing hormone, is used more commonly when investigating secondary hypothyroidism.
Q 121.14: A 62-year-old African-American male is seen for his yearly physical exam. He has no complaints. He denies any current medications or medical problems, but the occupational medicine nurse has taken his blood pressure several times in the past year and told him it was high. He denies any tobacco or alcohol use. His blood pressure is 156/92 today. What is the most likely cause of his elevated blood pressure? A sleep apnea B primary aldosteronism C pheochromocytoma D renal artery stenosis E essential hypertension
E The correct answer is (E). The most common cause of hypertension (HTN) is essential, also known as primary hypertension, making up about 95% of patients with hypertension. Secondary causes of hypertension are less common and include sleep apnea, primary aldosteronism, pheochromocytoma, and renal artery stenosis. There is no reason to suspect these other causes in this patient, who is otherwise healthy with stage 1 hypertension based on the stated history. Sutters Michael, "Chapter 11. Systemic Hypertension" (Chapter). McPhee SJ, Papadakis MA: CURRENT Medical Diagnosis & Treatment 2011: http://www.accessmedicine.com/content.aspx?aID=3177080.
Q 115.7: A 72-year-old male presents to you for a routine physical exam. He complains of increasing abdominal distention, weight loss, and night sweats over the past 2 months. He admits to having a blood transfusion as a young adult. His labs include +anti-HCV and HCV RNA. Which of the following is the most likely explanation for his history and laboratory findings? A acute hepatitis B B alcoholic hepatitis C chronic hepatitis A D Gilbert's syndrome E hepatocellular carcinoma
E The correct answer is (E). The patient's symptoms are consistent with a possible malignancy. Labs indicate chronic hepatitis C, likely longstanding with liver cirrhosis, which is associated with an increased risk of hepatocellular carcinoma. Chronic hepatitis B is also associated with an increased risk of hepatocellular carcinoma, but not acute hepatitis B, choice (A). Choices (B), (C), and (D) are not consistent with the laboratory results and do not increase the patient's risk of hepatocellular carcinoma. (Fauci et al., 2008, Chapter 298), (Greenberger et al, 2009, Chapter 49)
Q 110.3: You are called for a consult of a 30-year-old female who appears well, but is noted to have hepatomegaly, spider nevi, and elevated serum amino transaminases greater than 1000 units/L. Because of your suspected diagnosis, you decide to give her a trial of corticosteroids resulting in improvement of her serum amino transaminases. What is the most likely diagnosis? A hepatitis A B hepatitis B C hepatitis C D hepatocellular carcinoma E autoimmune hepatitis
E The correct answer is (E). The symptoms indicate chronic hepatitis. Autoimmune hepatitis is generally the only hepatitis of the choices that responds to corticosteroids. Patients with autoimmune hepatitis may develop cirrhosis and may be at higher risk for hepatocellular carcinoma, but the history is more consistent with autoimmune hepatitis. A liver biopsy is indicated. The patient likely has a positive antinuclear antibody (ANA). (McPhee and Papadakis, 2011, Chapter 16)
Q 80.7: An 84-year-old female complains of intermittent dyspnea and dizziness over the past 4 months. She is found to have an irregular rhythm on exam. Her EKG findings are below. Based on the EKG findings, which single laboratory test may lead to finding a secondary cause? Source: (Tintinalli, et al., 2010, Chapter 22) A B12 level B folic acid C lipid profile D erythrocyte sedimentation rate (ESR) E thyroid-stimulating hormone (TSH)
E The correct answer is (E). This patient has atrial fibrillation, which can occur in patients with hyperthyroidism, Therefore, it is important to check a TSH in patients who present with atrial fibrillation, especially in the elderly. Choices (A), (B), (C), and (D) are not secondary causes of atrial fibrillation. (McPhee and Papadakis, 2011, Chapter 26)
Q 104.7: Your patient is a 47-year-old female who complains of leg cramps and fatigue over the past few weeks. Her examination is completely normal. She is taking an unknown medication for hypertension, which she did not bring with her. Labs include a normal complete blood count (CBC) and a BMP that reveals a potassium level of 3.2 m Eq/L, otherwise normal. Which of the following is the most likely cause of her laboratory abnormalities? A quinapril B labetalol C verapamil D valsartan E hydrochlorothiazide (HCTZ)
E The correct answer is (E). This patient's symptoms are likely due to hypokalemia, which is a potential side effect of thiazide diuretics such as hydrochlorothiazide. Hyponatremia may also be another possible side effect. Choice (A), an ACE inhibitor, and choice (D), an ARB, can both cause hyperkalemia. Choice (B), a beta blocker, and choice (C), a calcium channel blocker, do not cause hypokalemia but may be associated with bradycardia. (McPhee and Papadakis, 2011, Chapter 11)
Q 104.9: This is a protrusion of pharyngeal mucosa that develops at the phayngoesophageal junction, between the inferior pharyngeal constrictor and the cricopharyngeus. It often presents in older patients, with symptoms of dysphagia and regurgitation that develop slowly over many years. Late in the disease process patients may note halitosis, spontaneous regurgitation of undigested food, or nocturnal choking. Which of the following choices is the correct term for this condition? A Meckel diverticulum B Hiatal hernia C Esophageal web D Achalasia E Zenker diverticulum
E The correct answer is Zenker diverticulum. Meckel's diverticulum is a pouch on the wall of the lower part of the small bowel, which is congenital. A hiatal hernia is a protrusion of a portion of the stomach into the chest through a hole in the diaphragm. An esophageal web is a thin, diaphragm-like membrane of squamous mucosa. Achalasia is a motility disorder, characterized by loss of peristalsis in the distal two-thirds of the esophagus and impaired relaxation of the LES. (McPhee SJ, Papadakis MA. Current Medical Diagnosis & Treatment, 2010, p. 538)
Q 26.7: A 34-year-old male presents to the primary care office with a complaint of heartburn. He has symptoms two to three times a week, and it occurs about 30 minutes after eating. He has tried over-the-counter antacids, and they were helping relieve his symptoms for a few months, but they are not working well now. In discussing lifestyle modifications with this patient, to help reduce his symptoms, which of the following is recommended? A Eating larger meals B Increasing the intake of acidic foods C Lying down within 30 minutes after eating D Increasing the intake of fatty foods E Elevating the head of the bed six inches
E The correct answer is elevating the head of the bed six inches. All of the other choices are known to exacerbate reflux symptoms.
Q 86.1: What is the radionuclide imaging pattern noted during a thyroid scan in patients with subacute thyroiditis? A Single area of increased uptake B Diffusely high uptake C Multiple areas of increased uptake D Single area of low uptake E Diffusely low uptake
E The correct choice is E, diffusely low uptake. Acute inflammation is occurring during subacute thyroiditis, causing leakage of stored thyroid hormone into the circulation. The thyroid is not metabolically active, and therefore there is low radioactive iodine uptake on scan. This is in distinction to the diffuse increased uptake seen on scan in Graves' disease, choice B. Choice A, a single area of increased uptake, is commonly seen with a "hot" or toxic nodule. Choice C, multiple areas of increased uptake, are found in patients with multinodular goiters. This patient had no nodules noted on exam. Choice D, single area of low uptake, is seen as cold nodules and may be related to a benign nodule or thyroid cancer. (Cooper et al., 2007, Chapter 8)
Q 95.4: A 47-year-old man presents with signs and symptoms of Cushing's syndrome. He has no history of depression, alcoholism, anorexia, or medication use. His past medical history is negative for any known malignancies. An MRI of the pituitary gland is not definitive for any masses. You are trying to decide if the symptoms are caused by an occult pituitary adenoma or occult malignancy in another area of the body. Which of the following is the definitive test used to differentiate pituitary from non-pituitary ACTH dependent Cushing's syndrome? A 12:00 a.m. plasma cortisol B Random plasma ACTH C 24-hour urine free cortisol D MRI of the adrenal glands E Inferior petrosal sinus ACTH
E The correct choice is E, inferior petrosal sinus ACTH. This test can definitively indicate whether or not an elevated plasma ACTH level is due to secretion from the pituitary or an ectopic source. Simultaneous measurements of ACTH from the peripheral circulation and the inferior petrosal sinus after CRH stimulation can help to locate the surge's origin. Highly accurate results can be obtained if this test is performed by a skilled interventional radiologist. Although the other choices listed are used in the evaluation of patients with hypercortisolemia, they do not detect any related ACTH surge source. (Aron et al., 2007, Chapter 10)
Q 98.39: A patient is recovering from having a total thyroidectomy two days ago for medullary thyroid cancer. An extensive neck dissection was required during the surgery. Post-operative lab testing reveals a low serum calcium level. Which of the following clinical presentations will most likely occur in this patient? A Constipation B Anorexia C Polyuria D Bone pain E Paresthesias
E The correct choice is E, paresthesias. Circumoral paresthesias are signs of hypocalcemia. Hypocalcemia can occur after any type of neck surgery that may have resulted in destruction of the parathyroid glands. Choices A through D are symptoms of hypercalcemia and may be seen in hyperparathyroidism.
Q 88.6: What is the definitive treatment of choice for elderly patients diagnosed with Graves' disease? A Beta blocking agents B Levothyroxine C Methimazole D Total thyroidectomy E Radioactive iodine
E The correct choice is E, radioactive iodine. This is the treatment of choice in the elderly because it is efficient, easy to take, and inexpensive. Choice A, beta blocking agents, are useful in the treatment of symptoms of hyperthyroidism, such as palpitations, but they are not a definitive treatment for the disorder. Choice B, levothyroxine, is used for thyroid hormone supplementation in patients with hypothyroidism. Choice C, methimazole, is an anti-thyroid drug that has increased toxicity in the elderly and is more useful in younger patients with mild hyperthyroidism. Choice D, total thyroidectomy, has a limited role as a treatment for hyperthyroidism, and is associated with increased morbidity in the elderly. (Greenspan et al., 2007, Chapter 24)
A 52-year-old male has a 200-pound file cabinet fall on his right leg. He comes to the emergency department complaining of pain and swelling to the right leg. He is also complaining of parasthesias to the leg also. Based on these findings on history, what part of the leg would be the most likely site of compartment syndrome? A Anterior compartment of the leg B Deep posterior compartment of the leg C Dorsal compartment of the forearm D Lateral compartment of the leg E Superficial posterior compartment of the leg
Q 141
Q 39.23: A 12 year old female found a tick on her leg after a camping trip. The tick was removed without incident. According to CDC recommendations, what is the appropriate testing for Lyme disease? A ELISA, if + or equivocal, follow up with Western Blot B Western Blot, if + or equivocal, follow up with ELISA C Cbc with diff and skin biopsy D Skin biopsy and gram stain
The CDC recommends first using ELISA to test for Lyme disease. If this is positive, then a Western Blot should be performed as confirmation. Acute and convalescent titers should be tested as only 20-30% of patients have a positive response in the acute phase. That percentage rises to 70-80% in the convalescent phase.
Q 77.4: During a hospitalization for acute exacerbation of COPD, troponin levels are drawn on a 62-year-old man with a history of hypertension, hyperlipidemia, and chronic tobacco use, and found to be elevated above the 99 th percentile of normal. Which of the following choices would qualify this patient for the most recent ACC/AHA consensus guideline's definition of myocardial infarction? A Ischemic symptoms B New right bundle branch-block on EKG C J wave on EKG D Pulmonary vascular congestion on CXR E Elevated WBC count
The Correct Answer is: A Choice A is the most appropriate choice, as troponin elevation may occur in the setting of patients who do not suffer from acute coronary syndrome. Therefore, the 2007 consensus guidelines recommended that the definition of myocardial infarction be applied to those patients who not only had troponin elevation above the 99 th percentile, but also met one of the following criteria: "ischemic symptoms, new left bundle branch block (not right bundle branch-block as in choice B), new ST and T-wave changes, new Q waves, or imaging evidence of a new loss of viable myocardium or new regional wall-motion abnormality." Choice C, J wave, is characteristic of patients with hypothermia. Choice D, pulmonary vascular congestion, is frequently noted on CXR of patients with congestive heart failure. Choice E, an elevated WBC count, is indicative of an infectious process. (Tintinalli et al., 2011, Chapter 52)
Q 77.10: What range of time does it take for a PPD test to become positive as an immune response? A 2 to 10 weeks B 12 to 20 weeks C 21 to 40 weeks D >40 weeks E Immediately after exposure
The Correct Answer is: A The typical timeframe is shortly after the immune system can register the new strain into the system. False positives can occur in individuals who have had prior infections. (Chesnutt MS, Prendergast TJ. Current Medical Diagnosis and Treatment, 2011, Chapter 9, Pulmonary Disorders)
Q 98.51: A 32-year-old lactating female presents to the surgical clinic with a fluctuant mass of her left breast. The area directly above the lesion is erythematous and tender to touch. You make the diagnosis of localized breast abscess. Which of the following pathogens is the most likely cause of the patient's symptoms? A Staphylococcus aureus B Viridens streptococcus C Pseudomonas aeruginosa D Escherichia coli
The Correct Answer is: A A breast abscess is commonly seen in conjunction with lactation. The most common bacterial pathogen is Staphylococcus aureus. While antibiotics are the treatment of choice, the patient should be instructed to continue breast feeding in order to promote drainage of the breast. If the abscess does not respond to antibiotics, then an aspiration should be considered.
Q 7.5: A 63 year-old woman complains of fatigue, loss of appetite, a sore-red tongue, paresthesias of her feet and hands, and unsteadiness of her gait. Which of the following tests is be used to confirm the patient's suspected diagnosis? A Anti-intrinsic factor antibodies B Antiparietal cell antibodies C Coomb's test D Schilling Test E Serum folate levels
The Correct Answer is: A A diagnosis of pernicious anemia (PA) can be supported through the presence of anti-intrinsic factor antibodies. Antiparietal (B) cell antibodies aren't associated with the development of PA. The Coomb's test (C) is used in the evaluation of hemolytic anemias. Serum folate (D) levels are beneficial in evaluating macrocytic anemias, but will not establish a diagnosis of PA. The Schilling test (D) was once commonly used to diagnosis PA, but is no longer available due to lack of available radiolabeled human intrinsic factor.
Q 113.10: A patient is being evaluated for fatigue, weight loss, a two-week cough, and erythema nodosum. Today, the patient noted eye symptoms and your exam reveals iritis. A chest xray reveals hilar and paratracheal lymphadenopathy. You suspect sarcoidosis. Which of the following is the most appropriate next step of evaluation to determine the diagnosis? A Bronchoscopy with tissue biopsies B CT chest with contrast C MRI chest D PET Scan E Serum ACE level
The Correct Answer is: A A diagnosis of sarcoidosis must incorporate clinical findings and radiologic findings, ruling out other conditions and obtaining definitive information. A biopsy should be performed, and may be performed utilizing tissue from any affected organ, including skin or from a transbronchial biopsy, which has a high conclusive yield. Histologically, sarcoidosis is associated with noncaseating granulomas; however, other granulomatous disease must be ruled out. The additional testing listed may also be utilized for further evaluation, but is considered adjunctive (see Figure 322-8 for more information). (Fauci et al., Harrison's Principles of Internal Medicine, 17e, Chapter 322, Sarcoidosis) Proposed approach to management of patient with possible sarcoidosis. Presence of one or more of these features supports the diagnosis of sarcoidosis: uveitis, optic neuritis, hypercalcemia, hypercalciuria, seventh cranial nerve paralysis, diabetes insipidus.
Q 109.17: A chest x-ray on an 81-year-old male with a four-day history of productive cough, dyspnea, fever, chills, and shortness of breath reveals a left sided pleural effusion. Pleural fluid analysis reveals a decreased glucose level, elevated lactate dehydrogenase, and 20,000 polymorphonuclear white blood cells/mcL. What is the most likely cause of this effusion? A Bacterial pneumonia B Congestive heart failure C Malignancy D Pulmonary embolus E Tuberculosis
The Correct Answer is: A A pleural effusion is a collection of fluid within the pleural space, due to an increased rate of fluid formation with decreased absorption. Pleural effusions are classified as transudative versus exudative, based on the underlying cause. This patient exhibits pleural fluid analysis results that are consistent with a parapneumonic effusion. Malignancy and tuberculosis also cause exudative effusions, with different fluid analysis results and various patient presentations (see Table 9-23). Congestive heart failure and pulmonary embolism are associated with transudative effusions. (McPhee SJ, Papadakis MA. Current Medical Diagnosis & Treatment 2011, Chapter 9, Pulmonary Disorders)
Q 108.1: A known patient who carries the diagnosis of hemophilia A wants to know what medications they can use to relieve the symptoms of the flu. Which of the following is the most appropriate to recommend? A Acetaminophen B Acetylsalicylic Acid C Diclofenac D Ibuprofen E Naproxen
The Correct Answer is: A Acetaminophen is the only product on the list that won't exacerbate or interfere with platelet aggregation. All other products are to be avoided, as they interfere with platelet aggregation and exacerbate bleeding. (Lichtman et al., Williams Hematology 8e, Chapter 124, Hemophilia A and Hemophilia B)
Q 49.19: This common malignancy is diagnosed in patients younger than 15 years of age, has an incidence peak during early childhood (2 to 4 years old), and is seen more prominently in industrialized nations. What is the name of this malignancy? A Acute Lymphoblastic Leukemia B Acute Myelogenous Leukemia C Chronic Lymphocytic Leukemia D Chronic Myelogenous Leukemia E Lymphoma
The Correct Answer is: A Acute Lymphoblastic Leukemia is the most common malignancy diagnosed in patients younger than age 15, and has a peak incidence of 2 to 4 years of age. It is seen more often in males, and the pattern affected suggests a leukemogenic contribution from factors associated with industrialization.
Q 120.16: A 43-year-old female patient presents with back pain and hematuria. The patient reports having this problem earlier this year and recalls her previous clinician telling her, "they're just cysts." Denying any history of urinary tract infections, the patient reports her mother was on dialysis before passing away. The patient is afebrile and her physical examination is positive for diffuse back tenderness and bilateral flank masses with palpation. Urine dipstick is positive for 3+ blood and is negative for leukocytes and nitrites. What is this patient's most likely diagnosis? A adult polycystic kidney disease B renal cyst C horseshoe kidney D renal cell carcinoma
The Correct Answer is: A Adult polycystic kidney disease is a hereditary condition that almost always has a bilateral presentation (95% of the cases). It does not appear until after the age of 40, and dialysis or kidney transplantation is necessary for survival. Renal cysts and renal cell carcinoma generally present unilaterally. A horseshoe kidney (fusion of the renal tissue) may be palpated bilaterally; otherwise, the patient is asymptomatic. (McAninch, 2008, pp. 507-512) McAninch JW. Disorders of the kidneys. In: Tanagho EA , McAninch JW, eds. Smith's General Urology. 17th ed. New York, NY: McGraw-Hill; 2008:506-520. McAninch JW. Disorders of the penis and male urethra. In: Tanagho EA , McAninch JW, eds. Smith's General Urology. 17th ed. New York, NY: McGraw-Hill; 2008:625-637.
Q 97.99: A 36-year-old presents to the office for evaluation of painful breasts, which is worse before her period. She complains of them feeling fuller and lumpier before onset of her menses. She has tried acetaminophen and ibuprofen with minimal relief. Her symptoms resolve at the end of her menses. What is the most likely cause of the symptoms? A Normal cyclic hormone fluctuation B Abnormal cyclic hormone fluctuation C No cyclic hormone elevation D Hormone secreting tumor E Hypothalamic dysfunction
The Correct Answer is: A Age, cyclic history, and resolution are essential in the diagnosis of benign fibrocystic breast disease.
Q 23.9: A 45-year-old male comes to your primary care office complaining of recurrent panic attacks that have led to a disabling fear of being in places from which escape might be difficult, such as a bus or a train. He now finds he can barely leave his home. You diagnose him with panic disorder and which of the following complications? A Agoraphobia B Generalized anxiety disorder C Obsessive-compulsive disorder D Posttraumatic stress disorder E Social phobia
The Correct Answer is: A Agoraphobia (A) is a complication of panic disorder in which the attacks are associated with being in a crowd or around others. Generalized anxiety disorder (B) is a more overarching term, but does not necessarily include panic attacks. Obsessive-compulsive disorder (C) involves recurrent intrusive thoughts and rituals. Social phobia (E) is more focused on performance in normal social situations and post-traumatic stress (D) requires a traumatic trigger event.
Q 18.3: A 58 year-old man with a hemoglobin of 11.1 mg/dL and hematocrit of 33% is noted to have a serum folic acid level of 0.2 ng/dL. Which of the following risk factors for folate deficiency should this patient be assessed for? A Alcoholism B Diabetes C Down's syndrome D Smoking E Vegetarian diet
The Correct Answer is: A Alcoholism and the nutritional status that results is a common cause of folate deficiency. Diabetes (B) and smoking (D) are not directly associated with folate deficiency. Down's syndrome (C) risk can be reduced through the administration of prenatal folic acid. A vegetarian (E) diet provides excellent sources of folate.
Q 81.8: A 28-year-old male has Hemophilia A. He is in for genetic counseling on the risk of his children developing the disease. What is the likelihood that a son of his will develop Hemophilia A? A 0% B 25% C 50% D 75% E 100%
The Correct Answer is: A All daughters of a hemophilic male are carriers of hemophilia, whereas all sons are normal. Hemophilia A is one of only two sex-linked pattern-bleeding disorders, and as such the disease occurs almost exclusively in males. Sons of carriers have a 50% chance of being affected and daughters of carriers have a 50% chance of being carriers themselves. (Lichtman et al., Williams Hematology 8e, Chapter 124, Hemophilia A and Hemophilia B)
A 20-year-old male presents to the emergency department complaining of pain to the right shoulder region while playing basketball. He states that his arm was pulled back and rotated while he was moving forward, and then felt a popping sensation in the shoulder. Since then he has not been able to move the shoulder at all due to pain and immobility. It is suspected that he has a dislocation. Given this scenario, what would be the most likely type? What type of shoulder dislocation did he most likely experience? A Anterior B Inferior C Multidirectional D Posterior E Superior
The Correct Answer is: A All of the various types of dislocations mentioned above are possible, but anterior dislocations are by far the most common (>95%) and they are the most common of all joint dislocations. Most occur as a result of a fall or other traumatic event and they may become recurrent. The shoulder is most susceptible to an anterior dislocation when it is abducted and externally rotated. The shoulder joint is considered a very mobile joint, but this also renders it very susceptible to injury.
Q 20.10: A 20-year-old male presents to the emergency department complaining of pain to the right shoulder region while playing basketball. He states that his arm was pulled back and rotated while he was moving forward, and then felt a popping sensation in the shoulder. Since then he has not been able to move the shoulder at all due to pain and immobility. It is suspected that he has a dislocation. Given this scenario, what would be the most likely type? What type of shoulder dislocation did he most likely experience? A Anterior B Inferior C Multidirectional D Posterior E Superior
The Correct Answer is: A All of the various types of dislocations mentioned above are possible, but anterior dislocations are by far the most common (>95%) and they are the most common of all joint dislocations. Most occur as a result of a fall or other traumatic event and they may become recurrent. The shoulder is most susceptible to an anterior dislocation when it is abducted and externally rotated. The shoulder joint is considered a very mobile joint, but this also renders it very susceptible to injury.
Q 97.7: A 72-year-old male is found to have an absolute lymphocyte count of 13,500 (1000 to 3500). Marrow examination demonstrates infiltration with leukemic lymphocytes, with a lacy or interstitial pattern. What is the most common physical finding that would be associated with this patient? A Cervical and supraclavicular lymphadenopathy B Proptosis and headache C Rhinitis and polyneuropathy D Splenomegaly and hepatomegaly E Weight loss and night sweats
The Correct Answer is: A All of these findings can be present in this patient with chronic lymphocytic leukemia, but the most common physical finding in these patients are lymphadenopathy with over 80% presenting with nontender adenopathy. All others are findings that may occur, but are significantly less common. (Lichtman et al., Williams Hematology 8e, Chapter 94, Chronic Lymphocytic Leukemia and Related Diseases)
Q 27.2: A 19-year-old male is brought by his mother to your primary care office. She is at her wits' end as he has just gotten out of juvenile detention but continues to engage in the behaviors that resulted in his incarceration. He lies, cheats, steals, and seems to disregard the rights and needs of others to the point of endangering their safety. This pattern has been present since early childhood, since at least age four. He does not feel that there is a problem, but his family and others around him are disturbed by his attitudes and behaviors. What is the most likely diagnosis? A Antisocial personality disorder B Borderline personality disorder C Histrionic personality disorder D Narcissistic personality disorder E Schizoid personality disorder
The Correct Answer is: A All personality disorders involve a persistent pattern of behavior in which there is a disturbance in impulse control, interpersonal behavior, interpretation of people and events, and/or emotional response that begins no later than adolescence. Antisocial PD involves a reckless disregard for others, usually manifesting in lawlessness, lying, cheating, and a lack of remorse. Borderline PD (B) involves interpersonal reactivity and impulsivity often focused on perceived abandonment. Histrionic PD (C) involves a need to be the center of attention, often manifesting in behavior that is theatrical or seductive. Narcissistic PD (D) involves a lack of empathy and grandiosity. Schizoid PD (E) involves symptoms similar to schizophrenia.
Q 89.7: Which of the following exerts its action by inhibiting cell wall synthesis? A amoxicillin B ciprofloxacin C doxycycline D erythromycin E gentamicin
The Correct Answer is: A All β-lactam antibiotics, including the penicillins (eg, amoxicillin) and cephalosporins, prevent bacterial growth by inhibiting cell wall synthesis. Fluoroquinolones (eg, ciprofloxacin) block bacterial DNA synthesis. Erythromycin, doxycycline, and gentamicin all inhibit protein synthesis but via different mechanisms. (Craig and Stitzel, 2004, pp. 519, 526, 538, 544) Craig CR , Stitzel RE. Modern Pharmacology with Clinical Applications. 6th ed. Baltimore, MD: Lippincott Williams & Wilkins; 2004.
Q 48.5: A 5-year-old girl presents to the office with her mother, who states the child is experiencing excessive tearing, itching, and redness of her eyes. On physical exam you note marked injection with chemosis without discharge. The patient lacks adenopathy. Which of the following is the most likely diagnosis? A Allergic conjunctivitis B Bacterial conjunctivitis C Inclusion conjunctivitis D Keratoconjunctivitis E Viral conjunctivitis
The Correct Answer is: A Allergic conjunctivitis (A) is characterized by itching, tearing, redness, and chemosis, with itching being uncommon in other common forms of conjunctivitis. The absence of other viral symptoms (E) and discharge (B) make other diagnoses less likely. Preauricular adneopathy typically occurs in viral (E) or chlamydial conjunctivitis (C).
Q 102.10: A 54-year-old male with a known history of hypercholesterolemia presents to your office complaining of vision changes in his right eye. He states that this morning, while working on his computer, he noticed darkening of his vision, which he described as a "shade being pulled down over my eye." This was followed by approximately three minutes of vision loss in the right eye, which resolved. He denies any other symptoms and does not have a history of eye problems. Which of the following is the most likely diagnosis? A Amaurosis fugax B Intracranial tumor C Retinal detachment D Retinal vascular spasm E Uveitis
The Correct Answer is: A Amaurosis fugax, often characterized as transient blindness or vision impairment lasting 1 to 5 minutes, can result when blood flow to the retina is disrupted. Emboli, often due to atherosclerosis in carotid arteries or cardiac sources, impact the retinal arteries, causing the visual changes. As the body resolves the emboli and restores blood flow to the retina, symptoms improve. Retinal detachment is also associated with vision changes and may be described as having a curtain effect. This typically starts in the superior temporal area of vision and progresses, with central vision remaining intact, unless the macula is involved. Retinal vascular spasm and vasculitis are more rare causes of transient vision loss. Intracranial tumor, depending on location, may impact vision, with transient symptoms less likely. Acute uveitis typically presents with unilateral pain, redness, vision changes, and photophobia. (McPhee SJ, Papadakis MA. Current Medical Diagnosis & Treatment 2011, Chapter 7, Disorders of the Eyes & Lids)
Q 86.8: A 38-year-old woman presents with a history of frequent headaches that begin behind her right eye and are associated with a visual aura, photophobia, and phonophobia. She feels her headaches are worse due to job-related stress and insomnia. She is having six to eight instances of headache a month. Her medical history is remarkable for exercise-induced asthma. Which of the following agents is the best prophylactic agent for this patient? A amitriptyline (Elavil) B celecoxib (Celebrex) C propranolol (Inderal) D sumatriptan (Imitrex) E butalbital/caffeine (Midrin)
The Correct Answer is: A Amitriptyline would be an appropriate prophylactic agent that may also treat her insomnia. Celecoxib and sumatriptan are common abortive agents that also have a limited role as prophylactic agents. They are inappropriate choices for this patient due to the frequency of her headaches and ill-defined pattern. Propranolol is contraindicated because of her history of exercise-induced bronchospasm. Butalbital/caffeine is not indicated for use as a prophylactic agent. (Wells et al., 2009, p. 609) Wells BG , DiPiro JT , Schwinghammer TL , et al. Pharmacotherapy Handbook. 7th ed. New York, NY: McGraw-Hill; 2009.
Q 101.7: Two days following an uneventful 4-vessel CABG, a 57-year-old man develops a sudden onset of lightheadedness and palpitations. His vital signs are stable, and physical examination demonstrates no abnormalities. Given the results of his EKG, as shown (Figure 2), which of the following is the most appropriate next step in management? A Direct-current cardioversion B Nitroglycerin patch C Digoxin 0.125 mg PO daily D Neurology consult E Meclizine 25 mg PO Q6H
The Correct Answer is: A Among the choices offered here, choice A is the most appropriate next step in management of a patient with new onset atrial flutter, as determined by EKG; it most effectively converts most patients to normal sinus rhythm. Choice B is inappropriate, as the patient is not demonstrating angina pectoris, and the EKG does not demonstrate evidence of ischemia or infarction. Choice C is inappropriate, as it is the least effective agent for slowing the ventricular response when compared to beta blockade or calcium channel blockers, all of which act by blocking the AV node (digixon may occasionally convert atrial flutter to atrial fibrillation). Choice D is inappropriate, as the patient's symptoms of lightheadedness do not stem from neurologic changes. Choice E is inappropriate, as the patient's symptoms do not stem from vertigo. (McPhee et al., 2011, Chapter 10)
Q 117.8: A 45-year-old male presents with a complaint of itching and burning of his upper eyelid. This has been present for a week. Some dried discharge is seen clinging to his eyelashes. Which of the following is the most likely diagnosis? A Blepharitis B Chalazion C Dacryocyctitis D Hordeolum E Conjunctivitis
The Correct Answer is: A Anterior Blepharitis is common and can be bilateral. Causes include staphylococcus and seborrhea. Posterior blepharitis is caused by meibomian gland dysfunctions. (Riordan et al., 2008, Chapter 4)
Q 22.4: A 52-year-old male has a 200-pound file cabinet fall on his right leg. He comes to the emergency department complaining of pain and swelling to the right leg. He is also complaining of parasthesias to the leg also. Based on these findings on history, what part of the leg would be the most likely site of compartment syndrome? A Anterior compartment of the leg B Deep posterior compartment of the leg C Dorsal compartment of the forearm D Lateral compartment of the leg E Superficial posterior compartment of the leg
The Correct Answer is: A Anterior compartment syndrome is most commonly found in the anterior compartment of the leg, with the volar compartment of the forearm also a common location. The mechanism is generally an acute crushing trauma to the affected area that causes an increase in pressure within the compartment that inhibits venous outflow and a decrease in arterial blood flow. This adversely affects tissue perfusion and ischemia of the involved tissues can occur. The symptoms can also occur with chronic exertion or when there is a dramatic increase in the amount of exercise being performed. Anterior compartment syndrome of the leg is likely most common due to its vulnerable location and susceptibility to injury in athletics and motor vehicle accidents. The rate of occurrence of acute anterior compartment syndrome is also likely due to the fact that the anterior compartment lies adjacent to the tibia and the tibia is the most frequently fractured long bone. Compartment syndrome could occur in any compartment of the upper or lower extremities if the necessary circumstances were in place (either acute trauma to the area or excessive use of the muscles that are associated with each of the compartments).
Q 98.32: A 52-year-old male has a 200-pound file cabinet fall on his right leg. He comes to the emergency department complaining of pain and swelling to the right leg. He is also complaining of parasthesias to the leg also. Based on these findings on history, what part of the leg would be the most likely site of compartment syndrome? A Anterior compartment of the leg B Deep posterior compartment of the leg C Dorsal compartment of the forearm D Lateral compartment of the leg E Superficial posterior compartment of the leg
The Correct Answer is: A Anterior compartment syndrome is most commonly found in the anterior compartment of the leg, with the volar compartment of the forearm also a common location. The mechanism is generally an acute crushing trauma to the affected area that causes an increase in pressure within the compartment that inhibits venous outflow and a decrease in arterial blood flow. This adversely affects tissue perfusion and ischemia of the involved tissues can occur. The symptoms can also occur with chronic exertion or when there is a dramatic increase in the amount of exercise being performed. Anterior compartment syndrome of the leg is likely most common due to its vulnerable location and susceptibility to injury in athletics and motor vehicle accidents. The rate of occurrence of acute anterior compartment syndrome is also likely due to the fact that the anterior compartment lies adjacent to the tibia and the tibia is the most frequently fractured long bone. Compartment syndrome could occur in any compartment of the upper or lower extremities if the necessary circumstances were in place (either acute trauma to the area or excessive use of the muscles that are associated with each of the compartments).
Q 47.3: A 10-year-old girl was recently diagnosed with type 1 diabetes. She appears to be adjusting well to her regimen of insulin and glucose monitoring, and follows a careful meal plan that she and her mother work on together. At a follow-up visit, she says that she will "die" if she can't participate on her summer swim team, but her mother is clearly concerned about her becoming hypoglycemic if she exercises that vigorously. In addition to reminding them that regular aerobic exercise is an important part of her overall well-being, which of the following is the most appropriate advice for them? A add an extra snack before exercising to the diet plan B ask the coach for a less-rigorous practice regimen C avoid dextrose-containing beverages during exercise D increase the amount of insulin on practice and competition days E try a less energy-intensive sport than swimming
The Correct Answer is: A Appropriate advice for children with type 1 diabetes is to have extra snacks before exercise. In addition, they should drink dextrose-containing fluids (C) during exercise and decrease insulin (D), while monitoring glucose before, during, and after exercise. Exercising less rigorously (B) and avoiding any particular sport (E) is usually not necessary with careful management of the diabetes.
Out of all cervical vertebrae, which two are responsible for the greatest amount of rotation? A C1 & C2 B C2 & C3 C C3 & C4 D C4 & C5 E C5 & C6
The Correct Answer is: A Approximately 50% of cervical rotation takes place between the C1 (atlas) and C2 (axis) vertebrae. These first two cervical vertebrae have a different shape from the other cervical vertebrae that allow for this greater range of motion. The remaining 50 % of cervical rotation is split fairly evenly between the remaining vertebrae. Approximately 50 % of flexion and extension occurs between the occiput at the base of the skull and C1 with the remaining 50% distributed fairly evenly between the remaining vertebrae with a slightly higher percentage occurring at the C5 & C6 level.
Q 98.37: Out of all cervical vertebrae, which two are responsible for the greatest amount of rotation? A C1 & C2 B C2 & C3 C C3 & C4 D C4 & C5 E C5 & C6
The Correct Answer is: A Approximately 50% of cervical rotation takes place between the C1 (atlas) and C2 (axis) vertebrae. These first two cervical vertebrae have a different shape from the other cervical vertebrae that allow for this greater range of motion. The remaining 50 % of cervical rotation is split fairly evenly between the remaining vertebrae. Approximately 50 % of flexion and extension occurs between the occiput at the base of the skull and C1 with the remaining 50% distributed fairly evenly between the remaining vertebrae with a slightly higher percentage occurring at the C5 & C6 level.
Patients diagnosed with an auricular hematoma are at increased risk of developing which condition? A Cartilage necrosis B Cholesteatoma C Coagulopathy D Exostosis E Otomycosis
The Correct Answer is: A Auricular hematomas occupy the subperichondral space, leading to decreased or absent diffusion from the perichondrium to the cartilage and resulting in increased risk of necrosis (A). Coagulopathy (C) may predispose a patient to experiencing a hematoma. Cholesteatoma (B) may result from TM trauma, but not blunt trauma to the outer ear.
Q 116.9: A 21-year-old college student presents with headache, fever, and a stiff neck. You perform a lumbar puncture for suspected meningitis. Which of the following cerebrospinal fluid analysis results is consistent with bacterial meninigitis? A Elevated opening pressure, markedly elevated white blood cell count, decreased glucose, increased protein concentration B Elevated opening pressure, mildly elevated white blood cell count, markedly elevated red blood cell count with noted xanthochromia C Mildly elevated opening pressure, white blood cell count elevated but less than 1000/mL, normal glucose, protein slightly increased D Mildly elevated opening pressure, elevated white blood cell count, low glucose, high protein, positive spirochetes E Normal opening pressure, normal white blood cell count, elevated red blood cell count
The Correct Answer is: A Bacterial meningitis is associated with specific cerebrospinal fluid analysis results, although some overlap with other diagnoses does exist. The CSF for bacterial meningitis should reveal a markedly elevated white blood cell count, including a high percentage of polymorphonuclear leukocytes. Additional findings should include a decreased glucose concentration and elevated protein values. Gram stain analysis will reveal bacteria morphology and suggest an infectious etiology, although at times additional specialized testing may be needed, such as a fluorescent treponemal antibody absorption (FTA-ABS) test, in cases of suspected neurosyphilis. Choice B suggests subarachnoid hemorrhage, choice C suggests viral meningitis, choice D suggests syphilis, and choice E suggests a traumatic tap. (Ropper AH, Samuels MA. Adams and Victor's Principles of Neurology, 9e, Chapter 2, Special Techniques for Neurologic Diagnosis)
Q 65.2: A 53 year-old female is diagnosed with bladder cancer after an extensive work-up. Which pathology would be expected on her results, considering the most common cause of bladder cancer? A Urothelial (transitional) cell carcinoma B Squamous cell carcinoma C Adenocarcinoma D Clear cell carcinoma E Basal cell carcinoma
The Correct Answer is: A Baldder carcinoma pathology is transitional (urothelial) cell carcinoma (~90% of cases) (A); squamous cell carcinoma (B), (~7% of cases); adenocarcinoma (C), (~2% of cases). Clear cell carcinoma (D) and basal cell carcinoma (E) are not common findings in bladder cancer.
Q 11.8: A 36-year-old female presents to your family practice office concerned about a breast lump she discovered in the shower last night. You obtain a detailed history and these are some of your findings: • G2P2 • LMP: 2 weeks ago, normal • Sister diagnosed with breast cancer at age 42. • Mother is currently being treated for ovarian cancer. • Father is of Ashkenazi Jewish ancestry. Which of the following is an appropriate recommendation for this patient? A Enhanced surveillance, including mammography alternating with MRI, every six months. B Genetic counseling and testing for BRCA1 and BRCA2 status. C Referral to a breast surgeon for mastectomy D Chemoprophylaxis with an aromatase inhibitor weekly.
The Correct Answer is: A Because of her ethnicity (Ashkenazi Jewish descent) and family history (two first degree relatives with breast and ovarian cancers), this patient is at a higher than normal risk for breast and ovarian cancers. With a physical finding of a palpable breast mass, this is even more ominous. She should be referred for an immediate mammogram but not necessarily MRI - and have continued enhanced surveillance annually and not every 6 months regardless of the result - as well as a referral to a breast surgeon for immediate consideration of a biopsy. A mastectomy is a possible consideration but not mandatory. She is certainly an appropriate candidate for consideration of referral to a genetic counselor and genetic testing to ascertain her BRCA1 and BRCA2 status. She is also an appropriate candidate for chemoprophylaxis.
Q 73.1: Which of the following is a potential complication of acute pyelonephritis? A perinephric abscess B renal vein thrombosis C allergic interstitial nephritis D struvite stones E hepatic failure
The Correct Answer is: A Because pyelonephritis is an infectious disease, the most likely complication is a perinephric abscess, which occurs as the result of inadequate therapy. Since it is not vascular in origin, renal vein thrombosis would not occur. Allergic interstitial nephritis is caused by an antigen-antibody reaction, which does not occur with acute pyelonephritis. Struvite stones are due to chronic infection with urease-producing organisms, such as Proteus and Pseudomonas, not to an acute infection. Hepatic failure can be a complication of acute renal failure, but not acute pyelonephritis. (Stoller et al., 2009, p. 830) Stoller ML , Kane CJ , Meng MV. Urology. In: Tierney LM , McPhee SJ , Papadakis MA, eds. Current Medical Diagnosis and Treatment. 48th ed. New York, NY: McGraw-Hill; 2009.
Q 21.2: Which of the following joints has the lowest occurrence rate of osteoarthritis? A Elbows B Hands C Hips D Knees E Spine
The Correct Answer is: A Because the elbow is not a weight bearing bone, the rate of osteoarthritis in the elbow is considerably less that what is found in locations like the hips, knees and spine. The hands have one of the highest rates of occurrence of osteoarthritis, likely due to their near constant use and propensity for minor (or major) injury. When elbow arthritis does develop it is often post-traumatic osteoarthritis related to a significant injury in the past that disrupted joint surface integrity or as a result of rheumatoid arthritis, a systemic illness. Osteoarthritis of the elbow will generally present with pain, stiffness, and decreased range of motion. Osteophytes that form on the medial elbow might be implicated should neurological symptoms develop that correlate with ulnar nerve distribution as this nerve does pass in close proximity to the elbow on the medial side.
Q 21.9: A 58-year-old male comes to your primary care office complaining of sadness, insomnia, loss of appetite, weight loss, and feelings of guilt or hopelessness for the past month. On further questioning you find that his wife of 30 years died of breast cancer shortly before the onset of symptoms. Which of the following is the most likely diagnosis? A Bereavement B Dysthymia C Depressive episode D Depressed mood E Major depression
The Correct Answer is: A Bereavement (A) is the normal grief response experienced after a significant loss and includes all the symptoms of depression and, by definition, lasts no longer than two months, although many will suffer from some symptoms for longer. Dysthymia (B) is a longstanding depressed mood for at least two years but not meeting the full criteria for a depressive episode (C). Major depression (E) requires at least one depressive episode, which requires at least five of the symptoms, one of which is depressed mood (D).
Q 49.9: A 5-year-old male child in the clinic is being evaluated for a firm, painful lump that is slightly reddened and approximately 3 cm in diameter, in his right axilla. His mother tells you the lump has been there for a couple of days. The boy does not look acutely ill. The mother informs you that they got a new kitten and puppy about a month ago but otherwise nothing else is new at home. Which of the following is the most likely etiology for his rash? A Bartonella henselae B parvovirus C Hodgkin disease D Osgood-Schlatter disease
The Correct Answer is: A Cat scratch disease (CSD) is caused by the gram-negative bacillus, Bartonella henselae. The disease is more common in the fall and winter months and more males than females are affected. Typically (approximately 90%), patients report handling a cat or kitten and up to 70% will report a scratch by a cat. The most common complication of CSD is encephalitis. About half of the patients with CSD will develop a primary cutaneous papule at the site of inoculation, most often (approximately 50%) on the hands or upper extremities, 3 to 10 days after the exposure. Regional lymphadenopathy will usually develop in about 1 to 7 weeks after the cutaneous lesions and will affect the nodes draining the site of the scratch or bite. The affected lymph nodes may be inflamed and are usually tender. Occasionally, the involved nodes may suppurate. The lymphadenopathy resolves in about 2 months, but may last as long as 4 to 8 months. Treatment is usually not indicated for this self-resolving disease. However, suppurative lesions may need to be aspirated for pain relief. It has been shown that 5 days of treatment with azithromycin has helped to speed recovery for some patients. Because Hodgkin disease involves the lymph nodes, it should be considered as a differential diagnosis when evaluating a child for CSD. However, it typically presents as a cervical lymphadenopathy. Fifth disease (erythema infectiosum) is a childhood disease caused by the human parvovirus. This common community-acquired disease does not usually require treatment, but respiratory isolation is recommended for 7 days following the onset of symptoms. The initial stage of the disease presents as red cheeks that appear to be "slapped" or "slapped cheeks" with circumoral pallor. Osgood-Schlatter disease is an orthopedic problem in children. It is the result of repetitive microtraumas to the patellar ligament at its point of insertion into the tibial tubercle. Usually, rest and anti-inflammatory medications are helpful in alleviating the pain associated with this condition.
Q 98.2: A patient is scheduled for a mitral valve replacement. Which of the following pharmacologic agents would be recommended for surgical prophylaxis? There are no known drug allergies. A Cefazolin B Vancomycin C Ciprofloxacin D Nafcillin
The Correct Answer is: A Cefazolin is used as prophylaxis for the majority of clean surgical procedures. For cases in which there is an increased likelihood of encountering gram-negative organisms or anaerobic bacteria, a second-generation cephalosporin is recommended to provide broader coverage. Vancomycin is an alternative if the patient has an allergy to cephalosporin antibiotics.
Q 47.2: A 3-year-old girl is being followed by the neurologist to evaluate her motor spasticity that resulted from anoxia during labor and delivery. Which of the following is the most likely cause of this patient's spasticity? A cerebral palsy B congenital hypothyroidism C meningitis D multiple sclerosis
The Correct Answer is: A Cerebral palsy is caused by perinatal injury to the nervous system and results in motor spasticity. The history of perinatal anoxia is consistent with cerebral palsy. Congenital hypothyroidism is typically asymptomatic at birth and diagnosed through routine screening tests. Neonatal meningitis may result in anoxia, but this patient's anoxia is attributed to the birthing process. Multiple sclerosis is caused by inflammation, demyelination, and scarring.
Q 21.8: A 43 year-old woman is brought to the emergency department in critical care secondary to a traumatic brain injury. Which of the following best describes her Cheyne-Stokes respiratory pattern? A Alternating periods of shallow and deep breathing B Difficult or labored breathing C Difficult of labored breathing while supine D Periods of absent breathing E Sudden awakening due to shortness of breath
The Correct Answer is: A Cheyne-Stokes respirations are characterized by shallow breaths that increase in rate and depth followed by periods of apnea (A). Dyspnea is difficult or labored breathing (B) and if it occurs while supine (C) is termed orthopnea. Apnea is noted periods of absent breathing (D). Sudden awakening due to shortness of breath (E) is paroxysmal nocturnal dyspnea.
A 24-year-old HIV-positive man comes to the emergency department complaining of severe left-sided chest discomfort, which radiates through to the left trapezius region. On coming into the room, you note that he is sitting up and hunched forward. On physical examination, the patient's blood pressure is 135/78, with a pulse of 85 bpm, and a pericardial friction rub is noted. Laboratory findings demonstrate elevated serum creatine kinase levels and normal serial troponin levels. His EKG demonstrates global ST segment elevation. His CXR demonstrates no acute process. Which of the following is the most likely diagnosis in this patient? A Acute pericarditis B Acute myocardial infarction C Acute bacterial endocarditis D Acute ascending aortic dissection E Acute costochondritis
The Correct Answer is: A Choice A is the most likely finding, as this patient is exhibiting signs, symptoms, and EKG findings pathognomonic for acute pericarditis, which is likely infectious in the setting of a patient with HIV. A pericardial friction rub is heard best with the patient in a seated position, during expiration, and is frequently found in patients with pericarditis. Choice B, an acute myocardial infarction, is less likely in a patient of this age, especially with normal serial troponins. Acute pericarditis can sometimes present with elevated serum creatine kinase levels when the epicardium is also involved. Choice C, acute bacterial endocarditis, is less likely in a patient with these EKG changes. Choice D, aortic dissection, would present with chest pain; however, the patient would be markedly hypotensive, less stable on presentation, and a CXR would demonstrate widening of the superior mediastinum.
Q 97.5: A 24-year-old HIV-positive man comes to the emergency department complaining of severe left-sided chest discomfort, which radiates through to the left trapezius region. On coming into the room, you note that he is sitting up and hunched forward. On physical examination, the patient's blood pressure is 135/78, with a pulse of 85 bpm, and a pericardial friction rub is noted. Laboratory findings demonstrate elevated serum creatine kinase levels and normal serial troponin levels. His EKG demonstrates global ST segment elevation. His CXR demonstrates no acute process. Which of the following is the most likely diagnosis in this patient? A Acute pericarditis B Acute myocardial infarction C Acute bacterial endocarditis D Acute ascending aortic dissection E Acute costochondritis
The Correct Answer is: A Choice A is the most likely finding, as this patient is exhibiting signs, symptoms, and EKG findings pathognomonic for acute pericarditis, which is likely infectious in the setting of a patient with HIV. A pericardial friction rub is heard best with the patient in a seated position, during expiration, and is frequently found in patients with pericarditis. Choice B, an acute myocardial infarction, is less likely in a patient of this age, especially with normal serial troponins. Acute pericarditis can sometimes present with elevated serum creatine kinase levels when the epicardium is also involved. Choice C, acute bacterial endocarditis, is less likely in a patient with these EKG changes. Choice D, aortic dissection, would present with chest pain; however, the patient would be markedly hypotensive, less stable on presentation, and a CXR would demonstrate widening of the superior mediastinum. (Fauci et al., 2001, pp. 810-811, 1365-1366, 1431)
Q 94.2: A 60-year-old marathon runner has noticed a progressive decline in his exercise tolerance over the past year. He is now dypsneic while walking up one flight of stairs, and notices increased lower extremity edema. He experienced an episode of chest pressure while playing racquetball, and underwent a nuclear stress test, which was found to be normal. On physical exam, a grade 2/6 crescendo-decrescendo murmur is noted, radiating to the carotid arteries. An echocardiogram is ordered. Which of the following results would be expected, based on the patient's history and physical exam findings? A Bicuspid aortic valve with severe stenosis B Small patent foramen ovale C Tricuspid regurgitation D Pulmonic stenosis E Apical aneurysm
The Correct Answer is: A Choice A, a bicuspid aortic valve, would be the most likely finding on echocardiogram given the patient's history and physical exam findings. The patient is demonstrating symptoms suggestive of congestive heart failure at a relatively young age, with relatively few cardiac risk factors. A congenital bicuspid aortic valve does not usually cause symptoms until the fifth or sixth decade, after many years of calcification have led to severe stenosis. Symptom onset can be gradual or sudden. The patient's physical exam findings are pathognomonic for aortic stenosis, so choices C and D are inappropriate. Choice B, patent foramen ovale, can result in congestive heart failure if the PFO is large and if a large shunt is present; therefore, a small PFO is unlikely to be the cause of this patient's symptoms. Choice E, apical aneurysm, can also lead to congestive heart failure if large, but would not present with this patient's physical exam findings. (McPhee et al., 2011, Chapter 10)
Q 115.18: A 24-year-old HIV-positive man comes to the emergency department complaining of severe left-sided chest discomfort, which radiates through to the left trapezius region. On coming into the room, you note that he is sitting up and hunched forward. On physical examination, the patient's blood pressure is 135/78, with a pulse of 85 bpm, and a pericardial friction rub is noted. Laboratory findings demonstrate elevated serum creatine kinase levels and normal serial troponin levels. His EKG demonstrates peaked T waves. His CXR demonstrates a "water bottle" cardiac silhouette. Which of the following are serious consequences of acute pericarditis, which require careful monitoring? A Pericardial effusion B Aortic dissection C Myxedema D Chylopericardium E Acute myocardial infarction
The Correct Answer is: A Choice A, pericardial effusion, is a serious consequence of acute pericarditis, which requires careful monitoring to ensure that progression of the pericardial effusion does not lead to cardiac tamponade, which can be fatal if not treated promptly. Choice B is unlikely in a patient with acute pericarditis. Choices C, D, and E are noninfectious causes, not consequences of pericarditis. (Fauci et al., 2001, p. 1368)
Q 37.5: Examination of a female newborn demonstrating cyanosis after feeding reveals a systolic ejection click that is heard best at the third left intercostal space, and a short grade III/VI systolic ejection murmur that is heard best at the second left intercostal space. Given this patient's physical exam findings, which of the following is the most likely finding on an echocardiogram? A Pulmonic stenosis B Aortic stenosis C Mitral regurgitation D Aortic regurgitation E Mitral stenosis
The Correct Answer is: A Choice A, pulmonic stenosis, is the most likely finding on echocardiogram, as severe pulmonic stenosis frequently presents with a newborn infant presenting with cyanosis after feeding, and a systolic ejection murmur heard best at the second left intercostal space. Congenital pulmonic stenosis occurs in 1 in 10 patients with congenital heart disease. Most patients demonstrate only mild to moderate pulmonary stenosis and are asymptomatic, but infants with critical stenosis require intervention with balloon valvuloplasty. Choice B, aortic stenosis, if critical, would present with pulmonary congestion and heart failure within the first few weeks of birth, with a lack of peripheral pulses and no murmur. Choice C, mitral regurgitation, would not present with a cyanotic infant; however, in more severe mitral regurgitation, failure to thrive, frequent respiratory infections, and heart failure can occur. It would also present with a holosystolic blowing murmur, heard best at the apex. Choice D, mitral stenosis, would present with symptoms of failure to thrive and dyspnea, and an accentuated S1 and loud pulmonary closure sound. A presystolic crescendo murmur can be heard best at the apex.
Q 74.1: A 57-year-old woman with a history of rheumatic fever is seen complaining of dyspnea while vacuuming her apartment, which has been worsening over the last few months. On physical exam, a very soft systolic murmur is auscultated. With inspiration, the murmur increases, and is heard best at the left lower sternal border. A large and early v jugular venous wave is noted. Which of the following is the most likely finding on echocardiogram given this patient's physical exam findings? A Tricuspid regurgitation B Ventricular septal defect C Atrial septal defect D Aortic stenosis E Mitral stenosis
The Correct Answer is: A Choice A, tricuspid regurgitation, is the most likely finding in a patient with a history of rheumatic fever who is presenting with a systolic murmur that increases with inspiration. It is heard best at the left lower sternal border. Tricuspid stenosis is also frequently seen in association with tricuspid regurgitation. The murmur of tricuspid stenosis, however, is a diastolic murmur, which increases with inspiration, and is also heard best at the left lower sternal border. Choice B, ventricular septal defect, would present with a harsh holosystolic murmur, heard best at the third and fourth left interspaces along the sternum. Choice C, an atrial septal defect, if large, could present with similar symptoms of exertional dyspnea secondary to a large shunt, but auscultation would reveal a moderately loud systolic ejection murmur heard best in the second and third interspaces. This is secondary to increased pulmonary arterial flow. Choice D, aortic stenosis, presents with a systolic ejection murmur heard best at the right upper sternal border, frequently radiating to the carotid arteries. The murmur of mitral stenosis, choice E, is also frequently associated with a history of rheumatic fever; however, it is heard best in the left lateral decubitus position, with the bell of the stethoscope at the apex, and inspiration does not increase it. In patients with tricuspid stenosis, mitral stenosis should always be suspected. (LeBlond et al., 2009, Chapter 8)
Q 64.6: A 57-year-old woman with a history of rheumatic fever is seen complaining of dyspnea while vacuuming her apartment, which has been worsening over the last few months. On physical exam, jugular venous distension is appreciated. Auscultation of the chest reveals a possible opening snap, loud S 1 , and a very soft diastolic rumbling murmur is auscultated at the left lower sternal border. When the patient is placed in the left lateral decubitus position, the murmur is accentuated, and heard best at the apex. Both hepatomegaly and splenomegaly are noted. On transthoracic echocardiogram, which of the following findings is most likely to be seen in addition to mitral valve stenosis? A Tricuspid stenosis B Atrial septal defect C Aortic stenosis D Ventricular septal defect E Left ventricular hypertrophy
The Correct Answer is: A Choice A, tricuspid stenosis, is frequently seen in association with mitral valve stenosis in patients with a history of rheumatic heart disease (between 5% and 10% of the time). Both murmurs are similar in character, with the main difference that the murmur of tricuspid stenosis is heard best at the left lower sternal border, and the murmur of mitral stenosis is heard best in the left lateral decubitus position, with the bell at the apex. Because they are similar in nature, a high level of suspicion for tricuspid stenosis should be maintained, so that tricuspid stenosis is not overlooked. Tricuspid regurgitation may also be seen in association with tricuspid stenosis. Patients frequently present with signs and symptoms of right-sided heart failure, including hepatic congestion and splenomegaly. Choices B, C, D, and E are not seen as frequently as tricuspid stenosis in association with rheumatic heart disease and mitral valve stenosis.
Q 97.115: A 23-year-old woman presents with complaints of pelvic discomfort and a vaginal discharge for the past 3 days. She finished her period last week. She is taking oral contraceptives as directed. Her medical history is significant for a therapeutic abortion with no other hospitalizations or pregnancies. She has had three sexual partners in the past 6 weeks and does not use condoms. Her most recent partner reported that he was treated recently for gonorrhea. On examination, she has a mucopurulent discharge with "strawberry" cervix on speculum examination. After collecting the appropriate specimens, the best therapeutic option for this patient is A ofloxacin 400 mg i po × 1 dose plus azithromycin 250 mg iiii po × 1 dose B fluconazole 150 mg i po × 1 dose C metronidazole 500 mg iiii po × 1 dose D ceftriaxone 250 mg IM × 1 dose
The Correct Answer is: A Clinical presentation is consistent with cervicitis in a young woman with risk factors for sexually transmitted infection. She has likely been exposed specifically to Neisseria gonorrhea. Coinfection with Chlamydia trachomatis is common. While test results are pending, the Centers for Disease Control and Prevention in its 2006 Guidelines for STD treatment recommend treating for both with single doses (improved compliance) of ofloxacin and azithromycin first-line. In areas with quinolone resistance, intramuscular ceftriaxone is an option but coverage for Chlamydia is still necessary. Metronidazole is the appropriate therapy for trichomoniasis and fluconazole for vaginal candidiasis.
Q 92.7: Which of the following is the appropriate treatment for acute Clostridium tetani infection? A tetanus immune globulin, tetanus toxoid, and metronidazole B tetanus immune globulin and penicillin C tetanus toxoid and penicillin D tetanus immune globulin, tetanus toxoid, and penicillin E tetanus immune globulin and tetanus toxoid
The Correct Answer is: A Clostridial tetani infection is a vaccine-preventable disease that results in approximately 50 cases/yr in the United States. Even with modern medical resources, one of four or one of five patients with generalized tetanus dies. Almost all cases occur in individuals who are not properly immunized. Sixty percent of cases occur in older adults for whom immunity has waned. Tetanus presents in different forms including generalized, localized, cephalad, and neonatal. Generalized is the most common and symptoms include mood changes, trismus, diaphoresis, dysphagia, and drooling. Later symptoms include painful flexion and adduction of the arms and pain with extension of the legs. Convulsions and spasms are possible, along with a variety of autonomic symptoms. Treatment includes airway protection, benzodiazepines for muscle spasm, tetanus immune globulin immediately, and three doses of tetanus toxoid given by the standard schedule. Metronidazole has been demonstrated to be the most effective antimicrobial. Labetalol may be used for catecholamine-induced hypertension but the patient must also be monitored for hypotension and bradycardia. (Bartlett, 2007, pp. 2205-2207) Bartlett JG. Clostridial infections. In: Goldman L , Ausiello D, eds. Cecil Medicine. 23rd ed. Philadelphia, PA: Saunders Elsevier; 2007.
Q 117.4: A 42 year old male presents to your office with a 5 day history of multiple episodes of intense, unilateral retroorbital pain of short duration associated with lacrimation and flushing. He describes the pain as a "hot poker" sensation with occurrences at about the same time each day, although not limited to this time. Which of the following is the most likely diagnosis? A Cluster headache B Migraine headache C Sinus headache D Temporal arteritis E Tension headache
The Correct Answer is: A Cluster headaches are typically described as explosive, with marked unilateral retroorbital pain. Patients will often note some relation to time of day, but the headaches can occur at other times as well. Cluster headache patients often have exacerbations with recurrent symptoms over a period of time. Some patients will develop refractory or prolonged episodes, which will require additional management. Cranial parasympathetic autonomic activation leads to the associated symptoms of conjuntival injection, lacrimation, rhinorrhea, and nasal congestion. Cluster headaches must be differentiated from other diagnoses, utilizing history and physical exam findings. (Fauci et al., Harrison's Principles of Internal Medicine, 17e, Chapter 15, Headache)
Q 32.6: A 42 year-old woman presents complaining of shortness of breath, three days of fever as high as 103F, and has a cough productive of green sputum. On physical examination, you hear crackles in her lungs. A chest radiograph reveals a consolidation in the left lower lobe. What do you expect to hear when you percuss this patient's left lower thorax? A Dull B Flat C Hyperresonant D Resonant E Tympanic
The Correct Answer is: A Consolidation leads to dullness (A) to percussion. Flat sounds (B) are noted with organ percussion (i.e., hepatic flatness), hyperresonant (C) sounds are noted in pneumothorax, normal lung parenchyma results in resonance (D), and percussion of the gastric air bubble leads to tympanic sounds.
Q 44.5: A 4-year-old female is brought in by her parents due to an increased nightly cough and low grade temperature. The x-ray shown reveals a classic finding for which of the following diagnoses? A Croup B Epiglottitis C Foreign body aspiration D Peritonsillitis E Tracheal carcinoma
The Correct Answer is: A Croup, also known as laryngotracheobronchitis, is associated with upper tracheal narrowing and edema, which is visible on an anteroposterior soft tissue neck x-ray. This is termed the "steeple sign." Epiglottitis is associated with a thickened epiglottis on a lateral soft-tissue neck x-ray, termed the "thumb" sign. Foreign body aspiration and tracheal carcinoma may have x-ray findings based on the location, size, and components present. Peritonsillitis is best visualized on physical exam. If assessing for a potential peritonsillar abscess, a contrasted CT is recommended.
Q 43.8: A neonate presents with meconium ileus that is successfully unobstructed. The infant returns at her 4-month appointment with signs of failure to thrive. Which of the following is the most likely diagnosis for this patient? A cystic fibrosis B Wilson disease C intussusception D volvulus
The Correct Answer is: A Cystic fibrosis (CF) is a major cause of gastrointestinal and pulmonary morbidity in children due to mutations in the CF genes. The mutations lead to a deficiency in cystic fibrosis transmembrane conductance regulator protein that controls movement of salt and water into and out of epithelial cells and results in production of abnormally thick mucus. About 15% of patients with CF present with meconium ileus at birth. This is typically treated with enema for disimpaction and rarely surgery. Approximately half of the infants with CF will present with failure to thrive, which is diagnosed by lack of growth for 2 consecutive months in patients younger than 6 months of age. They may also present with respiratory compromise. However, not all patients present in childhood. Diagnosis of CF is confirmed by a sweat chloride level above 60 meq/L or with genetic testing. Treatment for patients with CF is mainly symptomatic therapy for obstructions of the digestive and respiratory tract. In addition, there is pancreatic enzyme supplementation to aid in digestion and vitamin and calorie supplementation for deficiencies in the diet. Gene therapy is now being looked at for future treatment. Intussusception (telescoping of the small intestine) typically presents in an infant with paroxysmal abdominal pain, vomiting, and diarrhea that may progress into bloody stools. Volvulus is normally the result of intestinal malrotation that causes occlusion of the superior mesenteric artery and eventual bowel necrosis. Infants typically present within 3 weeks of life with bile-stained vomiting and bowel obstruction. Wilson's disease is the defect in the ability to excrete copper in the bile that results in accumulation of copper in the liver.
Q 34.1: At 12 hours of age, a physical examination is performed on a neonate who has intrauterine growth retardation. He is noted to have microcephaly, jaundice, and hepatosplenomegaly. Which of the following is the MOST likely congenital viral infection in this neonate? A cytomegalovirus B herpes simplex virus C rubella D syphilis
The Correct Answer is: A Cytomegalovirus (CMV) is one of the congenital neonatal TORCH infections (toxoplasmosis, other [syphilis, varicella-zoster, and parvovirus in this list], rubella, cytomegalovirus, and herpes simplex/hepatitis/HIV). CMV is the most common congenital infection. The disease-specific manifestations for CMV include microcephaly with periventricular calcifications, neonatal jaundice with direct hyperbilirubinemia, and hepatosplenomegaly. Other associated manifestations include intrauterine growth retardation, thrombocytopenia, and purpura. Disease-specific manifestations for herpes simplex virus include skin/eye/mouth vesicles, encephalitis, respiratory distress, and sepsis. Disease-specific manifestations of rubella include congenital heart lesions (patent ductus arteriosus, pulmonary artery stenosis, aortic stenosis, ventricular defects), thrombocytopenic purpura characterized by purple macular lesions ("blueberry muffin" appearance), cataracts, retinopathy, and sensorineural deafness. Disease-specific manifestations of syphilis include mucocutaneous lesions (snuffles), periostitis, osteochondritis, and hemolytic anemia. Often, these babies are stillborn. Syphilis is caused by a spirochete, Treponema pallidum, not a virus.
Q 15.2: A 32-year-old female professional golfer presents with a 3-week history of pain along her thumb and down her wrist. She denies any trauma and states that it is aggravated with any movement of her wrist and thumb. Her physical examination is unremarkable. You perform the following test: This test requires her to cup her thumb in a closed fist and ulnar deviate, which reproduces her pain. Considering the suspected diagnosis based on the positive examination above, what are the affected anatomical structures? A Extensor pollicis brevis and abductor pollicis longus B Flexor pollicis longus and flexor digitorum profundus C Extensor carpi radialis longus and brevis D Ulnar collateral ligament
The Correct Answer is: A De Quervain tenosynovitis is a common condition that occurs in patients who have experienced excessive use of the thumb or wrist. This is a tenosynovitis of the extensor pollicis brevis and abductor pollicis tendons (A), where the tendons lie in the groove of the radial styloid. The diagnosis of De Quervain tenosynovitis is supported by a history of pain in this location along with a painful range of motion of the thumb and further confirmation may be provided by a positive Finkelstein test. The flexor pollicis longus and flexor digitorum profundus (B) are affected in carpal tunnel syndrome. Extensor carpi radialis longus and brevis (C) are extrinsic extensor muscles of the hand. The ulnar collateral ligament (D) is injured in Gamekeeper's thumb.
Q 121.8: A 45-year-old woman comes to the office for evaluation of "yeast infection again." Review of the chart shows that she has had six episodes of vaginal candidiasis in the past 18 months. She and her husband have been mutually monogamous during their 23-year marriage; he had a vasectomy following the birth of their second child. Her two children weighed 10# 2 oz. and 10# 4 oz. at delivery. Her last menstrual period occurred one week ago and was slightly heavier than normal, but her preceding two periods were scanty and "off schedule." Her medications include a thiazide diuretic for mild hypertension and a senior multivitamin. She is 62" tall and weighs 198#. Assuming her examination demonstrates candidiasis, this woman needs most urgently to be screened for which of the following? A diabetes mellitus B hyperlipidemia C hypothyroidism D perimenopause E polycystic ovarian syndrome
The Correct Answer is: A Diabetes causes recurrent vaginal candidiasis in women with uncontrolled glucose. Type 2 diabetes is further suggested by her obesity and history of macrosomia in offspring. Furthermore, thiazide diuretics may increase glucose levels. She needs also to be screened for hyperlipidemia (B), given her weight and probable diabetes, but that is not as urgent. She may be hypothyroid (C), although the clinical picture more strongly suggests diabetes; hypothyroidism is also more likely to cause menorrhagia than scanty periods. She may also be perimenopausal (D), given irregular menses. Polycystic ovarian syndrome (E) is unlikely given no history of infertility. Masharani U, Diabetes Mellitus & Hypoglycemia, in Current Medical Diagnosis and Treatment, 52 nd ed. 2013.
Q 18.7: Patients in which of the following age groups are least likely to experience a dislocation or sprain when a significant stress is placed on their joints? A 5-10 years old B 15-20 years old C 35-40 years old D 55-60 years old E 75-80 years old
The Correct Answer is: A Dislocations and ligamentous injuries are uncommon in prepubertal children as the ligaments and joints are quite strong as compared to the adjoining growth plates. Excessive force applied to a child's joint is more likely to cause a fracture through the growth plate than a dislocation or sprain.
Q 91.9: Which of the following antineoplastic medications is most likely to cause cardiac toxicity and precipitate heart failure? A doxorubicin B cisplatin C cyclophosphamide D tamoxifen E 6-mercaptopurine
The Correct Answer is: A Doxorubicin is a common antineoplastic drug used for a variety of cancers, including breast, bladder, ovarian, and endometrial, among many others. Unfortunately, it has a well-established, dose-dependent adverse effect on the heart that is linked to free-radical formation. (Medina and Fausel, 2008, p. 2099) Medina PJ , Fausel C. Cancer treatment and chemotherapy. In: DiPiro JT , Talbert RL , Yee GC, et al., eds. Pharmacotherapy: A Pathophysiologic Approach. 7th ed. New York: McGraw-Hill; 2008.
Q 101.1: While you are doing a funduscopic exam on an 80-year-old female with progressive vision loss, you notice drusen formations on her retinas. What is the most likely diagnosis? A Macular degeneration B Retinal detachment C Central retinal artery occlusion D Diabetic retinopathy E Central retinal vein occlusion
The Correct Answer is: A Drusen are yellow colored collagen deposits in Bruch's membrane of the retina. They can be diffuse, discrete, or confluent. Retinal pigment changes and atrophy are see in "dry" macular degeneration. "Wet" macular degeneration demonstrates choroidal neovascularization, or serous retinal pigment hemorrhages and retinal detachments. (Riordan et al., 2008, Chapter 6)
Q 63.6: Which of the following symptoms is most frequently reported in patients that have peptic ulcer disease? A Dyspepsia B Sharp abdominal pain C Relief of pain with food D Pain that awakens the patient at night E Significant vomiting
The Correct Answer is: A Dyspepsia (epigastric pain) is the hallmark of peptic ulcer disease and is present in 80 to 90% of patients. The abdominal pain is typically described as dull, gnawing, aching or "hunger like," but isn't typically described as sharp. Relief of pain with food is reported in about 50% of patients. Furthermoe, 66% of patients with duodenal ulcers and 33% of patients with gastric ulcers report nocturnal pain that awakens them at night. Significant vomiting is not typical of uncomplicated peptic ulcer disease, and is more suggestive of gastric outlet obstruction or gastric malignancy.
Q 32.33: You are getting ready to evaluate a patient who has a past history of gout. The chief complaint is that the patients gout has flared up again, causing pain. Based on your knowledge of gout, which joint is most commonly the site of an initial gout attack? A 1st metatarsaphalangeal joint B Knee C Wrist D Elbow E Ankle
The Correct Answer is: A Eighty percent of gout attacks affect only one joint. The most common joint to be involved is the 1st metatarsal phalangeal joint. This common phenomenon is called podagra. Gout can affect other joints as well, including the knee, ankle and tarsal joints of the foot. Upper extremities are not commonly affected by gout. The knee is the most common joint to be affected by pseudogout followed by the wrist, MCP joints of the hands, hips, shoulders, elbows and spine.
You are getting ready to evaluate a patient who has a past history of gout. The chief complaint is that the patients gout has flared up again, causing pain. Based on your knowledge of gout, which joint is most commonly the site of an initial gout attack? A 1st metatarsaphalangeal joint B Knee C Wrist D Elbow E Ankle
The Correct Answer is: A Eighty percent of gout attacks affect only one joint. The most common joint to be involved is the 1st metatarsal phalangeal joint. This common phenomenon is called podagra. Gout can affect other joints as well, including the knee, ankle and tarsal joints of the foot. Upper extremities are not commonly affected by gout. The knee is the most common joint to be affected by pseudogout followed by the wrist, MCP joints of the hands, hips, shoulders, elbows and spine.
Q 98.15: A 45-year-old man presents to the office with the complaint of perianal pain and bleeding. Examination reveals an anal/perianal mass complex. Biopsy is taken and the results are positive for epidermoid carcinoma of the anus. Which of the following treatment options would be the most appropriate therapy for this patient? A Local resection, chemotherapy, and external beam radiation B Abdominoperineal resection C Chemotherapy only D Local resection only E Radiation only
The Correct Answer is: A Epidermoid carcinoma of the anal canal is a slow-growing tumor that often presents as an anal or perianal mass. Wide excision, followed by 5-fluorouracil, mitomycin, and external beam radiation (Nigro Protocol), typically results in a greater than 80% cure rate. The presence of inguinal lymph node metastasis is a poor prognostic indicator. Management of recurrences is typically achieved by performing an abdominoperineal resection.
Q 69.1: A 45-year-old man presents to the office with the complaint of perianal pain and bleeding. Examination reveals an anal/perianal mass complex. Biopsy is taken and the results are positive for epidermoid carcinoma of the anus. Which of the following treatment options would be the most appropriate therapy for this patient? A Local resection, chemotherapy, and external beam radiation B Abdominoperineal resection C Chemotherapy only D Local resection only E Radiation only
The Correct Answer is: A Epidermoid carcinoma of the anal canal is a slow-growing tumor that often presents as an anal or perianal mass. Wide excision, followed by 5-fluorouracil, mitomycin, and external beam radiation (Nigro Protocol), typically results in a greater than 80% cure rate. The presence of inguinal lymph node metastasis is a poor prognostic indicator. Management of recurrences is typically achieved by performing an abdominoperineal resection. (Bullard, 2010, p. 1053) Bullard Dunn KM , Rothenberger DA. Colon, rectum and anus. In: Brunicardi FC, et al., eds. Schwartz's Principles of Surgery. 9th ed. New York, NY: McGraw-Hill; 2010.
Q 97.52: A 29-year-old woman comes in for evaluation of an increased vaginal discharge for the past week. She describes it as "sort of whitish gray with a disgusting odor." She has no other symptoms. She has had no new sexual partners, has taken no antibiotics, and has not used any new hygiene products. Examination reveals no vulvar erythema. She does have an adherent whitish discharge in the vaginal vault, but no vaginal erythema. Whiff test of the secretions is positive. Microscopic examination of saline and potassium hydroxide preparations is most likely to reveal which of the following? A Clue cells B Increased polymorphonucleocytes C Motile flagellates D Small, rounded parabasal epithelial cells E Spores and filaments
The Correct Answer is: A Examination of vaginal secretions in a woman with bacterial vaginosis demonstrates the presence of clue cells, which are epithelial cells that appear granulated due to G vaginalis cells adhering to them. Increased PMNs (B) may be present in a number of vaginal conditions, but not typically in bacterial vaginosis. Motile flagellates (C) are present in vaginal trichomoniasis, while small parabasal epithelial cells (D) are present in atrophic vaginitis. Spores and filaments (E) (pseudohyphae) are typical of candidiasis.
Q 101.2: A 51-year-old female undergoing chemotherapy for stage IIC breast cancer presents to her primary care provider with a complaint of fatigue, becoming tired very easily, and an inability to perform any of her normal daily activities. What is the most common reversible cause of chemotherapeutic -related fatigue? A anemia B anxiety C chronic pain D neutropenia with sepsis E sleep disturbance
The Correct Answer is: A Fatigue is one of the most common symptoms in cancer patients, experienced by 70% to 100% of those receiving cancer treatment. It is most commonly related to the chemotherapeutic agent itself and will resolve when treatment is completed. In the interim, anemia is the most common cause and this can be treated with hematopoietic growth factors, erythropoietin (epoetin alfa and epoetin beta), granulocyte colony-stimulating factor (G-CSF), granulocyte-macrophage colony-stimulating factor (GM-CSF). (Kantarjian, et al., 2006, Chapter 41)
Q 31.6: A 40- year-old female patient comes to the office because she notices that she is easily fatigued but cannot pinpoint a direct cause of her fatigue. She has experienced muscle tenderness to the shoulders and other large muscle groups, sensitivity to touch these areas, and has also felt depressed lately. Based on this history and the vague physical exam findings, what is the most likely diagnosis? A Fibromyalgia B Polyarteritis nodosa C Polymyositis D Scleroderma E Sjogren's
The Correct Answer is: A Fibromyalgia syndrome (FMS) primarily affects woman between the ages of 20 and 60 and is now the second most common condition seen in rheumatologists' offices behind only rheumatoid arthritis. This condition spares the joints, but causes tender areas throughout the soft tissue as well as generalized pain and fatigue. There are several common trigger points, both anteriorly and posteriorly that help confirm the diagnosis. Polyarteritis nodosa is a systemic necrotizing vasculitis that generally affects medium size muscular arteries. Patients can present with many of the same symptoms as those with fibromyalgia (fatigue, muscle pain, and others), but these patients often have skin manifestations, joint pain, and fever, along with renal, GI, and cardiovascular signs not seen in fibromyalgia. Polymyositis is an inflammatory condition of the muscles that is characterized by proximal muscle weakness, with much less pain than is seen in fibromyalgia. Despite the proximal muscle weakness there is generally not significant atrophy. Sjogren's syndrome does not generally present with any muscular manifestations. It is a chronic inflammatory disorder that affects the salivary and lacrimal glands resulting in dry eyes and a dry mouth. Scleroderma is classically associated with thickened and hardened skin, but it can have internal organ involvement as well affecting many different body systems. When it affects the musculoskeletal system it tends to affect joints and the areas where tendons cross joints. It can cause contractures, pain, and swelling as well as fatigue and weakness.
Q 97.58: A 40- year-old female patient comes to the office because she notices that she is easily fatigued but cannot pinpoint a direct cause of her fatigue. She has experienced muscle tenderness to the shoulders and other large muscle groups, sensitivity to touch these areas, and has also felt depressed lately. Based on this history and the vague physical exam findings, what is the most likely diagnosis? A Fibromyalgia B Polyarteritis nodosa C Polymyositis D Scleroderma E Sjogren's
The Correct Answer is: A Fibromyalgia syndrome (FMS) primarily affects woman between the ages of 20 and 60 and is now the second most common condition seen in rheumatologists' offices behind only rheumatoid arthritis. This condition spares the joints, but causes tender areas throughout the soft tissue as well as generalized pain and fatigue. There are several common trigger points, both anteriorly and posteriorly that help confirm the diagnosis. Polyarteritis nodosa is a systemic necrotizing vasculitis that generally affects medium size muscular arteries. Patients can present with many of the same symptoms as those with fibromyalgia (fatigue, muscle pain, and others), but these patients often have skin manifestations, joint pain, and fever, along with renal, GI, and cardiovascular signs not seen in fibromyalgia. Polymyositis is an inflammatory condition of the muscles that is characterized by proximal muscle weakness, with much less pain than is seen in fibromyalgia. Despite the proximal muscle weakness there is generally not significant atrophy. Sjogren's syndrome does not generally present with any muscular manifestations. It is a chronic inflammatory disorder that affects the salivary and lacrimal glands resulting in dry eyes and a dry mouth. Scleroderma is classically associated with thickened and hardened skin, but it can have internal organ involvement as well affecting many different body systems. When it affects the musculoskeletal system it tends to affect joints and the areas where tendons cross joints. It can cause contractures, pain, and swelling as well as fatigue and weakness.
Q 111.1: A 42-year-old African American male is admitted to the hospital with heme positive urine and anemia. He recently completed a course of trimethoprim/sulfamethoxazole for a urinary tract infection. What is the most likely cause of these findings? A Glucose-6-phophate dehydrogenase deficiency B Factor VIII disorder C Idiopathic thrombocytopenic purpura D Sickle cell anemia E Thalassemia
The Correct Answer is: A G6PD deficiency is an X-linked recessive disorder affecting 10 to 15% of African American males. Hemolysis occurs as a result of oxidative stress on the red cells after exposure to infection or certain drugs, and leads to a hemolytic episode. Common drugs include sulfonamides. Factor VIII and ITP are coagulation disorders, not anemias. Sickle cell and thalassemia are not transient and do not usually present this late in life. (McPhee SJ, Papadakis MA, Tierney LM. Current Medical Diagnosis and Treatment, 2010, Chapter 13, Blood Disorders)
Q 67.2: Assuming that a patient has maintained a normal baseline creatinine of 1.0 mg/dL with a normal glomerular filtration rate (GFR) of 100 mL/min, which of the following indicates a more significant change in the GFR? A increase in creatinine from 1.0 to 2.0 mg/dL B increase in creatinine from 2.0 to 4.0 mg/dL C increase in creatinine from 4.0 to 8.0 mg/dL D increase in creatinine from 8.0 to 16.0 mg/dL
The Correct Answer is: A GFR describes the amount of blood passing through the kidneys per minute. There is an inverse relationship between GFR and serum creatinine. In a patient with normal renal function, doubling of the serum creatinine represents a loss of approximately 50% of GFR. Using this information, the loss of GFR can be estimated from changes in the serum creatinine. For example, assume normal creatinine levels of 1.0 mg/dL and normal GFR of 100 mL/min. A doubling of the serum creatinine from 1.0 mg/dL to 2.0 mg/dL represents an approximate reduction in GFR from 100 mL/min to 50 mL/min (50% of GFR has been lost). Each additional doubling of the creatinine decreases the remaining GFR by approximately one half. When renal function is severely impaired, large increases in the creatinine (ie, from 8.0 to 16.0 mg/dL) represent only small decreases in GFR (from about 12 to 6 mL/min). This example emphasizes the importance of detecting increases in serum creatinine early. However, serum creatinine level does not become abnormal until ˜25% of renal function is lost. Therefore, other methods of estimating GFR are more useful in detecting early decreases in GFR. (Levey, 1999; Stevens and Perrone, 2008) Levey AS. Bosch JP , Lewis JB , et al. A more accurate method to estimate glomerular filtration rate from serum creatinine: a new prediction equation. Ann Intern Med. 1999;130:461-470. 10075613 Stevens L , Perrone RD. Assessment of kidney function: serum creatinine; BUN; and GFR. In Rose BD, ed. 2008. http://www.uptodate.com , version 16.3
Q 34.9: A 3-day-old infant has bilateral copious, yellow-green eye discharge and conjunctival inflammation. A Gram stain of this discharge reveals gram-negative intracellular diplococci. Which of the following antibiotics is the drug of choice for this infection? A ceftriaxone B cephalexin C erythromycin D gentamicin
The Correct Answer is: A Gonococcal ophthalmia neonatorum presents as a unilateral or bilateral serosanguineous discharge and then within 24 hours the discharge becomes mucopurulent, followed by conjunctival injection and edema of the eyelids. The usual incubation period for Neiserria gonorrhea is 2 to 5 days; however, the infection may be present at birth or delayed greater than 5 days if there has been instillation of silver nitrate prophylaxis. A presumptive diagnosis is made by the demonstration of gram-negative intracellular diplococci on Gram stain. Definitive diagnosis is made by culture. Following a positive Gram stain and pending culture results, treatment should be promptly initiated with ceftriaxone (50 mg/kg/24 hours IV or IM for one dose not to exceed 125 mg), a third-generation cephalosporin with good coverage for gram-negative bacteria. An alternate drug is cefotaxime (100 mg/kg/24 hours IV or IM every 12 hours for 7 days or 100 mg/kg as a single dose), which is also a third-generation cephalosporin. Although erythromycin drops (0.5%) are used prophylactically for N gonorrhea, this is not an effective treatment. Gentamicin would be used for Pseudomonas, and Chlamydia is treated with erythromycin. Cephalexin as a first-generation cephalosporin does not have coverage for gram-negative bacteria.
A 45-year-old obese male presents with pain and swelling to his left knee. It is reddened and tender to touch. He states that this has happened before but the swelling had gone down over the course of several days. Anti-inflammatories have helped some in the past. An analysis of the arthrocentesis fluid reveals needle-shaped crystals. Based on the results of the fluid analysis found within the synovial fluid, what is the most likely diagnosis? A Gout B Lupus C Osteoarthritis D Pseudogout E Rheumatoid arthritis
The Correct Answer is: A Gout is a condition found in people that have hyperuricemia (but not all people with hyperuricemia will develop the symptoms of gout). Hyperuricemia is defined as an elevated uric acid level in the blood. Uric acid is a product of purine metabolism. Gout occurs when needle-shaped monosodium urate crystals deposit in soft tissue, which leads to pain and inflammation at the deposition site. The diagnosis can be confirmed by analyzing joint aspirate to identify the classic negatively birefringent urate crystals that have a needle-like shape. Pseudogout can present with a similar clinical picture, but an analysis of joint aspirate shows calcium pyrophosphate crystals that are weakly positive, birefringent rhomboid shaped crystals. Lupus, osteoarthritis, and rheumatoid arthritis all can cause joint inflammation and pain, but none would show any significant crystal deposition during an analysis of a synovial fluid aspirate.
Q 25.5: A 45-year-old obese male presents with pain and swelling to his left knee. It is reddened and tender to touch. He states that this has happened before but the swelling had gone down over the course of several days. Anti-inflammatories have helped some in the past. An analysis of the arthrocentesis fluid reveals needle-shaped crystals. Based on the results of the fluid analysis found within the synovial fluid, what is the most likely diagnosis? A Gout B Lupus C Osteoarthritis D Pseudogout E Rheumatoid arthritis
The Correct Answer is: A Gout is a condition found in people that have hyperuricemia (but not all people with hyperuricemia will develop the symptoms of gout). Hyperuricemia is defined as an elevated uric acid level in the blood. Uric acid is a product of purine metabolism. Gout occurs when needle-shaped monosodium urate crystals deposit in soft tissue, which leads to pain and inflammation at the deposition site. The diagnosis can be confirmed by analyzing joint aspirate to identify the classic negatively birefringent urate crystals that have a needle-like shape. Pseudogout can present with a similar clinical picture, but an analysis of joint aspirate shows calcium pyrophosphate crystals that are weakly positive, birefringent rhomboid shaped crystals. Lupus, osteoarthritis, and rheumatoid arthritis all can cause joint inflammation and pain, but none would show any significant crystal deposition during an analysis of a synovial fluid aspirate.
A 23-year-old college basketball player twists her ankle while practicing. She explains the injury that is consistent with an inversion mechanism. Based on this history, what ligament would you expect to be the most likely injured in the ankle of this patient? A Anterior talofibular B Anterior tibiofibular C Calcaneofibular D Deltoid E Posterior talofibular ligament
The Correct Answer is: A Greater than 25,000 ankle sprains happen in the USA every day and the vast majority of those are inversion ankle sprains. The anterior talofibular ligament is the first, and often only, ligament damaged in inversion ankle sprains. As the force of the inversion increases, other lateral ankle ligaments can be involved. When the anterior tibiofibular ligament is involved, this is referred to as a high ankle sprain and such injuries generally have a prolonged recovery time. Calcaneofibular ligaments are generally the second most frequently injured of the lateral ankle ligaments and when injury occurs it is typically in combination with the anterior talofibular ligament. The deltoid ligament is a very strong ligament on the medial aspect of the ankle. Eversion stresses the deltoid ligament, but strong eversion forces are rare and when they do occur, an avulsion fracture of the medial malleolus is more likely than a significant ligament tear. The posterior talofibular ligament is one of the lateral ankle ligaments and can be injured in an inversion injury, but the rate of injury to this ligament lags far behind the anterior talofibular or calcaneofibular ligaments.
Q 19.8: A 23-year-old college basketball player twists her ankle while practicing. She explains the injury that is consistent with an inversion mechanism. Based on this history, what ligament would you expect to be the most likely injured in the ankle of this patient? A Anterior talofibular B Anterior tibiofibular C Calcaneofibular D Deltoid E Posterior talofibular ligament
The Correct Answer is: A Greater than 25,000 ankle sprains happen in the USA every day and the vast majority of those are inversion ankle sprains. The anterior talofibular ligament is the first, and often only, ligament damaged in inversion ankle sprains. As the force of the inversion increases, other lateral ankle ligaments can be involved. When the anterior tibiofibular ligament is involved, this is referred to as a high ankle sprain and such injuries generally have a prolonged recovery time. Calcaneofibular ligaments are generally the second most frequently injured of the lateral ankle ligaments and when injury occurs it is typically in combination with the anterior talofibular ligament. The deltoid ligament is a very strong ligament on the medial aspect of the ankle. Eversion stresses the deltoid ligament, but strong eversion forces are rare and when they do occur, an avulsion fracture of the medial malleolus is more likely than a significant ligament tear. The posterior talofibular ligament is one of the lateral ankle ligaments and can be injured in an inversion injury, but the rate of injury to this ligament lags far behind the anterior talofibular or calcaneofibular ligaments.
Q 38.10: A 5-year-old child presents for her kindergarten checkup. The clinician notes that over the past couple of years, her height decreased from the 50th percentile to the 5th percentile. On examination, the clinician also notes truncal adiposity. Her CBC and lead levels were normal. Which of the following is the most likely diagnosis? A growth hormone deficiency B Cushing disease C congenital hypothyroidism D congenital adrenal hyperplasia
The Correct Answer is: A Growth hormone (GH) deficiency is defined as a decreased growth velocity, delay in skeletal maturation, absence of other explanations for poor growth (lack of intake), and laboratory tests demonstrating decreased GH secretion. Etiology of GH deficiency can be congenital, genetic, acquired, or idiopathic, which is the most common. Infants usually have a normal birth weight and may have a slightly decreased length. In addition, most infants present with other endocrine deficiencies like hypoglycemia, hypothyroidism, and/or adrenal insufficiency. Children may present with truncal adiposity because growth hormone promotes lipolysis. Serum GH or intrinsic growth factor levels may or may not be decreased. In patients who do not have a demonstrated decrease in these hormones, a trial period with GH is indicated. These patients and positive GH-deficient patients receive a once-daily subcutaneous injection of recombinant human GH. Congenital hypothyroidism typically presents with short stature (typically noted after the 4-month newborn visit), delayed epiphyseal development, delayed closure of fontanelles, and retarded dental eruption in addition to other signs of hypothyroidism. Cushing disease typically presents with truncal adiposity with thin extremities, muscle wasting, decreased growth rate, and moon facies. Laboratory results show elevated adrenocorticosteroids both in urine and serum, hypokalemia, eosinopenia, and lymphocytopenia. Typically, in patients younger than the age of 12, Cushing disease is secondary to administration of ACTH or glucocorticoids. Congenital adrenal hyperplasia typically presents with pseudohermaphroditism in females or salt-losing crisis in males with or without isosexual precocity. There is an increased linear growth and advanced skeletal maturation.
Q 97.34: Which of the following tests should an HIV-positive pregnant woman undergo in each trimester of pregnancy? A CD4+ lymphocyte count B cytomegalovirus serology C postpartum depression with controls D shielded chest radiography E venereal disease research laboratory (VDRL)
The Correct Answer is: A HIV-positive pregnant women should undergo CD4+ serology each trimester. Early in the pregnancy, they should undergo shielded chest radiography, CMV baseline testing, and tuberculosis testing with controls. Syphilis testing should be completed initially and as usually recommended later in pregnancy.
Q 97.55: You are providing care for a 28-year-old who presents to clinic for her 27-week obstetrical check. She has no complaints and the fetus is active. Physical exam reveals the following: P 88, BP 142/90, FHT 148, UA negative. CBC is normal. Her BP pre-partum was 110/70. What is the most likely diagnosis at this point? A Pregnancy induced hypertension B Preeclampsia C Eclampsia D Chronic hypertension E stress
The Correct Answer is: A HTN is one of the triad of symptoms of preeclampsia, but at this point there are no other symptoms or proteinuria. She will need to be carefully monitored, as 50% of these presentations will progress to preeclampsia.
Q 39.14: A 9-year-old girl is brought to the office by her mother who has noticed that the child's neck looks "odd." The girl states that it feels "like something is there," indicating the base of the neck, but that she has no difficulty swallowing. A thorough examination reveals only a firm, nontender, and diffusely enlarged thyroid with no thrills or bruits. All parameters on a thyroid function panel are within normal limits. Low levels of thyroid antibodies are present. What is this child's most likely diagnosis? A Chronic lymphocytic thyroiditis (Hashimoto thyroiditis) B Graves disease C Papillary carcinoma of the thyroid D Pituitary tumor E Subacute thyroiditis (deQuervain thyroiditis)
The Correct Answer is: A Hashimoto thyroiditis, the most common thyroid disorder in the United States, is an autoimmune disease. In children, girls 8-15 years of age are most commonly affected. Graves disease is most common in women ages 20-40. The thyroid enlargement is diffusely enlarged but may be asymmetric and may be accompanied by a bruit. In addition, the patient will have signs and symptoms of hyperthyroidism. Thyroid antibodies and hormones are elevated and TSH suppressed. Thyroid carcinoma is also more likely in women and papillary carcinomas are the post common. They present as a firm nodule in the gland, most often between 10 and 40 years after exposure to radiation. Thyroid-hormone-secreting pituitary tumors (rare) usually present with signs and symptoms of hyperthyroidism without enlargement of the gland and normal or high serum TSH levels. DeQuervain thyroiditis presents with signs and symptoms of hyperthyroidism and an enlarged tender gland.
Q 73.4: A 27-year-old female presents to the emergency department with severe RUQ pain. Ultrasonography reveals gallstones. In preparation for a potential cholecystectomy, a CBC is obtained that reveals a normocytic anemia with an increased mean corpuscular hemoglobin concentration (MCHC). She is slightly jaundiced and you are able to palpate her spleen on examination. What is her underlying diagnosis? A Hereditary spherocytosis B Iron deficiency anemia C Sickle cell anemia D Thalassemia E Von Willebrand's disease
The Correct Answer is: A Hereditary Spherocytosis (HS) is characterized by jaundice, an enlarged spleen, and often gallstones; gallstones are more frequently seen in young people, triggering the investigation into HS. An increased MCHC is a characteristic feature of HS and is almost the only condition in which this finding is seen. Iron deficiency anemia does not have an increased MCHC. Sickle Cell has characteristic findings different than the presenting findings. Thalassemia is noted for a microcytosis and Von Willebrand's is a coagulation disorder. (Fauci, et al, Harrison's Principles of Internal Medicine, 17e, Chapter 101, Hemolytic Anemias and Anemia Due to Acute Blood Loss)
Q 113.1: An 18-year-old female presents with a history of draining abscesses in the axilla and groin, with large, open comedones. Currently, she has multiple scars in the axillae. What is the most likely diagnosis? A hidradenitis suppurtiva B nodulocystic acne C acanthosis nigricans D methicillin resistant staphylococcus aureus (MRSA)
The Correct Answer is: A Hidradenitis suppurtiva affects females more than males, and may show a family history of nodulocystic acne and/or hidradenitis suppurtiva. Skin lesions are usually tender nodules and abscesses that may spontaneously drain. Open comedones, including double comedones, are common. Eventually, sinus tracts may form. Nodulocystic acne consists of nodules and cysts, ranging in size from 1 to 4 cm in diameter. These lesions are distributed on the face, back, and chest. Acanthosis nigricans is described as a velvety, hyperpigmented plaque distributed around the neck, in the axillae, and in the groin. MRSA can have several different presentations, ranging from erythrasma to the presence of papules and pustules. Comedones are not associated with MRSA. (Wolff et al., 2009, Page 16)
Q 97.23: A 22-year-old sexually active woman presents for her annual gynecologic evaluation. She reports one partner for the past 6 months and takes oral contraceptive pills as directed. Her periods have been regular. Her examination is unremarkable and her Pap smear returns with atypical squamous cells of undetermined significance and positive for human papillomavirus-16. The next most appropriate step for this patient is to A proceed with colposcopy B repeat Pap smear in 12 months C repeat Pap smear in 24 months D schedule her for a loop electrosurgical excision procedure (LEEP)
The Correct Answer is: A Human papillomavirus subtypes 6, 11, 16, and 18 increase risk for the development of cervical cancer. In a young woman over 21 years old with atypical squamous cells of undetermined significance and positive HPV 16 subtype, the next step in evaluation is the colposcopic evaluation. Alternatively, she could be followed with Pap smears at 6 and 12 months. The LEEP procedure is indicated for those with recurrent histologic finding of cervical intraepithelial neoplasm grade 2 or 3.
Q 101.9: A 42-year-old male exhibits an unstable, dance-like gait, dystonia, irritability, and depression, which is limiting his ability to work. His older sister has similar symptoms. What is the most likely diagnosis? A Huntington's disease B Wilson's disease C Medication-induced chorea D Parkinson's disease E Hyperbilirubinemia
The Correct Answer is: A Huntington's disease, an inherited nervous system disorder seen in families, typically arises in adulthood. It progresses to include mental and motor disturbances. Classic findings include chorea of various severity, dyskinesia, progressive rigidity, and mental changes such as irritability, depression, antisocial personality, and suicidality. This must be differentiated from other nervous system disorders, as well as other causes of chorea and nervous system impairment. (McPhee SJ, Papadakis MA. Current Medical Diagnosis & Treatment 2011, Chapter 24, Nervous System Disorders)
Q 35.5: A patient presents with hyperleukocytosis (WBC 100,000). The bone marrow is consistent with T-cell ALL. The uric acid is less than 7.0 mg/dl. Prior to starting chemotherapy, what is the best therapy to initiate on this patient? A Allopurinol and IV fluids B Aluminum hydroxide and calcium carbonate C Glucocorticoids and vincristine D Leukapheresis and cranial irradiation E Sevelamer and mercaptopurine
The Correct Answer is: A Hyperuricemia is often a finding in patients with hyperleukocytosis. The optimal treatment is to start IV fluids, due to numbers of circulating white cells and allopurinol, to treat hyperuricemia. The use of aluminum hydroxide is appropriate if they have hyperphosphatemia, and calcium carbonate if they have a low serum calcium concentration. Glucocorticoids and vincristine are used with hyperleukocytosis of >400,000. Leukapheresis and cranial irradiation are used for patients with extreme leukocytosis of >400,000. Sevelamer is used to treat hyperphosphatemia, and mercaptopurine is a byproduct produced during production of leukemic cells.
Q 64.7: A 58-year-old man is in the hospital postoperative day 3 after a laparoscopic right colon resection. Your morning labs reveal a serum potassium level of 2.9 mEq/L (normal 3.5-5.0 mEq/L) despite aggressive potassium replacement during the previous shift. At this time you should check which of the following laboratory values? A Magnesium B Calcium C Phosphorous D Sodium E Albumin
The Correct Answer is: A Hypokalemia is a common electrolyte disturbance in surgical patients. It can be caused by enhanced losses, hyperaldosteronism, inappropriate replacement, and intracellular shifts caused by alkalosis. Symptoms of hypokalemia may include constipation, neuromuscular weakness, diminished tendon reflexes, paralysis, and distinctive electrocardiographic changes. Concomitant deficiencies in magnesium can contribute significantly to the development of hypokalemia as well as hypocalcemia. In the surgical patient with persistent hypokalemia refractory to potassium administration, one should check magnesium levels and correct as appropriate.
Q 98.35: A 58-year-old man is in the hospital postoperative day 3 after a laparoscopic right colon resection. Your morning labs reveal a serum potassium level of 2.9 mEq/L (normal 3.5-5.0 mEq/L) despite aggressive potassium replacement during the previous shift. At this time you should check which of the following laboratory values? A Magnesium B Calcium C Phosphorous D Sodium E Albumin
The Correct Answer is: A Hypokalemia is a common electrolyte disturbance in surgical patients. It can be caused by enhanced losses, hyperaldosteronism, inappropriate replacement, and intracellular shifts caused by alkalosis. Symptoms of hypokalemia may include constipation, neuromuscular weakness, diminished tendon reflexes, paralysis, and distinctive electrocardiographic changes. Concomitant deficiencies in magnesium can contribute significantly to the development of hypokalemia as well as hypocalcemia. In the surgical patient with persistent hypokalemia refractory to potassium administration, one should check magnesium levels and correct as appropriate.
Q 107.7: A 45-year-old woman presents with weight gain, fatigue, dry skin, constipation, and oligomenorrhea. On physical exam, bradycardia and slow deep tendon reflexes are noted. Her free T 4 is low and TSH is elevated. Which of the following medications may be responsible for her condition? A amiodarone B beta-blockers C levadopa D hydrocholorthiazide
The Correct Answer is: A Hypothyroidism is reported in up to 10% of patients taking amiodarone, an antiarrhythmic medication. With the high iodine content of the medication and the structural similarities to thyroxine, thyroid abnormalities occur. Common side effects of amiodarone include bradycardia and constipation, so laboratory evaluation for thyroid dysfunction must be used. (Fitzgerald, 2009, p. 978) Fitzgerald PA. Endocrine diseases. In: McPhee SJ , Papadakis MA, eds. Current Medical Diagnosis and Treatment. 48th ed. New York, NY: McGraw-Hill; 2009.
Q 115.4: A 24-year-old man with a recent history of a viral illness comes to the emergency room complaining of severe left-sided chest discomfort, which radiates through to the left trapezius region. On coming into the room, you note that he is sitting up and hunched forward. On physical examination, the patient's temperature is 39°C, blood pressure is 135/78, with a pulse of 85 bpm, and a pericardial friction rub is noted. Laboratory findings demonstrate elevated serum creatine kinase levels and normal serial troponin levels. Which of the following would be the most likely electrocardiographic findings? A Diffuse ST segment elevation B Peaked T waves C Inferior Q waves D Loss of R-wave amplitude E U waves
The Correct Answer is: A In a patient with these signs, symptoms, and lab findings, acute pericarditis is the most likely diagnosis. In patients with acute pericarditis, EKG changes occur secondary to inflammation of the subepicardium, leading to widespread elevation of the ST segments, often with upward concavity, which returns to normal after several days, followed by T wave inversion. No significant QRS complex changes are noted, so choice C, the development of inferior Q waves (frequently associated with an inferior myocardial infarction), is incorrect. Choice B is frequently noted with severe hyperkalemia. Loss of R-wave amplitude, choice D, is associated with myocardial infarction. Choice E, U waves, are associated with hypokalemia. (Fauci et al., 2001, pp. 1289-1270, 1366)
Q 97.32: A patient seen at the prenatal clinic develops Graves disease at 25 weeks' gestation. Which of the following is the most appropriate treatment? A PTU 100 mg po tid B methimazole 10 to 30 mg po qd C propranolol 80 mg po qid D radioactive iodine therapy (RAI, 131I) E levothyroxine 0.1 mg po qd
The Correct Answer is: A In nonpregnant patients, PTU and methimazole are the drugs of choice for the management of Graves disease. During pregnancy, PTU has a lower incidence of crossing the placental barrier than does methimazole. It also is excreted into breast milk to a lesser degree than is methimazole. Propranolol will help with the symptoms of Graves but not treat it. It can also cause low birth weight in the infant. RAI is contraindicated in pregnancy. Levothyroxine will worsen a Graves patient's hyperthyroidism.
Q 113.7: Septic arthritis in adults younger than 30 years is usually caused by A Neisseria gonorrhea B Staphylococcus aureus C Pseudomonas aeruginosa D Streptococcus pyogenes E Salmonella species
The Correct Answer is: A In patients younger than 30 years, gonococcus is the most common cause of septic arthritis. When all patients are considered, Staphylococcus aureus is the most common cause. Patients with prevalent joint disease and intravenous drug users are especially susceptible to Staphylococcus. Pseudomonas is also a common cause of septic arthritis in intravenous drug users. Salmonella is not a common cause of joint infection. (Lange and Lederman, 2007, p. 966) Lange C , Lederman MM. Infections involving bones and joints. In: Andreoli TE , Carpenter CCJ , Griggs RC, et al., eds. Andreoli and Carpenter's Cecil Essentials of Medicine. 7th ed. Philadelphia, PA: Saunders Elsevier; 2007.
A 50-year-old woman with a history of hypertension complains of chest tightness and dyspnea while walking up one flight of stairs. She recently experienced an episode of near-syncope while walking her dog. She denies a history of rheumatic fever. On auscultation, a crescendo-decrescendo systolic ejection murmur is heard at the upper right sternal border radiating to the carotids bilaterally. Troponin levels are negative at 0, 3, and 6 hours. Her EKG demonstrates evidence of left ventricular hypertrophy. A transthoracic echocardiogram reveals significant aortic stenosis secondary to congenital bicuspid aortic valve, left ventricular hypertrophy, and normal left ventricular systolic function. Which of the following is the most appropriate next step in management? A Cardiac catheterization followed by aortic valve replacement B Monitoring via repeat transthoracic echocardiogram in 6 months C Monitoring via transesophageal echocardiogram in 6 months D Treadmill exercise stress test E Automatic internal cardiac defibrillator placement
The Correct Answer is: A In symptomatic patients demonstrating significant aortic stenosis, aortic valve replacement after cardiac catheterization, to evaluate for coronary artery disease and possible concomitant coronary artery bypass surgery with aortic valve replacement, is indicated. As the patient is demonstrating the classic symptoms of severe aortic stenosis, choice A is the most appropriate next step in management. Choices B and C are thus inappropriate, as the patient is already symptomatic. If the patient were not symptomatic, choice B would be a viable choice compared to choice C, because it is less invasive than transesophageal echocardiogram. Choice D would be inappropriate, as strenuous physical activity should be avoided in patients with severe aortic stenosis. Choice E is appropriate therapy for patients at risk for ventricular tachycardia/fibrillation.
Q 74.4: A 50-year-old woman with a history of hypertension complains of chest tightness and dyspnea while walking up one flight of stairs. She recently experienced an episode of near-syncope while walking her dog. She denies a history of rheumatic fever. On auscultation, a crescendo-decrescendo systolic ejection murmur is heard at the upper right sternal border radiating to the carotids bilaterally. Troponin levels are negative at 0, 3, and 6 hours. Her EKG demonstrates evidence of left ventricular hypertrophy. A transthoracic echocardiogram reveals significant aortic stenosis secondary to congenital bicuspid aortic valve, left ventricular hypertrophy, and normal left ventricular systolic function. Which of the following is the most appropriate next step in management? A Cardiac catheterization followed by aortic valve replacement B Monitoring via repeat transthoracic echocardiogram in 6 months C Monitoring via transesophageal echocardiogram in 6 months D Treadmill exercise stress test E Automatic internal cardiac defibrillator placement
The Correct Answer is: A In symptomatic patients demonstrating significant aortic stenosis, aortic valve replacement after cardiac catheterization, to evaluate for coronary artery disease and possible concomitant coronary artery bypass surgery with aortic valve replacement, is indicated. As the patient is demonstrating the classic symptoms of severe aortic stenosis, choice A is the most appropriate next step in management. Choices B and C are thus inappropriate, as the patient is already symptomatic. If the patient were not symptomatic, choice B would be a viable choice compared to choice C, because it is less invasive than transesophageal echocardiogram. Choice D would be inappropriate, as strenuous physical activity should be avoided in patients with severe aortic stenosis. Choice E is appropriate therapy for patients at risk for ventricular tachycardia/fibrillation. (McPhee et al., 2011, Chapter 10)
Q 22.9: A 57-year-old woman with a history of rheumatic fever is seen complaining of dyspnea while vacuuming her apartment, which has been worsening over the last few months. On physical exam, a possible opening snap, loud S 1 , and a very soft diastolic rumbling murmur is auscultated. When the patient is placed in the left lateral decubitus position, the murmur is accentuated, and heard best at the apex. With inspiration, the murmur does not increase in amplitude. On transthoracic echocardiogram, severe mitral valve stenosis and mitral regurgitation is noted. Which of the following is the most appropriate therapy or treatment for this patient? A Cardiac catheterization, followed by mitral valve replacement B Monitoring via repeat transthoracic echocardiogram in 6 months C Monitoring via transesophageal echocardiogram in 6 months D Treadmill exercise stress test E Automatic internal cardiac defibrillator placement
The Correct Answer is: A In symptomatic patients demonstrating significant mitral valve stenosis, mitral valve replacement after cardiac catheterization,is recommended, to evaluate for associated valvular disease and coronary artery disease. This allows planning for possible concomitant coronary artery bypass surgery, with mitral valve replacement and or other valve replacement, if necessary. As the patient is symptomatic and demonstrates both severe mitral stenosis and mitral regurgitation, choice A is the most appropriate next step in management. Choices B and C are thus inappropriate, as the patient is already symptomatic. If the patient were not symptomatic, choice B would be a more viable choice compared to choice C, because it is less invasive than transesophageal echocardiogram. Choice D would be inappropriate, as strenuous physical activity should be avoided in patients with severe mitral valve stenosis. Choice E is appropriate therapy for patients at risk for ventricular tachycardia/fibrillation.
Q 97.82: A 30-year-old G 2 P 1 woman comes to the emergency department at 37 weeks gestation with a chief complaint of worsening abdominal pain for the past two hours. On examination, the uterus is tense and tender. External fetal monitoring indicates frequent uterine contractions with late decelerations. An ultrasound suggests a retroplacental hemorrhage. What is the most appropriate course of action at this time? A Emergency operative delivery B Expectant management at home C Monitoring in the labor suite with the mother on her side D Tocolytic administration until the infant is 38 weeks gestation E Vaginal delivery with induction
The Correct Answer is: A Indications for emergency cesarean delivery in a woman with placental abruption include fetal heart tracings that are not reassuring. Furthermore, a retroplacental hemorrhage carries a direr prognosis than a subchorionic one. Expectant management (B), monitoring (C), attempting to stop labor (D), and attempting a vaginal delivery (E) put both mother and fetus at severe risk of death.
Q 67.7: Assuming no contraindications, which of the following class of medications is considered the preferred long-term control therapy for asthma? A inhaled corticosteroids B leukotriene antagonists C long-acting B2 agonists D methylxanthines E muscarinic antagonists
The Correct Answer is: A Inhaled corticosteroids (eg, beclomethasone, fluticasone, triamcinolone, etc) are the preferred long-term control therapy for persistent asthma in all patients because of their potency and consistent effectiveness. Low- to medium-dose inhaled corticosteroids offer several advantages over other medications, including the ability to reduce bronchial hyper-responsiveness, improve overall lung function, and reduce severe exacerbations that often lead to emergency department visits and hospitalizations. (Chesnutt et al., 2008, p. 209; Kelly and Sorkness, 2008, pp. 485-486;) Chesnutt MS , Murray JA , Prendergast TJ. Pulmonary disorders. In: Tierney LM Jr , McPhee SJ , Papadakis MA, eds. Current Medical Diagnosis & Treatment. 47th ed. New York: McGraw-Hill; 2008. Kelly HW , Sorkness CA. Asthma. In: DiPiro JT , Talbert RL , Yee GC, et al., eds. Pharmacotherapy: A Pathophysiologic Approach. 7th ed. New York: McGraw-Hill; 2008.
Q 98.3: A 70-year-old man with a history of hypertension, DM Type 2, and hyperlipidemia is seen for preoperative evaluation prior to left knee replacement. On auscultation, a very soft high-frequency decrescendo early diastolic murmur is heard at the upper left sternal border. Which of the following maneuvers would be the most appropriate to choose to increase the intensity of the murmur for better identification? A Isometric hand grip exercise B Listening with the bell at the apex with the patient in the left lateral decubitus position C Inspiration, followed by the patient holding his/her breath D The Valsalva maneuver E Having the patient lie flat with the knees bent
The Correct Answer is: A Isometric hand grip exercises will increase the intensity of the murmur of aortic regurgitation, which is usually described as a high-frequency decrescendo early diastolic murmur heard best at the left upper sternal border or at the right upper sternal border. Radiation, if it occurs, is frequently to the lower left sternal border and the apex. Isometric hand exercises increase arterial and left ventricular pressure, which increases the flow across the aortic valve, thereby increasing the murmur's intensity. Choice B is best used when listening to the murmur of mitral stenosis. Choice C will increase the AP diameter, making it more difficult to hear the murmur. Choice D, the Valsalva maneuver, will decrease the intensity of the murmur. Choice E is the best position for the abdominal exam, especially in males.
Q 44.8: A 16-year-old female has tried topical clindamycin and tretinoin. In addition, she recently finished a six-month course of doxycycline 100 mg bid. She has not noticed much improvement in her acne. The patient continues to complain of large, painful lesions, as well as numerous comedonal lesions. What is the next appropriate step in treatment? A isotretinoin 0.5 mg/kg/day B clotrimazole cream bid C keflex 500 mg bid D elidel cream bid
The Correct Answer is: A Isotretinoin is indicated for nodulocystic acne, as well as acne that is resistant to topical treatments and oral antibiotics. Clotrimazole is an antifungal medication that is indicated for the treatment of cutaneous candidiasis and tinea. Keflex is a first generation Cephalosporin. It is not a first line treatment for acne. Elidel is indicated as a second line treatment for atopic dermatitis in patients over two years of age. It does not treat acne.
Q 49.20: A 4-year-old boy presents with 5 days of fever, conjunctivitis, strawberry tongue, red lips, and injected throat. He has large, swollen, slightly tender lymph nodes in his neck and a peeling rash in the palms and soles. The most likely cause is A Kawasaki syndrome B respiratory syncytial virus C coxsackievirus D fifth disease
The Correct Answer is: A Kawasaki syndrome occurs throughout the world, primarily in children. It is thought to be infectious but the etiologic agent has never been isolated. The syndrome is composed of fever and four of five of the following symptoms: bilateral conjunctivitis, some type of mucous membrane change, a peripheral extremity change, transverse grooves on the nails, a polymorphous rash, and cervical lymph nodes >1.5 cm. It can be complicated by arteritis. Treatment may include aspirin, immune globulin, plasmapheresis, or corticosteroids.
A 36-year-old auto mechanic presents to the emergency department after hurting his back on the job. While lifting an object, he experienced sudden pain in his lower back with radiation to the right buttock. He was initially treated for muscle strain with a nonsteroidal anti-inflammatory drug (NSAID) after x-rays of his lumbosacral spine demonstrated no pathology. He continued to complain of this low back pain now radiating posteriorly down his left leg to the mid-thigh. Physical examination is unremarkable. The most likely diagnosis is A lumbosacral strain B left S1 radiculopathy C cauda equina syndrome D L5-S1 disc herniation E lateral femoral cutaneous neuropathy
The Correct Answer is: A Low back pain is one of the more common presenting neurologic complaints to a primary care provider. Most acute pain syndromes are benign, self-limiting conditions, with pain arising from myofascial sources. Patients with back pain and normal neurologic examinations are unlikely to have any serious underlying pathology and further diagnostic testing is usually unrevealing.
Q 64.2: Which of the following is most useful in diagnosing renal artery stenosis? A magnetic resonance angiography (MRA) B computed tomography (CT) scanning C captopril renal scan D renal artery biopsy E intravenous pyelogram (IVP)
The Correct Answer is: A Magnetic resonance angiography, enhanced with gadolinium, is 99% to 100% sensitive and 71% to 96% specific for diagnosing renal artery stenosis (RAS). This study has largely replaced the captopril renal scan and contrast-enhanced arteriography in diagnosing RAS. The principle behind the captopril renal scan is that ACE inhibitors lower GFR. In a kidney with already-compromised blood flow due to RAS, administration of the ACE inhibitor further decreases GFR in the affected kidney despite maintenance of adequate plasma volume. GFR in the contralateral kidney remains normal. Subsequent injection of a radionuclide reveals delayed uptake in the compromised kidney. Although arteriography provides the most definitive diagnosis, it carries its own risks of contrast-induced injury and bleeding. MRA is a low-risk procedure due to its noninvasive nature. Renal artery biopsy would not yield this diagnosis. IVP is utilized to visualize the anatomical structure of the urinary tract in situations such as urinary tract trauma and outflow obstruction, although increasingly it too is being replaced by noninvasive testing, such as ultrasound, CT scanning, and MRI. It remains a useful test to pinpoint the location of a calculus in the urinary tract.
Q 97.33: A 17-year-old female presents to your office with intermittent menstrual pain. She is sexually active with a single male partner, consistently utilizing condoms. She reports that she has had this pain before, most commonly two weeks before her period, and that it has been increasing in severity over the past few months. You perform a pelvic examination and she has no lesions, discharge, or discomfort on bimanual and speculum exam. She is urinary chorionic gonadotropin (UCG) negative. Of the following, what is the most appropriate treatment for this adolescent? A Tell her she has mittelschmerz and prescribe an anti-inflammatory p.r.n. B Tell her she has primary dysmenorrhea and send her home on pain medications. C Recommend that she see a gynecologist to have an exploratory laparoscopic surgery to rule out endometriosis. D Refer her for a vaginal ultrasound to rule out uterine fibroids. E Draw a serum β-hCG.
The Correct Answer is: A Midcycle pain (mittelschmerz) is common in women with regular menstrual periods who are not taking birth control pills. These patients may commonly have midcycle spotting caused by an estrogen surge. There is no fever and no other abnormal bleeding such as that resulting from trauma to the cervix (e.g., coitus, douching). Pain usually occurs over several cycles. There is no history of intermittent lower abdominal pain. Examination at the time of mittelschmerz may reveal some lower quadrant tenderness with or without rebound. Bimanual examination may show localized tenderness. A palpable ovary may be present, but a history of regular menses, lack of fever, and negative pregnancy tests confirm the diagnosis. Mild analgesics, especially nonsteroidal anti-inflammatory drugs (NSAIDs), and reassurance are usually adequate for these patients.
Q 89.10: Which of the following patterns of stiffness is most characteristic of patients with rheumatoid arthritis? A morning stiffness lasting at least 1 hour B exacerbation of joint stiffness with walking C frequent, brief episodes of stiffness after inactivity D stiffness reflected by a major delay in muscle relaxation E stiffness evidenced by increased resistance to passive movement
The Correct Answer is: A Morning stiffness lasting at least 1 hour is characteristic of rheumatoid arthritis (RA). Exacerbation of joint stiffness with weight bearing (such as walking) and frequent, brief episodes of stiffness (lasting <30 minutes) after inactivity are both more characteristic of degenerative joint disease, not RA. Stiffness reflected by a major delay in relaxation after muscle contraction is seen in myotonic dystrophy. Stiffness evidenced by increased resistance to passive movement describes the "rigidity" associated with parkinsonism. (Hellmann and Imboden, 2008, pp. 720-721) Hellmann DB , Imboden JB. Arthritis and musculoskeletal disorders. In: Tierney LM , McPhee SJ , Papadakis MA, eds. 2008 Current Medical Diagnosis and Treatment. New York, NY: McGraw-Hill; 2008:703-756.
Q 116.3: A patient with myasthenia gravis would likely experience symptomatic benefit with which of the following? A acetylcholinesterase inhibitors B muscarinic antagonists C α1-blockers D β-blockers E dopamine agonists
The Correct Answer is: A Myasthenia gravis is characterized by autoantibodies directed against nicotinic cholinergic receptors at neuromuscular junctions. By inhibiting the enzyme responsible for metabolizing acetylcholine (acetylcholinesterase), the synaptic concentration of acetylcholine increases and can bind more frequently to functional nicotinic receptors yet to be affected by the disease. This can alleviate the symptoms such as limb weakness, difficulty swallowing, and difficulty chewing associated with the disease. Examples of acetylcholinesterase inhibitors include neostigmine and pyridostigmine. (Aminoff, 2008, pp. 892-893) Aminoff MJ. Nervous system disorders. In: Tierney LM Jr , McPhee SJ , Papadakis MA, eds. Current Medical Diagnosis & Treatment. 47th ed. New York: McGraw-Hill; 2008.
Which of the following is the drug of choice for acute hypertensive encephalopathy? A Labetolol B Clonidine C Furosemide D Nifedipine E Nitroglycerin
The Correct Answer is: A Neurologic emergencies associated with elevated blood pressure must be throurougly evaluated to determine the diagnosis and appropriate treatment plan. In the case of hypertensive encephalopathy, immediate attention must be focused on blood pressure reduction. Multiple agents are known to decrease blood pressure, but selection must also focus on how rapidly each agent works, how titratable the agent is, and any potential sequelae from using a particular agent. Sodium nitroprusside was classically the agent of choice for rapid blood pressure management, but it has fallen out of favor due to its monitoring requirements and rate of toxicity. Labetolol, a beta-blocker, is recommended for acute management, except in the case of cocaine intoxication. It has a rapid onset of action, can be titrated, and is given IV. Use of labetolol should be avoided in patients with known asthma, COPD, congestive heart failure, bradycardia, and second or third degree heart block. Additional agents appropriate for hypertensive encephalopathy include enalaprilat, esmolol, fenoldopam, hydralazine, and nicardipine, with each being considered carefully (based on patient condition and other factors). Agents with known CNS adverse effects, such as clonidine, should be avoided. Use of clonidine must also be monitored due to potential rebound hypertension. Nifedipine use is discouraged in hypertensive emergencies. Nitroglycerin should not be used for hypertensive encephalopathy because it increases intracranial pressure
Q 22.7: A 22-year-old male hurts his right knee while playing football on artificial turf. He states that he planted his foot and went to turn, but his leg didn't turn with his body. He instantly felt a popping sensation in the knee. A few hours later he develops an effusion. Based on the history of the injury, which knee structure was likely injured? A Anterior cruciate ligament (ACL) B Lateral collateral ligament (LCL) C Medial collateral ligament (MCL) D Posterior cruciate ligament (PCL) E Quadriceps tendon
The Correct Answer is: A Non-impact rotational or hyperextension forces are the most common mechanisms for sustaining a tear of the ACL. One third of patients report hearing an audible popping sound as their ACL tear occurred. Because the ACL is a vascular structure, when it tears a rapid bloody effusion (hemarthrosis) usually develops which effects mobility of the joint. Lateral collateral ligaments are the least likely to be injured as the type of force necessary to cause injury would be a varus stress which is unlikely to occur in typical circumstances. Medial collateral ligament injuries are fairly common and produced by a valgus force that stresses the ligament. This can occur in many sporting events including those in which another competitor might fall on or dive into the lateral aspect of the knee. Trauma to a knee can result in tears of both the ACL and MCL in certain situations. A tear of the quadriceps tendon usually occurs when a person falls on a knee that is partially flexed. As the quadriceps muscle contract to prevent excessive flexion, the force and momentum of the fall may overwhelm the knee extension mechanism and cause the rupture. No such mechanism occurred in our scenario. Posterior cruciate injuries occur when the tibia is driven posterior in relation to the femur as may happen when a car dashboard is driven into the tibias during a major front impact collision. A powerful hyperextension force can result in both ACL and PCL tears (usually in that order). PCL tears are much more uncommon than ACL tears and don't generally occur with basic rotational forces as described in our patient scenario.
Q 98.40: A 22-year-old male hurts his right knee while playing football on artificial turf. He states that he planted his foot and went to turn, but his leg didn't turn with his body. He instantly felt a popping sensation in the knee. A few hours later he develops an effusion. Based on the history of the injury, which knee structure was likely injured? A Anterior cruciate ligament (ACL) B Lateral collateral ligament (LCL) C Medial collateral ligament (MCL) D Posterior cruciate ligament (PCL) E Quadriceps tendon
The Correct Answer is: A Non-impact rotational or hyperextension forces are the most common mechanisms for sustaining a tear of the ACL. One third of patients report hearing an audible popping sound as their ACL tear occurred. Because the ACL is a vascular structure, when it tears a rapid bloody effusion (hemarthrosis) usually develops which effects mobility of the joint. Lateral collateral ligaments are the least likely to be injured as the type of force necessary to cause injury would be a varus stress which is unlikely to occur in typical circumstances. Medial collateral ligament injuries are fairly common and produced by a valgus force that stresses the ligament. This can occur in many sporting events including those in which another competitor might fall on or dive into the lateral aspect of the knee. Trauma to a knee can result in tears of both the ACL and MCL in certain situations. A tear of the quadriceps tendon usually occurs when a person falls on a knee that is partially flexed. As the quadriceps muscle contract to prevent excessive flexion, the force and momentum of the fall may overwhelm the knee extension mechanism and cause the rupture. No such mechanism occurred in our scenario. Posterior cruciate injuries occur when the tibia is driven posterior in relation to the femur as may happen when a car dashboard is driven into the tibias during a major front impact collision. A powerful hyperextension force can result in both ACL and PCL tears (usually in that order). PCL tears are much more uncommon than ACL tears and don't generally occur with basic rotational forces as described in our patient scenario.
Q 32.4: You are evaluating a patient with multiple articular complaints that are consistent with osteoarthritis. The patient is a 60-year-old female who complains of pain to her joints which worsen as the day progresses. On examination you notice that the patient has Heberdens nodes. Based on the history and physical exam findings, what structures become progressively damaged and eroded over time? A Articular cartilage B Fascia C Ligaments D Muscles E Tendons
The Correct Answer is: A Osteoarthritis is a progressive, irreversible disease that leads to loss of articular cartilage in the joints. Osteoarthritis can affect the weight bearing joints in the lower extremities and spine, but can also be commonly found in the hands and other joints. The occurrence and severity of the condition can be affected by history of past trauma to a joint, long-term wear and tear on a joint, as well as obesity, genetics, and progressive aging. The condition can produce pain, swelling, stiffness, decreased range of motion, joint deformity, crepitus and decreased ability to do tasks of daily living. Osteoarthritis does tend to lead to direct damage to tissues like fascia (which is a fibrous connective tissue that surrounds muscles), muscles or tendons (tendons connect muscles to bones). As osteophytes form and joints become distorted, this could have a secondary impact on ligaments that are responsible for providing joint stability, but ligaments are not primarily affected by osteoarthritis.
Q 15.10: What age group is most at risk to develop osteoid osteoma? A Adolescents B Elderly C Middle-aged D Neonates E Toddlers
The Correct Answer is: A Osteoid osteoma is a benign bone forming tumor that usually develops during a patient's second decade of life. This type of tumor is much more common in boys than girls and typically affects the lower extremities (femur and tibia primarily) and spine more than other areas of the body. Patients typically present with gradually progressive bone pain that is worse at night and does not correlate with activity level. The tumor produces high levels of prostaglandins, so symptoms usually improve in 20-25 minutes if the patient takes a medication like ibuprofen, ASA or other NSAIDS that are prostaglandin inhibitors. A lack of improvement in symptoms with these medications should lead health care providers to consider a different diagnosis. The pain of this condition may cause those afflicted in a leg to limp and have swelling, muscle atrophy or contractures and exquisite point tenderness. The condition usually resolves on its own over time, but symptomatic patients may require surgical resection or radioablation of the tumor.
What age group is most at risk to develop osteoid osteoma? A Adolescents B Elderly C Middle-aged D Neonates E Toddlers
The Correct Answer is: A Osteoid osteoma is a benign bone forming tumor that usually develops during a patient's second decade of life. This type of tumor is much more common in boys than girls and typically affects the lower extremities (femur and tibia primarily) and spine more than other areas of the body. Patients typically present with gradually progressive bone pain that is worse at night and does not correlate with activity level. The tumor produces high levels of prostaglandins, so symptoms usually improve in 20-25 minutes if the patient takes a medication like ibuprofen, ASA or other NSAIDS that are prostaglandin inhibitors. A lack of improvement in symptoms with these medications should lead health care providers to consider a different diagnosis. The pain of this condition may cause those afflicted in a leg to limp and have swelling, muscle atrophy or contractures and exquisite point tenderness. The condition usually resolves on its own over time, but symptomatic patients may require surgical resection or radioablation of the tumor.
Q 19.10: A 32-year-old man presents to the urgent care center with a concern of scrotal tenderness that began 3 days ago and has now worsened. Physical examination reveals a temperature of 100.7°F, positive tenderness in the posterolateral aspect of the right testis with swelling, spermatic cord tenderness with palpation, and no transillumination. What is this patient's most likely diagnosis? A epididymitis B orchitis C epididymo-orchitis D testicular torsion
The Correct Answer is: A Pain and swelling are prominent features of epididymitis; fever and abdominal pain may also be present. Epididymitis is caused by an ascending infection that without treatment will continue to the testicles, causing a significant swelling that will make it difficult for the clinician to distinguish between the epididymis and the testicles (epididymo-orchitis). Orchitis alone is most commonly viral (mumps) and observed in prepubertal boys. In men younger than 30 years, epididymitis can be confused with torsion.
Q 71.10: A patient who resides in northern Arizona presents with signs and symptoms that are consistent with pneumonia. He is afebrile and appears non-toxic. His physical exam is unremarkable, and blood work is within normal limits. A chest x-ray is ordered and reveals bilateral upper lobe infiltrates. Based on this information, what is the best medication treatment for this patient? A Itraconazole B Erythromycin C Oseltamivir D Doxycycline E Amoxicillin
The Correct Answer is: A Patient is presenting with clinical signs and symptoms of fungal pneumonia. The presentation of being afebrile, along with upper lobe infiltrates bilaterally in a region of the country that consistently has more fungal pneumonia's would lead the clinician with the reasoning that a fungal process is present. The azole's are the best treatment for fungal pathogens. (Proia LA. Current Diagnosis and Treatment in Pulmonary Medicine, Section X, Diseases of the Pleura, Chapter 38, Fungal Pneumonias)
Q 118.18: Generally, patients who are malingering: A use illness to attain a goal B have avoidant personalities C follow prescribed treatment regimens D have a history that agrees with their physical symptoms
The Correct Answer is: A Patients who are malingerers do not want to improve until their goal is met. Goals may be financial, occupational, or legal. These patients will act differently when they think they are not being observed. They may fake their symptoms in order to be admitted to a hospital or to obtain drugs. These patients have an antisocial personality disorder. (Ford, 2008, p. 417; Sadock and Sadock, 2008, pp. 421-422) Ford, CV. Somatoform disorders. In: Ebert MH , Loosen PT , Nurcombe B , Leckman JF, eds. Current Diagnosis and Treatment in Psychiatry. New York: McGraw-Hill; 2008. Sadock BJ , Sadock VA. Concise Textbook of Clinical Psychiatry, 3rd ed. Philadelphia, PA: Lippincott, Williams & Wilkins; 2008.
Q 117.6: A phobia is an excessive fear of an object or place that leads to or can be preceded by: A panic attack B depression C hallucinations D delusions E confabulations
The Correct Answer is: A Patients who have a phobia realize it is an irrational fear and try to avoid whatever they have the fear of. In attempts to avoid the "problem," patients can develop anxiety or panic attacks. (Sadock and Sadock, 2008, p. 250; Shelton, 2008, p. 361) Sadock BJ , Sadock VA. Concise Textbook of Clinical Psychiatry, 3rd ed. Philadelphia, PA: Lippincott, Williams & Wilkins; 2008. Shelton RC. Other psychotic disorders. In: Ebert MH , Loosen PT , Nurcombe B , Leckman JF, eds. Current Diagnosis and Treatment in Psychiatry. New York: McGraw-Hill; 2008.
Q 100.1: A 65-year-old man with a history of emphysema, obstructive sleep apnea, prior inferior myocardial infarction, and pulmonary hypertension presents with complaints of increasing dyspnea over the last 6 months. He has also recently noticed increasing lower extremity edema. On physical exam, jugular venous distension is appreciated. Auscultation of the chest demonstrates a blowing holosystolic murmur along the lower left sternal border. Hepatomegaly, ascites, and lower extremity edema are noted. Which of the following would be the most likely finding on echocardiogram given the patient's physical exam findings? A Tricuspid regurgitation B Aortic stenosis C Atrial septal defect D Aortic regurgitation/insufficiency E Mitral stenosis
The Correct Answer is: A Patients with pulmonary hypertension and right heart failure frequently demonstrate right ventricular hypertrophy, which leads to tricuspid regurgitation; therefore, choice A is the most likely of the choices offered. This patient is demonstrating signs and symptoms of right heart failure, and with a prior inferior infarction is at risk for the development of tricuspid regurgitation. In addition, the patient demonstrates the blowing holosystolic murmur at the left lower sternal border, which is characteristic for tricuspid regurgitation. Choice B, aortic stenosis, presents with a systolic ejection murmur. An atrial septal defect, choice C, if large, could present with similar symptoms of exertional dyspnea secondary to a large shunt, but auscultation would reveal a moderately loud systolic ejection murmur heard best in the second and third interspaces. This is secondary to increased pulmonary arterial flow. Choice D, aortic regurgitation, presents with a soft diastolic murmur heard best at the left sternal border. (Fauci et al., 2008, Chapter 230)
Q 105.12: A 62-year-old obese woman presents with progressive numbness and tingling in her feet for the past 3 months. On physical examination, the patient is found to have decreased sensation to pinprick and vibration, absence of ankle reflexes, and difficulty with tandem walking. Which is the most common etiology of her symptoms? A diabetes mellitus B alcoholism C vitamin B12 deficiency D spinal cord tumor E rheumatoid arthritis
The Correct Answer is: A Peripheral neuropathy is a syndrome that is manifested by muscle weakness, paresthesias, decreased deep tendon reflexes, and autonomic disturbances most commonly in the hands and feet, such as coldness and sweating. There are many causes of peripheral neuropathy ranging from metabolic conditions to malignant neoplasm, rheumatoid arthritis, and drug and alcohol use. The increase in non-insulin-dependent diabetes mellitus due to obesity in the American population has increased the incidence of associated disease states. (Aminoff et al., 2005, pp. 213-214) Aminoff MJ , Greenberg DA , Simon RP. Clinical Neurology. 6th ed. New York, NY: McGraw-Hill; 2005.
Q 115.9: A 28-year-old man has a positive HIV ELISA and Western Blot but has never had an opportunistic infection. Of the following laboratory parameters, which, if present, is consistent with a diagnosis of AIDS in this man? A CD4 lymphocyte count of 175/mL B HHV-8 titer of 1:160 C HSV-2 titer of 1:80 D platelet count of 10,000/mL E total white blood cell count of 1500/mL
The Correct Answer is: A Persons with a CD4 count less than 200/mL or a CD4 percentage below 14% are now included in the Centers for Disease Control and Prevention category of "definitive AIDS diagnoses with laboratory evidence of HIV infection." Persons with HIV-AIDS may have positive herpes titers or depressed platelet or white blood cell counts, but these are not diagnostic of AIDS in the absence of symptoms. (Katz and Zolopa, 2009, p. 1177) Katz MH , Zolopa AR. HIV infection. In: McPhee SJ , Papadakis MA, eds. Current Medical Diagnosis and Treatment. 48th ed. New York, NY: McGraw-Hill; 2009.
Q 110.11: A patient presents with epistaxis from the right nares, along with direct pressure to the nares and elevation of the head. Which of the following is an appropriate initial treatment? A Phenylephrine spray and anterior packing B Triamcinolone spray and anterior packing C Azelastine spray and anterior packing D Momentasone spray and anterior packing E Cromolyn sodium spray and anterior packing
The Correct Answer is: A Phenylephrine is a topical decongestant, and acts as a vasoconstrictor to aid in stopping minor anterior septal epistaxis. Triamcinolone and momentasone are nasal steroids used for allergic rhinitis. Cromolyn sodium is a mast cell stabilizer, and azelastine is a topical H1 selective antihistamine, used for allergic rhinitis. (McPhee et al., 2011, Chapter 6)
Q 97.91: A 30-year-old woman presents to the office with polyuria, fatigue, and a chronic white vaginal discharge with vaginal pruritis. She has been having the discharge off and on for the past 6 months with recurrent treatment failures. Which of the following is the most likely diagnosis? A type 2 diabetes mellitus B hyperthyroidism C hypothyroidism D diabetes insipidus
The Correct Answer is: A Polyuria, polydipsia, and fatigue are all findings that can be consistent with both type 1 and type 2 diabetes. Any woman who presents with a chronic vaginal discharge or chronic vaginal pruritis should be screened for type 2 diabetes.
Q 119.5: A 30-year-old woman presents to the office with polyuria, fatigue, and a chronic white vaginal discharge with vaginal pruritis. She has been having the discharge off and on for the past 6 months with recurrent treatment failures. Which of the following is the most likely diagnosis? A type 2 diabetes mellitus B hyperthyroidism C hypothyroidism D diabetes insipidus
The Correct Answer is: A Polyuria, polydipsia, and fatigue are all findings that can be consistent with both type 1 and type 2 diabetes. Any woman who presents with a chronic vaginal discharge or chronic vaginal pruritis should be screened for type 2 diabetes. (Masharami, 2009, p. 1056) Masharami U. Diabetes mellitus and hypoglycemia. In: McPhee SJ , Papadakis MA, eds. Current Medical Diagnosis and Treatment. 48th ed. New York, NY: McGraw-Hill; 2009.
Q 102.7: A 56 year-old woman develops a fever of 102.1 F on her first post-operative surgery day following cholecystectomy. Which of the following is the most likely diagnosis? A Atelectasis B Catheter-related phlebitis C Drug reaction D Urinary tract infection E Wound infection
The Correct Answer is: A Post-operative fever that occurs in the first 24 hours is most likely caused by atelectasis (A). Catheter-related phlebitis (B), drug reactions (C) and urinary tract infection (D) typically present between post-operative days 2 and 4, while wound infection (E) typically presents on or after post-operative day 5.
Q 109.1: A 57-year old male with hypovolemia and decreased cardiac output is diagnosed with acute kidney injury (acute renal failure). What is the most likely cause of acute kidney injury? A Prerenal azotemia B Ischemia-associated injury C Glomerulonephritis D Antibiotics E Postrenal acute kidney injury
The Correct Answer is: A Prerenal azotemia (A) is the most common form of acute kidney injury (AKI). Azotemia is a buildup of nitrogen waste products in the body, resulting in a rise in serum Cr or BUN concentration, due to inadequate renal plasma flow and intraglomerular hydrostatic pressure to support normal glomerular filtration. The most common clinical conditions associated with prerenal azotemia are hypovolemia, decreased cardiac output, and medications that interfere with renal autoregulatory responses such as nonsteroidal anti-inflammatory drugs (NSAIDs) and inhibitors of angiotensin II. Intrinsic AKI includes glomerulonephritis, ischemia (B), sepsis/infection, vascular injury, and nephrotoxins including antibiotics (D)/(C). Postrenal AKI (E) occurs when the normally unidirectional flow of urine is acutely blocked either partially or totally, leading to increased retrograde hydrostatic pressure and interference with glomerular filtration. Postrenal AKI includes bladder outlet obstruction. Waikar SS, Bonventre JV. Chapter 279. Acute Kidney Injury. In: Longo DL, Fauci AS, Kasper DL, Hauser SL, Jameson JL, Loscalzo J, eds. Harrison's Principles of Internal Medicine. 18th ed. New York: McGraw-Hill; 2012. http://www.accessmedicine.com/content.aspx?aID=9129805 . Accessed March 6, 2013.
Q 85.1: A middle-aged woman presents with elevated cholestatic liver enzyme levels. She is not taking any medications, does not drink alcohol, and does not complain of abdominal pain. She has not had any previous biliary tract surgery. Which of the following is the most likely diagnosis? A primary biliary cirrhosis B pancreatitis C cholecystitis D fatty liver E primary sclerosing cholangitis
The Correct Answer is: A Primary biliary cirrhosis affects women typically between ages 40 and 60. It is often discovered incidentally when the serum alkaline phosphatase level is found to be elevated. Many patients do not have pain, which is more common in cholecystitis or pancreatitis. Primary sclerosing cholangitis is more likely to occur in a patient with known inflammatory bowel disease. (Friedman, 2009, pp. 607-608, 622) Friedman LS. Liver, biliary tract, and pancreas disorders. In: McPhee SJ , Papadakis MA, eds. Current Medical Diagnosis and Treatment. 48th ed. New York, NY: McGraw-Hill; 2009.
Q 105.7: Your 65-year-old patient with a history of tobacco abuse was recently diagnosed with stage III lung cancer. He has not started treatment yet and presents to his oncologist with complaints of nausea, anorexia, and increasing fatigue over the last several days. He has been eating less than usual but has been able to maintain a normal fluid intake. His wife reports that he has been more forgetful and confused than usual. His medical history includes hypertension, for which he has been taking 25 mg of hydrochlorothiazide for 12 years, and gastro-esophageal reflux disease (GERD), for which he takes omeprazole. He has no history of significant side effects from his medications. You order labs, and the calcium level is elevated at 11.9 mg/dL. What is the most likely etiology of his hypercalcemia? A malignancy B hyperparathyroidism C thiazide diuretic use D dehydration E vitamin D toxicity
The Correct Answer is: A Primary hyperparathyroidism and malignancy account for 90% of all cases of hypercalcemia. *Ten to twenty percent of patients with cancer develop hypercalcemia, most commonly because of breast, lung, kidney, head and neck carcinomas, and multiple myeloma and lymphoma.* Given this patient's history of lung cancer, this is the most likely etiology of his hypercalcemia. Although it is possible that the patient's symptoms could be due to hyperparathyroidism, this is a relatively rare disorder, affecting only about 0.1% of the population, making malignancy a much more likely etiology. He is taking a low dose of hydrochlorothiazide, which has been stable for years; therefore, this is unlikely to be causing excessive fluid losses and dehydration with hemoconcentration. However, this medication could be exacerbating the hypercalcemia. He is not taking vitamin D, so there is nothing to suggest vitamin D toxicity. (Cho et al., 2009, pp. 778-780; Fitzgerald, 2009, pp. 1007-1009; Rugo, 2009, pp. 1483-1484) Cho K , Fukagawa M , Kurokawa K. Fluid and electrolyte disorders. In: Tierney LM , McPhee SJ , Papadakis MA, eds. Current Medical Diagnosis and Treatment. 48th ed. New York, NY: McGraw-Hill; 2009. Fitzgerald PA. Endocrine disorders. In: Tierney LM , McPhee SJ , Papadakis MA, eds. Current Medical Diagnosis and Treatment. 48th ed. New York, NY: McGraw-Hill; 2009. Rugo HS. Cancer. In: Tierney LM , McPhee SJ , Papadakis MA, eds. Current Medical Diagnosis and Treatment, 48th ed. New York, NY: McGraw-Hill; 2009.
Q 98.73: Following emergent appendectomy, a 58-year-old obese male develops a temperature of 102.4˚F, 18 hours postoperatively. His respiratory rate is 26 and his pulse is 116bpm. A physical exam reveals scattered fine rales. What is the most likely diagnosis? A Atelectasis B Aspiration pneumonitis C Pleural effusion D Pneumonia E Pulmonary embolus
The Correct Answer is: A Pulmonary alveoli collapse, also known as atelectasis, occurs during operative procedures for a variety of reasons, including decreased clearance of secretions and decreased intra-alveolar pressure. Postoperatively, often due to pain, patients may not mobilize secretions appropriately, also contributing to atelectasis. Atelectasis is the most common postoperative pulmonary complication, and is often associated with emergent and prolonged surgeries, especially those of the thorax and abdomen. Atelectasis is associated with fever, an increased respiratory rate, an increased pulse, and lung exam findings ranging from normal to rales and decreased breath sounds. Symptoms usually present within the first 48 hours postoperatively. Pulmonary aspiration pneumonitis, although possible, is less likely due to appropriate preoperative and intraoperative measures being utilized to decrease risk. Pneumonia is also a common postoperative complication, due to the same contributing factors as atelectasis. Physical exam findings may also be similar. However, postoperative pneumonia is likely to become evident between 24 and 96 hours postoperatively. A postoperative pleural effusion may form, due to free peritoneal fluid as well as a complication of atelectasis, but has a lower incidence of occurrence than atelectasis alone. Patient symptoms will be based upon the size of the effusion, associated inflammation, and whether or not the effusion is infectious. Consideration must be given to pulmonary emboli for any post-surgical patient with tachypnea, tachycardia, and dyspnea. Pulmonary emboli may occur at any point postoperatively, but atelectasis remains a more common cause of postoperative fever and respiratory changes.
Q 116.13: Which of the following radiographic studies is indicated for the initial evaluation of a questionable palpable mass in the area of the kidney, with no other complaints by the patient? A renal ultrasound B intravenous pyelogram (IVP) C abdominal computed tomography D magnetic resonance imaging
The Correct Answer is: A Renal masses are initially identified by ultrasound. Ultrasound will be able to distinguish between a solid mass and a cyst. It is not uncommon to find some texts state that an intravenous pyelogram is noted as the initial test. Intravenous pyelograms have limited value, especially in differentiating small tumors. Whether a mass or a cyst, these findings are usually referred to a urologist who will follow-up with their own IVP and CT. (McAninch, 2008, p. 511) McAninch JW. Disorders of the kidneys. In: Tanagho EA , McAninch JW, eds. Smith's General Urology. 17th ed. New York, NY: McGraw-Hill; 2008:506-520. McAninch JW. Disorders of the penis and male urethra. In: Tanagho EA , McAninch JW, eds. Smith's General Urology. 17th ed. New York, NY: McGraw-Hill; 2008:625-637.
Q 97.6: A 37-year-old G3P2 female at 39 weeks gestation presents to the labor and delivery unit complaining of abdominal pain. Laboratory evaluation reveals anemia coagulopathy that is felt to be consumptive. What is the most likely diagnosis to have caused this? A Placental abruption B Placenta previa C Preeclampsia D Labor E Pre-existing anemia
The Correct Answer is: A Reproductive placental abruption is the most common cause of coagulopathy in pregnancy; the hemorrhage may be concealed and is not always evident. In the presence of pain, anemia, and coagulopathy, abruption should be assumed until proved otherwise.
What is the most common pathogen that causes an acute bronchitis? A Viral B Bacterial C Fungal D Unknown E Spirochetal
The Correct Answer is: A Respiratory viruses are the most common cause of acute bronchitis. In clinical medicine, it is rare to obtain cultures for patients who present with bronchitis symptoms.
Q 118.19: What is the most common pathogen that causes an acute bronchitis? A Viral B Bacterial C Fungal D Unknown E Spirochetal
The Correct Answer is: A Respiratory viruses are the most common cause of acute bronchitis. In clinical medicine, it is rare to obtain cultures for patients who present with bronchitis symptoms. (Tallman TA. Tintinalli's Emergency Medicine, Section 8. Pulmonary Emergencies, Chapter 67, Acute Bronchitis and Upper Respiratory Tract Infections)
Q 112.5: Which of the following findings would be evidence of a patient who has longstanding chronic obstructive pulmonary disease (COPD)? A a decreased FEV1 on spirometry that is not fully reversible with nebulizer treatment B chest radiograph with an elevated hemidiaphragm C FEV1/FVC ratio > 0.7 D a decreased A-a-Do2 on arterial blood gas E an abnormal sweat test
The Correct Answer is: A Spirometry provides objective information about pulmonary function and assesses the results of therapy. Pulmonary function tests early in the course of COPD reveal only evidence of abnormal closing volume and reduced midexpiratory flow rate. Reductions in FEV 1 and in the ratio of forced expiratory volume to vital capacity (FEV 1 % or FEV 1 /FVC ratio) occur later. In severe disease, the FVC is markedly reduced. Lung volume measurements reveal a marked increase in residual volume (RV), an increase in total lung capacity (TLC), and an elevation of the RV/TLC ratio, indicative of air trapping, particularly in emphysema. Arterial blood gas measurements characteristically show no abnormalities early in COPD other than an increased A-a-DO2. Indeed, they are unnecessary unless (1) hypoxemia or hypercapnia is suspected, (2) the FEV 1 is < 40% of predicted, or (3) there are clinical signs of right heart failure. Radiographs of patients with chronic bronchitis typically show only nonspecific peribronchial and perivascular markings. Plain radiographs are insensitive for the diagnosis of emphysema; they show hyperinflation with flattening of the diaphragm in less than half of cases. (McPhee and Papadakis, 2011, Chapter 9)
Q 97.87: A 27-year-old female is 8 weeks postpartum with her first child and has been exclusively nursing since discharge at the hospital. She has a 5-day history of engorgement in her right breast, which is red, tender, and feels warm to the touch. She states she is feverish but has not taken her temperature. On physical examination you see the breast as shown below. What is the usual causative agent of your suspected diagnosis? A Staphylococcus aureus B Streptococcus pyogenes C Group A Beta-hemolytic streptococcus D Fungal E Group B streptococci
The Correct Answer is: A Staphylococcus aureus is usually the causing agent in puerperal mastitis. Streptococcus infection can cause mastitis but is much less frequent than staph infections. Streptococcus pyogenes (B) is the cause of Group A beta-hemolytic streptococcus (C), which may cause necrotizing fasciitis, among other infections. Group B streptococci (E) is commonly seen colonized in the lower genital tract of females. It is a common cause of neonatal sepsis. Fungal infections (D) of the breast would not typically present in this way.
Q 107.3: Which of the following types of renal calculi is associated with an infectious cause? A struvite B uric acid C calcium oxalate D cystine E calcium phosphate
The Correct Answer is: A Struvite stones form when urea-splitting organisms, such as Proteus, Klebsiella, Pseudomonas, and Staphylococcus, are present in the urinary tract. Ammonia is formed when urease breaks down urea. This results in an alkaline urine, which decreases the solubility of struvite, favoring the production of stones. Calcium stones result from hyperabsorption of calcium in the intestine, impaired renal tubular reabsorption of calcium, primary hyperparathyroidism, intestinal hyperabsorption of oxalate, and hypocitraturia. Uric acid stones are due to hyperuricosuria or a urinary pH <5.5, which causes uric acid to dissociate. They are also the only radiolucent calculi. Cystinuria, an inborn error of metabolism, results in cystine stones. (Stoller et al., 2009, pp. 833-837) Stoller ML , Kane CJ , Meng MV. Urology. In: Tierney LM , McPhee SJ , Papadakis MA, eds. Current Medical Diagnosis and Treatment. 48th ed. New York, NY: McGraw-Hill; 2009.
Q 108.3: A patient presented to your office with multiple somatic complaints. During the mental status exam, you notice that the patient loses the thread of conversation and discusses irrelevant topics based on an external stimulus. The patient never gets back to the main point he or she was trying to express. What is this thought process called? A tangentiality B circumstantiality C looseness of association D word salad E neologisms
The Correct Answer is: A Tangentiality is a disturbance in thought causing the person to start a train of thought, but never getting to the point. Circumstantiality is seen in someone who eventually gets to the point after a delay in the thought process. Word salad is a mixture of words and phrases that are incoherent. Looseness of association is when the ideas shift between subjects that are totally unrelated to each other. Neologisms are the creation of new words. (Nurcombe and Ebert, 2008, p. 48) Nurcombe B , Ebert MH. The psychiatric interview. In: Ebert MH , Loosen PT , Nurcombe B , Leckman JF, eds. Current Diagnosis and Treatment in Psychiatry. New York: McGraw-Hill; 2008.
Q 13.6: A 43 year-old woman presents to the clinic due to the presence of a bulls eye rash on her right upper back. She is prescribed tetracycline. What is the mechanism of tetracycline? A Binds to the 30S ribosomal subunit to inhibit protein synthesis B Binds to the 50S ribosomal subunit to inhibit protein synthesis C Inhibits dihydropteroate synthase and folate production D Inhibits DNA replication by binding to DNA gyrase and topoisomerase IV E Inhibits the transpeptidation reaction
The Correct Answer is: A Tetracycline binds to the 30S ribosomal subunit to inhibit protein synthesis (A). Macrolide antibiotics bind to the 50S ribosomal subunit to inhibit protein synthesis (B). Sulfamethoxazole inhibits dihydropteroate synthase and folate production (C). Ciprofloxacin inhibits DNA replication by binding to DNA gyrase and topoisomerase IV (D). Beta lactam antibiotics inhibit the transpeptidation reaction leading to cell wall destruction (E).
Q 121.9: Which of the following concerning changes in nevi can be associated with melanoma? A asymmetry, irregular borders, color changes, and growth of the lesion B asymmetry, enlarged border, color uniformity, diameter <4mm C asymmetry, bleeding, color uniformity, diameter <4mm D symmetry, well defined borders, color irregularity, diameter <4mm E symmetry, sharp borders, no color, and growth of the lesion
The Correct Answer is: A The ABCDE's of moles are asymmetry, border irregularity, color change or irregularity, diameter >5mm, and evolving (changing in some way). These are all signs that the mole should be evaluated for possible dysplastic or malignant changes. (Wolff et al., 2009, Page 301)
Q 71.2: A 22 year-old male is involved in a motor vehicle crash resulting in fracture of the left femur and left ribs 3 through 6. Approximately 24 to 36 hours after admission he becomes mildly confused and his RR increases to 40. Chest x-ray reveals diffuse pulmonary opacities. ABG shows pH 7.39, PCO 2 34, PO 2 55. What is the most likely diagnosis? A ARDS B Cardiac contusion C Pleural effusion D Pneumothorax E Pulmonary thromboembolism
The Correct Answer is: A The acute onset of respiratory distress after trauma is consistent with ARDS (A). The chest x-ray findings are inconsistent with cardiac contusion (B), pleural effusion (C), pneumothorax (D) and pulmonary thromboembolism (E). (Mention by letter)
Shoulder dislocation commonly involves injury to which nerve? A Axillary B Median C Peroneal D Radial E Ulnar
The Correct Answer is: A The axillary nerve is in close proximity to the glenohumeral joint, thus making it vulnerable to injury during a shoulder dislocation. The median, radial, ulnar nerves are more distal in the upper extremity and and the peroneal nerve is in the leg.
Q 37.11: Which physical exam finding differentiates acne rosacea from acne vulgaris? A The absence of comedones B The absence of telangiectasias C The presence of erythema D The presence of inflammatory papules
The Correct Answer is: A The characteristic lesions of acne rosacea are small papules, papulopustules, and telangiectasias with flushing. There are no comedones present in acne rosacea.
Q 47.6: A woman brings her 13-year-old daughter to clinic, concerned about the fact that she has not yet had her first menstrual cycle. The patient is PMH negative, Social Hx negative, FMH: mothers' age of menarche was 13. Physical exam reveals the child to be a well-developed, well-nourished female, height and weight at the 50% for age and gender. Thelarche is present and has sparse pubic hair. The remainder of her physical exam is normal for age. The most appropriate course of action is? A Reevaluation in 6 months B Thyroid evaluation C Pelvic exam D Pelvic ultrasound E hCG
The Correct Answer is: A The child is in the expected age for puberty, and she shows signs of hormonal changes.
Q 97.120: A woman brings her 13-year-old daughter to clinic, concerned about the fact that she has not yet had her first menstrual cycle. The patient is PMH negative, Social Hx negative, FMH: mothers' age of menarche was 13. Physical exam reveals the child to be a well-developed, well-nourished female, height and weight at the 50% for age and gender. Thelarche is present and has sparse pubic hair. The remainder of her physical exam is normal for age. The most appropriate course of action is? A Reevaluation in 6 months B Thyroid evaluation C Pelvic exam D Pelvic ultrasound E hCG
The Correct Answer is: A The child is in the expected age for puberty, and she shows signs of hormonal changes.
Q 97.68: A 37-year-old woman, G3P2 at 30 weeks gestation, complains of lower extremity swelling and her weight is up 5 pounds this week. Her PMH is insignificant, and her other pregnancy was uncomplicated. Her BP baseline is now 142/92. On exam her BP is unchanged, her UA shows 2+ protein, and FHTs are 152. What is the cause of the protein in her urine? A Glomeruloendotheliosis B Glomerulonephritis C Renal vasospasm D Glomerular hemorrhage E Glomerular infarct
The Correct Answer is: A The classic histological change that occurs in the renal system (in preeclampsia) is swelling and inflammation of the endothelium and of the glomeruli, which leads to endothelial leaking.
Q 36.7: An 8-year-old male with a history of atopic dermatitis presents with a localized rash, consisting of vesicles and eroded lesions. He has a low-grade fever, but no other symptoms. What is the appropriate treatment? A Oral acyclovir for 5 to 10 days B Oral keflex for 7 to 10 days C Topical aclometasone ointment for 10 to 14 days D Topical mupirocin ointment for 7 to 10 days
The Correct Answer is: A The classic lesion of eczema herpeticum is described as a "punched out" lesion, which refers to vesicles that have become eroded. Mild cases of eczema herpeticum can be treated on an outpatient basis with oral acyclovir. More severe cases must be treated on an inpatient basis with IV acyclovir and oral antibiotics if superinfected.
Q 109.2: A 60-year-old male presents with scaling feet for several months. The nails are spared. The patient has tried over-the-counter hydrocortisone cream with no help. KOH shows branching hyphae and spores. Which of the following should be part of an appropriate treatment regimen? A topical clotrimazole bid x 4 weeks B lotrisone cream bid x 2 weeks C fluticasone ointment bid x 2 weeks D mupirocin ointment bid x 4 weeks
The Correct Answer is: A The clinical presentation and KOH results are consistent with tinea pedis or athlete's foot. Since the nails are unaffected, topical treatment with clotrimazole is appropriate. Lotrisone contains an antifungal and a steroid; this combination medication is not appropriate for this patient. Fluticasone is a class 3 topical steroid that would worsen this patient's condition. Mupirocin is a topical antibiotic and will not help resolve a fungal infection. (Wolff et al., 2009, Pages 695-699)
Q 97.96: A sexually active 19-year-old woman presents with clusters of painful vesicles on an erythematous base on the vulva and cervix, accompanied by temperature of 100°F and mild malaise. She reports a history of a similar outbreak last month, which resolved in 10 days. Microscopic examination of cells from the basement of a blister treated with Giemsa stain is likely to reveal A multinucleated giant cells B gram-positive cocci in clusters C gram-positive cocci in chains D gram-negative rods E hyphae and buds
The Correct Answer is: A The clinical presentation is consistent with herpes simplex. The appropriate microscopic study is a Tzanck smear, prepared by staining cells from the floor of a vesicle using Papanicolau, Giemsa, or Wright methods. The Tzanck smear will show multinucleated giant cells. It has a sensitivity of 60% to 70% and as a result should be confirmed by viral culture. Gram-positive cocci are consistent with staphylococcal or streptococcal infection and gram-negative rods are usually enteric pathogens. Hyphae and buds are seen on KOH prep with candidal infection.
Q 6.7: A 66-year-old female is admitted to the hospital with atrial fibrillation. Her past medical history has hypertension, diabetes mellitus type II, hypercholesterolemia, and rheumatoid arthritis. During her evaluation for work up it is noted that the patient has had recurring symptomatic episodes of atrial fibrillation for the last year, some resulting in the patient not being able to ambulate due to hypotensive events. Her current blood pressure is 146/83, and her heart rate is 87. Given this clinical scenario, what is the most appropriate procedure for this patient? A Ablation therapy B Cardiac catheterization C Synchronized cardioversion D Permanent pacemaker insertion E Pacemaker and defibrillator insertion
The Correct Answer is: A The clinical scenario for patients who have recurrent symptomatic episodes related to atrial fibrillation is ablation therapy (A). Cardiac catheterization (B) is sometimes performed based on suspicion of coronary artery disease, but does not address the electrophysiological reasons for the patient's episodes. Cardioversion (C) would not address a patient who is having recurrent episodes, and there is no clinical indication for pacing or defibrillation needs (D and E).
Q 4.6: A 23-year-old male presents to the clinic complaining of left anterior neck pain that developed over the past week following recovery from an acute upper respiratory infection. On physical exam a tender mass is felt anterior to the left sternocleidomastoid muscle from the mandible inferiorly to the level of the cricoid cartilage. Which of the following is the most likely diagnosis? A Branchial cleft cyst B Dermoid cyst C Peritonsillar abcess D Salivary gland tumor E Thryoglossal duct cyst
The Correct Answer is: A The development of a neck mass in a young adult following URI is consistent with branchial cleft cyst (A) and thyroglossal duct cyst (E). The location of this mass away from the midline and anterior to the SCM is most consistent with branchial cleft cyst (A). The location of the mass and history are inconsistent with dermoid cysts (B), which are typically midline, peritonsillar abcesses (C), which would be located in the retropharyngeal space, and salivary gland tumors (D), which would be located in the parotid, submandibular, or submental salivary regions.
Q 27.10: A 26-year-old woman who is nursing presents to clinic complaining of 2 to 3 days of increasing pain and redness in her left breast. She continues to feed her infant and has good milk supply. She is 4 weeks postpartum. She complains of some general fatigue, but no headaches, body aches, or fever. On physical exam, results are as follows: P 80, T 99.1°F, and BP 120/70, CV RRR, and lungs CTA. Her left breast has a 5-cm area of induration, in the upper outer quadrant a 2-cm mass is noted, which is tender to the touch. In addition, milk expressed is non-purulent. What is the most likely cause of the infection? A Staphylococcus aureus B Streptococcus viridians C Blastomycosis D Sporotrichosis E Streptococcus pyogenes
The Correct Answer is: A The discrete nodules indicate a breast abscess, which can commonly occur in nursing women. Fungal infections can occur, but are rare, and Streptococcal infections are not as likely as staph infections.
Q 97.113: A 26-year-old woman who is nursing presents to clinic complaining of 2 to 3 days of increasing pain and redness in her left breast. She continues to feed her infant and has good milk supply. She is 4 weeks postpartum. She complains of some general fatigue, but no headaches, body aches, or fever. On physical exam, results are as follows: P 80, T 99.1°F, and BP 120/70, CV RRR, and lungs CTA. Her left breast has a 5-cm area of induration, in the upper outer quadrant a 2-cm mass is noted, which is tender to the touch. In addition, milk expressed is non-purulent. What is the most likely cause of the infection? A Staphylococcus aureus B Streptococcus viridians C Blastomycosis D Sporotrichosis E Streptococcus pyogenes
The Correct Answer is: A The discrete nodules indicate a breast abscess, which can commonly occur in nursing women. Fungal infections can occur, but are rare, and Streptococcal infections are not as likely as staph infections.
Q 97.83: A 19-year-old G1 P0 presents to the emergency department complaining of abdominal pain. Her LMP was 2 months ago, and she has been spotting for the last two days. Her HCG is positive, and transvaginal ultrasound reveals no intrauterine gestational sac. The patient has an ectopic pregnancy, which has likely implanted in the fallopian tube. Why is the fallopian tube the most common non-uterine implantation site? A Lack of sub-mucosal layer in the fallopian tube B Inflammatory response in the endometrium C Proliferative endometrium D Extra myometrial tissue (fibroids) E Excessive trophoblastic proliferation
The Correct Answer is: A The fallopian tube is the most common site of ectopic pregnancy, accounting for over 95% of ectopic pregnancies. The lack of a submucosal layer allows for easy wall access and implantation of the fertilized ovum. The increasing rate of Chlamydia infections in the U.S. also impacts the physical anatomy of the fallopian tube, and impacts where the ovum implants. Excessive trophoblastic activity does not impact where the ovum implants, and proliferative myometrium enhances uterine implantation if the fertilized ovum makes it to the uterine cavity.
Q 24.10: Upon testing a patient for function of the hip extensors, which muscle is considered the primary muscle responsible for most extension? A Gluteus maximus B Pectineus C Semimembranosus D Semitendinosus E Vastus lateralis
The Correct Answer is: A The gluteus maximus is a large muscle that is partially responsible for giving shape to the buttocks. It is the dominant muscle responsible for hip extension. It is easily palpable with a patient in the prone position with buttocks squeezed together or with the hip extended and the knee flexed. The pectineus muscle is considered a secondary hip adductor. The Semimembranosus and Semitendinosus are two of the three hamstring muscles. They are primary movers in knee flexion, but only secondary contributors to hip extension. Vastus lateralis is one of the four quadriceps muscles and plays a role in knee extension, but not hip extension.
Q 70.2: What is the most common embolic source of acute arterial occlusion in the lower extremities? A Atrial fibrillation B Aortic aneurysm C Myocardial infarction D Prosthetic cardiac valve E Iliac artery thrombus
The Correct Answer is: A The heart accounts for 80% of all emboli, with atrial fibrillation making up 70% of that. Aortic aneurysms are frequently lined with thrombus but infrequently embolize; aneurysmal disease only accounts for 6% of all acute arterial occlusion. Acute myocardial infarction (especially those associated with left ventricular thrombus) accounts for 25% of cardioembolism, with peripheral embolization often the first sign of a previously "silent" MI. Prosthetic cardiac valves make up a still small but increasingly prevalent source of emboli. Peripheral arterial thrombi account for only 3% of acute occlusion.
Q 98.63: What is the most common embolic source of acute arterial occlusion in the lower extremities? A Atrial fibrillation B Aortic aneurysm C Myocardial infarction D Prosthetic cardiac valve E Iliac artery thrombus
The Correct Answer is: A The heart accounts for 80% of all emboli, with atrial fibrillation making up 70% of that. Aortic aneurysms are frequently lined with thrombus but infrequently embolize; aneurysmal disease only accounts for 6% of all acute arterial occlusion. Acute myocardial infarction (especially those associated with left ventricular thrombus) accounts for 25% of cardioembolism, with peripheral embolization often the first sign of a previously "silent" MI. Prosthetic cardiac valves make up a still small but increasingly prevalent source of emboli. Peripheral arterial thrombi account for only 3% of acute occlusion.
Q 66.2: A 12-year-old male begins to sneeze, and develops itchy, watery eyes about 15 minutes after being exposed to a cat. There is no respiratory difficulty. What phase of allergic response is he in? A Humoral B Priming C Cellular D Seasonal E Perennial
The Correct Answer is: A The humoral or early phase occurs in the first 15 minutes of being exposed to an allergen. The symptoms are caused by release of histamine. The cellular phase is the late phase, and occurs after four to six hours of allergen exposure. Seasonal allergic rhinitis occurs in a regular pattern each year, corresponding to pollen exposure. Perennial rhinitis occurs year round, and may be more linked with indoor allergen exposures.
Q 66.9: A 54-year-old female presents complaining of decreased visual acuity to her right eye over the past few hours. She denies pain, and describes having wavy vision and seeing flashes of light. Her visual acuity in the affected eye is 20/200. What condition best describes the following physical finding? A Retinal detachment B Central retinal artery occlusion C Open angle glaucoma D Angle closure glaucoma E Optic neuritis
The Correct Answer is: A The image demonstrates a detached retina. The superior aspect of the retina appears wavy and flowing. A central retinal artery occlusion is characterized by a pale retina, as well as a cherry red spot on the macula. Open angle glaucoma does not cause acute vision loss. Angle closure glaucoma causes painful acute vision loss. Optic neuritis is characterized by painful visual loss and a swollen optic disc.
Q 36.2: These lesions are visible on a 14-year-old female's forehead. What medication is this disorder best treated with? A topical retinoids B topical erythromycin C topical Benzoyl peroxide and erythromycin D oral Doxycycline 100 mg bid
The Correct Answer is: A The lesions are comedones (open and closed). Optimal treatment should be with topical retinoids such as tretinoin and adapalene, as these are comedolytic. Topical erythromycin is indicated in inflammatory acne, not comedonal acne as pictured. Benzoyl peroxide only has mild comedolytic activity and erythromycin has none. This combination medication would be more appropriate for inflammatory acne. Doxycycline has no comedolytic activity.
Q 106.10: A 33-year-old woman presents with a 3-year history of a persistent, unfluctuating depressed mood. She also notes persistent insomnia, poor concentration, and very little appetite. She denies previous similar symptoms, substance abuse, current prescriptive drug use, and has had no change in her overall life circumstances. She remains functional at work and in most relationships. On the basis of the information presented, what is the most likely diagnosis? A dysthymic disorder B premenstrual dysphoric disorder C major depressive disorder D cyclothymic disorder
The Correct Answer is: A The main historical component that points to this diagnosis is the long-term (equal to or greater than 2 years), unfluctuating symptoms without mention of manic or hypomanic symptoms that would be typical of cyclothymic disorders. No variances with menstrual cycles are mentioned. Major depressive disorder is generally associated with more intense symptoms, including suicidal ideation, and only requires a 2-week duration of symptoms to diagnose. (Loosen and Shelton, 2008, p. 328; Sadock and Sadock, 2008, p. 226) Loosen PT , Shelton RC. Mood disorders. In: Ebert MH , Loosen PT , Nurcombe B , Leckman JF, eds. Current Diagnosis and Treatment in Psychiatry. New York: McGraw-Hill; 2008. Sadock BJ , Sadock VA. Concise Textbook of Clinical Psychiatry, 3rd ed. Philadelphia, PA: Lippincott, Williams & Wilkins; 2008.
Q 115.1: A 17-year-old female (Ht: 5'6", Wt: 105 lbs) is noted to have macrocytic anemia. Her serum vitamin B12 level is 298 (normal range 203 to 339). What is the most likely diagnosis? A Folate deficiency B Hemolytic anemia C Iron deficiency anemia D Sideroblastic anemia E Vitamin B12 deficiency
The Correct Answer is: A The most common cause of folate deficiency is inadequate dietary intake. This patient is anorexic, which increases her chances of developing a folate deficiency, a common cause of macrocytic anemias. Her vitamin B12 level is in the normal range, making this diagnosis unlikely. Iron deficiency causes a microcytic anemia. Sideroblastic anemia is diagnosed by examining the bone marrow, and the MCV is usually normal. Hemolytic anemias generally do not affect MCV. (McPhee SJ, Papadakis MA, Tierney LM. Current Medical Diagnosis and Treatment, 2010, Chapter 13, Blood Disorders)
Q 97.67: A 54 year-old female returns for pathology results after being diagnosed with ovarian cancer. Which of the following is the most likely cause of ovarian epithelial malignancy? A Serous tumors B Endometrioid tumors C Mucinous tumors D Clear cell tumors E Transitional cell tumors
The Correct Answer is: A The most common of the ovarian epithelial malignancies are serous tumors (50%); tumors of mucinous (25%) (C), endometrioid (15%) (B), clear cell (5%) (D), and transitional cell (E) histology or Brenner tumors (1%) represent smaller proportions of epithelial ovarian tumors. Over half of all epithelial ovarian cancers have serous histology. The second most common histologic type of epithelial ovarian cancers are endometrioid adenocarcinomas.
Q 105.19: A 28-year-old woman who is HIV positive presents with substernal discomfort and painful swallowing for the past week. Her physical examination is unremarkable, but on endoscopy she has extensive adherent yellowish plaques on the esophageal mucosa. What is the most likely diagnosis? A candidal esophagitis B cytomegalovirus (CMV) esophagitis C gastroesophageal reflux disease (GERD) D herpetic esophagitis
The Correct Answer is: A The most common symptoms of infectious esophagitis in immunocompromised persons are dysphagia and odynophagia. Endoscopic evaluation is highly accurate. Candidal esophagitis is characterized by the yellow-white plaques described. CMV esophagitis is characterized by a few large, shallow, superficial ulcerations while herpetic esophagitis has many small deep ones. About half of patients with GERD will have erosions or ulcers distally at the squamocolumnar junction. (Katz and Zolopa, 2009, p. 520) Katz MH , Zolopa AR. HIV infection. In: McPhee SJ , Papadakis MA, eds. Current Medical Diagnosis and Treatment. 48th ed. New York, NY: McGraw-Hill; 2009.
Q 106.12: A 48-year-old woman presents with a chief complaint of gradually progressing difficulty in climbing stairs over the past 3 months. The physical examination shows there is notable proximal muscle weakness of the upper and lower extremities. The remainder of the examination is unremarkable. The laboratory evaluation shows an elevated serum creatinine phosphokinase level, and a muscle biopsy reveals lymphoid inflammatory infiltrates. Which of the following is the appropriate initial treatment of choice in this patient? A prednisone B azathioprine C methotrexate D immunoglobulin E hydrochloroquine
The Correct Answer is: A The most likely diagnosis in this patient is polymyositis. This is supported by the finding of a gradual progressive proximal muscle weakness and elevation of creatinine phosphokinase level. The finding of lymphoid inflammatory infiltrates on muscle biopsy confirms the diagnosis. Initial treatment of choice in this condition is the use of a corticosteroid (prednisone). Patients who do not respond to prednisone may then benefit from the use of methotrexate or azathioprine. Both intravenous immune globulin and hydroxychloroquine are effective for the treatment of patients with dermatomyositis that is resistant to prednisone therapy. (Hellmann and Imboden, 2008, pp. 733-735) Hellmann DB , Imboden JB. Arthritis and musculoskeletal disorders. In: Tierney LM , McPhee SJ , Papadakis MA, eds. 2008 Current Medical Diagnosis and Treatment. New York, NY: McGraw-Hill; 2008:703-756.
Q 66.11: A 32-year-old female is brought into the emergency department by her partner. His report indicates that she had been in her usual state of good health until a couple of days ago. At that time she started to complain of feeling fatigued. She now appears jaundiced and lethargic, and is complaining of chest pain. On exam, her spleen is palpable. Hemoglobin is 6 g/dl and she is Coombs positive. What is the most likely diagnosis? A Autoimmune hemolytic anemia B Glucose-6-phosphate dehydrogenase deficiency C Hereditary spherocytosis D Pyruvate kinase deficiency E Thalassemia
The Correct Answer is: A The onset of autoimmune hemolytic anemia (AHA) is often abrupt and dramatic. Anemia can develop in days, along with jaundice and splenic enlargement. When this triad is present, the suspicion for AHA must be high. The diagnostic test for AHA is the Coombs test. If positive, it confirms the presence of the antibody on the red cells. All other diseases listed are Coombs negative hemolytic anemias. (Fauci, et al, Harrison's Principles of Internal Medicine, 17e, Chapter 101, Hemolytic Anemias and Anemia Due to Acute Blood Loss)
Q 8.2: A 58 year-old male presents with a history of dyspnea on exertion and chronic cough worse with arising in the mornings. He has a 60-pack year history of cigarette use. On examination there is increased AP diameter and decreased breath sounds with a prolonged expiratory phase. Pulse oximetry reveals an oxygen saturation of 93% on room air. Current medications include varenicline and ipratropium. Which of the following is the most appropriate intervention at this time? A Influenza and pneumococcal vaccine B Montelukast C Oxygen therapy D Prednisone E Prophylactic antibiotic therapy
The Correct Answer is: A The patient has chronic obstructive pulmonary disease being treated with bronchodilators (ipratropium) and he is undergoing tobacco cessation therapy (varenicline). Additional health maintenance requirements include administration of influenza and pneumococcal vaccine (A). Antibiotics (E) are beneficial during acute COPD exacerbations. Oxygen therapy (C) is not required as evidenced by his pulse oximetry of 93%. Prednisone (D) is used in later stage disease that fails to respond to additional inhaled medications (long-acting beta agonists, corticosteroids). Montelukast (B) is used in the management of asthma.
Q 46.9: A 2-month-old female presents for a well child check. The mother has no concerns and feels that the child is doing well. On exam, there is no evidence of cyanosis and the peripheral pulses are normal and equal. However, there is a fixed and widely split S2, a right ventricular heave, and a systolic ejection murmur present. The murmur is heard best at the left sternal border second intercostal space. What is the most likely diagnosis? A Atrial septal defect B Coarctation of the aorta C Patent ductus arteriosus D Tetralogy of fallot E Aortic stenosis
The Correct Answer is: A The patient in this scenario is exhibiting the classic signs of an atrial septal defect. Coarctation of the aorta has absent or diminished femoral pulses and a blowing systolic murmur. A patent ductus arteriosus (PDA) is not associated with cyanosis, and the description of this murmur is classically described as a rough machinery systolic murmur. Tetralogy of fallot can have associated cyanosis with hypoxemic spells during infancy, easy fatigability, and dyspnea on exertion. Tetralogy of fallot also has the presence of a right ventricular lift and a rough, systolic ejection murmur, present along the left sternal border in the third intercostal space that radiates to the back. Aortic stenosis has a harsh systolic ejection murmur present at the right sternal border, and associated thrill in the carotid arteries.
Q 94.4: A 25-year-old female presents with a complaint of dry, cracking hands for two months. She has never had any rashes or similar problems. She does not work outside the home. She is the mother of a 6-month-old healthy female. Over-the-counter lotions have been tried, but they sting with application. What is the next appropriate step? A treat with triamcinolone 2.5% ointment bid, moisturize with petrolatum, and protect hands from moisture B patch testing to determine allergen C punch biopsy at periphery D KOH test
The Correct Answer is: A The patient most likely has an irritant dermatitis, secondary to increased water exposure, from having an infant. Appropriate treatment includes a topical steroid, like triamcinolone. Ointments are better vehicles than creams, as they penetrate better and moisturize. Using petroleum based moisturizers are more effective than oil based in repairing the epidermis. There is no evidence at this point of an allergen causing the outbreak. If the disorder is resistant to treatment, then patch testing to determine the allergen may be an appropriate step. A punch biopsy is not indicated, unless there is no response to treatment. A KOH test would be indicated if the rash was suspicious for a fungal infection. If the patient does not respond to treatment with a topical steroid, KOH testing may be indicated. (Wolff et al., 2009, Page 24)
Q 20.5: A 73-year-old, male with a 30 pack-year smoking history presents to the clinic with complaints of headache and dizziness. Labs reveal a hemoglobin of 21.3 g/dl, hematocrit of 63%, and platelet count of 498,000. He tests positive for the JAK2 mutation. Which of the following should be recommended to this patient to prevent secondary complications from his diagnosis? A Aspirin 81 mg daily B Ferrous Sulfate 325 mg twice daily C Eat a diet rich in vitamin B12 D Eat a diet rich in folate E Maintain stable consumption of foods that contain vitamin K
The Correct Answer is: A The patient most likely has polycythemia vera and is at risk for thrombotic events that can be reduced through the use of daily aspirin. In addition, he should be counseled on smoking cessation if he is still currently smoking. The patient should not take iron supplements (B) as they compete with phlebotomy therapy that seeks to reduce iron stores. Patient's with PCV similarly do not require vitamin B 12 (C) or folate (D) supplementation, and are not impacted by fluctuations in vitamin K consumption (E) unless they are being treated with warfarin for a current/recent thrombotic event.
Q 4.2: A 23-year-old man presents to the outpatient clinic for follow-up from a recent urgent care visit. He complains of sore throat, fever, fatigue, myalgias, and a rash that started 5 days ago, and have worsened since he was seen in the urgent care 3 days ago. The patient appears non-toxic with a temperature of 39.4 degrees Celsius. Physical exam reveals pharyngeal and tonsillar erythema without exudates, generalized lymphadenopathy, a morbilliform rash on his trunk, and no hepatosplenomegaly. A rapid strep screen and Monospot performed at the local urgent care were reportedly negative. Which of the following prevention strategies should be recommended to this patient? A Abstain from sexual activity B Avoid aspirin C Avoid contact sports and rest D Bedrest and increased fluids E Take the full course of antibiotics
The Correct Answer is: A The patient presentation is consistent with acute retroviral syndrome. The patient is highly contagious and should be counseled on strategies to prevent transmission of HIV to others (A). Aspirin use in viral syndromes (B) is associated with Reye's syndrome, but most often occurs in children with influenza or varicella. Avoiding contact sports (C) is appropriate patient education for a patient with infectious mononucleosis, and patients with group A strep pharyngitis should be instructed to take the full course of their antibiotics (E).
Q 36.3: A 1 day-old boy develops progressing abdominal distension, bilious vomiting and failure to pass a meconium stool. Abdominal radiographs show dilated loops of small bowel. Which of the following is the most likely diagnosis in this patient? A Cystic Fibrosis B Hypothyroidism C Imperforate anus D Intussusception E Pyloric Stenosis
The Correct Answer is: A The patient presents with a meconium ileus consistent with a diagnosis of cystic fibrosis (A). Pyloric stenosis (E) typically presents between 3 and 6 months of age, while intussusception (D) presents later (6 to 24 months). Imperforate anus (C) presents at birth, but infants typically lack acute abdominal distention and bilious vomit. Other common causes of intestinal obstruction in a newborn include meconium plug syndrome, Hirschsprung Disease, Intestinal Atresia, and Midgut Volvulus.
Q 50.1: A 1 day-old boy develops progressing abdominal distension, bilious vomiting and failure to pass a meconium stool. Abdominal radiographs show dilated loops of small bowel. Which of the following is the most likely diagnosis in this patient? A Cystic Fibrosis B Hypothyroidism C Imperforate anus D Intussusception E Pyloric Stenosis
The Correct Answer is: A The patient presents with a meconium ileus consistent with a diagnosis of cystic fibrosis (A). Pyloric stenosis (E) typically presents between 3 and 6 months of age, while intussusception (D) presents later (6 to 24 months). Imperforate anus (C) presents at birth, but infants typically lack acute abdominal distention and bilious vomit. Other common causes of intestinal obstruction in a newborn include meconium plug syndrome, Hirschsprung Disease, Intestinal Atresia, and Midgut Volvulus.
Q 98.4: A 1 day-old boy develops progressing abdominal distension, bilious vomiting and failure to pass a meconium stool. Abdominal radiographs show dilated loops of small bowel. Which of the following is the most likely diagnosis in this patient? A Cystic Fibrosis B Hypothyroidism C Imperforate anus D Intussusception E Pyloric Stenosis
The Correct Answer is: A The patient presents with a meconium ileus consistent with a diagnosis of cystic fibrosis (A). Pyloric stenosis (E) typically presents between 3 and 6 months of age, while intussusception (D) presents later (6 to 24 months). Imperforate anus (C) presents at birth, but infants typically lack acute abdominal distention and bilious vomit. Other common causes of intestinal obstruction in a newborn include meconium plug syndrome, Hirschsprung Disease, Intestinal Atresia, and Midgut Volvulus.
Q 37.7: A 24-month-old infant presents for his routine physical examination. The parents state that he has been following all of his developmental milestones. On examination, the clinician hears a grade II/VI murmur along the left sternal border, which radiates into the left axilla and the left side of the back. The child also has decreased femoral pulses bilaterally. The clinician orders a chest X-ray. Which of the following is the expected finding on X-ray based on the presentation? A notching or scalloping of the ribs B boot-shaped heart—right ventricular hypertrophy C "egg on string"—narrowed mediastinum D absence of the main pulmonary artery
The Correct Answer is: A The patient's presentation is consistent with findings of coarctation of the aorta. The pathognomonic finding in coarctation is decreased or absent femoral pulses. However, the majority of children show no signs of coarctation in infancy and develop signs and symptoms during childhood, most notably unequal pulses and blood pressure between arms and legs (arms greater than legs). In addition, a grade II/VI ejection murmur is heard at the aortic area and left sternal border that radiates into the left axilla and left back. Chest X-ray shows a normal-sized heart, a prominent aorta, indents at the level of the coarctation, and a dilated poststenotic segment resulting in the "figure 3" sign. Scalloping or notching of the ribs is due to enlargement of the intercostal arteries. Echocardiography is used to directly visualize the coarctation and estimate the obstruction. Asymptomatic infants and children are encouraged to have corrective surgery prior to age 5, after which they are at increased risk for myocardial dysfunction and hypertension, and require exercise testing prior to participation in aerobic activities. The boot-shaped heart is seen in patients with tetralogy of Fallot secondary to right ventricular hypertrophy; the narrowed mediastinum finding with "egg on a string" is typically seen in patients with transposition of the great vessels.
Q 26.1: A 49-year-old male presents to the clinic with symptoms of nausea, occasional vomiting, vague epigastric pain, fatigue, and weight loss of 35 lbs. in the past few months. On exam, you find a palpable abdominal mass. What is his most likely diagnosis? A Gastric adenocarcinoma B Crohn's disease C Peptic ulcer disease D Nonfamilial adenmatous polyposis E Anorexia nervosa
The Correct Answer is: A The patient's signs and symptoms in this question correlate most closely with gastric adenocarcinoma. With Crohn's disease, the patient may have weight loss and their signs and symptoms often include intermittent bouts of low-grade fever, diarrhea, right lower quadrant abdominal pain, and right lower quadrant tenderness on physical exam. Peptic ulcer disease typically presents with dyspepsia, but you would not see the associated weight loss or abdominal mass. Nonfamilial adenomatous polyposis consists of polyps in the colon, which are most typically asymptomatic; however, the most common symptom would be intermittent hematochezia if the polyp was large and ulcerated. Patients with anorexia nervosa have a disturbance of body image and weight loss, but they would not have the other symptoms present (the finding of an abdominal mass).
Q 30.9: A 43-year-old female presents to the outpatient clinic complaining of itching and irritation of her right eye. She denies decreased vision or photophobia. On physical exam the patient's eye has the following appearance: Which of the following is the most likely diagnosis for this patient? A Anterior blepharitis B Dacrocystitis C Posterior belpharitis D Preseptal cellulitis E Psoriasis
The Correct Answer is: A The patient's symptoms of itching and irritation of the lid margin with an inflamed eyelid and eyelash scaling is consistent with anterior blepharitis (A). Dacrocystitis (B) is an infection of the lacrimal sac and would be isolated to that location. Posterior blepharitis (C) affects the inner lid margins and meibomian glands. The symptoms and physical exam findings aren't consistent with preseptal cellulitis (D) or psoriasis (E); however, seborrhea is commonly associated with anterior blepharitis.
Q 16.9: A 43-year-old female presents to the outpatient clinic complaining of itching and irritation of her right eye. She denies decreased vision or photophobia. On physical exam the patient's eye has the following appearance: Which of the following is the most appropriate management for this patient's condition? A Daily lid cleansing and application of bacitracin ophthalmic ointment 500 units/g B Doxycycline 100 mg by mouth once daily C Incision and drainage D Referral to an ophthalmologist E Warm compresses every 3 hours until resolution
The Correct Answer is: A The patient's symptoms of itching and irritation of the lid margin with an inflamed eyelid and eyelash scaling is consistent with mild anterior blepharitis, which is initially treated with cleansing and the potential addition of a topical antistaphlococcal antibiotic (A). Answers (C) and (E) are appropriate treatments for a hordeolum. Doxycycline once daily (B) can be used as a long-term treatment for posterior blepharitis. The condition doesn't warrant referral (D).
Q 86.2: A 29-year-old woman comes in for evaluation of "panic attacks." She has no history of anxiety or depression. She says that during these episodes, which have been getting more frequent and more severe over the past month, she perspires heavily, feels highly anxious ("as though the end is coming"), and as if her heart is "going to jump out of my chest." Recently she has also begun experiencing headaches during these attacks. Her best friend told her that her face gets really "blotchy" during the attacks and then "awfully red" afterwards. Her examination today is remarkable only for blood pressure of 160/100. What is the most sensitive test for diagnosing her condition? A plasma fractionated free metanephrines B serum chromogranin A C serum thyroid stimulating hormone D urine fractionated metanephrine and creatinine E urine toxicology screen
The Correct Answer is: A The plasma fractionated free metanephrine test is the most sensitive test for a pheochromocytoma which this woman's symptoms strongly suggest. Serum chromogranin A (B) is elevated in about 90% of patients and its level correlates with tumor size. Serum TSH (C) would not be appropriate since the symptoms are not as suggestive of hyperthyroidism as they are of pheochromocytoma. A positive plasma test should be followed by a urine fractionated metanephrine test and creatinine level (D). A urine toxicology screen (E) would be appropriate if cocaine use were suspected; however the symptoms more strongly suggest pheochromocytoma. Fitzgerald PA, Endocrine Disorders, in Current Medical Diagnosis and Treatment, 52 nd ed. 2013.
Q 9.8: A 28-year-old male presents to the outpatient clinic with complaints of a painful, red right eye, with profuse drainage that started 2 days ago. He has increased tearing and photosensitivity. Physical exam reveals the following appearance of the upper tarsal conjunctiva. Which of the following is the most likely causative agent for this patient's conjunctivitis? A Chlamydia trachomatis B Herpes simplex virus C Neisseria gonorrhea D Pseudomonas E Streptococcus pneumoniae
The Correct Answer is: A The presence of acute follicular conjunctivitis is most consistent with inclusion conjunctivitis resulting from chlamydial infection (A). Follicles are most often seen in viral conjunctivitis and some forms of parasitic conjunctivitis, but not typically seen in bacterial conjunctivitis (C, D, and E).
Q 8.5: A 23-year-old female presents to the outpatient clinic with complaints of a painful, red right eye, with profuse drainage that started 2 days ago. She has increased tearing and photosensitivity. Physical exam reveals the following appearance of the upper tarsal conjunctiva. Which of the following is the most appropriate treatment for this patient? A Azithromycin 1 gram by mouth B Ceftriaxone 1 gram IM C Olopatadine 0.1% ophthalmic solution twice daily D Sulfacetamide 10% ophthalmic solution three times a day for 5 days E Valacyclovir 500 mg by mouth twice daily for 10 days
The Correct Answer is: A The presence of acute follicular conjunctivitis is most consistent with inclusion conjunctivitis resulting from chlamydial infection. Follicles are most often seen in viral conjunctivitis and some forms of parasitic conjunctivitis, but not typically seen in bacterial conjunctivitis. The first-line treatment for inclusion conjunctivitis is azithromycin 1 gram orally (A).
Q 31.3: A 58-year-old female presents to the outpatient clinic complaining of 1 week of rhinorrhea, nonproductive cough, and hoarseness. On physical exam she is noted to have erythematous nasal mucosa and decreased phonation without significant nasal discharge, sinus tenderness, pharyngeal erythema, or lymphadenopathy. Which of the following is the most likely diagnosis? A Laryngitis B Laryngeal cancer C Vocal cord hemorrhage D Vocal cord paralysis E Vocal cord polyp
The Correct Answer is: A The presence of acute hoarseness associated with an upper respiratory infection is consistent with laryngitis (A).
Q 6.4: A 56-year-old male patient is diagnosed with prostatitis. Which of the following is the least appropriate antibiotic to prescribe in the family practice setting? A ceftriaxone B doxycycline C levofloxacin D trimethoprim-sulfamethoxazole (TMP-SMX) E All are appropriate antibiotics for this patient in this setting.
The Correct Answer is: A The response to antibiotics in acute bacterial prostatis is usually prompt, perhaps because drugs penetrate readily into the acutely inflamed prostate Antibiotic selection should be guided by results of urine cultures and susceptibility results. Appropriate empiric antibiotics include a fluroquinolone (i.e.levofloxacin 500 mg once daily) or TMP/SMX (one double-strength tablet every 12 hours). Patients who are too ill for oral therapy or are septic on presentation should be hospitalized for initial parenteral treatment (intravenous quinolones with or without an aminoglycoside). Ceftriaxone would not be recommended as first-line.
Q 108.2: A 46-year-old female complains of heartburn, steatorrhea, and a 20-lb weight loss. She was diagnosed with a solitary ulcer in the duodenal bulb, but it has been refractory to treatment. Which of the following laboratory findings are most likely to be present in this patient? A Increased fasting serum gastrin B Increased fasting serum glucagon C Increased fasting serum insulin D Increased fasting serum glucose E Increased fasting lipids
The Correct Answer is: A The suspected diagnosis is Zollinger-Ellison syndrome, and the most sensitive and specific method for identifying this syndrome is by the presence of an increased fasting serum gastrin level. A gastrinoma would not cause elevation of any of the other choices listed. (McPhee SJ, Papadakis MA. Current Medical Diagnosis & Treatment, 2010, p. 556)
Q 18.1: A 23-year-old patient who has recently been on a ski trip presents with pain to the right hand after sustaining a fall. It is difficult to move, and there is pain on flexion of the digit. Based on this history what ligament would the patient most likely have injured? A 1st MCP joint ulnar collateral ligament B 2nd MCP joint ulnar collateral ligament C 3rd MCP joint ulnar collateral ligament D 4th MCP joint ulnar collateral ligament E 5th MCP joint ulnar collateral ligament
The Correct Answer is: A The ulnar collateral ligament at the base of the thumb, or 1st MCP joint, is often injured in forced abduction, such as a fall while skiing or during other sporting activities. An injury to this ligament has traditionally been called Gamekeeper's Thumb, but the origin of this term referred to a more chronic injury sustained by English gamekeepers as a result of the way they killed rabbits using their hands. Any of the MCP joint ulnar collateral ligaments could be injured in a fall if the mechanism of injury creates significant forces on the ligaments, but the 1st MCP joint is far more commonly injured than the others mentioned above.
Q 70.7: A 48-year-old African American male presents with dyspnea, 2-pillow orthopnea, and swelling to his lower legs that has developed over the last month. He also complains of fatigue and decreased exercise tolerance, stating that he has trouble climbing one flight of steps. On physical examination, his blood pressure is 178/98, pulse rate is 102, and respiratory rate is 20. There is 5 cm JVD, crackles at the bilateral lung fields, tachycardia, and an S3 is heard on cardiac auscultation. There is 2+ pitting edema to the lower extremities. His electrocardiogram reveals a sinus tachycardia at a rate of 105 and left ventricular hypertrophy. The chest x-ray reveals cardiomegaly with increased interstitial markings in all lung fields. There is a small right pleural effusion that blunts the costophrenic angle. He is on a thiazide diuretic. On repeat examination, the patient's blood pressure remains high. Which medication should be added to better control the patient's blood pressure? A Lisinopril B Diltiazem C Spironolactone D Amiodarone E Hydralazine
The Correct Answer is: A The use of an ACE inhibitor is a logical second order medication for this patient. Along with the diuretic, the ACE will further lower blood pressure. Calcium channel blockers are not an optimal choice, as they can worsen heart failure. ACE inhibitors can also have a protective effect on renal function, as well as improve morbidity and mortality in diabetics. While hydralazine can also be used for pressure control, ACE inhibitors would be a choice above hydralazine. (Bashore et al., Current Medical Diagnosis and Treatment, Chapter 10)
Q 81.2: A 27-year-old female carries a diagnosis of hereditary spherocytosis. She has been noted to have a few small gallstones that currently do not require a cholecystectomy. While her mean corpuscular hemoglobin concentration is increased, she maintains a hemoglobin level around 10. What is the best way treat her condition? A Annual examinations B Immediate splenectomy C Regular transfusions to maintain HGB above 12.0 D Splenectomy followed by anti-pneumococcal vaccination E Splenectomy with a cholecystectomy
The Correct Answer is: A There is currently no treatment recommended for patients with HS with a compensated anemia. This patient has a slight anemia that is compensated and would not require transfusions, splenectomy, or other treatment or therapy at this time. Annual examinations would be appropriate. (Fauci, et al, Harrison's Principles of Internal Medicine, 17e, Chapter 101, Hemolytic Anemias and Anemia Due to Acute Blood Loss)
Q 116.1: A 48-year-old African American male presents with dyspnea, 2-pillow orthopnea, and swelling to his lower legs that has developed over the last month. He also complains of fatigue and decreased exercise tolerance, stating that he has trouble climbing one flight of steps. On physical examination, his blood pressure is 178/98, pulse rate is 102, and respiratory rate is 20. There is 5 cm JVD, crackles at the bilateral lung fields, and tachycardia and an S 3 is heard on cardiac auscultation. There is 2+ pitting edema to the lower extremities. His electrocardiogram reveals a sinus tachycardia at a rate of 105 and left ventricular hypertrophy. The chest x-ray reveals cardiomegaly with increased interstitial markings in all lung fields. There is a small right pleural effusion that blunts the costophrenic angle. What is the initial medication of choice for treatment of this patient's edema? A Hydrochlorothiazide B Diltiazem C Amiodarone D Terazosin E Metoprolol
The Correct Answer is: A Thiazide diuretics are indicated for the initial treatment of fluid overload related to dilated cardiomyopathy. Calcium channel blockers are to be avoided, as they can worsen heart failure. Amiodarone is utilized for arrhythmic events and not purely for heart failure. (Bashore et al., Current Medical Diagnosis and Treatment, Chapter 10)
Q 31.4: A 52-year-old woman presents with vaginal discharge that is white curd-like in appearance but is not malodorous. She has a 19-year history of obesity and poorly controlled type 2 diabetes mellitus. Microscopic examination of the discharge with 10% potassium hydroxide demonstrates filaments and spores. Which of the following is the most likely etiologic agent? A Candida B Gardnerella C Lactobacillus D Staphylococcus epidermidis E Trichomonas vaginalis
The Correct Answer is: A This case has several clues pointing to a Candida infection, including the fact that diabetes mellitus can predispose patients to Candida infections and the presence of the white curd-like discharge that is not malodorous. In Trichomonas vaginalis, the discharge is malodorous and yellow-green in color. With Gardnerella, there is also a malodorous discharge. Lactobacillus is the predominant, normal microorganism of the vagina and keeps it slightly acidic to help reduce the growth of potentially harmful organisms. Staphylococcus epidermidis is also part of the natural flora of the vagina.
Q 97.97: A 52-year-old woman presents with vaginal discharge that is white curd-like in appearance but is not malodorous. She has a 19-year history of obesity and poorly controlled type 2 diabetes mellitus. Microscopic examination of the discharge with 10% potassium hydroxide demonstrates filaments and spores. Which of the following is the most likely etiologic agent? A Candida B Gardnerella C Lactobacillus D Staphylococcus epidermidis E Trichomonas vaginalis
The Correct Answer is: A This case has several clues pointing to a Candida infection, including the fact that diabetes mellitus can predispose patients to Candida infections and the presence of the white curd-like discharge that is not malodorous. In Trichomonas vaginalis, the discharge is malodorous and yellow-green in color. With Gardnerella, there is also a malodorous discharge. Lactobacillus is the predominant, normal microorganism of the vagina and keeps it slightly acidic to help reduce the growth of potentially harmful organisms. Staphylococcus epidermidis is also part of the natural flora of the vagina.
Q 45.6: A 2-week-old male infant presents for a routine checkup. The mother complains that he nurses every hour, but vomits (nonbilious) after every time he eats. He has only had three bowel movements since he has been home. On examination, the infant has not gained any weight since leaving the hospital, and the clinician notes gastric peristaltic waves. Which of the following is the treatment of choice for this patient? A pyloromyotomy B metoclopramide C laparotomy D omeprazole
The Correct Answer is: A This infant is presenting with signs and symptoms of pyloric stenosis. Infants typically have vomiting (projectile at times) after every feeding and it normally starts between the age of 2 and 4 weeks. The infant nurses fervently and is hungry. In addition, there may be dehydration, constipation, weight loss, and apathy. Abdomen may be distended with gastric peristaltic waves. Occasionally, an olive-sized mass can be felt in the right upper quadrant with deep palpation after the child has vomited. Vomitus is typically nonbilious. Diagnosis is confirmed by an upper gastrointestinal series with delayed gastric emptying, enlarged pyloric muscle, and characteristic semilunar impressions on the gastric antrum. In addition, an ultrasound is needed to verify the hypertrophic muscle. The treatment of choice for these patients is pyloromyotomy, which can be done laparoscopically. These patients make full recoveries and have an excellent prognosis.
Q 121.4: A 26-year-old female presents with a history of a rash around her neck, off and on for several years. It has been treated with mid potency prescription topical steroids, only to recur again. She reports that during treatment she will discontinue wearing jewelry until the rash resolves. The patient complains of pruritis but no other symptoms. What is the most likely diagnosis? A contact dermatitis B atopic dermatitis C herpes zoster D tinea Corporis
The Correct Answer is: A This is a classic contact dermatitis, secondary to nickel allergy. Nickel is a very common metal that is contained in metals, clothing, and jewelry. It is a delayed, cell mediated hypersensitivity reaction, so it takes multiple exposures before an allergic response is exhibited. Atopic dermatitis is usually manifested prior to the age of six. The classic distribution is the flexural surfaces of the extremities. Herpes zoster presents with prodromal neuralgic pain two to three weeks prior to outbreak. The rash has vesicles and erythema in a dermatomal distribution that crust over after three to five days. The pain may last after resolution of the lesions. Presentation of tinea corporis is pruritic, annular scaling patches that enlarge with central clearing. (Wolff et al., 2009, Page 26)
Q 8.9: An 8-year-old male presents to his primary care provider with the onset of a new rash, consisting of small, oval, discrete scaling plaques on his trunk, and a large red plaque with overlying thin, silvery scales in the gluteal cleft. Which of the following is a potentially important historical finding in this patient? A a recent history of group A strep infection B a family history of atopy C exposure to nickel in clothing D a personal history of allergies
The Correct Answer is: A This is a classic guttate psoriasis. An acute strep infection is a known precipitating factor of guttate psoriasis. All patients need to be checked and treated for a strep infection. Atopy has no correlation with guttate psoriasis. It is not caused by contact with an allergen or irritant. It is also not caused by an allergic reaction.
Q 39.20: A mother presents with her four-month-old infant for a well child check. While examining the child, you notice ill defined bluish macules on the back and lumbosacral regions. What is the appropriate next step? A reassure the parent that these frequently spontaneously resolve in early childhood B call Child Protective Services on suspicion of abuse C educate the parents that there may be an increased risk of melanoma in the lesions D educate the parents that the only effective treatment is laser therapy
The Correct Answer is: A This is a common presentation of hypermelanosis, sometimes commonly referred to as Mongolian spots. These usually occur in patients with more pigmented complexions. They usually spontaneously resolve prior to the child entering grade school. Hypermelanosis can sometimes be mistaken for child abuse by an inexperienced practitioner; however, hypermelanosis is a common benign condition that occurs in patients with pigmented skin. There is no increased incidence associated with hypermelanosis, and no treatment is required or available.
A 30-year-old man is brought to the emergency department by his wife for abdominal pain, nausea, vomiting, and diarrhea. She says he has been getting "worse and worse" for at least the past two months. He is "too tired" and his "muscles hurt too much" to play golf, formerly their favorite leisure activity. He has been depressed and reticent, instead of his usual talkative self. He won't eat much of anything, even when she cooks his favorite meals and she is sure he has lost considerable weight because his clothes "hang off him." He has refused medical evaluation until today, when the abdominal pain of approximately a week increased and he began having the vomiting and diarrhea. Initial impression is that the man is quite tanned, although it is the middle of winter. Closer examination reveals dark pigmentation in his skin folds and on the buccal mucosa. This presentation most strongly suggests which of the following? A adrenocortical insufficiency B diabetic ketoacidosis C hypercortisolism D hyperglycemic hyperosmolar state E hypothyroidism
The Correct Answer is: A This is a fairly classic presentation of adrenocortical insuffiency (Addison's disease). Diabetes mellitus, type 1, that has led to ketoacidosis (B) could also present with weight loss, nausea, and vomiting; but is also accompanied by hyperphagia and polydipsia, and does not have the hyperpigmentation associated with Addison's disease. Patients with hypercortisolism (Cushing Syndrome) (C) present with central obesity, hirsutism, thin skin, poor wound healing, and a host of other problems including emotional lability. Those with hyperglycemic hyperosmolar state (D) are typically older, have a high body mass index, and present with lethargy, confusion, and dehydration. Patients with hypothyroidism (E) usually have weight gain and constipation along with lethargy, fatigue, and weakness.
Q 97.9: A 27-year-old G3P2 at 39 weeks gestation has been in labor for 6 hours. Her membranes ruptured 3 hours ago and revealed a large amount of clear fluid. She has previously delivered a 7 pound 2 ounce infant. She is having good quality contractions, and has been completely dilated and pushing for 55 minutes. The fetal head is +4, FHT 128. What would be the most appropriate management at this time? A Continue pushing B Instrument delivery C Emergency caesarean section D Non emergent caesarean section E Epidural pain control
The Correct Answer is: A This is a multigravida patient with an adequate pelvis. She is making steady progress and there are no signs of maternal or fetal distress, so continued labor management is expected.
Q 97.46: A 54-year-old female presents to clinic complaining of vaginal fullness and leaking of urine for 10 years, which is progressively getting worse. She is a G4P4 and is postmenopausal. Nothing seems to improve her symptoms, and coughing or running makes them worse. On physical exam her vitals are as follows: Ht 5'4", Wt 135 lb, T 98°F, BP 130/72. Her abdomen is soft and non-tender with no masses, and her pelvic exam reveals the anterior wall to be at the level of the hymen. In addition, UA dip is negative. She is on no medication. What would the most appropriate initial management include? A Pessary B Surgery C Renal CT D Renal functions E Cystogram
The Correct Answer is: A This is a stage 2 cystocele by the Baden Walker System, with urinary incontinence. It is best treated with conservative measures. If they fail, then surgery should be considered.
Q 117.2: A 62-year-old female diabetic patient complains of a pruritic rash under her breasts. A physical exam shows an eroded red plaque with satellite papules. What is the most likely cause? A Candida albicans B Staph aureus C group A strep D herpes simplex
The Correct Answer is: A This is consistent with a yeast infection of the skin, caused by Candida albicans. Diabetic patients are particularly susceptible to these infections. A staph infection of the skin will have either honey colored crusting or inflammatory papules and pustules with erythema of the skin. A strep infection of the skin will have inflammatory papules and pustules with erythema of the skin. A herpes simplex infection will have vesicles that crust over after a few days. There is often preceding neurogenic pain prior to the outbreak. (Wolff et al., 2009, Page 720-721)
Q 107.10: A 25-year-old male presents with complaints of a transient lesion in the groin. This has been present off and on for years. He states it is mildly pruritic, and notices it after he has had headaches or mild aches for which he takes acetaminophen. The lesion occurs at the same site each time. What is the most likely diagnosis? A fixed drug eruption B herpes zoster C herpes simplex D tinea cruris
The Correct Answer is: A This lesion is the classic presentation of a fixed drug eruption (FDE). FDEs can be precipitated by numerous drugs, including acetaminophen. Herpes zoster is a unilateral vesicular rash that occurs along a dermatome. There is neurogenic pain prior to and following an outbreak. It is not precipitated by a drug. Herpes simplex is a vesicular rash that crusts over after several days. The outbreak is frequently preceded by neurogenic pain. Tinea cruris is a well demarcated red scaling rash that may have satellite papules which occur in the groin. It is not precipitated by a drug. (Wolff et al., 2009, Pages 567-568)
In the emergency department, you are asked to evaluate a 77-year-old man with a history of HTN who had a syncopal episode while chasing after his dog. He admits to recent episodes of chest discomfort, also associated with activity, as well as dyspnea at lower levels of activity including walking up one flight of stairs. On physical exam, a grade III/IV crescendo-decrescendo systolic ejection murmur can be heard best over the right upper sternal border. His EKG demonstrates NSR @ 80 bpm, with evidence of left ventricular hypertrophy. His troponin levels are negative for ischemia. What is the next most appropriate test or procedure? A Echocardiography B VQ scan C CT scan of the head D Serum D-dimer levels E MRI of the heart
The Correct Answer is: A This patient exhibits all the signs of progression of aortic stenosis, thus echocardiography is the next most appropriate test. A determination of severity can then be made, with possible cardiac catheterization if severe aortic stenosis is suspected, in preparation for surgical intervention if necessary. A VQ scan is appropriate if pulmonary embolism were suspected. A CT scan of the head could be considered if a head injury was suspected, but would not be the next step in the management of this patient. Serum D-dimer levels might be used to rule out pulmonary embolism, although it is a fairly nonspecific test. An MRI of the heart is not considered standard of care for aortic stenosis.
Q 121.16: In the emergency department, you are asked to evaluate a 77-year-old man with a history of HTN who had a syncopal episode while chasing after his dog. He admits to recent episodes of chest discomfort, also associated with activity, as well as dyspnea at lower levels of activity including walking up one flight of stairs. On physical exam, a grade III/IV crescendo-decrescendo systolic ejection murmur can be heard best over the right upper sternal border. His EKG demonstrates NSR @ 80 bpm, with evidence of left ventricular hypertrophy. His troponin levels are negative for ischemia. What is the next most appropriate test or procedure? A Echocardiography B VQ scan C CT scan of the head D Serum D-dimer levels E MRI of the heart
The Correct Answer is: A This patient exhibits all the signs of progression of aortic stenosis, thus echocardiography is the next most appropriate test. A determination of severity can then be made, with possible cardiac catheterization if severe aortic stenosis is suspected, in preparation for surgical intervention if necessary. A VQ scan is appropriate if pulmonary embolism were suspected. A CT scan of the head could be considered if a head injury was suspected, but would not be the next step in the management of this patient. Serum D-dimer levels might be used to rule out pulmonary embolism, although it is a fairly nonspecific test. An MRI of the heart is not considered standard of care for aortic stenosis. (Crawford et al., 2009, Chapter 7)
Q 45.5: A 3-year-old male on laboratory examination has a hemoglobin of 6 (9.5 to 15.0), platelet count of 43,000 (150,000 to 450,000), and Leukocyte count of 9.6 (4.5 to 11.0). Blasts are noted in peripheral smear and on marrow examination. What other laboratory examination is essential in this child? A Cerebrospinal fluid examination B Coagulopathy studies C MRI of the chest D Skeletal roentgenography E Urinalysis
The Correct Answer is: A This patient has acute lymphoblastic Leukemia (ALL). Examination of the CSF is essential to rule in/out CNS leukemia. A chest x-ray is adequate to determine if there is enlargement of the thymus or mediastinal nodes. An MRI would not be indicated at this point. Severe bleeds is uncommon and bleeding times are not typically effected. Skeletal roentgenography is not necessary for management of this patient, even if abnormalities are detected. A urinalysis may show microscopic hematuria and the presence of uric acid crystals, but is not essential in the diagnostic workup of this patient.
Q 41.6: A 9-year-old male presents with an acute onset of petechiae, ecchymoses, and gingival bleeding. He has pallor, fatigue, and bony pain. A pancytopenia is noted on CBC. Ebstein-Barr is negative. Lymphoblasts are noted on smear. Vitals reveal a temperature of 100.8F, HR 74, and RR 20. A few shoddy cervical nodes are noted. What is the most likely diagnosis? A Acute lymphoblastic leukemia B Aplastic anemia C Infectious lymphocytosis D Infectious mononucleosis E Lymphoma
The Correct Answer is: A This patient has acute lymphoblastic leukemia (ALL). While all the diseases would be in the differential, only ALL fits all the findings. Infectious mononucleosis is excluded with a negative EBV. Infectious lymphocytosis would not have a pancytopenia or increased blasts on smear. Aplastic anemia would have a pancytopenia but not bony pain. Lymphoma typically does not have a pancytopenia associated with the disease.
A 33-year-old female presents to the emergency department for the complaint of abdominal pain that has been present for the last month. During her workup, she has an ECG completed. She denies any shortness of breath, palpitations, orthopnea, dyspnea on exertion, syncope, weakness, or headaches. On physical exam, she is alert, awake, and in no distress. Her vital signs are as follows: temperature is 98.9, pulse is 100, respiratory rate is 18, and blood pressure is 132/90. Her HEENT is within normal limits, her neck is supple with a slightly enlarged thyroid that is non-tender without nodules, her lungs are clear, and cardiac is a regularly irregular rhythm. Based on these findings, what is the best therapy for this patient? A No therapy B Ablation therapy C Beta blockers D Digoxin E Verapamil
The Correct Answer is: A This patient has asymptomatic bigeminy. In this case, the patient is otherwise healthy and has no other medical problems. Patients with this presentation require no therapy, and this is an incidental finding on her evaluation. Only monitoring of the patient is considered.
Q 86.3: A 33-year-old female presents to the emergency department for the complaint of abdominal pain that has been present for the last month. During her workup, she has an ECG completed. She denies any shortness of breath, palpitations, orthopnea, dyspnea on exertion, syncope, weakness, or headaches. On physical exam, she is alert, awake, and in no distress. Her vital signs are as follows: temperature is 98.9, pulse is 100, respiratory rate is 18, and blood pressure is 132/90. Her HEENT is within normal limits, her neck is supple with a slightly enlarged thyroid that is non-tender without nodules, her lungs are clear, and cardiac is a regularly irregular rhythm. Based on these findings, what is the best therapy for this patient? A No therapy B Ablation therapy C Beta blockers D Digoxin E Verapamil
The Correct Answer is: A This patient has asymptomatic bigeminy. In this case, the patient is otherwise healthy and has no other medical problems. Patients with this presentation require no therapy, and this is an incidental finding on her evaluation. Only monitoring of the patient is considered. (Bashore et al., Current Medical Diagnosis and Treatment, Chapter 10)
Q 95.10: A 54 year old male is undergoing treatment for chronic myelogenous leukemia. On examination today his WBC is increased from 17,000 to 67,000, Hgb decreased from 13 to 9, spleen is now palpable on examination and he complains of abdominal fullness, an increased number of marrow blasts are noted on bone marrow biopsy and he "just doesn't feel good". What is the most likely diagnosis? A Accelerated phase of chronic myelogenous leukemia B Blast phase of chronic myelogenous leukemia C Development of treatment related lymphoma D Molecular transformation phase of chronic myelogenous leukemia E Transformation of chronic myelogenous leukemia to Acute myelogenous leukemia
The Correct Answer is: A This patient has classic signs of transformation from the chronic phase of CML to the accelerated phase. Objective findings to confirm this are increased percentages of blood blasts, thrombocytopenia development, increased basophils, and new clonal cytogenetics abnormalities. Blast crisis is the most severe form of accelerated phase and would indicate the disease is transformed to overt acute leukemia. There is not a phase described as molecular transformation, and the findings are not consistent with a secondary malignancy in this patient. (Lichtman et al., Williams Hematology 8e, Chapter 90, Chronic Myelogenous Leukemia and Related Disorders)
Q 88.1: A patient is undergoing induction treatment for acute myelogenous leukemia. They received daunorubicin and cytarabine seven days ago. They present in the emergency department with a temperature of 38.4°C, HR: 90, RR: 20. They are complaining of cough, fatigue, and diarrhea. They appear pale, fatigued, and slightly tachypenic. Lung fields are clear to auscultation interrupted by cough, and bowel sounds are hyperactive. What is the next best step in caring for this patient? A Obtain blood cultures and start cefepime B Obtain chest x-ray and blood cultures C Obtain CBC, LFTs, and blood cultures D Obtain CT of the chest and admit E Obtain pulse ox and start O2 by nasal cannula
The Correct Answer is: A This patient has developed a neutropenic fever following induction chemotherapy. The most important next step is to obtain blood cultures and start IV antibiotics. Additional steps can include urine and sputum culture. Consideration should be given for both gram positive and negative coverage. The lungs are clear due to the inability to form pus with neutropenia. A chest x-ray or CT may be appropriate, but are not the next best steps. Currently, there is no reason to start oxygen and this would not be the next best step. (Lichtman et al., Williams Hematology 8e, Chapter 89, Acute Myelogenous Leukemia) (Lichtman et al., Williams Hematology 8e, Chapter 22, Treatment of Infections in the Immunocompromised Host)
Q 38.9: An 8-year-old male with a history of atopic dermatitis presents with a widespread rash consisting of vesicles and eroded lesions. What is the causative organism? A Herpes simplex virus B Human papilloma virus C Staphylococcus aureus D Varicella zoster virus
The Correct Answer is: A This patient has eczema herpeticum. This is caused by the herpes simplex virus. Transmission can occur innocuously via the parent. Atopic dermatitis is a risk factor for eczema herpeticum, secondary to the impaired barrier function of the skin. This impaired barrier function allows the virus to spread rapidly.
Q 11.4: In your family practice, you perform an ankle brachial index (ABI) on your 66-year-old diabetic who smokes with the results being 0.71 on the left and 0.68 on the right. Which of the following is the most appropriate next step? A Begin the patient on aspirin 81 mg and clopidogrel 75 mg daily. B Begin the patient on prasugrel 60 mg loading dose followed by 10 mg daily and refer to cardiology. C Begin the patient on a low-molecular weight heparinoid (LMWH) and refer to a vascular surgeon for further evaluation. D Begin the patient on warfarin 5 mg daily and titrate to an INR of 2.0 to 3.0.
The Correct Answer is: A This patient has peripheral arterial disease (PAD). This can be treated with antiplatelet agents, including aspirin and/or clopidogrel. Warfarin is an anticoagulant and is not FDA-approved for use in PAD. Your other consideration is referral to a cardiologist and/or vascular surgeon for further evaluation, depending upon the degree of symptoms. Patients with PAD have approximately one in six chance of have significant atherosclerosis in at least one other vascular bed, including carotids and coronary, and with a diabetic history, aggressive treatment of all co-morbidities could be life-saving. Effient, LMWH, and warfarin are not FDA-approved for treatment of PAD.
Q 16.7: A 51-year-old male patient presents to your family practice office complaining of genital discomfort with dysuria. His digital rectal exam reveals an enlarged, tender prostate. His prostate-specific antigen (PSA) returns elevated with a value of 11.1 mg/mL, which you fractionate, and this reveals approximately 75% free PSA. His urinalysis reveals moderate white cells and trace blood. What would be your next step in treating this patient? A Begin him on 6 weeks of doxycycline to treat his prostatitis and when resolved, repeat his PSA level. B Immediately refer him to a urologist for prostate biopsy to rule out prostate cancer. C Immediately refer him to a urologist for cystoscopy to rule out bladder cancer and perform a computed tomography (CT) scan of the abdomen and pelvis in the interim. D Order a stat testicular sonogram to rule out torsion. E Order a CT scan of the abdomen and pelvis.
The Correct Answer is: A This patient has signs and symptoms consistent with prostatitis. Additionally, while his PSA is elevated, this is common in prostatits as well as prostate cancer, and his free PSA is of a percentage that prostate cancer is unlikely. However, it would be prudent to recheck his PSA after treatment and resolution of his symptoms to confirm that an underlying cancer is not smoldering.
Q 37.3: A 4 year old male presents with a fever for 5 days. His highest temperature was 39.4C. His mother brings him to the ED because she noticed this morning that his palms and soles were red. Now, there is blotchy erythema on the trunk with bulbar conjunctivitis and diffuse erythema on the tongue and prominent papillae. CBC shows leukocytosis. What is the appropriate management of this patient? A Hospitalization and IvIg B Hospitalization ad IV antibiotics C Outpatient antibiotics x 10 days D Outpatient symptomatic treatment
The Correct Answer is: A This patient is exhibiting classic signs and symptoms of Kawasaki Disease. Complications of Kawasaki Disease include coronary artery aneurysms, myocarditis, myocardial ischemia or infarction, and stroke. Recommended treatment is hospitalization to monitor for complications and administration of IvIg with aspirin.
Q 27.4: A 64-year-old African American female presents to the clinic for evaluation of her hypertension, which she has had for several years. In the past she had been taking hydrochlorothiazide and lisinopril, with little effect on her blood pressure management. At today's visit, she has no complaints and feels well. Her vitals show T m 96.6, P 85, R 18, BP 191/99. She has no jugular venous distention seen on the neck exam, her lungs are clear, and cardiac exam has a regular rate and rhythm without murmur or gallop. Her abdomen is soft, non-tender, and a bruit is appreciated at the mid-abdomen just a few centimeters below the epigastric region. There is no fullness or enlargement of the abdominal aorta on palpation. Based on the history and clinical findings, what is the most likely diagnosis? A Renal artery stenosis B Abdominal aortic aneurysm C Mitral stenosis radiating murmur D Tricuspid stenosis E Coarctation of the aorta
The Correct Answer is: A This patient is exhibiting hypertension that appears to be quite high, along with evidence of a bruit on abdominal exam. Because there are no appreciable murmurs on cardiac exam, it would be less likely that a cardiac valvular murmur (C) is the reason. The abdominal aorta is of normal size, and most coarctation murmurs (E) are better appreciated in the back of the chest than on abdominal exam. Because the patient has uncontrolled hypertension despite being on two antihypertensive medications, the most likely reason for the bruit is renal artery stenosis (A). An abdominal aortic aneurysm (B) is less likely given the normal size of the aorta on exam, and tricuspid stenosis (D) would not present with these symptoms.
Q 97.10: A 24-year-old male is brought to the clinic by his mother, who is concerned because her son believes that a local TV news anchor is in love with him. The mother states that this thought has been persistent for the last two to three months, and that he goes around town telling everyone about their relationship; however, she knows that her son does not even know the TV news anchor. The mother notes that this belief has not impaired his daily functioning, but has significantly affected his social functioning. The patient is exhibiting signs most consistent with which of the following psychiatric disorders? A Delusional disorder B Histrionic disorder C Paranoid disorder D Schizoid disorder E Schizotypal disorder
The Correct Answer is: A This patient is exhibiting signs most consistent with delusional disorder. Delusional disorder is a psychosis in which the person has persistent beliefs that are non-bizarre, such as being persecuted, being related to or loved by a well-known person, or that their partner is unfaithful. In this disorder, the delusions tend not to affect the patient's intellectual and occupational activities, but social and marital functioning are significantly affected. Clinical findings of histrionic personality disorder include being dependent, immature, seductive, egocentric, vain, and emotionally labile. Clinical findings of someone who has paranoid personality disorder would include defensiveness, being overly sensitive, secretive, suspicious, and hyper-alert, and having a limited emotional response. Schizoid personality disorder is characterized by shyness, introversion, being withdrawn, and avoiding close relationships. Schizotypal disorder is characterized by being superstitious, socially isolated, and suspicious, with limited interpersonal ability, odd speech, and eccentric behaviors. (McPhee SJ, Papadakis MA. Current Medical Diagnosis & Treatment, 2010, p. 952)
Q 115.17: A 28-year-old male smoker presents with a complaint of numbness and pain in his fingers. He notices this after being exposed to the cold. He states that his fingers appear pale or even blue at times. After warming, his fingers turn red before returning to their normal color. What should be included in appropriate management of this condition? A Counsel the patient to stop smoking B Systemic glucocorticoids C Take aspirin prior to cold exposure D Topical ketoconazole each morning
The Correct Answer is: A This patient is experiencing Raynaud phenomenon. This is digital ischemia that can occur after exposure to cold or emotional stress. It is more common in smokers or patients whose occupation involves using vibratory tools. Management includes patient education to include cold avoidance behavior and wearing loose-fitting clothing. Cessation of smoking is imperative. Drug therapy is used in patients with progressive and severe Raynaud's. (Wolff et al., 2009, Fig. 14-32, pg. 394-395)
Q 38.5: A 14-year-old female presents with a 24-hour history of episodic outbreaks on her hands and feet. She describes the outbreaks as beginning on the sides of her fingers and toes, with small intensely pruritic vesicles. What should be the next step in treatment? A bacterial culture and KOH B punch biopsy and viral culture C viral culture and shave biopsy D shave biopsy and KOH
The Correct Answer is: A This patient is experiencing probable dyshidrotic eczema. It is necessary to rule out a secondary bacterial infection, so a bacterial culture is necessary. It is also necessary to rule out a fungal infection or parasitic by performing a KOH. This does not have any features of a viral infection, so a viral culture is not necessary. A punch biopsy is also not necessary because it can be diagnosed with a non-invasive procedure. A shave biopsy is not indicated because non-invasive techniques can be used to diagnose. Shave biopsies are not indicated when vesicles are present.
Q 63.4: A 77-year-old female has been treated for the last two weeks for a community-acquired pneumonia. While on oral azithromycin, the patient continues to develop fevers, some as high as 103 0 F. Her oral intake has decreased, and her effort to breathe continues to be labored. On examination, the patient continues to have rhonchi and some mild rales that are best appreciated in the anterior right lung region. A follow-up chest x-ray reveals a consolidated infiltrate of the right middle lobe. A CT of the chest reveals a loculated, fluid-filled area of the right middle lobe with no evidence of a foreign body. Based on these new findings, what is the most likely pathogen causing this ongoing infection? A Staphylococcal aureus B Pseudomonas aeurginosa C Hemophilus influenza D Klebsiella pneumoniae E Chlamydia pneumoniae
The Correct Answer is: A This patient is having a history and physical exam that is consistent with an empyema. This loculated collection of fluid will harbor bacteria, the most common pathogen being Staphylococcus aureus. Intravenous antibiotics as well as surgical drainage are warranted.
Q 106.6: An 18-year-old female presents to your office with the complaint of palpitations for the last 2 months. The episodes are frequent and accompanied with lightheadedness and shortness of breath. The patient's mother has taken her pulse when some of the episodes occur and states that the rate gets as high as 170 beats per minute. On exam, she is alert, awake, and oriented. Her resting pulse is 55 and her blood pressure is 122/65. Her lungs are clear throughout, and her cardiac exam revealed a regular rate and rhythm, without murmurs, rubs, or gallops. An ECG is obtained, as shown. Based on her history, physical exam, and ECG, what is the best pharmacologic treatment plan for this patient? A Flecanide B Hydrochlorothiazide C Lisinopril D Adenosine E Digoxin
The Correct Answer is: A This patient is presenting with Wolff-Parkinson-White syndrome, as evidenced by the delta waves on the ECG. These conditions will generally occur in individuals at the onset of early adulthood. Management for this condition pharmacologically includes the use of class IC drugs, such as flecanide. Other choices include procainamide, sotalol, and amiodarone. Digoxin therapy may worsen and widen the QRS complex and place the patient into a ventricular tachycardia. (Calkins H. Hurst's the Heart, Chapter 38, Supraventricular Tachycardia: AV Nodal Reentry and Wolff-Parkinson-White Syndrome)
Q 37.9: A 7-year-old child is brought into the office by her mother who states that the child "is still wetting the bed at night." The child has already decreased liquid intake and uses the bathroom before going to bed. The mother is worried that there is something wrong with the child. Upon examination there is no abnormality. Urinalysis is negative. Which of the following is the treatment of choice for this disorder? A bed-wetting alarm B desmopressin acetate (DDAVP) C imipramine D amitriptyline
The Correct Answer is: A This patient is presenting with signs and symptoms of primary nocturnal enuresis, which is the wetting only at night during sleep without any sustained period of dryness. It is mainly considered a parasomnia occurring in deep sleep. The incidence of enuresis is higher in boys, is typically related to a developmental delay, and most children become continent by adolescence. Patients need to be tested for structural abnormalities and infections, in addition to neurologic diseases, diabetes mellitus and insipidus, and seizure disorders. Treatment includes limiting liquids at bedtime and routine bathroom training during the day. If these are unsuccessful, the next option is a bed-wetting alarm. This device is attached to the child's undergarment and vibrates when the child is wet to arouse the child to be aware of their need to urinate. If the alarm is unsuccessful, then the next step is medication—DDAVP (desmopressin acetate) or imipramine.
Q 91.6: A 59-year-old woman with a known history of rheumatoid arthritis presents with relatively severe complaints of pain, notable bony deformity of the hands with extra-articular findings of cutaneous nodules, scleritis, and pleurisy. On physical examination, the patient is found to have splenomegaly. Which of the following is the most appropriate laboratory evaluation to order to further evaluate the suspected diagnosis? A complete blood count (CBC) B uric acid C C-reactive protein D antinuclear antibodies E erythrocyte sedimentation rate
The Correct Answer is: A This patient presentation of known rheumatoid arthritis with severe deformities, extra-articular findings, and splenomegaly is most likely Felty syndrome. Felty syndrome is characterized by the triad of deforming rheumatoid arthritis, splenomegaly, and neutropenia. The appropriate laboratory test to order would be a CBC to evaluate for neutropenia. Uric acid testing is helpful in evaluating gout but is not relevant to this patient presentation. Ordering an erythrocyte sedimentation rate or C-reactive protein is not necessarily helpful in diagnosing Felty syndrome; in an acute inflammatory flare, both would most likely be elevated. Antinuclear antibodies could be present in 20% to 40% of patients but are not diagnostic of Felty syndrome. (Hellmann and Imboden, 2008, p. 721; Linker, 2008, p. 438) Hellmann DB , Imboden JB. Arthritis and musculoskeletal disorders. In: Tierney LM , McPhee SJ , Papadakis MA, eds. 2008 Current Medical Diagnosis and Treatment. New York, NY: McGraw-Hill; 2008:703-756. Linker CA. Blood disorders. In: Tierney LM , McPhee SJ , Papadakis MA, eds. 2008 Current Medical Diagnosis and Treatment. New York, NY: McGraw-Hill; 2008:422-472.
Q 110.13: A 22-year-old African American male presents to the emergency department with complaints of syncope, which occurred during intense physical exertion. He did not have symptoms prior to exercise, but states that he started having chest pain and shortness of breath right before the syncopal episode. Upon physical examination, he is afebrile, his pulse rate is 93, his respiratory rate is 16, and his blood pressure is 142/100. His lungs are clear, and a cardiovascular examination reveals a bisferiens carotid pulse and a loud S 4 . The electrocardiogram shows a normal sinus rhythm with ventricular hypertrophy, and q-waves in the septal leads. Which of the following tests is most useful to render an accurate diagnosis? A Echocardiography B Holter monitor C Exercise treadmill D Chest x-ray E Electrophysiology study
The Correct Answer is: A This patient presents with a history that is consistent with restrictive cardiomyopathy. While this is not a common diagnosis, it usually presents in younger males who experience symptoms while exerting themselves. Echocardiography is the best assessment test to determine wall motion and thickness of the myocardium. It can also assess any valvular disorders or areas of decreased wall motion abnormalities. (Bashore et al., Current Medical Diagnosis and Treatment, Chapter 10)
Q 38.14: A 15 year old male presents with lesions on his palms, dorsum of his hands and lower arm. They began as red macules that developed a central vesicle over a few days. The lesions are pruritic with no other symptoms. What is the most common etiology? A Herpes simplex infection B Insect bites C Psoriasis vulgaris D Atopic dermatitis
The Correct Answer is: A This patient presents with the classic iris or target lesion of erythema multiforme (EM). The most common cause of recurrent EM is herpes simplex outbreak which usually precedes EM by a few days.
Q 116.16: A 48-year-old man presents to the emergency department with acute right upper quadrant tenderness, fever, and mild jaundice. Which of the following is most likely to be elevated in the blood? A bilirubin B creatinine C glucose D ketones E uric acid
The Correct Answer is: A This patient's signs and symptoms correlate with a suspected case of cholecystitis. Jaundice is associated with hyperbilirubinemia, in which the excess bilirubin can deposit in tissues such as the skin, sclera, and nails, causing a yellowish discoloration. Bilirubin is the waste product generated from the metabolism of hemoglobin. (Greenberger and Paumgartner, 2008, pp. 1995-1996; Pratt and Kaplan, 2008, p. 261) Greenberger NJ , Paumgartner G. Diseases of the gallbladder and bile ducts. In: Fauci AS , Braunwald E , Kasper DL, et al., eds. Harrison's Principles of Internal Medicine. 17th ed. New York, NY: McGraw-Hill Medical; 2008. Pratt DS , Kaplan MM. Jaundice. In: Fauci AS , Braunwald E , Kasper DL, et al. eds. Harrison's Principles of Internal Medicine. 17th ed. New York, NY: McGraw-Hill Medical; 2008.
A 33-year-old male is brought by rescue to the emergency department after he had a collision on the side of the road on his motorcycle. He is complaining of pain and swelling of the right ankle, with decreased range of motion and pain to the extremity. You decide to order a radiograph of his right ankle (with results shown below). Based on the radiography provided, what is the nature of this injury? A Bimalleolar ankle fracture involving the distal tibia and fibula B Distal fibula fracture only C Distal tibia fracture only D Lisfranc fracture - dislocation E Talus fracture
The Correct Answer is: A This radiograph clearly shows fractures of both the distal fibula and tibia (see arrows below). Bimalleolar ankle fractures can also present with a clear fibular fracture and signs of deltoid ligament disruption on the medial side (a palpable gap on the medial side, deltoid ligament tenderness or laxity with an eversion-type stress). This type of injury is considered unstable and generally requires surgical management. Choices "B" and "C" were incorrect since there is clear evidence of fractures on both the lateral and medial aspect of the joint pictured. A Lisfranc fracture - dislocation would seem unlikely as the tarsometarsal joints appear intact, but this is not an ideal view to rule out that injury conclusively. This x-ray does not have any evidence of a talus fracture, but additional views would be needed to be sure no such fracture is present.
Q 28.7: A 33-year-old male is brought by rescue to the emergency department after he had a collision on the side of the road on his motorcycle. He is complaining of pain and swelling of the right ankle, with decreased range of motion and pain to the extremity. You decide to order a radiograph of his right ankle (with results shown below). Based on the radiography provided, what is the nature of this injury? A Bimalleolar ankle fracture involving the distal tibia and fibula B Distal fibula fracture only C Distal tibia fracture only D Lisfranc fracture - dislocation E Talus fracture
The Correct Answer is: A This radiograph clearly shows fractures of both the distal fibula and tibia (see arrows below). Bimalleolar ankle fractures can also present with a clear fibular fracture and signs of deltoid ligament disruption on the medial side (a palpable gap on the medial side, deltoid ligament tenderness or laxity with an eversion-type stress). This type of injury is considered unstable and generally requires surgical management. Choices "B" and "C" were incorrect since there is clear evidence of fractures on both the lateral and medial aspect of the joint pictured. A Lisfranc fracture - dislocation would seem unlikely as the tarsometarsal joints appear intact, but this is not an ideal view to rule out that injury conclusively. This x-ray does not have any evidence of a talus fracture, but additional views would be needed to be sure no such fracture is present.
Q 98.64: A 33-year-old male is brought by rescue to the emergency department after he had a collision on the side of the road on his motorcycle. He is complaining of pain and swelling of the right ankle, with decreased range of motion and pain to the extremity. You decide to order a radiograph of his right ankle (with results shown below). Based on the radiography provided, what is the nature of this injury? A Bimalleolar ankle fracture involving the distal tibia and fibula B Distal fibula fracture only C Distal tibia fracture only D Lisfranc fracture - dislocation E Talus fracture
The Correct Answer is: A This radiograph clearly shows fractures of both the distal fibula and tibia (see arrows below). Bimalleolar ankle fractures can also present with a clear fibular fracture and signs of deltoid ligament disruption on the medial side (a palpable gap on the medial side, deltoid ligament tenderness or laxity with an eversion-type stress). This type of injury is considered unstable and generally requires surgical management. Choices "B" and "C" were incorrect since there is clear evidence of fractures on both the lateral and medial aspect of the joint pictured. A Lisfranc fracture - dislocation would seem unlikely as the tarsometarsal joints appear intact, but this is not an ideal view to rule out that injury conclusively. This x-ray does not have any evidence of a talus fracture, but additional views would be needed to be sure no such fracture is present.
Q 45.10: A 16-year-old male complains of a recurrent rash that is noticed each year during the summer. He states that the rash is asymptomatic but is spreading. A physical exam shows small hypopigmented macules with fine scale. A KOH exam shows budding yeast. What is the most likely diagnosis? A tinea versicolor B atopic dermatitis C post inflammatory hypopigmentation D tinea corporis
The Correct Answer is: A Tinea versicolor is a yeast infection that primarily affects teens and young adults. The area of infection is usually the upper back, upper chest, and lower face. It occurs in warm, humid environments and can recur yearly. The appearance of the infection can be hypo- or hyper-pigmented, slightly scaling macules. The characteristic microscopic appearance on a KOH is described as "spaghetti and meatballs," due to the shortened hyphae and spores of the yeast. Atopic dermatitis in a teen patient would normally be described as red scaling plaques that affect the flexural surface. Atopic dermatitis is also described as being pruritic. Post inflammatory hypopigmentation occurs following an inflammatory reaction on the skin. The hypopigmentation is not scaling and will have a negative KOH. Tinea corporis can be hypopigmented and scaling, but usually exhibits central clearing and pruritis. It does not recur seasonally and affects exposed surfaces. A KOH will show branching hyphae and spores.
Q 95.1: Which of the following medications is NOT recommended by the American Neurological Association based upon Level I evidence to treat severe and unrelenting neuropathic pain because of significant side effect potential? A amitryptyline B duloxetine C gabapentin D methadone E pregabalin
The Correct Answer is: A Tricyclic antidepressants (TCAs) may be utilized to treat neuropathic pain. However, amitryptyline has relatively higher adverse events, particularly in the elderly population, especially when compared to secondary amine TCAs, such as desipramine. (Fauci et al., 2008, Chapter 12)
Q 97.9: Of the following choices, which regimen is considered first-line therapy for Helicobacter pylori-positive individuals with peptic ulcer disease? A omeprazole & clarithromycin & amoxicillin B omeprazole & ranitidine & clarithromycin C esomeprazole & clarithromycin & ampicillin D ranitidine & amoxicillin & bismuth subsalicylate E misoprostol & clarithromycin & metronidazole
The Correct Answer is: A Triple-therapy regimens consisting of a proton pump inhibitor (PPI) and two antibiotics are considered first-line therapy for the eradication of Helicobacter pylori. PPI-based regimens that combine clarithromycin and amoxicillin or clarithromycin and metronidazole have been shown to have the most effective eradication rates. There are also 4-drug regimens that include bismuth subsalicylate that have been shown to be effective as well. Because of lower eradication rates, it is not recommended that histamine receptor antagonists like ranitidine be substituted for a PPI. Misoprostol is used for reducing the risk of nonsteroidal anti-inflammatory agent (NSAIA)-induced gastric ulcer in patients at high risk of developing complications from these ulcers and in patients at high risk of developing gastric ulceration. It has no effect on H pylori eradication. (Berardi and Welage, 2008, pp. 577-578) Berardi RR , Welage LS. Peptic ulcer disease. In: DiPiro JT , Talbert RL , Yee GC, et al., eds. Pharmacotherapy: A Pathophysiologic Approach. 7th ed. New York: McGraw-Hill; 2008.
Q 114.10: A 78-year-old male presents to the office due to increasing exertional dyspnea and cough for the past week. Physical exam reveals an S3 gallop, mild JVD, and 2+ pitting edema of the bilateral lower extremities. The patient has had mild congestive heart failure in the past. A chest x-ray reveals prominent pulmonary vasculature without any additional complications. Which of the following is released by the ventricular myocardium in response to elevated ventricular pressure and overload? A B-Type natriuretic peptide (BNP) B Creatine kinase MB (CK-MB) C Plasma d-dimer D Total creatine kinase (CK) E Troponin
The Correct Answer is: A Two markers, B-type natriuretic peptide (BNP) and N-terminal pro-BNP, provide representation of the ventricular response to volume and pressure overload, and are elevated in hypervolemic states. These markers provide diagnostic information, including differentiating dyspnea causes. They may also be used to monitor CHF patient prognosis and progression. Increased levels of the markers, although sensitive, are not necessarily specific for CHF, and can be elevated in multiple disease processes and patient populations, such as the elderly and women. (McPhee SJ, Papadakis MA. Current Medical Diagnosis & Treatment 2011, Chapter 10, Heart Disease)
Q 63.10: A 25-year-old woman presents not feeling well 1 week after returning from a trip to central Africa. She has had steadily increasing fever, abdominal distention, and diarrhea. She also has rashes on her abdomen, chest, and back, which are characterized by 3-mm pink papules, which blanch with pressure. Heart rate is 60 beats/min. Blood culture is positive but final identification is pending. Most likely diagnosis is A typhoid fever B yellow fever C malaria D hepatitis E shigellosis
The Correct Answer is: A Typhoid fever is caused by Salmonella typhus. It is contracted by contaminated food or water. There are several endemic areas throughout Africa. Symptoms and signs may be nonspecific but often include blanchable, pink, papular rash over the trunk and fever that increases in stepwise fashion. Blood culture is positive in 80% of cases in the first week. Abdominal symptoms may include distention and constipation, initially, followed by diarrhea and, possibly, splenomegaly. Prevention is accomplished by multidose oral or single-dose vaccine.
Q 70.4: A 75-year-old woman, mother of four, presents to your office to establish care. Appearing healthy, she reports a past medical history positive for hypertension and denies any additional problems. However, when specifically asked she admits to having urinary incontinence for "a couple of years" and now describes symptoms that have recently worsened, with the patient experiencing the need to void almost hourly. These desires to urinate are so severe that she is now using four to five adult incontinence pads per day to manage the urine she leaks. What is the most likely diagnosis? A urge incontinence B stress incontinence C overflow incontinence D functional incontinence
The Correct Answer is: A Urinary incontinence is defined as involuntary urine loss. Urge incontinence is the result of uninhibited urge sensations that are so strong that the patient experiences an involuntary urine loss. Women particularly experience this problem with the changes associated with aging (weakened pelvic muscles secondary to childbirth as well as estrogen depletion causing weakening of the detrusor muscle). The problem may be worsened by the use of diuretics to treat hypertension. Stress incontinence is associated with increases in intra-abdominal pressure (laughing, sneezing, coughing, etc.). Overflow incontinence is associated with leaking small amounts of urine from mechanical factors that affect an already distended bladder. Functional incontinence is associated with patients who exhibit cognitive impairment (severe dementia). (Johnston et al., 2009, pp. 66-68) Johnston CB , Harper GM , Landefeld CS. Geriatric disorders. In: McPhee SJ , Papadakis MA, eds. Current Medical Diagnosis and Treatment. 48th ed. New York, NY: McGraw-Hill; 2009:56-71.
Q 39.29: A 16-year-old boy is seen for a sports physical prior to starting football. On auscultation, a grade II/IV holosystolic murmur is appreciated at the apex. Which of the following maneuvers would be the most appropriate to choose to increase the intensity of the murmur for better identification? A Isometric hand grip exercise B Listening with the bell at the apex, with the patient in the left lateral decubitus position C Inspiration, followed by the patient holding his/her breath D Valsalva maneuver E Having the patient lie flat with the knees bent
The Correct Answer is: A Utilizing isometric hand grip exercises, the murmur increases in intensity and may be heard radiating to the axilla. Isometric hand grip exercises increase the intensity of the murmur of mitral regurgitation by increasing arterial and left ventricular pressure, which increases the flow across the mitral valve, thereby increasing the murmur's intensity. Choice B is best used when listening to the murmur of mitral stenosis. Choice C will increase the AP diameter, making it more difficult to hear the murmur. With the Valsalva maneuver, choice D, the murmur decreases in intensity. Choice E is the best position for the abdominal exam, especially in males.
Q 77.3: A 62 year old male presents with a concern regarding the numerous brown, warty looking papules on his back. They are asymptomatic but sometimes get caught on clothing which can cause irritation and bleeding. What is the most likely diagnosis? A Seborrheic keratosis B Melanocytic nevus C Verruca vulgaris D Malignant melanoma
The Correct Answer is: A Seborrheic Keratoses are described as brown warty looking papules that have a "stuck on" appearance. They are more common in patients over 50 years old. (Wolff & Johnson, pg 215) (Fig 9-49a & c, Wolff K, Johnson RA: Fitzpatrick's Color Atlas & Synopsis of Clinical Dermatology, 6 th ed: http://www/accessmedicine.com)
Q 77.6: A 34-year-old male has a one and one-half day history of fever, chills, a non-productive cough, and malaise. He is otherwise healthy with no long-standing medical history, and is taking no chronic medications. On examination, the patient has a temperature of 101.3 0 F, BP 123/63, P 78, R 18. His HEENT reveals mild rhinorrhea, moist mucous membranes, clear lung sounds, and a regular rate and rhythm. The rapid nasal viral test for influenza B is positive. Based on this information, what is the medication treatment for this patient? A Amantadine B Oseltamavir C Ramantadine D Famciclovir E Azithromycin
The Correct Answer is: B Oseltamivir is the best antiviral medication for the treatment of acute influenza. This medication is ideally started within the first onset of illness, usually within the first 24 to 36 hours. Amantadine and Ramantidine have been shown to not be effective, and there is growing resistance to the medication. (Shandera WX, Patel S. Current Medical Diagnosis and Treatment, 2011, Chapter 32, Viral and Riskettsial Infections)
Q 77.5: A 72-year-old farmer presents with multiple rough adherent scaly lesions on his scalp. They are better felt than seen. He reports mild pain when he inadvertently scratches them. What would the most effective management include for this patient? A shave excision B 5-FU cream 5% C doxycycline D no treatment is required
The Correct Answer is: B These lesions are consistent with actinic keratoses. Treatment is indicated, as 10% of these lesions progress to squamous cell carcinoma. First line treatment is 5-Fu cream 5%. Excision is not indicated for actinic keratoses. Doxycycline is an antibiotic used to treat acne and various bacterial infections, and it is not an effective treatment for actinic Keratosis. (Wolff et al., 2009, Page 268)The Correct Answer is: B These lesions are consistent with actinic keratoses. Treatment is indicated, as 10% of these lesions progress to squamous cell carcinoma. First line treatment is 5-Fu cream 5%. Excision is not indicated for actinic keratoses. Doxycycline is an antibiotic used to treat acne and various bacterial infections, and it is not an effective treatment for actinic Keratosis. (Wolff et al., 2009, Page 268)
Q 34.4: A 13-year-old female is seen for the first time to establish care. She is known to carry the diagnosis of beta-thalassemia major and has been maintained on regular transfusions for her anemia. She is short for her age and has not achieved menarche. Her glucose level is elevated and she has developed signs of diabetes mellitus. These findings are consistent with which of the following? A End stage thalassemia B Inadequate iron chelation C Inadequately transfused anemia D Stage 3 thalassemia E Untreated thalassemia
The Correct Answer is: B *Inadequate iron chelation in patients with thalassemia major results in the absence of a pubertal growth spurt and failure of menarche. These patients may also develop diabetes mellitus, as well as other endocrine disturbances. * There is no staging to thalassemia, and untreated or inadequately transfused anemia results in increased infections early in life, usually causing death, spontaneous fractures, and other deformities. Additionally, thrombocytopenia and leucopenia may develop.
Q 99.1: A 45-year-old male presents with a non-tender nodule protruding from his lower eyelid. There is some surrounding erythema to the conjunctiva, but no discharge is seen. He states that it has been there for one month. He has no visual problems. What is your diagnosis? A Blepharitis B Chalazion C Dacryocyctitis D Hordeolum E Conjunctivitis
The Correct Answer is: B A chalazion is a sterile, chronic, and non-painful granulomatous nodule, caused by a previous acute infection in a meibomian gland. It can develop over a period of a few weeks. Treatment is intralesional steroids or surgical curettage. (Riordan et al., 2008, Chapter 4)
Q 12.6: What diagnosis should be given to a patient who has nonbizarre delusions for at least a month and no other symptoms? A schizoaffective disorder B delusional disorder C brief psychotic disorder D schizophreniform disorder
The Correct Answer is: B A delusional disorder presents with nonbizarre delusions for at least a month. The disorder does not present with any other symptoms related to schizophrenia or a mood disorder. A brief psychotic disorder has symptoms that last for 1 day to 1 month. Schizophreniform has symptoms that last at least a month, but no longer than 6 months. In schizoaffective disorders, depression or mania develop along with schizophrenic symptoms.
Q 22.10: A 29-year-old female presents with a "bump" on her wrist for the past week. She denies any pain or history of trauma. Her past medical history is unremarkable. On physical examination you discover a 2.0 cm nontender, firm cystic lesion on the dorsum of the wrist. Considering your suspected diagnosis what is the best treatment option? A IV antibiotics B Aspiration and injection of a corticosteroid C Physical therapy D Hold pressure on the cyst until it ruptures
The Correct Answer is: B A ganglion cyst is a cystic collection of synovial fluid within a joint or tendon sheath. Treatment options include cyst aspiration and corticosteroid injection (B), or surgical excision. However, none are typically performed in the ED. Surgery is generally effective but ganglion cysts may reoccur. Referral to a hand surgeon is indicated for persistent or recurrent pain or cosmetic deformity. IV antibiotics (A) would not help since ganglion cysts are not known to be infectious. Physical therapy (C) would aggravate the cyst and would not reduce the size typically. Rupturing the cyst (D) would temporarily reduce the size but recurrence is high.
Q 23.10: What type of fracture is not related to an acute bony trauma? A Greenstick B Stress C Oblique D Comminuted E Spiral
The Correct Answer is: B A stress or fatigue fracture is caused by small, repetitive forces that usually involve the metatarsal shafts, the distal tibia, and the femoral neck (though many other bones may be affected). These fractures may not be seen on initial radiographs. A greenstick fracture is an incomplete traumatic fracture with angular deformity seen in children. An oblique fracture is a traumatic fracture with an angulated fracture line. A comminuted fracture is a traumatic fracture in which there are more than two fracture segments. A spiral fracture is a traumatic fracture that has a multiplanar and complex fracture line usually caused by an excessive rotational force on a bone.
Q 98.67: What type of fracture is not related to an acute bony trauma? A Greenstick B Stress C Oblique D Comminuted E Spiral
The Correct Answer is: B A stress or fatigue fracture is caused by small, repetitive forces that usually involve the metatarsal shafts, the distal tibia, and the femoral neck (though many other bones may be affected). These fractures may not be seen on initial radiographs. A greenstick fracture is an incomplete traumatic fracture with angular deformity seen in children. An oblique fracture is a traumatic fracture with an angulated fracture line. A comminuted fracture is a traumatic fracture in which there are more than two fracture segments. A spiral fracture is a traumatic fracture that has a multiplanar and complex fracture line usually caused by an excessive rotational force on a bone.
Q 13.1: A parent brings her child into the office with the concern of the way he stands. To the parent the child looks like they are knocking knees. If the child is indeed presenting this way, what would be the best description of this orthopedic abnormality? A Angulation of an extremity at a joint with the more distal part angled anteriorly. B Angulation of an extremity at a joint with the more distal part angled away from the midline. C Angulation of an extremity at a joint with the more distal part angled toward the midline. D Angulation of an extremity at a joint with the more distal part angled posteriorly. E Angulation of an extremity at a joint with the more proximal part angled away from the midline.
The Correct Answer is: B A valgus deformity involves angulation of an extremity at a joint with the more distal part angled away from the midline. When looking at the anterior aspect of a patient in the anatomic position, a valgus deformity of the left knee or left elbow would look like the letter L.
Q 97.69: A 26-year-old athlete presents complaining of scant menses x 4 months. She is a G0P0, menarche was at age 13, and her menses have been mostly regular. She is a non-smoker and non-drinker, and has been trying to achieve pregnancy x 8 months. Physical exam reveals a thin, white female in no distress. Vitals are normal, BMI is 17.5, her pelvic exam is normal, and STD cultures are negative. A pregnancy test is negative in clinic. What is the most likely cause for her amenorrhea? A Idiopathic B Hypothalamic C Hypothyroid D Polycystic ovarian syndrome E Androgenic
The Correct Answer is: B A young healthy woman with a low BMI, no other signs of virilization, and a history of normal menarche is most likely to have induced amenorrhea, due to suppression of the hypothalamic axis from low weight and fat index.
Q 97.56: A 29-year-old G3P2 presents to the office for her obstetrical visit. She is currently 16 weeks gestation by ultrasound. Her pregnancy has had no complications to date. She is a non-smoker and takes her prenatal vitamin. What routine obstetric lab should be offered at this visit? A Amniocentesis B Alpha fetal protein screen C 3D ultrasound D D. Glucose challenge E E. HIV testing
The Correct Answer is: B AFP testing is only available between 15 and 20 weeks gestation. HIV is done on initial visit and amniocentesis is offered for risk factors, advanced age, or abnormal AFP. 3D ultrasound is not routine standard of care, and diabetic screening is done between 24 and 32 weeks gestation.
Q 110.1: In a patient who does not have thyroid disease, an elevated serum thyroid stimulating hormone (TSH) may be found under which of the following conditions? A acute corticosteroid administration B acute psychiatric admission C development of an hCG-secreting tumor D pregnancy E use of amphetamines
The Correct Answer is: B About 15% of patients who are admitted for an acute psychiatric illness will have an elevated TSH in the absence of thyroid disease. Acute corticosteroid administration (A), hCG-secreting tumors (C), pregnancy (D), and use of amphetamines (E) are all associated with low TSH levels. Fitzgerald PA, Endocrine Disorders, in Current Medical Diagnosis and Treatment, 52 nd ed. 2013.
Q 95.5: A 30-year-old man presents complaining of back pain radiating down his right leg. On examination, you note that his knee jerk reflex is absent on the right. This finding suggests compression of which spinal nerve root? A L1-L2 B L3-L4 C S1-S2 D T11-T12 E C5-C6
The Correct Answer is: B Absence of the knee jerk reflex suggests compression of the L3-L4 spinal nerve root. The four most commonly tested deep tendon reflexes are the Achilles (ankle jerk) reflex, quadriceps (knee jerk) reflex, triceps reflex, and the biceps reflex. The nerve roots that each tests in ascending order are 1 and 2, 3 and 4, 5 and 6 (biceps), and 7 and 8 (triceps). One only needs to remember that the ankle jerk is a sacral nerve root, the knee jerk is a lumbar nerve root, and the biceps and triceps are cervical nerve roots. (Aminoff et al., 2005, p. 367) Aminoff MJ , Greenberg DA , Simon RP. Clinical Neurology. 6th ed. New York, NY: McGraw-Hill; 2005.
Q 97.125: A 21-year-old female returns to clinic for treatment of her abnormal pap smear. She had been sexually active since age 18. She uses oral contraceptives for birth control. Her pap smear showed mild cervical intraepithelial neoplasm, with positive high-risk HPV. What is the most appropriate next step for management of her pap smear? A HPV testing in 6 months B Pap smear in 6 months C Colposcopy with directed biopsy D LEEP procedure E STD testing
The Correct Answer is: B According to new ACOG guidelines, cancer screening should begin at age 21. Routine follow up is based on normal pap smear and no history of high-risk HPV. The presence of high-risk HPV necessitates the exclusion of a higher grade cervical lesion.
Q 112.1: A 35-year-old female presents with persistent erythema, with telangiectasias on the central portion of her face. She also experiences exacerbations of red papules and small pustules. What is the most likely diagnosis? A acne vulgaris B acne rosacea C periorificial dermatitis D systemic lupus erythematosus
The Correct Answer is: B Acne rosacea is differentiated from acne vulgaris by the absence of comedonal lesions. The hallmark of rosacea is inflammatory lesions and telangiectasias distributed in the central face. In addition, flushing is common in response to change in temperature, spicy foods, and stress. Acne vulgaris consists of comedonal lesions and inflammatory papules and pustules. There is no association of telangiectasias with acne vulgaris. This is an inflammatory condition, with a slight preponderance in females. It consists of small, red papules located around the mouth and sometimes adjacent to the nose. The rash from systemic lupus erythematosus is described as well defined erythema in the malar distribution. There may be fine scaling, erosions, or crusts. (Wolff et al., 2009, Page 9)
Q 97.118: Which of the following is the drug of choice for treating herpes simplex virus (HSV) types 1 and 2? A amantadine B acyclovir C zidovudine D nystatin E zanamivir
The Correct Answer is: B Acyclovir is the treatment of choice for HSV disease, typically in oral doses of 200 mg five times daily or 400 mg three times daily. In situations where oral acyclovir cannot be absorbed effectively by the GI tract or tolerated by the patient, intravenous acyclovir can be administered at a rate of 15 mg/kg/day.
Q 117.12: Which of the following is the drug of choice for treating herpes simplex virus (HSV) types 1 and 2? A amantadine B acyclovir C zidovudine D nystatin E zanamivir
The Correct Answer is: B Acyclovir is the treatment of choice for HSV disease, typically in oral doses of 200 mg five times daily or 400 mg three times daily. In situations where oral acyclovir cannot be absorbed effectively by the GI tract or tolerated by the patient, intravenous acyclovir can be administered at a rate of 15 mg/kg/day. (Knodel, 2008, p. 1926) Knodel LC. Sexually transmitted diseases. In: DiPiro JT , Talbert RL , Yee GC, et al., eds. Pharmacotherapy: A Pathophysiologic Approach. 7th ed. New York: McGraw-Hill; 2008.
Q 99.8: Which of the following are common adverse effects associated with aminoglycosides? A diarrhea and bone marrow depression B ototoxicity and nephrotoxicity C blurred vision and hyperglycemia D headache and hypoglycemia E rash and dyspepsia
The Correct Answer is: B All aminoglycosides are ototoxic and nephrotoxic. The likelihood of experiencing these toxicities occurs when treatment lasts beyond 5 days, at higher doses, in elderly patients, and those suffering from renal insufficiency. Other agents that produce either of these toxicities should not be used concurrently. (Craig and Stitzel, 2004, pp. 541-542) Craig CR , Stitzel RE. Modern Pharmacology with Clinical Applications. 6th ed. Baltimore, MD: Lippincott Williams & Wilkins; 2004.
Q 70.9: Your patient has symptoms consistent with perennial allergic rhinitis, and after performing a history and physical examination, you elect to perform an IgE-specific serum antibody test for both food and respiratory allergens. The results return and the patient does not have an IgE positive response to a single allergen tested, yet the patient's total serum IgE is elevated dramatically. What would be the most appropriate next step in the diagnosis and treatment of this patient? A Repeat the test—it is clear that an error was made or samples were switched. B Perform a more detailed history carefully looking for the offending allergen(s) and expand the serum IgE-specific antibody testing to include additional potential triggers. C Perform the more sensitive serum IgM-specific antibody testing. D Refer the patient to an allergist for definitive skin testing. E Clearly this is one of the many forms of nonallergic rhinitis; treat the patient accordingly.
The Correct Answer is: B Allergen-specific serum IgE testing is an easy and accurate method for determining the presence of atopic allergy, and with newer in vitro technology available, in vitro testing is at least equivalent to skin testing in efficacy. In vitro assays are safe, specific, cost-effective, and reproducible, and do not require the patient to be free of antihistamines and other medications that may interfere with skin testing. They are also easy and quick and are therefore preferred, especially in children and in anxious patients. Although the original in vitro assay, the RAST test (radioallergosorbent test), is no longer performed, its name is still used today to generally describe IgE-specific blood testing. However, not all in vitro assays available today are alike. The newer assays tend to be faster, more reliable, and more efficient than previous tests. The ImmunoCAP is an excellent example of this newer technology. Not using a reliable assay may affect the diagnosis of atopy and therefore the prescribing of appropriate therapy In vitro testing can be cost-effective if an appropriately chosen inhalant screening battery of 10 to 12 allergens consisting of the most prevalent pollens, molds, dust mites, and animals in the local environment is used. In children, common allergenic foods are substituted or added. No further testing is necessary if this battery is negative. If the screening battery is positive and if no immunotherapy is considered, additional allergy testing can be performed. (Lalwani, 2008, Chapter 13)
Q 71.3: A 47-year-old female patient is diagnosed with a duodenal ulcer. She was determined to have H. pylori infection. Which of the following medications used to treat peptic ulcer disease is specifically used to eradicate H. pylori? A Omeprazole B Amoxicillin C Ranitidine D Lansoprazole E Misoprostol
The Correct Answer is: B Amoxicillin is an antibiotic that is specifically used to eradicate the H. pylori bacteria. Omeprazole and lansoprazole are proton pump inhibitors, and are used in peptic ulcer disease to suppress acid production in the stomach. Ranitidine is an H2 receptor antagonist, and its role is also to reduce acid production in the stomach. Misoprostol is a prostaglandin analog that stimulates gastric and duodenal mucus and bicarbonate secretion. (McPhee SJ, Papadakis MA. Current Medical Diagnosis & Treatment, 2010, p. 550)
Q 109.5: Which of the following clinical manifestations is most characteristic of polymyalgia rheumatica (PMR)? A subcutaneous inflammatory lesions B pain and stiffness of proximal muscle groups C insidious onset of symmetrical joint involvement D widespread musculoskeletal pain and tender points E symmetrical weakness initially in the legs that progresses caudally
The Correct Answer is: B An abrupt onset of proximal muscle pain and stiffness in the shoulder and pelvic girdle areas, usually associated with fever, malaise, and weight loss, is characteristic of polymyalgia rheumatica. Subcutaneous inflammatory lesions denote erythema nodosum. These lesions are associated with pregnancy and several systemic disorders, such as sarcoidosis, tuberculosis (TB), and streptococcal infections. Insidious onset of symmetrical joint involvement is most commonly associated with rheumatoid arthritis. Widespread musculoskeletal pain and tender points, referred to as "trigger points," are seen with fibromyalgia syndrome. Trigger points may be found anywhere on the body but are most common in the neck, shoulders, hands, low back, and knees. Symmetrical weakness initially in the legs that progresses caudally is characteristic of Guillain-Barré syndrome. (Hellmann and Imboden, 2008, p. 739) Hellmann DB , Imboden JB. Arthritis and musculoskeletal disorders. In: Tierney LM , McPhee SJ , Papadakis MA, eds. 2008 Current Medical Diagnosis and Treatment. New York, NY: McGraw-Hill; 2008:703-756. Explanation
Q 35.4: A mother presents with a 2-month-old infant with a concern of bald spots in the child's scalp. She states that there were ulcerated areas present at birth that healed within a few days. What is the most likely diagnosis? A Alopecia areata B Aplasia cutis congenita C Epidermolysis bullosa D Seborrheic dermatitis
The Correct Answer is: B Aplasia cutis congenita is a rare condition that is present at birth, and presents as asymptomatic ulcerations of the scalp. These ulcerations heal with scarring in a matter of weeks. The cause is believed to be incomplete neural tube closure or cessation of skin development of the embryo.
Q 43.9: A 6-year-old male child presents to the clinic for a cough that occurs only after he has been running, according to his mother. She says she first noticed this about 6 months ago, after he had had one of his usual winter colds, and his cough persisted for about a week. On the basis of this history, what is the most likely diagnosis? A airway foreign body B asthma C cystic fibrosis D laryngomalacia
The Correct Answer is: B Asthma, in this case exercise-induced, is the most likely cause of this problem. The symptoms commonly associated with acute exacerbations of asthma include wheezing, cough, dyspnea, and chest pain. Some symptoms that might be suggestive of asthma include exercise-induced cough, nighttime cough, cough after cold air exposure, and cough after laughing. Airway foreign bodies, though not common, are an acute problem that may present as sudden cough, choking, and wheezing. Cystic fibrosis (CF) is the most common, lethal, genetic disease affecting the Caucasian population. Up to 50% of patients with CF are diagnosed in infancy, but others may not be diagnosed until adolescence or adulthood. Chronic or recurrent cough should be an indicator for consideration of CF as a differential diagnosis. Laryngomalacia is the most common cause of stridor in infants. It is the incomplete development of the cartilaginous support of the laryngoglottic structures. This congenital condition is usually self-limiting and occurs most commonly in infants at or just after birth. The inspiratory collapse of the epiglottis or arytenoid cartilages is heard as stridor.
Q 12.7: A 64-year-old female presents to your family practice office with a complaint of shortness of breath, dizziness, and fatigue over the past seven days. She states that she has experienced a similar episode twice in the past seven months, but it has resolved spontaneously in the past. However, this time she states it is different and is concerned about her condition. Her vital signs are stable and ECG is below. What is your diagnosis and which of the following is the appropriate initial treatment? A paroxysmal supraventricular tachycardia; start her on metoprolol ER 50 mg once daily and refer her to cardiology B persistent atrial fibrillation with a controlled rate; start her on warfarin 5.0 mg and check her INR in 5 days C sinus arrhythmia; reassure her that this is perfectly normal, associated with respirations, and that nothing further needs to be done D tell her that her ECG shows some irregularity; order a 24-hour holter monitor E she is having an anterior-inferior wall myocardial infarction; give her a full-dose aspirin and call 911
The Correct Answer is: B Atrial fibrillation is the most common chronic arrhythmia, with an incidence and prevalence that rises with age, so that it affects approximately 10% of individuals over 80 years old. It occurs in rheumatic and other forms of valvular heart disease, dilated cardiomyopathy, atrial septal defect (ASD), hypertension, and coronary heart disease. It also occurs in patients with no apparent cardiac disease; it may be the initial presenting sign in thyrotoxicosis, and this condition should be excluded with the initial episode. The heart rate may range from quite slow to extremely rapid, but is uniformly irregular unless there is a underlying complete heart block and a permanent ventricular pacemaker is in place. The surface ECG typically demonstrates erratic, disorganized atrial activity between discrete QRS complexes occurring in an irregular pattern. The atrial activity may be very fine and difficult to detect on the ECG, or quite course and often mistaken for atrial flutter. Atrial fibrillation often appears paroxysmally before becoming the established rhythm.
Q 111.8: The diagnosis of systemic lupus erythematosus (SLE) is supported by a positive initial antibody screen; however, this test is not specific. Which of the following tests is most specific in the diagnostic evaluation of SLE? A gliadin antibody B antibody to double-stranded DNA (anti-dsDNA) C antinuclear antibody (ANA) D anticentromere antibody E antiribosomal P antibody
The Correct Answer is: B Autoantibody production is the primary immunological abnormality seen in patients with systemic lupus erythematosus (SLE); the antinuclear antibody (ANA) is most characteristic of SLE and seen in 95% of patients with SLE but is not specific for the diagnosis of SLE. A positive ANA can also be found in patients with lupoid hepatitis, scleroderma, rheumatoid arthritis, Sjögren disease, dermatomyositis, and polyarteritis. ANA testing should be employed as the initial screening test in a patient suspected of having SLE. A negative total ANA test is strong evidence against the diagnosis of SLE, whereas a positive test is not confirmatory of the diagnosis. The most specific antibody tests for SLE are antibodies to double-stranded DNA (anti-dsDNAs) and anti-Smith (anti-SM). Although these tests are more specific for SLE, they are less sensitive than the ANA test. Anti-dsDNA is positive in 60% of patients with SLE and anti-SM is positive in 30% of patients. Anti-dsDNA is more likely to reflect disease activity. Gliadin antibody assay is utilized to assess patients with suspected celiac disease. Anticentromere antibody is associated with CREST (calcinosis, Raynaud phenomenon, esophageal dysmotility, sclerodactyly, and telangiectasia) syndrome in scleroderma. Antibodies to ribonucleoprotein are present in patients with a mixture of overlapping rheumatological symptoms known as "mixed connective tissue disease." (Hellmann and Imboden, 2008, pp. 725-729) Hellmann DB , Imboden JB. Arthritis and musculoskeletal disorders. In: Tierney LM , McPhee SJ , Papadakis MA, eds. 2008 Current Medical Diagnosis and Treatment. New York, NY: McGraw-Hill; 2008:703-756.
Q 11.10: A 23 year-old man presents to the clinic complaining of fever, productive cough and fatigue for 3 days. Physical exam reveals decreased breath sounds in the left lower lobe. He is prescribed azithromycin. Which of the following statements best describes the mechanism of action of azithromycin? A Binds to the 30S ribosomal subunit to inhibit protein synthesis B Binds to the 50S ribosomal subunit to inhibit protein synthesis C Inhibits dihydropteroate synthase and folate production D Inhibits DNA replication by binding to DNA gyrase and topoisomerase IV E Inhibits the transpeptidation reaction
The Correct Answer is: B Azithromycin (B) is a macrolide antibiotic that binds to the 50S ribosomal subunit to inhibit protein synthesis. Beta lactam antibiotics (A) inhibit the transpeptidation reaction leading to cell wall destruction, Ciprofloxacin (C) inhibits DNA replication by binding to DNA gyrase and topoisomerase IV, tetracycline (E) binds to the 30S ribosomal subunit to inhibit protein synthesis, and sulfamethoxazole (D) inhibits dihydropteroate synthase and folate production.
Q 97.86: A 27-year-old female presents to the office complaining of a lump on one side of her vagina. It has been present for several weeks, and causes only slight discomfort with intercourse. She has no new sexual partners and no other vaginal or systemic symptoms. On physical exam you note a 1.5-cm area of swelling in the left posterior labia majora. It is firm, well circumscribed, and minimally tender, and no changes are noted in the skin. No vaginal discharge or other lesions are noted. What is the most likely diagnosis? A Vulvar abscess B Bartholin gland duct cyst C Vulvar lipoma D Acrochordon E Lichen planus
The Correct Answer is: B Bartholin obstruction leads to an asymmetrical posterior labia majora or vestibule, which is not typically painful unless it abscesses. Lipomas are less likely, and lichen planus causes pain and itching without a discrete mass.
Q 120.2: A patient is found with a microcytic anemia. Further laboratory findings include a serum iron value of 26 mcg/dl (50 to 175 mcg/dl), a total iron binding capacity of 376 mcg/dl (250 to 370 mcg/dl), a lactic dehydrogenas of 143 U/L (100 to 190 U/L) and a creatinine level of 0.8 mg/dl (0.1 to 1.5 mg/dl). Which of the following is the best recommendation for therapy? A Ascorbic acid 250 mg po daily B Ferrous sulfate 325 mg po three times daily C Glucophage 500 mg po twice daily D Sodium ferric gluconate 1.5 GMs IM monthly E Vitamin B12 1000 mcg po daily
The Correct Answer is: B Based on laboratory values, this patient has iron deficiency anemia. The most important part of treatment is identifying the cause of the blood loss. Therapy consists of replacement of iron through the use of ferrous sulfate. If the patient is unable to tolerate oral iron dosing, IM sodium ferric gluconate may be used, but only after intolerance of po dosing is demonstrated. Glucophage is used for diabetes. Vitamin B12 is used in vitamin B12 anemia (a macrocytic anemia). Ascorbic acid (vitamin C) is used to increase iron absorption if needed. (McPhee SJ, Papadakis MA, Tierney LM. Current Medical Diagnosis and Treatment, 2010, Chapter 13, Blood Disorders)
Q 118.10: A 42-year-old male presents to the office after awakening with left facial droop, slurred speech, mild left ear pain, and drooling from the left side of his mouth. On physical exam, left facial droop is present, along with the inability to fully close the left eye or raise the left eyebrow. There are no other findings on physical exam. Which of the following is the most likely diagnosis? A Acute transient ischemic attack B Bell palsy C Encephalitis D Intracranial neoplasm E Temporal arteritis
The Correct Answer is: B Bell palsy is classically described as a sudden onset of symptoms involving a unilateral facial nerve, with resultant paralysis. The full face is involved on the affected side, unlike some strokes. The differential diagnoses of stroke, tumor, and various infections must be considered. (McPhee SJ, Papadakis MA. Current Medical Diagnosis & Treatment 2011, Chapter 24, Nervous System Disorders)
A 61-year-old man arrives at the emergency department (ED) suffering an acute myocardial infarction as a result of coronary artery thrombosis. One of the agents administered to the patient is a thrombolytic agent. From the choices below, which drug is a thrombolytic agent? A abciximab B alteplase C warfarin D heparin E clopidogrel
The Correct Answer is: B Both warfarin and heparin are anticoagulants that are indicated for the prevention of thrombi. They do not actively lyse clots, but are capable of preventing further thrombogenesis. Both abciximab and clopidogrel are considered antiplatelet agents. Abciximab inhibits the activation of glycoprotein IIb/IIIa receptors on platelets, which helps to reduce platelet aggregation. Clopidogrel blocks adenosine diphosphate (ADP) receptors on platelets. The binding of ADP to these receptors is an important cellular mechanism in stimulating platelet aggregation. *Alteplase converts plasminogen to plasmin, which then actively dissolves the fibrin threads associated with a thrombus.*
Q 20.1: A 4-month-old, full-term infant female presents to the clinic for a well-child visit. The mother states the baby is nursing well and her growth chart progression is within normal limits. Which of the following supplements is most important for this infant to receive? A Fluoride B Iron C Omega-3 fatty acids D Protein E Vitamin K
The Correct Answer is: B Breastfed infants require additional dietary iron, either through supplements or through consumption of iron rich foods (i.e., fortified cereal). Without supplementation, the infant will deplete her iron stores by about 5-6 months of age and develop iron deficiency anemia. Fluoride (B) supplementation should begin at 6 months of age. Omega-3 fatty acids (C) and protein supplementation (D) are less important in this full-term infant, and vitamin K (E) supplementation is used in the presence of specific diseases (i.e., chronic liver disease).
Q 17.8: A 35-year-old male presents with pain and decreased range of motion after sustaining a fall in which the patient tried to grab onto a bar which pulled his entire arm in the process. Given this clinical scenario, at what cervical motor neuron level would the biceps reflex be testing? A C4 B C5 C C6 D C7 E C8
The Correct Answer is: B C5 is the primary motor neuron being tested in a biceps reflex. C6 contributes to the brachioradialis reflex primarily, but does have a small role in the biceps reflex. The C7 motor neuron is primarily involved in the triceps reflex. C4 and C8 do not contribute to any primary reflexes.
Q 39.24: A 12-year-old boy presents to the clinic for follow-up regarding his recently diagnosed partial seizures. He reports no seizures or side effects since starting carbamazepine (Tegretol) 1 month ago. What study should be ordered to monitor this patient's treatment? A blood glucose B complete blood cell count C electroencephalogram D vitamin B12 E urinalysis
The Correct Answer is: B Carbamazepine is an anti-epileptic drug that potentially causes blood dyscrasias and requires CBC monitoring. Disorders of carbohydrate metabolism, vitamin B 12 deficiency, or renal toxicity are not commonly reported. EEG is used to help establish a diagnosis of a seizure disorder.
Q 98.54: A 22-year-old man is brought to the emergency department by paramedics after having sustained a single stab wound along the left sternal border at the fourth intercostal space. Upon arrival to the emergency department, he was hypotensive and tachycardic. The neck veins were distended and heart sounds were muffled. Which of the following interventions is the most appropriate first-line management of this patient? A Left tube thoracostomy B Pericardiocentesis C Fluid resuscitation D Immediate intubation
The Correct Answer is: B Cardiac tamponade is classically described by the triad of jugular venous distension (JVD), arterial hypotension, and muffled heart sounds. In the emergency department, suspicion of this clinically entity is usually confirmed by ultrasonography and is acutely treated by pericardiocentesis, which will be diagnostic, therapeutic, and buy time until a definitive procedure can be done. A left tube thoracostomy may be indicated in this patient but would not relieve symptoms. Fluid resuscitation though applied to all trauma patients would help stabilize the patient until more therapeutic interventions could be completed. Immediate intubation, even if indicated, would require a prophylactic tube thoracostomy to prevent the development of tension pneumothorax in the event of an unrecognized lung injury. Emergency thoracotomy will relieve the signs and symptoms associated with cardiac tamponade and allow for repair of any underlying cardiac injuries.
Q 117.11: A 57-year-old woman with a history of rheumatic fever is seen complaining of dyspnea while vacuuming her apartment, which has been worsening over the last few months. On physical exam, a possible opening snap, loud S 1 , and a very soft diastolic rumbling murmur is auscultated. Which of the following maneuvers would be the most appropriate to choose to increase the intensity of the murmur for better identification? A Isometric hand grip exercise B Listening with the bell at the apex, with the patient in the left lateral decubitus position C Inspiration, followed by the patient holding his/her breath D Valsalva maneuver E Having the patient lie flat with the knees bent
The Correct Answer is: B Choice B is best used when listening to the murmur of mitral stenosis, which is the murmur auscultated in this patient. Choice A, isometric hand grip exercises, increase the intensity of the murmur of mitral regurgitation by increasing arterial and left ventricular pressure, which increases the flow across the mitral valve, thereby increasing the murmur's intensity. Choice C will increase the AP diameter, making it more difficult to hear the murmur. With the Valsalva maneuver, choice D, the murmur decreases in intensity. Choice E is the best position for the abdominal exam, especially in males. (Crawford et al., 2009, Chapter 1)
Q 89.4: A 55-year-old woman with a history of emphysema, who is undergoing chemotherapy for lung cancer, is sent to see you by her oncologist regarding a sudden increase in dyspnea, with exertion and fatigue. On physical exam, pulsus paradoxus is noted. Which of the following is the most likely diagnosis given the patient's physical exam findings? A Aortic stenosis B Cardiac tamponade C Mitral regurgitation D Hypertrophic cardiomyopathy E Atrial fibrillation
The Correct Answer is: B Choice B is the best choice, as pulsus paradoxus is frequently seen in patients with a pericardial effusion, which may lead to cardiac tamponade. Patients diagnosed with cancer, particularly of the lung and breast, may accumulate fluid within the pericardial sac, leading to cardiac tamponade. The finding of pulsus paradoxus is defined as a decrease in systolic arterial pressure of greater than 10 mmHg. It is an accentuation of the normal decrease in systolic arterial pressure of less then 10mm Hg that normally accompanies inspiration. Pulsus paradoxus is not typically seen in the conditions listed in choices A, C, D, and E. (LeBlond et al., 2009, Chapter 8)
A 65-year-old recent alcoholic comes to the emergency department with recent onset of dyspnea with exertion, 3 pillow orthopnea, lower extremity edema, and palpitations, in which he describes his heart as racing. Which of the following is the most appropriate treatment for his high-output congestive heart failure? A IV dextrose alone B IV thiamine C IV enalapril D IV dopamine E IV diltiazem
The Correct Answer is: B Choice B is the most appropriate treatment, as the patient is demonstrating high output congestive heart failure secondary to beriberi, or thiamine deficiency. In 50% of patients, IV thiamine administration, along with other vitamins and glucose, will resolve the patient's symptoms. Choice A, IV dextrose alone in patients with very low thiamine stores, can worsen signs and symptoms of heart failure. Choice C, IV enalapril, is appropriate therapy for patients in need of better blood pressure control, and as an ACE inhibitor, in patients with left ventricular systolic dysfunction, which is not the cause of this patient's heart failure. Choice D, IV dopamine, is useful in patients in need of pressor support, but will not help treat high-output heart failure secondary to thiamine deficiency. Choice E, IV diltiazem, is useful for heart rate control in patients with atrial fibrillation with a rapid ventricular rate.
A 55-year-old woman with a history of hypertension and 2 vessel CABG presents to the emergency department with increasing dyspnea while walking up one flight of stairs. She denies chest pain and discomfort, but states that for the last 24 hours she has also noticed palpitations. On physical examination, her vital signs are stable. On EKG, she demonstrates atrial flutter with 2:1 AV block. Her echocardiogram demonstrates normal LV systolic function and normal valvular function. Which of the following is the most appropriate therapy for this patient? A IV ibutilide after 4 weeks of anticoagulation with warfarin B IV ibutilide alone C IV quinidine after 4 weeks of anticoagulation with warfarin D IV quinidine alone E IV dopamine
The Correct Answer is: B Choice B, IV ibutilide, is the most appropriate choice for this patient. Therapy for patients with atrial flutter and atrial fibrillation is the same in regards to anticoagulation; therefore, in a patient with a CHADS2 score of 1 and with symptoms of less than 48 hours duration, cardioversion to normal sinus rhythm, whether chemically or electrically, is recommended. Out of all the choices listed, IV ibutilide has been found to be most effective in converting atrial flutter to sinus rhythm. Choices C and D are contraindicated, regardless of the type of anticoagulation paired with it, as quinidine is a class I antiarrhythmic. The atrial conduction may decrease to the point that 1:1 atrial to ventricular conduction can occur with the administration of class I antiarrhythmics. The ventricular rate can then increase to rates greater than 200 bpm, and hemodynamic collapse may occur. Choice E is useful for pressor support, which is not indicated in this patient who is quite stable.
Q 110.19: A 57-year-old man with a history of HTN, hyperlipidemia, and chronic tobacco use, presents to the office with complaints of chest tightness that occurs every time he begins raking leaves. If he stops and rests, it is relieved within 5 minutes. He has no associated nausea or diaphoresis, but does admit to associated dyspnea. Which of the following is the most appropriate next step in the management of this patient? A Cardiac catheterization B Exercise nuclear stress test C Holter monitor D Tilt table test E Transesophageal echocardiogram
The Correct Answer is: B Choice B, exercise nuclear stress testing, would provide information regarding exercise tolerance and exercise-induced dysrhythmias, as well as information regarding myocardial ischemia. Choice A, cardiac catheterization, would be utilized in patients diagnosed with acute myocardial infarction or after a stress test suspicious for myocardial ischemia. Choice C, Holter monitor, is a useful diagnostic tool for the evaluation of patients with palpitations occurring on a daily basis. Choice D, tilt table testing, is utilized in evaluation of patients suffering from near-syncope or syncope. Choice E, transesophageal echocardiogram, is helpful in more direct visualization of heart valves, especially when transthoracic echocardiogram is unclear. (Fauci et al., 2008, Chapter 238)
Q 119.2: A 66-year-old man with a history of a cardiac murmur since childhood presents with complaints of increasing dyspnea while walking up one flight of stairs and increased lower extremity edema. On physical examination, a late-peaking crescendo-decrescendo murmur, preceded by a systolic ejection click, is noted. An S4 gallop is appreciated. Hepatomegaly and splenomegaly are appreciated. An EKG demonstrates right ventricular hypertrophy, and no acute ST or T wave changes. Which of the following is the most appropriate next diagnostic study? A Chest x-ray B Transthoracic echocardiogram C Holter monitor D Treadmill exercise stress test E Cardiac catheterization
The Correct Answer is: B Choice B, transthoracic echocardiogram, is a simple, sensitive and non-invasive diagnostic tool, which can evaluate for the presence of valvulopathy. However, in patients with pulmonic stenosis, it offers limited direct visualization of the pulmonic valve, and although it is the most appropriate next diagnostic study it is usually followed by other diagnostic procedures, such as transesophageal echocardiogram, which offer better visualization of the pulmonic valve directly. Choice A might be able to give evidence of cardiomegaly, or calcification of heart valves, but would not be sensitive enough to detect the degree of valvulopathy if present. Choice C is a useful diagnostic tool for the evaluation of patients complaining of palpitations, but is incorrect for this patient who has no symptoms of palpitations. Choice D, although a useful diagnostic tool for the evaluation of exercise tolerance and in patients complaining of chest pain, does not allow direct visualization of the heart valves to evaluate the degree of pulmonic stenosis. Choice E, cardiac catheterization in patients demonstrating severe pulmonic stenosis, is not only diagnostic, but also therapeutic, as percutaneous balloon valvuloplasty is the preferred method of treatment for critical pulmonic stenosis. (Crawford et al., 2009, Chapter 28)
A 24-year-old HIV-positive man comes to the emergency department complaining of severe left-sided chest discomfort, which radiates through to the left trapezius region. On coming into the room, you note that he is sitting up and hunched forward. On physical examination, the patient's blood pressure is 135/78, with a pulse of 85 bpm, and a pericardial friction rub is noted. Laboratory findings demonstrate elevated serum creatine kinase levels and normal serial troponin levels. His EKG demonstrates peaked T waves. His CXR demonstrates a "water bottle" cardiac silhouette. Which of the following diagnostic studies would be considered the most appropriate next step in management of this patient? A Cardiac catheterization B Transthoracic echocardiography C CT of the thorax D VQ scan E Lower extremity venous doppler
The Correct Answer is: B Choice B, transthoracic echocardiography, would allow for monitoring of a patient with acute pericarditis, to determine if a pericardial effusion and/or cardiac tamponade develops. Choice A, cardiac catheterization, would be appropriate in a patient with acute myocardial infarction. Choice C, CT of the thorax, is not as effective or specific as transthoracic echocardiography for the development of a pericardial effusion and/or tamponade. Choices D and E would be appropriate diagnostic studies if pulmonary embolism is suspected, but not as the next step in management of this patient with pericarditis.
Q 80.5: A 24-year-old HIV-positive man comes to the emergency department complaining of severe left-sided chest discomfort, which radiates through to the left trapezius region. On coming into the room, you note that he is sitting up and hunched forward. On physical examination, the patient's blood pressure is 135/78, with a pulse of 85 bpm, and a pericardial friction rub is noted. Laboratory findings demonstrate elevated serum creatine kinase levels and normal serial troponin levels. His EKG demonstrates peaked T waves. His CXR demonstrates a "water bottle" cardiac silhouette. Which of the following diagnostic studies would be considered the most appropriate next step in management of this patient? A Cardiac catheterization B Transthoracic echocardiography C CT of the thorax D VQ scan E Lower extremity venous doppler
The Correct Answer is: B Choice B, transthoracic echocardiography, would allow for monitoring of a patient with acute pericarditis, to determine if a pericardial effusion and/or cardiac tamponade develops. Choice A, cardiac catheterization, would be appropriate in a patient with acute myocardial infarction. Choice C, CT of the thorax, is not as effective or specific as transthoracic echocardiography for the development of a pericardial effusion and/or tamponade. Choices D and E would be appropriate diagnostic studies if pulmonary embolism is suspected, but not as the next step in management of this patient with pericarditis. (Fauci et al., 2001, pp. 1365-1366)
Q 2.10: Which of the following is a risk factor for cholesteatoma formation? A Acute otitis media B Chronic tympanic membrane retraction C Diabetes mellitus D Hyperlipidemia E Smoking
The Correct Answer is: B Cholesteatomas result from negative pressure in the middle ear chronically retracting the pars flaccida (B). Acute otitis media (A) may lead to short-term TM retraction or perforation, but without perforation or the placement of PE tubes it is unlikely to lead to cholesteatoma formation. Diabetes (C), hyperlipidemia (D), and smoking (E) are not associated with this localized pathology.
Q 110.6: Your 25-year-old female patient is a smoker, takes oral contraceptives, and complains of shortness of breath and wheezing, which forced her to stop smoking less than a week ago. She has no cough and her lungs are clear on your examination. Her vital signs are as follows: Pulse 72, respirations 14, blood pressure 115/70 mm Hg, and her pulse oximetry is 94%, and her height is 64 inches. In an effort to distinguish between various pathologies, you order spirometry followed by a beta2-agonist nebulizer treatment, and then after 10 minutes a repeat spirometry. Her repeat spirometry FEV 1 improves by 225 ml which is approximately 16% and from this you tell her that you are diagnosing her with which of the following? A acute exacerbation of chronic bronchitis B asthma C chronic obstructive pulmonary disease D hyperventillation syndrome E pulmonary embolism
The Correct Answer is: B Clinicians are able to identify airflow obstruction on examination, but they have limited ability to assess it or to predict whether it is reversible. The evaluation for asthma should include spirometry (FEV 1 , FVC, FEV 1 /FVC) before and after the administration of a short-acting bronchodilator. These measurements help determine the presence and extent of airflow obstruction and whether it is immediately reversible. Airflow obstruction is indicated by a reduced FEV 1 /FVC ratio. Significant reversibility of airflow obstruction is defined by an increase of 12% and 200 mL in FEV 1 or 15% and 200 mL in FVC after inhaling a short-acting bronchodilator. A positive bronchodilator response strongly confirms the diagnosis of asthma but a lack of responsiveness in the pulmonary function laboratory does not preclude success in a clinical trial of bronchodilator therapy. Severe airflow obstruction results in significant air trapping, with an increase in residual volume and consequent reduction in FVC, resulting in a pattern that may mimic a restrictive ventilatory defect. (McPhee and Papadakis, 2011, Chapter 9)
Q 97.112: A 24-year-old female, with a history of type 2 diabetes, presents with the inability to conceive after 14 months of unprotected sexual intercourse with her husband. Her vital signs are unremarkable and you calculate a BMI of 31. Physical examination reveals acne vulgaris and hirsutism. Which of the following treatment options for her infertility would be the most effective considering your suspected diagnosis? A Medroxyprogesterone acetate B Clomiphene citrate C Metformin D Spironolactone E Mini-pill (progestin only)
The Correct Answer is: B Clomiphene citrate is highly effective as the first line treatment for infertility in PCOS. It can be accompanied with metformin, weight loss, exercise, and exogenous gonadotropins when clomiphene fails. PCOS in over half of patients is accompanied with obesity, abnormalities in insulin control, metabolic syndrome, and infertility. Medroxyprogesterone acetate (A) and the mini-pill (E) are used for endometrial protection and with oral contraceptive pills. Metformin (C) will help with her diabetes. Spironolactone (D) is a diuretic, which acts as a weak androgen receptor antagonist.
Q 39.13: At a 2-month-old well-child checkup, a female infant is noted to have the following physical findings: widely open anterior and posterior fontanels, large protruding tongue, coarse facial features, low-set hair line, and an umbilical hernia. In the newborn period, there was a prolongation of physiologic icterus. The results of the newborn screening test are abnormal. Which of the following is the MOST likely diagnosis? A congenital adrenal hyperplasia B congenital hypothyroidism C Crigler-Najjar syndrome D galactosemia
The Correct Answer is: B Congenital hypothyroidism is one of the most common disorders tested for in newborn screening tests, revealing an elevated TSH (thyroid stimulating hormone) and a decreased T 4 (thyroxine). Symptoms suggestive of congenital hypothyroidism in the neonate include hypotonia, coarse facial features, hirsute forehead, large fontanels (anterior and posterior), widely open sutures, umbilical hernia, protruding/large tongue, hoarse cry, distended abdomen, and prolonged jaundice. Signs of congenital hypothyroidism include lethargy or hypoactivity, poor feeding, constipation, mottling, and hypothermia. Congenital adrenal hyperplasia (CAH) is not universally screened for in the newborn screening test, as it is included in only 14 of the 50 states. In females with CAH, there may be virilization with abnormalities of the external genitalia varying from mild enlargement of the clitoris to complete fusion of the labioscrotal folds. Signs of adrenal insufficiency (salt loss) may present in the first few days of life. Crigler-Najjar syndrome is not one of the disorders tested for in the standard newborn screening tests. It is an inherited disease producing congenital nonobstructive, nonhemolytic, unconjugated severe hyperbilirubinemia. The physical findings in this infant do not correlate with Crigler-Najjar syndrome. Galactosemia is tested for in the newborn screening test in nearly all 50 states. The infant may have symptoms of cataract, hepatomegaly, and prolonged jaundice. Often, these neonates have Escherichia coli sepsis, leading to death in the first 2 weeks of life if not treated promptly.
A 68-year-old female presents to the emergency department due to exertional dyspnea and increased cough for the past week. The patient states that this has happened in the past, but she cannot recall which medication helped her. Physical exam reveals a sustained left ventricular impulse, an S3 gallop, and mild JVD. The chest x-ray is shown. What is the most likely diagnosis? A Acute respiratory distress syndrome B Congestive heart failure C Hypertrophic cardiomyopathy D Left pleural effusion E Right perihilar pneumonia
The Correct Answer is: B Congestive heart failure is common and increases in prevalence with age. Heart failure may be left sided or right sided, with patients often exhibiting signs of both as heart failure progresses. Patients will often present with shortness of breath, which may progress to increased dyspnea at rest and orthopnea. Physical exam findings may range from overt to subtle and may include jugular venous distention, adventitious lung sounds such as rales and rhonchi, hepatic enlargement, peripheral edema, and cardiac findings of a sustained left ventricular impulse, diminished first heart sound, S3 gallops, and an S4.
Q 31.2: A 32-year-old female professional golfer presents with a 3-week history of pain along her thumb and down her wrist. She denies any trauma and states that it is aggravated with any movement of her wrist and thumb. Her physical examination and x-rays are unremarkable. You perform the following test: This test requires her to cup her thumb in a closed fist and ulnar deviate, which reproduces her pain. What is the most likely diagnosis based on this exam finding? A Colle's fracture B De Quervain tenosynovitis C Carpal tunnel syndrome D Ganglion cyst E Gout
The Correct Answer is: B De Quervain tenosynovitis (B) commonly occurs in patients who have experienced excessive use of the thumb or wrist. Often, no plausible cause can be found. This is a tenosynovitis of the tendons that lie in the groove of the radial styloid. The patient usually presents with pain along the radial aspect of the wrist that may radiate to the thumb or extend into the forearm. The diagnosis of De Quervain tenosynovitis is supported by a history of pain in this location along with a painful range of motion of the thumb and local tenderness over the distal portion of the radial styloid. Further confirmation of the diagnosis may be provided by a positive Finkelstein test, in which the patient grasps the thumb in the palm of the hand and the examiner ulnar deviates the thumb and hand. This stretches the tendons over the radial styloid and produces sharp pain along the involved tendons. A ganglion cyst (D) and carpal tunnel syndrome (C) would not appear on physical examination in this presentation. Gout (E) would be painful and involve more of the joint. A Colle's (A) fracture would show on her x-ray.
Q 88.2: A 48-year-old male presents with an 8-day history of productive cough, subjective fevers, and malaise. He is otherwise healthy without any active medical problems. He is a social drinker of alcohol, and denies any tobacco or drug use. On physical examination, the patient is alert and oriented. His temperature is 100.4 0 F, pulse rate is 56, respiratory rate is 18, and blood pressure is 133/64. HEENT is within normal limits and a chest exam has diffuse expiratory wheeze with decreased sounds to the right lower lung fields. Blood labs reveal WBC 14.4, Hgb 11.3, Plt 233, ALT 65, AST 102, and PO 4 2.1. A chest x-ray reveals a dense consolidation with bulging fissures. Based on these findings, what is the most likely pathogen affecting this patient? A Klebsiella pneumoniae B Legionella pneumophila C Pseudomonas aeurginosa D Mycoplasma pneumoniae E Streptococcus pneumoniae
The Correct Answer is: B Dense consolidation with bulging fissures is pneumonia consistent with Legionella pneumophila. Pleural effusions may also occur, as well as nodular irregularities in the immunocompromised host. (Sabria M, Yu VL. Harrisons Online, Part 7, Infectious Disease, Section 6, Diseases Caused by Gram-Negative Bacteria, Chapter 141, Legionella Infection)
Q 97.36: A 27-year-old woman complains of years of menstrual irregularity and increasing facial and chest hair. PMH: significant for ovarian cyst and left cytectomy. She is a non-smoker and non-drinker. Labs include a negative uCG, elevated LH, and low FSH. She desires fertility and she has not responded to three cycles of clomiphene. What would be the next choice of medication that may return ovulation? A Insulin B Metformin C Dexamethasone D Spironalactone E Finasteride
The Correct Answer is: B Dexamethasone, finasteride, and spironalactone all treat symptoms of hirsutism, but do not treat the underlying cause of PCO or improve fertility outcomes. PCO has an underlying insulin resistance that can be treated with oral hypoglycemics and improve sensitivity to insulin. Adding insulin does not improve the resistance.
Q 109.16: A 56-year-old chronic alcoholic presents with signs of anemia. Laboratories reveal a moderate anemia with a hematocrit of 30% (45 to 62%) and a dimorphic population of red cells, one normal and one hypochromic. The MCV (mean corpuscular volume) is normal, the serum iron level is elevated, and the transferrin saturation is high. The diagnosis was made using which diagnostic study? A 24-hour urine for porphyrin B Bone marrow biopsy C Echocardiogram D Ferritin level E Liver biopsy
The Correct Answer is: B Diagnosis of sideroblastic anemia is made by examination of the bone marrow, using Prussian blue staining and noting the presence of ringed sideroblasts, which are cells with iron deposits encircling the red cell nucleus. None of the other studies are useful in making this diagnosis. (McPhee SJ, Papadakis MA, Tierney LM. Current Medical Diagnosis and Treatment, 2010, Chapter 13, Blood Disorders)
A 44-year-old female is involved in a motor vehicle accident, during which she suffered blunt trauma to the left chest and abdomen from the car door. She presents via rescue with marked dyspnea, tachypnea, and an oxygen saturation of 87% on room air. You obtain the chest x-ray shown. Based on the following x-ray, what is the diagnosis? A Cor pulmonale B Diaphragmatic rupture C Hemothorax D Pericarditis E Pneumothorax
The Correct Answer is: B Diaphragmatic rupture is often the result of direct injury to the diaphragm or increased intra-abdominal or intrathoracic pressure. Patient symptoms are often the result of impaired lung expansion and decreased oxygenation. Additional symptoms may include bowel obstruction or other nonspecific bowel complaints. *The chest x-ray shows elevation and irregularity of the left diaphragmatic border, with decreased left lung volume. Also of note is a widened mediastinum, which may suggest additional injury, including to the aorta. Lung markings extend through the lung space, which is not consistent with a hemothorax or pneumothorax.* Evaluation for a pericardial effusion and/or pericarditis should be performed on this patient, based on the history, with evaluation including echocardiography and EKG. The typical x-ray result for pericardial effusion is termed a "water bottle" heart. However, this would be difficult to assess in the setting of a diaphragmatic rupture. Cor pulmonale is not associated with trauma, and is due to lung disease or pulmonary vascular disease.
Q 4.7: A 43 year-old woman undergoes pulmonary function tests (PFTS) to evaluate progressive dyspnea and cough. The diffusion capacity is noted to be significant reduced. What is the most likely type of lung disease this patient is experiencing? A Congenital B Interstitial C Obstructive D Restrictive E Fixed extra-thoracic obstruction
The Correct Answer is: B Diffusion capacity is reduced in conditions that effect alveolar gas exchange such as, interstitial lung disease (B), atelectasis, pneumonia pulmonary vasculature disease, and late stage emphysema (due to destruction of alveoli, not obstructive lung disease).
A 16-year-old male was hit on the left side of his face by a line drive baseball. Marked swelling is noted externally to the left eye. There was no loss of consciousness. Upon physical exam, he complains of diplopia during extraocular motion testing. Enophthalmos is noted, as well as decreased sensation of the left cheek. Plain x-rays of the face demonstrate an air-fluid level in the left maxillary sinus, and a fracture of the orbit. Based on this information, what is the most likely diagnosis? A Zygomatic arch fracture B Orbital blowout fracture C Le Fort I fracture D Le Fort II fracture E Le Fort III fracture
The Correct Answer is: B Diplopia is common in an orbital blow out fracture, due to entrapment of the inferior rectus and inferior oblique muscles. Loss of infraorbital sensation occurs from disruption or swelling of the infraorbital nerve. A Le Fort I fracture describes a transverse fracture separating the body of the maxilla from the pterygoid plate and nasal septum. A Le Fort II fracture describes a pyramidal through the central maxilla and hard palate. Movement of the hard palate and nose occurs, but not the eyes. A Le Fort III fracture describes a craniofacial disjunction, wherein the entire face is separated from the skull due to fractures of the frontozygomatic suture line, across the orbit and through the base of the nose, and ethmoids. The entire face shifts, with the globes held in place only by the optic nerve.
Q 49.18: A 16-year-old male was hit on the left side of his face by a line drive baseball. Marked swelling is noted externally to the left eye. There was no loss of consciousness. Upon physical exam, he complains of diplopia during extraocular motion testing. Enophthalmos is noted, as well as decreased sensation of the left cheek. Plain x-rays of the face demonstrate an air-fluid level in the left maxillary sinus, and a fracture of the orbit. Based on this information, what is the most likely diagnosis? A Zygomatic arch fracture B Orbital blowout fracture C Le Fort I fracture D Le Fort II fracture E Le Fort III fracture
The Correct Answer is: B Diplopia is common in an orbital blow out fracture, due to entrapment of the inferior rectus and inferior oblique muscles. Loss of infraorbital sensation occurs from disruption or swelling of the infraorbital nerve. A Le Fort I fracture describes a transverse fracture separating the body of the maxilla from the pterygoid plate and nasal septum. A Le Fort II fracture describes a pyramidal through the central maxilla and hard palate. Movement of the hard palate and nose occurs, but not the eyes. A Le Fort III fracture describes a craniofacial disjunction, wherein the entire face is separated from the skull due to fractures of the frontozygomatic suture line, across the orbit and through the base of the nose, and ethmoids. The entire face shifts, with the globes held in place only by the optic nerve.
Q 68.2: Mr. Smith leaves home and does not return nor does he go to work. A friend of Mr. Smith sees him in another state while on vacation. When he approaches Mr. Smith, he does not recognize him and has a total different demeanor. What type of disorder does Mr. Smith have? A amnesia B dissociative fugue C schizophrenia D dissociative identity disorder E depersonalization
The Correct Answer is: B Dissociative or psychogenic fugue is precipitated by a stressful event that causes the patient to develop amnesia, leave home, and assume another identity. (Sadock and Sadock, 2008, p. 297; Eisendrath and Lichtmacher, 2009, p. 915) Eisendrath SJ , Lichtmacher JE. Psychiatric disorders. In: McPhee SJ , Papdakis MA, eds. Current Medical Diagnosis and Treatment, 48th ed. New York: McGraw-Hill; 2009. Sadock BJ , Sadock VA. Concise Textbook of Clinical Psychiatry, 3rd ed. Philadelphia, PA: Lippincott, Williams & Wilkins; 2008.
Q 80.10: A patient with HIV positive status, a CD4 count of 277/mcL, and is on chronic HIV medication therapy is diagnosed with pneumonia. What is the most common pathogen that would result in pneumonia for this patient? A Pneumocystosis jiroveci B Streptococcus pneumoniae C Pseudomonas aeurginosa D Klebsiella pneumoniae E Hemophilus influenzae
The Correct Answer is: B Due to the tight control of most of the HIV population, the most common pathogen is actually the most common community-acquired pathogen in the United States, Streptococcus pneumoniae. When the patient has lower CD4 counts and higher viral loads they are more susceptible to pathogens such as pneumocystosis jiroveci. (Chesnutt MS, Prendergast TJ. Current Medical Diagnosis and Treatment, 2011, Chapter 9, Pulmonary Disorders)
A 62-year-old man presents to the emergency department with aphasia and right lower extremity weakness that started about 4 hours ago. He now has progressing right upper extremity weakness, worsening right lower extremity weakness, and decreased sensation throughout his right side. This cerebral ischemia is best characterized as A transient ischemic attack B stroke in evolution C completed stroke D subarachnoid hemorrhage E global cerebral ischemia
The Correct Answer is: B During a stroke in evolution, symptoms will worsen or new symptoms will appear. A completed stroke is one in which neurologic symptoms have stabilized, whereas a transient ischemic attack produces deficits that resolve over time. This patient's symptoms do not match those of an acute subarachnoid hemorrhage. Global cerebral ischemia as seen in sudden cardiac arrest would involve loss of consciousness.
Q 97.6: A 62-year-old man presents to the emergency department with aphasia and right lower extremity weakness that started about 4 hours ago. He now has progressing right upper extremity weakness, worsening right lower extremity weakness, and decreased sensation throughout his right side. This cerebral ischemia is best characterized as A transient ischemic attack B stroke in evolution C completed stroke D subarachnoid hemorrhage E global cerebral ischemia
The Correct Answer is: B During a stroke in evolution, symptoms will worsen or new symptoms will appear. A completed stroke is one in which neurologic symptoms have stabilized, whereas a transient ischemic attack produces deficits that resolve over time. This patient's symptoms do not match those of an acute subarachnoid hemorrhage. Global cerebral ischemia as seen in sudden cardiac arrest would involve loss of consciousness. (Aminoff et al., 2005, pp. 286-297) Aminoff MJ , Greenberg DA , Simon RP. Clinical Neurology. 6th ed. New York, NY: McGraw-Hill; 2005.
Q 97.73: A 23-year-old complains of chronic pelvic pain. It is worse with intercourse, several days before her period, and throughout her period. She is a GOPO, LMP two weeks ago. She is married and has had one partner for the last three years. Her symptoms have been increasing over the last year. What would the most appropriate initial management of her symptoms include? A Expectant management B Combined oral contraceptives C Surgical intervention D Acetaminophen E Androgen therapy
The Correct Answer is: B Endometriosis treatment is aimed at reducing pain and preserving fertility. Surgical interventional is not first line therapy until medication has been tried. NSAIDS are used, as opposed to acetaminophen, to lower the prostaglandin levels. Combined oral contraceptives suppress ovulation, decrease menstrual flow, and decidualize implants.
The Correct Answer is: C This genetic mutation is found in patients with MEN1 syndrome, formerly known as Wermer Syndrome. About two-thirds of patients with this syndrome present with hyperparathyroidism. Gastrinomas (A) occur in about one-third and pituitary adenomas (E) in about half. Hirschsprung disease (B) and medullary thyroid carcinomas (D) are found in MEN2a syndrome. Fitzgerald PA, Endocrine Disorders, in Current Medical Diagnosis and Treatment, 52 nd ed. 2013.
The Correct Answer is: B Entamoeba histolytica has two stages in its life cycle. In the active stage in the human intestine, it causes symptoms of dysentery, abdominal pain, stool mucus, and tenesmus. In the dormant stage, the cystic form is excreted in the stool and in developing nations frequently contaminates the supply of drinking water. When the amoeba is in the dormant stage, the cystic form can be excreted in the stool and, in the case of food handlers with poor personal hygiene, be transmitted to others. In addition, because of the cystic stage, individuals engaging in anal intercourse can transmit the infection unknowingly. Diagnosis is made by microscopic evaluation of a stool wet prep and confirmed by serology. Treatment includes agents such as metronidazole or tinidazole. (Schuster and Glaser, 2007, pp. 2404-2405) Schuster FL , Glaser CA. Amebiasis. In: Goldman L , Ausiello D, eds. Cecil Medicine. 23rd ed. Philadelphia, PA: Saunders Elsevier; 2007.
Q 79.9: Which of the following viral exanthems includes a fever followed by a diffuse maculopapular rash that spares the face and resolves in about 2 days? A measles B erythema subitum C erythema infectiosum D rubella
The Correct Answer is: B Erythema subitum or roseola is caused by the human herpesvirus 6 and presents clinically as described. The fever resolves when the rash begins and the entire process is self-limiting with usual full recovery with supportive care. The key to this question is that the rash spares the face. Erythema infectiosum (fifth disease), rubella, and measles also begin with a febrile prodrome, but each of the associated rashes starts on the head or face before progressing to other parts of the body. (Kaye and Kaye, 2008, p. 122) Kaye ET , Kaye KM. Fever and rash. In: Fauci AS , Braunwald E , Kasper DL, et al., eds. Harrison's Principles of Internal Medicine. 17th ed. New York, NY: McGraw-Hill; 2008.
Q 10.1: A 67 year-old with chronic kidney disease presents to the clinic with a hemoglobin of 8.6 mg/dl and hematocrit of 24%. Which of the following agents, if administered, can further increase this patient's risk of thrombotic vascular events? A Cyanocobalamin B Erythropoietin C Ferrous sulfate D Folic acid E Niacin
The Correct Answer is: B Erythropoietin (B) increases red blood cell mass and toxicity may lead to increased blood viscosity and potential thrombotic events. Cyanocobalamin (A), ferrous sulfate (C), folic acid (D), and niacin (E) don't increase thrombotic risk.
Q 30.5: A 30-year-old female presents with a year history of chronic aching pain and stiffness throughout her entire body. She denies any trauma, new activities, or change in symptoms throughout the day. Her past medical history is significant for trouble sleeping at night and irritable bowel symptoms. On physical examination, the only finding is pain produced on palpation of the trapezius, medial knee, and lateral epicondyle of the elbow bilaterally. Her vitals are normal. All laboratory work including a thyroid panel, complete metabolic panel, erythrocyte sedimentation rate, and rheumatoid factor are within normal limits. What is the most likely diagnosis based on these findings? A Polymyositis B Fibromyalgia C Polymyalgia rheumatica D Rheumatoid arthritis E Systemic lupus erythematosus
The Correct Answer is: B Fibromyalgia (B) is a diagnosis of exclusion most frequent in women aged 20-50 years. The patient with fibromyalgia complains of chronic aching pain and stiffness, frequently involving the entire body but with prominence of pain around the neck, shoulders, low back, and hips. Fatigue, sleep disorders, subjective numbness, chronic headaches, and irritable bowel symptoms are common. Objective signs of inflammation are absent and laboratory studies are normal. Physical examination is normal except for "trigger points" of pain produced by palpation of various areas such as the trapezius, the medial fat pad of the knee, and the lateral epicondyle of the elbow. Polymyositis (A) produces weakness rather than pain. Polymyalgia rheumatica (C) produces shoulder and pelvic pain and is associated with an elevated ESR, occurring after age 50. Rheumatoid arthritis (D) and systemic lupus erythematosus (SLE), (E), present with objective physical findings or abnormalities on routine testing.
Q 69.5: Which of the following treatments is first-line therapy for sputum culture-positive Legionnaire's pneumonia in an immunocompetent patient? A ampicillin/sulbactam B erythromycin C ceftriaxone D vancomycin E clindamycin
The Correct Answer is: B First-line therapy for legionella pneumonia (mild to moderate) in the immunocompetent host is erythromycin 500 mg to 1 g IV qid or 500 mg po qid for 14 to 21 days. Another option for first-line therapy is doxycycline 200 mg IV or po once daily for 14 to 21 days. Alternatives include levofloxacin 500 mg IV or po q day for 7 to 10 days or azithromycin 500 mg IV or po q day for 3 days. Severe infection or treatment in the immunocompromised patient is levofloxacin or azithromycin. (Edelstein, 2007, pp. 2264-2265) Edelstein PH. Legionella infection
Q 97.109: A 19 year old female presents for her first pelvic examination and is noted to have a palpable, non-tender right ovary. She is sexually active, and uses condoms for birth control and protection from STDs. Her LMP was 20 days ago. She underwent transvaginal ultrasound and was found to have a 3-cm follicular cyst. What does this right ovarian mass represent? A Retained hemorrhagic products B Intrafollicular fluids C Solid intracellular material D Germ cell layers E Keratinized squamous epithelium
The Correct Answer is: B Follicular cysts occur prior to ovulation and cause expansion of the follicular antrum, serious fluid collection, and subsequent follicular cyst formation. Corpus luteum cysts form after ovulation and may accumulate blood products. Germ cell layers and keratinized squamous epithelium are solid components of teratomas.
Q 98.60: A 64-year-old man has been experiencing intermittent left lower abdominal pain associated with alternating diarrhea and constipation. The pain has been increasing over the past 24 hours and is now associated with a fever. The abdomen is tender with evidence of peritoneal signs. Which of the following diagnostic studies is most appropriate to evaluate this patient? A Barium enema B Computed tomography (CT) C Sigmoidoscopy D Colonoscopy
The Correct Answer is: B For a patient with diverticular disease, the preferred study to evaluate complications, such as a perforation or abscesses, is a CT scan. A barium enema or endoscopic procedure is contraindicated due to increased risk of perforation during an acute exacerbation.
Q 92.9: A 64-year-old man has been experiencing intermittent left lower abdominal pain associated with alternating diarrhea and constipation. The pain has been increasing over the past 24 hours and is now associated with a fever. The abdomen is tender with evidence of peritoneal signs. Which of the following diagnostic studies is most appropriate to evaluate this patient? A Barium enema B Computed tomography (CT) C Sigmoidoscopy D Colonoscopy
The Correct Answer is: B For a patient with diverticular disease, the preferred study to evaluate complications, such as a perforation or abscesses, is a CT scan. A barium enema or endoscopic procedure is contraindicated due to increased risk of perforation during an acute exacerbation. (Su, 2006, p. 208; Otterson, 2006, p. 1133) Su LT , Fry R. The colon, rectum and anus. In: Alturi P , Karakousis GC , Porrett PM , Kaiser LR, eds. The Surgical Review: An Integrated Basic and Clinical Science Study Guide. 2nd ed. Philadelphia, PA: Lippincott Williams & Wilkins; 2006. Otterson MF , Korus GB. Diverticular disease. In: Mulholland MW , Lillemoe KD , Doherty GM , Maier RV , Upchurch GR, eds. Greenfield's Surgery: Scientific Principles and Practice. 4th ed. Philadelphia, PA: Lippincott Williams & Wilkins; 2006.
Q 20.8: Your patient is a 22-year-old male who has experienced significant dysfunction for the past eight months. He exhibits only one symptom of schizophrenia, but that one symptom has been present for a significant part of each day. Which of the following would qualify the patient to be diagnosed with schizophrenia according to the DSM-IV TR? A A firm belief that someone is following him B An auditory hallucination in which two or more voices are conversing C A sensation that bugs are crawling on his skin D Refusal to go out without a hat, certain he will become ill without one E Speaking entirely in rhymes that are related but mostly nonsensical
The Correct Answer is: B Generally, two or more characteristic symptoms of schizophrenia are required in order to make a diagnosis unless the one symptom present is a bizarre delusion or an auditory hallucination (B) that consists of either a running commentary or two voices conversing. Positive symptoms of schizophrenia also include disorganized or catatonic behavior and disorganized speech as well as delusions and hallucinations. Negative symptoms include affective flattening, alogia and avolition. (A) and (D) are both potentially delusions, but neither is particularly bizarre. A sensation of bugs on the skin (C, fomication) is a type of hallucination and speaking in rhyme (E) qualifies as disorganized speech. Each of these might count as one of the two required symptoms but would not be sufficient without a second symptom.
Q 20.7: According to the ATP III revised guidelines, which of the following should be your primary focus of treatment in patients with dyslipidemia? A lowering apolipoprotein B B lowering LDL cholesterol C Lowering non-HDL cholesterol D lowering triglycerides E raising HDL cholesterol
The Correct Answer is: B Given their proven efficacy, ease of administration, and enhanced patient compliance over other classes of medications, statin agents are the drugs of first choice for most patients. In particular, patients with diabetes or those in the highest risk category derive special benefits from their use due to their innate anti-inflammatory effects. Myopathy and elevated liver enzymes are the main potential side effects from statin agents. An increase of serum aminotransferase levels to more than three times normal occurs in 1% of patients taking high doses of statins. Monitoring of liver function tests at six weeks, 12 weeks, six months, and annually thereafter can help identify patients with hepatic side effects and facilitate prompt discontinuation of the agents. Rhabdomyolysis occurs in less than 0.1% of cases. It can be prevented by the prompt discontinuation of the agent when muscle pain and elevated muscle enzymes occur. Unexplained pain in large muscle groups should prompt investigation for myopathy; however, routine monitoring of muscle enzymes is not supported by any evidence. Side effects from statins may not be class specific. Therefore, a side effect with one agent should not prevent a trial with another statin agent. Prior concerns about statins causing cataracts or cancer have been alleviated by the release of two large meta-analyses in 2001.
Q 26.2: A 65-year-old woman presents with a complaint of blood in her urine, intermittently for the last month. The patient denies any fever, chills, flank pain, or dysuria. Social history is positive for tobacco use (45 pack years), but patient reports stopping her tobacco use last year. What is the most likely cause of her hematuria? A urinary tract infection (UTI) B bladder cancer C renal calculi D pyelonephritis
The Correct Answer is: B Hematuria in women older than 60 years is consistent with a bladder malignancy. Bladder cancer causes episodic, gross hematuria that is usually painless. Cigarette smoking is a risk factor that also increases the incidence of bladder cancer. Painful hematuria associated with suprapubic discomfort or dysuria (or both) is more indicative of cystitis or calculi. Pyelonephritis is associated with chills, fever, and flank pain.
Q 47.8: A 10 year old male presents with bright red, well-demarcated petechiae and palpable purpura located on bilateral lower extremities. He also complains of abdominal pain and mild joint pain. His mother report the child had an upper respiratory infection about a week ago. Punch biopsy shows IgA immunoreactivity around post-capillary venules. What is the most likely diagnosis? A Toxic Epidermal Necrolysis B Henoch Schonlein purpura C Erythema multiforme D Fixed Drug eruption
The Correct Answer is: B Henoch Schonlein purpura (HSP) is commonly described as palpable purpura. It is precipitated by an upper respiratory infection and can also be associated with abdominal pain and joint pain.
Q 91.1: A patient presents complaining of a painful rash on his lips. What is the causative pathogen for the rash shown in the image? A Herpex zoster B Herpes simplex type 1 C Staphylococcus aureus D Herpes simplex type 2 E Staphylococcus pyogenes
The Correct Answer is: B Herpes simplex type 1 causes an orolabial and gingival vesicular rash. Herpes simplex type 2 causes genital lesions and is sexually transmitted. Herpes zoster or shingles causes a dermatomal, unilateral, and painful vesicular rash. Staphylococcus aureus and beta hemolytic streptococci cause bacterial skin infections. (McPhee et al., 2011, Chapter 6)
Q 44.12: Two and a half weeks after a camping vacation in upstate New York, a 12-year-old boy develops a fever and generalized headache. He complains that any light hurts his eyes, that he aches all over, and that he "just can't get warm." On examination he has a diffuse erythematous macular rash that spares the palms and soles, mild cervical lymphadenopathy, and slight hepatomegaly. He does not remember a tick bite, but his mother reports warning signs about ticks in the restrooms of the campground. Of the following tick-borne diseases, which is most likely? A Babesiosis B Human granulocytic ehrlichiosis C Lyme disease D Rocky Mountain Spotted Fever
The Correct Answer is: B Human granulocytic ehrlichiosis is commonly found in the upper Midwest and Northeast and has this clinical presentation. The incubation period is 5-21 days. Babesiosis and Lyme are spread by the same tick vector as HGE so must be considered. The incubation period for Babesiosis is 1-3 weeks and it is most common in coastal New England, northern California, and Washington State and around the lakes of the upper Midwest. Symptoms are similar, but usually include dark urine. Hepatosplenomegaly is rare. (A) The incubation period and distribution of Lyme disease are similar to HGE, but the rash is more typically that of erythema chronicum migrans. Early symptoms range from none to nonspecific low-grade fever, headache and myalgias. (C) Rocky Mountain spotted fever is more common along the eastern seaboard and the southeastern and south-central states. Its incubation period is 3-12 days and its rash tends to involve the palms, soles, and extremities and spread centrally. The headache is typically retroorbital. (D)
Q 76.2: You are reviewing laboratory results on a 60-year-old male from 2 days prior and note that the patient's potassium was 5.6 mEq/L but otherwise his BMP is normal. You speak to the patient on the phone; he states he feels fine. Which of the following medications would most likely be responsible for the abnormal potassium? A clonidine B enalapril C hydralazine D nebivolol E felodipine
The Correct Answer is: B Hyperkalemia is a potential adverse reaction of ACE inhibitors such as enalapril. ACE inhibitors should be suspected as a cause of hyperkalemia and may require discontinuation. Choices (A), (C),(D), and (E) do no cause hyperkalemia. (McPhee and Papadakis, 2011, Chapter 11)
Q 97.15: A 24-year-old delivers twins by cesarean section. The twins are monozygotic. The placenta was fused. What does this implies? A Single ovum and single chorion B Single ovum and double chorion C Double ovum and single chorion D Double ovum and double chorion E Triple chorion
The Correct Answer is: B Identical twins indicate single ovum. If the placenta is fused or double it means there are two chorions, and that the trophoblast differentiation occurred before day 3.
Q 12.2: A 34-year-old male presents to the primary care office with a complaint of heartburn that has been present for three months. He has symptoms two to three times a week, which occurs about 30 minutes after eating. He has tried over-the-counter antacids and they were helping to relieve his symptoms for a few months, but they are not working well now. He denies dysphagia, odynophagia, or weight loss. You decide to treat him with a proton pump inhibitor at this visit, and he achieves good symptomatic relief with this therapy. What length of therapy is appropriate in this patient? A Two to four weeks B Eight to twelve weeks C Four to six months D One year E Continue indefinitely
The Correct Answer is: B If a patient achieves good symptomatic relief with a course of an empiric, once-daily proton pump inhibitor, therapy may be discontinued after eight to twelve weeks.
Q 97.65: A 37-year-old female presents to the labor and delivery department complaining of intermittent pain and contractions. Upon arrival, she also complains of vaginal bleeding. She is a G3P2 at 39 weeks gestation; no other prenatal complications are noted. She is a non-smoker. A physical exam reveals the following: P 90, BP 130/80, T 98.7°F, abdomen gravid, positive bowel sounds, and left lower quadrant tenderness noted. A sterile speculum exam reveals the cervix to be dilated 8, fetus is cephalic, and membranes are intact. The fetal monitor reveals heart tones in the 140s with mild, decreased variability and good quality contractions noted. Delivery is felt to be imminent, and vaginal delivery has been determined to be the best course of action. What will likely decrease bleeding and shorten time to delivery? A Increased activity level B Amniotomy C Oxytocin therapy D Epidural placement E IV sedation
The Correct Answer is: B If the fetus is mature and vaginal delivery (versus c-section) has been determined to be the best course of action, then amniotomy may diminished amnionic fluid volume. This might also allow for better spiral artery compression, and serve to both decrease bleeding from the implantation site and reduce entry of thromboplastin into the maternal circulation.
Q 68.8: Which drug can potentially lead to oropharyngeal candidiasis, and which agent can be used to treat this type of infection? A albuterol; ketoconazole B triamcinolone; fluconazole C fluticasone; amantadine D cromolyn sodium; levofloxacin E flunisolide; metronidazole
The Correct Answer is: B If they coat the mouth and throat, inhaled corticosteroids (eg, triamcinolone, fluticasone, flunisolide) can alter the local bacteria and fungal population, thereby enhancing fungal growth. In cases of oropharyngeal candidiasis (thrush), white spots on the tongue and hard palate can be visualized, and the patient usually has pain on swallowing. In the asthma patient, the utilization of a spacer with a metered dose inhaler (MDI) can help minimize the chances of oropharyngeal candidiasis, as can routine gargling and rinsing following each inhaled treatment. Fluconazole is an antifungal agent that is effective in treating oropharyngeal candidiasis. (Kelly and Sorkness, 2008 pp. 486-487; Schindler et al., 2008, p. 190; Chesnutt et al., 2008, p. 209) Kelly HW , Sorkness CA. Asthma. In: DiPiro JT , Talbert RL , Yee GC, et al., eds. Pharmacotherapy: A Pathophysiologic Approach. 7th ed. New York: McGraw-Hill; 2008. Schindler J , Lustig L , Jackler RK , et al. Ear, nose & throat. In: Tierney LM Jr , McPhee SJ , Papadakis MA, eds. Current Medical Diagnosis & Treatment. 47th ed. New York: McGraw-Hill; 2008. Chesnutt MS , Murray JA , Prendergast TJ. Pulmonary disorders. In: Tierney LM Jr , McPhee SJ , Papadakis MA, eds. Current Medical Diagnosis & Treatment. 47th ed. New York: McGraw-Hill; 2008.
Q 109.19: A 28-year-old woman presents with nervousness and palpitations associated with heat intolerance. On examination, there is no evidence of thyromegaly, but there is a palpable nodule that is "hot" on a thyroid scan. The TSH was low and T3 and T4 were both elevated. Which of the following is the recommended treatment for this patient? A Propylthiouracil (PTU) B Thyroid lobectomy C Total thyroidectomy D Radioiodine ablation
The Correct Answer is: B In Graves' disease, the thyroid is diffusely enlarged in contrast to a toxic adenoma in which the thyroid is normal sized but with a palpable nodule. Surgery is the treatment of choice for a toxic adenoma. Surgical treatment of a toxic adenoma is a thyroid lobectomy and isthmusectomy. A subtotal or total thyroidectomy is indicated for toxic multinodular goiters or Plummer disease. Thionamides and radioiodine ablation are not effective therapies for toxic adenomas. (Coe, 2006, pp. 404-406) Coe NPW. Surgical endocrinology: thyroid gland. In: Lawrence PF, ed. Essentials of General Surgery. 4th ed. Philadelphia, PA: Lippincott Williams & Wilkins; 2006.
Q 97.14: A 30-year-old woman delivers a viable 7 pound 6 ounce female infant by normal spontaneous vaginal delivery. The infant was delivered and held below the introitus. The infant was dried, stimulated, and apgars were assigned. The cord was then clamped and the infant was placed on the maternal abdomen. Which of the following is a complication that can arise from this? A Hypovolemia B Hyperbilirubinemia C Hypoglycemia D Hyperglycemia E Hypoxia
The Correct Answer is: B In a normal delivery, after the infant is delivered through the introitus it should not be held below it; excessive fluids can be passed to the infant, resulting in increased hematocrit and hemoglobin, which will hemolyze and cause hyperbilirubinemia.
Q 88.8: A 56-year-old male is diagnosed with an H. pylori associated ulcer. The ulcer was 0.8 mm in size on endoscopy. He is placed on a proton pump inhibitor, clarithromycin, and amoxicillin. If his dyspeptic symptoms resolve after starting this regimen, how many days should he take the proton pump inhibitor to complete the treatment? A 5 to 7 days B 10 to 14 days C 14 to 21 days D 4 to 6 weeks E Indefinitely
The Correct Answer is: B In an H. pylori associated ulcer that is < 1cm in size and dyspeptic symptoms resolve with therapy, 10 to 14 days is the recommended length of treatment. For patients with large or complicated ulcers, antisecretory therapy should be continued for an addition two to four weeks (duodenal ulcer) or four to six weeks (gastric ulcer) after completion of the antibiotic regimen. (McPhee SJ, Papadakis MA. Current Medical Diagnosis & Treatment, 2010, p. 551)
Q 98.31: What is the most common joint dislocation in children? A Ankle B Elbow C Finger D Knee E Shoulder
The Correct Answer is: B In children the elbow is the most commonly dislocated joint and it is the third most common joint dislocation for adults. The shoulder and finger are dislocated more frequently than the elbow in adults. Knee dislocations at the femur-tibia joint are rare (patellofemoral dislocations are more common) and ankle dislocations are also relatively rare. (Sarwark, Ed., Essentials of Musculoskeletal Care, 4 th Edition, 2010)
Q 40.6: A 4-year-old is diagnosed with severe thrombocytopenia, causing spontaneous bleeding two weeks after having influenza. The child has no evidence of anemia, neutropenia, or anything else that raises a suspicion for an alternate diagnosis, and there are no atypical findings on the blood film. What is the most likely diagnosis? A Autoimmune hemolytic anemia B Immune thrombocytopenic purpura C Thalassemia D Thrombotic thrombocytopenic purpura E Von Willebrand's disease
The Correct Answer is: B In children with no other potential causes, the most likely diagnosis is immune thrombocytopenic purpura. The disease must be distinguished from acute lymphoblastic leukemia in this population prior to finalizing the diagnosis. Autoimmune hemolytic anemia would present with an anemia. Thalassemia is an inherited hematologic disorder, von Willebrand is a bleeding disorder typically with normal platelet counts, and thrombotic thrombocytopenic purpura typically has an associated anemia and a lack of bleeding.
Q 118.15: What is the most common ECG abnormality in patients with a pulmonary embolism (PE)? A Atrial fibrillation B Sinus tachycardia C Ventricular ectopy D Sinus bradycardia E High grade AV block
The Correct Answer is: B In most cases, sinus tachycardia is the only abnormality in patients with a PE. You may also find some ECGs that will have non-specific ST-T wave changes. Sinus bradycardia and AV blocks are not common findings that are associated with PE. (Chesnutt MS, Prendergast TJ. Current Medical Diagnosis and Treatment, 2011, Chapter 9, Pulmonary Disorders)
Q 99.4: A 70-year-old man, with a history of HTN and aortic valve replacement 3 months ago, presents with complaints of arthralgia, myalgia, anorexia, fatigue, and weight loss over the last month, with recent dyspnea on exertion and lower extremity edema. Vital signs are as follows: Temperature 38°C, BP 102/64, P 98, RR 20. On physical exam, a new high-pitched, blowing, decrescendo diastolic murmur is noted along the left lower sternal border. Two separate blood cultures are positive for S. aureus, and found to be methicillin-resistant. A transesophageal echocardiogram demonstrates a paravalvular abscess. Which of the following is the most appropriate therapy in the management of this patient? A IV vancomycin B IV vancomycin, IV gentamicin, and PO rifampin with surgical treatment C IV amphotericin plus flycytosine, and surgical treatment D Outpatient IV ceftriaxone E IV penicillin G
The Correct Answer is: B In patients with prosthetic valve infection with methicillin-resistant S. aureus, the treatment of choice is IV vancomycin for 6 to 8 weeks, plus IV or IM gentamicin for the initial 2 weeks secondary to nephrotoxicity, and PO rifampin for 6 to 8 weeks, with susceptibility to gentamicin determined before initiation of rifampin. Surgical therapy decreases mortality in patients with S. aureus endocarditis, from over 70% with medical therapy alone to 25%, and should be considered in patients with paravalvular abscesses and symptoms suggestive of moderate to severe refractory congestive heart failure. Therefore, choice B is the most appropriate next step in the management of this patient. Choice A does not offer sufficient coverage for methicillin-resistant S. aureus. Choice C is appropriate therapy for infective endocarditis when the causative organism is Candida. Outpatient antibiotic therapy is only appropriate in patients who are stable, without clinical or echocardiographic findings to suggest complications, and IV Ceftriaxone is not appropriate for the treatment of methicillin-resistant S. aureus prosthetic valve endocarditis. Choice E is appropriate therapy for pencillin-susceptible streptococci, such as S. bovis. (Fauci et al., 2001, pp. 813-815)
Q 97.88: A 51-year-old female presents to the office complaining of intermittent vaginal spotting for three months. She has a history of well-controlled hypertension. She is a nonsmoker. Her LMP was two years ago. Her family history is significant for colon cancer. Physical exam and pelvic exam were performed and unremarkable. What is the best diagnostic step in evaluating her vaginal bleeding? A Transvaginal ultrasound B Endometrial sampling C Pap smear D Pregnancy test E STD testing
The Correct Answer is: B In postmenopausal women with a family history of colon cancer, there is a 30% risk of endometrial cancer. Pregnancy and STD are less likely, and do not exclude endometrial cancer. Pap smear and ultrasound are useful but can be negative. Endometrial sampling is required to confirm or rule out cancer.
Q 38.18: A 4-month-old male presents for a well child check. He is healthy and the mother feels that the child is eating and growing well. On examination, there is no evidence of cyanosis. The peripheral pulses are normal and equal. There is a medium-pitched harsh pansystolic murmur that is heard best at the left sternal border at the fourth intercostal space. There is no heave or thrill present. The murmur radiates over the entire precordium and the S2 is physiologically split. What is the most likely finding on ECG? A Left ventricular hypertrophy B Normal ECG C Right axis deviation D Supraventricular tachycardia E Sick sinus syndrome
The Correct Answer is: B In this scenario the patient most likely has a small left-to-right shunt of a ventricular septal defect, given the clinical exam findings. The ECG is most frequently normal in a patient with a small ventricular septal defect. If the patient had a large left-to-right shunt left ventricular hypertrophy would be a possibility. The other choices are not commonly seen on ECG when a ventricular septal defect is present.
Q 48.4: A 1400-gram preterm infant has a symptomatic patent ductus arteriosus. What medication has the best chance of closing the patent ductus arteriosus in this patient? A Acetaminophen B Indomethacin C Propranolol D Ranitidine E Sildenafil
The Correct Answer is: B Indomethacin is an NSAID and inhibits prostaglandin synthesis, allowing for closure of the patent ductus arteriosus in preterm infants. None of the other medications listed play a role in closure of a patent ductus arteriosus.
Q 98.27: When a bone is fractured, there are the 3 stages of healing. What is the proper order in which the healing occurs? A Inflammatory, Remodeling, Reparative B Inflammatory, Reparative, Remodeling C Remodeling, Inflammatory, Reparative D Remodeling, Reparative, Inflammatory E Reparative, Inflammatory, Remodeling
The Correct Answer is: B Inflammatory changes happen after fractures and this is followed by a reparative phase and ultimately, a remodeling phase. The healing from acute injuries generally starts with inflammation,. Once the integrity of the bone has been restored on the macro level, bone remodeling continues until full healing has occurred.
Q 115.8: A 48-year-old male has a positive PPD and a negative chest x-ray. Which of the following laboratory studies must be conducted before initiating standard isoniazid therapy? A Complete blood count B Hepatic function panel C Lipid profile D PT/PTT E Thyroid-stimulating hormone
The Correct Answer is: B Isoniazid has been associated with hepatotoxicity, as well as severe and sometime fatal hepatitis. Monitoring should begin at initiation and continue throughout the course of therapy. Isoniazid is also associated with peripheral neuropathy development, especially in patients with an increased predisposition for developing a neuropathy. Alteration of renal function and visual changes have also been associated with tuberculosis therapies, indicating that renal function testing and routing eye examination should also be performed. Additional laboratory testing may be utilized for monitoring overall patient health and determining if any other conditions are also associated with the patient, such as the other choices provided. (Fauci et al., Harrison's Principles of Internal Medicine, 17e, Chapter 158)
Q 117.9: Koplik spots are a differentiating diagnostic feature of which of the following viral exanthems? A rubella B measles C varicella D parvovirus E Kawasaki disease
The Correct Answer is: B Koplik's spots, white lesions on the buccal mucosa, are characteristic of measles. The rash in measles usually presents as a red-brown rash starting with the head and moving caudally. It follows a 3- to 4-day prodrome consisting of fever, nasal drainage, conjunctivitis, and cough. Varicella may also present with mucosal lesions but they are vesicular on an erythematous base. Parvovirus, rubella, and Kawasaki disease generally do not have mucosal involvement. (Shandera and Carrales-Medina, 2009, p. 1217) Shandera WX , Corrales-Medina VF. Viral & rickettsial infections. In: McPhee SJ , Papadakis MA, eds. Current Medical Diagnosis and Treatment. 48th ed. New York, NY: McGraw-Hill; 2009.
A 66-year-old woman presents to the emergency department with a complaint of abdominal pain and distension for the past 3 days. Examination reveals a protuberant abdomen with diminished bowel sounds and tympany to percussion. Flat and upright abdominal radiographs reveal distended loops of bowel with prominent haustral markings. Which of the following etiologies is the most likely cause of the patient's condition? A Volvulus B Adenocarcinoma C Diverticular disease D Strangulated hernia E Adhesions
The Correct Answer is: B Large bowel obstructions are most commonly caused by an adenocarcinoma (65%). This is followed in decreasing incidence by diverticular scarring and volvulus. Adhesions are the most common cause of small-bowel obstruction but are rare as a cause of large bowel obstruction. The presence of haustral markings on radiographic evaluation helps differentiate between small and large bowel involvement.
Q 39.8: A mother presents with her 6-year-old child who has a rash on his arm. The rash which is a discrete, red or flesh-colored, flat-topped, thickened area that is linear in nature with papules with scale has remained unchanged despite the use of topical steroids for one month. The lesion is asymptomatic. What is the most likely diagnosis? A Atopic dermatitis B Lichen striatus C Morphea D Verruca vulgaris
The Correct Answer is: B Lichen striatus is a benign rash consisting of linearly configured, shiny, and flat lesions that occur on any skin surface. This rash occurs suddenly and resolves on its own in several weeks. The etiology is unknown.
Q 12.8: A 23 year-old vegan presents to the clinic complaining of fatigue. Initial CBC reveals a hemoglobin of 11.1 mg/dL and an MCV of 113 fL. Which of the following best describes the cause of the abnormality pictured in the patient's peripheral smear? (note photo taken from figure 57-5- in Harrison's) A Absent or non-functioning spleen B Failure of nuclear maturation C Intravascular hemolysis D Lead intoxication E Presence of uremia
The Correct Answer is: B Macrocytes are present on the peripheral smear and result from failure of nuclear maturation commonly secondary to vitamin B 12 or folate deficiency. Howell-Jolly bodies result from the lack of removal of nuclear material due to an absent or non-functioning spleen (A). Intravascular hemolysis (C) creates schizocytes, lead intoxication (D) and thalassemia cause basophilic stippling, and uremia (E) is associated with Burr cells.
Q 82.10: A 24-year-old male with thalassemia major who has received adequate transfusions, chelation therapy, and regular health checkups is in to establish care. He is 5'4" with a BMI of 17. He eats a balanced healthy diet and gets regular exercise. You know that there are complications of this disease. What is the best next step for this patient? A Increase threshold for blood transfusions B Maintain sustained reduction of body iron C Obtain a Dexa scan for osteoporosis D Obtain an ECHO to evaluate for cardiac siderosis E Obtain regular testosterone levels and treat
The Correct Answer is: B Maintaining sustained reductions in body iron has demonstrated increased overall survival rates through reductions in cardiac disease specifically due to siderosis. While these patients are at increased risk for osteoporosis and cardiac siderosis, the next best step in this patient is to maintain reduced iron levels. There is no place for increased blood transfusion or obtaining regular testosterone levels. (Lichtman et al., Williams Hematology 8e, Chapter 47, The Thalassemias: Disorders of Globin Synthesis)
Q 97.44: For the past year, a 30-year-old woman and her husband have been trying unsuccessfully to become pregnant. Over-the-counter ovulation tests have indicated that she is ovulating. Neither partner smokes cigarettes, uses any mind-altering drugs, or has a history of sexually transmitted infection. She has no history of abdominal surgery or pelvic procedures, diethylstilbestrol (DES) exposure, or major illness. On examination, she is 66" tall, weighs 135#, and appears healthy. Her thyroid is nonpalpable, and pelvic examination is unremarkable. What is the most appropriate next step in evaluation of this couple's infertility? A Endometrial biopsy B Examination of the husband C Pelvic ultrasonography D Performing a hysterosalpingogram E Referral to an infertility clinic
The Correct Answer is: B Male factors account for 25-40% of infertility. Since the woman's initial evaluation appears normal and ovulation tests are positive, the next step is to evaluate the husband before embarking on a more detailed evaluation of the wife. An endometrial biopsy (A) may be appropriate during the late luteal phase to evaluate the endometrial lining. Pelvic ultrasonography (C) may help identify ovarian cysts, endometrial implants, leiomyomas and other treatable conditions. Hysterosalpingography (D) is useful in evaluating the patency of the fallopian tubes and the size and shape of the uterine cavity. Since this woman is only 30 years of age and many other studies can be performed in ob-gyn setting, referral to an infertility clinic (E) is premature.
Q 69.10: A 67-year-old female presents for a follow-up visit for chronic obstruction pulmonary disease (COPD). Her most recent FEV1 is <80% predicted. Her room air oxygen saturation is 94%. She is currently managed on a short acting beta-agonist as needed, and has recently been on a taper-dosed corticosteroid for an exacerbation. Which of the following is the most appropriate next step of management for this patient? A Increase the dosage of the short acting beta-agonist B Add an anticholinergic, such as tiotropium C Begin oral theophylline D Begin chronic oxygen therapy E Begin daily oral corticosteroids
The Correct Answer is: B Management of COPD patients focuses on improving symptoms and decreasing the severity of exacerbations. The initial management should focus on smoking cessation in all patients that smoke. Medications may be utilized to allow bronchodilation, but must be used appropriately, to avoid side effects and potential harm. Anticholinergic agents have been shown to improve symptoms, FEV1, and reduce exacerbations, with less side effects than high dose beta-agonists. Long-acting beta-agonists have been shown to have similar benefits, with caution being needed when using these agents in certain populations. Corticosteroids, both inhaled and systemic, have been shown to have a vital role in COPD exacerbations, but benefits regarding mortality or limiting lung function decline have not been shown, with these agents not being considered a vital part of long-term COPD management. Oral theophylline, which provides bronchodilation and anti-inflammatory properties, is a fourth-line COPD agent, based upon its narrow therapeutic index and potential for adverse side effects. Oxygen therapy has been shown to improve the progression of COPD in patients with resting hypoxemia, defined by most as a resting O 2 saturation <88% or <90% with other comorbid findings. (McPhee SJ, Papadakis MA. Current Medical Diagnosis & Treatment 2011, Chapter 9, Pulmonary Disorders)
Q 46.5: A 4 year-old girl presents to the clinic due to severe allergic rhinitis and recurrent asthma exacerbations. Which of the following leukotriene pathway inhibitors is indicated for use in this child? A Beclomethasone B Montelukast C Salmeterol D Zafirlukast E Zileuton
The Correct Answer is: B Montelukast (B) is indicated for use in children 1 year of age or older, zafirlukast (D) is indicated for children > 5 years of age, and zileuton (E) is indicated for children > 12 years of age. Beclomethasone (A) is an inhaled corticosteroid and salmeterol (C) is an inhaled long-acting beta-2 agonist.
Q 120.3: A 60-year-old male complains of right scapular pain that is sharp for the last two days. He reports that the pain was preceded by tenderness and a tingling sensation. What is the most likely diagnosis? A Contact dermatitis B Herpes zoster virus C Impetigo D Molluscum contagiosum
The Correct Answer is: B More than 2/3 of cases of the herpes zoster virus occurs in patients over the age of 50. A herpes zoster virus flare occurs unilaterally in a dermatomal distribution. The prodromal stage can consist of neuritic pain or paresthesias prior to eruption of the rash in two to three weeks. The rash consists of vesicles that eventually crust and heal. The pain of herpes zoster can continue for months or years after the rash has resolved. (Wolff et al., 2009, Fig. 27-41, pg. 837-840)
Q 15.3: A 1 day-old boy develops progressing abdominal distension, bilious vomiting and failure to pass a meconium stool. Abdominal radiographs show dilated loops of small bowel. Which of the following genetic mutations should the patient be evaluated for? A ΔF508 B G551D C Q1412X D R117H E W1282X
The Correct Answer is: B Mutations in CFTR protein function resulting from genotype G551D mutations are amenable to treatment with ivacaftor (B). Approximately 5% of CF patients have the G551D mutation and all patients should be assessed for potential ivacaftor therapy. ΔF 508 (A) is the most common genotype occurring in 60-66% of all CF patients and is not amenable to ivacaftor therapy.
Q 13.3: A 16-year-old girl presents to the clinic complaining of strong desires to sleep at inappropriate times. She is very concerned because she "felt paralyzed" while falling asleep on the couch last night. Which of the following is the best diagnostic test to confirm this patient's diagnosis? A CT of the head B multiple sleep latency test C Tensilon test D thyroid stimulating hormone E polysomnography
The Correct Answer is: B Narcolepsy is characterized by hypersomnolence, loss of muscle tone prior to sleep, hallucinations upon initiating or arising from sleep, and episodes of sleep paralysis. The diagnostic test that is used in conjunction with clinical history to establish the diagnosis is the multiple sleep latency test. The Tensilon test is utilized to assess for the presence of myasthenia gravis. Polysomnography can be useful in excluding other sleep disorders, but it does not assess sleep latency time necessary to support the diagnosis of narcolepsy.
Q 19.1: Which of the following diseases is associated with the development of nasal polyps? A Amyloidosis B Allergic rhinitis C Chronic sinusitis D Chronic obstructive pulmonary disease E Wegener's granulomatosis
The Correct Answer is: B Nasal polyps are most commonly idiopathic although they may develop secondary to allergic rhinitis (B), or cystic fibrosis. Chronic sinusitis (C) may result from obstruction of the sinus drainage secondary to a polyp. Amyloidosis (A) and Wegener's granulomatosis (E) lead to the development of lesions with histology and appearance different than a benign nasal polyp.
Q 106.8: A 63-year-old male with type 2 diabetes mellitus and hyperlipidemia is being seen for routine blood work to assess his renal function. Blood pressure is 130/90 and pulse is 75. His blood chemistries show hypoalbuminemia and hypoproteinemia. His urinalysis shows urine protein excretion of 3.5 grams per 24 hours, microscopically shows oval fat bodies in the urine, and Maltese crosses under polarized light. Which of the following findings would support the suspected diagnosis? A Maculopapular rash B Peripheral edema C Jaundice D Hematuria E Costovertebral angle tenderness
The Correct Answer is: B Nephrotic syndrome is diagnosed with bland urine sedimentation, urine protein excretion > 3 g per 24 hours, hypoalbuminemia of < 3g/dl, peripheral edema, hyperlipidemia and oval fat bodies in the urine. Peripheral edema (B) is a hallmark sign of nephrotic syndrome, which occurs when the serum albumin concentration is < 3 g/dL (30 g/L). Edema is most likely due to sodium retention. Initially, this presents in the dependent areas of the body subject to gravity (lower extremities); such edema can become generalized as in periorbital edema. Patients can experience dyspnea due to pulmonary edema, pleural effusions and diaphragmatic compromise with ascites. In adults, roughly one-third of patients diagnosed with nephrotic syndrome also have a concurrent systemic disease such as diabetes mellitus, amyloidosis or systemic lupus erythematosus. Serum creatinine may or may not be abnormal at the time of presentation, depending on the severity, acuity and chronicity of the disease. Only nephrotic syndrome would show oval fat bodies in his urine. Maculopapular rash (A) is seen with scarlet fever, measles, and syphilis. Jaundice (C) is usually seen with liver abnormalities. Hematuria (D) is seen with glomerulonephritis. CVA tenderness (E) is seen with pyelonephritis Watnick S, Dirkx T. Chapter 22. Kidney Disease. In: Papadakis MA, McPhee SJ, Rabow MW, eds. CURRENT Medical Diagnosis & Treatment 2013. New York: McGraw-Hill; 2013. http://www.accessmedicine.com/content.aspx?aID=11374 . Accessed March 6, 2013.
Q 72.3: A 42-year-old African American male is admitted to the hospital with heme positive urine and anemia. He recently completed a course of trimethoprim/sulfamethoxazole for a urinary tract infection. Treatment for his condition should include which of the following? A Begin vancomycin 1 GM IV q 12 for untreated infection B No treatment C Restart bactrim IV for an undertreated UTI D Transfuse packed red cells E Transfuse platelets
The Correct Answer is: B No treatment is necessary, except to avoid known oxidant drugs. The patient is not exhibiting signs of an untreated infection, so A is wrong. The anemia is transient so no transfusions are needed. (McPhee SJ, Papadakis MA, Tierney LM. Current Medical Diagnosis and Treatment, 2010, Chapter 13, Blood Disorders)
Q 118.4: A 60-year-old male presents with complaints of irritation and a white plaque on his tongue. He denies pain. During physical exam you are unable to remove the white plaque from the mucosa with a tongue depressor. What is the most likely diagnosis, represented as follows? A Oral thrush B Leukoplakia C Geographic tongue D Glossitis E Lichen planus
The Correct Answer is: B Oral leukoplakia cannot be removed from the mucosa using a tongue depressor like oral thrush can. Lichen planus can mimic candidiasis, squamous cell carcinoma, or hyperkeratosis, and requires a biopsy to diagnose. Glossitis is a generalized inflammation, and loss of papillae of the tongue is caused by vitamin deficiencies, medication reactions, auto immune reactions, or psoriasis. Geographic tongue is an asymptomatic serpiginous area of atrophy and erythema of the anterior tongue. The condition is self-limiting. (Tintinalli et al., 2011, Chapter 117) (McPhee et al., 2011, Chapter 8)
Q 14.10: You are evaluating a 67-year-old female who has history and physical exam findings consistent with osteoarthritis. Based on your knowledge of the disease, which joints are most commonly affected? A Carpometacarpal (CMC) B Distal interphalangeal (DIP) C Metacarpophalangeal (MCP) D Proximal interphalangeal (PIP) E Radiocarpal
The Correct Answer is: B Osteoarthritis can affect all of the joints mentioned, but generally has the highest prevalence in the DIP joints, especially the second DIP joint. Osteophyte formation at the DIP joints produce enlargements referred to has Heberden's nodes. The first carpometacarpal joint is likely the second most commonly affected joint. This painful and potentially debilitating condition at the base of the thumb can make grasping activities difficult and be exacerbated by prolonged or strenuous use of the thumb. While metacarpophalangeal and proximal interphalangeal joints can definitely be damaged in osteoarthritis, they are the more classic locations for rheumatoid arthritis changes to be manifested. Radiocarpal joints at the wrist are susceptible to osteoarthritis, but not at the same high frequency as the DIP joints.
Q 106.17: A 74-year-old man with end-stage renal failure is suffering from a number of bone abnormalities, including osteomalacia. Which of the following is most likely diminished in this patient? A blood urea nitrogen (BUN) B production of 1,25-dihydroxycholecalciferol C secretion of parathyroid hormone (PTH) D secretion of thyroid hormones E serum concentration of creatinine
The Correct Answer is: B Osteomalacia represents a softening of the bone due to inadequate amounts of calcium. Hypocalcemia often develops in end-stage renal failure due to the inability of the kidneys to activate vitamin D into the form known as 1,25-dihydroxycalciferol. The kidneys possess an enzyme (1-alpha-hydroxylase) that performs this activation, but in end-stage renal failure, the activity of this enzyme declines. 1,25-Dihydroxycalciferol is essential for adequate calcium absorption in the small intestine. Without 1,25-dihydroxycholecalciferol, calcium is excreted in higher amounts in the feces. The secretion of PTH is likely to be elevated (and not diminished) in patients with end-stage renal failure, as the main stimulus for PTH secretion is hypocalcemia. Thyroid hormones do not play a role in calcium homeostasis. Both the BUN and the serum creatinine level would elevate in the patient with end-stage renal failure, as the glomerular filtration rate is diminished to the point where both urea and creatinine cannot be adequately filtered from the plasma. (Costanzo, 2006, pp. 435-437) Costanzo LS. Physiology. 3rd ed. Philadelphia, PA: Saunders Elsevier; 2006.
Q 17.3: How many types of primary osteoporosis are there? A 1 B 2 C 3 D 4 E 5
The Correct Answer is: B Osteoporosis is a condition characterized by low bone mass, which increases the fragility of bones and leads to an increased risk of fracture. Osteoporosis is defined as being either primary or secondary and primary osteoporosis is further broken down into Type 1 and Type 2. Type 1 is related to decreased hormone levels - estrogen in women and testosterone in men, and is sometimes referred to as "postmenopausal osteoporosis." It is six times more common in women and results in loss of trabecular bone. Type 1 primary osteoporosis often presents with vertebral compression fractures or fractures of the distal radius after a fall. Type 2 primary osteoporosis is sometimes referred to as "senile osteoporosis" and generally occurs in patients over 70 years of age. It is twice as common in women as men and occurs due to a diminished capacity to make new bone. The most common types of fractures found in this type of osteoarthritis are hip and pelvic fractures. Secondary osteoporosis occurs at a somewhat higher rate in men versus women and is caused by some other medical condition that produces bone loss. Common causes include long-term steroid use, various endocrine abnormalities, and neoplastic diseases such as multiple myeloma.
Q 97.2: How many types of primary osteoporosis are there? A 1 B 2 C 3 D 4 E 5
The Correct Answer is: B Osteoporosis is a condition characterized by low bone mass, which increases the fragility of bones and leads to an increased risk of fracture. Osteoporosis is defined as being either primary or secondary and primary osteoporosis is further broken down into Type 1 and Type 2. Type 1 is related to decreased hormone levels - estrogen in women and testosterone in men, and is sometimes referred to as "postmenopausal osteoporosis." It is six times more common in women and results in loss of trabecular bone. Type 1 primary osteoporosis often presents with vertebral compression fractures or fractures of the distal radius after a fall. Type 2 primary osteoporosis is sometimes referred to as "senile osteoporosis" and generally occurs in patients over 70 years of age. It is twice as common in women as men and occurs due to a diminished capacity to make new bone. The most common types of fractures found in this type of osteoarthritis are hip and pelvic fractures. Secondary osteoporosis occurs at a somewhat higher rate in men versus women and is caused by some other medical condition that produces bone loss. Common causes include long-term steroid use, various endocrine abnormalities, and neoplastic diseases such as multiple myeloma.
Q 115.15: A patient presents to your office claiming that the FBI is trying to poison him. What would these types of beliefs be called? A somatic delusion B delusion of persecution C illusion D delusion of grandeur E hallucination
The Correct Answer is: B Patients who have delusions of persecution often feel that people are taking pictures and tape recording them. Patients often believe that external agencies or relatives are attempting to harm them. (Sadock and Sadock, 2008, p. 185; Shelton, 2008, p. 294) Sadock BJ , Sadock VA. Concise Textbook of Clinical Psychiatry, 3rd ed. Philadelphia, PA: Lippincott, Williams & Wilkins; 2008. Shelton RC. Other psychotic disorders. In: Ebert MH , Loosen PT , Nurcombe B , Leckman JF, eds. Current Diagnosis and Treatment in Psychiatry. New York: McGraw-Hill; 2008.
Q 118.9: Patients who suffer with acne rosacea can relate a history of which outcome? A worsening with onset of menses B worsening with exposure to hot temperatures, spicy foods, or alcoholic beverages C improvement with onset of menopause D improvement with use of steam rooms
The Correct Answer is: B Patients who suffer with acne rosacea can relate a history of their condition worsening with exposure to hot temperatures, spicy foods, or alcoholic beverages. This is in response to increased reactivity of capillaries. Acne that worsens with the onset of menses is characteristic of acne vulgaris, not acne rosacea. Acne rosacea may resolve spontaneously; however, it is usually present in some form for a lifetime. Exposure to high temperatures, such as those in a steam room, can worsen acne rosacea. (Wolff et al., 2009, Page 9)
Q 65.8: You are evaluating a 77-year-old male with stage 3 chronic kidney disease (CKD). He denies any complaints today. His renal function has been stable over the past 6 months. However, you note that his Hgb has decreased from 10 g/dl to 8.5g /dl despite erythropoietin injections. Hemoccult stool times 3 is negative. What should you do next? A Order a blood transfusion. B Order an iron profile with ferritin. C Increase his frequency of erythropoietin to daily. D Double his dose of erythropoietin weekly. E Begin dialysis.
The Correct Answer is: B Patients with CKD may develop an anemia of chronic kidney disease, which will usually respond to subcutaneous erythropoietin q2-4weeks. However, iron is necessary for appropriate response. Patients with CKD who are iron deficient with either decreased ferritin or iron saturations will need to have their iron stores replaced by either p.o. or intravenous iron. Increasing the dose of erythropoietin or increasing the frequency is not appropriate since this will be costly and increase patient risk without benefit. Also, daily dosing or doubling the dose weekly is not recommended or indicated and may be dangerous. Dialysis is not indicated at this time. (McPhee and Papadakis, 2011, Chapter 22)
Q 107.4: A 54-year-old male patient presents to your office complaining of pain to the left eye with nausea, vomiting, and a headache after being brushed in the eye with his grandchild's stuffed animal. On examination the conjunctiva is not injected, and the cornea has a steamy appearance. You cannot visualize the retina. The pupil is fixed and 4 mm. When you stain the eye you are unable to see any lesions or scratches. You suspect: A acute bacterial conjunctivitis B acute narrow angle glaucoma C allergic conjunctivitis D herpes simplex ophthalmicus E traumatic iritis
The Correct Answer is: B Patients with acute glaucoma usually seek treatment immediately because of extreme pain and blurred vision, though there are subacute cases. The blurred vision is associated with halos around lights. Nausea and abdominal pain may occur. The eye is red, the cornea steamy, and the pupil moderately dilated and nonreactive to light. Intraocular pressure is usually over 50 mm Hg, producing a hard eye on palpation. (McPhee and Papadakis, 2011, Chapter 7)
Q 27.7: A 22-year-old non-obese female presents to the clinic with sign and symptoms of deep vein thrombosis (DVT). She denies recent trauma, prolonged immobilization, smoking, use of oral contraceptives, or changes in her health. Her last menstrual period was one week ago. She should avoid exposure to which of the following medications? A Aspirin B Combined oral contraceptives C Enoxaparin D Progesterone-based contraceptives E Warfarin
The Correct Answer is: B Patients with factor V Leiden mutation are at risk for thrombotic events when exposed to combined oral contraceptives. Aspirin (A) is not effective for the treatment of DVT and increases the risk of bleeding if co-administered with enoxaparin or warfarin. A patient may be treated with progesterone only-based contraceptives (D) as a contraception option, and may require enoxaparin (C) or warfarin (E) to manage thrombotic risk or events.
Q 97.107: A 22-year-old non-obese female presents to the clinic with sign and symptoms of deep vein thrombosis (DVT). She denies recent trauma, prolonged immobilization, smoking, use of oral contraceptives, or changes in her health. Her last menstrual period was one week ago. She should avoid exposure to which of the following medications? A Aspirin B Combined oral contraceptives C Enoxaparin D Progesterone-based contraceptives E Warfarin
The Correct Answer is: B Patients with factor V Leiden mutation are at risk for thrombotic events when exposed to combined oral contraceptives. Aspirin (A) is not effective for the treatment of DVT and increases the risk of bleeding if co-administered with enoxaparin or warfarin. A patient may be treated with progesterone only-based contraceptives (D) as a contraception option, and may require enoxaparin (C) or warfarin (E) to manage thrombotic risk or events.
Q 26.4: A 72-year-old female presents with a 4-month history of pain and stiffness in her shoulders and hips. She identifies the pain being worse in the morning and aggravated with getting in and out of the car along with difficulty brushing her hair. She also reports malaise and a 10-pound weight loss over the past few months. Her blood work shows an erythrocyte sedimentation rate (ESR) of 74 mm/h. What is the best treatment for the suspected diagnosis? A Clindamycin B Prednisone C Acetaminophen D Tramadol E Oxycodone
The Correct Answer is: B Patients with polymyalgia rheumatica (a clinical diagnosis based on pain and stiffness of the shoulder and pelvic girdle areas, frequently in association with fever, malaise, and weight loss) are treated with prednisone 10-20 mg/day orally. Usually after 2-4 weeks of treatment, slow tapering of the prednisone can be attempted. Most patients require some dose of prednisone for a minimum of approximately 1 year; 6 months is too short in most cases. Clindamycin (A) is an antibiotic that will not help polymyalgia rheumatic. Acetaminophen (C) alone has not been shown as an effective treatment. Tramadol (D) and oxycodone (E) are pain medications that will not help with the specific condition.
Q 38.3: A 4-year-old boy presents to the outpatient clinic for a well child visit. Developmental assessment is normal. Upon visual acuity testing his vision is noted to be 20/30 right eye, 20/40 left eye, and 20/30 with both eyes. Which of the following is the most appropriate management of this child's vision? A Re-assess his vision in 3 months B Re-assess his vision in one year C Re-assess his vision prior to enrolling in kindergarten D Refer to an optometrist E Refer to a pediatric ophthalmologist
The Correct Answer is: B Pediatric ophthalmology referral criteria for 3- to 5-year-old children include visual acuity of less than 20/40 in either eye or greater than a two line difference between eyes, so referral to an optometrist (D) or pediatric ophthalmologist (E), which is preferred by the American Academy of Pediatrics, is not warranted and his vision can be re-assessed at his next well child visit in one year (B).
Q 11.1: A 19-year-old female patient presents to her family practice office for her annual Pap test and her first dose of the quadravalent human papillomavirus (HPV) vaccine. As her physician assistant, when would you schedule her to come in for her second dose of the HPV vaccine? A two weeks B two months C three months D four months E six months
The Correct Answer is: B People are frequently confused by differences between the quadravalent and bivalent versions of this vaccine. But dosing schedules are not one of the confusing issues. The Centers for Disease Control and Prevention (CDC), Advisory Committee on Immunization Practices (ACIP), and the manufacturers of both the HPV4 and the HPV2 vaccines all agree: The dosing and administration schedules are the same for HPV4 and HPV2. Each dose is 0.5 mL, administered intramuscularly, preferably in a deltoid muscle. The vaccines are administered in a three-dose schedule. The second dose is administered one to two months after the first dose, and the third dose is administered six months after the first dose.
Q 97.27: A 19-year-old female patient presents to her family practice office for her annual Pap test and her first dose of the quadravalent human papillomavirus (HPV) vaccine. As her physician assistant, when would you schedule her to come in for her second dose of the HPV vaccine? A two weeks B two months C three months D four months E six months
The Correct Answer is: B People are frequently confused by differences between the quadravalent and bivalent versions of this vaccine. But dosing schedules are not one of the confusing issues. The Centers for Disease Control and Prevention (CDC), Advisory Committee on Immunization Practices (ACIP), and the manufacturers of both the HPV4 and the HPV2 vaccines all agree: The dosing and administration schedules are the same for HPV4 and HPV2. Each dose is 0.5 mL, administered intramuscularly, preferably in a deltoid muscle. The vaccines are administered in a three-dose schedule. The second dose is administered one to two months after the first dose, and the third dose is administered six months after the first dose.
Q 97.108: A 28-year-old G1P0 at 37.5 weeks gestation complains of a thin, watery discharge for the last 5 hours. She has soaked 3 pads. She has no pain and the fetus continues to be active. An external fetal monitor reveals heart tones in the 140s, with variability and no contractions. The pH of the vaginal fluids is 8. What does this indicate? A Normal vaginal secretions B Amniotic fluid C Yeast vaginitis D Bacterial vaginitis E Urine
The Correct Answer is: B Premature rupture of membranes is the rupture of membranes before the onset of labor (within 2 hours); preterm rupture is the rupture prior to 37 weeks gestation. The absence of contraction on the monitor, in addition to no complaints indicates no labor. Urinary incontinence is common, but the ph of 8 indicates amniotic fluid.
Q 89.6: A 65-year-old male presents to you with complaints of decreasing hearing, along with difficulty discerning words when in conversations in noisy environments, such as restaurants. His only medication is simvastatin for hyperlipidemia. The following is his audiogram. He has bilateral decreased high frequency hearing loss, and decreased speech recognition. What is the most likely diagnosis? A Vestibular schwannoma B Presbycusis C Presbystasis D Cerumen impactions E Vestibulobasilar insufficiency
The Correct Answer is: B Presbycusis is age related bilateral loss of high frequency hearing, and decreased word recognition. Presbystasis is age related balance disorder. Vestibular schwannoma (acoustic neuroma) causes unilateral hearing loss. Vestibulobasilar insufficiency results from atherosclerosis of the vertebral arteries, and can cause many symptoms including double vision, speech defects, vertigo, ataxia, and drop attacks. (Lalwani A.K., 2008, Chapter 53)
Q 97.42: A 23-year-old woman presents to clinic complaining of amenorrhea for 3 months. She also complains of increasing facial hair and weight gain. On exam, PMH: menarche age 13. Physical exam reveals a well-developed, slightly obese female with a BMI of 29. Her amenorrhea can likely be improved with which therapy? A Testosterone administration B Progesterone administration C Estrogen administration D Estrogen suppression E Testosterone suppression
The Correct Answer is: B Progesterone administration slows GnRH pulses, thereby improving FSH secretion and follicular maturation.
Q 68.9: A 65-year-old man presents with complaints of acute onset of pain and swelling of the right great toe. He denies recent alcohol ingestion or trauma to the area. On physical examination, the patient is afebrile, and the first metatarsophalangeal joint is erythematous, swollen, and warm to the touch. Laboratory evaluation reveals a WBC (white blood cells) count of 12,000/μL and a normal differential. Serum uric acid level is found to be 5 mg/dL. Synovial fluid analysis reveals the presence of rhomboid-shaped crystals. Which of the following is the most likely diagnosis? A acute gout B pseudogout C psoriatic arthritis D infectious arthritis E rheumatoid arthritis
The Correct Answer is: B Pseudogout presents similarly to acute gout and is best diagnosed by the finding of the rhomboid-shaped crystals of calcium pyrophosphate in joint aspirates. Joints commonly involved in pseudogout are the knees and wrists and other joints such as the metacarpophalangeals, hips, shoulders, ankles, and elbows. The diagnosis of pseudogout is further supported by the finding of a normal serum uric acid level. Acute gout would more likely be associated with an elevated serum uric acid level. Psoriatic arthritis commonly presents with asymmetrical oligoarticular involvement of two to four joints, and in a higher percentage of patients, there is known presence of the dermatological expression of psoriasis. Infectious arthritis is ruled out with the findings of an afebrile patient and WBC count of 12,000/μL. In acute infectious arthritis, the WBCs would be expected to be elevated in the range of 50,000 to 200,000/μL. Rheumatoid arthritis usually presents with symmetrical polyarticular involvement of three or more joints.
Q 120.7: A renal ultrasound would be most beneficial for diagnosing which of the following? A nephrotic syndrome B polycystic kidney disease C glomerulonephritis D acute tubular necrosis E lupus nephritis
The Correct Answer is: B Renal ultrasound is useful for assessing kidney size and thickness of the cortex, and for the presence of masses, cysts, obstruction, and hydronephrosis. Intrinsic disease is best assessed by establishing the clinical context, analyzing the urine for protein, cells, and casts, and possibly by doing a biopsy. Loss of cortical thickness is a nonspecific finding, and ultrasound does not establish an etiology. (Bazari, 2008, pp. 810-811; Watnick and Morrison, 2009, p. 805) Bazari H. Approach to the patient with renal disease. In Goldman L , Ausiello D, eds. Cecil Medicine. 23rd ed. Philadelphia, PA: Saunders; 2008. Watnick S , Morrison G. Kidney. In: Tierney LM , McPhee SJ , Papadakis MA, eds. Current Medical Diagnosis and Treatment. 48th ed. New York, NY: McGraw-Hill; 2009.
Q 21.7: A 16-year-old boy presents to the office with complaints of a rash, low-grade fever, headache, and malaise. Symptoms began yesterday after he spent most of his free time in the past 4 days deer hunting in the woods around his house. He reports that he does check himself for ticks every night. He often finds them but has not noticed any this season that were latched on to his skin. On examination, his temperature is noted to be 99.9°F, his HEENT is unremarkable, and he has 1 to 2 mm red macules over wrists and ankles with remainder of skin clear. Heart, lungs, and abdomen unremarkable. The most likely diagnosis in this patient is A Lyme disease B Rocky Mountain spotted fever C ehrlichiosis D Q fever
The Correct Answer is: B Rocky Mountain spotted fever (RMSF) is a rickettsial infection caused by Ricketsia ricketsii. The organism is transmitted to humans through the bite of the dog tick and is more common among those who spend time outdoors in a wooded area. The illness begins with generalized symptoms of fever, headache, nausea, vomiting, malaise, and myalgias. The rash of RMSF begins as a macular rash and progresses to nonblanching petechiae. The rash begins over the wrists and ankles and progresses to the arms, legs, and trunk. Untreated, it can progress to respiratory failure and/or central nervous system involvement. Serologic confirmation is not usually valid until 7 to 10 days after clinical symptoms begin so treatment is often begun empirically. Drug of choice is doxycycline 100 mg po bid until the patient is afebrile and clinically better for 2 to 3 days. Lyme disease is distinguished from RMSF by the pattern of the rash. Lyme disease is characterized by the classic erythema chronicum migrans rash, usually on the trunk. Ehrlichiosis usually does not manifest with a rash. It begins with the same general symptoms but can progress to a toxic shock syndrome. Q fever can be transmitted by ticks, but it is often acquired through contact with sheep, cattle, and goats. It has similar generalized symptoms but can progress to a cough and pneumonia. It is usually without rash. All of these illnesses respond to doxycycline.
Q 35.6: A 6-year-old female child presents with neck pain and fever for 2 days. Her remote history consists of 2 to 3 days of diarrhea and vomiting. She attends a local daycare where other kids had similar nausea/vomiting, but recovered. The LP was positive for gram-negative bacilli, decreased glucose, increased protein, and increased neutrophils. Which of the following is the most likely etiologic agent? A rotavirus B Salmonella species C Corynebacterium diphtheria D Clostridium botulinum
The Correct Answer is: B Salmonella species are gram-negative bacilli that are classified as Enterobacteriaceae, along with E Coli. While extremely uncommon as an etiology for meningitis, salmonella can cause lethal meningitis infections and must be watched. While there is typically no treatment for mild to moderate diarrhea from salmonella infections, these patients should be monitored for complete resolution. Viral meningitis typically does not have a positive Gram stain, unless there is contamination. Corynebacterium and clostridium are gram-positive bacilli.
Q 101.3: Which of the following best describes simple partial seizures? A Focal seizure activity with associated altered level of consciousness, and decreased command response with postictal phase B Seizure activity related to a focal region of the brain that may be motor, sensory, or autonomic in nature, and not accompanied by impaired consciousness C Sudden and brief loss of muscle tone with impaired level of consciousness, but without postictal state D Sudden lapse of consciousness with a brief period of autonomic changes or tonic, atonic, or clonic activity, with no postictal confusion E Sudden loss of consciousness with the onset of tonic muscle contraction, followed by clonus of musculature, and accompanied by impaired respiration and postictal state
The Correct Answer is: B Simple partial seizures are due to focal brain cortex stimulation. The seizure symptoms will be related to the specific region of the brain cortex involved, and may include motor, sensory, and autonomic changes. Partial seizures are differentiated as simple versus complex, based on the alteration or lack of alteration of consciousness, with simple not involving a change in consciousness. Seizures are also differentiated by the absence or presence of a postictal state. Simple partial seizures do not have an associated postictal state (for further seizure classification information, see Table 24-2). (McPhee SJ, Papadakis MA. Current Medical Diagnosis & Treatment 2011, Chapter 24, Nervous System Disorders)
Q 23.3: What term is used to describe the forward movement of one vertebral body on the vertebra below it, as shown in this figure? A Spina bifida B Spondylolisthesis C Spondylolysis D Sprengel's deformity E Sustentaculum tali
The Correct Answer is: B Spondylolisthesis is the forward movement of one vertebral body on the vertebra below. This most commonly occurs with L5 on S1 (more than 85% of cases) or L4 on L5. It is often due to a spondylosis (a bony defect) in the pars articularis as pictured above which is an acquired condition that may develop as a result of a stress fracture and it is not uncommon in children or adolescents. A small number of spondylolithesis cases are congenital. Spina bifida refers to a non-union of the vertebral arch at the spinous process, which is a congenital condition. Sprengel's deformity refers to a scapula that only partially descends from the neck to the thorax. The higher than normal position of the scapula results in a shortened appearance of the neck. The sustentaculum tali refers to an anatomical landmark in the foot that supports the talus and serves as an attachment for the spring ligament. It is part of the calcaneus bone.
Q 98.36: What term is used to describe the forward movement of one vertebral body on the vertebra below it, as shown in this figure? A Spina bifida B Spondylolisthesis C Spondylolysis D Sprengel's deformity E Sustentaculum tali
The Correct Answer is: B Spondylolisthesis is the forward movement of one vertebral body on the vertebra below. This most commonly occurs with L5 on S1 (more than 85% of cases) or L4 on L5. It is often due to a spondylosis (a bony defect) in the pars articularis as pictured above which is an acquired condition that may develop as a result of a stress fracture and it is not uncommon in children or adolescents. A small number of spondylolithesis cases are congenital. Spina bifida refers to a non-union of the vertebral arch at the spinous process, which is a congenital condition. Sprengel's deformity refers to a scapula that only partially descends from the neck to the thorax. The higher than normal position of the scapula results in a shortened appearance of the neck. The sustentaculum tali refers to an anatomical landmark in the foot that supports the talus and serves as an attachment for the spring ligament. It is part of the calcaneus bone.
Q 43.2: Which of the following is the initial treatment step in an adolescent who presents to the emergency department with status epilepticus? A IV glucose B stabilize airway C arterial blood gas D IV diazepam therapy
The Correct Answer is: B Status epilepticus is a medical emergency and is defined as seizure activity that lasts a minimum of 30 minutes. This results in hypoxia, acidosis, cerebral edema, and structural damage. In addition, fever, respiratory depression, hypotension, and death may occur. There are both convulsive and nonconvulsive types of status epilepticus. Because of its emergency status and potential complications, the clinician needs to initiate the ABCs (airway, breathing, circulation). Therefore, the first line of treatment is to establish and maintain an airway, oxygen is next, and then circulation, which encompasses pulse, blood pressure, and IV access. Once the IV is established, the orders should be for administering glucose-containing fluids and IV drug therapy with diazepam, lorazepam, or midazolam as well as administer phenytoin and phenobarbital. Arterial blood gases should be ordered and any abnormalities should be corrected appropriately. Finally, the clinician should determine the underlying cause: trauma, structural disorder, infection, lactic acidosis, toxins, and uremia. Maintenance drug therapy is necessary until the underlying cause is determined and rectified.
Q 114.8: Which of the following patients, without laboratory evidence of HIV, meets the Centers for Disease Control and Prevention case definition for acquired immunodeficiency syndrome (AIDS)? A 29-year-old man with pulmonary tuberculosis B 32-year-old man with Kaposi sarcoma C 35-year-old woman with invasive cervical cancer D 36-year-old man with recurrent Salmonella septicemia E 40-year-old woman with recurrent pneumonia
The Correct Answer is: B The Centers for Disease Control and Prevention AIDS case definition includes the following diseases that, with or without laboratory evidence of HIV infection, constitute a definitive diagnosis of AIDS: candidiasis of the esophagus, trachea, bronchi, or lungs; extrapulmonary cryptococcosis; cryptosporidiosis with diarrhea persisting more than 1 month; cytomegalovirus disease of an organ other than liver, spleen, or lymph nodes; herpes simplex virus infection causing a mucocutaneous ulcer that persists longer than 1 month or causing bronchitis, pneumonitis, or esophagitis; Kaposi sarcoma in a patient younger than 60; lymphoma of the brain in a patient younger than 60; disseminated Mycobacterium avium complex or Mycobacterium kansasii disease; Pneumocystis jiroveci pneumonia; progressive multifocal leukoencephalopathy; or toxoplasmosis of the brain. Other conditions in the case definition require laboratory evidence of HIV infection. (Katz and Zolopa, 2009, p. 1177) Katz MH , Zolopa AR. HIV infection. In: McPhee SJ , Papadakis MA, eds. Current Medical Diagnosis and Treatment. 48th ed. New York, NY: McGraw-Hill; 2009.
Q 2.7: A man who is sexually active with men develops an ulcer on the rim of the anus. This ulcer has a clean, erythematous base and firm, raised margins. He says it is not painful, but he noticed it while wiping himself following a bowel movement several days ago. He has no other symptoms, but does have rubbery nontender lymphadenopathy in the genital area. Of the following, which of the following is the most appropriate test to order at this stage to confirm the suspected diagnosis of syphilis? A Darkfield microscopic examination B IgG enzyme immunoassay (EIA) C Polymerase chain reaction (PCR) D Rapid Plasma Reagin (RPR) test E Venereal Disease Research Laboratory (VDRL) test
The Correct Answer is: B The EIA or CIA (chemoluminescence assay) is faster and less expensive than traditional syphilis testing, so current algorithms suggest beginning with one of these tests. The darkfield examination of fresh exudate from a lesion is usually only available in specialized laboratories as it requires expertise in obtaining and examining specimens. (A) PCR testing is increasingly used, but not widely commercially available in the U.S. (C) The RPR and VDRL tests (D, E) do not become positive until 1-3 weeks following the appearance of a chancre.
Q 83.6: A 75-year-old man is involved in a motor vehicle accident and strikes his forehead on the windshield. He complains of neck pain and severe burning in his shoulders and arms. His physical examination reveals weakness of his upper extremities. What type of spinal cord injury does this patient have? A anterior cord syndrome B central cord syndrome C Brown-Séquard syndrome D complete cord transection E cauda equina syndrome
The Correct Answer is: B The central cord syndrome involves loss of motor function that is more severe in the upper extremities than in the lower extremities, and is more severe in the hands. There is typically hyperesthesia over the shoulders and arms. Anterior cord syndrome presents with paraplegia or quadriplegia, loss of lateral spinothalamic function with preservation of posterior column function. Brown-Séquard syndrome consists of weakness and loss of posterior column function on one side of the body distal to the lesion with contralateral loss of lateral spinothalamic function one to two levels below the lesion. Complete cord transection would affect motor and sensory function distal to the lesion. Cauda equina syndrome typically presents as low back pain with radiculopathy. (Hauser and Ropper, 2008, p. 2580) Beal MF , Hauser SL. Trigeminal neuralgia, Bell's palsy and other cranial nerve disorders. In: Fauci AS , Braunwald E , Kasper DL, et al., eds. Harrison's Textbook of Medicine. 17th ed. New York, NY: McGraw-Hill; 2008.
Q 121.20: A cerebrospinal fluid analysis reveals the following results: opalescent color, increased protein, decreased glucose, and increased polymorphonuclear white blood cells (WBCs). The most likely diagnosis would be A subarachnoid hemorrhage B bacterial meningitis C viral meningitis D multiple sclerosis E encephalitis
The Correct Answer is: B The cerebrospinal fluid (CSF) analysis in bacterial meningitis includes a cloudy appearance with a markedly elevated protein and white cell content. The white cells are predominantly polymorphonuclear leukocytes (polys). Bacterial utilization of CSF glucose causes it to be low. Gram stain may or may not be positive for bacteria. The diagnosis of bacterial meningitis requires a culture of the CSF. CSF pressures at the time of the lumbar puncture are elevated in 90% of cases. In viral meningitis, the CSF white count is usually 1,000/mL. The cell types are lymphocytes or monocytes but early in the disease polys may predominate. CSF glucose is normal in viral meningitis and protein is elevated. Gram stain will be negative and the culture will show no growth. The CSF in multiple sclerosis may have a mild lymphocytosis with an increased protein concentration. CSF protein electrophoresis in multiple sclerosis shows discrete bands of IgG called oligoclonal bands. These oligoclonal bands are present in 90% of patients with multiple sclerosis. The CSF in subarachnoid hemorrhage is grossly bloody. Because bleeding can be caused by a traumatic puncture, the red blood cell (RBC) count should be done on the first and last tubes and the counts compared. In subarachnoid hemorrhage, the RBC count will be the same, whereas in a traumatic lumbar puncture, the RBCs will not be present in the last tube that is collected. The CSF in subarachnoid hemorrhage may reveal xanthochromia. This is a yellow appearance in the centrifuged CSF supernatant caused by the degradation of RBCs in the CSF. The CSF becomes xanthochromic after it has been exposed to blood for several hours. (Aminoff et al., 2005, p. 12) Aminoff MJ , Greenberg DA , Simon RP. Clinical Neurology. 6th ed. New York, NY: McGraw-Hill; 2005.
A 23-year-old patient presents to the Emergency Department complaining about a bloody left eye that occurred after being struck with a tennis ball. The patient denies any decreased vision or photophobia. A photo of the left eye reveals the following (see below). Which of the following is the most appropriate management for this patient? A CT scan of orbit B Observation and reassurance C Olopatadine 0.1% ophthalmic solution twice daily D Refer to ophthalmology E Sulfacetamide 10% ophthalmic solution three times a day for 5 days
The Correct Answer is: B The classic appearance of a bright red, flat collection of blood is consistent with a subconjunctival hemorrhage best managed with observation and reassurance (B) by the primary care provider. The mechanism of injury is inconsistent with orbital fracture (A), and the patient lacks infectious (E) or allergic (C) conjunctivitis.
Q 16.5: A 23-year-old patient presents to the Emergency Department complaining about a bloody left eye that occurred after being struck with a tennis ball. The patient denies any decreased vision or photophobia. A photo of the left eye reveals the following (see below). Which of the following is the most appropriate management for this patient? A CT scan of orbit B Observation and reassurance C Olopatadine 0.1% ophthalmic solution twice daily D Refer to ophthalmology E Sulfacetamide 10% ophthalmic solution three times a day for 5 days
The Correct Answer is: B The classic appearance of a bright red, flat collection of blood is consistent with a subconjunctival hemorrhage best managed with observation and reassurance (B) by the primary care provider. The mechanism of injury is inconsistent with orbital fracture (A), and the patient lacks infectious (E) or allergic (C) conjunctivitis.
Q 6.8: A 23-year-old male with a recent diagnosis of Non-Hodgkin's lymphoma presents complaining of swelling of the neck and face, cough, and dyspnea on exertion. On exam you note dilated neck veins. Which of the following is the most appropriate initial treatment for this patient? A Beta-blockers B Glucocorticoids C IV fluids D Trendelenburg position E Vena cava stenting
The Correct Answer is: B The classic presentation signs and symptoms of superior vena cava syndrome (SVCS) are present in this patient. Patients with lymphoma often respond with a decrease in tumor mass and improvement in SVCS with glucocorticoid therapy. Beta-blockers (A) and vena cava stenting (E) aren't routinely used in the management of SVCS. Other initial symptomatic therapies include low salt diet, diuretics, head elevation, and diuretics. IV fluids (C) and Trendelenburg position (D) are contraindicated.
Q 28.10: Which bone is the most susceptible and most often fractured at birth? A Calcaneus B Clavicle C Femur D Humerus E Patella
The Correct Answer is: B The clavicle is the most common bone broken during childbirth. It often is associated with shoulder dystocia, but clavicular fractures can occur in uncomplicated pregnancies. They are usually of the greenstick variety and heal without complications. Calcaneal and patellar fractures are highly unlikely to occur since they are not long bones which are much more vulnerable to fracture. Fractures of the humerus and femur are possible during childbirth, but generally only in traumatic births. Humerus and femur fractures are much less common than clavicular fractures.
Q 97.104: Which bone is the most susceptible and most often fractured at birth? A Calcaneus B Clavicle C Femur D Humerus E Patella
The Correct Answer is: B The clavicle is the most common bone broken during childbirth. It often is associated with shoulder dystocia, but clavicular fractures can occur in uncomplicated pregnancies. They are usually of the greenstick variety and heal without complications. Calcaneal and patellar fractures are highly unlikely to occur since they are not long bones which are much more vulnerable to fracture. Fractures of the humerus and femur are possible during childbirth, but generally only in traumatic births. Humerus and femur fractures are much less common than clavicular fractures.
Q 35.8: An 8-year-old boy began vomiting earlier in the day and complained of "feeling terrible." His mother says he has slept most of the day and is hot to the touch, although she has not taken his temperature. He is also complaining of a sore throat and that his neck is sore. On examination, his temperature is 103F and he appears unwell. His pharynx is beefy red, the tonsils are enlarged, and covered with a thick exudates. His anterior cervical nodes are tender and markedly enlarged. What additional physical finding is most consistent with this presentation? A bloody diarrhea B circumoral pallor C diffuse rales D petechiae on the distal extremities E vesicular skin lesions with a honey-colored crust
The Correct Answer is: B The clinical picture is that of a Group A Streptococcal infection which, in this age group, presents with fever, malaise, repeated vomiting, a sore throat with tonsillar exudates, and tender, enlarged anterior cervical nodes. Additional physical findings include petechial lesions on the soft palate and mucosal surfaces, circumoral pallor, and a coated tongue. Diarrhea is not common in streptococcal infections. (A) Localized or diffuse rales are characteristic of a pneumococcal pneumonia in this age group (C) and petechial rashes on the extremities and trunk of meningococcal disease. (D) Vesicular lesions that rupture and become covered with a honey-colored crust are typical of streptococcal impetigo. (E)
Q 98.19: Which of the following signs and symptoms is associated with the abdominal pain secondary to chronic intestinal ischemia? A Guarding and rigidity B Fear of eating C Nausea and vomiting D Bloody diarrhea E Positive obturator and psoas signs
The Correct Answer is: B The clinical symptoms associated with chronic intestinal ischemia include severe epigastric pain following meals, which results in weight loss and fear of eating. Nausea, bloody diarrhea, and vomiting as well as guarding and rigidity are consistent with acute intestinal ischemia. Obturator and psoas signs are indicative of acute appendicitis.
Q 66.5: Which of the following signs and symptoms is associated with the abdominal pain secondary to chronic intestinal ischemia? A Guarding and rigidity B Fear of eating C Nausea and vomiting D Bloody diarrhea E Positive obturator and psoas signs
The Correct Answer is: B The clinical symptoms associated with chronic intestinal ischemia include severe epigastric pain following meals, which results in weight loss and fear of eating. Nausea, bloody diarrhea, and vomiting as well as guarding and rigidity are consistent with acute intestinal ischemia. Obturator and psoas signs are indicative of acute appendicitis. (McKinsey, 2006, p. 457; Shelton, 2006, p. 678) McKinsey JF , Lawrence PF , Gewertz BL. Diseases of the vascular system. In: Lawrence PF , Bell RM , Dayton MT , Ahmed MI, eds. Essentials of General Surgery. 4th ed. Philadelphia, PA: Lippincott Williams & Wilkins; 2006. Shelton AA , Chang G , Welton ML. Small intestine. In: Doherty GM , Way LM, eds. Current Surgical Diagnosis & Treatment. 12th ed. New York, NY: Lange Medical Books/McGraw-Hill; 2006.
Q 69.7: A 45-year-old male presents with purulent discharge from his right ear for three weeks. He states that despite being treated by his family doctor for an ear infection one month ago, the problem continues to get worse. Upon exam, you note purulent discharge in the ear canal, an erythemic tympanic membrane, and a possible perforation. What are the pathogens most likely to culture positive? A Strep pneumoniae B Pseudomonas aeroginosa C Escherichia coli D Candida albicans E Mycoplasma pheumoniae
The Correct Answer is: B The clinical vignette describes a chronic otitis media. Usually, this refers to a complication of acute otits media with perforation. Pathogens that culture from these infections are usually pseudomonas, proteus, or staphylococcus aureus. Strep pneumoniae is often seen in acute otitis media. E.coli is a urinary tract pathogen. Candida albicans is a cause of vaginitis, and mycoplasma is a respiratory pathogen. (McPhee et al., 2011, Chapter 8)
Q 37.6: A 3-year-old boy presents complaining of left ear pain since early this morning. The mother states he has had cold symptoms for 3 days and awoke crying, with left ear pain, and a temperature of 102.6˚ F. Which of the following physical exam findings most accurately establishes the anticipated diagnosis? A Fever greater than 101.5˚ F B Immobile or hypo-mobile tympanic membrane C Post-auricular adenopathy D Tenderness with palpation of the external ear E Tympanic membrane erythema
The Correct Answer is: B The diagnosis of acute otitis media is best established through the use of pneumatic otoscopy demonstrating decreased tympanic membrane mobility (B). Fever (A), pain with palpation of the auricle (D), and tympanic membrane erythema (E) are all non-specific findings.
Q 47.4: Which of the following is the causative agent in patients with Fifth disease? A herpesvirus 6 (HHV-6) B human parvovirus B19 C paramyxovirus D varicella-zoster virus (VZV) E none of the above
The Correct Answer is: B The disease is caused by parvovirus B19. It appears sporadically, but often in epidemics in communities. Children are infectious during the prodromal stage, which is unapparent or mild and usually indistinguishable from an upper respiratory infection. The rash is an immune-mediated phenomenon that occurs after the infection, so children with the rash are not infectious and should not be restricted from school or other activities.
Q 7.10: A disk herniation that is putting pressure on the L5 nerve root may present with weakness of what muscle(s)? A Anterior tibialis B Extensor hallucis longus C Gastrocnemius-soleus D Iliopsoas E Peroneus longus and brevis
The Correct Answer is: B The extensor hallucis longus muscle's motor function is associated the L5 motor neuron, which also supplies the gluteus medius and extensor digitorum longus and brevis muscles. The anterior tibialis muscle is supplied by the L4 motor neuron. Nerves emanating from T12, L1, L2 and L3 supply the iliopsoas. Gastrocnemius, soleus and peroneus longus and brevis are all supplied by nerves coming from the S1 area. The plantar flexing gastrocnemius and soleus muscles also are supplied by S2.
Q 102.3: A 67-year-old male presents for his annual physical in February. On examination, you note nontender cervical and axillary adenopathy. He states that he has felt those lumps for several weeks, has noted feeling more fatigued, and has decided that he had a cold. On further inspection, you note a mildly enlarged spleen, and the patient states that he hasn't been eating as much as he usually does. A CBC reveals an elevated lymphocyte count. These findings are most consistent with which diagnosis? A Acute infection lymphocytosis B Chronic lymphocytic leukemia C Rickettsiosis D Stress lymphocytosis E Systemic lupus erythematosus
The Correct Answer is: B The findings in this patient are consistent with chronic lymphocytic leukemia. Rickettsiosis and systemic lupus erythematosus tend to cause a lymphocytopenia rather than lymphocytosis. Acute infection lymphocytosis do not have either lymph node enlargement or splenomegaly, and stress lymphocytosis is short lived following trauma, surgery, or other insults to the body. (Lichtman et al., Williams Hematology 8e, Chapter 94, Chronic Lymphocytic Leukemia and Related Diseases) (Lichtman et al., Williams Hematology 8e, Chapter 81, Lymphocytosis and Lymphocytopenia)
Q 21.4: A 36 year-old smoker with a past medical history of asthma presents to the clinic for follow-up care. What is the first step that should be performed to promote smoking cessation? A Assess how long after awakening until their first cigarette B Ask about their smoking and obtain permission to discuss the behavior C Determine their prior quit attempts and results D Prescribe varencline E Share information about the harms of smoking
The Correct Answer is: B The first step in promoting positive behavioral change is to ask if the patient if they smoke and obtain permission to discuss the behavior (B). Assessing the time from awakening to first cigarette (A) is helpful in determining the severity of their nicotine dependence and understanding prior attempts and results (C) forms a basis for assisting smokers interested in quitting. Sharing the harms of smoking (E) should be re-framed positively for patients through advisement on the benefits of quitting. Varnecline (D) is one option available to assist smokers in their attempt to quit.
Q 7.2: A 5-year-old male is being evaluated for an acute injury to the right ankle. On the x-ray of the ankle there is a distal tibia fracture that involves the separation of the epiphysis, as well as a small non-displaced chip fracture of the metaphysis of the tibia. Based on these findings, what type of Salter-Harris fracture does this child have? A I B II C III D IV E V
The Correct Answer is: B The growth plate is the most fragile part of the bone prior to bone maturation and thus is usually the first structure disrupted when force is applied. Statistically, Type II fractures are most common - those that involve both the growth plate and a chip fracture of the metaphysis.
Q 15.6: Upon testing a patient for function of the hip flexors, which muscle is considered the primary muscle responsible for most flexion? A Gracilis B Iliopsoas C Rectus femoris D Sartorius E Vastus intermedius
The Correct Answer is: B The iliopsoas muscle is the primary hip flexor muscle. It originates at T12 and L1-5 vertebrae and intervertebral disks as well as the iliac fossa of the pelvis and connects to the femur at the lesser trochanter. The gracilis muscle is considered a secondary hip adductor. Rectus femoris does help with hip flexion, but in a secondary role to the iliopsoas. Rectus femoris is also involved in knee extension. Sartorius is also involved in hip flexion, but in a secondary role. The vastus intermedius muscle is one of the four quadriceps muscles and is involved with knee extension and is not involved in hip flexion.
Q 98.34: A 58-year-old man presents with to the emergency department with the acute onset of the "worst headache of his life," which was associated with a brief loss of consciousness. The patient is complaining of increased pain with movement of his neck as well as photophobia. What is the recommended first-line study to evaluate this patient? A Lumbar puncture B Noncontrast CT scan of the head C Magnetic resonance imaging (MRI) of the brain D Four-vessel angiogram
The Correct Answer is: B The initial study to diagnose a subarachnoid hemorrhage is a noncontrast CT scan of the head. If the CT scan is nondiagnostic for a SAH and the clinical suspicion is high, then proceed with a lumbar puncture for the presence of red blood cells in the cerebrospinal fluid; xanthochromia can be seen with an old SAH.
Q 14.5: Which peripheral nerve is involved in the most common compression neuropathy in the upper extremity? A Axillary B Median C Radial D Sciatic nerve E Ulnar
The Correct Answer is: B The median nerve is commonly compressed as it passes through the carpal tunnel in the wrist. This syndrome is most often diagnosed in middle aged or pregnant female patients. The axillary nerve passes through the axilla and is often compressed when patients use crutches improperly and bear weight on the axillary area. The ulnar nerve is second only to the median nerve and can be compressed as it passes through the cubital tunnel at the elbow or as it passes through the humeral and ulnar heads of the flexor carpi radialis muscle. Compression of the radial nerve (and its branches) as it passes through the radial tunnel on the lateral side of the elbow is often confused with lateral epicondylitis. The sciatic nerve is associated with the lower extremities and pain is often elicited as a result of a lumbar disk herniation causing nerve root impingement.
Q 98.16: Which peripheral nerve is involved in the most common compression neuropathy in the upper extremity? A Axillary B Median C Radial D Sciatic nerve E Ulnar
The Correct Answer is: B The median nerve is commonly compressed as it passes through the carpal tunnel in the wrist. This syndrome is most often diagnosed in middle aged or pregnant female patients. The axillary nerve passes through the axilla and is often compressed when patients use crutches improperly and bear weight on the axillary area. The ulnar nerve is second only to the median nerve and can be compressed as it passes through the cubital tunnel at the elbow or as it passes through the humeral and ulnar heads of the flexor carpi radialis muscle. Compression of the radial nerve (and its branches) as it passes through the radial tunnel on the lateral side of the elbow is often confused with lateral epicondylitis. The sciatic nerve is associated with the lower extremities and pain is often elicited as a result of a lumbar disk herniation causing nerve root impingement.
Which peripheral nerve is involved in the most common compression neuropathy in the upper extremity? A Axillary B Median C Radial D Sciatic nerve E Ulnar
The Correct Answer is: B The median nerve is commonly compressed as it passes through the carpal tunnel in the wrist. This syndrome is most often diagnosed in middle aged or pregnant female patients. The axillary nerve passes through the axilla and is often compressed when patients use crutches improperly and bear weight on the axillary area. The ulnar nerve is second only to the median nerve and can be compressed as it passes through the cubital tunnel at the elbow or as it passes through the humeral and ulnar heads of the flexor carpi radialis muscle. Compression of the radial nerve (and its branches) as it passes through the radial tunnel on the lateral side of the elbow is often confused with lateral epicondylitis. The sciatic nerve is associated with the lower extremities and pain is often elicited as a result of a lumbar disk herniation causing nerve root impingement.
Q 37.13: The most common cause of nephrotic syndrome in children is A post-streptococcal glomerulonephritis B minimal change disease C diabetes mellitus D NSAIDs E polycystic kidney disease
The Correct Answer is: B The most common cause of nephrotic syndrome in children is minimal change disease. Diffuse injury to the capillaries is the underlying cause, resulting in significant proteinuria, edema, hypoalbuminemia, and hyperlipidemia. It accounts for 65% of cases of nephrotic syndrome in children; however, 10% of adults with nephrotic syndrome have minimal change disease. Treatment is with corticosteroids for 2 to 4 weeks, dietary sodium restriction, and sometimes diuretics to reduce the edema. Relapse and lack of response to corticosteroids can occur. If the latter occurs, renal biopsy is indicated to rule out other causes of the nephrotic syndrome, such as focal glomerulosclerosis and membranoproliferative glomerulonephritis.
Q 39.28: Which of the following is the recommended treatment for a 4-year-old child with presumed bacterial meningitis? A cefotaxime or ceftriaxone plus ampicillin B cefotaxime or ceftriaxone plus vancomycin C gentamicin plus ampicillin D ampicillin plus chloramphenicol
The Correct Answer is: B The most common etiologic organisms for bacterial meningitis in children are S pneumoniae, N Meningitidis, and H influenzae. Because of an increase in resistant S pneumoniae, coverage with vancomycin and a third-generation cephalosporin such as cefotaxime or ceftriaxone is needed for best coverage. Gentamicin can be used but, as with all aminoglycosides, caution is needed regarding toxicity. Ampicillin, rifampin, and chloramphenicol are alternative treatments if necessary. (
Q 97.100: A 22-year-old female present complaining of a lump in her left breast. She noticed it two days ago while taking a shower. She is a non-smoker and has three to four drinks per week. PMH is negative and FMH is negative. On physical exam, vitals are normal, and a breast exam reveals a 1-cm discrete, soft, and rubbery lesion in the upper outer quadrant—it is non-tender, and the remainder of the breast exam is normal. What is the most likely diagnosis? A Abscess B Fibroadenoma C Phylloides tumor D Cyst E Carcinoma
The Correct Answer is: B The most common mass in premenopausal women by far is a fibroadenoma. While any lesion needs to be followed, soft, mobile, non-tender, and small lesions in young women without family history are very characteristic of fibroadenoma.
Q 49.3: The mother of a four-month-old brings her son in for evaluation of cyanosis. The mother noted the cyanosis in the last two days, and it is most evident when he is feeding or crying. He was previously healthy, with no medical problems. On physical examination, a grade III/VI systolic ejection murmur is present at the left sternal border in the third intercostal space, and radiates to the back. Which of the following diagnostic studies will best help you in establishing the diagnosis? A Electrocardiography B Echocardiography C Angiocardiography D Chest x-ray E Serial cardiac enzymes
The Correct Answer is: B The most likely diagnosis, given the history and physical examination, is tetralogy of fallot. Echocardiography usually establishes the diagnosis by visualizing the large ventricular septal defect, the right ventricl infundibular stenosis, and the enlarged aorta. Electrocardiography may reveal right ventricular hypertrophy, but cannot establish the diagnosis of tetralogy of fallot. Angiocardiography will show the anomalies in the coronary arteries, but this alone cannot be used to establish the diagnosis. Chest x-ray may reveal a boot-shaped heart, but alone cannot be used to establish the diagnosis. Serial cardiac enzymes are measured when there is a concern for a myocardial infarction, but are not used in diagnosing tetralogy of fallot.
Q 79.1: Which of the following is a complication of Barrett esophagus? A achalasia B adenocarcinoma C diffuse spasm D varices E stricture
The Correct Answer is: B The most serious complication of Barrett esophagus is esophageal adenocarcinoma, which arises from dysplastic epithelium. Patients with Barrett esophagus have a significantly increased risk compared to those patients who do not. (Poneros, 2009, pp. 148-150) Poneros JM. Barrett esophagus. In: Greenberger NJ, ed. Current Diagnosis & Treatment: Gastroenterology, Hepatology, & Endoscopy. New York, NY: McGraw-Hill; 2009.
Q 99.9: A 63 year-old woman presents to the clinic complaining of increasing dyspnea over the last two weeks. Which of the following diagnoses is supported by the chest x-ray below? A Dissecting aortic aneurysm B Pleural Effusion C Pneumonia D Pneumothorax E Pulmonary Embolism
The Correct Answer is: B The patient has a left-sided pleural effusion (B). The mediastinum is not widened as seen in dissection aortic aneurysms (A) and although the lung markings are absent in the left lower lobe region this is due to the accumulation of fluid (B) and not air as seen in pneumothorax (D). Clear signs of pneumonia (D) or pulmonary embolism (E) aren't present on the chest x-ray, but are potential underlying causes of effusion.
Q 64.4: A 64 year-old woman with past medical history of hypertension presents to the clinic complaining of increasing cough and dyspnea over the past two weeks. Physical exam reveals dullness to percussion and decreased breath sounds at the bilateral bases. A chest x-ray is available below: Which of the following is the most appropriate intervention for this patient? A Cardiac catheterization B Diuresis with a loop diuretic C Obtain a spiral CT of the chest D Perform thoracentesis E Treat with empiric antibiotics
The Correct Answer is: B The patient has developed a pleural effusion most likely due to CHF based on the symptoms and presence of small bilateral pleural effusions. Initial interventions include diuresis and monitoring for resolution of symptoms and the effusion (B). If the effusion fails to improve or the patient develops dyspnea at rest then thoracentesis (E) is indicated. Additional therapy or intervention (A, C, E) should be considered if the patient's condition worsens and are informed by the results of thoracentesis.
Q 120.11: Your patient complains of some weight loss and anxiety over the past 6 months, which she attributes to family issues. On examination you note the following physical examination findings, shown below. What is the most likely diagnosis? Source: (Wolff et al., 2008, Chapter 152) A hypothyroidism B Grave's disease C hypoparathyroidism D subacute thyroiditis E hyperparathyroidism
The Correct Answer is: B The patient has exophthalmos, which is most commonly associated with Graves' disease. Subacute bacterial thyroiditis, choice (D), usually has an initial hyperthyroid phase followed by hypothyroidism and would unlikely cause these physical examination changes. None of the other listed conditions would cause these physical examination findings. (Wolff et al., 2008, Chapter 152)
Q 23.5: A 26-year-old female patient presents to the emergency department complaining of left eye discharge, without significant pain. She has no known drug allergies. Physical exam reveals the following: A gram stain reveals gram-negative intracellular diplococci. Which of the following is the most appropriate clinical intervention? A Ceftriaxone 1 gram IM and ophthalmologic follow-up in 48 hours B Ceftriaxone 1 gram IM and emergent referral to ophthalmologist C Ciprofloxacin 500 mg twice daily for 10 days and ophthalmogic follow-up in 48 hours D Ciprofloxacin 500 mg twice daily for 10 days and emergent referral to ophthalmologist E Emergent referral to ophthalmologist
The Correct Answer is: B The patient has gonococcal conjunctivitis that is an ophthalmologic emergency due to potential risk of corneal perforation (C). Aggressive antibiotic therapy for gonorrhea (i.e., ceftriaxone) with immediate ophthalmology evaluation is the best course of action.
Q 9.7: A 29-year-old woman presents to the clinic with a complaint of severe diarrhea occurring over the last 3 to 4 days. Upon examination, the patient displays poor skin turgor and has a temperature of 100.2°F. In the supine position, the patient's blood pressure is 88/64 mm Hg and her heart rate is 112 beats/min. Upon standing, her heart rate further increases to 126 beats/min. Which of the following accounts for the further increase in the patient's heart rate upon standing? A decreased systemic vascular resistance B decreased venous return C increased preload D increased myocardial contractility E increased peripheral vasodilation
The Correct Answer is: B The patient is displaying signs of hypovolemia likely because of her chronic diarrhea. Upon standing, most of her low blood volume pools in the veins of her lower extremities because of the effects of gravity. As a result, even less blood returns to the heart, which leads to a decrease in both stroke volume and cardiac output as well as orthostatic hypotension. This elicits the baroreceptor reflex, which attempts to increase and maintain arterial blood pressure by raising the heart rate.
Q 74.5: A 24-year-old female presents with hyperpigmented macules on her cheeks, nose, and upper lip. They have been present for a couple of months. Her current medications include oral LoEstrin 24 Fe, cetirizine, and a multivitamin daily. What is the most likely diagnosis? A congenital nevus B melasma C post-inflammatory hyperpigmentation D café-au-lait macule
The Correct Answer is: B The patient is experiencing melasma secondary to the use of oral contraceptives. This is a frequent cause of melasma. Melasma can also be precipitated by hormonal changes that occur during pregnancy. The condition will resolve upon discontinuation of the oral contraceptive. A congenital nevus is a nevus that presents within the first year of life. It is monitored in the same way as acquired nevi. They can be larger than acquired nevi, with only a slight increase in chance of malignant change over time. Post-inflammatory hyperpigmentation includes darker areas of pigmentation that can result after inflammation on the skin. Common causes include acne and atopic dermatitis. The hyperpigmentation will resolve over time. A Café-au-lait macule is a type of birthmark. It is usually light tan to light brown in appearance, and can vary greatly in size. They are usually benign, but can be associated with neurofibromatosis when more than six, with a diameter greater than 1.5 cm, are present. (Wolff et al., 2009, Pages 344-346)
Q 39.19: A 2-year-old boy presents to the clinic for his routine well child visit. On physical exam of his mouth and dentition you observe the following. Which of the following is the most likely diagnosis? A Dental abcess B Dental caries C Dental contusion D Periodontitis E Stomatitis
The Correct Answer is: B The patient is noted to have whitish discoloration at the gingiva margin consistent with the early demineralization of a dental carie (B).
Q 40.4: A 3 year-old boy is brought to the emergency department due to acute onset of cough and wheezing. Physical exam reveals focal wheezing in the right lower lobe. Which of the following should be ordered to confirm the suspected diagnosis? A Arterial blood gas B Inspiratory and forced expiratory chest x-rays C PA and lateral chest x-ray D Peak expiratory flow rate E Spirometry
The Correct Answer is: B The patient most likely has aspirated a foreign body. This is best evaluated through the demonstration of inspiratory localized hyperinflation and expiratory mediastinal shift (B) on chest x-ray. ABG (A) results will vary depending on the severity of airway obstruction. PA and lateral chest x-rays (C) are typically normal. PEFR (D) and Spirometry (E) are not typically able to accurately assess this localized airway obstruction.
Q 19.6: A 22 year-old male presents with a 2-week history of an upper respiratory infection that hasn't improved after taking amoxicillin for 6 days. He notes persistent sore throat, intermittent fever, and a worsening nonproductive cough. Physical examination reveals bilateral diffuse crackles. What is the most appropriate antibiotic to initiate after discontinuing the amoxicillin? A Amoxicilin and clavulanic Acid B Azithromycin C Cephalexin D Ciprofloxacin E Trimethoprim/Sulfamethoxazole
The Correct Answer is: B The patient most likely has atypical pneumonia that responds best to macrolide antibiotics (B). Antibiotics that inhibit cell wall synthesis (A, C) are generally ineffective against these atypical organisms that are either intracellular or lack a cell wall.
Q 86.6: A 58-year-old female two days status-post coronary artery bypass graft (CABG) surgery is being treated with heparin to prevent thrombosis. Her routine CBC reveals a hemoglobin of 11.2 mg/dL, hematocrit of 35%, WBC count of 5.6, and platelet count of 22,000. In addition to discontinuing heparin, which of the following is the most appropriate intervention? A Administer prednisone B Administer warfarin C Bone marrow aspirate D CT of the abdomen E Observation
The Correct Answer is: B The patient most likely has developed heparin-induced thrombocytopenia (HIT), which is associated with qualitative platelet function changes that result in increased risk of thrombosis. She should begin warfarin (B) and be evaluated for thrombosis (e.g., lower extremity Dopplers). Prednisone (A) is not indicated for the treatment of HIT, and a bone marrow aspirate (C) would not aid in establishing the diagnosis. CT of the abdomen (D) would be indicated if she had symptoms consistent with thrombosis in that region, and observation (E) fails to address her increased thrombotic risk. Konkle B. Chapter 115. Disorders of Platelets and Vessel Wall. In: Longo DL, Fauci AS, Kasper DL, Hauser SL, Jameson JL, Loscalzo J, eds. Harrison's Principles of Internal Medicine. 18th ed. New York: McGraw-Hill; 2012. http://www.accessmedicine.com/content.aspx?aID=9100733 . Accessed March 22, 2013.
Q 7.4: A 76 year-old man with long-standing asthma presents to the clinic complaining of increased use of his albuterol and 2-3 nighttime awakenings over the last month. Which of the following is the most appropriate therapy? A Beclomethasone via a pressurized meter dose inhaler B Fluticasone dry powder inhaler C Montelukast orally D Prednisone orally E Salmeterol dry powder inhaler
The Correct Answer is: B The patient requires inhaled corticosteroid therapy for moderate-persistent asthma. Fluticasone (B) dry powder inhaler is a preferred agent in this geriatric patient who is at increased risk of having poor inhaler technique. Beclomethasone (A) via a pMDI is prone to deposition in the oropharynx, decreased efficacy, and increased side effects. These concerns can be mitigated through the use of a spacer. Montelukast (C) and salmeterol (E) are not preferred agents for this stage of asthma.
Q 32.25: A 62 year-old woman with a 40 pack year smoking history presents with a history of progressive dyspnea of several months duration. She denies hemoptysis, cough, fever, or orthopnea. Sputum production is positive. On physical exam, she shows evidence of accessory muscle use when breathing, sits in a slightly bent forward position, uses pursed lips, and you notice she is very thin. Auscultation reveals decreased vesicular breath sounds with prolonged expiration. Which of the following is the most likely diagnosis? A CHF B COPD C Lung cancer D Pulmonary emboli E Pulmonary fibrosis
The Correct Answer is: B The patient scenario is consistent with COPD (B) based on the history of progressive dyspnea and physical exam findings of obstructive lung disease (i.e., pursed lip breathing and decreased breath sounds with prolonger expiratory phase. The patient lacks physical exam findings of pulmonary edema due to CHF (A). She is at increased risk of lung cancer (C), but lacks red flag symptoms at this time. Pulmonary embolism (D) typically occurs in patients with venous stasis, hyper coagulable state, and vascular wall injury. Pulmonary fibrosis (E) would present with sign of restrictive lung disease (e.g., shallow breaths with rapid expiration)
A 53-year-old man presents to the emergency department because of fever, headache, and confusion. On physical examination, you note an obtunded man who appears acutely ill with temperature of 104°F, blood pressure of 128/76 mm Hg, pulse of 98, and respiratory rate of 20. The patient has stomatitis, nuchal rigidity, and a positive Kernig sign. CSF examination shows increased opening pressure, 80 WBC/mL (normal < 10/mL), mildly elevated protein, and normal glucose. Which of the following tests would confirm the most likely causative organism? A CT of the head B polymerase chain reaction test for herpes simplex virus C blood culture for herpes simplex virus D serum IgG for herpes simplex virus E MRI of the head
The Correct Answer is: B The patient's presentation is consistent with viral meningitis with potential encephalitis. The presence of active stomatitis indicates herpes simplex virus as the most likely causative organism. A CT of the head could be considered prior to performing a lumbar puncture and may show temporal lobe abnormalities that support a diagnosis of herpes virus encephalitis, but like an MRI will not identify the causative organism and has limited sensitivity. Of the three herpes tests described, the PCR technique is the most likely to identify the herpes simplex virus as the causative organism in the CSF due to its high sensitivity and specificity. Serum IgG indicates prior infection from herpes simplex virus but does not confirm the causative organism of the patient's encephalitis. Viral blood cultures for herpes simplex would likely show no growth even in the presence of herpes simplex virus encephalitis.
Q 114.7: A 53-year-old man presents to the emergency department because of fever, headache, and confusion. On physical examination, you note an obtunded man who appears acutely ill with temperature of 104°F, blood pressure of 128/76 mm Hg, pulse of 98, and respiratory rate of 20. The patient has stomatitis, nuchal rigidity, and a positive Kernig sign. CSF examination shows increased opening pressure, 80 WBC/mL (normal < 10/mL), mildly elevated protein, and normal glucose. Which of the following tests would confirm the most likely causative organism? A CT of the head B polymerase chain reaction test for herpes simplex virus C blood culture for herpes simplex virus D serum IgG for herpes simplex virus E MRI of the head
The Correct Answer is: B The patient's presentation is consistent with viral meningitis with potential encephalitis. The presence of active stomatitis indicates herpes simplex virus as the most likely causative organism. A CT of the head could be considered prior to performing a lumbar puncture and may show temporal lobe abnormalities that support a diagnosis of herpes virus encephalitis, but like an MRI will not identify the causative organism and has limited sensitivity. Of the three herpes tests described, the PCR technique is the most likely to identify the herpes simplex virus as the causative organism in the CSF due to its high sensitivity and specificity. Serum IgG indicates prior infection from herpes simplex virus but does not confirm the causative organism of the patient's encephalitis. Viral blood cultures for herpes simplex would likely show no growth even in the presence of herpes simplex virus encephalitis. (Aminoff et al., 2005, p. 30) Aminoff MJ , Greenberg DA , Simon RP. Clinical Neurology. 6th ed. New York, NY: McGraw-Hill; 2005.
Q 5.5: A 43-year-old woman presents to the outpatient clinic complaining of right eye redness, photophobia, and pain. She notes some blurred vision and denies the presence of discharge. On physical exam her visual acuity is 20/20 left eye, and 20/60 right eye. Her right eye has circumcorneal injections and the pupil is 3 mm and responds poorly to light. Her left pupil is 5 mm and responds well. Fluorescein staining of the eye is unremarkable and intraocular pressures are normal. Which of the following treatment regimens should be prescribed? A Homatropine 5% solution four times daily B Homatropine 5% solution four times daily and prednisolone 1% solution every 1 or 2 hours while awake C Prednisolone 1% solution every 1 or 2 hours while awake D Prednisolone 1% solution every 1 or 2 hours while awake and sulfacetamide 10% solution three times a day E Prednisone 60 mg by mouth once daily
The Correct Answer is: B The patient's presentation of acute uveitis is best treated with topical corticosteroids and cycloplegics (B) once infectious causes (e.g., HSV) have been ruled out. The addition of a cycloplegic helps reduce pain. Antibiotic drops (D) aren't indicated for acute uveitis.
Q 97.2: A 43-year-old woman presents complaining of a "pins and needles" sensation that started bilaterally in her feet 2 days ago. The sensation now extends up to her mid-thighs. On physical examination, she is noted to have mild sensory loss, weakness, and absent reflexes bilaterally in her legs. Which of the following is the most likely diagnosis? A diabetic peripheral neuropathy B Guillain-Barré syndrome C multiple sclerosis D myasthenia gravis E hypothyroidism
The Correct Answer is: B The pattern of sensory, motor, and reflex findings occurring over an acute time period is consistent with Guillain-Barré Syndrome. Diffuse diabetic peripheral neuropathy develops more insidiously than this case scenario. Multiple sclerosis presents with central nervous system (CNS) lesions that are unlikely to occur in this pattern. Myasthenia gravis causes intermittent motor symptoms without sensory involvement. Hypothyroidism may cause weakness and delayed reflexes, but is not the single best answer for this question. (Aminoff et al., 2005, p. 212) Aminoff MJ , Greenberg DA , Simon RP. Clinical Neurology. 6th ed. New York, NY: McGraw-Hill; 2005.
A 19-year-old woman presents to the emergency department complaining of headache. The headaches are generalized and increasing in intensity. They have not responded to over-the-counter (OTC) medications. She complains of approximately 1 week of blurred vision, intermittent diplopia, and vague dizziness. Her medical history includes obesity and acne. She takes Accutane and oral contraceptives. She is found to have bilateral papilledema, visual acuity of 20/30 on physical examination, and a normal MRI of the brain. The next most appropriate step would be A CT scan of the head B lumbar puncture C therapy with high-dose prednisone D stat cerebral arteriogram E reassurance and follow-up in the office in 6 months
The Correct Answer is: B The presence of headache associated with papilledema raises the concern for a brain tumor. The MRI excluded a mass lesion, raising a strong suspicion of pseudotumor cerebri. This is also known as benign intracranial hypertension. It is not a benign condition, however, since it causes severe headache and may result in visual loss. It is particularly frequent in obese adolescent girls and young women. The etiology is unknown but may be associated with the use of oral contraceptives, vitamin A, and tetracycline. *The presentation consists of headaches caused by an increase in intracranial pressure and blurring of vision. There may be diplopia, but the remainder of the neurologic examination is unremarkable. Papilledema is virtually always part of the presentation.* The mental status is normal. The differential diagnosis includes venous sinus thrombosis, sarcoidosis, and tuberculosis or carcinomatous meningitis. The last two are excluded by lumbar puncture. An abnormal cerebrospinal fluid is not consistent with pseudotumor cerebri. The diagnosis is made by excluding mass lesions with CT scan or MRI and demonstrating markedly increased intracranial pressure by lumbar puncture. The treatment involves weight loss, diuretics, and steroids. Repeat lumbar punctures to remove cerebrospinal fluid and decrease intracranial pressure are effective. In cases that are unresponsive to these measures, lumbar-peritoneal shunting is effective, as is unilateral optic nerve sheath fenestration. Effective treatment can improve headaches and prevent vision loss.
Q 97.49: You are monitoring a 30-year-old G2P1 at 40 weeks gestation, who is in an active stage of labor and is 6-cm dilated. The fetal heart tracing has a baseline heart rate of 140, with 7 to 10 beats of variability. With the last five contractions you have noted late decelerations. What would be the next most appropriate course of action? A Close observation of FHR tracing B Assessment of dilatation C Augment contractions with oxytocin D Intravenous analgesic E Surgical intervention
The Correct Answer is: B The presence of recurrent late decelerations should raise the suspicion for fetal distress. Vaginal evaluation for change in dilatation or cord prolapse, and to assess the fetal response to stimulation, are the first steps in evaluating the need for intervention.
Q 31.9: It is September and the radio is flooded with public service announcements recommending people get their flu shots early; flu is not yet endemic. Your patient, an 18 year old female, has come to see you in your family practice clinic with complaints of "flu-like" symptoms. She hoped the symptoms would resolve on their own, but it is now a week later and she is still experiencing them, and requests a rapid flu test. Which of the following is true regarding your patient and the rapid influenza tests? A It is best she waits at least a week before having you perform a rapid test, since a false negative may result if seen too soon from onset of symptoms. B You may warn your patient that given this time of year, the low prevalence in the community, and her delay in testing, the results are virtually useless and you would not recommend testing. C After obtaining a specimen to run the rapid test, you inform you patient that you will have her results in two to three days. D The greatest cost benefit would be achieved if antibiotics were prescribed once you receive a positive rapid test for her.
The Correct Answer is: B The rapid tests vary in terms of sensitivity and specificity. Research indicates that sensitivities are approximately 50% to 70%, while specificities are approximately 90% to 95%. Specimens to be used with rapid tests generally should be collected as close as is possible to the start of symptoms and usually no more than four to five days later in adults. In very young children, influenza viruses can be shed for longer periods; therefore, in some instances, testing for a few days after this period may still be useful. Most importantly, the positive and negative predictive values vary considerably depending upon the prevalence of influenza in the community. False-positive (and true-negative) influenza test results are more likely to occur when disease prevalence is low, which is generally at the beginning and end of the influenza season, as is the case here. False-negative (and true-positive) influenza test results are more likely to occur when disease prevalence is high, which is typically at the height of the influenza season. When disease prevalence is relatively low, the positive predictive value (PPV) is low and false-positive test results are more likely. By contrast, when disease prevalence is low, the negative predictive value (NPV) is high, and negative results are more likely to be true.
A 27-year-old African American female presents to the emergency department with low blood pressure of 100/40, palpitations, and shortness of breath. She is currently under treatment for Wolff-Parkinson-White syndrome and has been taking procainamide for the last two years. An electrocardiogram is obtained on the monitor and reveals the rhythm strip shown. What is the treatment of choice for this patient? A Intravenous calcium B Intravenous magnesium C Oral potassium D Subcutaneous epinephrine E Metoprolol
The Correct Answer is: B The rhythm strip reveals ventricular tachycardia in the form of torsades de pointes. In this case, the primary medical intervention is to administer magnesium sulfate to counter the irregular activity. Antiarrhythmics, antidepressants, and some antibiotics can be responsible for this arrhythmia. In addition to the magnesium, administration of beta-blockers can also be helpful.
Q 100.2: A 26-year-old male has a history of schizophrenia that has been characterized by marked incoherence and a silly affect. Which classification of schizophrenia does this most closely resemble? A Catatonic B Disorganized C Paranoid D Residual E Undifferentiated
The Correct Answer is: B The signs and symptoms that this patient has exhibited is most consistent with disorganized schizophrenia. Catatonic schizophrenia is characterized by severe psychomotor disturbances of either rigidity with mutism or excitement. Paranoid schizophrenia is characterized by persecutory or grandiose delusions, often accompanied by hallucinations. Residual schizophrenia is characterized by an episode that warrants a diagnosis of schizophrenia, but there are no current psychotic symptoms present; however, they continue to exhibit milder signs, such as social withdrawal, flat affect, and eccentric behaviors. In undifferentiated schizophrenia, the symptoms are not specific enough to allow for categorization in any of the other subtypes. (McPhee SJ, Papadakis MA. Current Medical Diagnosis & Treatment, 2010, p. 952)
Q 87.1: A 49-year-old male presents to the clinic with symptoms of nausea, occasional vomiting, vague epigastric pain, fatigue and weight loss of 35 lbs. over the past few months. On exam you find a palpable abdominal mass. Which of the following laboratory findings are most consistent with the suspected diagnosis? A Elevated CEA B Iron deficiency anemia C Elevated CA 125 D Megaloblastic anemia E Elevated CRP
The Correct Answer is: B The suspected diagnosis is gastric carcinoma. The most common laboratory finding is iron deficiency anemia related to chronic blood loss. Tumor markers are of no value, and an elevated CRP is not commonly present. Megaloblastic anemia is not the type of anemia typically present in gastric carcinoma. (McPhee SJ, Papadakis MA. Current Medical Diagnosis & Treatment, 2010, p. 1471)
Q 65.4: A 24-year-old male has an eight-month history of loose thought associations, social withdrawal, auditory hallucinations, and deterioration in his personal appearance and hygiene. Upon examination, he is noted to have a flat affect and perceptual distortions, and he behaves like he is detached from his own actions. He is started on a neuroleptic medication, and a few weeks later he is noted to pace frequently and seems to be unable to sit or stand still. What is the extrapyramidal symptom this patient is exhibiting called? A Acute dystonia B Akathisia C Drug-induced parkinsonism D Tardive dyskinesia E Verbigeration
The Correct Answer is: B The symptom this patient is exhibiting is akathisia, the most common extrapyramidal symptom of the neuroleptic medications. Acute dystonias from neuroleptic medications consist of bizarre muscle spasms of the head, neck, and tongue. Drug-induced parkinsonism consists of the same symptoms as idiopathic parkinsonism, including signs of reduced facial and arm movements, festinating gait, rigidity, and pill-rolling tremor. Tardive dyskinesia usually appears months or years after starting neuroleptic medication, and consists of involuntary stereotyped movements of the face, mouth, tongue, trunk, and limbs. Verbigeration is a symptom of schizophrenia and other psychotic disorders that consists of repetition of senseless words or phrases, but is not a side effect of neuroleptic medication.
Q 117.15: What is the most common cause of treatment failure in tuberculosis? A Drug resistance B Noncompliance to therapy C Inappropriate selection of medication D Sepsis E Death
The Correct Answer is: B The usual reason for failure is simply due to the patient not continuning their treatment plan, regardless of the severity of the disease. Drug resistence, while present in some cases does not preclude the patient from treatment failure. Ongoing sepsis is not a reason to have treatment failure. (Chesnutt MS, Prendergast TJ. Current Medical Diagnosis and Treatment, 2011, Chapter 9, Pulmonary Disorders)
Q 65.3: A 56-year-old woman is currently being treated with daily warfarin for thrombophlebitis. She has contracted a serious lower respiratory tract infection and is admitted to the hospital. The patient is started on ciprofloxacin upon admission, and after 3 days of treatment, her INR increases from 2.7 to 7.4. She also reports a nosebleed on the third night in the hospital. Her lower respiratory function has improved slightly, but the infection has still not resolved. Which of the following is the most likely explanation for the increase in the patient's INR? A decreased warfarin absorption in the small intestine B decreased warfarin metabolism by the liver C increased plasma protein binding of warfarin D increased warfarin absorption in the small intestine E increased warfarin metabolism by the liver
The Correct Answer is: B There are several clinically important warfarin drug interactions, with most of them causing an increase in the drug's anticoagulant effect (ie, increasing the INR). Warfarin metabolism occurs via hepatic cytochrome P450 enzymes that can be inhibited by a large number of drugs, including the fluoroquinolones. When this inhibition occurs, plasma levels of warfarin rise, thereby enhancing the anticoagulant effect.
Q 39.27: A six-month-old infant presents to the primary care provider with complaints of a spreading rash. The physical exam shows multiple yellow-brown macules and plaques that urticate when stroked. What would an appropriate treatment regimen include? A referral to child protective services B patient education to avoid NSAIDS and extremes of temperature C reassure the parents that it will resolve spontaneously within a week D prescribe ketoconazole cream bid x 2 week
The Correct Answer is: B This condition is consistent with urticarial pigmentosa, and it will resolve over time. However, certain things such as NSAIDS, codeine, and scopolamine, as well as extreme temperatures, can cause such reactions as anaphylaxis. This condition is frequently mistaken for child abuse, as the lesions can look like small finger sized bruises. It is consistent, however, with urticaria pigmentosa, which is an accumulation of mast cells in the skin, as indicated by urtication of the lesion after gentle stroking. Urticaria pigmentosa will resolve; however, it will take longer than a week to resolve. Ketoconazole cream is an antifungal that is used to treat fungal infections.
Q 76.9: A 30 year old female complains of a dome shaped slightly erythematous nodule on her right thigh. Upon clinical examination it exhibits the "dimple sign". The patient states the lesion is not changing and is asymptomatic. What is the most likely diagnosis? A Basal cell carcinoma B Dermatofibroma C Closed comedone D Hypertrophic scar
The Correct Answer is: B This is the classic presentation of a Dermatofibroma. The dimple sign is when a depression forms after the lesion is laterally compressed between the fingers (Wolff & Johnson, p 224-226). (Fig 9-54, Wolff K, Johnson RA: Fitzpatrick's Color Atlas & Synopsis of Clinical Dermatology, 6 th Ed: http://www/accessmedicine.com)
Q 32.22: The term "nursemaid's elbow" refers to which of the following physical conditions? A A fracture of the humerus B A subluxation of the radial head C Inflammation at the lateral epicondyle D Inflammation of the medial epicondyle E Olecranon bursitis
The Correct Answer is: B This is the most common elbow injury in children under the age of 5. The injury generally occurs when the child's arm is forcefully pulled when the elbow is in an extended position and the forearm is pronated. Fibers of the annular ligament that encircle the radial neck become trapped between the radius and ulna. On presentation, children hold their arm in slight flexion and pronation. There is no fracture associated with nursemaid's elbow and the olecranon bursa is unaffected. Nursemaid's elbow is an acute injury, while medial and lateral epicondylitis (also known as golfer's elbow and tennis elbow respectively) are generally chronic conditions.
The term "nursemaid's elbow" refers to which of the following physical conditions? A Fracture of the humerus B Subluxation of the radial head C Inflammation at the lateral epicondyle D Inflammation of the medial epicondyle E Olecranon bursitis
The Correct Answer is: B This is the most common elbow injury in children under the age of 5. The injury generally occurs when the child's arm is forcefully pulled when the elbow is in an extended position and the forearm is pronated. Fibers of the annular ligament that encircle the radial neck become trapped between the radius and ulna. On presentation, children hold their arm in slight flexion and pronation. There is no fracture associated with nursemaid's elbow and the olecranon bursa is unaffected. Nursemaid's elbow is an acute injury, while medial and lateral epicondylitis (also known as golfer's elbow and tennis elbow respectively) are generally chronic conditions.
Q 63.2: A 25-year-old female presents for a skin exam. She has no family history of skin cancer, and has no moles that itch, bleed, or ulcerate. She is concerned about a mole on her arm that is surrounded by a hypopigmented area. She states that the mole appears to be decreasing in size. What would appropriate management of the lesion include? A excision with 2mm margin B reassurance C hydrocortisone 2.5 % ointment bid x 2 weeks D ketoconazole cream bid x 2 weeks
The Correct Answer is: B This lesion is consistent with a halo nevus. The depigmented macule that surrounds the nevus is similar to vitiligo and may consume the nevus; therefore, reassurance that this will resolve is appropriate for this patient. There is no need for excision with margins as this is not indicative of malignant changes. Hydrocortisone 2.5% ointment is a low potency topical steroid. This treatment is not indicated for a halo nevus. Ketoconazole is an antifungal used to treat fungal infections. There are no indications of fungal infection in this halo nevus.
Q 10.2: A 71-year-old male who has a history of hypertension presents with a new finding of atrial fibrillation. He is independent, drives his own car, and tends to his daily activities without assistance. He is currently having no symptoms, and his heart rate is 90, with a blood pressure of 146/76. Given this clinical scenario, what is the best pharmacologic anticoagulation treatment for this patient? A No anticoagulation B Aspirin C Aspirin and warfarin D Aspirin and clopidogrel E Warfarin and clopidogrel
The Correct Answer is: B This patient has a lower CHADS2 score of 1 (HTN) and would have adequate risk reduction with the therapy of aspirin alone (B). Higher risk patients with a CHADS2 score of 2 or higher qualify for adding warfarin (C), and the use of aspirin and clopidogrel (D) is not indicated. No anticoagulation (A) would put the patient at risk for CVA, and warfarin and clopidogrel (E) would create an over anticoagulated environment and increase risk for bleeding.
Q 34.5: A 12-year-old male presents with hyperkeratotic papules located on both hands. What is the causative organism? A Herpes simplex virus B Human papilloma virus C Parvovirus B19 D Varicella zoster virus
The Correct Answer is: B This patient has verruca vulgaris or the common wart. The causative organism is the human papillomavirus (HPV). They can affect patients of any age and can occur on any skin surface. There is a predilection for the hands and fingers.
A 66-year-old male with a history of hypertension, diabetes mellitus, and hypercholesterolemia presents by emergency medical services (EMS) to the emergency department complaining of severe chest pain with radiation into his back. The patient states that he was feeling well in the morning, but while performing some light activity he felt a "ripping" sensation in his back, which he initially thought was a pulled muscle. The pain continued and the patient started to have chest pain, shortness of breath, and lightheadedness. On initial examination the patient is still in pain, pale, diaphoretic, and has a blood pressure of 85/40. His chest is clear to auscultation, and he has a 3/6 diastolic murmur best appreciated at the base of the heart. Given this clinical scenario, what is the most likely diagnosis? A Pneumothorax B Dissecting thoracic aneurysm C Acute myocardial infarction D Pulmonary embolus E Esophageal perforation
The Correct Answer is: B This patient is exhibiting a history and physical examination that is consistent with a thoracic aneurysm. The patient's history of hypertension, along with the "ripping" sensation in his back and hypotension give a clinical presentation that is suggestive of a thoracic aneurysm dissection (B). A pneumothorax (A) would have more pleuritic characteristics, and chest pains without the ripping sensation or loud diastolic murmur would be more likely in a situation of myocardial infarction (C). Pulmonary embolus (D) and esophageal perforation (E) would typically not present with the above complaints or physical exam findings.
Q 121.11: A 66-year-old male with a history of hypertension, diabetes mellitus, and hypercholesterolemia presents by emergency medical services (EMS) to the emergency department complaining of severe chest pain with radiation into his back. The patient states that he was feeling well in the morning, but while performing some light activity he felt a "ripping" sensation in his back, which he initially thought was a pulled muscle. The pain continued and the patient started to have chest pain, shortness of breath, and lightheadedness. On initial examination the patient is still in pain, pale, diaphoretic, and has a blood pressure of 85/40. His chest is clear to auscultation, and he has a 3/6 diastolic murmur best appreciated at the base of the heart. Given this clinical scenario, what is the most likely diagnosis? A Pneumothorax B Dissecting thoracic aneurysm C Acute myocardial infarction D Pulmonary embolus E Esophageal perforation
The Correct Answer is: B This patient is exhibiting a history and physical examination that is consistent with a thoracic aneurysm. The patient's history of hypertension, along with the "ripping" sensation in his back and hypotension give a clinical presentation that is suggestive of a thoracic aneurysm dissection (B). A pneumothorax (A) would have more pleuritic characteristics, and chest pains without the ripping sensation or loud diastolic murmur would be more likely in a situation of myocardial infarction (C). Pulmonary embolus (D) and esophageal perforation (E) would typically not present with the above complaints or physical exam findings. Source: http://www.accessmedicine.com/content.aspx?aID=3651494
Q 26.6: An 84-year-old male is admitted to the hospital for the chief complaint of syncope. The history provided states that the patient was in his normal state of health and feeling well when he had fallen, with an apparent loss of consciousness for 25-30 seconds. He has a history of hypertension and arthritis, and is well managed on medications that include lisinopril and acetaminophen. During his time on the telemetry unit it's noted that the patient has periods of sinus bradycardia in the 30s, followed by normal sinus rhythms that fluctuate in the 60 to 120 range. During several of the bradycardia episodes the patient becomes symptomatic with shortness of breath, lightheadedness, and dizziness. His blood pressure during these episodes is measured at 88/56. Given this clinical scenario, what is the most likely diagnosis for this patient? A Ventricular tachycardia B Sick sinus syndrome C First-degree AV block D Wolff-Parkinson-White syndrome E Premature atrial contractions
The Correct Answer is: B This patient is exhibiting signs and symptoms related to sick sinus syndrome (B). There is no evidence in the clinical scenario to suggest any ventricular ectopy (A), and the patient's age and characteristics of the arrhythmia are not consistent with Wolff-Parkinson-White syndrome (D), or other skipped beat abnormalities (C and E).
Q 63.7: A 29-year-old female complains of a two-month history of easy bruising. She describes that the bruising is located primarily on her shins, but has noted them on other areas as well. She also describes red freckles on her lower extremities. On exam, you note non-blanching and non-palpable purpura to both legs and petechiae. She denies recent illnesses and states that she has essentially been feeling fine. The PE is normal other than the skin findings. Which diagnostic study would be most helpful as a first choice? A Antiplatelet antibody test B Complete blood count C Direct antiglobulin test D Reticulocyte count E Thyroid function
The Correct Answer is: B This patient is exhibiting signs of Idiopathic (Immune) thrombocytopenic purpura. The diagnosis is based on history, physical examination, blood count, and blood film. The American Society of Hematology guidelines recommend no further diagnostic studies. The other studies may be useful if you believe another underlying disease is causing the symptoms and thrombocytopenia, but a complete blood count should be sufficient.
Q 45.4: A 6-year-old male presents with crusted erythematous lesions on the nose, mouth, and chin. He has a history of atopic dermatitis. Which of the following should be part of an appropriate treatment regimen? A hydrocortisone 2.5% ointment bid x 2 weeks B topical mupirocin ointment bid x 7-10 days C lotrisone cream bid x 2 weeks D ketoconazole cream bid x 7 -10 days
The Correct Answer is: B This patient is suffering from impetigo. Impetigo is easily treated with a topical antibiotic, such as mupirocin. Hydrocortisone is a topical steroid and may worsen the infection. Lotrisone is a combination medication that includes an antifungal and a topical steroid, neither of which is indicated in this patient. Ketoconazole cream is used to treat fungal infections, not bacterial infections.
Q 26.8: A 22-year-old African American male presents to the emergency department with shortness of breath, which started 2 hours prior to arrival. He does not have a history of pulmonary disease that he is aware of, and he states that in the past at random events he has had similar episodes. He does nothing to get the episodes to stop, and he also states that he feels his chest pounding at the same time of the shortness of breath. He has no medical history that he is aware of, and he takes no medications or any illicit drugs. On examination he is alert, awake, and oriented. His vital signs show T 99.0, P 142, R 18, and BP 132/82. His chest x-ray is negative for any acute cardiopulmonary disease, and his electrocardiogram has an irregularly irregular rhythm and a rate of 142 with visible delta waves. Based on the information provided, what is the most likely diagnosis for this patient? A Torsade de pointes B Atrial fibrillation with Wolff-Parkinson-White syndrome C Atrial fibrillation with re-entrant tachycardia D Junctional tachycardia E Atrio-ventricular re-entrant tachycardia
The Correct Answer is: B This patient most likely has Wolff-Parkinson-White (WPW) syndrome (B), based on the explanation of the delta wave findings on EKG. In addition, the patient is also having atrial fibrillation with a rapid ventricular response. Patients that present with a new finding of WPW can often present with atrial fibrillation, along with symptoms such as those described above. There is nothing noted that gives the history of ventricular tachycardia (D), or re-entrant tachycardia (C and E). Torsade de points (A) is an ventricular tachycardia without evidence of a noticeable rhythm.
Q 31.5: A 69-year-old female presents with her family to her primary care provider. The family is concerned that the patient is increasingly forgetful and has difficulty remembering the names of people and places involved in her life for many years. The symptoms have worsened over the past five months. She often gets lost while driving and is found in unusual parts of the house. She has not had any physical complaints, has had no weight changes, and denies recent illnesses. Which of the following is the most likely explanation for these symptoms? A Delirium B Dementia C Depression D Hydrocephalus E Hypothyroidism
The Correct Answer is: B This patient represents a deterioration of mental functioning over time, with an absence of physical illness symptoms. The decreased cognition is causing an impairment of activities of daily living. There is no indication of altered consciousness. The onset was not rapid, nor associated with recent medical conditions or other physical symptoms. This is most consistent with dementia. The other etiologies among the answer choices must be ruled out, although this patient does not exhibit the classic findings for any of the other diagnoses.
Q 97.123: A 42-year-old woman has experienced recent weight gain, heavy periods, fatigue, cold intolerance, and constipation. She has a rough voice, and her rate of speech is slow. Physical exam is significant for an enlarged thyroid, slow reflexes, and the presence of brittle and coarse hair. She denies any history of bipolar disease or treatment with lithium. Laboratory tests show an elevated TSH and low free T 4 . What is the most appropriate treatment for this patient? A propylthiouracil (PTU) B levothyroxine C surgical resection D radioiodide ablation
The Correct Answer is: B This patient's signs and symptoms are consistent with hypothyroidism. Treatment of choice is levothyroxine, which is partially converted in the body to T 3 . Significant increases are seen within 1 to 2 weeks, with maximum levels reached in 3 to 4 weeks.
Q 116.14: A 42-year-old woman has experienced recent weight gain, heavy periods, fatigue, cold intolerance, and constipation. She has a rough voice, and her rate of speech is slow. Physical exam is significant for an enlarged thyroid, slow reflexes, and the presence of brittle and coarse hair. She denies any history of bipolar disease or treatment with lithium. Laboratory tests show an elevated TSH and low free T 4 . What is the most appropriate treatment for this patient? A propylthiouracil (PTU) B levothyroxine C surgical resection D radioiodide ablation
The Correct Answer is: B This patient's signs and symptoms are consistent with hypothyroidism. Treatment of choice is levothyroxine, which is partially converted in the body to T 3 . Significant increases are seen within 1 to 2 weeks, with maximum levels reached in 3 to 4 weeks. (Friedman and Herman-Bonert, 2009, pp. 652-653) Friedman TC , Herman-Bonert VS. Thyroid gland. In: McPhee SJ , Papadakis MA, eds. Current Medical Diagnosis and Treatment. 48th ed. New York, NY: McGraw-Hill; 2009.
Q 98.11: A 64-year-old man has been experiencing signs and symptoms compatible with diverticular disease for the past 3 weeks. He now presents to the emergency department malnourished with severe left-sided lower abdominal pain. After appropriate workup and hydration, he is taken to the operating room where a perforated sigmoid colon is discovered with gross contamination. What is the most appropriate surgical intervention at this time? A Left colectomy with primary anastomosis B Hartmann procedure C Proctocolectomy D Abdominoperineal resection E Low anterior resection
The Correct Answer is: B This vignette is consistent with an emergent resection in an unprepared patient. The most appropriate therapy for an acute perforation is a Hartmann procedure, which includes resection of the affected portion of the bowel, a temporary diverting colostomy, and oversewing of the distal rectal stump; the second stage of the procedure will involve taking down the colostomy with anastomosis to the rectal stump. A colectomy with a primary anastomosis should not be done when the bowel is unprepared due to the significant risk of infection and leakage of the bowel at the site of the anastomosis. Abdominoperineal resection is used in the treatment of malignant disease of the lower rectum. In this procedure, a permanent colostomy is created and the entire rectum, anal canal, and anus are removed. In the management of benign disease of the lower rectum, a proctocolectomy is appropriate to preserve anal function.
Q 97.35: A 49-year-old woman has had irregular menses for more than two years. Most of the time she requires only 1-2 mini-pads per day to handle the flow, but recently she soaked through a dozen maxi-pads in a day. On physical examination, she appears well and her pelvic exam in unremarkable. Her serum progesterone is low and her hematocrit is 39%. A transvaginal ultrasound reveals an endometrial stripe of 11 mm. Which of the following is the most appropriate next step in her management? A Abdominal hysterectomy B Endometrial biopsy C Fractional curettage D Hysteroscopy E Watchful waiting
The Correct Answer is: B This woman's excessive bleeding and endometrial stripe > 5 mm raises the suspicion for endometrial hyperplasia or cancer. The next step is an endometrial biopsy, which can be performed without anesthesia in the outpatient setting. Hysterectomy (A) is inappropriate without first establishing a diagnosis for the bleeding. Fractional curettage (C) is the definitive procedure for diagnosis of endometrial carcinoma, but requires anesthesia with its attendant risks. Hysteroscopy (D) is more invasive than endometrial biopsy and may spread tumor cells into the peritoneal cavity. Watchful waiting (E) is inappropriate given with size of the endometrial stripe.
Q 30.3: A 64-year-old African American female presents to the clinic for evaluation of her hypertension, which she has had for several years. In the past she had been taking hydrochlorothiazide and lisinopril, with little effect on her blood pressure management. At today's visit, she has no complaints and feels well. Her vitals show T m 96.6, P 85, R 18, BP 191/99. She has no jugular venous distention seen on the neck exam, her lungs are clear, and cardiac exam has a regular rate and rhythm without murmur or gallop. Her abdomen is soft, non-tender, and a bruit is appreciated at the mid-abdomen just a few centimeters below the epigastric region. There is no fullness or enlargement of the abdominal aorta on palpation. Based on the history and clinical findings, what is the most appropriate procedure for this patient? A Hepatic venous angiography B Renal artery angiography C Cardiac catheterization D Lower extremity arteriography E Venous duplex ultrasound of lower extremities
The Correct Answer is: B To appreciate the extent of this patient's renal artery stenosis by history and physical exam, the patient would benefit the best by renal artery angiography (B). This test will help determine the extent of stenosis, as well as map the surrounding arterial supply. Hepatic venous arteriography (A) and cardiac catheterization (C), and lower extremity arteriography (D) would not be indicated given the physical exam findings. Venous duplex ultrasounds would not yield the appropriate information given this clinical scenario.
Q 72.9: A 62-year-old man with a history of hypertension, diabetes mellitus type 2, hyperlipidemia, and chronic tobacco use presents to the office with complaints of a retrosternal chest pressure, associated with diaphoresis, nausea, and dyspnea, radiating down his left arm for the last 45 minutes after mowing his lawn. The patient's vital signs are stable, and on physical examination a new systolic murmur is appreciated. His EKG demonstrates evidence of acute anterolateral myocardial infarction on EKG, with ST segment elevation across the precordial leads, indicative of left anterior descending coronary artery stenosis. Which of the following cardiac markers would be expected to remain elevated one week later? A CK-MB B Troponin I C BNP D Creatine kinase E Myoglobin
The Correct Answer is: B Troponin elevation in acute myocardial infarction may be noted within two hours after myocardial infarction. It is usually elevated within 6 to 10 hours, peaks at 12 hours, and may remain elevated for 7 to 10 days; thus, choice B is the answer. Troponin elevation is rapidly replacing CK-MB as the diagnostic cardiac marker of choice for AMI. Choice A, CK-MB, peaks earlier than creatine kinase, and is cleared within two days. Choice C, BNP, is a cardiac marker followed in patients with congestive heart failure, and unless the patient develops heart failure in the next 7 days, is unlikely to be elevated. Creatine kinase becomes elevated within 4 to 8 hours, peaks within 12 to 24 hours, and returns to normal within 3 to 4 days. Serum myoglobin levels rise within 3 hours of symptoms and are elevated at 6 to 8 hours. Myoglobin peaks at 4 to 9 hours, and, with normal kidney function, returns to baseline within 24 hours. (Tintinalli et al., 2011, Chapter 52)
Q 38.13: First-time parents present to the clinic with their 2-week old infant for a well-child visit. Which of the following strategies should be recommended to parents to help prevent sudden infant death syndrome (SIDS)? A Avoid pacifier use B Encourage tummy time while awake C Increase room temperature D Place infant on side for sleep E Use home monitors
The Correct Answer is: B Tummy time (B) helps infants develop strength to avoid situations that compromise breathing. Pacifiers (A) are recommended to help avoid bottle feedings as a sleep aid since they are associated with an increased risk of SIDS. Excess warmth (C) and prone or side (D) sleeping positions also increase the risk of SIDS. Home monitors (E) are not effective in the prevention of SIDS.
A 46-year-old African American male is seen in the emergency department with upper right quadrant pain that radiates to the right infrascapular area. The pain is colicky and was precipitated by a meal of fried fish and French fries. Which of the following diagnostic studies is the initial study of choice for this patient? A Plain abdominal x-ray B Ultrasonography C Radionuclide scan (HIDA scan) D Computed tomography (CT)
The Correct Answer is: B Ultrasonography is the first-line study in the evaluation of patients presenting with signs and symptoms of biliary disease. The sensitivity and specificity is 95%. It can detect stones, dilation of biliary ducts, thickening of the gallbladder, and pericolic collections of fluid and can also provide information pertaining to associated liver or pancreatic pathology.
Q 98.50: A 46-year-old African American male is seen in the emergency department with upper right quadrant pain that radiates to the right infrascapular area. The pain is colicky and was precipitated by a meal of fried fish and French fries. Which of the following diagnostic studies is the initial study of choice for this patient? A Plain abdominal x-ray B Ultrasonography C Radionuclide scan (HIDA scan) D Computed tomography (CT)
The Correct Answer is: B Ultrasonography is the first-line study in the evaluation of patients presenting with signs and symptoms of biliary disease. The sensitivity and specificity is 95%. It can detect stones, dilation of biliary ducts, thickening of the gallbladder, and pericolic collections of fluid and can also provide information pertaining to associated liver or pancreatic pathology.
Q 91.4: A 46-year-old African American male is seen in the emergency department with upper right quadrant pain that radiates to the right infrascapular area. The pain is colicky and was precipitated by a meal of fried fish and French fries. Which of the following diagnostic studies is the initial study of choice for this patient? A Plain abdominal x-ray B Ultrasonography C Radionuclide scan (HIDA scan) D Computed tomography (CT)
The Correct Answer is: B Ultrasonography is the first-line study in the evaluation of patients presenting with signs and symptoms of biliary disease. The sensitivity and specificity is 95%. It can detect stones, dilation of biliary ducts, thickening of the gallbladder, and pericolic collections of fluid and can also provide information pertaining to associated liver or pancreatic pathology. (Danziger, 2006, p. 339) Danzinger RG , Nauta R , Park J. Biliary tract. In: Lawrence PF, ed. Essentials of General Surgery. 4th ed. Philadelphia, PA: Lippincott Williams & Wilkins; 2006.
Q 89.9: The best initial diagnostic modality to diagnose cholelithiasis is which one of the following? A CT scan of the abdomen B ultrasound of the abdomen C oral cholecystogram D abdominal plain film E MRI of the abdomen
The Correct Answer is: B Ultrasound has replaced oral cholecystograms as the test of choice for diagnosing cholelithiasis. CT is useful in the evaluation of the acute abdomen but the sensitivity for viewing the gallstones is poor. KUB is also not a sensitive study for cholelithiasis. MRI is expensive and not recommended as an initial screening exam for gallstones but can be used if ultrasound is equivocal. (Paumgartner, 2009, pp. 541-542) Paumgartner G , Greenberger NJ. Gallstone disease. In: Greenberger NJ, ed. Current Diagnosis & Treatment: Gastroenterology, Hepatology, & Endoscopy. New York, NY: McGraw-Hill; 2009.
Q 36.4: A 1-year-old female is having a 2-day history of fever (102 0 F oral), rhinorrhea, and dry cough, with a decreased appetite. The mother states that her daughter has been less active, and her fluid intake has decreased for her age. On exam, the child is non-toxic appearing, has a rectal temperature of 100.2 0 F, and has nasal flaring and a respiratory rate of 45, rhinorrhea, moist mucous membranes, and a minimal wheeze heard bilaterally. Her chest x-ray has no specific findings. Based on these findings, what is the initial ancillary test to confirm the diagnosis? A Acid fast bath test B Viral nasal washings C Sputum culture and sensitivity D Blood cultures E Throat culture
The Correct Answer is: B Viral nasal washings are the best choice for determining RSV infection that causes bronchiolitis. Sputum and blood cultures do not grow the agents that would typically cause the infection to occur, and you are obtaining a sample from a different part of the respiratory tract and blood.
Q 32.1: A 33 year-old man with long-standing, recurrent, abdominal pain is diagnosed with anemia secondary to vitamin B12 deficiency. Which of the following gastrointestinal disorders most likely explains his diagnoses? A Achalasia B Crohn's disease C Duodenal ulcer D Pancreatitis E Ulcerative colitis
The Correct Answer is: B Vitamin B 12 complexed with intrinsic factor produced by gastric parietal cells is absorbed in the ileum, which is commonly affected by Crohn's disease (B). Achalasia (A) can lead to malnutrition and anemia secondary to vitamin B12, folate, and iron, but presents as difficulty swallowing, chest pain, and heartburn. Duodenal ulcer (C) and ulcerative colitis (E) may result in chronic blood loss and the development of iron deficiency anemia. Pancreatitis (D) causes malabsorption of fat soluble vitamins and may also lead to vitamin B12 deficiency in rarer instances.
Q 109.15: A 74-year-old male has been diagnosed with aplastic anemia. His temperature is 37.4°C, heart rate is 68, respiratory rate is 20, and blood pressure is 130/86. Mild petechiae are noted to the extremities and there is some scattered bruising. The patient is currently taking Captopril, HCTZ, Lipitor, and Flo-max. Laboratory findings include: WBC: 2.9 HGB: 10.4 PLT: 54,000 Initial management for this patient should include which of the following? A Broad spectrum antibiotics B Discontinuation of Captopril C Immediate assessment for allogeneic stem cell transplant D Irradiated and leukocyte depleted red cell transfusion E Platelet transfusion
The Correct Answer is: B While the laboratory findings demonstrate a pancytopenia, levels are not low enough or physical findings are not suggestive of the need for transfusions at this time. Given the patients age, he is not eligible for an allogeneic stem cell transplant. He has no physical findings suggestive of a systemic infection requiring antibiotics. Captopril has a low risk of causing pancytopenias. It is best to discontinue this drug first as a possible cause of the pancytopenia, while continue to treat this patient symptomatically. (Lichtman et al., Williams Hematology 8e, Chapter 34, Aplastic Anemia: Acquired and Inherited)
Q 13.8: A 59-year-old female patient with no complaints is undergoing routine physical examination in your family practice office and has a hypochromic, microcytic anemia that was not present on her physical examination last year. Which of the following is the most important cause to rule out in this patient? A acute pathology secondary to aplastic anemia B chronic anemia secondary to colon cancer C Falconi's anemia D iron deficiency anemia E medication-induced anemia
The Correct Answer is: B While there are many causes of hypochromic, microcytic anemia, high on the differential is blood loss, commonly occult and long standing in nature. This can be from heavy menstruation or gastrointestinal loss. This later is commonly caused by an occult colon cancer. This patient should initially receive a sensitive screen such as an immunochemical fecal occult blood test (IFOBT) and, given her age, a colonoscopy should be strongly considered.
Q 105.13: All other things being equal, the level of coding allowed in the outpatient setting is most dependent upon which of the following? A chronicity of the patient complaint B complexity of medical decision-making and/or time spent with the patient C number of items detailed within each section during the review of systems D patient insurance E patient acuity
The Correct Answer is: B With all other things being equal, the complexity of the medical decision-making is usually what determines coding levels. A patient visit may also be coded based upon the time spent with the patient. A minimum number of items must be documented in the History of Present Illness, the physical examination, and the review of systems, along with other factors, but medical decision making cannot be impacted positively by completing what is ostensibly unnecessary information. (LeBlond et al., 2009, Chapter 2)
Q 117.7: Myasthenia gravis symptomology is due to an autoimmune attack on which of the following receptors? A Alpha-adrenergic B Acetylcholine C Calcium D Dopamine E Norepinephrine
The Correct Answer is: B Within normal neuromuscular junctions, the release of acetylcholine and stimulation of the acetylcholine receptors result in muscle contraction. In myasthenia gravis, the available acetylcholine receptors are decreased due to autoimmune attack. This leads to muscle weakness and decreased functionality. Alpha-adrenergic receptors are the receptors for catecholamines, such as norepinephrine. The functions are diverse, based upon subtype and location, but may include vasoconstriction of arteries and veins and decreased gastrointestinal smooth muscle motility. Dopamine, a neurotransmitter and form of catecholamine, has unique receptors. The calcium-sensing receptor in the parathyroid regulates calcium homeostasis, which is unrelated to myasthenia gravis. (Fauci et al., Harrison's Principles of Internal Medicine, 17e, Chapter 381, Myasthenia Gravis and Other Diseases of the Neuromuscular Junction)
Q 23.1: You are performing a mental status exam on a 19-year-old male patient who was brought to the Emergency Department because of bizarre behavior. As you are speaking with him he keeps repeating words that rhyme with words that either you or he say, but make no sense and are unrelated to your questions. How will you document this behavior in his chart? A Circumstantiality B Clanging C Flight of ideas D Perseveration E Tangentiality
The Correct Answer is: B Word association based on rhyme is called clang association (B) and may be seen in psychotic disorders. Circumstantiality (A) is a disturbance in fluency where the speaker meanders on many side topics before returning to the topic at hand. Flight of ideas (C) is a rapid transition from thought to thought, leading to the speaker losing track of the original idea. Perseveration (D) is a fixed focus returning again and again to the same thought. Tangentiality (E) refers to a disturbance in continuity where the speaker shifts from one thought to another that may be only vaguely related.
You are performing a mental status exam on a 19-year-old male patient who was brought to the Emergency Department because of bizarre behavior. As you are speaking with him he keeps repeating words that rhyme with words that either you or he say, but make no sense and are unrelated to your questions. How will you document this behavior in his chart? A Circumstantiality B Clanging C Flight of ideas D Perseveration E Tangentiality
The Correct Answer is: B Word association based on rhyme is called clang association (B) and may be seen in psychotic disorders. Circumstantiality (A) is a disturbance in fluency where the speaker meanders on many side topics before returning to the topic at hand. Flight of ideas (C) is a rapid transition from thought to thought, leading to the speaker losing track of the original idea. Perseveration (D) is a fixed focus returning again and again to the same thought. Tangentiality (E) refers to a disturbance in continuity where the speaker shifts from one thought to another that may be only vaguely related.
Q 44.9: A 28-week-premature infant is noted to have increasing tachypnea and difficulty breathing, with diffusely decreased breath sounds on exam shortly after delivery. A chest x-ray reveals diffuse, bilateral atelectasis and air bronchograms. Which of the following is the most likely diagnosis? A Acute asthma exacerbation B Hyaline membrane disease C Meconium aspiration D Pleural effusion E Spontaneous pneumothorax
The Correct Answer is: B Premature infants, due to a lack of surfactant, develop marked atelectasis and decreased lung compliance, with acute respiratory distress. This is termed hyaline membrane disease and is the most common cause of respiratory distress in preterm infants. Meconium aspiration is more likely in full-term or near-term infants who experience fetal distress, and the x-ray may demonstrate hyperexpansion and irregular infiltrates. Spontaneous pneumothorax can occur at birth. Exam findings will include decreased breath sounds on the affected side, and x-ray findings should indicate the pneumothorax or pneumomediastinum. Pleural effusion may be present in hydropic infants, or it may be due to an underlying disorder or chylothorax. The x-ray would reveal opacity of the affected side, with blunting of the costophrenic recess. Asthma exacerbation should involve airway hyperresponsiveness in relation to a trigger exposure, and would demonstrate bilateral hyperinflation with diaphragm flattening on x-ray.
Q 67.9: What is the most appropriate management for the lesion shown, which is noticed on a 50-year-old female? A punch biopsy B imiquimod cream 3x week C excision with 1 cm margin D cryotherapy
The Correct Answer is: C *The lesion is asymmetric with irregular margins. The optimal treatment of this lesion would be excision with 1 cm margins. A punch biopsy would only be performed if excision cannot be performed.* Cryotherapy would destroy the lesion and prohibit a diagnosis and staging. (Wolff et al., 2009, Page 332)
Q 80.8: The source of most cases of epistaxis comes from what anatomic location? A Posterior nasal septum B Frontal sinuses C Anterior nasal septum D Anterior palate E Maxillary sinuses
The Correct Answer is: C 95 percent of epistaxis come from Kesselbach's plexus, which is a superficial, fragile group of arterioles and veins that are the most likely cause of nosebleeds. Five percent are posterior bleeds that originate along the sphenopalentine artery. (Tintinalli et al., 2011, Chapter 239)
Q 30.8: You are evaluating a 53-year-old post-menopausal patient during her routine annual examination in a primary care office. You notice that the patient has not been properly assessed for risk for osteoporosis, and does have some complaints that are of concern for osteoporosis. Based on this history, and the standard of care, what test is used to confirm osteoporosis in this patient? A Bone scan B CT scan C DEXA scan (dual energy x-ray absorptionmetry) D MRI scan E X-ray
The Correct Answer is: C A DEXA scan is the best test to determine bone density and is commonly used as a screening tool for those at risk of developing osteoporosis. This test is fast, reproducible and exposes patients to a relatively low dose of radiation. The results are reported as Z and T scores. Z scores indicate how the patients bone density compares to peers and T scores compare the patient to young, health individuals. Z and T scores are presented in the form of standard deviations below the comparison group. If the T score is 0 to greater than -1 the test is interpreted as normal. T scores of -1 to -2.5 are indicative of someone with osteopenia and a T score lower than -2.5 is consistent with osteoporosis. None of the other imaging options presented are ideal for determining bone density, but they each play important roles in other aspects of patient care. Bone scans are best suited for imaging studies designed to look for occult fractures, tumors, inflammatory or infectious processes within the bones, or metabolic bone diseases. CT is useful for a detailed examination of bone when looking for fractures or lesions and intra-articular pathology. MRI is helpful in the evaluation of spinal column pathology and various soft tissue injuries involving the muscles, tendons, ligaments, and cartilage. MRI is also useful in the evaluation of stress and occult fractures, osteomyelitis and early osteo necrosis. Radiographs (x-ray) are useful for most initial evaluation of musculoskeletal pathology, but are somewhat limited in their usefulness in visualizing some conditions. Low bone density can be perceived on x-ray, but only after the bone loss has been rather extensive. It is not a great early screening tool for osteoporosis.
Q 17.6: What types of connective tissue are injured in a sprain? A Bones and muscles B Fascia and joint capsules C Ligaments and joint capsules D Muscles and tendons E Tendons and bones
The Correct Answer is: C A sprain involves injury to those tissues that give support to joints - ligaments and joint capsules. Injury to muscles, tendons, and fascia would all be classified as a strain. Injuries to bone would be classified as a fracture.
Q 98.17: What types of connective tissue are injured in a sprain? A Bones and muscles B Fascia and joint capsules C Ligaments and joint capsules D Muscles and tendons E Tendons and bones
The Correct Answer is: C A sprain involves injury to those tissues that give support to joints - ligaments and joint capsules. Injury to muscles, tendons, and fascia would all be classified as a strain. Injuries to bone would be classified as a fracture.
Q 97.40: A 37-year-old woman who takes no medication and is otherwise healthy has developed a spontaneous "bloody discharge" from her left breast. Examination reveals no tenderness, masses, dimpling, or asymmetry. Gentle pressure at the margin of the areola reveals single duct involvement. This most likely represents which of the following? A Fibroadenoma B Fibrocystic changes C Intraductal papilloma D Malignancy E Pituitary adenoma
The Correct Answer is: C A unilateral serous or serosanguinous nipple discharge from a single duct is more likely a benign intraductal papilloma A less-likely intraductal malignancy (D), however, is possible and must be ruled out. Fibroadenomas (A) and fibrocystic changes (B) are not usually associated with nipple discharge. A pituitary adenoma (E) is usually associated with galactorrhea, rather than a bloody discharge, from multiple ducts in both breasts.
Q 38.11: The American Academy of Pediatrics (AAP) recommends which of the following treatments for a two-year-old child with an acute otitis media who has a fever of 103.7˚F, had an ear infection two months ago, and in whom you suspect penicillin-resistant strep bacteria? A Amoxicillin 45 mg/kg, divided into BID-dosing and administered for 10 days. B Amoxicillin 90 mg/kg, divided into BID-dosing and administered for 10 days. C Amoxicillin-clavulanate 45 to 90 mg/kg, divided into BID dosing and administered for 10 days D Cephalexin 20 mg/kg, divided into QID-dosing and administered for 10 days E Ceftriaxone 50 mg/kg, IM daily for three days.
The Correct Answer is: C AAP recommends consideration of amoxicillin as a first-line anti-infective for antibiotic naïve patients, and those at low risk. For patients who have received antibiotics in the previous 90 days, or those who have been exposed in day care or a medical facility, escalating antibiotics to cover penicillin-resistant streptococcus bacteria—such as amoxicillin-clavulanate—and a higher dose should be a consideration.
Q 32.2: A 64-year-old female presents with acute onset of severe right knee pain. She denies any trauma and her history is only significant for diabetes mellitus. On physical examination her knee has significant edema and erythema, with warmth on palpation. After an aspiration of the synovial fluid the results are: Color: opaque yellow fluid WBC: 90,000/mcL PMN: 90% Considering your suspected diagnosis what is the best treatment for this patient? A IV analgesics and observation B IV fluids C Hospitalization and IV antibiotics D PO antibiotics and follow-up in 3 days E MRI for further evaluation
The Correct Answer is: C Acute bacterial septic arthritis is associated with white blood cell counts commonly > 50,000/mcl with 90% or more polymorphonuclear cells. Compromised immunity, such as diabetes mellitus, increases the risk of septic arthritis. The effective treatment of septic arthritis requires appropriate antibiotic therapy together with drainage of the infected joint. Hospitalization is always necessary to prevent sepsis and administer IV antibiotics (C). IV analgesics (A) can be used but are not a necessary treatment option. IV fluids (B) may help but would not treat the septic joint. Oral antibiotics would not work as quickly and the patient should be monitored until the infection is controlled, sending them home on PO antibiotics would not be recommended (D). Imaging tests (E) generally add little to the diagnosis of septic arthritis.
A 54-year-old man with a history of chronic alcohol abuse presents to the emergency department with complaints of a subjective fever and severe epigastric pain radiating to the back. The pain has been present for the past 8 hours and is associated with nausea and vomiting, which has not relieved the pain. Laboratory data reveal a WBC of 14,000/mm 3 and a serum amylase of 500 U/L (reference range 0-286 U/L). Plain films of the abdomen were unremarkable. Which of the following is the most likely diagnosis? A Perforated duodenal ulcer B Acute cholecystitis C Acute pancreatitis D Mesenteric ischemia E Choledocholithiasis
The Correct Answer is: C Acute pancreatitis typically presents with severe, steady midepigastric abdominal pain that radiates through to the back; pain is associated with fever, nausea, and vomiting. The most common causes of acute pancreatitis are gallstones and alcohol. Laboratory studies will show elevated WBC and serum amylase levels. Amylase elevations are nonspecific and can be elevated with perforated ulcers and mesenteric ischemia. A perforated ulcer will show evidence of free air on plain film; mesenteric ischemia will not present with fever or an elevated WBC unless there is the presence of infarcted bowel at which point the patient would appear septic. Acute cholecystitis may be associated with elevations in amylase but they are typically only a modest increase.
Q 98.45: A 54-year-old man with a history of chronic alcohol abuse presents to the emergency department with complaints of a subjective fever and severe epigastric pain radiating to the back. The pain has been present for the past 8 hours and is associated with nausea and vomiting, which has not relieved the pain. Laboratory data reveal a WBC of 14,000/mm 3 and a serum amylase of 500 U/L (reference range 0-286 U/L). Plain films of the abdomen were unremarkable. Which of the following is the most likely diagnosis? A Perforated duodenal ulcer B Acute cholecystitis C Acute pancreatitis D Mesenteric ischemia E Choledocholithiasis
The Correct Answer is: C Acute pancreatitis typically presents with severe, steady midepigastric abdominal pain that radiates through to the back; pain is associated with fever, nausea, and vomiting. The most common causes of acute pancreatitis are gallstones and alcohol. Laboratory studies will show elevated WBC and serum amylase levels. Amylase elevations are nonspecific and can be elevated with perforated ulcers and mesenteric ischemia. A perforated ulcer will show evidence of free air on plain film; mesenteric ischemia will not present with fever or an elevated WBC unless there is the presence of infarcted bowel at which point the patient would appear septic. Acute cholecystitis may be associated with elevations in amylase but they are typically only a modest increase.
Q 120.14: A 54-year-old man with a history of chronic alcohol abuse presents to the emergency department with complaints of a subjective fever and severe epigastric pain radiating to the back. The pain has been present for the past 8 hours and is associated with nausea and vomiting, which has not relieved the pain. Laboratory data reveal a WBC of 14,000/mm 3 and a serum amylase of 500 U/L (reference range 0-286 U/L). Plain films of the abdomen were unremarkable. Which of the following is the most likely diagnosis? A Perforated duodenal ulcer B Acute cholecystitis C Acute pancreatitis D Mesenteric ischemia E Choledocholithiasis
The Correct Answer is: C Acute pancreatitis typically presents with severe, steady midepigastric abdominal pain that radiates through to the back; pain is associated with fever, nausea, and vomiting. The most common causes of acute pancreatitis are gallstones and alcohol. Laboratory studies will show elevated WBC and serum amylase levels. Amylase elevations are nonspecific and can be elevated with perforated ulcers and mesenteric ischemia. A perforated ulcer will show evidence of free air on plain film; mesenteric ischemia will not present with fever or an elevated WBC unless there is the presence of infarcted bowel at which point the patient would appear septic. Acute cholecystitis may be associated with elevations in amylase but they are typically only a modest increase. (Sharp, 2006, pp. 355-357) Sharp KW , Goldin SB , Lomis KD. Pancreas. In: Lawrence PF, ed. Essentials of General Surgery. 4th ed. Philadelphia, PA: Lippincott Williams & Wilkins; 2006.
Q 81.10: A physician assistant student suffers a needlestick injury while caring for an HIV-positive patient whose viral load is currently undetectable. Of the following, which is the most appropriate management for the student? A no drug treatment unless HIV testing performed immediately and at 6 weeks, 3 months, and 6 months results become positive B administration of zidovudine and lamivudine until results of baseline testing are received C administration of zidovudine and lamivudine for 4 weeks D administration of zidovudine, lamivudine, and indinavir for 4 weeks
The Correct Answer is: C After a needle-stick injury, a health-care worker should have baseline testing with follow-up testing at 6 weeks, 3 months, and 6 months. Risk of seroconversion is approximately 1:300. Administration of antiviral therapy decreases this risk by 79%, so the worker should be offered treatment with zidovudine and lamivudine as soon as possible after the injury, for a total of 4 weeks. However, workers with a high-risk exposure (source patient with advanced disease, a viral load >50,000, or with resistant organisms) should have a protease inhibitor added to the prophylactic regimen. (Katz and Zolopa, 2009, p. 1192) Katz MH , Zolopa AR. HIV infection. In: McPhee SJ , Papadakis MA, eds. Current Medical Diagnosis and Treatment. 48th ed. New York, NY: McGraw-Hill; 2009.
Q 107.5: A 29-year-old female has a long history of supraventricular tachycardia, for which she has been treated with long-term flecanide, as well as prior therapy with verapamil. She continues to have repeated episodes, sometimes two to three times a week, along with shortness of breath and at times hypotension that has been recorded. What is the next best therapy for this patient? A Synchronized cardioversion B Cardiac catheterization C Ablation therapy D Pacemaker insertion E Long-term telemetry monitoring
The Correct Answer is: C After exhaustion of non-invasive therapies, ablation therapy can be used to try to negate the aberrant pathway for SVT. Pacemakers will not allow for an override of the pathway, and cardioversion is only a temporary solution to an acute event. Implantable telemetry monitoring is only diagnostic and not therapeutic to treat. (Calkins H. Hurst's the Heart, Chapter 38, Supraventricular Tachycardia: AV Nodal Reentry and Wolff-Parkinson-White Syndrome)
Q 63.5: A 68-year-old female presents to the emergency department with signs and symptoms of an acute ischemic stroke. The initial CT scan is normal. Her blood pressure is 164/105. What is the most appropriate treatment for the blood pressure of this patient? A Atenolol PO B Clonidine PO C Close monitoring D Labetolol IV E Nicardipine IV
The Correct Answer is: C Aggressively lowering blood pressure may decrease blood flow to the ischemic tissue, thus decreasing the chances of recovery or increasing the risk of further infarction. In the setting of an acute ischemic stroke, blood pressure elevation should be monitored closely, with some elevation expected. This elevation is expected to decline without medication in the first few hours to days, but if elevation continues to a systolic blood pressure greater than 220mmHg, or mean arterial pressure greater than 120mmHg, medication is advised. Medications may include intravenous labetolol or nicardipine, with close monitoring of the patient. After the acute phase following a stroke, appropriate oral medications may be considered for outpatient hypertension management.
Q 119.15: You are asked to examine an 88-year-old female resident of a nursing home, who presents with a red eye. Her notes from the nursing home say that the patient has had this problem for six months, but now seems to be getting worse despite using daily artificial tears and occasional topical antibiotic drops. On physical exam you notice markedly injected conjunctiva to the right eye, with no discharge. The lower lid appears to be curled in toward the bulbar conjunctiva, with the eyelashes pointing inward. What is the name of this condition? A Conjunctivitis B Dacryoadenitis C Entropion D Ectropion E Endophthalmos
The Correct Answer is: C Aging causes a relaxation in the lower lid retractors, resulting in an entropion. This causes chronic irritation to the bulbar conjunctiva and corneal abrasions. Treatments include taping the lower lid to the cheek, botulinum toxin injection, or surgery. (Riordan et al., 2008, Chapter 3)
Q 35.7: A 5-year-old girl presents to the office with her mother, who states the child is experiencing excessive tearing, itching, and redness of her eyes. On physical exam you note marked injection with chemosis without discharge. The patient lacks adenopathy. Which of the following is the most appropriate treatment? A Azithromycin 1 gram by mouth B Observation and reassurance C Olopatadine 0.1% ophthalmic solution twice daily D Sulfacetamide 10% ophthalmic solution three times a day for 5 days E Valacyclovir 500 mg by mouth twice daily for 10 days
The Correct Answer is: C Allergic conjunctivitis is characterized by itching, tearing, redness, and chemosis, with itching being uncommon in other common forms of conjunctivitis and is treated with antihistamines/mast cell stabilizers (C). Azithromycin (A) is indicated for the treatment of conjunctivitis caused by chlamydia and antibiotic drops (D) are indicated for bacterial conjunctivitis. Observation and reassurance (B) is indicated for viral conjunctivitis. The patient lacks symptoms of herpes infection (E).
Q 36.6: A seven-week-old male infant is believed to have had a febrile seizure. The mother states that he has had rhinorrhea and fever x 2 days, and this afternoon he had a witnessed generalized tonic-clonic seizure for approximately two minutes. There is no history of seizures, trauma, new medications, or rash. A physical exam reveals an irritable infant with an elevated temperature of 102.4˚F, an erythematous bulging left tympanic membrane, and clear rhinorrhea with mucosal swelling. Which of the following is the most appropriate next management step for this infant? A Anticonvulsant medication B Electroencephalogram C Lumbar puncture D Skull x-rays E Watchful waiting
The Correct Answer is: C Although febrile seizures are common in the pediatric population, newborns and young infants with irritability, seizure activity, and fever should be evaluated for bacterial meningitis, according to the American Academy of Pediatrics. This requires a lumbar puncture. A single simple seizure, occurring in the presence of temperature greater than or equal to 100.4˚F (38˚C), lasting less than 15 minutes, and occurring in a child between six months and five years of age that is nontoxic appearing, does not require blood work, lumbar puncture, or imaging, but a source should be identified when possible. Anticonvulsant therapy is not indicated, as the medication risks outweigh the risks of another simple seizure and the primary target is to find and address the cause while addressing the fever. EEG is recommended for new onset afebrile seizures, or in the presence of recurrent seizures. Emergent neuroimaging with CT or MRI should be pursued if trauma is suspected.
Q 66.4: Which of the following conditions is the most common cause of massive lower gastrointestinal bleeding? A Hemorrhoids B Colon cancer C Diverticular disease D Upper gastrointestinal hemorrhage E Meckel diverticulum
The Correct Answer is: C Although hemorrhoids are a common cause of lower gastrointestinal hemorrhage, they do not cause massive hemorrhage. Massive lower gastrointestinal hemorrhage is most commonly due to diverticular disease. Upper gastrointestinal hemorrhage may present as massive lower hemorrhage due to the cathartic effect of blood. Colon cancer usually presents with occult bleeding. Meckel diverticulum may also present significant lower gastrointestinal hemorrhage, but it affects only approximately 2% of the population and is therefore not as common.
Q 98.12: Which of the following conditions is the most common cause of massive lower gastrointestinal bleeding? A Hemorrhoids B Colon cancer C Diverticular disease D Upper gastrointestinal hemorrhage E Meckel diverticulum
The Correct Answer is: C Although hemorrhoids are a common cause of lower gastrointestinal hemorrhage, they do not cause massive hemorrhage. Massive lower gastrointestinal hemorrhage is most commonly due to diverticular disease. Upper gastrointestinal hemorrhage may present as massive lower hemorrhage due to the cathartic effect of blood. Colon cancer usually presents with occult bleeding. Meckel diverticulum may also present significant lower gastrointestinal hemorrhage, but it affects only approximately 2% of the population and is therefore not as common.
Q 37.12: A 16-year-old girl is brought to the emergency department by ambulance after reportedly ingesting "a bottle of aspirin." Vital signs are temperature 37.8°C oral; pulse 94/min; respirations 30/min; blood pressure 100/68 mm Hg. What would you expect the blood gases to show that would confirm she had swallowed the aspirin? A anion gap metabolic acidosis with respiratory acidosis B nonanion gap metabolic acidosis with respiratory alkalosis C anion gap metabolic acidosis with respiratory alkalosis D nonanion gap metabolic acidosis with respiratory acidosis
The Correct Answer is: C An acute salicylate overdose (greater than 150 mg/kg) will produce symptoms of salicylate intoxication. Chronic salicylate intoxication occurs with ingestion of greater than 100 mg/kg/day for at least 2 days. Salicylates affect most organ systems, leading to various metabolic abnormalities. Because salicylates are a gastric irritant, symptoms of vomiting and diarrhea occur soon after the overdose, which may contribute to the development of dehydration. Salicylates stimulate the respiratory center leading to hyperventilation and hyperpnea resulting in respiratory alkalosis and compensatory alkaluria. A characteristic feature of salicylate intoxication is the coexistence of a respiratory alkalosis with a widened anion gap metabolic acidosis.
Q 10.3: In taking the family history of your 25-year-old male patient, you discover that he has numerous relatives with breast and ovarian cancers. In the past, his mother received genetic counseling and testing for the BRCA1 and BRCA2 gene mutations and was found to be positive for a mutant allele. What is his risk for developing this genetic cancer? A His chance is 0%, because this is not transmitted to men. B 25% C 50% D 75% E 100%
The Correct Answer is: C BRCA1 and BRCA2 gene mutations are expressed in the cells of breast and other tissue, where they help repair damaged DNA, or destroy cells if DNA cannot be repaired. If the BRCA1 or BRCA2 itself is damaged, damaged DNA is not repaired properly and this increases risks for certain cancers. These genes are inherited in an autosomal dominant manner so if his mother is positive (and his father is not), his risk is 50%, and he is also at increased risk for not only breast cancer, but also prostate, pancreatic, and other cancers.
Q 98.38: A patient presented with intermittent rectal bleeding associated with decreased caliber in the size of his stool. On examination, there were no palpable abdominal masses, but the fecal occult testing was positive. A barium enema was obtained with the results pictured below. What is the most likely diagnosis for this patient? A Diverticular disease B Crohn disease C Colorectal carcinoma D Intussusception
The Correct Answer is: C Barium enema finding of carcinoma of the sigmoid colon causing high-grade obstruction shows the classic "apple core" lesion. Crohn's disease is typically associated with the string sign, which is an area of stricture or stenosis that shows up as a narrow line of contrast, giving the appearance of a string associated with the stricture. Diverticular disease is associated with outpouchings from the colon that will be filled with barium.
Q 115.3: A patient presented with intermittent rectal bleeding associated with decreased caliber in the size of his stool. On examination, there were no palpable abdominal masses, but the fecal occult testing was positive. A barium enema was obtained with the results pictured below. What is the most likely diagnosis for this patient? (Reproduced, with permission, from Fauci AS, Braunwald E, Kasper DL, et al. Harrison's Principles of Internal Medicine, 17th ed. New York: McGraw-Hill, 2008:577.) A Diverticular disease B Crohn disease C Colorectal carcinoma D Intussusception
The Correct Answer is: C Barium enema finding of carcinoma of the sigmoid colon causing high-grade obstruction shows the classic "apple core" lesion. Crohn's disease is typically associated with the string sign, which is an area of stricture or stenosis that shows up as a narrow line of contrast, giving the appearance of a string associated with the stricture. Diverticular disease is associated with outpouchings from the colon that will be filled with barium. (Chang, 2006, p. 1111; Mellinger, 2006, p. 291; Dayton, 2006, p. 313) Chang AE , Morris AM. Colorectal cancer. In: Mulholland MW , Lillemoe KD , Doherty GM , Maier RV , Upchurch GR, eds. Greenfield's Surgery: Scientific Principles and Practice. 4th ed. Philadelphia, PA: Lippincott Williams & Wilkins; 2006. Mellinger JD , Macfadyen BV , Mercer DW , et al. Small intestine and appendix. In: Lawrence PF, ed. Essentials of General Surgery. 4th ed. Philadelphia, PA: Lippincott Williams & Wilkins; 2006. Dayton MT , Tradel JL. Colon, rectum, and anus. In: Lawrence PF, ed. Essentials of General Surgery. 4th ed. Philadelphia, PA: Lippincott Williams & Wilkins; 2006.
Q 18.8: A 64-year-old African American female presents to the clinic for evaluation of her hypertension, which she has had for several years. In the past she had been taking hydrochlorothiazide and lisinopril, with little effect on her blood pressure management. At today's visit, she has no complaints and feels well. Her vitals show T m 96.6, P 85, R 18, BP 191/99. She has no jugular venous distention seen on the neck exam, her lungs are clear, and cardiac exam has a regular rate and rhythm without murmur or gallop. Her abdomen is soft, non-tender, and a bruit is appreciated at the mid-abdomen just a few centimeters below the epigastric region. There is no fullness or enlargement of the abdominal aorta on palpation. Based on the history and clinical findings, what is the most appropriate next test for this patient? A Computed tomography of the abdomen B Nuclear exercise stress test C Renal duplex ultrasound D Echocardiogram E Electrocardiogram
The Correct Answer is: C Based on the information of the presentation, the patient most likely has renal artery stenosis, which is causing uncontrolled hypertension. The most appropriate test for confirmation of this is a renal duplex ultrasound (C). This test will have a higher specificity and sensitivity than the other listed tests (A, B, D, and E), as well as determine flow.
Q 120.20: You are evaluating a patient who is complaining of facial drooping , and inability to close his eye. During the cranial nerve exam you notice he is unable to wrinkle his forehead. Based on this information what is the most likely diagnosis? A Cerebrovascular accident B Transient ischemic attack C Bell's palsy D Horner's syndrome E Isolated oculomotor palsy
The Correct Answer is: C Bell's palsy affects cranial nerve VII, the facial paralysis conforms to the all branches of the peripheral nerve including the side of the face, eyelid and forehead muscles. An acute cerebrovascular accident would present only with a facial droop, the ability to close the eye and wrinkle the forehead would be preserved and there would likely be other focal weakness on physical exam. Horner's syndrome is miosis, ptosis and facial flushing and anhydrosis caused by abnormalities of the supercervical ganglion along the internal carotid artery. (Riordan et al., 2008, Chapters 65, 68)
Q 20.2: Which condition is suggested by urethritis, arthritis, and conjunctivitis? A chlamydial infection B gonococcal infection C reactive arthritis D tertiary syphilis
The Correct Answer is: C Both chlamydia and gonorrhea infections can result in urethritis. Gonococci can disseminate to the joints and cause septic arthritis. Chlamydia is typically asymptomatic but can cause chronic conjunctivitis in adolescents and young adults. Reactive arthritis (also known as Reiter syndrome) is a result of an untreated chlamydia infection, and although typically characterized, in texts, by the triad of urethritis, arthritis, and conjunctivitis, all of the symptoms may not be present or not identified at the time of presentation. Tertiary syphilis is characterized by neurologic and cardiovascular disease, gumma, auditory and ophthalmic involvement, and cutaneous lesions.
Q 97.105: Which condition is suggested by urethritis, arthritis, and conjunctivitis? A chlamydial infection B gonococcal infection C reactive arthritis D tertiary syphilis
The Correct Answer is: C Both chlamydia and gonorrhea infections can result in urethritis. Gonococci can disseminate to the joints and cause septic arthritis. Chlamydia is typically asymptomatic but can cause chronic conjunctivitis in adolescents and young adults. Reactive arthritis (also known as Reiter syndrome) is a result of an untreated chlamydia infection, and although typically characterized, in texts, by the triad of urethritis, arthritis, and conjunctivitis, all of the symptoms may not be present or not identified at the time of presentation. Tertiary syphilis is characterized by neurologic and cardiovascular disease, gumma, auditory and ophthalmic involvement, and cutaneous lesions.
An 8-month-old female is diagnosed with respiratory syncytial virus bronchiolitis while in the emergency department. Which of the following strongly indicates a need for admission to the hospital and continued monitoring? A Age of 8 months B Birth at 38 weeks C Feeding difficulty with decreased oxygen saturation D Oxygen saturation of 96% E Respiratory rate of 45 bpm
The Correct Answer is: C Brochiolitis patients must be considered at risk of developing severe disease and/or apnea when certain criteria are present, thus requiring admission. This includes, but may not be limited to, the following: birth <37 weeks gestation, age <12 weeks, witnessed apnea, underlying cardiopulmonary disease, immunodeficiency, tachypnea based on expected respiratory rate per age, decreased oral intake or feeding difficulty with associated decreased oxygen saturation, decreased oxygen saturation with varying ranges based on source (most being <95%), a history of previous intubation, and a caregiver ability to adequately provide care and monitoring.
Q 49.7: An 8-month-old female is diagnosed with respiratory syncytial virus bronchiolitis while in the emergency department. Which of the following strongly indicates a need for admission to the hospital and continued monitoring? A Age of 8 months B Birth at 38 weeks C Feeding difficulty with decreased oxygen saturation D Oxygen saturation of 96% E Respiratory rate of 45bpm
The Correct Answer is: C Brochiolitis patients must be considered at risk of developing severe disease and/or apnea when certain criteria are present, thus requiring admission. This includes, but may not be limited to, the following: birth <37 weeks gestation, age <12 weeks, witnessed apnea, underlying cardiopulmonary disease, immunodeficiency, tachypnea based on expected respiratory rate per age, decreased oral intake or feeding difficulty with associated decreased oxygen saturation, decreased oxygen saturation with varying ranges based on source (most being <95%), a history of previous intubation, and a caregiver ability to adequately provide care and monitoring.
Q 110.16: During a spirometry test, a patient is asked to forcibly expel as much air from the lungs as possible. Which of the following represents the amount of air that remains in the patient's lungs following this maximal forced expiration? A expiratory reserve volume B functional residual capacity C residual volume D tidal volume E vital capacity
The Correct Answer is: C By definition, residual volume is the volume of air remaining in the lungs after a maximal forced expiration. The residual volume is important physiologically, as it prevents total collapse of the alveoli and minimizes the pressure and energy required to inflate the lungs during inspiration. (Boulpaep, 2009, pp. 625-626; Costanzo, 2006, pp. 185-187) Boulpaep EL. The microcirculation. In: Boron WF , Boulpaep EL, eds. Medical Physiology. 2nd ed. Philadelphia, PA: Saunders Elsevier; 2009. Costanzo LS. Physiology. 3rd ed. Philadelphia, PA: Saunders Elsevier; 2006.
Q 75.5: Which one of the following is a characteristic finding on computed tomography (CT) of the abdomen in a patient with acute diverticulitis? A toxic megacolon B air-fluid levels C soft tissue inflammation of the pericolic fat D thinning of the colon wall E paucity of bowel gas in the colon
The Correct Answer is: C CT findings consistent with diverticulitis include soft tissue thickening of the pericolic fat (98%), diverticula, and thickening of the bowel wall. In immunosuppressed patients, findings may include intraperitoneal and extraperitoneal gases without fluid or abscess formation. (Travis, 2009, p. 249) Travis AC , Blumberg RS. Diverticular disease of the colon. In: Greenberger NJ, eds. Current Diagnosis & Treatment: Gastroenterology, Hepatology, & Endoscopy. New York, NY: McGraw-Hill; 2009.
Q 73.7: A 37-year-old male presents to your office with a history of vision loss in his right eye. He denies any pain, and states that the vision loss occurred suddenly. He noted there was a wavy, "curtain-like" visual disturbance preceding the vision loss. Upon physical exam you notice a cherry red spot over the macula and retinal pallor. What is the most likely diagnosis? A Macular degeneration B Retinal detachment C Central retinal artery occlusion D Cerebrovascular accident E Central retinal vein occlusion
The Correct Answer is: C Central retinal artery occlusion is characterized by a sudden, painless vision loss. A cherry red spot is characteristic on the macula, along with pallor to the retina. (Tintinalli et al., 2011, Chapter 236)
Q 97.39: A 30-year-old woman and her husband have been trying unsuccessfully to become pregnant for the past year. Over-the-counter ovulation tests have indicated that she is ovulating. As part of her evaluation she undergoes a hysterosalpingogram that reveals tubal scarring. Which of the following is the most likely explanation for this finding? A Congenital anomaly of the tubes B Diethylstilbestrol exposure C Past asymptomatic chlamydial infection D Previous abdominal surgery E Scarring from prior uterine instrumentation
The Correct Answer is: C Chlamydial infection can cause "silent" pelvic inflammatory disease, leading to scarring with subsequent tubal obstruction that can cause infertility or ectopic pregnancy. Congenital anomalies of the tubes (A) may also contribute to infertility, but is not associated with scarring. DES exposure (B) in utero may lead to reproductive-system anomalies in offspring. Past abdominal surgery (D) may lead to adhesions that inhibit fertility. Uterine instrumentation (E) may lead to intrauterine synechiae.
Q 34.2: A 2-week-old male infant is being seen in the clinic for a profuse mucoid discharge from both eyes, with some associated tearing. On examination, you notice both eyes are hyperemic and the eyelids are red and swollen. Which of the following is the most likely cause of this patient's ophthalmia neonatorum (conjunctivitis in the newborn)? A allergic B gonococcal C chlamydial D viral
The Correct Answer is: C Chlamydial infections are the most common cause of conjunctivitis in newborns in developed countries. Other causes of ophthalmia neonatorum include reactions to silver nitrate prophylaxis, other bacterial infections such as gonococcal or staphylococcal, or viral organisms such as adenovirus or echovirus. Chlamydia trachomatis causes conjunctivitis and pneumonia in neonates. Treatment for chlamydial conjunctivitis should be with systemic erythromycin to treat the conjunctivitis and as prophylaxis against pneumonia.
Q 106.1: A 55-year-old woman with a history of emphysema, who is undergoing chemotherapy for lung cancer, is sent to see you by her oncologist regarding a sudden increase in dyspnea, with exertion and fatigue. Which of the following physical exam findings would predict a cardiac etiology for her dyspnea instead of a pulmonary etiology? A Crackles at the lung bases bilaterally B Tachycardia C Pulsus paradoxus D Soft S1, S2 E Wheezing
The Correct Answer is: C Choice C is the best choice, as pulsus paradoxus is frequently seen in patients with cardiac tamponade. Patients diagnosed with cancer, particularly of the lung and breast, may accumulate fluid within the pericardial sac, leading to cardiac tamponade. The finding of pulsus paradoxus is defined as a decrease in systolic arterial pressure of greater than 10 mmHg. It is an accentuation of the normal decrease in systolic arterial pressure of less than 10mm Hg that normally accompanies inspiration. Symptoms of dyspnea and fatigue in a patient with lung cancer and emphysema are not uncommon. The other choices are nonspecific to a cardiac etiology. Choice A, crackles at the bases, could be in pulmonary etiology. Choice B, tachycardia, is also nonspecific, and can be found in patients who are febrile, or with lung disease. Choice D, a soft S1, S2, may be found in patients with emphysema and an increased AP diameter. Choice E is likely pulmonary. (LeBlond et al., 2009, Chapter 8)
Q 68.10: A 70-year-old man with a history of congestive heart failure, CABG x 2, HTN, and diabetes mellitus, type 2, presents to the office with complaints of increasing dyspnea while walking across the room, 3 pillow orthopnea, and worsening lower extremity edema since he returned from a cruise. He admits to taking all of his medications regularly, including lisinopril 20 mg daily, carvedilol 12.5 mg twice daily, spironolactone 25 mg daily, furosemide 20 mg daily, and aspirin 81 mg daily. On physical exam, his vital signs are stable, although he has gained 10 lbs compared to his last office visit three months ago. He is also demonstrating jugular venous distention to the angle of the jaw, at 90 degrees of truncal elevation. There are crackles at the lung bases bilaterally, and 1+ pitting edemas to the knees bilaterally. His EKG is unchanged from previously (see Figure 7). Which of the following is the most likely etiology for this patient's acute exacerbation of congestive heart failure? A Labile hypertension B Acute myocardial infarction C Dietary noncompliance D Acute chordae tendinae rupture E Severe anemia
The Correct Answer is: C Choice C is the most likely etiology for this patient's acute exacerbation of congestive heart failure. Although choices A, B, D, and E are all possible causes, a more common, though mundane cause of acute exacerbation of congestive heart failure, is dietary noncompliance, especially related to sodium restriction. Patients with congestive heart failure should maintain low sodium diets, which is difficult in everyday settings, and nearly impossible aboard a cruise ship while on vacation. There is no evidence to support labile hypertension as the cause of this patient's symptoms, as his blood pressure is currently well-controlled. Choice B is also less likely than choice C, given that the patient is not demonstrating chest discomfort and no EKG changes are noted. Choice d is less likely than choice C, as on physical exam there is no new murmur noted. Choice E is less likely than choice C in this patient, as no history or physical exam findings are noted that would raise the suspicion for severe anemia. (McPhee et al., 2011, Chapter 10)
A 33-year-old IV drug user presents to the emergency department with chills, diaphoresis, anorexia, and malaise. On physical exam, her temperature is 40°C, BP 98/55, P 115 bpm, and RR 22. Two separate blood cultures are positive for S.aureus. Which of the following physical exam findings would confirm a clinical diagnosis of infective endocarditis, according to the Duke criteria? A Increase in valvular regurgitation B Irregularly irregularly pulse C Osler's nodes D Buccal hemmorhages E Koplik spots
The Correct Answer is: C Choice C, Osler's nodes, confirms the clinical diagnosis of infective endocarditis, as it is a minor criteria. The Duke criteria for the clinical diagnosis of infective endocarditis requires the documentation of two major criteria, or one major criteria and three minor criteria, or five minor criteria. The patient demonstrates the presence of one major criteria (two separate blood cultures with typical microorganisms for infective endocarditis) and two minor criteria (fever greater than 38.0°C and predisposing condition of IV drug use). Only a new valvular regurgitation, not an increase or change in preexisting murmur, is considered sufficient to qualify as a major criteria, so choice A is incorrect. An irregularly irregular pulse, choice B, is commonly seen in patients with atrial fibrillation, not with infective endocarditis. Choice D, conjunctival hemorrhages, not buccal hemorrhages, are one of the minor criteria. Choice E, Koplik spots, are buccal lesions seen in patients infected with measles, whereas the presence of Roth's spots does fulfill one of the minor criteria.
A 55-year-old woman with a history of emphysema, who is undergoing chemotherapy for lung cancer, comes to the emergency department complaining of a sudden increase in dyspnea, with exertion and fatigue. On physical exam, hypotension, pulsus paradoxus, and muffled heart sounds are noted. On transthoracic echocardiography, cardiac tamponade is noted, with over 200 mL of pericardial fluid described. Which of the following would be this patient's most likely electrocardiographic finding (Figure 7)? A Torsades de pointes B U waves C Electrical alternans with sinus tachycardia D Peaked T waves E Convex elevation of the J point
The Correct Answer is: C Choice C, electrical alternans with sinus tachycardia, a beat-to-beat alteration in one or more components of the ECG signal, is considered a specific sign of pericardial effusion, often with cardiac tamponade, as it represents the periodic swinging motion of the heart in the effusion at a frequency that is ½ the heart rate. Choice A, torsades de pointes, is a type of ventricular tachycardia frequently seen, and is associated with electrolyte disturbances or the use of certain types of antiarrhythmic drugs. Choice B, U waves, are associated with hypokalemia. Choice D is frequently noted with severe hyperkalemia. Choice E, convex elevation of the J point, is seen in patients suffering from hypothermia.
Q 119.14: A 57-year-old man with a history of HTN, hyperlipidemia, and chronic tobacco use presents to the office with complaints of chest tightness that occurs every time he begins raking leaves. If he stops and rests, it is relieved within 5 minutes. He has no associated nausea or diaphoresis, but does admit to associated dyspnea. Which of the following is the most likely diagnosis? A Pericarditis B Acute myocardial infarction C Stable angina pectoris D Prinzmetal angina E Myocarditis
The Correct Answer is: C Choice C, stable angina pectoris, is chest or arm discomfort that is reliably precipitated by activity and/or emotional distress, and relieved with rest or sublingual nitroglycerin. Choice A, pericarditis, would present with chest discomfort that is worse while supine and improves while sitting up, as well as a pericardial friction rub. Choice B, acute myocardial infarction, requires troponin elevation to establish the diagnosis. Choice D, prinzmetal angina, or variant angina pectoris, is defined as coronary artery spasm associated with ST-segment elevation, usually occurring at rest, and frequently at the same time of the day. Choice E, myocarditis, is usually preceded by a viral prodrome. (Fauci et al., 2008, Chapter 238)
Q 66.7: A 57-year-old woman with a history of rheumatic fever is seen complaining of dyspnea while vacuuming her apartment, which has been worsening over the last few months. On physical exam, jugular venous distension is appreciated. Auscultation of the chest reveals a possible opening snap, loud S 1 , and a very soft diastolic rumbling murmur is auscultated at the left lower sternal border. When the patient is placed in the left lateral decubitus position, the murmur is accentuated, and heard best at the apex. Both hepatomegaly and splenomegaly are noted. On EKG, no evidence of right ventricular hypertrophy is noted, despite the obvious signs and symptoms of right heart failure. Which of the following valvulopathies should be suspected given this patient's history and physical exam findings? A Mitral stenosis and aortic stenosis B Mitral stenosis and aortic regurgitation C Mitral stenosis and tricuspid stenosis D Aortic stenosis and mitral regurgitation E Mitral stenosis and pulmonic stenosis
The Correct Answer is: C Choice C, tricuspid stenosis, is frequently seen in association with mitral valve stenosis in patients with a history of rheumatic heart disease (between 5% and 10% of the time). Both murmurs are similar in character, with the main difference that the murmur of tricuspid stenosis is heard best at the left lower sternal border, and the murmur of mitral stenosis is heard best in the left lateral decubitus position with the bell at the apex. Because they are similar in nature, a high level of suspicion for tricuspid stenosis should be maintained, so that tricuspid stenosis is not overlooked. Tricuspid regurgitation is also frequently seen in association with tricuspid stenosis. Patients frequently present with signs and symptoms of right-sided heart failure, including hepatic congestion and splenomegaly. As tricuspid stenosis and regurgitation are present, right ventricular hypertrophy may not occur, and thus would not be demonstrated on EKG. Choices A, B, D, and E are not seen as frequently as tricuspid stenosis in association with rheumatic heart disease and mitral valve stenosis. (Fauci et al., 2008, Chapter 230)
Q 120.15: A 23-year-old female presents with a history of rigid thought patterns and a need for control. She sees herself as a perfectionist. She discloses that she feels a need to check the locks on her doors at home once every 30 minutes. She is consumed with these thoughts about locking the doors. What is the most likely classification for her personality disorder? A Histrionic B Narcissistic C Obsessive-compulsive D Paranoid E Schizotypal
The Correct Answer is: C Clinical findings of obsessive-compulsive disorder include being a perfectionist, egocentric, and indecisive, with rigid thought patterns and need for control. Clinical findings of histrionic personality disorder include being dependent, immature, seductive, egocentric, vain, and emotionally labile. Narcissistic personality disorder presents with the clinical findings of grandiosity, a preoccupation with power, lacking interest in others, and excessive demands for attention. Clinical findings of someone who has paranoid personality disorder would include defensiveness, being overly sensitive, secretive, suspicious, hyper-alert, and with a limited emotional response. Schizotypal clinical findings include being superstitious, socially isolated, and suspicious, and having limited personality ability, odd speech, and eccentric behaviors. (McPhee SJ, Papadakis MA. Current Medical Diagnosis & Treatment, 2010, p. 951)
Q 38.16: A 1-day-old infant being examined in the newborn nursery is noted to have a central, 4 mm cataract affecting his right eye. Which of the following diagnostic studies should be performed as a result of this finding? A Fasting blood glucose B MRI of the eye and orbit C Rapid plasma reagin (RPR) D Serum cortisol level E Serum thyroid stimulating hormone level
The Correct Answer is: C Congenital cataracts may result from transmission of maternal infections such as herpes simplex virus, cytomegalovirus, toxoplasmosis, or syphilis and require further evaluation for potential systemic infection. A quantitative RPR (C) should be performed to assess for congenital syphilis. Endocrine disorders such as diabetes (A), Cushing's syndrome (D), or hypothyroidism (E) aren't common causes of congenital cataracts.
Q 8.6: A 45-year-old male comes into your family practice office for his second follow-up appointment since being diagnosed with type 2 diabetes. He does not complain of any symptoms. He is currently taking metformin 1000 mg BID and his HgA 1c at this visit 7.6%. You ask him about his diet and if he is regularly monitoring his blood glucose. He says that he does not really watch what he eats, but he does check his glucose levels daily. He tells you that on average his fasting plasma glucose (FPG) is usually around 88 mg/dl and his postprandial glucose (PPG) is around 180 mg/dl. What would be the BEST next appropriate step in management for this patient? A Nothing, his levels are within normal limits. B Consider switching to basal insulin therapy to control his FPG. C Add a dipeptidyl peptidase 4 (DPP-4) inhibitor to control his PPG. D Add a sulfonylurea to control his PPG. E Increase his metformin to 1500 mg twice daily.
The Correct Answer is: C DPP-4 inhibitors modulated glucagon-like peptide-1 (GLP-1). Their mechanism of action is thought to result from increased incretin levels, especially GLP-1. GLP-1 inhibits glucagon release which, in turn, results in increased insulin secretion, delays gastric emptying, and decreases serum glucose levels. The class is particularly appropriate to utilize in patients who have near-normal HbA 1C and elevated postprandial serum glucose because they work only when food enters the gut and have little to any chance of hypoglycemia.
Q 48.2: While seeing a 12-week-old baby girl for her well-child checkup, it is noticed that she has tearing from her left eye. There is a small reddened area that is swollen and she cries when it is touched. The swollen area is just below the medial inferior eyelid. There is also constant tearing from this same eye. Her mother says it just started about 2 days ago and is getting worse. What is the most likely cause of this problem? A blepharitis B conjunctivitis C dacryocystitis D anterior uveitis
The Correct Answer is: C Dacryocystitis, whether acute or chronic, is usually secondary to bacterial infections. It presents as an acutely inflamed swelling and tender area over the lacrimal sac just medial and inferior to the inner canthus of the eye. Because the lacrimal sac is inflamed and blocked there is tearing and usually purulent discharge from the eye. There may also be an orbital cellulitis. Treatment consists of oral and topical antibiotics and warm compresses, and surgical drainage may also be indicated. After the acute episode and for chronic cases, surgical correction of the nasolacrimal obstruction is required. Anterior uveitis typically presents with pain, photophobia, blurred vision, and injection without exudates. Blepharitis is an inflammation of the lid margin that presents with crusty debris along the lashes. Unless there is a concomitant conjunctival infection, there is typically no injection noted.
Q 97.31: A 25-year-old woman brings in her menstrual calendar as part of a preconceptional counseling visit. Her cycles are regular, occurring every 30 days and lasting 3-4 days. She has mild cramping on days 1 and 2 that is easily relieved by ibuprofen or acetaminophen. On what day of her cycle is she most likely ovulating? A 12 B 14 C 16 D 18 E 20
The Correct Answer is: C Day 1 of menses is the start of a new menstrual cycle. In normally menstruating women, the luteal phase is stable at 14 days, i.e., ovulation ordinarily occurs 14 days before the onset of the next menses. Therefore, in a woman with a very regular 30-day cycle it is most likely to occur on day 16. She may, however, become pregnant if she times intercourse for two days before (B) or after (D) the day of ovulation, due to other factors such as the duration of activity of sperm. Days 12 (A) and 20 (E) are at the margins of the period of fertility, but neither is the day of likely ovulation.
Q 36.1: Which of the following daily maintenance fluid requirements is the closest approximation for a 24-kg child who is refusing to eat? A 1,080 mL B 1,200 mL C 1,580 mL D 2,000 mL
The Correct Answer is: C Dehydration is a common pathophysiologic alteration in fluid balance in children. The body has a maintenance fluid requirement to replace daily normal losses that occur through the skin, kidney, intestines, and respiratory tract. The following formula can be used to calculate the usual amount of fluid a healthy child requires by mouth to maintain hydration: 100 mL/kg for the first 10 kg of body weight 50 mL/kg for the next 10 kg of body weight 20 mL/kg for the weights above 20 kg For this question, a 24-kg child would require: 100 mL/kg × 10 kg = 1,000 mL for the first 10 kg 50 mL/kg × 10 kg = 500 mL for the next 10 kg 20 mL/kg × 4 kg = 80 mL for the next 4 kg Total = 1,580 mL 24 kg
Q 74.3: A 45-year-old man with a history of NSTEMI, CABG X 3, HTN, and hyperlipidemia presents to your office with complaints of progressive dyspnea over the last three weeks, to the point that he is now dyspneic while walking across the room. In the last few days, he has noticed bilateral lower extremity edema. Which of the following findings on physical exam would meet the criteria for a diagnosis of congestive heart failure, according to the modified Framingham clinical criteria for the diagnosis of heart failure? A Nocturnal cough B Tachycardia C Third heart sound D Pleural effusion E Hepatomegaly
The Correct Answer is: C Diagnosis of heart failure requires that the findings of two major criteria, or one major and two minor criteria, cannot be attributed to another medical condition. The patient demonstrates two minor criteria: bilateral lower extremity edema and dyspnea on ordinary exertion. Only choice C (third heart sound) falls under the heading of major criteria. Choice A, B, D, and E all fall under the heading of minor criteria. A third heart sound, or S3, is representative of early rapid filling of the left ventricle, and can occur in any condition in which there is an increased left ventricular volume, such as congestive heart failure. (Senni et al., 1998, 98:2282)
Q 32.20: A 62-year-old male diabetic patient presents to your family practice office with a fever and cough, and after a physical examination you perform a chest x-ray, which reveals a right middle lobe pneumonia. You prescribe levofloxacin 750 mg PO for five days and give the patient his first dose in the office prior to sending him home. Within five minutes, he develops generalized urticaria, shortness of breath with a pronounced wheeze, and becomes hypotensive. While your staff calls 911, what is the most important agent to administer first? A cimetidine B dipenhydramine C epinephrine D methylprednisolone E none of the above
The Correct Answer is: C Early recognition of an anaphylactic reaction is mandatory, since death occurs within minutes to hours after the first symptoms. Mild symptoms such as pruritus and urticaria can be controlled by administration of 0.3 to 0.5 mL of 1:1000 (1.0 mg/mL) epinephrine SC or IM, with repeated doses as required at 5- to 20-minute intervals for a severe reaction.
Q 112.3: A 48-year-old African American male presents with dyspnea, 2-pillow orthopnea, and swelling to his lower legs that has developed over the last month. He also complains of fatigue and decreased exercise tolerance, stating that he has trouble climbing one flight of steps. On physical examination, his blood pressure is 178/98, pulse rate is 102, and respiratory rate is 20. There is 5 cm JVD, crackles at the bilateral lung fields, and tachycardia and an S 3 is heard on cardiac auscultation. There is 2+ pitting edema to the lower extremities. His electrocardiogram reveals a sinus tachycardia at a rate of 105 and left ventricular hypertrophy. The chest x-ray reveals cardiomegaly with increased interstitial markings in all lung fields. There is a small right pleural effusion that blunts the costophrenic angle. Which initial diagnostic test gives you the best information regarding this patient's pathology? A Exercise stress test B Pulmonary function test C Echocardiogram D Ventilation/perfusion scan E CT scan of chest
The Correct Answer is: C Echocardiogram is indicated in determining the extent of the cardiomyopathy. This test will examine aspects of wall motion and valvular competency, as well as estimate an ejection fraction. Exercise stress tests are utilized more for evaluation of ischemia than function of the myocardium. (Bashore et al., Current Medical Diagnosis and Treatment, Chapter 10)
Q 45.8: The eggs of this parasite are detected by microscopic examination of clear adhesive tape that has been pressed to the child's anus in the morning, prior to bathing. What parasite is most likely to be identified by this test method? A Ancylostoma duodenale (hookworm) B Ascaris lumbricoides (ascaris) C enterobiasis (pinworm) D trichuriasis (whipworm)
The Correct Answer is: C Enterobiasis or pinworms is a worldwide infection that affects people of all ages and socioeconomic levels. It especially affects children. The classic manifestation of this problem is nocturnal anal pruritis and sleeplessness. The sleeplessness may be secondary to the migration of female worms to the perianal area to lay eggs, during which the tape may pick up the larvae. Transmission of the worms occurs when children ingest the eggs that are present on their hands (from scratching), in the bedclothes, or in house dust. After hatching in the stomach, the larvae migrate to the cecum where they mature into adults. The treatment of choice for pinworms is pyrantel pamoate or mebendazole. Albendazole may also be used. For eradication of this parasite, often the entire family must be treated at once. Ascaris is a helminthiasis infection that is ingested and excreted in the stool. Diagnosis is made by stool examination for the characteristic eggs. Hookworms are found in warm, damp soil and penetrate the skin. From there the infection can spread to the lungs where they ascend into the trachea to be swallowed and live in the intestine. Diagnosis is made by stool examination for the eggs. Whipworm is ingested from the soil and lives in the intestine; detection is also made by egg in the feces.
Q 38.7: A 2-day-old infant presents with numerous red macules with central vesicles and pustules. The rash spares only the palms and soles. The infant has no fever and is nursing normally. What is the most likely diagnosis? A Acne neonatorum B Congenital herpes simplex C Erythema toxicum D Milia
The Correct Answer is: C Erythema toxicum is a benign rash seen in newborns. The cause of the rash is unknown and resolves spontaneously. The rash appears as erythematous macules, which may develop central vesicles within 24 to 48 hours. The palms and soles are spared.
Q 110.5: A 36-year-old male returns to the office for follow up of his heartburn symptoms. He continues to have heartburn three to four times a week, even though he has been compliant with his proton pump inhibitor medication since it was prescribed three months ago. He underwent an endoscopy three weeks ago, which was normal. He denies dysphagia, early satiety, or weight loss. Which of the following tests is the most appropriate next step in evaluating this patient? A Repeat endoscopy B Barium esophagography C Esophageal manometry D Abdominal CT scan E Urea breath test
The Correct Answer is: C Esophageal monometry is the answer, and this is useful in patients who have persistent symptoms, despite PPI therapy and individuals who have had normal findings on endoscopy. There is no reason to repeat the endoscopy, as the first one was normal. Barium esophagography plays a limited role in GERD, and could be helpful if the patient complained of dysphagia. Abdominal CT is not indicated in this case. Urea breath test is used in the detection of H. pylori, which is an uncommon cause of dyspepsia in the absence of peptic ulcer disease. (McPhee SJ, Papadakis MA. Current Medical Diagnosis & Treatment, 2010, p. 531)
Q 49.13: A 20-month-old boy is brought into the emergency department by his parents. They state he has not been feeling well for 2 days and this morning noted he was "shaking all over" and was not responding to commands. This went on for less than 10 minutes and has never happened before. His current rectal temperature is 100.7°F. The seizures are characteristic of A absence seizures B Lennox-Gastaut syndrome C febrile seizures D infantile spasms E juvenile myoclonic epilepsy
The Correct Answer is: C Febrile seizures can occur in children younger than 5 years when accompanied by a fever. They are characterized by a brief generalized motor seizure. Absence seizures are generalized seizures characterized by a loss of consciousness without motor involvement, typically seen in older children. Lenox-Gastaut syndrome presents in childhood as well but is usually associated with developmental delay and seizures of akinetic and myoclonic nature (referred to as drop attacks). Infantile spasms occur without relation to systemic illness and are massive myoclonic events with bending at the waist. Juvenile myoclonic epilepsy evolves in the teenage years and is characterized by repeated episodes of myoclonic seizure activity.
Q 48.7: A 5-year-old child with no known drug allergies is diagnosed in your clinic with bilateral acute otitis media. Which of the following is the drug of choice? A levofloxacin B nitrofurantoin C amoxicillin D doxycycline E gentamicin
The Correct Answer is: C First choice antibiotic treatment for acute otitis media includes a 10-day course of amoxicillin (80 to 90 mg/kg/day in two divided doses) or a combination of erythromycin (50 mg/kg/day) and a sulfonamide (150 mg/kg/day). Reasons for amoxicillin therapy include spectrum of activity including both susceptible and intermediate resistant S pneumoniae, safety, cost, and tolerability.
Q 29.1: A 23-year-old female presents to the clinic for evaluation of a pre-employment physical examination with evidence of a first-degree AV block on ECG. She is otherwise healthy and without any medical history, is not on any medications, and is symptom free. Based on this history, what is the treatment for this patient? A Beta-blockers B Definitive electrophysiology study C No treatment, only monitoring D Ablation therapy E Calcium channel blockers
The Correct Answer is: C First-degree AV block needs no immediate therapy. The treatment is to monitor the patient for any changes that may occur if a new disease presents itself.
A 7-year-old is diagnosed with an acute case of hematogenous osteomyelitis accompanied with fever and leukocytosis. Based on your knowledge of the disease, which bone is most likely to present with the infection? A Feet B Hands C Long bones D Pelvis E Vertebrae
The Correct Answer is: C Fortunately, hematogenous osteomyelitis is not common in children, but when it does occur it primarily is found in the long bones. The femur, tibia and humerus are the most typical locations for osteomyelitis in children. The highly vascular metaphysis of long bones contribute to the potential for hematogenous spread of the implicated pathogen. Osteomyelitis can occur at any of the locations mentioned in the answer choices given, but at a significantly lower rate than in the long bones. The rate of occurrence at several selected locations is given below: Feet - 9% Femur 25% Hands - 6% Humerus - 13% Pelvis - 8% Radius/ulna - 6% Tibia/fibula - 28% Vertebrae - 2%
Q 26.9: A 7-year-old is diagnosed with an acute case of hematogenous osteomyelitis accompanied with fever and leukocytosis. Based on your knowledge of the disease, which bone is most likely to present with the infection? A Feet B Hands C Long bones D Pelvis E Vertebrae
The Correct Answer is: C Fortunately, hematogenous osteomyelitis is not common in children, but when it does occur it primarily is found in the long bones. The femur, tibia and humerus are the most typical locations for osteomyelitis in children. The highly vascular metaphysis of long bones contribute to the potential for hematogenous spread of the implicated pathogen. Osteomyelitis can occur at any of the locations mentioned in the answer choices given, but at a significantly lower rate than in the long bones. The rate of occurrence at several selected locations is given below: Feet - 9% Femur 25% Hands - 6% Humerus - 13% Pelvis - 8% Radius/ulna - 6% Tibia/fibula - 28% Vertebrae - 2%
Q 97.84: A women is being evaluated at her 36-week obstetrical appointment. She is not obese, her bladder is empty, and she does not have any complications. The fetus is in a cephalic position by Leopold maneuver. You measure her fundal height. What should it measure? A 32 to 34 cm B 32 to 36 cm C 34 to 38 cm D 35 to 39 cm E 32 to 39 cm
The Correct Answer is: C Fundal height in an uncomplicated, normal weight pregnancy should be within 1 to 2 cm per week of gestation in pregnancies above 20 weeks.
Q 30.2: A 46-year-old female presents with pain to her left wrist. She complains that it is painful and swollen as she points to the volar aspect of the wrist on the radial side. On examination, there is a small, soft bump on the dorsum of her wrist with a jelly-like consistency. What is the most likely diagnosis? A Cancerous tumor B Fracture C Ganglion cyst D Hematoma E Lipoma
The Correct Answer is: C Ganglion cysts commonly occur on the dorsal or volar aspect of the wrist. They result when a joint capsule or tendon sheath is damaged, allowing synovial fluid to escape producing a one-way valve, which allows fluid into the cyst, but not back out. The accumulating fluid forms the ganglion cyst. These cysts may or may not be tender and can fluctuate in size depending on activity level of the affected extremity. Cancerous tumors would tend to be much more firm, but also may be relatively pain free. Fractures would generally be exquisitely tender and if the bump is due to a displaced bone, it would be much more firm than a ganglion cyst. Hematomas are generally associated with acute trauma and would be tender and ecchymotic in many situations. Lipomas are benign fatty tumors that are more commonly seen on the thenar eminence than the dorsum of the wrist and their size does not change based on activity level.
Q 97.77: You are examining the wet prep of a young woman who presented complaining of vaginal discharge. You see 15 to 20 WBCs, 2+ bacteria, and clues cells. The KOH prep is whiff positive. What type of vaginitis does this indicate? A Yeast vaginitis B Viral vaginitis C Gardnerella vaginitis D Trichomonas vaginitis E Atrophic vaginitis
The Correct Answer is: C Gardnerella produces a positive amine test when the preparation is mixed with KOH. Viral Vaginitis, yeast, and Trichomonas do not produce a positive amine smell when mixed with KOH.
A 16-year-old high school boy presents to the emergency department 4 hours after sustaining an abrasion to his knee after a fall while rollerblading on the school playground. His school immunization record reveals that his last diphtheria, tetanus, and pertussis (DTaP) booster was administered at age 4. In this situation, which of the following is the MOST appropriate plan? A administer tetanus toxoid B administer adult tetanus and diphtheria toxoid (Td) C administer diphtheria, tetanus toxoid, and acellular pertussis (Tdap) vaccine D administer tetanus immune globulin
The Correct Answer is: C Generalized tetanus (lockjaw) is a neurologic disease caused by Clostridium tetani. Although any open wound is a potential source for contamination with C tetani, those with dirt, soil, feces, or saliva are at increased risk. Tetanus-prone wounds contain devitalized tissue, especially those caused by punctures, frostbite, crush injury, or burns. Recommendations for tetanus prophylaxis in a child with a laceration or abrasion depend upon the number of previous vaccinations, occurrence of last booster, type of wound (clean or tetanus-prone), and age of child. In this case, the patient is older than 7 years and had all of his previous immunizations; however, his most recent booster was greater than 10 years ago. Thus, he should receive an adult-type diphtheria and tetanus toxoid with acellular pertussis. In most cases, when tetanus toxoid is required for wound prophylaxis in a child older than 7 years, the Td instead of tetanus toxoid alone is recommended so that diphtheria immunity is maintained. If tetanus immunization is not up to date at the time of wound treatment, then the immunization series should be completed according to the primary immunization schedule. If a child is younger than 7 years, then the diphtheria, tetanus, acellular pertussis (DTaP) booster is indicated, unless there is a contraindication for pertussis, in which case the diphtheria and tetanus (DT) booster should be administered. Tetanus immune globulin (TIG) is recommended for treatment of tetanus. Under special circumstances, a patient infected with the human immunodeficiency virus (HIV) with a tetanus-prone wound should also receive TIG in addition to the prophylactic vaccine. (Ogle and Anderson, 2009, pp. 1144-1147; Centers for Disease Control and Prevention, 2009a) Ogle JW , Anderson MS. Infections: bacterial & spirochetal. In: Hays WW , Levin MJ , Sondheimer JM, et al., eds. Current Diagnosis & Treatment: Pediatrics. 19th ed. New York: McGraw-Hill; 2009. Centers for Disease Control and Prevention (2009a). Summary of recommendations for tetanus toxoid, reduced diphtheria toxoid and acellular pertussis vaccine (Tdap) and tetanus and diphtheria toxoids (Td) use among adolescents aged 11-18 years .
Q 39.5: A 16-year-old high school boy presents to the emergency department 4 hours after sustaining an abrasion to his knee after a fall while rollerblading on the school playground. His school immunization record reveals that his last diphtheria, tetanus, and pertussis (DTaP) booster was administered at age 4. In this situation, which of the following is the MOST appropriate plan? A administer tetanus toxoid B administer adult tetanus and diphtheria toxoid (Td) C administer diphtheria, tetanus toxoid, and acellular pertussis (Tdap) vaccine D administer tetanus immune globulin
The Correct Answer is: C Generalized tetanus (lockjaw) is a neurologic disease caused by Clostridium tetani. Although any open wound is a potential source for contamination with C tetani, those with dirt, soil, feces, or saliva are at increased risk. Tetanus-prone wounds contain devitalized tissue, especially those caused by punctures, frostbite, crush injury, or burns. Recommendations for tetanus prophylaxis in a child with a laceration or abrasion depend upon the number of previous vaccinations, occurrence of last booster, type of wound (clean or tetanus-prone), and age of child. In this case, the patient is older than 7 years and had all of his previous immunizations; however, his most recent booster was greater than 10 years ago. Thus, he should receive an adult-type diphtheria and tetanus toxoid with acellular pertussis. In most cases, when tetanus toxoid is required for wound prophylaxis in a child older than 7 years, the Td instead of tetanus toxoid alone is recommended so that diphtheria immunity is maintained. If tetanus immunization is not up to date at the time of wound treatment, then the immunization series should be completed according to the primary immunization schedule. If a child is younger than 7 years, then the diphtheria, tetanus, acellular pertussis (DTaP) booster is indicated, unless there is a contraindication for pertussis, in which case the diphtheria and tetanus (DT) booster should be administered. Tetanus immune globulin (TIG) is recommended for treatment of tetanus. Under special circumstances, a patient infected with the human immunodeficiency virus (HIV) with a tetanus-prone wound should also receive TIG in addition to the prophylactic vaccine. (Ogle and Anderson, 2009, pp. 1144-1147; Centers for Disease Control and Prevention, 2009a) Ogle JW , Anderson MS. Infections: bacterial & spirochetal. In: Hays WW , Levin MJ , Sondheimer JM, et al., eds. Current Diagnosis & Treatment: Pediatrics. 19th ed. New York: McGraw-Hill; 2009. Centers for Disease Control and Prevention (2009a). Summary of recommendations for tetanus toxoid, reduced diphtheria toxoid and acellular pertussis vaccine (Tdap) and tetanus and diphtheria toxoids (Td) use among adolescents aged 11-18 years .
Q 115.16: Use of which of the following medications can result in hearing loss? A Cefalexin B Erythomycin C Gentamycin D Ciprofloxacin E Hydrochlorothiazide
The Correct Answer is: C Gentamycin is an aminoglycoside, and can cause ototoxicity. Peak and trough levels must be drawn to determine the lowest effective dose. The remaining medications do not interfere with vestibular function. (Lalwani A.K., 2008, Chapter 53)
Q 97.70: You are providing care to a woman who is at 33 weeks gestation. Her pregnancy is complicated by gestational diabetes. She is being provided education by the dietician, and has weekly obstetrical appointments. What fasting blood sugar (FBS) readings should necessitate switching from diet control to insulin therapy? A FBS > 70 mg/dL B FBS > 90 mg/dL C FBS > 95 mg/dL D FBS > 110 mg/dL E FBS > 126 mg/dL
The Correct Answer is: C Gestational diabetes has different risks associated with it, in particular for the fetus. Stricter glycemic controls are recommended for pregnant versus non pregnant women by the ACOG and the ADA.
Q 46.6: A 10-year-old male who plays soccer presents with annularly configured dermal papules that are skin colored and shiny, and are located on his shins. His mother states that the lesions started as nodules, and have since enlarged. There is no scale present and they are asymptomatic. No one else in his home has similar findings. What is the most likely diagnosis? A Atopic dermatitis B Contact dermatitis C Granuloma annulare D Tinea corporis
The Correct Answer is: C Granuloma annulare (GA) is a self-limited condition that may appear as solitary lesions or in a more generalized distribution. They begin as small, shiny skin colored dermal papules that enlarge over time with central clearing. There are no epidermal manifestations. GA commonly occurs over bony surfaces, such as the shins or dorsa of the hands. It is frequently misdiagnosed as tinea corporis; however, the absence of epidermal scaling helps rule out this diagnosis.
Q 111.10: Which of the following viruses is rodent-borne and the cause of hemorrhagic fever and a pulmonary syndrome, which begins with a fever and may rapidly progress to shock and adult respiratory distress syndrome? A human T-cell lymphotropic virus (HTLV) B Flavivirus C Hantavirus D Filovirus E coronavirus
The Correct Answer is: C Hantavirus has a rodent vector and usually manifests in either hemorrhagic fever or Hantavirus pulmonary syndrome, which can be fatal. In the United States, outbreaks are usually in the southwest. There have been 300 cases since 1993. HTLV is a lymphotropic oncovirus associated with lymphoma. Dengue and yellow fever are both caused by Flaviviridae, which is carried by mosquitoes. Filoviruses cause Ebola fever and Marburg fever. The vector is unknown. Coronavirus is the etiologic agent in severe acute respiratory syndrome. During the 2002-2003 epidemic that began in Southeast Asia, it was postulated that it was carried by the masked palm civet. (Shandera and Corrales-Medina, 2009, pp. 1233-1234) Shandera WX , Corrales-Medina VF. Viral & rickettsial infections. In: McPhee SJ , Papadakis MA, eds. Current Medical Diagnosis and Treatment. 48th ed. New York, NY: McGraw-Hill; 2009.
Q 109.12: A 28-year-old male presents with a diagnosis of Hemophilia A. Which of the following would he be deficient in? A Christmas factor B Factor VII C Factor VIII D Factor IX E Von Wildebrand's factor
The Correct Answer is: C Hemophilia A is caused by a defective synthesis of Factor VIII. Factor IX and Christmas factor are deficiencies in Hemophilia B. Deficiency of factor VII can exacerbate bleeding issues, but are not typically seen in Hemophilia A. Von Willebrand's factor is a loss of this platelet-binding multimer, and most patients will have enough factor VIII but not the ability to bind factor VIII. In very severe cases of vWF, factor VIII may become low enough to produce symptoms similar to those found in factor VIII deficiency. (Lichtman et al., Williams Hematology 8e, Chapter 124, Hemophilia A and Hemophilia B)
Q 67.6: A 42-year-old male presents with a history of low grade fever, cough, and myalgias for five days. He states that these symptoms began after a cave exploration trip along the Ohio River two weeks ago, and have since worsened. An x-ray reveals focal consolidation, and you suspect Histoplasmosis pneumonia. Which of the following is the first line treatment of choice? A Azithromycin B Doxycycline C Itraconazole D Terbinafine E Trimethoprim-sulfamethoxazole
The Correct Answer is: C Histoplasmosis is caused by a dimorphic fungus, most commonly Histoplasma capsulatum (although other species exist). Within the United States, endemic areas include the Ohio and Mississippi river valleys. Additional areas of risk include other parts of North, South, and Central America, Africa, Mexico, and Central Asia. Large amounts of bird and bat droppings within specific soils promote the growth of the fungus, and exposure typically occurs during activities that disrupt the soil and aerosolize the spores. Depending upon the length and intensity of exposure, and the patient's immune system and previous lung history, infections may range from asymptomatic to severe. Treatment is based upon the patient's clinical picture, with mild to moderate disease being treated with oral antifungal agents. (McPhee SJ, Papadakis MA. Current Medical Diagnosis & Treatment 2011, Chapter 36, Mycotic Infections)
Q 97.98: A 26-year-old female presents to clinic complaining of increasing headaches for one year, irritability, bloating and fluid retention, and abdominal discomfort with loose stools during her menstrual cycle. The symptoms begin a day or two before her menses, and last until the middle of her cycle. She has tried acetaminophen and ibuprofen without improvement. On physical exam she is a well-developed, well-nourished female in no acute distress. Vitals are normal, CV and lungs are normal, pelvis exam is normal, pap smear is normal, and GC and Chlamydia testing are negative. What would be the most appropriate next step? A Pelvic ultrasound B FSH, LH levels C Fluoxetine on cycle day 21-7 D Paroxetine daily E Serum HCG
The Correct Answer is: C Hx and Px are key to diagnosing premenstrual syndrome. Laboratory and radiologic procedures are not useful, particularly in light of a normal exam. With failure of NSAIDS, treatment is aimed at reducing symptoms. For mild to moderate symptoms, SSRI therapy prior to and through the menstrual cycle has become a primary therapy.
Q 19.5: An 11-month-old African-American male presents to the pediatric office with lethargy, jaundice and splenomegaly. A CBC reveals hemoglobin of 8.0 mg/dl and a hematocrit of 25%. Peripheral smear appearance is available below. Which of the following treatments will reduce hemolysis and increase hemoglobin levels for this patient? A Allogeneic stem cell transplant B Folic acid supplementation C Hydroxyurea D Prophylactic penicillin E Pneumococcal vaccine
The Correct Answer is: C Hydroxyurea directly reduces hemolysis and increases levels of fetal hemoglobin and reduces complications and transfusion frequency. Sickle cell disease can be cured in 80% of individuals who receive a suitable transplant (A), but this procedure carries great risk and donors are difficult to identify. Folic acid supplementation (B) and pneumococcal vaccine (E) are indicated for preventive therapy of all patients with sickle cell disease, and prophylactic antibiotics (D) are used to prevent recurrent infections.
A 15-year-old boy suddenly collapses on the basketball court; his sports physical conducted at the beginning of the year did not elicit any abnormal findings. Basic life support initiated at the scene, however, is unsuccessful in resuscitation. Which of the following is the most likely etiology of his sudden death? A mitral valve prolapse B surgically corrected aortic stenosis C hypertrophic cardiomyopathy D rheumatic heart disease
The Correct Answer is: C Hypertrophic cardiomyopathy in adolescence is typically due to familial hypertrophic cardiomyopathy with an incidence of 1:500. Many patients are asymptomatic until a sporting event, which may cause symptoms, specifically sudden cardiac death. Examination may demonstrate a palpable or audible S 4 , an LV (left ventricular) heave, systolic ejection murmur (may need to stimulate cardiac activity), and/or a left precordial bulge. Echocardiography is the gold standard for diagnosis but family history should be assessed. Stress testing is indicated to assess for ischemia and arrhythmias. Strenuous activities are prohibited for these patients. The other cardiomyopathies (dilated and restrictive) are next but are not as common. Congenital structural abnormalities of the coronary arteries are the next most common cause. Valvular disorders, including surgically repaired aortic stenosis, are typically not causes of sudden death, but these patients should be screened for symptoms and stress tested as necessary.
Q 49.10: A 15-year-old boy suddenly collapses on the basketball court; his sports physical conducted at the beginning of the year did not elicit any abnormal findings. Basic life support initiated at the scene, however, is unsuccessful in resuscitation. Which of the following is the most likely etiology of his sudden death? A mitral valve prolapse B surgically corrected aortic stenosis C hypertrophic cardiomyopathy D rheumatic heart disease
The Correct Answer is: C Hypertrophic cardiomyopathy in adolescence is typically due to familial hypertrophic cardiomyopathy with an incidence of 1:500. Many patients are asymptomatic until a sporting event, which may cause symptoms, specifically sudden cardiac death. Examination may demonstrate a palpable or audible S 4 , an LV (left ventricular) heave, systolic ejection murmur (may need to stimulate cardiac activity), and/or a left precordial bulge. Echocardiography is the gold standard for diagnosis but family history should be assessed. Stress testing is indicated to assess for ischemia and arrhythmias. Strenuous activities are prohibited for these patients. The other cardiomyopathies (dilated and restrictive) are next but are not as common. Congenital structural abnormalities of the coronary arteries are the next most common cause. Valvular disorders, including surgically repaired aortic stenosis, are typically not causes of sudden death, but these patients should be screened for symptoms and stress tested as necessary.
Q 99.2: A 30-year-old patient presents 2 months postthyroidectomy. The patient has had symptoms of increased irritability, muscle spasms, and hair loss for the past month. On physical examination, a positive Chovstek sign is noted. Which of the following is the most likely diagnosis? A hypothyroidism B hypopituitarism C hypoparathyroidism D hypogonadism
The Correct Answer is: C Hypoparathyroidism commonly presents following thyroidectomy surgery. This patient has classic signs and symptoms of a low calcium level and hypoparathyroidism. Chovestek sign is a physical exam finding that is positive after tapping in front of the ear in the facial nerve region. When doing this, the muscle contracts. When the calcium level is low, this occurs. Hypothyroidism can occur following a thyroidectomy but the symptoms are not the same. (Fitzgerald, 2009, pp. 1004-1005) Fitzgerald PA. Endocrine diseases. In: McPhee SJ , Papadakis MA, eds. Current Medical Diagnosis and Treatment. 48th ed. New York, NY: McGraw-Hill; 2009.
Q 24.3: A 18 year-old man present to the clinic due to a recent positive tuberculosis screening test. Which of the following steps should be taken to avoid neurotoxicity associated with his prophylactic isoniazid (INH) treatment? A Perform an initial screening mental status exam B Supplement with folic acid C Supplement with pyridoxine (vitamin B6) D Take medication on a full stomach E Take the medication three times daily
The Correct Answer is: C INH leads to the development of peripheral neuropathy in 2% of patients secondary to pyridoxine deficiency. This can be prevented through the coadministration of pyridoxine (vitamin B6) (C).
Q 97.57: A 26-year-old female presents to the clinic for evaluation of her inability to conceive for 12 months. She menstruates monthly. Her past medical history is significant for PID x 2 and mild asthma. Her GC, Chlamydia, and pap smear are negative. Her TSH is 3.0 and her UCG today is negative. What is the most likely cause for her inability to conceive? A Anovulatory cycles B Hypothalamic state C Fallopian tube scarring D Hypothyroidism E Polycystic ovarian syndrome
The Correct Answer is: C In a young and otherwise healthy female with a negative work up and a history of PID, the most common cause of infertility is fallopian tube scarring secondary to STD and PID.
Q 49.14: A 5-year-old male presents with a first-time, one-week history of a cough, nasal congestion, and a temperature of 101˚ F orally. Upon physical exam, you note a distended, erythemic right tympanic membrane that has decreased mobility, with pneumatic otoscopy. There are no known drug allergies. What is the first line antibiotic indicated for this condition? A Ciprofloxacin B Moxifloxacin C Amoxicillin D Clindamycin E Cefaclor
The Correct Answer is: C In cases of non-resistant otitis media, amoxicillin, erythromycin, or a sulfonamide are first line antibiotics. Flouroquinolones, like ciprofloxacin or moxifloxacin, are contraindicated in children.
Q 39.30: In a 6 year old non-bleeding child with a platelet count of 20,000, and a new diagnosis of immune thrombocytopenic purpura, what is the best clinical intervention? A Glucocorticoids B Immunization against pneumococcus C No therapy D Splenectomy E Weekly transfusions of platelets
The Correct Answer is: C In non-bleeding children with platelet counts of 20,000, no therapy is indicated. More than two-thirds of children diagnosed with ITP will resolve in less than six months, and more than 80% after one year. Immunization against pneumococcus is appropriate if a splenectomy is planned. Glucocorticoids are useful, but may be avoided as an initial intervention and avoidance of side effects. Splenectomy is used only after one year of diagnosis, and when the child is at least five years old. Transfusion of platelets is not needed in non-bleeding individuals.
Q 79.4: A 55-year-old woman with a history of emphysema, who is undergoing chemotherapy for lung cancer, is sent to see you by her oncologist regarding a sudden increase in dyspnea, with exertion and fatigue. On physical exam, pulsus paradoxus and muffled heart sounds are noted. Which of the following diagnostic studies would be most effective in establishing a definitive diagnosis given this patient's physical exam findings? A Chest x-ray B CBC with differential C Transthoracic echocardiography D Pulmonary function tests E Tilt-table test
The Correct Answer is: C In patients with a history of malignancy, sudden worsening of dyspnea and physical exam findings of pulsus paradoxus and muffled heart sounds (two of the three components of Beck's triad), a clinical suspicion of cardiac tamponade should be part of a clinician's differential diagnosis. Choice C, transthoracic echocardiography, is most effective and will establish a definitive diagnosis of cardiac tamponade, as it allows direct visualization of the location and amount of pericardial fluid. A CXR may demonstrate a classic "water bottle" shape of the heart, but may also be normal. A CBC with differential would not give a definitive diagnosis of cardiac tamponade. Pulmonary function tests will likely be abnormal in the setting of a patient with emphysema and lung cancer, and thus would also not offer a definitive diagnosis if cardiac tamponade is suspected. Tilt-table testing is useful in patients with syncope, but not in this patient population. (Fauci et al., 2001, p. 1366)
Q 38.19: An Rh-negative, 5-year-old male child presents with acute onset of petechiae and purpura after an acute viral illness. In addition, he has episodes of epistaxis. Which of the following is a treatment option if his platelet count falls below 20,000/mm 3 , but he is not actively bleeding? A platelet transfusions B IV anti-D (WinRho SD) 50-70 mg/kg/dose C prednisone 2.4 mg/kg/24 hours × 2 weeks D splenectomy
The Correct Answer is: C In patients with idiopathic thrombocytopenic purpura, treatment options should be initiated when platelet counts *fall below 20,000, *regardless of whether there is active bleeding or not. Without active bleeding the treatment options include prednisone 2-4 mg/kg/24 hours for 2 weeks; IV immunoglobulin 1 g/kg/24 hours for 1 to 2 days, or IV anti-D 50-75 μg/kg/dose for Rh-positive patients. Splenectomy is indicated for life-threatening bleeding. There is currently no indication for platelet transfusion and none of the above treatments are considered optimal, because in the majority of children, it will resolve on its own within 6 months.
Q 97.50: A 22-year-old female presents to her obstetrical appointment at 39 weeks gestation. Her pregnancy to date has been uncomplicated. She is concerned that her infant may be larger than average, as her fundal height measures 41. On physical exam, her fetus is in a cephalic presentation, her cervix is soft and 1-cm dilated, and the fetus is at a -3 station. Her membranes are intact, she is not contracting, and her vitals are normal. She requests to be induced. For decreased risk of complication and optimal fetal outcome, when should she expect to be induced? A Now B At 40.5 weeks C 41.5 weeks D 42.5 weeks E Not expected
The Correct Answer is: C In the absence of complication, the recommendation from ACOG is to wait for labor to occur. Large for gestational age is not an indication for induction in the absence of diabetes. Gestations greater than 42 weeks increase risk of fetal stillbirth.
Q 24.1: Of the following sexual practices, which poses the greatest risk of HIV transmission when practiced with an infected partner but without the use of a reliable barrier method of prophylaxis? A insertive anal intercourse B insertive vaginal intercourse C receptive anal intercourse D receptive fellatio with ejaculation E receptive vaginal intercourse
The Correct Answer is: C In unprotected intercourse with an infected partner, receptive anal intercourse carries a risk of HIV transmission between 1:100 and 1:30. Insertive anal intercourse, receptive vaginal intercourse, and fellatio with ejaculation each carry a risk of about 1:1000. Insertive vaginal intercourse carries a risk of 1:10,000.
Q 97.94: Of the following sexual practices, which poses the greatest risk of HIV transmission when practiced with an infected partner but without the use of a reliable barrier method of prophylaxis? A insertive anal intercourse B insertive vaginal intercourse C receptive anal intercourse D receptive fellatio with ejaculation E receptive vaginal intercourse
The Correct Answer is: C In unprotected intercourse with an infected partner, receptive anal intercourse carries a risk of HIV transmission between 1:100 and 1:30. Insertive anal intercourse, receptive vaginal intercourse, and fellatio with ejaculation each carry a risk of about 1:1000. Insertive vaginal intercourse carries a risk of 1:10,000.
Q 106.19: A 68-year-old male patient presents to your office complaining of difficulty urinating, nocturia that is interfering with his sleep to the point where he is fatigued, along with lower back pain. On physical examination, his prostate is enlarged, irregular, and slightly tender. His U/A is normal. Of the following items that may be in your differential diagnosis must you pursue fully? A benign prostatic hypertrophy B prostatitis C prostate cancer D testicular torsion E urinary retention
The Correct Answer is: C Incidental or stage A (T1) carcinoma of the prostate presents no physical signs (it is nonpalpable) and is only diagnosed by the pathologist when prostate tissue is removed as treatment for symptomatic bladder outlet obstruction presumed to be caused by benign prostatic hyperplasia or is found by an elevated PSA (T1c). Patients with stage B (T2) or higher disease have a hard nodule on the prostate that can be felt during rectal examination. Previously, 50% of patients presented with evidence of metastases, including weight loss, anemia, bone pain (commonly in the lumbosacral area), or acute neurologic deficit in the lower limbs. Today, however, fewer than 20% of patients present in this way because of earlier diagnosis due to wide use of PSA screening (stage migration). (Doherty, 2010, Chapter 38)
Q 44.6: An aspirated peanut that is causing a partial obstruction of the trachea in a child is most likely to cause which of the following physical exam findings? A Aphonia B Inability to cough C Stridor D Progressive cyanosis E Rhonchi
The Correct Answer is: C Incomplete airway obstruction due to a foreign body will cause turbulent air flow in the airway and an inspiratory wheeze sound, known as stridor. Attempts should be made to remove the foreign body, leaving the patient with a partial obstruction to utilize the cough reflex to remove the foreign body. If unsuccessful, or if findings such as aphonia, an inability to cough, progressive cyanosis, or unconsciousness occur, a complete obstruction is present and definitive intervention must convene. Age appropriate measures, utilizing back blows and chest compressions in infants younger than the age of one and abdominal thrusts in children over the age of one, should be attempted.
Q 44.7: A mother brings in her 20-month-old female child to the office because she noticed pubic hair growing. On examination, the clinician notices that the clitoris is enlarged; the rest is unremarkable. Which of the following is an expected laboratory finding on this patient? A increased aldosterone B increased estrogen C increased androstenedione D increased luteinizing hormone
The Correct Answer is: C Infant girls presenting with signs of precocious puberty need to be screened for congenital adrenal hyperplasia (CAH). CAH most commonly presents with pseudohermaphroditism in females—urogenital sinus, enlarged clitoris, or other signs of virilization. In males, there tends to be isosexual precocity in older males and salt-losing crisis in infant males. Both children show increased linear growth and skeletal maturation. The most common type of CAH is a deficiency in the enzyme 21-hydroxylase and laboratory tests demonstrate increased urinary and plasma androgens (DHEA, androstenedione). There may be elevated progesterone, but typically there is no effect on estrogen. There is also decreased aldosterone and elevated urinary ketosteroids. There is also no effect on the levels of leuteinizing hormone or follicle-stimulating hormone. Treatment usually involves glucocorticoids, mineralocorticoids, and reconstructive surgery, if needed.
Q 97.24: A 24-year-old G1P1 presents to the office complaining of a red, tender area of her right breast. She is four weeks postpartum and is nursing her infant with good success. She complains of no other symptoms. On physical exam, her vitals are normal. Lungs CTA, CV RRR, left breast is normal, right breast has a 3-cm area that is warm with erythema, and no mass or area of fluctuance is noted. She has a MRSA mastitis. How did she most likely contract the infection? A Community acquired B Hospital acquired C From her infant D Self inoculated
The Correct Answer is: C Infants usually contract MRSA due to poor hand washing technique from the hospital staff, but it is then spread to the mother via the infant.
Q 97.72: A 27-year-old woman and her male partner come to the emergency department for assistance with emergency contraception. They experienced condom failure during intercourse an hour ago and neither desires pregnancy. Her last menstrual period was approximately two weeks ago and her cycles occur every 28-30 days. Her medical history includes a deep venous thrombosis during labor and delivery 5 years ago. What is the most appropriate course of action at this time? A Administration of an ethinyl estradiol and levonorgestrel combination now and in 12 hours B Dilation and curettage C Insertion of a copper-containing intrauterine device D Serial beta hCG determinations E Testing for factor V Leiden
The Correct Answer is: C Insertion of a copper-containing IUD is an effective means of preventing an unintended pregnancy in this case. The woman's history of clotting is a contraindication to use of combination oral contraceptives (A). Dilation and curettage (B) is neither appropriate nor effective for emergency contraception. Performing beta hCG determinations (D) would merely detect pregnancy if it were to occur. Given her history of clotting, testing for factor V Leiden (E) may be appropriate but will not affect management at this time.
A 23-year-old female who has a history of supraventricular tachycardia is having an acute episode again. She has attempted a valsalva maneuver without success in breaking the arrhythmia. The ECG confirms SVT. What is the next step in therapy for this patient? A Atropine B Amlodipine C Adenosine D Amiodarone E Metoprolol
The Correct Answer is: C Intravenous adenosine is the treatment of choice in this clinical situation. If successful the adenosine will break the cycle of tachycardia, usually with a pause. The initial dose is 6mg, followed by two, 12 mg doses if unsuccessful. While this is the standard protocol as guided by ACLS, additional doses of adenosine can be given in a hospital setting, as the half-life of the drug is extremely short. Amiodarone and amlodipine would not have an effect on the supraventricular effect of the arrhythmia.
Q 38.15: A 12-year-old female presents with a complaint of dry flaking skin that becomes fissured and painful. Her skin has always had dark plate-like scales. What is the most likely finding in the patient's history? A Her skin forms blisters with minimal contact B She was premature C There was a membrane present at birth D There were no symptoms present until the age of six
The Correct Answer is: C Lamellar ichthyosis is a condition in which a baby is born with a collodion membrane. Within a few weeks, this membrane is shed and replaced by large gray scales. These plate-like scales persist with no improvement over time. Painful fissures on the hands and feet are common.
Q 106.14: Which of the following drugs block the actions of leukotrienes and can be used for long-term control of mild persistent asthma? A cromolyn sodium B omalizumab C zafirlukast D nedocromil sodium E ipratropium bromide
The Correct Answer is: C Leukotrienes are inflammatory mediators that are generated within the lungs. When they bind to specific receptors, they induce a variety of responses, including bronchospasm and mucus production. Zafirlukast (and also montelukast) are leukotriene receptor antagonists that block these effects in the lungs and improve asthma symptoms. Zafirlukast is considered an alternative therapy for long-term control of asthma, as it has been shown to be less effective than inhaled corticosteroids. Both cromolyn and nedocromil are mast cell stabilizers and can also be used as an alternative treatment to inhaled corticosteroids. Omalizumab is an anti-IgE antibody, whereas ipratropium bromide is a muscarinic receptor antagonist. (Kelly, 2008, pp. 478, 488-490) Kelly HW , Sorkness CA. Asthma. In: DiPiro JT , Talbert RL , Yee GC, et al., eds. Pharmacotherapy: A Pathophysiologic Approach. 7th ed. New York: McGraw-Hill; 2008.
Q 14.8: A 73-year-old female with type 2 diabetes, hypertension, and hyperlipidemia presents to the outpatient clinic complaining of left ear pain, and a yellowish-green, foul-smelling discharge that began about 3 weeks ago. On physical examination, the patient is afebrile and examination reveals a markedly edematous left ear canal draining purulent, green discharge. The tympanic membrane is unable to be visualized. Which of the following is the most likely diagnosis? A Auricular cellulitis B Cerebrospinal fluid leakage C Malignant otitis externa D Otitis externa E Serous otitis media with perforation
The Correct Answer is: C Malignant otitis externa (C) is a more serious form of otitis externa (D) that most commonly occurs in patients with diabetes and is most commonly caused by pseudomonas. The case scenario describes the location as the external auditory canal without significant involvement of the auricle (A). Serous otitis media with perforation (E) and CSF leak (B) would present with a clear drainage.
Q 22.5: A 73-year-old female with type 2 diabetes, hypertension, and hyperlipidemia presents to the outpatient clinic complaining of left ear pain, and a yellowish-green, foul-smelling discharge that began about 3 weeks ago. On physical examination, the patient is afebrile and examination reveals a markedly edematous left ear canal draining purulent, green discharge. The tympanic membrane is unable to be visualized. Which of the following is the most likely causative agent for this patient's diagnosis? A Escherichia coli B Moraxella catarrhalis C Pseudomonas aeruginosa D Staphylococcus aureus E Streptococcus pneumoniae
The Correct Answer is: C Malignant otitis externa is most commonly caused by pseudomonas (C). E coli (A) and S aureus (D) are less common causes of otitis externa, while S pneumoniae (E) and M catarrhalis (B) are common etiologies of acute otitis media.
Q 83.3: A 72-year-old male presents with bony pain, Bence Jones protein in his urine, an elevated creatinine level of 2.0, hypercalcemia, and lytic lesions to the long bones of his legs. The best initial treatment to correct this patient's renal insufficiency is which of the following? A Bisphosphonate and plasma phoresis B Dialysis and calcitonin C Hydration and calcitonin D Hydration and rapid single infusion of bisphosphonate E Plasmaphoresis and hydration
The Correct Answer is: C Management of renal impairment in patients with multiple myeloma is primarily supportive care. Hydration and use of calcitonin is the mainstay. If the hypercalcemia needs rapid correction, a slow infusion of bisphosphonate may be used. Dialysis would be useful if the patient has renal failure but has difficulty removing light chains from the blood, which are causing the problem. Plasmaphoresis doesn't play a role in this setting. (Lichtman et al., Williams Hematology 8e, Chapter 109, Myeloma)
Q 98.1: What absolute tissue pressure generally is used as a guideline for diagnosing compartment syndrome? A 10 mm Hg B 20 mm Hg C 30 mm Hg D 40 mm Hg E 50 mm Hg
The Correct Answer is: C Many trauma surgery services use an absolute tissue pressure of approximately 30 mm Hg as the threshold for diagnosing compartment syndrome. Based on the entire clinical picture, patients with numbers in that range or higher will likely require surgical decompression with a fasciotomy, while lower numbers will probably be managed with a more conservative approach.
Q 40.10: A 12-year-old male presents with a rash, consisting of erythematous macules and papules involving the face, trunk, and extremities. He also complains of cough, coryza, and non-purulent conjunctivitis. The described exanthema and associated enanthem is pathognomonic for what disorder? A Erythema infectiosum B Hand, foot, and mouth disease C Measles virus D Varicella virus
The Correct Answer is: C Measles is one of the few exanthemas where the primary lesions are both macules and papules. These lesions may coalesce to become confluent. The pathognomonic lesion of measles is the Koplik spot, which is a blue-white papule with surrounding erythema appearing on the oral mucosa.
Q 40.7: A 6-year-old girl presents to the emergency department with abdominal distension of 1-day duration. She has not had a bowel movement or passed flatus in 72 hours. Examination reveals markedly diminished bowel sounds with tympany to percussion. She has also passed bloody mucus from her rectum. There is no evidence of hernia, and surgical history is negative. Which of the following is the most likely diagnosis? A Regional enteritis B Pyloric stenosis C Meckel diverticulum D Acute appendicitis
The Correct Answer is: C Meckel diverticulum is prevalent in 2% of the population, has a 2:1 male:female predominance, and is usually located 2 ft from the ileocecal valve. The most common clinical presentations are bleeding, intestinal obstruction, and inflammation. Bright red or maroon bleeding is the most frequent complication in children younger than 2 years of age. Obstruction may develop secondary to a volvulus that occurs at the site of the diverticulum or from an intussusception with the diverticulum acting as the lead point. An air contrast barium enema may be able to reduce intussusceptions in children.
Q 98.74: A 6-year-old girl presents to the emergency department with abdominal distension of 1-day duration. She has not had a bowel movement or passed flatus in 72 hours. Examination reveals markedly diminished bowel sounds with tympany to percussion. She has also passed bloody mucus from her rectum. There is no evidence of hernia, and surgical history is negative. Which of the following is the most likely diagnosis? A Regional enteritis B Pyloric stenosis C Meckel diverticulum D Acute appendicitis
The Correct Answer is: C Meckel diverticulum is prevalent in 2% of the population, has a 2:1 male:female predominance, and is usually located 2 ft from the ileocecal valve. The most common clinical presentations are bleeding, intestinal obstruction, and inflammation. Bright red or maroon bleeding is the most frequent complication in children younger than 2 years of age. Obstruction may develop secondary to a volvulus that occurs at the site of the diverticulum or from an intussusception with the diverticulum acting as the lead point. An air contrast barium enema may be able to reduce intussusceptions in children.
Q 110.12: Over a period of several months, a 62-year-old man has developed erectile dysfunction. He has no history of neurologic, kidney, or cardiovascular disease or diabetes mellitus. He takes a multivitamin and an occasional ibuprofen for aches and pains. He has never smoked cigarettes, drinks 1-2 glasses of wine with dinner on weekends, and uses no mind-altering drugs. Physical examination is remarkable only for bilateral gynecomastia. What is the most likely diagnosis? A breast cancer B depression C prolactinoma D steroid abuse E testicular cancer
The Correct Answer is: C Men with prolactinomas may experience erectile dysfunction, infertility, and, less commonly, gynecomastia. Breast cancer in men (A) presents as a usually as a unilateral mass. Men with depression (B) may have erectile dysfunction, but not gynecomastia. Steroid abuse (D) is associated with gynecomastia, but the patient would likely be showing other signs and symptoms. Testicular cancer (E), specifically germ cell cancer, is associated with gynecomastia in 5% of cases but this man has no testicular mass or swelling. Fitzgerald PA, Endocrine Disorders, in Current Medical Diagnosis and Treatment, 52 nd ed. 2013.
Q 76.7: A 43-year-old obese man presents for a health maintenance visit. On physical exam, it is noted that his waist circumference is 106 cm and blood pressure is 148/92 mm Hg. Which of the following fasting laboratory levels would suggest a diagnosis of metabolic syndrome (syndrome X) in this patient? A HDL of 45 mg/dL B LDL of 180 mg/dL C triglyceride of 190 mg/dL D glucose of 100 mg/dL
The Correct Answer is: C Metabolic syndrome is found in approximately 25% of Americans. It is defined as three or more of the following findings: waist circumference of greater than 102 cm in men or greater than 88 cm in women; serum triglyceride level of at least 150 mg/dL, HDL level of less than 40 mg/dL in men or less than 50 mg/dL in women; blood pressure of at least 130/85 mm Hg; and serum glucose level of at least 110 mg/dL. (Friedman and Herman-Bonert, 2009, p. 1105) Friedman TC , Herman-Bonert VS. Thyroid gland. In: McPhee SJ , Papadakis MA, eds. Current Medical Diagnosis and Treatment. 48th ed. New York, NY: McGraw-Hill; 2009.
Q 34.6: A 6-year-old female presents with small erythematous papules grouped around the mouth. The mother reports that she tried to treat with over-the-counter hydrocortisone 1% cream. The condition has worsened. What is the best step in management of this condition? A hydrocortisone valerate cream B keflex 500 mg bid C metronidazole 0.75% gel bid D clotrimazole cream bid
The Correct Answer is: C Metronidazole 0.75% gel bid is a first line treatment for perioral dermatitis. Topical steroids, such as hydrocortisone valerate cream, will actually worsen perioral dermatitis and create a granulomatous condition. Oral antibiotics, like Keflex 500 mg bid, are frequently used to treat perioral dermatitis. However, Keflex is not indicated for perioral dermatitis. Clotrimazole is an antifungal cream that is not used in perioral dermatitis.
Which type(s) of Salter-Harris fractures can generally be treated with closed reduction and cast immobilization? A Type I B Types I and II C Types I, II, and III D Types I, II, III, and IV E Types I, II, III, IV, and V
The Correct Answer is: C Minimally displaced Salter-Harris types I, II, and III fractures generally can be treated with immobilization only. Types IV and V involve the cartilage of both the articular surface and the growth plate. To ensure proper alignment and a congruous joint surfaces open reduction and internal fixation is usually necessary.
Q 97.110: A 25-year-old female presents for an ultrasound after having a positive home pregnancy test. She has an unremarkable past medical history and physical exam. She states she has been feeling fine without any abdominal discomfort or vaginal bleeding noted. On ultrasound you determine she is 10 weeks pregnant. You note a noncomplex unilateral mass on her left ovary measuring 2 cm in diameter. What is the most likely diagnosis? A Benign cystic teratoma B Serous cystadenoma C Functional ovarian cyst D Hemorrhagic ovarian cyst E Torsion of the adnexa
The Correct Answer is: C More than 90% of unilateral, noncomplex masses that are measured to be less than 5 centimeters in diameter that are identified in the first trimester are functional ovarian cysts. Benign cystic teratomas (A) make up 21% of pathologic ovarian neoplasms and serous cystadenomas (B) make up 21%. A hemorrhagic cyst (D) is a non-functional cyst. Torsion of the adnexa (E) is most commonly seen between 6-14 weeks gestation or in immediate puerperium but symptoms include abdominal pain and tenderness.
Q 106.7: A 33-year-old woman treated with trifluoperazine for the past 3 months is seen in the emergency department because of recent-onset fever, stiffness and tremor, as reported by her accompanying sister. The patient also appears to be mildly confused when asked about location, day, and time. Her temperature is 104.5°F, and her serum creatine kinase (CK) level is markedly elevated. Which of the following has most likely occurred? A a delayed allergic reaction has occurred with trifluoperazine B tardive dyskinesia has begun to develop in the patient C the patient has developed neuroleptic malignant syndrome D the patient has developed serotonin syndrome E the patient has overdosed on trifluoperazine
The Correct Answer is: C Neuroleptic malignant syndrome is an uncommon but serious complication with therapeutic doses of antipsychotic drug therapy, particularly the first-generation (typical) class. Cardinal signs and symptoms include a body temperature above 100.4°F, altered state of consciousness, autonomic dysfunction, and rigidity. (Crismon et al., 2008, pp. 1113-1114; Eisendrath and Lichtmacher, 2008, pp. 917-919) Crismon ML , Argo TR , Buckley PF. Schizophrenia. In: DiPiro JT , Talbert RL , Yee GC, et al. eds. Pharmacotherapy: A Pathophysiologic Approach. 7th ed. New York: McGraw-Hill; 2008. Eisendrath SJ , Lichtmacher JE. Psychiatric disorders. In: Tierney LM Jr , McPhee SJ , Papadakis MA, eds. Current Medical Diagnosis & Treatment. 47th ed. New York: McGraw-Hill; 2008.
Q 71.11: A 43-year-old pilot is interested in quitting his 20-year habit of smoking. His medical history includes type 2 diabetes mellitus diagnosed 6 years ago for which he is currently taking metformin. Which of the following would be most appropriate to recommend to this patient? A alprazolam B clonidine C nicotine replacement therapy D nortriptyline E varenicline
The Correct Answer is: C Nicotine replacement therapy is relatively safe in the majority of patients and comes in many forms (transdermal patches, gums, sprays, and inhalers). Both clonidine and nortriptyline are considered second-line smoking cessation agents because of their many side effects. Neither has been approved by the FDA for smoking cessation. Alprazolam is also not indicated, and there is currently no evidence that it aids in smoking cessation. *Varenicline is a relatively new agent for smoking cessation and is a partial agonist to α 4 -β 2 nicotinic acetylcholine receptors. It has been approved by the FDA; however, varenicline is banned from use by pilots and air traffic controllers as per the Federal Aviation Administration (FAA) *(Doering et al., 2008, pp. 1090-1094) Doering PL , Kennedy WK , Boothby LA. Substance-related disorders: Alcohol, nicotine and caffeine. In: DiPiro JT , Talbert RL , Yee GC, et al., eds. Pharmacotherapy: A Pathophysiologic Approach. 7th ed. New York: McGraw-Hill; 2008.
Q 19.9: A patient presents to your family practice office with classic renal colic with his pain being a 6 on a 1 to 10 scale, with 10 being the worst pain imaginable. You refer your patient for a stat computed tomography (CT) scan of the kidneys. Your radiologist calls and advises that the CT reveals that your patient has a 4 mm stone at the ureterovesical junction (UVJ) on the left side without evidence of hydronephrosis. What should you advise your patient? A Go directly to the emergency department for admission. B Go directly to the emergency department for hydration and pain management. C Return to the office, administer ketorolac IM, give a prescription for an opioid, increase fluids, strain their urine, go to the emergency department if the pain worsens, and return in 48 hours for a reevaluation. D Go home, take four Motrin IB tablets every eight hours for pain, and go to the emergency department if the pain worsens. E Refer the patient for immediate lithotripsy.
The Correct Answer is: C Nonsteroidal anti-inflammatory drugs (NSAIDs) in general, and ketoralac specifically more so than other, causes constriction of the renal afferent arteriole, reducing pressure on a kidney stone and providing significant pain relief. Generally speaking a stone of less than 5 mm, especially at the UVJ, will pass relatively rapidly (within 48 hours) and the patient only needs to strain their urine to collect the stone and receive analgesia.
Q 121.18: During the physical exam of a patient with a suspected pleural effusion, you ask the patient to make the sound "eee." You note on auscultation that the transmission is auscultated as "ay," suggestive of resonance through fluid. What is the name of this exam technique? A Bronchophony B Diaphragmatic excursion C Egophony D Tactile fremitus E Whispered pectoriloquy
The Correct Answer is: C Normal lungs transmit spoken sounds faintly and with indistinct syllables, except over main bronchi. An area of fluid, such as a pleural effusion, consolidation or atelectasis, and areas of fibrosis will increase sound transmission and alter the distinction of the sound. This occurs for both whispered and spoken sounds. The utilization of the spoken sound "eee," auscultating for a change to "ay" due to fluid within the lung fields, is termed egophony. The use of whispered sounds and generally spoken sounds to determine lung changes are termed whispered pectoriloquy and bronchophony respectively. Diaphragmatic excursion is performed to determine the thoracic diaphragmatic movement during respiration. Tactile fremitus assesses chest vibration during vocalization, with changes being noted in the presence of consolidation (increases fremitus) and pleural effusion (decreased or absent fremitus). (LeBlond RF, Brown DD, DeGowin RL. DeGowin's Diagnostic Examination, 9e, Chapter 8, The Chest: Chest Wall, Pulmonary, and Cardiovascular Systems; The Breasts)
Q 48.10: Which of the following is the most common childhood nutritional disorder in the United States? A binge eating disorder B folate deficiency C obesity D rickets
The Correct Answer is: C Obesity is the number one nutritional disorder in children in the United States. In 2004, 17% of American children aged between 9 and 19 were considered obese. Risk factors for obesity include other obese family members and infants born to diabetic mothers. Associated environmental factors include sedentary lifestyle, total caloric intake, television watching, and computer games. All are considered contributory factors in childhood obesity. Binge eating disorder is a relatively new eating disorder category. It is most frequent in overweight or obese individuals. This disorder includes recurrent episodes of binge eating (eating more than most individuals would in a 2-hour period) and a sense of lack of control over the impulse to eat, marked distress over the episode at least 2 days a week, and is not associated with regular compensatory activity such as purging or fasting. Folate deficiency anemia (megaloblastic) can occur in infants within a few weeks after birth. This deficiency may be a result of malabsorption, low dietary intake such as with goat's milk or home-prepared formulas that have been sterilized by heating, or formulas based on pasteurized milk. Infants who are breastfed or given supplemented cows' milk formulas do not have a problem with folate deficiency. In children, rickets is most commonly a result of poor dietary intake of vitamin D and inadequate exposure to direct sunlight. Vitamin D sources include milk, cheese, and baby formula. Vitamin D in humans is produced by activation of its inactive precursors in the skin after exposure to ultraviolet light.
Q 98.42: A patient presents with abdominal distension associated with nausea and vomiting. Which of the following findings is consistent with a paralytic ileus? A Crampy abdominal pain B Hyperactive bowel sounds C Obstipation and failure to pass flatus D Gas in small intestine only on KUB (kidney, ureter, bladder)
The Correct Answer is: C Obstipation and failure to pass flatus are actually symptoms of both paralytic ileus and a small bowel obstruction (SBO). However, patients with a paralytic ileus usually have minimal abdominal pain and hypoactive or absent bowel sounds due to hypomotility. Patients with an SBO will have crampy abdominal pain and increased bowel sounds with high-pitched sounds and rushes due to increased peristalsis. Plain films in paralytic ileus will show gas throughout the small and the large bowel on plain films as opposed to air confined to the small intestine only in SBO.
Q 97.8: A patient presents with abdominal distension associated with nausea and vomiting. Which of the following findings is consistent with a paralytic ileus? A Crampy abdominal pain B Hyperactive bowel sounds C Obstipation and failure to pass flatus D Gas in small intestine only on KUB (kidney, ureter, bladder)
The Correct Answer is: C Obstipation and failure to pass flatus are actually symptoms of both paralytic ileus and a small bowel obstruction (SBO). However, patients with a paralytic ileus usually have minimal abdominal pain and hypoactive or absent bowel sounds due to hypomotility. Patients with an SBO will have crampy abdominal pain and increased bowel sounds with high-pitched sounds and rushes due to increased peristalsis. Plain films in paralytic ileus will show gas throughout the small and the large bowel on plain films as opposed to air confined to the small intestine only in SBO. (Mellinger, 2006, pp. 297-299) Mellinger JD , Macfadyen BV , Mercer DW , et al. Small intestine and appendix. In: Lawrence PF, ed. Essentials of General Surgery. 4th ed. Philadelphia, PA: Lippincott Williams & Wilkins; 2006.
Q 66.3: A 52-year-old female is taking daily ibuprofen due to severe degenerative joint disease. She is concerned about developing an ulcer, but refuses to stop the ibuprofen. Which of the following medications is approved for the prevention of NSAID-induced gastric and duodenal ulcers, and should be recommended to this patient? A Ranitidine B Ondansetron C Omeprazole D Sucralfate E Dicyclomine
The Correct Answer is: C Omeprazole is a proton pump inhibitor, and is the only medication listed that is indicated for the prevention of NSAID induced ulcers. Ranitidine is an H2 antagonist and is indicated for active duodenal or benign gastric ulcers and GERD. Ondansetron is a 5-HT3 receptor antagonist and is indicated in the prevention of nausea and vomiting. Sucralfate is approved for treatment of an active duodenal ulcer and maintaining healed ulcers. Dicyclomine is an anticholinergic, and is indicated for the treatment of irritable bowel syndrome.
Q 106.2: Point of care (POC) testing has significant advantages and disadvantages. Which of the following is most correct regarding point of care testing? A It has not been proven to streamline workflow and reduce patient call-backs. B It has not been proven to decrease global costs. C It has been proven that more patients reach target "goals" and therefore improve patient outcomes with POC testing. D It has not been proven to reduce untoward outcomes. E Costs of POC testing generally preclude its use except in academic medical centers.
The Correct Answer is: C POC testing has a multitude of advantages including streamlining of work flow, immediate results precluding the need for patient call-backs to discuss results, allowing immediate interpretation, communication to patients, and patient education. This results in more patients reaching target goals and timely patient education with improved outcomes. Although more expensive, POC testing can reduce global costs if implemented correctly. (Kost, 2002, pp. 3-12)
Q 25.2: A 61-year old female patient who is a smoker is undergoing a routine physical examination in your family practice. She is otherwise asymptomatic but her urinalysis reveals microscopic hematuria. Your next definitive step would include which of the following? A Refer her for a spiral CT scan of the kidneys to ascertain where her renal calculi may be and to rule out hydronephrosis. B Repeat her urinalysis in the morning after asking the patient to hydrate with at least eight glasses of water in the interim. C Send her urine for cytology and refer her to a urologist to rule out bladder cancer. D Send her urine for a C&S and depending upon the result, start her on antibiotics E Start a course of levofloxacin to resolve her occult urinary tract infection.
The Correct Answer is: C Painless hematuria must always include bladder cancer in the differential diagnosis. Without another reasonable explanation—something that is not the case in this patient—bladder cancer must be ruled out, beginning with cytology and subsequent referral to an urologist.
Q 98.68: A 63-year-old white male is seen in the ambulatory outpatient clinic with complaints of midepigastric pain, weight loss, and jaundice. On examination, he is jaundiced and his sclerae are icteric. On palpation of the abdomen, you find a distended nontender gallbladder. Which of the following is the most likely diagnosis? A Gastric carcinoma B Chronic pancreatitis C Pancreatic carcinoma D Choledocholithiasis
The Correct Answer is: C Pancreatic carcinoma presents with weight loss, jaundice, and midepigastric pain. A palpable, nontender gallbladder (Courvoisier sign) is more often associated with a pancreatic malignancy than cholelithiasis, especially if the tumor is in the head of the pancreas. In acute cholecystitis, the obstruction in the cystic duct is associated with inflammation, resulting in a tender gallbladder on palpation of the right upper quadrant (Murphy sign); obstruction of the common bile duct in choledocholithiasis will result in jaundice but not weight loss. Gastric carcinoma will present with midepigastric pain and weight loss but not jaundice or a palpable gallbladder. Midepigastric pain is the most common symptom seen in chronic pancreatitis, and weight loss may be seen in association with malabsorption secondary to exocrine insufficiency.
Q 69.2: A 63-year-old white male is seen in the ambulatory outpatient clinic with complaints of midepigastric pain, weight loss, and jaundice. On examination, he is jaundiced and his sclerae are icteric. On palpation of the abdomen, you find a distended nontender gallbladder. Which of the following is the most likely diagnosis? A Gastric carcinoma B Chronic pancreatitis C Pancreatic carcinoma D Choledocholithiasis
The Correct Answer is: C Pancreatic carcinoma presents with weight loss, jaundice, and midepigastric pain. A palpable, nontender gallbladder (Courvoisier sign) is more often associated with a pancreatic malignancy than cholelithiasis, especially if the tumor is in the head of the pancreas. In acute cholecystitis, the obstruction in the cystic duct is associated with inflammation, resulting in a tender gallbladder on palpation of the right upper quadrant (Murphy sign); obstruction of the common bile duct in choledocholithiasis will result in jaundice but not weight loss. Gastric carcinoma will present with midepigastric pain and weight loss but not jaundice or a palpable gallbladder. Midepigastric pain is the most common symptom seen in chronic pancreatitis, and weight loss may be seen in association with malabsorption secondary to exocrine insufficiency. (Danziger, 2006, pp. 343-345; Paige, 2006, p. 264; Sharp, 2006, pp. 361-362) Danzinger RG , Nauta R , Park J. Biliary tract. In: Lawrence PF, ed. Essentials of General Surgery. 4th ed. Philadelphia, PA: Lippincott Williams & Wilkins; 2006. Paige J , O'Leary JP. Stomach and duodenum. In: Lawrence PF, ed. Essentials of General Surgery. 4th ed. Philadelphia, PA: Lippincott Williams & Wilkins; 2006. Sharp KW , Goldin SB , Lomis KD. Pancreas. In: Lawrence PF, ed. Essentials of General Surgery. 4th ed. Philadelphia, PA: Lippincott Williams & Wilkins; 2006.
Q 82.8: Which of the following is one standard of care to treat a first-time deep vein thrombosis (DVT) without pulmonary embolism (PE)? A Begin the patient on warfarin 5 mg PO once daily and check an INR in five days and adjust to therapeutic levels continuing warfarin for six months. B Begin the patient on enoxaparin 1 mg/kg SQ QD while also starting warfarin 5 mg PO once daily and check INR in five days and adjust to therapeutic levels continuing warfarin for six months and enoxaparin until therapeutic on the warfarin. C Begin the patient on enoxaparin 2 mg/kg SQ QD while also starting warfarin 5 mg PO once daily and check INR in five days and adjust to therapeutic levels continuing warfarin for six months and enoxaparin until therapeutic on the warfarin. D Begin the patient on heparin sodium IV and monitor and adjust levels to achieve therapeutic levels Q six hours and transition to warfarin as soon as possible. E Initiate a hypercoaguability work up and treat accordingly.
The Correct Answer is: C Patients with deep vein thrombosis (DVT) without pulmonary embolism (PE) do not necessarily require hospitalization. Outpatient therapy would include treatment with a low molecular weight heparinoid (LMWH), such as enoxaparin, subcutaneously while the patient becomes therapeutic on oral warfarin. The LMWH can be discontinued once therapeutic on warfarin, with continued monitoring of INRs for six months before considering discontinuation. (McPhee and Papadakis, 2011, Chapter 9)
A 21 year-old female presents to the emergency department after having a cast applied to her right arm earlier that day. Approximately one hour ago she began having extreme 10/10 pain in her right arm and is in visible distress. When considering the diagnosis of compartment syndrome, permanent damage to the muscle begins after how many hours of ischemia? A >2 hours B >6 hours C >8 hours D >12 hours E >24 hours
The Correct Answer is: C Permanent damage results after >8 hours (C) of ischemia. Nerves begin to lose conduction within 2 hours of onset of elevated pressures. Neurapraxia can occur within 4 hours, and irreversible damage occurs 8 hours after elevated pressures. Functional impairment is unlikely when compartment syndrome is diagnosed and treated within 6 hours of its onset. While deficits can occur prior to 8 hours, permanent damage is usually not seen before 8 hours (A and B). After 12 or 24 hours, permanent damage has already resulted in most cases (D and E). The key to early detection of compartment syndrome is a high index of clinical suspicion.
Q 112.4: A 38-year-old man presents to the emergency department experiencing a severe headache and heart palpitations. He appears to be anxious and perspiring heavily. On exam, he is found to be tachycardic and his blood pressure is 158/102 mm Hg. His urine catecholamines are increased. If imaging were performed, what is the most likely location where a lesion would be found? A pituitary gland B liver C adrenal gland D testicle E kidney
The Correct Answer is: C Pheochromocytomas produce, store, and secrete catecholamines. They are usually derived from the adrenal medulla, although they may be found in other locations. (Fitzgerald, 2009, pp. 1031-1034) Fitzgerald PA. Endocrine diseases. In: McPhee SJ , Papadakis MA, eds. Current Medical Diagnosis and Treatment. 48th ed. New York, NY: McGraw-Hill; 2009.
Q 45.7: A 2-year-old male presents with a 10-day history of fever, cough, and decreased appetite and fluid intake. He is normally healthy. On examination, the child appears ill, has a temperature of 102.2 0 F, a pulse rate of 122, and a respiratory rate of 36. On auscultation of the lungs there are rhonchi heard on the right lung fields, as well as a small amount of wheeze. A chest x-ray is ordered, which reveals the presence of pneumatoceles. Based on these findings, what is the most likely pathogen causing this patient's infection? A Streptococcus pneumoniae B Hemophilus influenza C Staphylococcal aureus D Pseudomonas aeurginosa E Chlamydia pneumoniae
The Correct Answer is: C Pneumatoceles, pyopneumothorax, and empyemas are frequently encountered in pediatric Staphylococcal aureus pneumonias.
Q 72.1: What is the most common hematologic finding in a patient with pulmonary hypertension? A Anemia B Thrombocytopenia C Polycythemia D Leukocytosis E Elevated mean corpuscular volume (MCV)
The Correct Answer is: C Polycythemia is the most common finding. Hematocrits that are >60% usually require phlebotomy, to reduce the numbers and prevent a hypercoagulable state. (Chesnutt MS, Prendergast TJ. Current Medical Diagnosis and Treatment, 2011, Chapter 9, Pulmonary Disorders)
Q 97.111: A 27-year-old female is 8 weeks postpartum with her first child and has been exclusively nursing since discharge at the hospital. She has a 5-day history of engorgement in her right breast, which is red, tender, and feels warm to the touch. She states she is feverish but has not taken her temperature. On physical examination you see the breast as shown below. Which of the following is the most likely diagnosis? Source: McPhee SJ, Papadakis, MA: Current Medical Diagnosis and Treatment 2011, 50 th Edition: http://www.accessmedicine.com A Galactorrhea B Fibroadenoma C Acute mastitis D Lobular breast cancer E Gynecomastia
The Correct Answer is: C Postpartum mastitis occurs sporadically in mothers that are nursing and usually develops after hospital discharge. The inflammation is usually unilateral and begins with an engorged breast or sore nipple. Frequently, mastitis begins within 3 months postpartum and affects first time nursing mothers more often. Cellulitis is usually seen in the affected breast and redness, tenderness, local warmth, and fevers and chills are common complaints. Fibroadenoma (B) and galactorrhea (A) do not look like this clinical picture. Gynecomastia (E) affects males. Inflammatory carcinoma can in rare cases be mistaken for mastitis. Lobular carcinoma (D) would not present this way.
Q 98.33: A 48-year-old patient is monitored post-operatively for appropriate heparin dosing, and his aPTT is twice the upper limit of the therapeutic range. Which of the following can be administered to reverse the effect of his heparin overdose? A Enoxaparin B Fresh frozen plasma C Protamine sulfate D Recombinant factor VII E Vitamin K
The Correct Answer is: C Protamine sulfate is the reversal agent for heparin, but administration carries a significant risk of thrombosis and allergic reaction, and should be used cautiously when the benefits outweigh the risks. Enoxaparin (A) should not be administered because it would be additive therapy to heparin. Fresh frozen plasma (B), recombinant factor VII (D), and vitamin K (E) are used in treating warfarin overdose.
Q 118.6: A 58-year-old man with a medical history of gouty arthritis presents with a red, swollen joint at the base of the great toe. His diet for the past 7 to 10 days consisted of large quantities of organ meats and fresh seafood. The increased metabolism of which of the following most likely contributed to the patient's symptoms? A amino acids B polysaccharides C purines D pyrimidines E triglycerides
The Correct Answer is: C Purines are normally metabolized into uric acid by the liver and can be found in high amounts in several foods, including organ meats, seafood, beans, peas, and many others. Higher levels of meat and seafood consumption are associated with an increased risk of gout because of the hyperuricemia that can occur via purine metabolism. (Hellman and Imboden, 2008, pp. 706-708) Hellman DB , Imboden JB Jr. Arthritis and musculoskeletal disorders. In: McPhee SJ , Papadakis MA, eds. Current Medical Diagnosis & Treatment. New York, NY: McGraw-Hill Medical; 2008.
Q 18.10: A 64-year-old postmenopausal female presents for an annual examination. She is 5'0" and weighs 92 pounds; compared to her examination 3 years ago she has lost an inch in height. After performing a dual-energy x-ray absorptiometry (DXA) scan the diagnosis of osteoporosis is confirmed and she is started on appropriate treatment. How often should she be recommended to follow-up with a DXA bone density scan? A Every six months B Every year C Every two years D Every ten years E No need to follow-up
The Correct Answer is: C Recommendations for patients diagnosed with osteoporosis without a fracture is every 2-3 years (C). This guideline is approved based on age, risk factors, or previous fractures. A post-menopausal woman at age 64 without a presenting fracture can be followed every 2-3 years with bone density imaging (DXA scan). Medications and other treatment options may need earlier or more frequent follow-up depending on the treatment plan. DXA is quite accurate and delivers negligible radiation. Performing a DXA at intervals less than two years has not shown of benefit (A and B). Every 10 years is too far of a time frame to follow osteoporosis (D). Follow-up is needed to see any changes in the bone density (E).
Q 97.59: A 20-year-old woman was just told by her new sexual partner that she needed to be checked for a sexually transmitted infection because he has developed dysuria and a profuse urethral discharge. She herself has had a subjective fever for the past two days, some nausea but no vomiting, diffuse lower abdominal pain, and a severe backache. On examination, she has a temperature of 100.5˚F, hypoactive bowel sounds, bilateral lower abdominal quadrant tenderness, a profuse mucopurulent cervical discharge and pronounced cervical motion tenderness. Serum pregnancy testing is negative. She is given an injection of ceftriaxone and a prescription for doxycycline for 14 days and an appointment for follow up the next day. Under which of the following conditions should metronidazole be added to her regimen? A If she does not appear improved by the following morning B If she develops vomiting or diarrhea C If she has a probable tubo-ovarian abscess D If she has an allergic reaction to the doxycycline E If she has had more than one partner in the past month
The Correct Answer is: C Recommended regimens for treatment of pelvic inflammatory disease include ceftriaxone or another parenteral third-generation cephalosporin and doxycycline or cefoxitin, probenecid, and doxycycline. Metronidazole (or clindamycin) should be added to either regimen if a tubo-ovarian abscess is present. If being treated as an outpatient and she has not improved in 3 days (A), or if she develops vomiting (B), she should be hospitalized rather than given metronidazole. Metronidazole is not a satisfactory substitute for doxycycline (D). The patient's condition rather than the number of sexual partners (E) determines the appropriate therapy.
Q 13.4: A 16-year-old girl is referred for a sports physical. Her blood pressure is 170/92 mm Hg. Urinalysis (UA) reveals 2+ protein. The girl's mother reports multiple episodes of urinary tract infections (UTIs) throughout childhood that were never investigated. The most likely diagnosis is A obstructive uropathy B orthostatic proteinuria C chronic reflux nephropathy D nephrotic syndrome E exercise-induced proteinuria
The Correct Answer is: C Retrograde flow of urine from the bladder damages the renal interstitium, causing inflammation and fibrosis. If untreated, irreversible damage to the kidneys will occur. Because this is a tubulointerstitial process, the urinalysis will be negative for protein in the early stages of damage. Most damage is done before age 5, but if undetected, glomerular damage will occur and protein will appear in the urine eventually. Hypertension develops as the GFR decreases.
Q 75.1: A patient presents with mild dyspnea, increased cough, and rhinorrhea. On physical exam, you auscultate low-pitched, sonorous, and adventitious sounds over the bilateral upper lung fields, which are suggestive of secretions. Which of the following terms is defined by these findings? A Crackles B Rales C Rhonchi D Vesicular breath sounds E Wheezes
The Correct Answer is: C Rhonchi are defined as low-pitched, often harsh breath sounds, with increased secretions and inflammation. Rhonchi due to secretions may improve with coughing. Crackles, also known as rales, are due to an increase of fluid shifting from the intravascular space into the alveoli, and are often described as brief, nonmusical sounds with popping. Wheezes, which are high-pitched, musical sounds, are due to the narrowing of the airway related to mucosal edema, secretions, and bronchospasm. Vesicular breath sounds are normal lung sounds found over the periphery. (McPhee SJ, Papadakis MA. Current Medical Diagnosis & Treatment 2011, Chapter 9, Pulmonary Disorders)
Q 20.6: A 23 year-old woman presents to the clinic complaining of abrupt onset of severe pallor, fatigue and dyspnea on exertion. Initial CBC reveals a hemoglobin of 9.6 mg/dL and an MCV of 87 fL. Which of the following best describes the cause of the abnormality pictured in the the patient's peripheral smear? (note photo taken from figure 29-13 in Harrison's) A Absent or non-functioning spleen B Failure of nuclear maturation C Intravascular hemolysis D Lead intoxication E Presence of uremia
The Correct Answer is: C Schizocytes are present on the peripheral smear and result from intravascular hemolysis. Howell-Jolly bodies result from the lack of removal of nuclear material due to an absent or non-functioning spleen (A). Failure of nuclear maturation (B) produces macrocytosis, lead intoxication (D) and thalassemia cause basophilic stippling, and uremia (E) is associated with Burr cells.
Q 73.3: A 25 year old female presents with multiple irregular brown macules on her upper back. They are asymptomatic. She has worked as a lifeguard for the past 7 years. She reports a couple of blistering sunburns and admits to maintaining a "healthy tan". What is the most likely diagnosis? A Pityriasis alba B Basal cell nevus syndrome C Solar lentigines D Metastatic melanoma
The Correct Answer is: C Solar lentigines are a result of skin damage from the sun. Unlike ephiledes, they do not fade once exposure to the sun has stopped. There is no risk of malignancy associated with solar lentigines. (Wolff & Johnson, p264-265) (Fig 14-20, Wolff K, Johnson RA: Fitzpatrick's Color Atlas & Synopsis of Clinical Dermatology, 6 th ed: http://www/accessmedicine.com )
Q 106.15: Large numbers of epithelial cells on urine sediment indicate A UTI B acute tubular necrosis C sample contamination D vaginitis in women E prostatitis in men
The Correct Answer is: C Squamous epithelial cells line the distal portion of the urethra in men and the entire urethra in women. They appear in the urine due to inadequate cleaning of the external urinary meatus prior to obtaining the sample and indicate that the sample is contaminated. In women, the source is usually vaginal/perineal. Uncircumcised men commonly have squamous epithelial cells in the urine sample. (McBride, 1998, p. 103) McBride L. Textbook of Urinalysis and Body Fluids. Philadelphia, PA: Lippincott Williams & Wilkins; 1998.
Q 30.4: A 14-year-old boy is brought to your primary care office as a new patient. He is new to the area and having a little bit of difficulty adjusting to school. He denies feeling depressed or being bullied. You wish to screen him for suicidality. Which is the best first question to ask? A "Do you have a gun at home?" B "Do you have friends who have committed suicide?" C "Do you sometimes feel life is not worth living?" D "Do you sometimes want to jump off a bridge" E "Have you considered what your death might do to those around you?"
The Correct Answer is: C Suicide risk should be routinely assessed. This is best done by asking a general question such as in (C), then following up with more specific questions about means, such as (A), or plans, such as (D). Having friends who have committed suicide (B) may increase risk, but is not helpful as a screening question. Questions should focus on the patient, not on family or friends as in (E).
Q 13.7: A 43 year-old woman with a past medical history of HIV infection presents to the clinic due to a 3-month history of dry, non-productive cough and progressive dyspnea. The patient is subsequently prescribed trimethoprim-sulfamethazole. What is the mechanism of action of this sulfamethazole? A Binds to the 30S ribosomal subunit to inhibit protein synthesis B Binds to the 50S ribosomal subunit to inhibit protein synthesis C Inhibits dihydropteroate synthase and folate production D Inhibits DNA replication by binding to DNA gyrase and topoisomerase IV E Inhibits the transpeptidation reaction
The Correct Answer is: C Sulfamethoxazole inhibits dihydropteroate synthase and folate production (C). Tetracycline binds to the 30S ribosomal subunit to inhibit protein synthesis (A). Macrolide antibiotics bind to the 50S ribosomal subunit to inhibit protein synthesis (B). Ciprofloxacin inhibits DNA replication by binding to DNA gyrase and topoisomerase IV (D). Beta lactam antibiotics inhibit the transpeptidation reaction leading to cell wall destruction (E).
Q 98.28: Which of the following is a major contraindication for surgical resection of a lung carcinoma? A Chest wall invasion B Non-malignant pleural effusion C Superior vena cava syndrome D Unilateral endobronchial tumor E Vagus nerve involvement
The Correct Answer is: C Surgical resection of lung carcinoma is contraindicated in cases of superior vena cava syndrome, extrathoracic metastases, heart, pericardial or great vessel involvement, recurrent laryngeal or phrenic nerve involvement, esophageal or carina involvement, malignant effusion, or contralateral mediastinal lymph nodes. Other contraindications are patient and staging dependent.
Q 104.6: An adult with a high risk for bacterial endocarditis is scheduled for a dental extraction. The patient has a history of penicillin allergy. Which of the following is an appropriate oral prophylactic drug to give this patient? A amoxicillin B vancomycin C clindamycin D doxycycline E gentamicin
The Correct Answer is: C The American Heart Association recommends that patients who are at moderate to high risk for bacterial endocarditis receive antibiotic prophylaxis prior to undergoing oral/dental, respiratory tract, or esophageal procedures. Amoxicillin 2.0 g orally 1 hour before the procedure is the standard regimen. Patients who have a history of amoxicillin/penicillin allergy may be given clindamycin, cephalexin, azithromycin, or clarithromycin. For adults, clindamycin is given at a dose of 600 mg po 1 hour before the procedure. (Schwartz and Chambers, 2009, pp. 1272-1273) Schwartz BS , Chambers HF. Bacterial and chlamydial infections. In: McPhee SJ , Papadakis MA, eds. Current Medical Diagnosis and Treatment. 48th ed. New York, NY: McGraw-Hill; 2009.
Q 8.3: Radiculopathy due to nerve root compression occurs most commonly at which nerve root within the brachial plexus? A C5 B C6 C C7 D C8 E T1
The Correct Answer is: C The C7 nerve root is affected the most often (approximately 45-60%). This radiculopathy can result from foraminal encroachment of the spinal nerve, cervical disk herniation, tumor, and multiple sclerosis. C7 radiculopathy can present with weakness in the triceps, which cause elbow extension, and finger flexion and extension. C6 is another common site of radiculopathy. C6 radiculopathy can present with weakness in the biceps, brachioradialis, and wrist extensor muscles. Cervical radiculopathy at the C5, C8, and T1 are less common, but still possible. C5 radiculopathy can present with deltoid and biceps muscle weakness. C8 radiculopathy can present with finger flexor weakness and T1 radiculopathy with finger abduction weakness
Q 8.10: An otherwise healthy 48-year-old male patient presents to your family practice clinic for a complete physical examination. He takes no medications. His vital signs, CBC, CMP, and TSH are normal. His fasting lipid panel reveals a total cholesterol of 280 mg/dL, LDL-C of 190 mg/dL. HDL-C of 38 mg/dL, and triglycerides of 151 mg/dL. What is your next step in his management? A Counsel him on target life style changes and recheck his lipid panel in three months and begin therapy if not to goal at that time. B Redraw his fasting lipid panel today, counsel him on target life style changes and begin therapy immediately if his follow up lipid panel is not to goal. C Counsel him on target life style changes, begin therapy with a statin at this time, and recheck his lipid panel in three months and adjust therapy. D Counsel him on target life style changes, begin therapy with a nicotinic acid derivative at this time, and recheck his lipid panel in three months and adjust therapy. E Counsel him on target life style changes, begin therapy with a fenofibrate at this time, and recheck his lipid panel in three months and adjust therapy
The Correct Answer is: C The National Cholesterol Education Program (NCEP) Adult Treatment Panel III (ATP-III) and subsequent revisions generally recommends that healthy male patients reach the following goals for cholesterol: low-density lipoprotein (LDL) of less than 100 mg/dL, high-density lipoprotein (HDL) of greater than 40 mg/dl in men and greater than 45 mg/dL in women, and triglycerides of less than 150 mg/dL. While counseling this patient on lifestyle changes is also important, it would be almost impossible for this patient to reduce his levels to goal without medication. A better strategy would be to combine lifestyle changes and initiate medication concurrently. Statins remain the first-line therapy for treatment of dyslipidemias.
Q 109.8: Which lobe is most affected by infection of tuberculosis? A Left lower lobe B Left upper lobe C Right upper lobe D Right lower lobe E Right middle lobe
The Correct Answer is: C The apical sections of the lung fields are the most typical areas where tuberculosis occurs. The right side is more prevalent than the left. Other findings on chest x-rays can include pleural effusions, Gohn lesions (calcified primary focus), and cavitation. (Chesnutt MS, Prendergast TJ. Current Medical Diagnosis and Treatment, 2011, Chapter 9, Pulmonary Disorders)
Q 1.3: A 66-year-old female is admitted to the hospital with a new onset of atrial fibrillation. Her past medical history has hypertension, diabetes mellitus type II, hypercholesterolemia, and rheumatoid arthritis. Her vital signs show a blood pressure of 136/78, pulse of 89, respirations 18, and oxygen saturation of 96%. Her lungs are clear to auscultation, and an irregularly irregular rhythm is appreciated. There is no edema on peripheral examination. Given this clinical scenario, what is the most appropriate test in determining if an intracardiac thrombus is present? A Transthoracic echocardiography B Electrocardiogram C Transesophageal echocardiography D Cardiac catheterization E Magnetic resonance arteriography
The Correct Answer is: C The best and most appropriate test to evaluate for an intra-atrial thrombus is the transesophageal echocardiogram (C). This test has the better sensitivity and specificity when compared to the transthoracic echocardiogram (A). Electrocardiograms (B), cardiac catheterizations (D), and magnetic resonance imaging (E) do not give any specific information related to the evaluation of an intracardiac thrombus.
Q 39.22: A 1-year-old female is having a 2-day history of fever (102 0 F oral), rhinorrhea, and dry cough, with a decreased appetite. The mother states that her daughter has been less active, and her fluid intake has decreased for her age. On exam, the child is non-toxic appearing, has a rectal temperature of 100.2 0 F, and has nasal flaring and a respiratory rate of 45, rhinorrhea, moist mucous membranes, and a minimal wheeze heard bilaterally. Her chest x-ray has no specific findings. What is the initial treatment of choice for this patient with these symptoms? A Antibiotics B Oxygen therapy C Supportive care D Antiviral medications E Plasmaphoresis
The Correct Answer is: C The choice for treatment of acute bronchiolitis is supportive care. Oxygen therapy is only reserved for those patients who are hypoxic, and antiviral medications have not proven to be effective in shortening or eradicating the infection.
Q 115.20: A 38-year-old female is at a follow-up visit for hypertension. She is accompanied by her husband. During the clinical visit the patient is noted to be passive, letting her husband do most of the talking. She also appears to lack confidence and self-esteem. The husband ends up making the decisions, and she is over-accepting of his dominance. What is the most likely classification of this personality disorder? A Avoidant B Borderline C Dependent D Histrionic E Schizotypal
The Correct Answer is: C The clinical findings of dependent personality disorder are the most consistent with the given clinical scenario. Avoidant personality disorder presents clinically as someone who fears rejection, overreacts to rejection and failure, has poor social endeavors, and low self-esteem. Borderline personality disorder clinical findings include impulsiveness, unstable and intense interpersonal relationships, lack of self control, suicidal ideations, aggressive behavior, and a high drug abuse rate. Clinical findings of histrionic personality disorder include being dependent, immature, seductive, egocentric, vain, and emotionally labile. Schizotypal clinical findings include being superstitious, socially isolated, and suspicious, and having limited personality ability, odd speech and eccentric behaviors. (McPhee SJ, Papadakis MA. Current Medical Diagnosis & Treatment, 2010, p. 951)
Q 98.23: A 74-year-old woman presented with the new onset of seizures. An MRI with gadolinium showed a parasagittal mass with homogenous enhancement and a "dural tail." What is the appropriate management of this patient? A Radiation therapy B Chemotherapy C Surgical resection D Surgical resection with chemotherapy
The Correct Answer is: C The clinical presentation is consistent with a meningioma. Meningiomas are commonly located in the parasagittal region, the convexity of the brain, sphenoid ridge, or posterior fossa. Radiographic features on MRI include homogenous enhancement and evidence of a "dural tail" indicating the origin of the tumor. Since meningiomas are a benign tumor, the primary treatment is surgical removal. In the event of a subtotal resection or if the meningioma is found to be malignant, surgical resection is followed by radiation therapy.
Q 97.89: A 33-year-old woman presents with an itchy vaginal discharge for the past 2 days. She has been healthy other than a recent sinus infection for which she took a 10-day course of amoxicillin. Her husband is her only sexual partner and he has no symptoms. On examination, the vulva is noted to be slightly erythematous and swollen with some evidence of excoriation. Discharge is white and clumpy. Provided the most likely diagnosis is confirmed on microscopy, first-line therapy is A metronidazole 500 mg i po bid for 1 week B metronidazole 500 mg 4 tablets po at HS × 1 night C fluconazole 150 mg i po × 1 day D rocephin 250 mg IM × 1 dose
The Correct Answer is: C The clinical presentation is consistent with vulvovaginal candidiasis. The recent oral antibiotic use increased her risk for developing the infection. The white clumpy discharge and relatively benign bimanual examination support the diagnosis, which is confirmed by 10% potassium hydroxide wet mount of the secretions. Treatment for an uncomplicated case may include topical or oral antifungals. Oral fluconazole in the one dose regimen is effective, convenient, and likely to increase compliance. The metronidazole regimens are appropriate for bacterial vaginosis and trichomoniasis, respectively. Rocephin is an option for gonococcal infection and would likely worsen the candidiasis.
Q 32.11: A 33-year-old woman presents with an itchy vaginal discharge for the past 2 days. She has been healthy other than a recent sinus infection for which she took a 10-day course of amoxicillin. Her husband is her only sexual partner and he has no symptoms. On examination, the vulva is noted to be slightly erythematous and swollen with some evidence of excoriation. Discharge is white and clumpy. Provided the most likely diagnosis is confirmed on microscopy, first-line therapy is A metronidazole 500 mg i po bid for 1 week B metronidazole 500 mg 4 tablets po at HS × 1 night C fluconazole 150 mg i po × 1 day D rocephin 250 mg IM × 1 dose
The Correct Answer is: C The clinical presentation is consistent with vulvovaginal candidiasis. The recent oral antibiotic use increased her risk for developing the infection. The white clumpy discharge and relatively benign bimanual examination support the diagnosis, which is confirmed by 10% potassium hydroxide wet mount of the secretions. Treatment for an uncomplicated case may include topical or oral antifungals. Oral fluconazole in the one dose regimen is effective, convenient, and likely to increase compliance. The metronidazole regimens are appropriate for bacterial vaginosis and trichomoniasis, respectively. Rocephin is an option for gonococcal infection and would likely worsen the candidiasis. (Nyirjesy, 2008, p. 642) Nyirjesy P. Vulvovaginal candidiasis and bacterial vaginosis.
Q 116.6: A 71 year old female presents with complaint of a severe headache for 2 days. The patient denies a history of headaches in the past. She complains of a 2 week periods of morning shoulder and pelvic stiffness. There also a history of jaw pain when she chews her food. The past medical history is remarkable for well controlled hypertension, hyperlipidemia. On physical exam there is a markedly tender scalp and left temporal artery. The neurologic exam is normal. Besides a temporal artery biopsy what other diagnostic test is most indicated? A CBC B Computed tomography of the head C Erythrocyte Sedimentation Rate D Electrolytes and renal function E International normalized ratio
The Correct Answer is: C The clinical scenario suggests giant cell arteritis (GCA) or temporal arteritis. An erythrocyte sedimentation rate( ESR) or C-reactive protein( CRP) can aid in the diagnosis of GCA when evaluating an inflammatory vasculitis such as GCA. Treatment is based on history and physical and not the result of the ESR. Temporal artery biopsy is the gold standard for diagnosis. (McPhee et al., 2011, Chapter 20)
Q 40.8: Which of the following childhood exanthems is characterized by a 10-day incubation period followed by 3 days of fever, runny nose, and conjunctivitis giving way to a maculopapular rash starting on the head, progressing to the trunk, and accompanied by white spots on the buccal mucosa? A herpes simplex infection B coxsackie virus C measles D rubella
The Correct Answer is: C The description best fits that of measles. It has a 7- to 10-day incubation period followed by 3 days of coryza, fever, and conjunctival involvement. The prodrome dissipates as the characteristic rash develops first on the head and face and then the trunk. Koplick spots are the pathognomonic white spots that occur on the buccal mucosa in measles' infection. Herpes simplex infection can present as painful vesicles on the mouth and lips (HSV-1) or genitalia (HSV-2). It is preceded by fever and a tingling or burning sensation at the site where the vesicle will develop. Coxsackievirus causes hand, foot, and mouth disease with lesion distribution in those three areas. Rubella has a longer incubation period (2 to 2.5 weeks) and is less contagious than measles. It is sometimes asymptomatic or produces a milder course than measles.
Q 117.10: A 78-year-old woman presents to the office complaining of a constant left-sided headache for 2 months. She has tried various over-the-counter (OTC) medications without relief. The patient admits to vision loss of her left eye last night for 10 minutes. The patient states that her vision then returned to normal. She denies pain in her eye. On review of systems, she relates several months of muscle aches and weight loss. On physical examination, she is found to have a tender, nonpulsatile superficial temporal artery. Her sedimentation rate is elevated at 90 mm/h. What is the next, most appropriate step in managing this patient? A stat MRI/MRA of the brain and cranial vessels B aspirin therapy C high-dose prednisone D lumbar puncture E sumatriptan (Imitrex) injection
The Correct Answer is: C The diagnosis is temporal arteritis. This is an arteritis of the temporal branch of the external carotid artery characterized by unilateral or bilateral headaches that may be localized to a tender temporal artery. The temporal artery may be thickened and tender and may be thrombosed and nonpulsatile late in the disease. Many patients present with malaise and have anemia and a low-grade fever. Fifty percent of patients report generalized muscle aches consistent with polymyalgia rheumatica. The most severe complication of temporal arteritis is blindness resulting from thrombosis of the ophthalmic artery. In some cases, this may be preceded by previous episodes of amaurosis fugax before the blindness becomes irreversible. Once blindness occurs in one eye, it may be prevented in the other by initiating treatment. The diagnosis is based on recognizing the clinical picture and obtaining a temporal artery biopsy. Treatment should not be delayed pending the biopsy. Early treatment with prednisone may prevent irreversible blindness. The efficacy of treatment can be measured with serial sedimentation rates. MRI and MRA have no value in establishing the diagnosis of temporal arteritis. Antiplatelet therapy would not be inappropriate but is inadequate for this diagnosis. The potentially unilateral headache should not be confused with a migraine for which Imitrex therapy would be appropriate. In addition, lumbar puncture has no role in establishing this diagnosis. (Langford and Fauci, 2008, p. 2127) Langford CA , Fauci AS. The Vasculitis syndromes. In: Fauci AS , Braunwald E , Kasper DL, et al., eds. Harrison's Textbook of Medicine. 17th ed. New York, NY: McGraw-Hill; 2008.
Q 29.6: A 32-year-old female professional golfer presents with a 3-week history of pain along her thumb and down her wrist. She denies any trauma and states that it is aggravated with any movement of her wrist and thumb. Her physical examination is unremarkable. Considering the suspected diagnosis you perform the following examination: This test requires her to cup her thumb in a closed fist and ulnar deviate, which reproduces her pain. Which test was performed? A Apley's B Phalen's C Finkelstein's D Tinel's E Empty Can
The Correct Answer is: C The diagnosis of De Quervain tenosynovitis is supported by a history of pain in the location of the radial aspect of the wrist with a painful range of motion of the thumb and occasional local tenderness over the distal portion of the radial styloid. Further confirmation of the diagnosis may be provided by a positive Finkelstein test (C), in which the patient grasps the thumb in the palm of the hand and the examiner ulnar deviates the thumb and hand. This stretches the tendons over the radial styloid and produces sharp pain along the involved tendons. A Tinel's (D) and Phalen's (B) sign are positive in carpal tunnel syndrome. The Apley's compression test (A) is used to evaluate meniscus tears in the knee. The Empty Can Test (E) is used to assess the rotator cuff supraspinatus muscle.
Q 105.6: A 26-year-old female presents with a whitish coating on her tongue and lips. When you attempt to rub the plaque with a tongue depressor, a small amount of bleeding is noted from the oral mucosa. Her past medical history includes asthma. What diagnostic test would you perform to confirm your diagnosis? A Strep screen B Culture and sensitivity C Wet mount D Viral culture E Gram stain
The Correct Answer is: C The history and physical are consistent with oral thrush. A wet mount potassium hydroxide preparation will reveal fungal spores and nonseptated mycelia. A culture and sensitivity is used for suspected bacterial infections. A gram stain is used to identify whether a bacterial pathogen is gram positive or gram negative. (McPhee et al., 2011, Chapter 8)
Q 121.15: A 43-year-old male farmer from the Southwest United States has been working in a very contaminated barn with rodent feces for the last week. He presents to your office with complaints of fever, non-productive cough, malaise, and decreased appetite. His physical exam reveals a temperature of 102 0 F, pulse rate of 98, blood pressure of 98/62, and O 2 saturation of 93%. Lung sounds have diffuse crackles throughout, and the rest of the exam is unremarkable. Based on the history and exam findings, what is the most likely pathogen for this type of illness? A Influenza pneumonia B Varicella pneumonia C Hantavirus pneumonia D Streptococcal pneumoniae E Cytomegalovirus
The Correct Answer is: C The history of the patient being exposed to the rodent feces is a typical presentation of a pneumonia caused by the hantavirus. There is no treatment for this type of pneumonia, only supportive care. (Lee et al., Current Diagnosis and Treatment in Pulmonary Medicine, Section X, Pulmonary Lung Disease, Chapter 37, Viral and Atypical Pneumonia)
Q 28.6: A 56-year-old African American female is currently on heparin therapy for a new diagnosis of atrial fibrillation, and warfarin therapy is being started. She has a past history of hypertension, hypercholesterolemia, diabetes, and end stage renal disease. All of her chronic medical problems are stable, and the patient now has a rate controlled atrial fibrillation at 77 beats per minute. Given this clinical scenario, what is the most appropriate international normalized ratio (INR) to remain therapeutic while being administered warfarin? A 0-1.0 B 1.0-2.0 C 2.0-3.0 D 3.0-4.0 E >4.0
The Correct Answer is: C The ideal target INR for a patient with non-valvular atrial fibrillation while on warfarin is 2.5 (C). Data suggests that anticoagulation with an INR >2.0 (D and E) not only reduces the risk of ischemic stroke, it also shows a reduction in the severity of stroke. However, higher INR levels >3.0 increase a patient's risk of side effects of warfarin, the most common being bleeding. Both choices A and B would not adequately anticoagulate the patient, and thus still put the patient at risk for development of thrombus formation.
Q 97.29: A 46 year-old female has just been diagnosed with ovarian cancer. Where would you expect the ovarian neoplasm to most likely arise from? A Germ cell B Stromal cell C Epithelial cell A Metastatic tumor E Endometrial cell
The Correct Answer is: C The most common and most lethal of the ovarian neoplasms arise from the ovarian epithelium found both on the surface of the ovary and in subsurface locations. The ovarian epithelium generally is in good health and appears as a simple epithelium. With neoplastic transformation there undergoes metaplastic changes into what is termed Müllerian epithelium. The Müllerian epithelium has a variety of subtypes, each providing a specific phenotype of the tumor and may have a different clinical presentation. Epithelial tumors are the most common ovarian neoplasm and may be: benign (50%); malignant (33%); or borderline malignancy (16%). Tumors may also metastasize (D) to the ovary from the breast, gastric, pancreatic, and colon primary cancers, but is not as common. Germ cell tumors (A) are more similar to testicular tumors in males. Stromal tumors (B) arise from steroid hormone producing cells. Endometrial (E) is usually specific to the uterus.
Q 75.4: Which of the following agents is a significant cause of pill-induced esophagitis? A fluoxetine B omeprazole C ibuprofen D Vitamin D E ciprofloxacin
The Correct Answer is: C The most common causes of pill-induced esophagitis are nonsteroidal medications. Other commonly prescribed medications causing esophageal injury include slow release of potassium chloride, iron sulfate, quinine sulfate, and alendronate sodium. (McQuaid, 2009, pp. 520-521) McQuaid KR. Gastrointestinal disorders. In: McPhee SJ , Papadakis MA, eds. Current Medical Diagnosis and Treatment. 48th ed. New York, NY: McGraw-Hill; 2009.
Q 97.79: A 30-year-old patient presents to labor and delivery complaining of bright red vaginal bleeding. She has no pain. The fetus is still active. She is 37 weeks pregnant. PMH is significant for in vitro fertilization. What is the most likely diagnosis? A Placental abruption B Placenta acreata C Placenta previa D Disseminated intravascular coagulopathy E Active labor
The Correct Answer is: C The most likely diagnosis is placenta previa, as the bleeding is bright red and painless. Labor and abruption are associated with discomfort and pain.
Q 43.7: A mother brings her 11-month-old son to you for a fever and rash. He had four days of fever worsening each day until last night when it measured 104.5°F taken orally before being given acetaminophen. When he awoke this morning, his fever was 100.4°F but he had developed a maculopapular rash to his trunk and extremities that blanches. Which of the following is the probable etiology? A Fifth disease B varicella C roseola D rubella E scarlet fever
The Correct Answer is: C The most prominent historical feature is the abrupt onset of fever, often reaching 40.6˚C, which lasts up to eight days (mean, four days) in an otherwise mildly ill child. The fever then ceases abruptly, and a characteristic rash may appear. Roseola occurs predominantly in children aged six months to three years, with 90% of cases occurring before the second year. Herpesvirus 7 (HHV-7) infection tends to occur somewhat later in childhood. These viruses are the most common recognized cause of exanthematous fever in this age group and are responsible for 20% of emergency department visits by children aged six to 12 months.
Q 97.13: You are caring for a pregnant woman who is Rh-negative. The father of the child is Rh-positive and heterozygous. What percent chance will the fetus have of being Rh-positive? A 0% B 25% C 50% D 75% E 100%
The Correct Answer is: C The mother is negative and has no genetically positive material to pass to the child; the father is heterozygous, so only half of his genetics contain the positive antigens.
Q 115.14: A 23-year-old patient with type 1 diabetes mellitus (DM) has been having difficulty sleeping at night. Usually around 3 am the patient will wake up feeling sweaty, nauseated, and tachycardic. He has recorded the following blood glucose levels: What advise is the best for this patient? A stop eating a bedtime snack B increase the evening regular dosage C decrease the evening Lente dosage D exercise before going to bed at night
The Correct Answer is: C The patient has described the Somogyi effect. This effect occurs because the patient is receiving too much intermediate insulin at dinnertime. This occurs when nocturnal hypoglycemia results in counter-regulatory hormones producing hyperglycemia. Either the intermediate insulin dosage can be shifted to a lower dosage at bedtime or the patient can eat a larger snack at bedtime. (Masharami, 2009, pp. 1074-1075) Masharami U. Diabetes mellitus and hypoglycemia. In: McPhee SJ , Papadakis MA, eds. Current Medical Diagnosis and Treatment. 48th ed. New York, NY: McGraw-Hill; 2009.
A 3 year-old boy is brought to the emergency department due to acute onset of cough and wheezing. Physical exam reveals focal wheezing in the right lower lobe. Which of the following is the most effective treatment option for the patient's suspected diagnosis? A Albuterol B Azithromycin C Bronchoscopy D Chest physiotherapy E Prednisone
The Correct Answer is: C The patient has most likely aspirated a foreign body. The most effective treatment is removal of the foreign body through bronchoscopy (C). Supportive therapy includes bronchodilators for wheezing/airway obstruction (A), antibiotics for associated infections (B), and chest physiotherapy (D) to promote recovery after removal of the foreign body.
Q 38.4: A 3 year-old boy is brought to the emergency department due to acute onset of cough and wheezing. Physical exam reveals focal wheezing in the right lower lobe. Which of the following is the most effective treatment option for the patient's suspected diagnosis? A Albuterol B Azithromycin C Bronchoscopy D Chest physiotherapy E Prednisone
The Correct Answer is: C The patient has most likely aspirated a foreign body. The most effective treatment is removal of the foreign body through bronchoscopy (C). Supportive therapy includes bronchodilators for wheezing/airway obstruction (A), antibiotics for associated infections (B), and chest physiotherapy (D) to promote recovery after removal of the foreign body.
Q 6.5: A 18 year-old man presents to the clinic with a lifelong history of transfusion dependent anemia and iron chelation therapy. Which of the following hemoglobin electrophoresis findings is most likely present? A Decreased HbA2 and increased HbF levels B Decreased HbA2 and HbF levels C Increased HbA2 and normal HbF levels D Increased HbA2 and decreased HbF levels E Normal HbA2 and HbF levels
The Correct Answer is: C The patient history is most consistent with beta thalassemia major that presents with increased HbA 2 and/or HbF levels. Alpha thalassemia is typically a less severe anemia and presents with a normal hemoglobin electrophoresis (E), since all adult hemoglobin are alpha-containing and affected equally. Beta thalassemia minor also typically presents with increased HbA 2 (C) and can typically be distinguished from beta thalassemia major by the clinical less sever clinical presentation.
Q 97.61: At her routine annual pelvic exam, a 39-year-old female presents to the clinic complaining of pelvic pressure and bloating for several months. She is a G3P2 who delivered vaginally. She is a nonsmoker. Her maternal aunt had a history of ovarian cancer. Her pelvic exam reveals an 8-cm ovarian mass in the right adnexal area. What is the most appropriate evaluation of the ovarian mass? A Pelvic ultrasound B Pelvic CT scan C Surgical evaluation D CA 125 level E Repeat pelvic exam 1 month
The Correct Answer is: C The patient is high risk, as she is premenopausal, has a family history of cancer, and the mass is large. Therefore, surgical evaluation should be undertaken. CA 125 can be negative in early disease, and pelvic US and CT are not sensitive enough. Repeat examination should be reserved for low risk women with smaller ovarian masses.
Q 97.11: A 24-year-old G2P2 delivered a viable female infant (8 lb 4 oz) via caesarean section, after a failed 20-hour induction for post date pregnancy. On day 2, she developed a postoperative fever of 101F (38.3C). She had slightly increasing abdominal cramping and pain, no change in loci, is voiding well, and has passed flatulence. Her WBC is 19,000. What is the mostly likely cause for her fever? A Urinary tract infection B Ileus C Metritis D Atelectasis E Tubo-ovarian abscess
The Correct Answer is: C The patient is passing urine and flatulence well, making choices A and B less likely. Fever greater the 38C is the most important indicator of metritis. Fever is not usually indicative of mild atelectasis. Tubo-ovarian abscess is usually a complication from PID. *The patient has many risk factors for metritis including c-sect, prolonged induction, and fever.*
Q 13.2: A 78-year-old obese male smoker presents with a gradual progression of fatigue and pallor over the last few months. Initial CBC results show a hemoglobin of 10.4 mg/dL, hematocrit of 32%, an MCV of 74 fL, and a reticulocyte count of 0.1%. Iron studies reveal an elevated ferritin, decreased serum iron and TIBC. Which of the following is the most likely cause of this patient's anemia? A Alpha thalassemia minor B Beta thalassemia C Anemia of chronic disease D Hemolytic anemia E Iron deficiency anemia
The Correct Answer is: C The patient most likely has an underlying chronic disease resulting in anemia, as evidenced by his adequate iron stores and inability to properly use that iron for RBC production. Iron studies in patients with thalassemia (A and B) are typically normal. Iron deficiency anemia (E) is confirmed with low ferritin, serum iron, and increased TIBC.
Q 6.1: A 28-year-old female presents with a gradual progression of fatigue and pallor over the last few months. Initial CBC results show a hemoglobin of 10.4 mg/dL, hematocrit of 32%, an MCV of 112 fL, and a reticulocyte count of 0.1%. Which of the following is the most likely pathophysiologic mechanism responsible for her anemia? A Acute blood loss B Defective bone marrow/stem cell function C Defective DNA production D Defective hemoglobin production E Increased destruction of red blood cells
The Correct Answer is: C The patient most likely has an underlying vitamin B 12 or folate deficiency resulting in macrocytosis (MCV of 112 fL). Defective DNA production (C) results in failure of RBC maturation and macrocytosis (elevated MCV). Acute blood loss (A) would present more acutely and with normal hemoglobin, hematocrit, and MCV until hemodilution occurs and lowers the hemoglobin concentration and hematocrit. Defective bone marrow/stem cell function (B) tends to produce normocytic red blood cells. Microcytic, hypochromic anemia results from defective hemoglobin production (D). Hemolysis (E) is less likely than vitamin B 12 and folate deficiency, but can be confirmed or ruled out through the assessment of the reticulocyte count (increased reticulocyte will increase the MCV and be inconsistent with a nutritional deficiency) and further hematologic labs as necessary.
Q 8.7: A 15-year-old male presents complaining of a sore throat, headache, and mild cough that started 8 days ago and has progressed to include a worsening cough and increasing fatigue. His chest x-ray reveals bilateral hilar infiltrates and a CBC is normal. Which of the following diagnostic tests will most likely confirm the suspected diagnosis? A Acid-fast bacilli smear and culture B Blood culture C PCR testing of sputum D Sputum culture E Sputum gram stain
The Correct Answer is: C The patient most likely has atypical pneumonia secondary to mycoplasma pneumoniae which is best confirmed by PCR testing of sputum (C), oropharyngeal or nasal secretions and isn't detected through standard cultures (B, D) or staining techniques (A, E).
Q 20.9: A 22 year-old male presents with a 2-week history of an upper respiratory infection that hasn't improved after taking amoxicillin for 6 days. He notes persistent sore throat, intermittent fever, and a worsening nonproductive cough. Physical examination reveals bilateral diffuse crackles. What is the most likely causative agent of this patient's symptoms? A Streptococcal pneumoniae B Klebsiella pneumoniae C Mycoplasma pneumoniae D Pneumocystis jiroveci E Staphlococcal aureus
The Correct Answer is: C The patient presentation is consistent with atypical pneumonia most commonly caused by viruses, mycoplasma pneumonia (C), chlamydia pneumonia, or legionella. The presentation of streptococcal pneumoniae (A) is more acute with productive cough and fever being early symptoms. Klebsiella pneumoniae (B) also presents with fever and a cough classically noted to be productive of "currant jelly" sputum. Pneumocystis jiroveci (D) causes pneumonia of insidious onset in immunocompromised patients. Staphlococcal (E) pneumonia also presents more acutely with productive cough.
Q 43.6: A 15-year-old male presents to the outpatient clinic complaining of a sore throat and fever that developed over the last week after feeling fatigued for two to three weeks. The patient appears non-toxic with a temperature of 39.0˚ Celsius. Physical exam reveals pharyngeal and tonsillar erythema with exudates and tonsillar hypertrophy, posterior cervical adenopathy, and splenomegaly. Which of the following laboratory tests is most likely to confirm the expected diagnosis? A HIV antibody testing B Liver function tests C Monospot D Rapid strep test E Rapid plasmin reagin (RPR)
The Correct Answer is: C The patient presentation is consistent with infectious mononucleosis (IM) and can be confirmed through a monospot test (C). The time course of the symptoms and presence of posterior cervical adenopathy makes IM more likely than strep pharyngitis (D). HIV antibody testing (A) is ineffective at diagnosing acute retroviral syndrome during its initial presentation. Liver function tests (B) may be elevated in IM, but are not confirmatory of the diagnosis.
Q 35.10: A 17-year-old boy high school wrestler is brought into the emergency department after he collapsed at a wrestling match. He spent time fully clothed in a hot sauna prior to the match to try to "make weight." Labs are ordered, and results come back as follows: Which IV fluid regimen would most effectively treat this patient's hypernatremia? A quarter normal (hypotonic) saline B half-normal saline C isotonic (normal) saline D dextrose 5% in water E lactated Ringer's
The Correct Answer is: C The patient presents with a combination of inadequate fluid intake and excessive losses due to perspiration, resulting in hypovolemia and hypernatremia. The most common causes of hypernatremia are inadequate fluid intake resulting in hemoconcentration and diabetes insipidus (DI), resulting in excessive renal fluid losses. Normal urine osmolality is 500 to 850 mOsm/kg but can range from 50 to 1,200 mOsm/kg depending on the patient's fluid intake. Urine osmolality >400 mOsm/kg indicates that the renal fluid-conserving mechanism is intact, as the kidneys are working to preserve volume. A lower urine osmolality would be consistent with DI, characterized by a lack of response to anti-diuretic hormone (ADH), resulting in excessive urinary losses of water with worsening hypernatremia. Treatment is directed at the cause. If the patient is dehydrated, restoring fluid volume is the goal. If the patient has DI, treating the underlying disease will lower the serum sodium level. For this dehydrated patient, the treatment would be to administer isotonic (normal) saline, which contains 0.9% sodium, because of the large free water deficit. Quarter-normal saline contains 0.25% sodium, half-normal saline contains 0.45% sodium, and lactated Ringer's solution is similar to half-normal saline in its sodium content. Dextrose 5% in water (D5W) contains no electrolytes. Isotonic saline is the appropriate choice because it treats not only the volume deficit but the serum osmolality as well. Its osmolality (308 mOsm/kg) is often lower than the plasma osmolality because of the hypovolemic state and, therefore, helps restore normal serum osmolality. Once serum osmolality becomes more normal, the isotonic saline can be replaced by D5W to replace the remaining free water deficit. If the free water deficit were less dramatic, initial IV fluid treatment could be half-normal saline, followed by D5W.
Q 114.3: A 45-year-old male presents with a history of thick, adherent yellow scaling in his scalp, and red scaling patches with fissuring in post auricular areas bilaterally. Which of the following are other areas of potential involvement? A antecubital and popliteal fossae B palms and soles C eyebrows, eyelashes, and beard area D trunk and neck
The Correct Answer is: C The patient presents with seborrheic dermatitis. Other areas of potential involvement include the eyebrows, eyelashes, and beard area. Antecubital and popliteal fossae are common areas of involvement in atopic dermatitis; not seborrheic dermatitis. The palms and soles are not involved in seborrheic dermatitis. The trunk and neck are not usually affected in seborrheic dermatitis. (Wolff et al., 2009, Page 49)
A 62-year-old man is brought to the emergency department after being found unresponsive in his car. On physical examination, his pupils are noted to be 7 mm on the right and 3 mm on the left. Which of the following diagnostic tests is most likely to identify the cause of the patient's signs and symptoms? A CBC with differential B serum electrolytes C MRI with contrast D liver function tests E skull X-rays
The Correct Answer is: C The patient's unilateral symptoms are best explained by a local anatomical cause (e.g., tumor) that would be detected with an imaging study (MRI). An MRI is preferred over skull X-rays to assess directly for intracranial pathology. CNS abnormalities arising from systemic causes are more likely to be symmetric.
Q 16.8: A 68-year-old man presents to the outpatient clinic complaining of decreased hearing in his left ear. The following is seen on otoscopic evaluation. Which of the following is the most likely diagnosis? A Acute otitis media B Cerumen impaction C Cholesteatoma D Chronic otitis media E Tympanic membrane perforation
The Correct Answer is: C The photo depicts a classic cholesteatoma (C) effecting the pars flaccida. The localized nature of the findings and lack of inflammation make otitis media (A and D) unlikely. Any perforation in the TM (E) would be secondary to the cholesteatoma and the visible TM is inconsistent with a diagnosis of impacted cerumen (B).
Q 32.19: A 24-year-old female comes into the clinic complaining of a severe sore throat. She was seen three days ago at an urgent care facility, and was given amoxicillin. She states that the pain is worse, she is unable to drink fluids, and is now having difficulty swallowing. She talks with a muffled voice. A physical exam reveals a markedly swollen and erythemic right tonsil and tonsillar pillar, with the uvula deviating to the left. The patient has extreme difficulty opening her mouth. What is the most likely diagnosis? A Tonsillitis B Uvulitis C Peritonsillar abcess D Tonsillar cellulitis E Diphtheria
The Correct Answer is: C The physical exam is highly suspicious for peritonsillar abcess, which must be considered first. Tonisllar cellulitis, or phlegmon, is swelling and enlargement of the tonsil and peritonsillar tissue, without the presence of fluctuant abcess. Uvulitis can exist with a peritonsillar abcess or tonsillitis, but isolated uvulitis usually includes symmetric swelling and erythema as a result of irritation (snoring), allergy (angioedema), or infection from upper respiratory pathogens. Diphtheria is a tonsillitis, with a characteristic gray pseudomembrane on the tonsils and upper airway, caused by corneybacterium diphtheriae.
Q 28.2: A 51-year-old female presents to her primary care provider for her annual physical. She is a healthy white female and a non-smoker. She has mild HTN, but an otherwise negative health Hx. In addition, FHx is negative and ROS is negative. Her LMP was 6 months ago. Her last mammogram was 3 years ago. When should she have her next mammogram? A In 2 years B In 1 year C Now D Only if self breast exam reveals abnormality E Only if provider breast exam reveals abnormality
The Correct Answer is: C The preponderance of data strongly supports the benefits of a screening mammography. New analyses of older randomized studies have suggested that screening may not work. While the design defects in some older studies cannot be retrospectively corrected, most experts, including panels of the American Society of Clinical Oncology and the American Cancer Society, continue to believe that screening conveys substantial benefit.
Q 97.64: A 51-year-old female presents to her primary care provider for her annual physical. She is a healthy white female and a non-smoker. She has mild HTN, but an otherwise negative health Hx. In addition, FHx is negative and ROS is negative. Her LMP was 6 months ago. Her last mammogram was 3 years ago. When should she have her next mammogram? A In 2 years B In 1 year C Now D Only if self breast exam reveals abnormality E Only if provider breast exam reveals abnormality
The Correct Answer is: C The preponderance of data strongly supports the benefits of a screening mammography. New analyses of older randomized studies have suggested that screening may not work. While the design defects in some older studies cannot be retrospectively corrected, most experts, including panels of the American Society of Clinical Oncology and the American Cancer Society, continue to believe that screening conveys substantial benefit.
Q 116.7: A 27-year-old male with enlarged cervical lymph nodes and shortness of breath is seen. Lungs are clear to auscultation but diminished breath sounds centrally are noted. A chest x-ray reveals a large mediastinal mass. A biopsy of the lymph node is performed and Reed-Sternberg cells are noted on exam. What is the most likely diagnosis? A Acute myelogenous leukemia B Chronic lymphocytic leukemia C Hodgkin lymphoma D Infectious mononucleosis E Multiple myeloma
The Correct Answer is: C The presence of Reed-Sternberg cells in the histologic examination of lymph nodes, along with a large mediastinal mass, is consistent with Hodgkin lymphoma. Reed-Sternberg cells are not seen in multiple myeloma, AML, CLL, or infectious mononucleosis. (Lichtman et al., Williams Hematology 8e, Chapter 99, Hodgkin Lymphoma)
Q 10.8: Upon testing a patient for function of the hip abductors, which muscle is considered the primary muscle responsible for most abduction? A Biceps femoris B Gluteus maximus C Gluteus medius D Gluteus minimus E Vastus medialis
The Correct Answer is: C The primary mover in the motion of hip abduction is the gluteus medius muscle. Gluteus minimus does play a supporting role in that motion. Biceps femoris is one of the three hamstring muscles and contributes to the motions of knee flexion (primary muscle) and hip extension (secondary muscle). Gluteus maximus is the primary mover for hip extension and vastus medialis is one of the four quadriceps muscles responsible for knee extension, but no hip movements.
Q 97.4: A 25-year-old woman and her husband have been using condoms and spermicidal foam for the 8 months since the birth of their baby. She plans to wean the baby from the breast sometime between a year and 18 months of age, but would like to begin a "less messy" method of contraception. Prior to her pregnancy she took combination oral contraceptives for several years without any difficulties. Which of the following is an appropriate recommendation for this woman? A "Breast feeding alone will prevent pregnancy until the baby is weaned." B "Condoms and spermicide are your only option until you wean the baby." C "Progestin-only pills are recommended for women who are breast feeding." D "A vaginal ring will not affect your milk production." E "You can start back on the same oral contraceptive you took before."
The Correct Answer is: C The progestin-only pill is ideal for breast feeding mothers because this pill does not interfere with lactation the way combination pills do. Breast feeding (A) is reasonably effective in preventing pregnancy only as long as breast milk is the infant's only source of nutrition. Condoms and spermicide (B) are options, but not the only ones for breastfeeding couples. A vaginal ring (D) does decrease the amount of milk production somewhat, but may be an effective option if lactation is well established. Combination oral contraceptives (E) are not recommended for breastfeeding women.
Q 74.2: A 46-year-old female complains of heartburn, steatorrhea, and a 20-lb weight loss. She was diagnosed with a solitary ulcer in the duodenal bulb, but it has been refractory to treatment. Imaging reveals the tumor but no hepatic metastasis. What is the recommended treatment in this patient to cure the disease? A Oral proton pump inhibitors B Resection of the entire duodenum C Resection of the tumor D Long term therapy with oral amoxicillin and an H2 antagonist E Systemic chemotherapy
The Correct Answer is: C The recommended treatment to cure localized disease in patients with Zollinger-Ellison syndrome is resection of the gastrinoma, before hepatic metastasis spread has occurred. (McPhee SJ, Papadakis MA. Current Medical Diagnosis & Treatment, 2010, p. 556)
A 78-year-old female trips and falls on an outstretched hand and now presents with right wrist pain. On examination, the patient complains of pain on palpation to the right radial side of the wrist near the anatomical snuffbox. Based on this clinical presentation, what is the most likely fracture that this patient has? A Capitate B Lunate C Scaphoid D Trapezium E Trapezoid
The Correct Answer is: C The scaphoid or navicular bone of the wrist, is the most commonly fractured carpal bone. This injury occurs most commonly in young men. The lunate is the second most commonly fractured carpal bone and the most commonly dislocated carpal bone. The capitate is the largest of the carpal bones and it is not known as a common isolated fracture site. Fractures of the trapezium and trapezoid are relatively uncommon as well.
Q 29.2: A 78-year-old female trips and falls on an outstretched hand and now presents with right wrist pain. On examination, the patient complains of pain on palpation to the right radial side of the wrist near the anatomical snuffbox. Based on this clinical presentation, what is the most likely fracture that this patient has? A Capitate B Lunate C Scaphoid D Trapezium E Trapezoid
The Correct Answer is: C The scaphoid or navicular bone of the wrist, is the most commonly fractured carpal bone. This injury occurs most commonly in young men. The lunate is the second most commonly fractured carpal bone and the most commonly dislocated carpal bone. The capitate is the largest of the carpal bones and it is not known as a common isolated fracture site. Fractures of the trapezium and trapezoid are relatively uncommon as well.
Q 31.7: Which rotator cuff muscle is most commonly injured? A Infraspinatus B Subscapularis C Supraspinatus D Teres Major E Teres Minor
The Correct Answer is: C The supraspinatus is involved in abduction and external rotation of the shoulder joint. It is often injured in repetitive overhead activities such as swimming and throwing sports. While all muscles of the rotator cuff can be injured, the supraspinatus has been shown to sustain the most frequent injuries. The infraspinatus and teres minor are also involved in abduction and external rotation, while the subscapularis and teres major assist in internal rotation. The teres major is not considered a rotator cuff muscle.
Which rotator cuff muscle is most commonly injured? A Infraspinatus B Subscapularis C Supraspinatus D Teres Major E Teres Minor
The Correct Answer is: C The supraspinatus is involved in abduction and external rotation of the shoulder joint. It is often injured in repetitive overhead activities such as swimming and throwing sports. While all muscles of the rotator cuff can be injured, the supraspinatus has been shown to sustain the most frequent injuries. The infraspinatus and teres minor are also involved in abduction and external rotation, while the subscapularis and teres major assist in internal rotation. The teres major is not considered a rotator cuff muscle.
Q 28.4: A 67-year-old, 220-pound female presents to your family practice office for a presurgical clearance examination for a total hip replacement in one week. Past medical history includes hypertension, atrial fibrillation, a prior percutaneous transluminal coronary angioplasty (PTCA) with drug-eluting stent seven months ago, and type 2 diabetes mellitus . Her medication include: lisinopril, metoprolol ER, a baby aspirin, clopidogrel, and metformin. Her blood pressure is 128/78 mm Hg, pulse is 72 beats/min, temperature 98.6˚F, and respirations of 17 breaths/min. Her heart and lung sounds are normal. The abdominal examination is benign. Her x-ray and EKG are normal, as are all required labs. Which of the following is recommended for this patient prior to her surgery? A Discontinue metoprolol one week prior to surgery. B Discontinue metformin one week prior to surgery. C Discontinue the aspirin and clopidogrel five to seven days prior to surgery. D Reschedule the surgery until patient has reduced their weight by 7%.
The Correct Answer is: C The timing and consideration of discontinuing anti-platelet agents is complicated. Generally speaking, for minor procedures (such as a tooth extraction) in patients with risk, discontinuation is not necessary. But for major and/or bloody procedures, such as a total joint replacement, discontinuation is necessary. When considering major surgeries, most anti-platelet therapies should be stopped five to seven days prior to the surgery. Consultation with the patient's cardiologist may be beneficial.
Q 120.13: What primitive, or immature, defense mechanism is demonstrated by a patient who attributes their own, unacknowledged, feelings onto others while they search for perceived wrongdoings, no matter how small? A acting out B isolation C projection D splitting
The Correct Answer is: C These patients are sensitive to any criticism and are constantly searching for any insult or mistreatments, no matter how small or unintentional they may be. Confrontation is to be avoided as it is only counterproductive and will reinforce their beliefs. This is commonly seen in paranoid personality disorders. (Sadock and Sadock, 2008, p. 30) Sadock BJ , Sadock VA. Concise Textbook of Clinical Psychiatry, 3rd ed. Philadelphia, PA: Lippincott, Williams & Wilkins; 2008.
A 27-year-old female presents to the emergency department after a motor vehicle accident. Imaging of the left lower extremity shows the following fracture pattern: How would this fracture be described? A No displacement B Complete dorsal displacement C Fifty percent dorsal displacement D Displacement with shortening
The Correct Answer is: C This Fracture is displaced, pertaining to any deviation from anatomical position or alignment to the extent to which the fracture fragments are nonconcentric or offset from each other. This fracture is only 50 percent displaced without angulation or shortening. The magnitude of displacement is expressed in either terms of measurement (i.e. incomplete) or percentage (i.e. 50%) ) of the width of the bone. The direction of displacement is based on the position of the distal fragment relative to the proximal fragment. Separation is the distance two fragments have been pulled apart. Shortening (D) is the amount the bone's length has been reduced, which is not seen in this fracture. Angulation is the degree of "bending" that makes the fragments unparalleled. This would not be described as "no displacement" (A) or "complete displacement" (B).
Q 49.8: A 12-year-old female presents with linearly distributed light brown papules on her arm. They are asymptomatic and have been present for several years. The mother states that they appear to grow as the child grows. What treatment is necessary? A Cryotherapy B Laser ablation C No treatment is necessary D Shave excision with biopsy
The Correct Answer is: C This condition is consistent with a linear epidermal nevus. They can appear at any age, but are usually present at or shortly after birth. The pigmented papules are arranged linearly and can occur on any skin surface. They are not symptomatic and will grow with the child. There is no treatment necessary.
Q 113.4: A 46-year-old male comes into the emergency department complaining of severe left eye pain after a champagne cork hit his eye while trying to open a bottle on his honeymoon night. Visual acuity to the affected eye is limited to identifying finger movements only. A physical exam reveals the following findings. What is your diagnosis? A Corneal abrasion B Orbital contusion C Hyphema D Hypopyon E Retinal detachment
The Correct Answer is: C This image demonstrates a traumatic hyphema. There is an air-fluid level of blood in the anterior chamber of the eye. Treatment includes elevation of the head and dilation of the pupil. A corneal abrasion would demonstrate fluorescein staining of the cornea. A hypopyon is a collection of wbc's, or pus in the anterior chamber. A retinal detachment would only be visualized in the posterior eye with a direct ophthalmoscope. (Tintinalli et al., 2011, Chapter 236)
Q 118.11: A 30 year old male presents with bright red erythema over her cheeks and nose after spending a week at the beach. She has had no prior rashes. She states that the rash is pruritic and she has had a low grade fever accompanied by lethargy. Lab results show a (+) ANA and anti DS-DNA with an elevated ESR. What is the most likely diagnosis? A Rosacea B Roseola C Systemic lupus erythematosus D Fixed Drug eruption
The Correct Answer is: C This is a classic presentation of systemic lupus erythematosus (SLE). This autoimmune disorder is more common in women who are in their 20s or 30s. Frequently it is precipitated or worsened by sun exposure. (Wolff & Johnson, pg 382) (Fig 14-34A, Wolff K, Johnson RA: Fitzpatrick's Color Atlas & Synopsis of Clinical Dermatology, 6 th Ed: http://www/accessmedicine.com)
Q 34.7: A 6-year-old male presents with multiple lesions on his shins. The lesions are annular dermal plaques with a central depression. There are no epidermal changes. He states the lesions are asymptomatic. The child has no other medical problems and is a normal active child. What is the most likely diagnosis? A necrobiosis lipoidica B tinea corporis C granuloma annulare D atopic dermatitis
The Correct Answer is: C This is the classic distribution of granuloma annulare. These lesions commonly occur over bony surfaces and are thought to be secondary to minor trauma (such as playing soccer, normal play activities, or insect bites). The lesions will spontaneously resolve and no treatment is indicated. The distribution in this patient is similar to that of classic necrobiosis lipoidica; however, the dermal changes are classic for granuloma annulare. Necrobiosis lipoidica starts as brown-red plaques that evolve to become waxy appearing. They are commonly misdiagnosed as tinea corporis; however, there are no epidermal changes such as scaling. The lesions are completely dermal. Atopic dermatitis in a 6-year-old child is most commonly distributed on the flexural surfaces and consists of red scaling plaques that are pruritic.
Q 81.6: A 28-year-old male presents with a tan-pink, well-demarcated waxy plaque, with raised firm borders located in the pretibial region of the left lower extremity. What is the next appropriate step to confirm the diagnosis? A liver function test B lipid panel C glucose tolerance test D complete blood cell count
The Correct Answer is: C This lesion is consistent with necrobiosis lipoidica, and is associated with diabetes mellitus. The patient should undergo glucose tolerance testing to be evaluated for diabetes. (Wolff et al., 2009, Page 428)
Q 86.10: A 70-year-old male is being followed for his chronic lymphocytic leukemia. On prior examination, he was staged at Binet Stage A, with three cervical lymph nodes and one right axillary lymph node palpated. Six months later he is seen for follow-up, where he now has five palpable cervical lymph nodes and two left and one right axillary node. His lymphocyte count has risen from 12,500 to 18,300. What is the best therapy at this time? A Alemtuzumab B Chlorambucil C Continue monitoring D Etoposide E Fludarabine
The Correct Answer is: C This patient continues to remain in Binet Stage A, and therefore would not benefit from beginning any additional therapy. Monitoring should be continued. If they would develop 3/5 lymphoid regions of involvement (cervical, axillary, ileofemoral, splenomegaly, or hepatomegaly) or doubling of the lymphocyte count in less than six months, then adding additional therapy would be indicated. Fludarabine is first-line therapy for progressing CLL. Chlorambucil is the main alkylating agent used in CLL. Alemtuzumab is a monoclonal antibody specific for human CD52, found on most lymphocytes and useful in CLL treatment. Etoposide is used in patients who failed alkylator-based chemotherapy. (Lichtman et al., Williams Hematology 8e, Chapter 94, Chronic Lymphocytic Leukemia and Related Diseases)
Q 95.3: A 31-year-old female is admitted to the hospital with a microangiopathic hemolytic anemia, thrombocytopenia, and elevated reticulocyte count and serum lactate dehydrogenase. Haptoglobin is decreased and a direct Coombs test is negative. There is a marked increase in schistocytes on blood smear. PT and aPTT are normal. What is the best first line of treatment? A Aspirin B Glucocorticoids C Plasma Exchange D Platelet transfusion E Vincristine
The Correct Answer is: C This patient has TTP (thrombotic thrombocytopenic purpura). The mainstay of therapy is plasma exchange. Glucocorticoids are used, but their efficacy has not been demonstrated conclusively. Aspirin is controversial, but has been used in additional therapy. Platelet transfusion is contraindicated. Vincristine is used if plasma exchange does not work. (Lichtman et al., Williams Hematology 8e, Chapter 133, Antibody-Mediated Thrombotic Disorders: Thrombotic Thrombocytopenic Purpura and Heparin-Induced Thrombocytopenia)
Q 109.3: A 48-year-old male presents with an 8-day history of a productive cough, subjective fevers, and malaise. He is otherwise healthy without any active medical problems. He is a social drinker of alcohol, and denies any tobacco or drug use. On physical examination, the patient is alert and oriented. His temperature is 100.4 0 F, pulse rate is 56, respiratory rate is 18, and blood pressure is 133/64. HEENT is within normal limits and a chest exam has diffuse expiratory wheeze with decreased sounds to the right lower lung fields. Blood labs reveal WBC 14.4, Hgb 11.3, Plt 233, ALT 65, AST 102, and PO 4 2.1. A chest x-ray reveals a dense consolidation with bulging fissures. Based on these findings, what is the best medication to treat this patient? A Ceftazidime B Vancomycin C Azithromycin D Penicillin E Gentamicin
The Correct Answer is: C This patient has a history and physical examination that is consistent with Legionella infection, as noted by the chest x-ray findings. The treatment of choice for the Legionella infection is the use of macrolides. In this case, azithromycin would be the best choice for this case. (Sabria M, Yu VL. Harrisons Online, Part 7, Infectious Disease, Section 6, Diseases Caused by Gram-Negative Bacteria, Chapter 141, Legionella Infection)
Q 81.5: A 48-year-old woman presents with new-onset headache that she describes as nonspecific, worse on awakening, intermittent throughout the day but can worsen with bending over or coughing. Her husband reports that she has not been herself since the headaches started about 4 to 6 weeks ago. Which of the following tests would be best for determining the etiology of her presenting symptoms? A noncontrast head CT scan B lumbar puncture C contrast-enhanced brain MRI D noncontrast brain MRI E cerebral angiography
The Correct Answer is: C This patient has an intracranial mass until proven otherwise. Headaches starting later in life and accompanied by other neurologic or cognitive problems should raise a high suspicion of a tumor. Obtaining a contrast-enhanced MRI of the brain will demonstrate an intracranial mass lesion. The contrast will follow blood flow distribution and help in determining possible tumor type. Some lesions are difficult to see without contrast enhancement. MRI scans have a much higher resolution for soft tissue over CT scans and are preferred for looking at brain parenchyma. Prior to the advent of CT and MRI, cerebral angiography was used to look for intracranial masses. Vascular tumors have characteristic blush patterns, and if a mass effect is present, it will distort the position of the blood vessels. (Aminoff et al., 2005, pp. 82-83) Aminoff MJ , Greenberg DA , Simon RP. Clinical Neurology. 6th ed. New York, NY: McGraw-Hill; 2005.
Q 85.4: A 45-year-old male is being treated with Imatinib Mesylate for his Ph chromosome positive disease. He obtained a complete cytogenetic response at six months. He is now one year out from diagnosis and initiation of therapy. Standard monitoring guidelines for this patient would include which of the following? A Bone marrow biopsy and cytogenetics yearly B Bone marrow biopsy and FISH for t(9;22) every six months C Bone marrow biopsy yearly and quantitative PCR on blood cells every three months D FISH for t(9;22) and quantitative PCR yearly E FISH for t(9;22) on blood cells and quantitative PCR every three months
The Correct Answer is: C This patient has chronic myelogenous leukemia Ph chromosome positive. With a complete cytogenetic response at six months, standard guidelines for monitoring of these patients include a yearly bone marrow biopsy and quantitative PCR on blood cells every three months. All other tests are done, but not in the frequency stated or necessarily following complete cytogenetic response. (Lichtman et al., Williams Hematology 8e, Chapter 90, Chronic Myelogenous Leukemia and Related Disorders)
Q 41.5: A 21-day-old male is brought to the pediatric clinic for concern of irritability, sweating with feeds, and fatigue. Upon examination, there is a discrepancy in the pulse between the arms and legs. The femoral pulses are diminished when compared to the brachial pulses. A murmur is present, and heard in the left axilla and the left back. An EKG is obtained, which show right ventricular hypertrophy. Echocardiography shows a localized narrowing of the aortic arch, just distal to the origin of the left subclavian artery. What is the best definitive treatment for this patient? A Intravenous infusion of normal saline B Placement of a pericardial patch C Balloon angioplasty of the abnormality D Long-term use of propranolol E Performing an atrial switch operation
The Correct Answer is: C This patient has coarctation of the aorta. Balloon angioplasty of the abnormality is the correct answer, and is a corrective repair. IV infusion of normal saline is not appropriate in this child, who is presenting with heart failure. Placement of a pericardial patch is performed for ventricular septal defects. Long-term use of porpranolol is an effective treatment in mitral valve prolapse. An atrial switch operation is performed with transposition of the great arteries.
Q 48.9: A mother presents with a four-month-old male infant complaining of a dry, itchy rash that never seems to completely resolve. There are days when it appears to improve. She currently uses baby wash and baby lotion to care for his skin. She recently discontinued the lotion because he screams when it is applied. Which of the following is this condition exacerbated by? A warm and humid environments B use of petroleum based moisturizers C frequent bathing and soap based cleansers D soap free cleansers and topical steroids
The Correct Answer is: C This patient has the classic presentation of infantile atopic dermatitis. There are numerous factors that can irritate this condition including frequent (more than once a day) or long baths, soap based cleansers, cold dry environments, illness, stress, itchy clothing, and allergies. Lotions may sting, especially if the skin is dry and the skin barrier is broken. Atopic dermatitis usually improves in warm, humid environments. Petroleum based moisturizers are an important part of the treatment of atopic dermatitis. Soap free cleansers are recommended for patients with atopic dermatitis, as they are less irritating and drying. Topical steroids are the first line treatment of atopic dermatitis.
A 62-year-old man with a history of hypertension, diabetes mellitus type 2, hyperlipidemia, and chronic tobacco use presents to the emergency department with complaints of retrosternal chest pressure associated with diaphoresis, nausea, and dyspnea, radiating down his left arm for the last 45 minutes after mowing his lawn. His EKG is shown (Figure 3). Which of the following is the most likely diagnosis given this patient's history and EKG findings? A Acute pulmonary embolism B Esophageal spasm C Acute myocardial infarction D Costochondritis E Aortic dissection
The Correct Answer is: C This patient is demonstrating evidence of acute anterolateral myocardial infarction on EKG, with ST segment elevation across the precordial leads, and reciprocal changes in the inferior leads, indicative of left anterior descending coronary artery stenosis. In addition, this patient's history increases the risk of acute myocardial infarction; therefore, choice C is the most likely diagnosis. Acute pulmonary embolism may also present with chest discomfort, and the patient's history of chronic tobacco use does increase this patient's risk of a hypercoaguable state. However, EKG changes with pulmonary embolism differ from those pictured, in that usually on an EKG, a pulmonary embolism presents with a prominent S wave in lead I, Q wave in lead III, and inverted T wave in III (S1/Q3/T3). Choices B, D, and E may also present with retrosternal chest discomfort; however, no EKG changes would be noted.
Q 111.2: A 24-year-old female presents with low self-esteem and lack of confidence. While obtaining the history, you learn that she has a long history of recurrent maladaptive behavior. She has a tendency to blame others and has almost no introspective ability. She also has significant difficulties with interpersonal relationships. What is the most likely diagnosis? A Delusional disorder B Dysthymia C Personality disorder D Schizoaffective disorder E Schizophrenic disorder
The Correct Answer is: C This patient is exhibiting all of the features of a personality disorder. Delusional disorder is characterized by symptoms of persistent, non-bizarre delusions that include minimal impairment of daily functioning. Dysthymia is characterized by a chronic depressive disturbance that is present for at least two years and includes symptoms of sadness, loss of interest, and withdrawal from activities. Schizoaffective disorder is characterized by affective symptoms that develop alongside psychotic manifestations. These cases do not fit easily within the schizophrenic or affective categories. Schizophrenic disorder is characterized by a severe disruption of thinking, mood, and overall behavior, as well as the inability to properly filter stimuli. (McPhee SJ, Papadakis MA. Current Medical Diagnosis & Treatment, 2010, p. 952)
Q 49.12: A two-week-old female is being evaluated, and on examination she is noted to have bounding pulses with a widened pulse pressure. There is a murmur present at the second left intercostal space, and it is described as a rough machinery murmur. Cyanosis is not present. What is the most likely diagnosis in this patient? A Atrial septal defect B Coarctation of the aorta C Patent ductus arteriosis D Tetralogy of fallot E Ventricular septal defect
The Correct Answer is: C This patient is exhibiting signs of a patent ductus arteriosus. Atrial septal defects may not have a murmur associated with them early in the infant's life, but may develop four to six weeks after birth and present as a nonspecific systolic murmur. The signs and symptoms of coarctation of the aorta consist of decreased or absent femoral pulses, with a murmur present in the left axilla and the left back. Tetralogy of fallot presents with cyanosis, easy fatigability, dyspnea on exertion, and variable digital clubbing. Ventricular septal defect presents with a holosystolic murmur at the lower left sternal border and a right ventricular heave, but presentation depends on the size of the defect and the pulmonary vascular resistance. The patient with a ventricular septal defect may also present with features of heart failure, failure to thrive, and diaphoresis with feedings.
Q 80.1: A 36-year-old female has just been diagnosed with hypertension. During the office visit, she exhibits an excessive emotional reaction. She is seen frequently for follow-up of her hypertension over the next three months, and during that time she is noted to exhibit signs of immaturity, seductiveness, egocentricity, and dependency. What is the most appropriate classification of the suspected personality disorder? A Antisocial B Borderline C Histrionic D Obsessive compulsive E Paranoid
The Correct Answer is: C This patient is exhibiting the signs most consistent with histrionic personality disorder. Antisocial personality disorder is characterized by selfishness, callousness, promiscuousness, and impulsive behavior, and an inability to learn from experience and legal problems. The clinical findings of borderline personality disorder include impulsiveness, unstable and intense interpersonal relationships, lack of self control, suicidal ideations, aggressive behavior, and a high drug abuse rate. Clinical findings of obsessive-compulsive disorder include being a perfectionist, egocentric, indecisive, and having rigid thought patterns and a need for control. Clinical findings of someone who has paranoid personality disorder would include defensiveness, being overly sensitive, secretive, suspicious, and hyper-alert, and having a limited emotional response. (McPhee SJ, Papadakis MA. Current Medical Diagnosis & Treatment, 2010, p. 951)
A 56-year-old male was admitted to the regular medical floor with a diagnosis of sepsis due to urinary tract infection. While on the floor he became acutely unstable and had a blood pressure of 70/40. A medical emergency code was called, which revealed the rhythm shown. There was no palpable pulse on exam, and no evidence of heart sounds on auscultation. The patient was not responding to any stimulus during this time, and there did not appear to be any spontaneous respirations noted. Based on this clinical scenario, what would be the most appropriate medical procedure to perform on this patient? A Emergent cardiac catheterization B Synchronized cardioversion C Unsynchronized cardioversion D Temporary pacemaker insertion E No intervention, the patient has not survived
The Correct Answer is: C This patient is exhibiting ventricular fibrillation with hypotension. Because this patient is unstable in this clinical setting, immediate unsynchronized cardioversion (C) is warranted. Once corrected, consideration for cardiac catheterization (A) is reasonable since patients may have native coronary disease that caused the fibrillation. Unsynchronized cardioversion (C) would not initiate a cardiac response, and temporary pacemaker (D) must first have an established electrical rhythm to appropriately pace the patient. The patient still has a low blood pressure, therefore no intervention (E) would prove to be fatal.
Q 16.3: A 56-year-old male was admitted to the regular medical floor with a diagnosis of sepsis due to urinary tract infection. While on the floor he became acutely unstable and had a blood pressure of 70/40. A medical emergency code was called, which revealed the rhythm shown. There was no palpable pulse on exam, and no evidence of heart sounds on auscultation. The patient was not responding to any stimulus during this time, and there did not appear to be any spontaneous respirations noted. Based on this clinical scenario, what would be the most appropriate medical procedure to perform on this patient? A Emergent cardiac catheterization B Synchronized cardioversion C Unsynchronized cardioversion D Temporary pacemaker insertion E No intervention, the patient has not survived
The Correct Answer is: C This patient is exhibiting ventricular fibrillation with hypotension. Because this patient is unstable in this clinical setting, immediate unsynchronized cardioversion (C) is warranted. Once corrected, consideration for cardiac catheterization (A) is reasonable since patients may have native coronary disease that caused the fibrillation. Unsynchronized cardioversion (C) would not initiate a cardiac response, and temporary pacemaker (D) must first have an established electrical rhythm to appropriately pace the patient. The patient still has a low blood pressure, therefore no intervention (E) would prove to be fatal.
Q 92.1: A 45-year-old patient presents 2 days postoperatively with a partial thyroidectomy. She has been experiencing vomiting with diarrhea. On physical exam, her temperature is 101°F and jaundice is noted. Her heart rate is irregularly irregular with a rate of 200 bpm. What would be the most appropriate pharmacological intervention? A radioactive iodine (131I) B propranolol 80 mg C PTU 600 mg D iopanoic acid 500 mg
The Correct Answer is: C This patient is in a thyrotoxic crisis or thyroid storm. She needs to be admitted for monitoring and supportive care. The initial treatment would be PTU 600 mg loading dose followed by 200 to 300 mg every 6 hours given either by nasogastric tube or rectally. (Fitzgerald, 2009, p. 988) Fitzgerald PA. Endocrine diseases. In: McPhee SJ , Papadakis MA, eds. Current Medical Diagnosis and Treatment. 48th ed. New York, NY: McGraw-Hill; 2009.
Q 97.28: A women presents to the labor department complaining of contraction every 3 to 4 minutes for the last 3 hours. She is a G1P0 at 40 weeks gestation. Her pregnancy is uncomplicated; her group B strep culture is negative. Physical exam vitals are normal, the baby is cephalic in a +2 station, and the bag of water is intact. The fetal heart monitor reveals fetal heart tones in the 140s with contractions every 3 minutes lasting 45 seconds. Her cervix is 4-cm dilated and 50% effaced. What is the expected rate of cervical dilation? A .5 cm per hour B 1 cm per hour C 1.2 cm per hour D 1.5 cm per hour E 1.7 cm per hour
The Correct Answer is: C This patient is in stage one active labor and is a primigravida. She should expect 1.2 cm of dilatation per hour; multigravida women can expect a faster rate of dilatation.
Q 115.6: A 30-year-old obese female presents with thick darkened skin in the bilateral axillae and around her armpit. She has tried over-the-counter hydrocortisone cream, with no help. There are no associated symptoms. Which of the following labs should be ordered? A complete blood cell count (cbc) with diff and liver function test (LFT) B LFT and basic metabolic panel (BMP) fasting C fasting insulin, glucose, and lipid panel D BMP and cbc with diff
The Correct Answer is: C This patient presents with acanthosis nigricans, which is most commonly associated with insulin resistance and metabolic syndrome. The most helpful labs for patients presenting in this manner are fasting insulin, glucose, and lipid panels. No helpful information will be gained from a cbc with diff and liver function test (LFT). The BMP will provide a glucose level, but not enough helpful information will be gained from these labs. (Wolff et al., 2009, Page 89)
Q 95.7: A 27-year-old man presents to the emergency department with a five-day complaint of substernal pleuritic chest pain, which worsens while lying supine. He is in no distress. A friction rub is noted over the precordium. The patient's vital signs are as follows: temperature is 100.4˚F, pulse rate is 94, respiratory rate is 20, and blood pressure is 136/84. An ECG reveals widespread diffuse ST elevations with PR interval depressions. He was recently treated for a viral respiratory infection. Which of the following is the most appropriate initial management for this patient? A Administer a broad spectrum antibiotic B Administer intravenous tissue plasminogen activator (T-PA) C Begin a nonsteroidal anti-inflammatory agent D Perform needle thoracentesis E Refer for immediate cardiac catheterization
The Correct Answer is: C This patient's diagnosis is acute inflammatory pericarditis. Viral infections are the most common cause of acute pericarditis, and males are the most commonly affected. A pericardial friction rub and EKG changes are characteristic of this diagnosis. Treatment is focused on the underlying inflammation, with NSAIDS being first-line and short course corticosteroids also being appropriate. Antibiotics are not indicated unless a bacterial etiology is confirmed or there are significant risk factors. Choices B, D, and E are not appropriate for this condition and could be harmful. (McPhee SJ, Papadakis MA. Current Medical Diagnosis & Treatment 2011, Chapter 10, Heart Disease)
Q 32.32: A 22-year-old man (refer to Figure 4-2) is being evaluated for extremity enlargement unlike anyone in his family. Over the past 2 years, he has noticed that his rings no longer fit and his feet are so wide that he cannot find shoes to fit. He has always been tall for his age, greater than the 95th percentile throughout his teenage years. He has very coarse facial features, macroglossia, and a very deep voice. What is the most likely cause of this patient's condition? (Reproduced, with permission, from Gagel R, McCutcheon IE. N Engl J Med, 1999;340:524. Copyright © 1999 Massachusetts Medical Society. All rights reserved.) A adrenal neoplasm B multinodular goiter C pituitary macroadenoma D Rathke cleft cyst E testicular neoplasm
The Correct Answer is: C This patient's signs and symptoms are consistent with acromegaly, which is caused by an increased secretion of GH. These are almost always caused by pituitary macroadenomas. The tumors may be locally invasive into the cavernous sinus but are typically not malignant.
Q 28.9: A 73-year-old man is brought into your office by his adult children with a concern of memory loss. They report their father's memory has been declining since the death of their mother a few months ago but are now concerned because he is losing weight, sleeping during the daytime, and is not keeping up with current events like he usually does. This type of behavior is most associated with which of the following? A Pick disease B Creutzfeldt-Jakob disease C depression D Alzheimer disease E vitamin B12 deficiency
The Correct Answer is: C This patient's symptoms are most consistent with situational depression over the loss of his spouse. Transient memory problems can be a component of depression as a result of decreased attention and interest. Dementia is a progressive impairment of higher cognitive function, and initially, the patient's social graces are preserved. It has many causes, of which Pick disease, Creutzfeldt-Jakob disease, and Alzheimer disease are irreversible. Vitamin B 12 deficiency can cause reversible form of cognitive impairment, in which the elderly are susceptible, so serum analysis of vitamin B 12 should be performed in diagnostic evaluations of dementia in this population.
Q 36.8: An 8-year-old male presents with hair loss and scalp pruritus with scaling. The mother states it has been present for approximately two weeks. On physical exam, posterior cervical lymphadenopathy is found to be present. What would be the best diagnostic test to perform at this stage? A cbc with diff B Wood's lamp examination C culture on dermatophyte test medium D thyroid function test
The Correct Answer is: C This presentation is classic for tinea capitis. Tinea capitis is extremely common in school aged children. It presents with a scaling, pruritic scalp and lymphadenopathy. There is often hair loss that is described as black dot alopecia. The diagnosis is confirmed by culture of dermatophyte test media (DTM). A cbc will not help diagnose a fungal infection. Only 10% of tinea infections will fluoresce with Wood's lamp examination; therefore, it is not effective for diagnosing tinea capitis. A thyroid function test will not help with a tinea infection.
Q 32.29: Your patient is a 42-year-old female diagnosed with depression. She has taken two different serotonin reuptake inhibitors (SSRIs) at the usual dosage, each for 3 weeks, and reports again that there have been no ill effects but that she doesn't feel that this medication has helped either. What is the most likely problem with her therapy? A She has a genetic resistance to the medication B The dosage was too low C The medication was discontinued too soon D The wrong diagnosis was made E The wrong medication was given
The Correct Answer is: C Treatments for depression generally require from 2 to 6 weeks of therapy in before effects can be evaluated (C). Too low a dose (B) is another common problem, but an increase in dose should not be considered until the medication has had time to take effect. The effect of these agents can be highly variable from one individual to another, but no specific genetic resistance (A) is known. There is always a possibility of making the wrong diagnosis (D), but there is no indication of this from the information given, and SSRIs are the first line therapy for depression (E), best combined with counseling.
Q 32.26: A 52-year-old male bus driver presents to the clinic with a chief complaint of intense, shooting pains in his left cheek, each lasting for only a few seconds. He avoids touching certain parts of his face and has started to chew food only on the right side of his mouth because he is afraid he will set off an attack of pain. In between attacks, the patient feels well. What is the most likely diagnosis? A cluster headache B tension-type headache C trigeminal neuralgia D giant cell arteritis E dental abscess
The Correct Answer is: C Trigeminal neuralgia is characterized by sharp, brief pain often described as "shooting, jabbing, electric shock, or stabbing." The history given for cluster headache (typically ipsilateral ocular headaches with tearing, and lasting for 2 hours) and tension-type headache is not at all like this patient's. The history for temporal arteritis is generally different, as it typically includes ocular symptoms, but it may be worth getting a sedimentation rate just to be sure. This location pain pattern is different than that of a focal dental problem.
Q 4.9: A 14-year-old female patient presents to your family practice clinic having received a 1% total body surface area first and second degree burn to the left forearm. Of the following, what would you recommend for your patient? A Deroof any blisters, apply bacitracin topically, and prescribe pain medication, with follow-up in 48 hours, B Deroof any blisters, apply silver sulfadiazine topically, and prescribe pain medication, with follow-up in 48 hours. C Leave any blisters intact, apply bacitracin topically, and prescribe pain medication, with follow-up in 48 hours. D Leave any blisters intact, apply silver sulfadiazine topically, and prescribe pain medication, with follow-up in 48 hours, E Refer to the emergency department immediately.
The Correct Answer is: C Unless a critical surface (face, genitalia or hands) is involved, first and second degree burns may be treated in the outpatient setting. Blisters may be left intact as a physiologic dressing, and deroofed after they rupture. The patient requires tetanus prophylaxis and a topical antibiotic cream, usually either silver sulfadiazine or, preferably, bacitracin. Sulfadiazine may permanently stain skin, so use it cautiously in potentially exposed skin areas for cosmetic reasons.
Q 32.28: A 66-year-old female patient presents to your family practice clinic for a complete physical exam. She is a smoker and you counsel her at length regarding smoking cessation. She is a well- controlled hypertensive taking lisinopril, along with an 81 mg enteric aspirin. You review her testing which reveals the following: • Comprehensive metabolic panel is normal. • Complete blood count is normal. • Urinalysis is normal, except for trace blood. • Fasting lipids are to goal. • Immunochemical fecal occult blood testing (IFOBT) is negative for occult blood. Of the following, what is a next appropriate step to take? A Schedule a colonoscopy to rule out colon cancer. B Refer her to pulmonary medicine for pulmonary function testing. C Refer her to a urologist to rule out bladder and renal cell carcinoma. D Stop her aspirin. E Send out her urine for culture and sensitivity, and in the interim start her on an empiric antibiotic.
The Correct Answer is: C Unless the patient has another reason to have hematuria, consideration should be made of the possibility of bladder or renal cell cancers, especially in a smoker. After ruling out other causes, the standard of care is to refer this patient to a urologist to rule out these potentially life threatening cancers.
Q 119.10: Which of the following is a result of untreated or partially treated otitis media, which presents with fever, ear pain, otorrhea, tenderness behind the ear, fluid collection, and destruction of air cells seen on head CT? A Suppurative otitis media B Peritonsillar abcess C Mastoiditis D Meningitis E Ethmoid sinusitis
The Correct Answer is: C Untreated or partially otitis media can result in mastoiditis. Tenderness, redness, and fluctuance over the mastoid bone is characteristic. Peritonsillar abcess symptoms include severe sore throat, drooling, dysphonia, and outpouching of the tonsillar pillar on the affected side and trismus. Ethmoid sinusitis presents with nasal congestion, discharge, and headache. Suppurative otits media is contained in the middle ear, without spreading to adjacent structures. (Knoop et al., 2010, Chapter 5)
Q 2.9: A 49-year-old male presents to the clinic with symptoms of nausea, occasional vomiting, vague epigastric pain, fatigue, and weight loss of 35 lbs. over the past few months. On exam you find a palpable abdominal mass. Which of the following diagnostic tests is the best initial test to obtain? A Barium upper GI series B Abdominal CT C Upper endoscopy D PET scan E Flexible sigmoidoscopy
The Correct Answer is: C Upper endoscopy allows for a biopsy, which is highly sensitive in detecting gastric carcinoma, the suspected diagnosis. An upper GI series cannot distinguish a benign from a malignant lesion. An abdominal CT would be most useful once gastric carcinoma has been diagnosed, to help in preoperative evaluation. A PET scan would be most useful in detecting metastasis. A flexible sigmoidoscopy would not allow for visualization of the stomach.
Q 91.3: A 52-year-old male presents to the office with a complaint of abdominal pain. He describes the pain as epigastric, dull, and achy. Antacids do help the pain for a few hours, but it then returns. He also notes that the pain often wakes him from sleep. He denies significant vomiting, dysphagia, heartburn, or weight loss. Which of the following diagnostic tests is the procedure of choice in this patient? A Abdominal CT B Barium upper GI series C Upper endoscopy D Colonoscopy E Esophageal manometry
The Correct Answer is: C Upper endoscopy is the procedure of choice for diagnosing peptic ulcer disease, which is the concern in this patient. Abdominal CT is obtained in patients suspected of having complications from an ulcer. Barium upper GI series is not as sensitive as upper endoscopy in detecting ulcer disease. Colonsocopy is used to evaluate the colon, and this patient's symptoms do not suggest a problem in that area. Esophageal manometry is used to evaluate dysphagia, which this patient denies having. (McPhee SJ, Papadakis MA. Current Medical Diagnosis & Treatment, 2010, p. 546)
Q 39.18: Which of the following is the most common congenital heart malformation? A atrial septal defect B tetralogy of Fallot C ventricular septal defect D transposition of the great vessels
The Correct Answer is: C Ventricular septal defect, a hole between the two ventricles, can be cyanotic or acyanotic based on the size of the defect, and accounts for 30% of cases of congenital heart disease. Atrial septal defect occurs in approximately 10% of congenital heart disease cases. Transposition of great vessels is an embryonic malformation resulting in the aorta arising from the right ventricle and the pulmonary artery arising from the left ventricle. It is responsible for about 10% of all congenital malformations. Tetralogy of Fallot, consisting of a ventricular septal defect, overriding aorta, pulmonic/subpulmonic stenosis, and right ventricular hypertrophy, accounts for 10% of congential heart disease. (
Q 98.58: A 45-year-old nonsmoking female is found to have a 2-cm mass in the periphery of the left lung on a routine chest radiograph. She has no significant medical history, known exposures, or contributory family history. A subsequent biopsy confirms malignancy. Which of the following is the most likely underlying cell type? A Squamous cell B Large cell C Adenocarcinoma D Small cell
The Correct Answer is: C While adenocarcinoma, like other lung cancer types, is more likely to appear in smokers; it is the most common lung cancer seen in nonsmokers, particularly women and younger patients. It generally originates in the periphery of the lung. All other answers are associated with significant smoking history and, with the exception of large cell carcinoma, originate centrally.
Q 111.9: A 45-year-old nonsmoking female is found to have a 2-cm mass in the periphery of the left lung on a routine chest radiograph. She has no significant medical history, known exposures, or contributory family history. A subsequent biopsy confirms malignancy. Which of the following is the most likely underlying cell type? A Squamous cell B Large cell C Adenocarcinoma D Small cell
The Correct Answer is: C While adenocarcinoma, like other lung cancer types, is more likely to appear in smokers; it is the most common lung cancer seen in nonsmokers, particularly women and younger patients. It generally originates in the periphery of the lung. All other answers are associated with significant smoking history and, with the exception of large cell carcinoma, originate centrally. (Minna, 2008, p. 552) Minna JD , Schiller JH. Neoplasms of the lung. In: Fauci AS , Kasper DL , Longo DL , Braunwald E , Hauser SL , Jameson JL , Loscalzo J, eds. Harrison's Principles of Internal Medicine. 17th ed. New York, NY: McGraw-Hill; 2008.
Q 97.41: A 60-year-old postmenopausal woman who has had negative annual Papanicolaou smears of the cervix for many decades reports that her husband has been in a nursing home for almost a year and that she is no longer sexually active. He was her only sexual partner. She has no personal or family history of cancer and has never taken or been exposed to any kind of hormone. She asks if she really needs to continue having testing every year. What is the correct advice for her? A At your age, you should really have a Pap smear every six months. B You should continue to have annual Pap smears. C You can safely extend the time between Pap smears to three years. D We can discontinue Pap smears altogether if you first have a negative colposcopy. E People who are not sexually active do not need Pap smears at all.
The Correct Answer is: C Women between the ages of 30 and 64 who have had 3 consecutive negative Pap smears and no additional risk factors may reduce the frequency of their screenings to every 2-3 years. Women who are HIV positive should have cervical screening every six months (A). Women with risk factors such as multiple partners, history of sexually transmitted infections or in utero exposure to diethylstilbestrol, immunosuppressive therapy should be screened annually (B). A colposcopy is not needed given her history (D). People who are not sexually active (E) may need to continual annual screenings if other risk factors exist.
Q 98.6: An 80-year-old male nursing home patient is brought to the emergency department with abdominal distension. A plain film of the abdomen is pictured below. Which of the following is the most likely diagnosis? A Small bowel obstruction B Cecal volvulus C Sigmoid volvulus D Toxic megacolon
The Correct Answer is: C A volvulus is an obstruction of the colon due to a loop of bowel that has rotated more than 180 degrees on its axis with the mesentery. The most common site for a volvulus is the sigmoid colon (65%). A sigmoid volvulus is associated with abdominal pain and distension. Plain films of the abdomen would show a characteristic "bent inner tube" appearance. Sigmoidoscopy can be used to decompress the bowel by gently releasing the area of obstruction. Following decompression, a rectal tube is inserted to act as a stent to prevent the bowel from twisting upon itself again.
Q 77.2: A 48-year-old African American male presents with dyspnea, 2-pillow orthopnea, and swelling to his lower legs that has developed over the last month. He also complains of fatigue and decreased exercise tolerance, stating that he has trouble climbing one flight of steps. On physical examination, his blood pressure is 178/98, pulse rate is 102, and respiratory rate is 20. There is 5 cm JVD, crackles at the bilateral lung fields, and tachycardia and an S 3 is heard on cardiac auscultation. There is 2+ pitting edema to the lower extremities. His electrocardiogram reveals a sinus tachycardia at a rate of 105 and left ventricular hypertrophy. The chest x-ray reveals cardiomegaly with increased interstitial markings in all lung fields. There is a small right pleural effusion that blunts the costophrenic angle. Which medication is the treatment of choice for controlling this patient's heart rate? A Amlopidine B Minoxidil C Isosorbide mononitrate D Metoprolol E Atropine
The Correct Answer is: D The use of beta-blockers is indicated for heart rate control. The other choices are not indicated for rate control and have no primary action on rate, but rather on blood pressure. (Bashore et al., Current Medical Diagnosis and Treatment, Chapter 10)
Q 34.3: An 8-year-old girl is rushed to the emergency department by her parents because she has become delirious. The child was diagnosed with influenza three days prior. Her parents say that she had begun vomiting yesterday, almost nonstop, and has not been able to hold down fluids. They also note that she has been breathing rapidly. Your exam reveals a tachypneic, disoriented female with hyperreflexia, a positive babinski reflex, and liver enlargement. CSF analysis reveals a normal protein and cell count. What is the most likely diagnosis? A Bacterial meningitis B Guillain Barre syndrome C Measles encephalitis D Reye's syndrome E Viral meningitis
The Correct Answer is: D *Although rare, Reye's syndrome is associated with viral infections, salicylate use during illness, and metabolic disorders. Illness is associated with liver fat deposition and degeneration, intractable vomiting, and mental status changes, which may progress to seizures, delirium, and coma. Cerebral edema contributes to these changes and other neurologic findings.* Meningeal signs are more consistent with meningitis. Measles encephalitis typically presents days to weeks after the pathognomic measles exanthem and clinical findings. Guillain Barre has been associated with influenza infection, and signs and symptoms would include evolving weakness with ascending paralysis and extremity dysesthesias.
Q 45.2: A 3-month-old infant had a mild microcytic, hypochromic anemia at birth and the screen was negative for sickle cell disease/trait. She was started on iron therapy and presents today for follow up. The hemoglobin (Hgb) electrophoresis laboratory results are: Hemoglobin 8.8 mg/dL (normal: 10.5-14.0) Hematocrit 25% (normal: 33-42) Mean corpuscular volume (MCV) 60 fL (normal: 70-90) Mean corpuscularhemoglobin concentration (MCHC) 32 g/dL (normal: 33-37) Hgb A 2 27% (normal: 1.5%-4%); Hgb A 1 30% (normal: 76%-99%); HgF (fetal hemoglobin) 50% (normal: 0%-20%); Bart Hgb 0% (normal: 0%) Which of the following is the MOST likely diagnosis? A heterozygous alpha thalassemia B homozygous alpha thalassemia C beta thalassemia major D beta thalassemia minor
The Correct Answer is: D *The typical hemoglobin electrophoresis for beta thalassemia minor has an elevated level of hemoglobin A2. In a normal infant there is mainly HgF and HgA 1 with minimal amounts of A2.* Bart hemoglobin is diagnostic for the alpha thalassemias after the neonatal period is over. Beta thalassemia major will only have fetal hemoglobin on electrophoresis. Because of the high incidence of false-negatives in hemoglobin screenings in the neonatal period, it is important for the provider to do a full work-up of microcytic, hypochromic anemias to ensure proper diagnosis.
Q 95.8: A 68-year-old female presents with symptoms of kidney failure. Her creatinine is elevated at 1.9, and monoclonal light chains are seen in the urine. She complains of fatigue and bony pain that has been increasing over the last several months. This patient is most likely experiencing symptoms from which disease? A Acute myelogenous leukemia B Chronic myelogenous leukemia C Hodgkin lymphoma D Multiple myeloma E Non-Hodgkin lymphoma
The Correct Answer is: D 30 to 50% of patients presenting with multiple myeloma will have some form of renal impairment. The most frequent cause is the formation of myeloma cast nephropathy. This is due to formation of tubular casts in the distal nephron, formed by the binding of light chains to uromodulin. These tubular casts obstruct the distal nephron and parts of the ascending loop of Henle, and contribute to development of interstitial nephritis. Light chains are not seen in the urine of any of the other diseases listed. (Lichtman et al., Williams Hematology 8e, Chapter 109, Myeloma)
Q 76.1: A 30-year-old male presents to your office complaining of sinus and facial pain, congestion, and purulent nasal discharge for one month. He has been treated with two courses of different antibiotics by another provider, and does not feel any improvement in his symptoms. What diagnostic test is indicated? A Plain sinus radiographs B MRI C Aspiration and culture of maxillary sinuses D CT scan E Ultrasound of sinuses
The Correct Answer is: D A CT scan is the current preferred method for sinus imaging of chronic sinusitis. CT imaging has better visualization of mucosal thickening air-fluid levels and bone structures. Plain radiographs and CT scans are of limited use in acute sinusitis, because viral pathogens that cause sinus abnormalities are indistinguishable from bacterial causes. (Lalwani A.K., 2008, Chapter 14)
Q 105.17: While interviewing a 29-year-old computer programmer, you find that he denies any close friends or prior sexual relationships and has no interest in developing them. He describes little enjoyment in any activities except role play video games. He denies past emotional difficulties or stressors. His exam reveals a flat affect throughout the visit but is otherwise normal. Which is the most likely diagnosis in this scenario? A antisocial personality disorder B adjustment disorder C seasonal affective disorder D schizoid personality disorder
The Correct Answer is: D A patient with ambivalence toward sexual relationships, no close contacts, and no desire for either, along with anhedonism and flat affect are typical for this disorder. The preference for solitary activities and use of fantasy furthers this picture. The lack of aggressiveness and risk-taking behavior lessons the antisocial diagnosis. The patient denied any precipitating event that would lend the problem to an adjustment disorder and the lack of variance, seasonal or otherwise, lessens the seasonal affective disorder diagnosis. (Eisendrath and Lichtmacher, 2009, pp. 925-926; Sadock and Sadock, 2008, p. 378) Eisendrath SJ , Lichtmacher JE. Psychiatric disorders. In: McPhee SJ , Papdakis MA, eds. Current Medical Diagnosis and Treatment, 48th ed. New York: McGraw-Hill; 2009. Sadock BJ , Sadock VA. Concise Textbook of Clinical Psychiatry, 3rd ed. Philadelphia, PA: Lippincott, Williams & Wilkins; 2008.
Q 34.8: A 4-year-old male has been experiencing a significant cough for the last 12 to 14 days, and initial episodes of coughing are characterized as frequent outbursts of 5 to 10 spastic coughs in a row. The patient does not report any fever, but does note that the coughing is worse at night. On examination, the patient is alert, awake, and oriented. His temperature is 97.7 0 F, pulse rate is 89, respiratory rate is 25, and blood pressure is 110/56. The HEENT is unremarkable, and lung sounds are clear to auscultation. You suspect that the patient may have an acute case of pertussis. Based on the history and physical exam findings, which is the test of choice for confirming a diagnosis of pertussis? A Complete blood count (CBC) B Throat culture C Chest x-ray D Nasopharyngeal culture E Sputum gram stain and culture
The Correct Answer is: D A special medium culture plate (such as a Bordet-Gengou agar) is required for the nasopharyngeal swab for the diagnosis of pertussis. Throat culture, chest x-rays, and complete blood counts are helpful in ruling out other disease patterns.
Q 19.4: What types of connective tissue are injured in a strain? A Bones and muscles B Fascia and joint capsules C Ligaments and joint capsules D Muscles and tendons E Tendons and bones
The Correct Answer is: D A strain involves injury to the muscles and tendons that are responsible for active movement of various body parts. Fascia is a part of the muscle-tendon unit, so injury to fascia would be considered a strain as well. Injury to ligaments and joint capsules would be considered a sprain and damage to bone would be classified as a fracture.
What types of connective tissue are injured in a strain? A Bones and muscles B Fascia and joint capsules C Ligaments and joint capsules D Muscles and tendons E Tendons and bones
The Correct Answer is: D A strain involves injury to the muscles and tendons that are responsible for active movement of various body parts. Fascia is a part of the muscle-tendon unit, so injury to fascia would be considered a strain as well. Injury to ligaments and joint capsules would be considered a sprain and damage to bone would be classified as a fracture.
Q 110.17: Which of the following represents a positive tuberculin skin test result? A A college student with 4mm of superficial erythema at site B An HIV-positive patient with 2mm induration C A low-risk individual with a pre-employment test result of 6mm induration D A nursing home resident with 12mm induration E A recent immigrant from Mexico with 8mm induration
The Correct Answer is: D A tuberculin purified protein derivative (also known as a TB test or PPD), is utilized to screen for latent Mycobacterium tuberculosis infection. Guidelines for interpreting test results, based upon induration, patient risk, and patient medical status, are published by the Centers for Disease Control and Prevention (summarized in Table 9-10 below). False-negative reactions may occur in immunosuppresed patients and those with extensive infection. False-positive and false-negative reactions can occur for various reasons, including previous vaccination with bacillus Calmette-Guirein (BCG), which may cause a false-positive. http://www.cdc.gov/TB/publications/factsheets/testing/skintesting.pdf
Q 41.2: Which of the following indicates a poor prognosis for someone diagnosed with schizophrenia? A acute onset B co-morbid mood disorder C obvious precipitating event D younger age at diagnosis
The Correct Answer is: D A younger age of onset/diagnosis along with an insidious onset, social isolation, family history of schizophrenia, and negative symptoms (affective flattening, alogia, apathy, anhedonia) all portend a poor prognosis. To the contrary, acute onset, late diagnosis, positive symptoms (hallucinations, delusions, disordered thought processes, etc.), and a concomitant mood disorder actually lend to a better prognosis.
Q 98.20: A 56-year-old man is diagnosed with a carcinoma of the sigmoid colon by colonoscopy. Which of the following tests should be performed as part of the preoperative evaluation for distant metastasis? A Carcinoembryonic antigen level B CT of the chest C Endorectal ultrasound D CT scan of the abdomen E Bone scan
The Correct Answer is: D Abdominal/pelvic CT scans and chest radiographs should be obtained as part of the preoperative staging of colon carcinoma for the evaluation of distant metastasis. CT scan of the chest is indicated only if the chest radiograph is abnormal. Endorectal ultrasound is useful in further staging rectal carcinoma. Bone scan is not indicated as a routine diagnostic study in the preoperative staging of colon cancer. Although carcinoembryonic antigen levels are performed preoperatively, they are not helpful in staging and are useful only in the postoperative follow-up of patients to monitor for a recurrence.
Q 116.17: According to the American Heart Association's most recent guidelines regarding infective endocarditis, which of the following patients requires infective endocarditis prophylaxis? A A 65-year-old man with a history of rheumatic fever prior to colonoscopy B A 29-year-old woman with a history of bicuspid aortic valve prior to vaginal hysterectomy C A 42-year-old man with a history of mitral valve regurgitation prior to vasectomy D A 22-year-old man with a history of mitral valve replacement prior to tooth extraction E A 44-year-old woman with a history of mitral valve prolapse prior to open cholecystectomy
The Correct Answer is: D According to the American Heart Association's most recent guidelines regarding infective endocarditis prophylaxis, only those patients with prosthetic heart valves, prior history of bacterial endocarditis, unrepaired or incompletely repaired cyanotic congenital heart disease (including those with palliative shunts and conduits), completely repaired congenital heart disease during the first 6 months after surgery if a prosthetic material or device was used, whether placed surgically or via catheter, repaired congenital heart disease that has residual defects at or adjacent to the site of repair, and heart transplant recipients with valvulopathy, and then in these populations of patients, only with certain procedures including tooth extraction. Therefore, the only patient among the choices offered who requires infective endocarditis prophylaxis is choice D. Choices A, B, C, and E are not considered at significant risk for infective endocarditis, regardless of the procedure. (American Heart Association, Inc., 2011, http://www.americanheart.org/presenter.jhtml?identifier=3047051)
Q 43.4: A 4-year-old child swallows several tablets of a medication that he found in his parent's bathroom cabinet underneath the sink. Approximately 2 to 3 hours after ingesting the tablets, there were no symptoms other than nausea and vomiting. Thirty hours after ingesting the tablets, elevated aminotransferase levels were detected followed by jaundice, hepatic encephalopathy, renal failure, and death. What did the child most likely swallow? A diazepam B aspirin C oxycodone D acetaminophen E phenobarbital
The Correct Answer is: D Acetaminophen toxicity may result from a single toxic dose, from repeated ingestion of large doses of acetaminophen (eg, 7.5 to 10 g daily for 1 to 2 days), or from chronic ingestion of the drug. Dose-dependent hepatic necrosis is the most serious acute toxic effect associated with overdose and is potentially fatal. Acetaminophen is the second most common cause of liver failure requiring transplantation in the United States.
Q 85.5: Administration of a drug that inhibits acetylcholinesterase would most likely lead to which of the following? A bronchodilation B decreased lacrimation C decreased salivation D increased gastric juice secretion E increased heart rate
The Correct Answer is: D Acetylcholinesterase is the enzyme responsible for metabolizing acetylcholine, which is the major neurotransmitter released from postganglionic neurons of the parasympathetic nervous system. Inhibition of acetylcholinesterase increases the level of acetylcholine at target tissues and can thereby create parasympathomimetic effects. These would include bronchoconstriction, increased lacrimation, increased salivation, bradycardia, and increased gastric juice secretion. (Costanzo, 2006, pp. 51-54) Costanzo LS. Physiology. 3rd ed. Philadelphia, PA: Saunders Elsevier; 2006.
Q 106.4: A 54-year-old male presents to you with a sudden onset of severe left eye pain and blurred vision. He states that he is nauseated and vomited twice. He denies any history of eye problems, other than having to wear glasses for reading. His only recent problem has been a mild upper respiratory infection, for which he is taking an over-the-counter decongestant. On physical exam, the vision in the affected eye is 20/200. His pupil is mid-sized and non-reactive to light, and the conjunctiva is markedly injected. What diagnosis must you consider first? A Retinal detachment B Central retinal artery occlusion C Open angle glaucoma D Angle closure glaucoma E Optic neuritis
The Correct Answer is: D Acute angle closure is characterized by sudden onset of severe eye pain, blurred vision, nausea, vomiting, visual halos, and headache. Physical exam findings can include conjunctival injection, a rock hard ocular globe on palpation, a cloudy cornea, and a mid-position fixed pupil. Normal intraocular pressure is below 21mm Hg. Acute angle closure glaucoma can develop pressures of 60 to 80mm Hg. (Tintinalli et al., 2011, Chapter 236)
Q 106.13: A 24-year-old woman comes to your office complaining of anxiety. The patient had witnessed a traumatic event 3 days earlier that made her feel fearful. She has not been able to tell her family about this experience. She now feels like she is numb and in a dazed, dreamlike state with poor concentration, and difficulty sleeping. She experienced a flashback of the event yesterday. What is the most likely diagnosis? A post-traumatic stress disorder B dissociative fugue C psychosis D acute stress disorder E depersonalization
The Correct Answer is: D Acute stress disorder is characterized by experiencing or witnessing a traumatic event where the person felt threatened by death or injury or the people they witnessed. The person feels fearful and helpless. Symptoms usually occur within a month of the event, last 2 days, and resolve in a month. The person feels numb, has lack of awareness of surroundings, and sees everything in a dreamlike state. Sometimes they develop amnesia. Flashbacks or recurrent images can occur with acute stress disorder. Difficulty sleeping, poor concentration, anhedonia, irritability, and despair are associated with this disorder. If not treated at the early stages, the patient is at risk of developing PTSD. (Johnson et al., 2008, pp. 377-378; Sadock and Sadock, 2008, p. 260) Johnson DC , Krystal JH , Southwick SM. Posttraumatic stress disorder and acute stress disorder. In: Ebert MH , Loosen PT , Nurcombe B , Leckman JF, eds. Current Diagnosis and Treatment in Psychiatry. New York: McGraw-Hill; 2008. Sadock BJ , Sadock VA. Concise Textbook of Clinical Psychiatry, 3rd ed. Philadelphia, PA: Lippincott, Williams & Wilkins; 2008.
Q 121.6: A 65-year-old female presents with a red irritation in her right eye. She states that this has been occurring intermittently for about two years. She also states that her eyelids are "droopy," and that she needs plastic surgery. On physical exam you notice a diffusely injected conjunctiva and an outwardly tilted lower eyelid. What is the most likely diagnosis for the abnormal physical finding? A Conjunctivitis B Dacryoadenitis C Entropion D Ectropion E Exophthalmos
The Correct Answer is: D Ageing causes a relaxation of the obicularis oris muscle, and will cause the lower eyelid to sag outwardly. This prevents the lower lid from protecting the eye, and frequently results in exposure conjunctivitis and keratitis. Treatment is surgical. (Riordan et al., 2008, Chapter 4)
Q 72.2: An 18-year-old female presents with two weeks of severe sore throat and fatigue. Her exam shows an exudative tonsillitis. A mono-spot test is positive, and a rapid strep test is positive. Which of the following medications should be avoided? A Erythromycin B Clindamycin C Cephalexin D Ampicillin E Prednisone
The Correct Answer is: D Ampicillin should be avoided, because a high percentage of mononucleosis patients develop a fine, non-allergic maculopapular rash when given ampicillin class drugs. The remaining antibiotics are appropriate for treating group A strep. Prednisone is used to reduce the pain and inflammation associated with severe tonsillitis. (McPhee et al., 2011, Chapter 8)
Q 19.3: An avulsion fracture at the base of the fifth metatarsal is commonly called which of the following? A Bennett fracture B Boxer's fracture C Chauffer's fracture D Jones fracture E Lisfranc fracture
The Correct Answer is: D An avulsion fracture at the base of the fifth metatarsal, usually secondary to plantar flexion and inversion is called a Jones fracture. Also called a ballet or dancer's fracture, it is the most common metatarsal fracture. The fracture occurs at the proximal diaphysis. A Bennett fracture is an oblique fracture of the first metacarpal near the carpometacarpal joint. A boxer's fracture is a fracture of the fifth metacarpal. This is the most common fracture of the hand. A chauffer's fracture is an oblique fracture through the base of the radial styloid in the forearm. A Lisfranc fracture is actually a fracture and dislocation involving the tarsometatarsal joints.
Q 5.3: An 8-month-old male presents to the clinic due to irritability, fatigue and parental concerns about developmental delays. He is fifth percentile for weight and 33rd percentile for height. An office based hemoglobin level is 8.4 mg/dL. The infant is prescribed iron supplementation, to begin today. Which of the following tests should be performed in 5-7 days to confirm response and adherence to iron supplementation? A Ferritin B Hemoglobin C Hematocrit D Reticulocyte count E Serum iron
The Correct Answer is: D An elevated reticulocyte count 1 week after initiation of iron supplementation confirms the presence of IDA and efficacy of therapy. The hemoglobin (B), hematocrit (C), serum iron (E) and ferritin (A) will improve subsequently to the increase in the reticulocyte count. After one month of iron supplementation the patient's hemoglobin should increase by 1-2 mg/dL and the hematocrit should increase by 3-6%.
Q 63.9: Which of the following statements about anemia associated with CKD is TRUE? A Iron and folic acid by mouth are the most effective treatments. B Transfusion of packed red blood cells monthly is the most effective treatment. C IM erythropoietin given monthly is the most effective treatment. D It is due to the inability of the kidney to transform erythropoietin into its physiologically active form. E It occurs early in the course of CKD.
The Correct Answer is: D Anemia associated with CKD is the result of inadequate erythropoietin synthesis by the kidneys. This hormone signals the bone marrow to synthesize red blood cells. A deficiency will result in anemia. In the absence of erythropoietin, iron would not be of use since red blood cell synthesis is inadequate. Folic acid would also not be of use and does not play a role in the etiology of this type of anemia. Transfusion is a tempering measure only, used to increase oxygen-carrying capacity in the case of symptomatic ischemia. Anemia due to erythropoietin deficiency generally does not occur until the GFR decreases to <60 mL/min, or approximately 50% of normal. Intramuscular administration of erythropoietin is the only effective treatment to induce red blood cell production. Depending on the formulation used, this can be given once a week or once every 2 weeks. Oral iron supplementation is needed to produce adequate hemoglobin for the increased de novo red cell production.
Q 116.2: A 22-year-old man presents with an insidious onset of low back pain over the last 6 months. He describes the pain as dull and has difficulty localizing the pain. The pain often radiates to his thighs. The pain is worse in the morning and associated with stiffening that lessens during the day. The patient notes that there is no history of trauma. The initial laboratory evaluation shows an elevated erythrocyte sedimentation rate, positive HLA-B27, and a negative rheumatoid factor. Plain films of the lumbar spine reveal bilateral blurring of the sacroiliac joints. Which of the following is the most likely diagnosis? A systemic lupus B lumbar disc disease C rheumatoid arthritis D ankylosing spondylitis E polymyalgia rheumatica
The Correct Answer is: D Ankylosing spondylosis is the most likely diagnosis in this patient. This condition is a chronic inflammatory disorder of the joints of the axial skeleton and commonly presents in the late teens or twenties. Male patients have a higher incidence than do female patients. A common presentation is pain in the lower back with radiation to the thighs and associated limitation of movement that may lessen during the day. Laboratory findings include an elevated erythrocyte sedimentation rate and positive HLA-B27. The HLA-B27 is not a specific test for ankylosing spondylitis; a small percentage of the normal population has a positive finding of this antigen. The earliest radiographic findings occur in the sacroiliac joints, with the detection of erosion and blurring of the joint space. Systemic lupus commonly affects women of childbearing years and presents with exacerbations and remissions of arthritis, rash, fatigue, and the potential for organ system involvement. Lumbar disc disease is usually seen in the age group of 35 to 45 years and is more likely to be associated with trauma. Rheumatoid arthritis does have the potential to affect this age group, but it would more likely be associated with smaller joints of the hands, along with a positive rheumatoid factor. Polymyalgia rheumatica more commonly affects patients older than 50 years and is associated with fatigue, malaise, chronic pain, and stiffness of the proximal muscles, shoulders, neck, and pelvic girdle. (Hellmann and Imboden, 2008, pp. 746-747) Hellmann DB , Imboden JB. Arthritis and musculoskeletal disorders. In: Tierney LM , McPhee SJ , Papadakis MA, eds. 2008 Current Medical Diagnosis and Treatment. New York, NY: McGraw-Hill; 2008:703-756.
Q 7.8: A 16-year-old male soccer player is complaining of pain to the right foot that has been getting progressively worse for the last 2 months. He states it hurts the most when he has all of his weight on his right foot as he plants to kick the ball. Most of the pain appears to be on weight bearing. You are concerned that this patient may be developing a stress fracture. Based on the patient's history and patient presentations, which bone is the most affected by stress fractures in the foot? A Calcaneus B Fifth metatarsal C First metatarsal D Second metatarsal E Talus
The Correct Answer is: D Any bone that is exposed to repetitive stress can have a stress fracture, but the long and thin metatarsal bones of the foot are the most commonly affected bones. Of the metatarsals, the second metatarsal has the highest number of stress fractures. These weight bearing bones can be particularly vulnerable to stress fracture if the patient is involved in long distance running, especially if he/she is wearing improper footwear for that activity or footwear that has lost most of its shock absorbing abilities. Some young female athletes may be training so hard that they become amenorrheic which can contribute to osteopenia resulting in weaker bones. Older patients with osteoporosis will also have a higher risk of stress fracture. Initially stress fractures of the metatarsals may present with a small area of localized pain and the dorsal forefoot may demonstrate a fairly diffuse area of swelling. If the stress fracture is not treated early, some patients will experience an audible pop or crack as the incomplete stress fractures progresses to a complete break. All types of fractures occur more easily in long thin bones like the metatarsals, than thicker bones like the calcaneus and talus.
Q 73.2: The organism shown in Figure 8-1 usually enters the body from infected soil through a break in the skin of the feet. It then is carried to the lungs, travels to the mouth, and is swallowed. Once in the gastrointestinal (GI) tract, it attaches to the wall and induces bleeding, leading to an iron deficiency anemia. Associated GI symptoms are uncommon. Additional symptoms include swelling and intense itching at the site in which the larva penetrates the skin. Which of the following organisms best fits this clinical picture and the organism shown in Figure 8-1? (Courtesy of Centers for Disease Control and Prevention, National Center for Infectious Diseases, Division of Parasitic Diseases.) A Strongyloides B whipworms C pinworms D hookworms
The Correct Answer is: D As described, hookworms generally enter through the skin and travel to the lungs. There they migrate to the mouth, are swallowed, and reproduce in the gut. Females lay thousands of eggs, which are subsequently excreted in the feces and mature in soil. The adult worms attach to the intestinal wall, causing bleeding and a subsequent anemia. They can also affect absorption, leading to nutritional deficiencies. Diagnosis is made through microscopic evaluation of a stool specimen and treatment is with albendazole. Whipworms (Trichuris trichiura) also lay eggs in the stool, which then reside in the soil. Many whipworm infections are asymptomatic. Strongyloides is different from other nematodes in that it can reproduce inside the intestine and persist for years. Pinworms (enterobiasis) are very small worms that exit the anus at night to lay eggs, causing intense itching and promoting the fecal-oral transmission. (Weller and Nutman, 2008, pp. 1322-1324; Kazura, 2007, pp. 2425-2431) Weller PF. Protozoal intestinal infections and trichomoniasis. In: Fauci AS , Braunwald E , Kasper DL, et al., eds. Harrison's Principles of Internal Medicine. 17th ed. New York, NY: McGraw-Hill; 2008. Kazura JW. Nematode infections. In: Goldman L , Ausiello D, eds. Cecil Medicine. 23rd ed. Philadelphia, PA: Saunders Elsevier; 2007.
Q 46.3: A 9-year-old male child presents with a painful rash of his upper extremity. His mom states it started 4 days ago and seems like it is spreading. Physical examination demonstrates a vesicular rash across the right upper arm and chest but does not cross the midline. Which of the following prescriptions would be most appropriate for this patient at today's visit? A hydration B nonsteroidal anti-inflammatory drugs (NSAIDs) C Varicella-Zoster immunoglobulin (VZIG) D oral acyclovir
The Correct Answer is: D As this patient is presenting with signs and symptoms of herpes zoster within the appropriate time frame for antiviral treatment, the treatment for this patient would be oral acyclovir. NSAIDs may help with the pain associated from zoster but will not hasten the length of the course of the virus as acyclovir will. Varicella-Zoster immunoglobulin (VZIG) is indicated for prophylaxis in exposed individuals who are immunocompromised
Q 50.2: A 9-year-old male child presents with a painful rash of his upper extremity. His mom states it started 4 days ago and seems like it is spreading. Physical examination demonstrates a vesicular rash across the right upper arm and chest but does not cross the midline. Which of the following prescriptions would be most appropriate for this patient at today's visit? A hydration B nonsteroidal anti-inflammatory drugs (NSAIDs) C Varicella-Zoster immunoglobulin (VZIG) D oral acyclovir
The Correct Answer is: D As this patient is presenting with signs and symptoms of herpes zoster within the appropriate time frame for antiviral treatment, the treatment for this patient would be oral acyclovir. NSAIDs may help with the pain associated from zoster but will not hasten the length of the course of the virus as acyclovir will. Varicella-Zoster immunoglobulin (VZIG) is indicated for prophylaxis in exposed individuals who are immunocompromised.
Q 115.12: A 14-year-old male presents for his asthma follow up. He states that he has symptoms three to four days per week and awakens from sleep three times a month, requiring the use of his inhaler. He occasionally has to sit out of gym class due to his symptoms, but overall he functions well. He currently uses a short-acting β2-agonist as needed. What is the preferred pharmacologic agent to add to this patient's regimen? A Inhaled long-acting β2-agonist B Leukotriene receptor antagonist C Long-acting mediator inhibitor D Low-dose inhaled corticosteroid E Low-dose systemic corticosteroid
The Correct Answer is: D Asthma management follows a stepwise approach, which is based upon medication action, disease progression, and patient compliance. It also involves determining the asthma classification of the patient, as well as previous response to medications. This approach also incorporates patient education, environmental control, and comorbidity management (see Figure 9-2). (McPhee SJ, Papadakis MA. Current Medical Diagnosis & Treatment 2011, Chapter 9, Pulmonary Disorders) Stepwise approach to managing asthma.
Q 79.8: A 48-year-old female complains of ear fullness, episodes of tinnitus, and vertigo. She also complains that her hearing is not as good as it used to be. She states that this has occurred sporadically over the past year. What is the most likely diagnosis? A Benign paroxysmal positional vertigo (BPPV) B Labyrinthitis C Vestibular neuronitis D Meniere's syndrome E Presbycusis
The Correct Answer is: D BPPV is characterized by sudden vertigo, made worse with head position change, and accompanied by nausea and vomiting. Meniere syndrome is characterized by episodic severe vertigo, fluctuating sensorineural hearing loss, tinnitus, and ear "fullness." Pathologically, there is distention of the endolymphatic system throughout the inner ear, presumably due to dysfunction of the endolymphatic sac. Labyrinthitis is characterized by severe vertigo and hearing loss, and is likely a result of a viral inner ear infection. Vestibular neuronitis is also a result of a viral inner ear infection, with symptoms of severe vertigo, nausea, and vomiting, without hearing loss. Both labyrinthitis and vestibular neuronitis resolve in one to two weeks. Presbycusis is age related hearing loss. (Lalwani A.K., 2008, Chapter 53)
Q 25.3: A 3 year-old male presents to the clinic with lethargy and fatigue. An initial CBC reveals a hemoglobin of 10.1 mg/dL and an MCV of 72 fL. Peripheral smear results are shown below. Which of the following best describes the cause of the abnormality pictured in the patient's peripheral smear? (note photo taken from figure 29-12 in Harrison's) A Absent or non-functioning spleen B Failure of nuclear maturation C Intravascular hemolysis D Lead intoxication E Presence of uremia
The Correct Answer is: D Basophilic stippling is noted in the peripheral smear and is associated with lead intoxication (D) and thalassemia. Howell-Jolly bodies result from the lack of removal of nuclear material due to an absent or non-functioning spleen (A). Failure of nuclear maturation (B) produces macrocytosis, intravascular hemolysis (C) leads to the creation of schizocytes, and uremia (E) is associated with Burr cells.
Q 98.72: What nerve is most commonly injured in a mid- or distal humeral shaft fracture? A Axillary B Median C Peroneal D Radial E Ulnar
The Correct Answer is: D Because of the radial nerves proximity to the humerus, mid and distal shaft fractures with significant displacement can cause a radial nerve injury. Median and ulnar injuries are more commonly associated with forearm injuries. Axillary nerve injuries are most common in anterior shoulder dislocations and peroneal nerve damage occurs as a result of lower leg insult
Q 14.2: What nerve is most commonly injured in a mid- or distal humeral shaft fracture? A Axillary B Median C Peroneal D Radial E Ulnar
The Correct Answer is: D Because of the radial nerves proximity to the humerus, mid and distal shaft fractures with significant displacement can cause a radial nerve injury. Median and ulnar injuries are more commonly associated with forearm injuries. Axillary nerve injuries are most common in anterior shoulder dislocations and peroneal nerve damage occurs as a result of lower leg insult.
What nerve is most commonly injured in a mid- or distal humeral shaft fracture? A Axillary B Median C Peroneal D Radial E Ulnar
The Correct Answer is: D Because of the radial nerves proximity to the humerus, mid and distal shaft fractures with significant displacement can cause a radial nerve injury. Median and ulnar injuries are more commonly associated with forearm injuries. Axillary nerve injuries are most common in anterior shoulder dislocations and peroneal nerve damage occurs as a result of lower leg insult.
Q 48.1: A premature infant is born with the inability to produce adequate amounts of pulmonary surfactant. Which of the following is the primary function of pulmonary surfactant? A increase the solubility of carbon dioxide in the alveoli B increase the solubility of oxygen in the alveoli C prevent infectious organisms from infiltrating the alveoli D prevent the collapse of small lung alveoli E stimulate the unloading of carbon dioxide from hemoglobin
The Correct Answer is: D Because of their small size, many lung alveoli are prone to collapse. Pulmonary surfactant contains a high concentration of amphipathic phospholipid molecules, which lowers the surface tension of alveoli. According to the law of Laplace, a reduction of surface tension reduces the collapsing pressure on small alveoli and allows them to remain open. Pulmonary surfactant production does not typically begin until the 24th week of gestation; hence, an infant born before this time is at great risk for having collapsed alveoli.
Q 101.8: A 21-year-old male with benign essential tremor asks to be placed on medication, to decrease his tremor for upcoming graduate school interviews. You discuss the potential benefits and side effects of medication and he is agreeable. Which of the following is the first-line agent of choice? A Alprazolam B Botulinum toxin C Primidone D Propanolol E Topiramate
The Correct Answer is: D Benign essential tremor is often worse during times of increased stress and decreased sleep. Management should include addressing these situations and providing nonpharmacologic therapy information. In specific cases such as this patient, beta-blocker therapy has been shown to effectively reduce tremor activity, with intermittent therapy being acceptable. Other medications, such as the others listed in the answer choices, may be used as alternatives to beta blockers, with each having individual risks and side effect profiles. (McPhee SJ, Papadakis MA. Current Medical Diagnosis & Treatment 2011, Chapter 24, Nervous System Disorders)
Q 6.3: Your 36-year-old obese female patient complained of a single episode of right upper quadrant pain after eating fast food (a double cheeseburger, fries, and a vanilla shake) last week. The symptoms have resolved and have not reoccurred when she presents to your family practice. Your psychical examination is completely normal, as is blood work, including a complete blood count, comprehensive metabolic panel, acute viral hepatitis panel, amylase, and lipase. You suspect cholecystitis and order an abdominal sonogram. The radiologist makes note that the gall bladder appears thickened with an associated calcification with a single, large gallstone. Of the following, what is the most appropriate next step? A Counsel the patient on diet, exercise, and lifestyle changes and revisit the issue should symptoms reoccur. B Refer the patient for an abdominal computed tomography (CT) scan. C Refer the patient to a gastroenterologist. D Refer the patient to a surgeon for a laparoscopic cholecystectomy due to the risk of gall bladder cancer. E Re-evaluate the patient for symptomatology in four to six weeks.
The Correct Answer is: D Calcification of the gall bladder, frequently referred to as a porcelain gall bladder, and single, solitary, large, gall stone are a common presentation of gall bladder cancer on imaging. Especially in the face of a symptomatic patient, cholecystectomy with an associated biopsy should be a strong consideration.
Q 44.13: An extremely heavy 12-year-old girl comes to the practice with her grandmother for new patient evaluation, bringing old records with her. Her blood pressure today is mildly elevated. Which of the following parameters will help determine whether her overweight and elevated blood pressure are due to Cushing syndrome (adrenocortical hyperfunction) rather than exogenous obesity? A advanced skeletal maturity B heavy thighs and legs C pinkish striae D short stature E slightly increased growth rate
The Correct Answer is: D Children with Cushing syndrome typically have short stature, while those who are obese due to exogenous factors have normal or tall stature. Likewise, they tend to have delayed skeletal maturity (A), truncal obesity with thin extremities (B), purplish striae (C), and a slowed growth rate (E), while obese children have advanced maturation, heavy extremities, pinkish striae, and an increased growth rate.
Q 116.4: A 24-year-old HIV-positive man comes to the emergency department complaining of severe left-sided chest discomfort, which radiates through to the left trapezius region. On coming into the room, you note that he is sitting up and hunched forward. Prior to examining him, you have reviewed his chart. Laboratory findings demonstrate troponins x 3, which are negative for myocardial ischemia. His EKG demonstrates diffuse ST segment elevations throughout. Which of the following physical exam findings would be most likely in this patient? A Roth spots B Splenic enlargement C Janeway lesions D Pericardial friction rub E Splinter hemorrhages
The Correct Answer is: D Choice D is the most likely finding, as this patient is exhibiting signs, symptoms, and EKG findings pathognomonic for acute pericarditis, which is likely infectious in the setting of a patient with HIV. A pericardial friction rub is heard best with the patient in a seated position, during expiration, and is frequently found in patients with pericarditis. Choice A B, C, and E are physical exam findings seen in acute bacterial endocarditis. (Fauci et al., 2001, pp. 1366-1367)
Q 80.11: A 42-year-old woman with a history of migraine cephalgia and Raynaud's phenomenon comes to the emergency department with complaints of severe chest discomfort that occurs at rest every morning (at approximately 10 AM). An EKG performed during an episode of chest discomfort demonstrates transient ST segment elevation, which is relieved with sublingual nitroglycerin. There is no troponin elevation. Cardiac catheterization is performed, and reveals coronary artery spasm, which corresponds with ST segment elevation, and no significant coronary artery stenosis. Which of the following is the most appropriate treatment regimen? A 24-hour nitroglycerin dermal patch B Thiazide diuretics C Loop diuretics D Calcium channel blockers E Aspirin
The Correct Answer is: D Choice D, calcium channel blockers, and long-acting nitrate therapy have been proven to be effective for preventing recurrences of episodes of Prinzmetal angina, with short-acting sublingual or IV nitroglycerin useful for relieving acute episodes. Choice A, 24-hour nitroglycerin dermal patch, is inappropriate, as patients can develop nitrate tolerance, and thus need a 12-hour nitrate-free period every day. Choices B and C, thiazide and loop diuretics, have no proven benefit in patients with Prinzmetal angina. Aspirin, choice E, may worsen episodes of prinzmetal angina, and thus is not recommended. (Fauci et al., 2008, Chapter 238)
Q 111.6: During a hospitalization for pneumonia, troponin levels are drawn on a 62-year old-man with a history of hypertension, hyperlipidemia, and chronic tobacco use, and found to be elevated above the 99 th percentile of normal. If acute myocardial infarction is ruled out, which of the following disease entities could also cause troponin elevation? A Mitral regurgitation B Gout C Parkinson's disease D Sepsis E Herpes zoster
The Correct Answer is: D Choice D, sepsis, is one of a long list of disease entities that can cause troponin elevation, including arrhythmias (both tachycardic and bradycardic), aortic valve disease, hypertrophic cardiomyopathy, invasive cardiac surgeries and procedures, severe pulmonary hypertension, pulmonary embolism, myocardial infiltrative diseases (such as amyloidosis, sarcoidosis, scleroderma, and hemochromatosis), acute respiratory failure, burns, pericarditis, endocarditis, myocarditis, and even occasionally due to extreme athletic activities such as marathon running. Not included on this long list, however, are choices A, B, C, and E. (Tintinalli et al., 2011, Chapter 52)
Q 32.30: Which of the following is the most common cause of hypoparathyroidism? A Familial hypoparathyroidism B Idiopathic hypoparathyroidism C Severe magnesium depletion D Surgical removal of the parathyroid E Iron deposition in the parathyroid
The Correct Answer is: D Choice D, surgical removal of the parathyroid glands, is the correct answer. Surgery for head and neck cancer, thyroidectomy, and parathyroidectomy are the most common causes of hypoparathyroidism. Choices A, B, C, and E are all causes of hypoparathyroidism that occur more infrequently.
Q 68.4: A 45-year-old man with a history of NSTEMI, CABG X 3, HTN, and hyperlipidemia presents to your office with complaints of progressive dyspnea over the last three weeks, to the point that he is now dyspneic while walking across the room. In the last few days, he has noticed bilateral lower extremity edema. His EKG is unchanged, demonstrating evidence of his prior infarction but no acute ST or T wave changes. Which of the following is the most appropriate next diagnostic study for this patient? A Transesophageal echocardiogram B Cardiac catheterization C Pulmonary function testing D Transthoracic echocardiogram E Holter monitor
The Correct Answer is: D Choice D, transthoracic echocardiogram, is usually necessary in patients demonstrating symptoms suggestive of congestive heart failure, as the history and physical examination are usually not sufficient to determine the etiology of the patient's heart failure. When compared to choice A, it is a less invasive test. It is an inexpensive, yet sensitive test for the evaluation of not only systolic, but also diastolic function, as well as valvular function, all of which can play a role in congestive heart failure. Choice B would not be the most appropriate next step, although it would be useful in evaluation of coronary artery stenosi if a nuclear stress test is found to be positive for myocardial ischemia. Choice C is a useful test if a pulmonary etiology is suspected; however, in this patient with a prior history of NSTEMI and CABG prior to the age of 45, a cardiac etiology should be ruled out prior to a pulmonary one. Choice E, Holter monitor, is inappropriate in a patient with no complaints of palpitations without arrhythmia on EKG. (Tintinalli et al., 2011, Chapter 57)
An 8-year-old girl is brought in to the emergency department with abdominal cramps, nausea, and vomiting since early this morning. She has had two loose stools but denies melena or hematochezia. She has had a low-grade fever. In the past hour, her vision has become blurry and she feels increasingly weak. Her mother has had similar but milder symptoms. Twenty-four hour dietary recall includes only chicken broth today. Last night for dinner they had meatloaf (fully cooked), mashed potatoes, and green beans. Her mother cans all their vegetables. Her medical history is unremarkable. She takes no medications. No known drug allergies. Examination reveals a temperature of 99°F, clear lungs, and mildly tachycardic heart with no murmur audible. Abdomen-bowel sounds present, soft with mild diffuse tenderness, no guarding. Neurologic examination is significant for decreased visual acuity and decreased motor strength (2/5) in the upper and lower extremities. The most likely etiology is A enterotoxic E coli B cholera species C pinworms D Clostridium botulinum
The Correct Answer is: D Clostridium botulinum produces a neurotoxin that can lead to life-threatening illness including respiratory paralysis. Botulism infection is caused by the spore-forming bacteria that lives in soil and can be foodborne. In the latter case, home-canned foods are often the cause. After a 12-hour to 3-day incubation period, botulism begins with classic symptoms of abdominal pain, nausea, vomiting, and mild diarrhea and, if unchecked, evolves into a progressive neurologic disorder marked by double vision, motor weakness, and ptosis. Respiratory muscle involvement may occur ultimately and result in death. Because of the virulence of the neurotoxin it has been used as an agent of bioterrorism. Cholera and enterotoxigenic E. coli cause a foodborne diarrheal illness that can result in significant morbidity and mortality, but they do not have neurologic manifestations. Pinworm infection is usually found among younger children, is marked by severe anal itching, and fecal-oral transmission.
Q 97.80: A 19-year-old presents complaining of vaginal discharge and itching for 3 days. She is sexually active and uses condoms most of the time. A physical exam reveals the following: vitals are normal, abdomen is soft and non-tender, + bowel sounds, the pelvic exam is notable for moderate discharge with no masses or tenderness. Her wet mount/KOH prep reveals 20 wbcs, 2+ bacteria, no hyphae, 5 to 7 clue cells. What is the most appropriate treatment? A Terconazole vaginal cream daily x 3 days B Cefixime 400 mg x one dose C Ciprofloxin 500 mg BID x 7 days D Metronidazole 500 mg BID x 7 days E Azithromycin 1 gram x 1 dose
The Correct Answer is: D Clues cells with bacteria and white blood cells are indicative of Gardnerella vaginitis, which is best treated with metronidazole.
Q 121.13: Which of the following will cause conductive hearing loss? A Mumps B Syphilis C Multiple sclerosis D Otitis media E Medications
The Correct Answer is: D Conductive hearing loss is the result of blockage of sound waves from the external canal to the inner ear. Causes include cerumen, middle ear effusion, otitis media, and occiscle disruption. Multiple sclerosis causes VIIIth cranial nerve disruption and neural hearing loss. Mumps and syphilis can cause sensoryneural hearing loss. (McPhee et al., 2011, Chapter 8) (Lalwani A.K., 2008, Chapter 52)
Q 98.49: Which of the following would raise your suspicions the most and likely warrant consideration of testing for an inherited thrombophilia? A a deep femoral vein deep vein thrombosis (DVT) after a flight from Mumbai, India B an iliac vein DVT after a round trip bus trip to Atlantic City and playing slots all day C any DVT after a total knee replacement D an upper extremity DVT after tripping falling down a flight of stairs E any DVT in a woman who recently started oral contraceptives and smokes
The Correct Answer is: D DVTs most commonly arise from the deep femoral veins and iliac arteries, most commonly in patients who smoke and take oral contraceptives, after immobilizing surgeries, and/or after immobilization due to long periods of time seated, including but not limited to airplane flights, bus rides, etc. Upper extremity DVTs are rare, even after trauma, and warrant a hypercoaguability work-up to rule out inherited disease.
Q 109.7: Which of the following would raise your suspicions the most and likely warrant consideration of testing for an inherited thrombophilia? A a deep femoral vein deep vein thrombosis (DVT) after a flight from Mumbai, India B an iliac vein DVT after a round trip bus trip to Atlantic City and playing slots all day C any DVT after a total knee replacement D an upper extremity DVT after tripping falling down a flight of stairs E any DVT in a woman who recently started oral contraceptives and smokes
The Correct Answer is: D DVTs most commonly arise from the deep femoral veins and iliac arteries, most commonly in patients who smoke and take oral contraceptives, after immobilizing surgeries, and/or after immobilization due to long periods of time seated, including but not limited to airplane flights, bus rides, etc. Upper extremity DVTs are rare, even after trauma, and warrant a hypercoaguability work-up to rule out inherited disease. (Current Diagnosis and Treatment: Surgery, Chapter 35)
Q 98.22: A 58-year-old man with a 20-year history of gastroesophageal reflux disease (GERD) presents with progressive dysphagia for 5 months associated with a 20-lb weight loss. Results from a barium swallow are pictured below (Figure 18-1). Which of the following is the most likely diagnosis? A Achalasia B Esophageal leiomyoma C Uncomplicated reflux esophagitis D Esophageal carcinoma
The Correct Answer is: D Dysphagia on a background of GERD is an alarm signal for cancer, since GERD is related to increased risk for esophageal adenocarcinoma. Esophageal cancer is associated with a progressive course of dysphagia, first to bulky foods, then softer foods, and then liquids as the tumor invades the esophagus; significant weight loss is almost universal at the time of presentation. Barium swallow demonstrates narrowing at the tumor site with normal appearance of the remainder of the esophagus. Achalasia is a motor disorder characterized by dysphagia to both liquids and solids as well as regurgitation of food. Patients with achalasia typically drink large amounts of liquids to force their food down and have problems with aspiration pneumonia. Barium swallow in achalasia typically shows a dilated esophagus with a narrowing at the lower esophageal sphincter (bird's beak). Leiomyomas are generally asymptomatic. Patients with reflux esophagitis will complain of epigastric or substernal pain that is worse when supine or leaning forward.
Q 105.18: A 58-year-old man with a 20-year history of gastroesophageal reflux disease (GERD) presents with progressive dysphagia for 5 months associated with a 20-lb weight loss. Results from a barium swallow are pictured below (Figure 18-1). Which of the following is the most likely diagnosis? (Reproduced, with permission, from Fauci AS, Braunwald E, Kasper DL, et al. Harrison's Principles of Internal Medicine, 17th ed. New York: McGraw-Hill, 2008:1849.) A Achalasia B Esophageal leiomyoma C Uncomplicated reflux esophagitis D Esophageal carcinoma
The Correct Answer is: D Dysphagia on a background of GERD is an alarm signal for cancer, since GERD is related to increased risk for esophageal adenocarcinoma. Esophageal cancer is associated with a progressive course of dysphagia, first to bulky foods, then softer foods, and then liquids as the tumor invades the esophagus; significant weight loss is almost universal at the time of presentation. Barium swallow demonstrates narrowing at the tumor site with normal appearance of the remainder of the esophagus. Achalasia is a motor disorder characterized by dysphagia to both liquids and solids as well as regurgitation of food. Patients with achalasia typically drink large amounts of liquids to force their food down and have problems with aspiration pneumonia. Barium swallow in achalasia typically shows a dilated esophagus with a narrowing at the lower esophageal sphincter (bird's beak). Leiomyomas are generally asymptomatic. Patients with reflux esophagitis will complain of epigastric or substernal pain that is worse when supine or leaning forward. (Patti, 2006, pp. 469-471; Goyal, 2008, pp. 1848-1850) Patti MG , Pietro T , Way LW. Esophagus and diaphragm. In: Doherty GM , Way LM, eds. Current Surgical Diagnosis & Treatment. 12th ed. New York, NY: Lange Medical Books/McGraw-Hill; 2006. Goyal RK. Diseases of the esophagus. In: Fauci AS , Kasper DL , Longo DL , Braunwald E , Hauser SL , Jameson JL , Loscalzo J, eds. Harrison's Principles of Internal Medicine. 17th ed. New York, NY: McGraw-Hill; 2008.
Q 97.51: A 19-year-old presents to clinic requesting emergency contraception. She is a G1P0Ab1 and a non smoker who has had intercourse and the condom broke. Her LMP was 3 weeks ago. Her PMH is negative. What would be the time frame for maximum efficacy for her to use emergency contraception? A 12 hours B 24 hours C 48 hours D 72 hours E 120 hours
The Correct Answer is: D Emergency contraception, both hormonal and IUD, reduces pregnancy rates for 120 hours, but there is a significant decrease in efficacy after 72 hours.
Q 120.12: Which of the following drugs is indicated for the treatment of anemia associated with chronic renal failure? A deferoxamine B warfarin C protamine sulfate D erythropoietin E argatroban
The Correct Answer is: D Erythropoietin (EPO) is a naturally occurring hormone synthesized and secreted by the kidneys. Synthetic forms of EPO include Epogen and Procrit. EPO works at the red bone marrow to stimulate erythropoiesis. In patients with chronic renal failure, EPO production is usually impaired, and this EPO deficiency leads to anemia. Deferoxamine is an iron-chelating compound that can be given systemically in situations of iron overdose. Warfarin and argatroban are both anticoagulants and do not typically affect red cell count. Protamine sulfate is a heparin-chelating compound that can be given in cases of heparin overdose. (Watnick and Morrison, 2008, p. 797) Watnick S , Morrison G. Kidney disease. In: Tierney LM Jr , McPhee SJ , Papadakis MA, eds. Current Medical Diagnosis & Treatment. 47th ed. New York: McGraw-Hill; 2008.
Q 80.9: A 62-year-old female presents to your office with a complaint of hematuria. She had a cardiac valve replacement three months ago and was placed on warfarin. You perform an in-office international normalized ratio (INR) and discover it is 6.8. Which of the following agents would provide the appropriate reversal of the effects of the warfarin? A folic acid B Fresh frozen plasma C vitamin B D vitamin K (phytonadione) E protamine sulfate
The Correct Answer is: D Excessive anticoagulant effect and bleeding from warfarin can be reversed by stopping the drug and administering oral or parenteral vitamin K1 (phytonadione). The speed and extent of reversal must be balanced against the risk of recurrent thromboembolism in patients who require therapeutic anticoagulation. For example, an over-anticoagulated patient with a prosthetic mitral valve may develop fatal thrombosis if supratherapeutic anticoagulation is rapidly and fully reversed. (Tintinalli, et al., 2011, Chapter 234)
Q 4.5: A 41-year-old alcoholic male, who lives primarily on the streets, appears pale, cachectic, and mildly icteric. He is complaining of several weeks of increasing fatigue. Laboratory findings note an elevated MCV of 128. What other physical finding would most support the diagnosis for megaloblastic anemia? A Decreased vibration and position sense B Dementia C Difficulty with balance D Glossitis E Parethesias
The Correct Answer is: D Features of folate deficiency are similar to vitamin B12 deficiency. However, there are none of the neurologic abnormalities associated with vitamin B12. Glossitis is the only non-neurologic finding in the PE that would support folate deficiency. Alcoholism and poor dietary intake also support the diagnosis of folate deficiency.
Q 109.14: A 46-year-old male presents to your office for consultation of supraventricular tachycardia (SVT). His history shows that he has had multiple episodes of SVT, and on three occasions has been treated in the emergency department with adenosine, which has converted his rhythm back to sinus. The patient is otherwise healthy, has no medical problems, and is on no medications. His vital signs are stable and his physical examination is within normal limits. Based on this history, which medication is the best choice for treating this patient long-term? A Magnesium sulfate B Spironolactone C Digoxin D Flecanide E Diltiazem
The Correct Answer is: D Flecanide doses up to 200 mg BID can be used to help control rate and prevent recurrences of reentry tachycardias. In this case, the use of digoxin would not be ideal, and the other medications would not have a direct effect on the nodal aberration that causes the tachycardia. (Calkins H. Hurst's the Heart, Chapter 38, Supraventricular Tachycardia: AV Nodal Reentry and Wolff-Parkinson-White Syndrome)
Q 32.24: A 72-year-old female presents with headache, jaw claudication, scalp tenderness, and visual changes in her left eye for the past week. She also reports a 4-month history of pain and stiffness in her shoulders and hips. She identifies the pain being worse in the morning and aggravated with getting in and out of the car along with difficulty brushing her hair. She also reports malaise and a 10-pound weight loss over the past few months. Her blood work shows an erythrocyte sedimentation rate (ESR) of 74 mm/h. What is the most likely cause of her jaw claudication? A Raynaud's phenomenon B Systemic lupus erythematosus C Fibromyalgia D Giant cell arteritis E Migraine
The Correct Answer is: D Giant cell (temporal) arteritis (D) is characterized by headache, jaw claudication, polymyalgia rheumatica, visual abnormalities, and a markedly elevated ESR. Giant cell arteritis is a systemic panarteritis affecting medium-sized and large vessels in patients over the age of 50. Giant cell arteritis is also called temporal arteritis because that artery is frequently involved. About 50% of patients with giant cell arteritis also have polymyalgia rheumatica. The classic symptoms suggesting that a patient has arteritis are headache, scalp tenderness, visual symptoms (particularly amaurosis fugax or diplopia), jaw claudication, or throat pain. Of these symptoms, jaw claudication has the highest positive predictive value. Polymyalgia rheumatica is a clinical diagnosis based on pain and stiffness of the shoulder and pelvic girdle areas, frequently in association with fever, malaise, and weight loss. Raynaud's phenomenon (A) affects the digits. SLE (B) is also an autoimmune disorder but mostly occurs in a younger patient population with different symptoms. Fibromyalgia includes multiple trigger point locations, not consistent with this clinical scenario of jaw claudication (C). A migraine (E) does not usually present with this presentation of symptoms.
Q 49.6: A 12-year-old boy presents to the urgent care center complaining of burning pain in his lower extremities with weakness. On examination, the clinician notes symmetric weakness with severely decreased active range of motion of the lower extremities. In addition, there is decreased position and vibratory sensation in the distal portions bilaterally. Upon further questioning, the patient admits to being diagnosed with mononucleosis 2 weeks ago. Which of the following is the most likely diagnosis? A poliomyelitis B botulism C Tick-bite paralysis D Guillain-Barré syndrome
The Correct Answer is: D Guillain-Barré syndrome is most likely due to a delayed hypersensitivity with T-cell-mediated antibodies to mycoplasma and viral infections (CMV, EBV, hepatitis B, campylobacter jejuni). The patients may mention a nonspecific respiratory or gastrointestinal infection 1 to 2 weeks prior to symptoms. Complaints may be paresthesias, weakness in bilateral lower extremities with occasional ascension into the arms, trunk, and face, and rarely ataxia and ophthalmoplegia in the Miller-Fisher variant. Examination findings demonstrate symmetric flaccid weakness, with impairment of position, vibration, and touch in the distal portions of the extremities. If a spinal tap is performed, it may show few polymorphonuclear neutrophils with high protein and normal glucose. EMG is positive for decreased nerve conduction. Laboratory tests may show high titers of suspected infections or active infection of hepatitis/bacterial pathogens. Guillain-Barré is normally a self-limiting disorder within a few weeks, unless there are issues with respiratory depression. Poliomyelitis is secondary to polioviruses and presents with fever, paralysis, meningeal signs, and asymmetrical weakness. Botulism secondary to infection with Clostridium botulinum in older children presents with blurred vision, diplopia, ptosis, choking, and weakness. In infants, botulism presents as constipation, poor suck and cry, apnea, lethargy, and choking. Tick-bite paralysis presents with rapid onset with ascending flaccid paralysis reaching upper extremities in a couple of days of onset and patients often present with paresthesia and pain. Finding of a tick is usually confirmatory for these patients.
Q 13.9: A 28-year-old patient who is a fire department paramedic presents for a routine physical examination to your family practice office. They are asymptomatic, but their PPD is positive. Suddenly, they relay that they have tested positive on their PPD in the past, "about five years ago," and that they were treated at that time with nine months of INH. What should your next step be in treating them? A Begin treatment for resistant tuberculosis (TB). B Nothing, inasmuch as they have already received a full course of therapy. C Order a chest x-ray and if normal, follow the patient with chest x-rays every two to three years or sooner if they become symptomatic D Order a chest x-ray, and if normal, clear the patient cautioning them to immediately contact you should they develop any signs or symptoms consistent with TB. E Refer the patient to ID or Pulmonary for definitive treatment of resistant TB.
The Correct Answer is: D Health care workers (HCWs) with positive PPD test results should have a chest radiograph as part of the initial evaluation of their PPD test; if negative, repeat chest radiographs are not needed unless symptoms develop that could be attributed to TB. However, more frequent monitoring for symptoms of TB may be considered for recent converters and other PPD-positive HCWs who are at increased risk for developing active TB (e.g., HIV-infected or otherwise severely immunocompromised HCWs). Regardless of whether the patient completes treatment for latent TB infection, serial or repeat chest radiographs are not indicated unless the patient develops signs or symptoms suggestive of TB disease.
Q 71.8: A 30-year-old female patient comes into your family practice office to discuss the fact that her mother just tested positive for the BRCA 1 and BRCA 2 genes. She has also been tested and is positive. What is her risk for developing breast cancer by the age of 70? A Her risk is identical to the population. B Her risk is approximately 25%. C Her risk is approximately 55%. D Her risk is approximately 85%. E Her risk is approaches 100%.
The Correct Answer is: D Hereditary breast and ovarian cancers put both male and female patients at increased risk for multiple cancers, including breast, ovarian, prostate, and pancreatic. In this case, the risk of breast cancer by the age of 70 is approximately 85%. When in doubt, consult a genetic counselor. (Kantarjian et al., 2006, Chapter 21)
A 35-year-old man is brought to the emergency department with unremitting, generalized convulsive status epilepticus. The initial, preferred treatment is intravenous administration of which of the following? A phenobarbital B valproate C phenytoin D lorazepam E donepezil
The Correct Answer is: D In most patients suffering from generalized convulsive status epilepticus (GCSE), benzodiazepines such as lorazepam and diazepam are effective initial therapies due to their relatively high lipid solubility. As a result, they are able to cross the blood-brain barrier easily, which gives them the potential to stop seizures quickly. Lorazepam's lipid solubility is less compared to diazepam, and it also redistributes to fat more slowly. Hence, lorazepam tends to have a longer duration of action (12 to 24 hours) than diazepam (20 to 30 minutes). Phenytoin is often administered immediately after benzodiazepine administration for long-term seizure control, as it has a long half-life (20 to 36 hours) compared to diazepam. Phenytoin is not given first because its lipid solubility is less than the benzodiazepines and therefore cannot enter the brain quickly enough to terminate seizure activity.
A 62-year-old man with a history of hypertension, diabetes mellitus type 2, hyperlipidemia, and chronic tobacco use presents to the office with complaints of a retrosternal chest pressure associated with diaphoresis, nausea, and dyspnea, radiating down his left arm for the last 45 minutes after mowing his lawn. The patient's vital signs are stable, and on physical examination a new systolic murmur is appreciated. His EKG demonstrates evidence of acute anterolateral myocardial infarction on EKG, with ST segment elevation across the precordial leads, indicative of left anterior descending coronary artery stenosis. Which of the following is the most appropriate next step in management of this patient? A Dobutamine stress echocardiogram B Transthoracic echocardiogram C Nuclear stress test D Cardiac catheterization E Exercise treadmill stress test
The Correct Answer is: D In patients suffering from acute ST elevation myocardial infarction (STEMI), cardiac catheterization with percutaneous coronary intervention within 90 minutes substantially decreases morbidity and mortality outcomes; thus, choice D is the most appropriate next step in management of this patient. A transthoracic echocardiogram, choice B, will likely be performed during this patient's hospitalization, as it is can assist in assessment of complications of MI. However, the primary goal for acute STEMI is reperfusion in a timely manner, and thus TTE would not be the next step in the management of this patient. Choices A, C, and E are all forms of stress testing, which are useful testing modalities for patients with chest pain who are not actively infarcting.
Q 114.1: A 62-year-old man with a history of hypertension, diabetes mellitus type 2, hyperlipidemia, and chronic tobacco use presents to the office with complaints of a retrosternal chest pressure associated with diaphoresis, nausea, and dyspnea, radiating down his left arm for the last 45 minutes after mowing his lawn. The patient's vital signs are stable, and on physical examination a new systolic murmur is appreciated. His EKG demonstrates evidence of acute anterolateral myocardial infarction on EKG, with ST segment elevation across the precordial leads, indicative of left anterior descending coronary artery stenosis. Which of the following is the most appropriate next step in management of this patient? A Dobutamine stress echocardiogram B Transthoracic echocardiogram C Nuclear stress test D Cardiac catheterization E Exercise treadmill stress test
The Correct Answer is: D In patients suffering from acute ST elevation myocardial infarction (STEMI), cardiac catheterization with percutaneous coronary intervention within 90 minutes substantially decreases morbidity and mortality outcomes; thus, choice D is the most appropriate next step in management of this patient. A transthoracic echocardiogram, choice B, will likely be performed during this patient's hospitalization, as it is can assist in assessment of complications of MI. However, the primary goal for acute STEMI is reperfusion in a timely manner, and thus TTE would not be the next step in the management of this patient. Choices A, C, and E are all forms of stress testing, which are useful testing modalities for patients with chest pain who are not actively infarcting. (McPhee et al., 2011, Chapter 10)
Q 110.9: An 18-year-old woman comes in for evaluation of "losing weight without meaning to." She also feels weak and in "always in the bathroom." Her appetite is normal but she "can't get enough to drink." Examination shows that she has lost 17# since her last visit a year ago. She is 66" tall and now weighs 120#. She is mildly orthostatic, but no other abnormalities are noted. A random blood sugar done in the office is 260mg/dl. Which results are most likely on measurement of her lipoproteins at this time? A decreased high density lipoprotein level B extremely elevated triglycerides C markedly increased total cholesterol D mildly elevated low density lipoproteins E normal profile
The Correct Answer is: D In persons with diabetes mellitus, type 1, low density lipoproteins, trigycerides (B), and total cholesterol (C) are likely to be slightly elevatated. High density lipoproteins (A) remain about the same as the patient's baseline. Once the glucose level is controlled, the lipid levels (E) on the profile typically return to normal. Masharani U, Diabetes Mellitus & Hypoglycemia, in Current Medical Diagnosis and Treatment, 52 nd ed. 2013.
Q 97.38: A progestin-only contraceptive, or "minipill," would be most appropriate for which of the following patients? A a 25-year-old woman in excellent overall health B a 28-year-old woman with a history of epilepsy C a 32-year-old woman with a history of pelvic inflammatory disease D a 37-year-old woman who smokes 2 packs per day and has a history of hypertension E a 38-year-old woman with a history of asthma and bronchitis
The Correct Answer is: D In the majority of cases, a combined hormonal contraceptive (ie, one that contains both an estrogen and progestin) is the preferred method of oral contraception because of its efficacy when used perfectly (>99%). However, for women older than 35 years of age who are smokers or are obese, or who have a history of hypertension or vascular disease, progesterone-only contraceptives are recommended. Ethinyl estradiol (EE), the most common estrogen found in combined hormonal contraceptives, has been associated with an increased risk of myocardial infarction in women older than 35 years of age who are smokers. Additionally, EE has also been shown to cause increases in blood pressure in both normotensive and mildly hypertensive women. Progestin-only contraceptives, however, tend to be less effective than the combined hormonal contraceptives.
Q 15.8: An 8-month-old male presents to the clinic due to irritability, fatigue and parental concerns about developmental delays. He is fifth percentile for weight and 33rd percentile for height. An office based hemoglobin level is 8.4 mg/dL. Which of the following is the most likely diagnosis? A Alpha thalassemia minor B Beta thalassemia minor C Folate deficiency D Iron deficiency anemia E Sideroblastic anemia
The Correct Answer is: D Infants with poor nutrition are at greatest risk of iron deficiency anemia between the ages of 6 and 24 months. Alpha thalassemia minor (A) and beta thalassemia minor (B) present with a markedly microcytic, but mild anemia. Folate deficiency (C) due to inadequate nutrition in the absence of co-morbid disease is rare. Sideroblastic anemia (E) is far less common than iron deficiency anemia and typically diagnosed based on the finding of ringed sideroblasts visible on peripheral smear.
Q 43.3: A 2-year-old baby girl is brought to the ED with a history of abdominal pain and diarrhea. Mother states that the child was playing normally and then "doubled over" with what appears to be abdominal pain. The abdomen appears slightly distended and is tender to palpation. While in the ED the child has a bloody, diarrheal bowel movement. Which of the following is the most likely diagnosis? A pyloric stenosis B mesenteric ischemia C Crohn disease D intussusception E Hirschsprung disease
The Correct Answer is: D Intussusception is the most frequent cause of intestinal obstruction in the first 2 years of life. The patient develops paroxysms of pain followed by bloody bowel movements. Pyloric stenosis typically presents prior to the age of 6 months with vomiting but not with diarrhea. Hirschsprung disease results from an absence of ganglion cells in the colon and typically presents early in life with failure to pass meconium, followed by vomiting and abdominal distension. The typical age of onset is later in adolescence in Crohn disease and in the elderly in mesenteric ischemia.
Q 97.47: A 35-year-old nulliparous female has had increasing and heavy flow with her menstrual cycles over the last six months. She is being followed by her gynecologist, who has diagnosed several small uterine fibroids. She presents in clinic today with complaints of tachypnea on exertion, tachycardia, and occasional palpitations. There is no family history of cardiac problems or hypertension. She denies symptoms of infectious etiology and has no fever. On physical examination, you note a mild tachycardia of 110 after she has been sitting for a few minutes. No other physical findings are noted. What would be the next most useful diagnostic study to help you confirm the diagnosis? A CBC B Chest x-ray C EKG D Serum ferritin E Transthoracic ECHO
The Correct Answer is: D Iron deficiency develops in stages. The first is depletion of iron stores. There is no anemia or change in RBC. The serum ferritin will become abnormally low and lead to symptoms of anemia, such as easy fatigability, tachycardia, palpitations, and tachypnea on exertion. Iron deficiency anemia is more frequently seen in women, especially those with heavy menstrual cycles, as they are unable to absorb enough iron in their diet to maintain stores—leading to depletion, signs of anemia, and progression if not treated.
Q 98.29: A 70-year-old man with a history of hypertension, DM Type 2, and hyperlipidemia is seen for pre-operative evaluation prior to left knee replacement. On auscultation, a very soft high-frequency decrescendo early diastolic murmur is heard at the upper left sternal border. Utilizing isometric hand grip exercises, the murmur increases in intensity and can be heard radiating to the left sternal border and apex. Given the patient's physical exam findings, which of the following is the most likely diagnosis? A Aortic stenosis B Ventricular septal defect C Mitral stenosis D Aortic regurgitation/insufficiency E Mitral regurgitation/insufficiency
The Correct Answer is: D Isometric hand grip exercises will increase the intensity of the murmur of aortic regurgitation, which is usually described as a high-frequency decrescendo early diastolic murmur that is heard best at the left upper sternal border or at the right upper sternal border. Radiation, if it occurs, is frequently to the lower left sternal border and the apex. Isometric hand exercises increase arterial and left ventricular pressure, which increases the flow across the aortic valve, thereby increasing the murmur's intensity. (Crawford et al., 2009, Chapter 1)
Q 43.1: A 3-week-old male infant is brought in by his mother due to his vomiting. The mother notes that a few days ago her son started vomiting after feeding, and it has become projectile in nature. The vomitus is non-bilious and contains no blood. The child seems hungry and nurses regularly, but the vomiting has become more frequent and is occurring with every feeding now. On physical examination, an oval mass is palpated in the right upper quadrant. What laboratory finding is most consistent with the suspected diagnosis? A Decreased hematocrit B Elevated amylase C Hyperkalemia D Hypochloremic alkalosis E Hyponatremia
The Correct Answer is: D Laboratory findings that are consistent with the suspected diagnosis of pyloric stenosis include hypochloremic alkalosis with potassium depletion. An elevated hemoglobin and hematocrit can also be present, and are due to dehydration. Changes in amylase or serum sodium are not present in cases of pyloric stenosis.
Q 38.8: A 6-year-old female child presents with complaints of chronic hip pain so severe that she has not been able to walk to the school bus. Examination shows severe tenderness at the left hip with markedly decreased active and passive range of motion. Radiologic examination demonstrates joint effusion with widening. Which of the following is the most likely diagnosis? A osteochondritis dissecans B slipped capital femoral epiphysis C septic hip arthritis D Legg-Calvé-Perthes disease
The Correct Answer is: D Legg-Calvé-Perthes disease is also known as avascular necrosis of the proximal femur. It typically occurs in children between 4 and 8 years old and persistent hip pain is the main symptom. On examination, the clinician notices a limp and/or limitation of motion of the affected hip. Radiologic examination demonstrates the necrosis with effusion and joint space widening with a negative aspirate. Treatment involves surgical hip replacement. Slipped capital femoral epiphysis (SCFE) is due to the displacement of the proximal femoral epiphysis owing to disruption of the growth plate. The head is normally displaced medially and posteriorly relative to the femoral neck. It typically occurs in adolescence, specifically obese males, and can also be associated with hypothyroidism. SCFE usually occurs after direct trauma to the hip or a fall. Patients complain of vague symptoms at first that progress into pain of the hip or of the knee. On examination, there is decreased internal rotation of the hip that can be confirmed by lateral X-ray of the hip. Septic hip arthritis is not common in children between the age of 5 and 12 years. The legs are held in external rotation to minimize pain and will have a positive aspirate. Osteochondritis dissecans typically presents in the knee, elbow, and talus and is characterized by a wedge-shaped necrosis of bone.
Q 88.7: When used for advanced carcinoma of the prostate, chronic administration of leuprolide inhibits the synthesis of androgens by _____________________ A blocking gonadotropin-releasing hormone (GnRH) receptors at the anterior pituitary B blocking luteinizing hormone (LH) receptors on interstitial (Leydig) cells of the testes C increasing the secretion of GnRH from the hypothalamus D inhibiting pulsatile secretion of gonadotropins from the anterior pituitary E upregulation of the number of GnRH receptors at the anterior pituitary
The Correct Answer is: D Leuprolide is a GnRH (LHRH) agonist that suppresses the pulsatile secretion of follicle stimulating hormone (FSH) and LH (gonadotropins) from the anterior pituitary when given chronically. Continuous administration of a GnRH agonist causes down-regulation of GnRH receptors on gonadotropes, which, in turn suppresses gonadotropin release and gonadal function. Decreased amounts of LH, in particular, lead to diminished production of androgens by the testes (especially DHT), which support prostate growth. It is believed that by interrupting the hormonal pathways that modulate prostatic growth, tumor development and metastasis is slowed. (Kolesar, 2008, p. 2212; Jones, 2009, pp. 1151-1152) Kolesar JM. Prostate cancer. In: DiPiro JT , Talbert RL , Yee GC, et al., eds. Pharmacotherapy: A Pathophysiologic Approach. 7th ed. New York: McGraw-Hill; 2008. Jones EE. The female reproductive system. In: Boron WF , Boulpaep EL, eds. Medical Physiology. 2nd. Philadelphia, PA; Saunders Elsevier; 2009.
Q 10.7: A 19 year-old woman presents to the clinic complaining of fever, and productive cough. Physical exam reveals a temperature of 102F, decreased breath sounds of her right middle lobe. She is prescribed levofloxacin. What is the mechanism of action of levofloxacin? A Binds to the 30S ribosomal subunit to inhibit protein synthesis B Binds to the 50S ribosomal subunit to inhibit protein synthesis C Inhibits dihydropteroate synthase and folate production D Inhibits DNA replication by binding to DNA gyrase and topoisomerase IV E Inhibits the transpeptidation reaction
The Correct Answer is: D Levofloxacin inhibits DNA replication by binding to DNA gyrase and topoisomerase IV (D). Tetracycline binds to the 30S ribosomal subunit to inhibit protein synthesis (A). Macrolide antibiotics bind to the 50S ribosomal subunit to inhibit protein synthesis (B). Sulfamethoxazole inhibits dihydropteroate synthase and folate production (C). Beta lactam antibiotics inhibit the transpeptidation reaction leading to cell wall destruction (E).
Q 30.10: Which physical complaint is the leading cause of lost work time and disability for patients under the age of 45? A Ankle pain B Hip pain C Knee pain D Low back pain E Shoulder pain
The Correct Answer is: D Low back pain is the most common causes of lost time from work and disability. Factors that can lead to back problems include repeated or heavy lifting and twisting or use of vibrating equipment along with poor fitness levels, smoking, poor job satisfaction and other psychosocial factors. Fortunately, 80% of patients experience significant recovery within one month. All of the other types of pain can occur based on multiple factors, but none of them occur with a frequency that rivals low back pain.
Which physical complaint is the leading cause of lost work time and disability for patients under the age of 45? A Ankle pain B Hip pain C Knee pain D Low back pain E Shoulder pain
The Correct Answer is: D Low back pain is the most common causes of lost time from work and disability. Factors that can lead to back problems include repeated or heavy lifting and twisting or use of vibrating equipment along with poor fitness levels, smoking, poor job satisfaction and other psychosocial factors. Fortunately, 80% of patients experience significant recovery within one month. All of the other types of pain can occur based on multiple factors, but none of them occur with a frequency that rivals low back pain.
Q 107.9: A 53-year-old man is taking a proton pump inhibitor for GERD symptoms, a beta blocker and a thiazide diuretic for hypertension, an SSRI for depression, and an over-the-counter NSAID as needed for aches and pains. He has developed gynecomastia and laboratory studies reveal an elevated prolactin level. If his hyperprolactinemai is due to one of his medications, which is the most likely cause? A the beta blocker B the NSAID C the proton pump inhibitor D the SSRI E the thiazide diuretic
The Correct Answer is: D Many medications cause hyperprolactinemia, including SSRIs, tricyclic antidepressants, and antipsychotics. Hydralazine and methyldopa, but not beta blockers (A), may also raise prolactin levels; likewise opioids, but not NSAIDS (B). Cimetidine and ranitidine, but not proton pump inhibitors (C) are included among possible pharmaceutical causes. Thiazide diuretics (E) are not know to raise prolactin levels.
A 54-year-old man presents to the emergency department with crampy abdominal pain, nausea, and vomiting. The patient has not passed gas or had a bowel movement for at least 10 hours. On examination, the abdomen is distended and there are high-pitched bowel sounds with rushes. A plain radiograph of the abdomen reveals cecal distension to 12 cm. What is the most appropriate definitive management for this patient? A Intravenous fluids B Nasogastric suction C Observation D Surgical exploration
The Correct Answer is: D Massive distention of the cecum, as detected on plain radiograph, is typically seen in "closed loop" obstructions where the ileocecal valve is competent. When distention approaches 12 cm, there is an increased risk of perforation and/or gangrene. Expedient surgical intervention is indicated. Although observation with intravenous fluids and nasogastric decompression are important adjuncts to management, surgical exploration is the only way to rapidly address this emergent situation.
Q 98.24: A 54-year-old man presents to the emergency department with crampy abdominal pain, nausea, and vomiting. The patient has not passed gas or had a bowel movement for at least 10 hours. On examination, the abdomen is distended and there are high-pitched bowel sounds with rushes. A plain radiograph of the abdomen reveals cecal distension to 12 cm. What is the most appropriate definitive management for this patient? A Intravenous fluids B Nasogastric suction C Observation D Surgical exploration
The Correct Answer is: D Massive distention of the cecum, as detected on plain radiograph, is typically seen in "closed loop" obstructions where the ileocecal valve is competent. When distention approaches 12 cm, there is an increased risk of perforation and/or gangrene. Expedient surgical intervention is indicated. Although observation with intravenous fluids and nasogastric decompression are important adjuncts to management, surgical exploration is the only way to rapidly address this emergent situation.
Q 102.8: A 34-year-old man presents to your office with the complaint of pain and fatigue in his right wrist. He states that the pain can sometimes wake him up at night and feels as if his thumb is falling asleep. He reports the problem started since he has been writing a book. On physical examination, you note a positive Tinel sign but no response to a Phalen maneuver. While sending him for a nerve conduction study, you tell him the most likely diagnosis is A ulnar nerve compression B radial nerve compression C thoracic outlet syndrome D median nerve compression E peroneal nerve compression
The Correct Answer is: D Median nerve compression can be precipitated by repetitive use of the wrist or hand or by compression of the median nerve within the carpal tunnel at the wrist from inflammation or trauma. Characteristic fatigue and pain, especially at night, may be accompanied by paresthesia in the median nerve distribution. The pain may be reproduced on examination by performing either the Tinel or Phalen maneuver but electrophysiology studies will usually confirm the entrapment. (Aminoff et al., 2005, pp. 221-222) Aminoff MJ , Greenberg DA , Simon RP. Clinical Neurology. 6th ed. New York, NY: McGraw-Hill; 2005.
Q 94.3: Your 45-year-old diabetic on a statin develops myalgias and you elect to check a creatine phosphokinase (CPK). While a normal CPK is 150 U/L, your patient has an elevated CPK. At what point must you discontinue the use of the statin? A > 150 U/L B > 300 U/L C > 450 U/L D > 1500 U/L E none of the above
The Correct Answer is: D Minor increases in creatine kinase (CK) activity in plasma are observed in some patients receiving reductase inhibitors, frequently associated with heavy physical activity. Rarely, patients may have marked elevations in CK activity, often accompanied by generalized discomfort or weakness in skeletal muscles. In all patients, CK should be measured at baseline. If muscle pain, tenderness, or weakness appears, CK should be measured immediately and the drug discontinued if activity is elevated significantly over baseline (> 1500 U/L). The myopathy usually reverses promptly upon cessation of therapy. If the association is unclear, the patient can be re-challenged under close surveillance. Myopathy in the absence of elevated CK has been reported. (Brunton, et al., 2006, Chapter 35)
Q 104.10: A 28-year-old woman who was born and brought up just outside of Washington, DC, comes in for evaluation of vague "problems with swallowing." She has no other symptoms except "my neck is bigger than it used to be." Examination reveals only a diffuse, somewhat irregular, nontender enlargement of the thyroid gland with distinct masses palpable within it. What is the most likely diagnosis? A endemic goiter B Graves disease C Hashimoto thyroiditis D multinodular goiter E thyroid carcinoma
The Correct Answer is: D Multinodular goiter is the most likely in a woman with these findings in the United States. It may be nontoxic as in this case or toxic, i.e., producing excessive thyroid hormones which cause symptoms of hyperthyroidism. Endemic goiter (A), which may present as a simple enlargement of the thyroid or as a multinodular one, is found almost entirely in iodine-deficient areas of the world and is extremely rare in the U.S. In Graves disease (B), the thyroid is enlarged and may exhibit a thrill and a bruit. In addition, the patient would have other signs of hyperthyroidism. The thyroid in Hashimoto thyroiditis (C) is diffusely enlarged and firm with fine nodules. A thyroid carcinoma (E) usually presents as a firm, nontender nodule in the gland. Fitzgeral PA, Endocrine Disorders, in Current Medical Diagnosis and Treatment, 52 nd ed. 2013.
Q 117.16: A 63-year-old male with type 2 diabetes mellitus and hyperlipidemia is being seen for routine blood work to assess his renal function. Blood pressure is 130/90 and pulse is 75. His blood chemistries show hypoalbuminemia and hypoproteinemia. His urinalysis shows urine protein excretion of 3.5 grams per 24 hours and microscopically shows oval fat bodies in the urine. The following lab results are within normal range: WBC, BUN, and creatinine. What is the suspected diagnosis? A Pyelonephritis B Glomerulonephritis C Acute renal failure D Nephrotic syndrome E Wilms tumor
The Correct Answer is: D Nephrotic syndrome (D) is diagnosed with bland urine sedimentation, urine protein excretion > 3 g per 24 hours, hypoalbuminemia of < 3g/dl, peripheral edema, hyperlipidemia, and oval fat bodies in the urine. In adults, roughly one-third of patients diagnosed with nephrotic syndrome also have a concurrent systemic disease such as diabetes mellitus, amyloidosis, or systemic lupus erythematosus. Serum creatinine may or may not be abnormal at the time of presentation, depending on the severity, acuity, and chronicity of the disease. Only nephrotic syndrome would show oval fat bodies in his urine. Glomerulonephritis (B) would should hypertension. Pyelonephritis (A) would show an increased WBC count. Acute renal failure (C) would show a change in BUN/creatinine. Wilms' tumor (E) is seen in children and does not have this presentation. Watnick S, Dirkx T. Chapter 22. Kidney Disease. In: Papadakis MA, McPhee SJ, Rabow MW, eds. CURRENT Medical Diagnosis & Treatment 2013. New York: McGraw-Hill; 2013. http://www.accessmedicine.com/content.aspx?aID=11374 . Accessed March 6, 2013.
Q 102.4: Which of the following is diagnostic of nephrotic syndrome? A hypoalbuminemia, hypolipidemia, proteinuria >10 g/24 h B hypoalbuminemia, hyperlipidemia, proteinuria >1 g/24 h C hypoalbuminemia, hyperlipidemia, proteinuria >2 g/24 h D hypoalbuminemia, hyperlipidemia, proteinuria >3.5 g/24 h E normal albumin, hyperlipidemia, proteinuria >10 g/24 h
The Correct Answer is: D Nephrotic syndrome is defined as proteinuria >3.5 g/24 h resulting in hypoalbuminemia (<3.0 g/dL), hyperlipidemia (total cholesterol >250 mg/dL), and edema, probably due to increased renal tubule permeability. Causes include diabetic nephropathy, HIV nephropathy, chronic hepatitis B and C, amyloidosis, systemic lupus erythematosus, constrictive pericarditis, Hodgkin's disease, minimal change disease, and many medications, including phenytoin and NSAIDs. (Watnick and Morrison, 2009, pp. 815-817) Watnick S , Morrison G. Kidney. In: Tierney LM , McPhee SJ , Papadakis MA, eds. Current Medical Diagnosis and Treatment. 48th ed. New York, NY: McGraw-Hill; 2009.
Q 104.4: Which of the following symptoms is most concerning for gastric cancer? A Dyspepsia exacerbated by meals B Alternating bowel habits from constipation to diarrhea C Dyspepsia that is exacerbated by recumbency D New-onset of dyspepsia in a 58 year-old patient E Steatorrhea
The Correct Answer is: D New-onset dyspepsia in a patient more than 55 years of age is the most concerning symptom of the ones listed for gastric cancer. It is quite common for GERD symptoms to be exacerbated by meals and recumbency. Alternating bowel habits, from constipation to diarrhea, can be associated with colon cancer. Steatorrhea can occur in many disease processes involving the small intestine, but is not typical in gastric cancer. (McPhee SJ, Papadakis MA. Current Medical Diagnosis & Treatment, 2010, p. 1472)
Q 69.4: A 62-year-old female complains of headaches and swelling in her legs for the past 3 months. Her current medications included in the choices below. Which of these medications is most likely responsible for both of her symptoms? A acetaminophen B hydrochlorothiazide (HCTZ) C olmesartan D nifedipine E atorvastatin
The Correct Answer is: D Nifedipine, a calcium channel blocker, is the most likely cause for the patient's complaints. Calcium channel blockers, especially those in the dihydropyridine class, are commonly associated with side effects of headaches and peripheral edema. Olmesartan, an ARB, is not commonly associated with headaches and peripheral edema. HCTZ is a diuretic that would more likely improve edema than cause it. Atorvastatin is a statin used for treatment of hyperlipidemia and is more likely to be associated with myalgias. Acetaminophen is unlikely to cause edema and may actually be used to treat headaches. (McPhee and Papadakis, 2011, Chapter 11)
Q 97.53: A concerned mother brings her 13-year-old daughter in for evaluation. All the child's female friends have begun to menstruate, and this child has "barely begun to develop." On examination, the teenager appears well. She is 5'4" tall and weighs 114 pounds. Her breasts demonstrate buds with a small amount of elevation of the breast itself, and fine hair that is just beginning to curl on the labia majora. What is the most appropriate education in this situation? A She will begin to menstruate in the next month or two. B Teens who are underweight often develop more slowly than others. C The other girls must have undergone precocious puberty. D Your daughter is developing at the time that is right for her. E We should perform some genetic testing on your daughter.
The Correct Answer is: D Normal puberty occurs within a wide range of ages. Girls are not considered "delayed" until they have shown no sexual development by age 14 or have not menstruated by age 17. This teen is approaching Tanner Stage 3 of development and should menstruate when she has attained Stage 4 (A) in another six months to a year. Very thin or highly athletic girls may not reach puberty as quickly or may cease to menstruate, but this girl's weight is within normal limits for her height. (B) The age of onset of puberty in the developed countries is between 8 and 13 (C). Since this girl is progressing normally and appears well, no genetic testing (E) is indicated.
Q 95.6: A 60-year-old man newly diagnosed with type 2 diabetes mellitus is most likely to have which of the following on ophthalmologic evaluation? A diabetic cataracts B glaucoma C nonproliferative retinopathy D normal fundi E proliferative retinopathy
The Correct Answer is: D Only about 20% of patients with type 2 diabetes have retinopathy at the time they are diagnosed. All patients with diabetes are at increased risk for premature development of cataracts (A) and glaucoma (B) occurs eventually in about 6% of diabetics. Nonproliferative retinopathy (C) is present in just under 2/3 of patients who have had type 2 diabetes for 16 years, and proliferative retinopathy (E) is far more common in patients with type 1. Masharani U, Diabetes Mellitus & Hypoglycemia, in Current Medical Diagnosis and Treatment, 52 nd ed. 2013.
Q 98.48: The rotator cuff is comprised of which four muscles? A Scalenes, infraspinatus, teres minor, subscapularis B Supraspinatus, infraspinatus, teres major, subscapularis C Supraspinatus, infraspinatus, teres minor, soleus D Supraspinatus, infraspinatus, teres minor, subscapularis E Supraspinatus, intercostals, teres minor, subscapularis
The Correct Answer is: D Only option D lists the correct muscles of the rotator cuff located in the shoulder area that contribute to arm elevation, internal rotation and external rotation. Of the teres muscle group, it is the teres minor, not teres major that is part of the rotator cuff. The scalene muscles are in the neck and cause rib elevation and neck movement. The soleus muscles cause plantar flexion of the feet and are located in the calf area of the posterior lower leg.
Q 109.20: A 28-year-old man presents with a complaint of new-onset headache. The pain awakens him early in the morning and is described as a sharp, lancinating pain around his right eye, which is 9 out of 10. When he looks in the mirror he notices tearing of his right eye as well as redness and a different sized pupil compared to the left. The pain lasts only for a few minutes but can recur later in the morning. This has happened for the past several days. The patient has a history of recurrent headaches that follow this pattern and usually last for 5 to 7 days. Prior to this occurrence it has been 4 years since his last episode. Which of the following is the most appropriate preventive treatment the patient should be offered at this time? A sumatriptan (Imitrex) B dihydroergotamine C verapamil (Calan) D oral corticosteroids E oxygen
The Correct Answer is: D Oral corticosteroids started immediately will often force a cluster cycle into remission and prevent future headaches. The infrequent recurrence pattern of the cluster headaches does not support the chronic use of verapamil. Sumatriptan, dihydroergotamine, and oxygen are very useful abortive agents but do not work on stopping the cluster cycle. Both sumatriptan and dihydroergotamine have maximum daily doses and may not be used for all attacks during the day as some cluster patients may have multiple attacks throughout the day. (Aminoff et al., 2005, pp. 90-91) Aminoff MJ , Greenberg DA , Simon RP. Clinical Neurology. 6th ed. New York, NY: McGraw-Hill; 2005.
Q 98.53: A 54-year-old man complains of persistent midepigastric abdominal pain 2 weeks following the diagnosis of acute pancreatitis. The patient also complains of anorexia but no fever or chills. There is a palpable mass in the midepigastrium; bowel sounds are normal in all four quadrants. Which of the following is most likely diagnosis? A Adynamic ileus B Pancreatic carcinoma C Infected pancreatic necrosis D Pancreatic pseudocyst
The Correct Answer is: D Pancreatic pseudocysts are the most common complication associated with acute pancreatitis. A pseudocyst should be suspected for a patient who has continued abdominal pain, the development of an abdominal mass, and continued elevations of amylase or lipase levels following an episode of acute pancreatitis. An adynamic ileus would be associated with abdominal distension and changes in bowel sounds; an infected area of necrosis within the pancreatic gland would be associated with fever. Pancreatic cancer may be seen in conjunction with chronic pancreatitis.
Q 106.3: A 54-year-old man complains of persistent midepigastric abdominal pain 2 weeks following the diagnosis of acute pancreatitis. The patient also complains of anorexia but no fever or chills. There is a palpable mass in the midepigastrium; bowel sounds are normal in all four quadrants. Which of the following is most likely diagnosis? A Adynamic ileus B Pancreatic carcinoma C Infected pancreatic necrosis D Pancreatic pseudocyst
The Correct Answer is: D Pancreatic pseudocysts are the most common complication associated with acute pancreatitis. A pseudocyst should be suspected for a patient who has continued abdominal pain, the development of an abdominal mass, and continued elevations of amylase or lipase levels following an episode of acute pancreatitis. An adynamic ileus would be associated with abdominal distension and changes in bowel sounds; an infected area of necrosis within the pancreatic gland would be associated with fever. Pancreatic cancer may be seen in conjunction with chronic pancreatitis. (Sharp, 2006, p. 359) Sharp KW , Goldin SB , Lomis KD. Pancreas. In: Lawrence PF, ed. Essentials of General Surgery. 4th ed. Philadelphia, PA: Lippincott Williams & Wilkins; 2006.
Q 98.30: A 60-year-old man presented with a mass in the left lobe of the thyroid. Fine needle aspiration was consistent with papillary carcinoma. There was no evidence of locally invasive or metastatic disease. Which of the following treatments is recommended for this patient? A Chemotherapy B External beam radiation C Preoperative radioiodine ablation D Total thyroidectomy
The Correct Answer is: D Papillary carcinoma is the most common type of thyroid malignancy. Treatment includes a thyroid lobectomy and isthmusectomy or total thyroidectomy. The decision regarding the extent of the surgery is based on the extent of the disease, the tumor size, and histiologic grade. A poor prognosis is seen in males, patients older than 50 years of age, primary tumors greater than 4 cm in size, tumors that are less well differentiated, or evidence of locally invasive or metastatic disease. Accordingly, the recommended treatment for this patient is a total thyroidectomy. Radioiodine ablation is recommended postoperatively.
Q 110.7: A 60-year-old man presented with a mass in the left lobe of the thyroid. Fine needle aspiration was consistent with papillary carcinoma. There was no evidence of locally invasive or metastatic disease. Which of the following treatments is recommended for this patient? A Chemotherapy B External beam radiation C Preoperative radioiodine ablation D Total thyroidectomy
The Correct Answer is: D Papillary carcinoma is the most common type of thyroid malignancy. Treatment includes a thyroid lobectomy and isthmusectomy or total thyroidectomy. The decision regarding the extent of the surgery is based on the extent of the disease, the tumor size, and histiologic grade. A poor prognosis is seen in males, patients older than 50 years of age, primary tumors greater than 4 cm in size, tumors that are less well differentiated, or evidence of locally invasive or metastatic disease. Accordingly, the recommended treatment for this patient is a total thyroidectomy. Radioiodine ablation is recommended postoperatively. (Coe, 2006, pp. 407-408) Coe NPW. Surgical endocrinology: thyroid gland. In: Lawrence PF, ed. Essentials of General Surgery. 4th ed. Philadelphia, PA: Lippincott Williams & Wilkins; 2006.
Q 26.10: A 17-year-old female distance runner with no significant PMH complains that she has diffuse, aching anterior knee pain that is worsened when she walks up or down stairs or when she squats down. There has been no acute trauma, but she has been increasing her running mileage. No effusion is present. What is the probable diagnosis? A Anterior cruciate ligament tear B Medial meniscal tear C Osteoarthritis of the knee joint D Patellofemoral syndrome E Posterior cruciate ligament tear
The Correct Answer is: D Patellofemoral syndrome is a common condition in active adolescents due to repetitive stresses on the patellofemoral joint. This can be exacerbated by altered patellofemoral tracking due to growth and development in adolescents. Other biomechanical issues that may contribute to this problem include poor flexibility, weakness of the vastus medialis muscle, which results in a more lateral tracking of the patella, and excessive foot pronation. This condition does not produce an effusion. Anterior and posterior ligament tears would be highly unlikely given that there was no history of an acutely traumatic event and because ligaments are fairly vascular structures, an effusion would be expected with any cruciate ligament tear. Meniscal tears in a young patient are generally associated with a traumatic twisting event and while not as vascular as a ligament, meniscal tears generally lead to the gradual development of an effusion. Older patients may experience a degenerative tear with minimal trauma, but that is unlikely in our 17-year-old patient. Osteoarthritis is generally associated with gradual wear and tear after many years or the process can be accelerated after a significant acute trauma to a joint. Neither scenario is applicable to this patient.
Q 46.7: Upon performing a newborn examination, the clinician notes a widened pulse pressure, paradoxical splitting of S 2 , and a "machine"-like murmur heard best at the second intercostal space, left sternal border, and inferior to the clavicle. Which of the following is the most likely diagnosis? A tetralogy of Fallot B ventricular septal defect C atrial septal defect D patent ductus arteriosus
The Correct Answer is: D Patent ductus arteriosus (PDA) is an isolated abnormality that occurs in infants. The ductus arteriosus is a normal fetal vessel that joins the aorta and the pulmonary artery and spontaneously closes after 3 to 5 days. Lack of closure results in the audible murmur that is "machine-like" and maximal at the second intercostal space (ICS), at the left sternal border (LSB), and inferior to the clavicle. It is typically a pansystolic murmur with bounding pulses and a widened pulse pressure. There is also a paradoxical splitting of S 1 and S 2 . Echocardiography confirms the PDA, the direction and degree of shunting, and the presence of lesions for which the PDA is needed to keep. If there are no other cardiac malformations requiring the PDA, then if the PDA is large, surgery should be completed before 1 year of age. Symptomatic PDAs that are relatively small may be closed with indomethacin in preterm infants. The murmur heard in atrial septal defect (ASD) usually is an ejection type, systolic murmur heard best at the LSB, second ICS with a wide, fixed S 2 and normal pulses. Ventricular septal defect (VSD) presents with a harsh, pansystolic murmur heard best at the third and fourth ICS. With increasing size of the VSD, heaves, thrills, and lifts are present along with radiation throughout the chest. Tetralogy of Fallot presents with a rough ejection, systolic murmur heard best at the LSB and the third ICS with radiation to the back.
Q 32.3: A 63-year-old female presents with deep bone pain in both legs. Laboratory evaluation reveals Bence-Jones proteinuria and a monoclonal spike on serum electrophoresis. Which of the following vaccines should this patient receive as part of her management? A Diphtheria B Human Papilloma Virus (HPV) C Pertussis D Pneumococcal E Varicella
The Correct Answer is: D Patients with multiple myeloma are at risk from infections, especially from encapsulated organisms such as Haemophilus influenzae and pneumococcus. Pneumococcal vaccines should be administered, but the patient response is decreased based on their current immunodeficiency. DTaP (A and C), HPV (B), and varicella (E) vaccines are appropriate for this patient based on general immunization guidelines for adults, but not of increased necessity based on her diagnosis.
Q 97.93: A 27-year-old G 0 P 0 woman has been trying to become pregnant for the past 10 months. Menarche occurred at age 13 and her cycles have "always been irregular" except during the 10-year period when she took oral contraceptives. Since stopping contraception two years ago, she and her partner of six years have used condoms consistently until they desired pregnancy. Neither partner has a history of sexually transmitted infection, and both have been mutually monogamous. On physical examination she is 64" tall and weighs 189 pounds. She has increased dark hair on her upper lip and chin and on her lower abdomen and says she has had this hair distribution as long as she can remember. Of the following, what is the most likely cause of her apparent infertility? A Androgen-secreting ovarian neoplasm B Hypothyroidism C Pituitary adenoma D Polycystic ovarian syndrome E Premature ovarian failure
The Correct Answer is: D Patients with polycystic ovarian syndrome typically are obese, infertile, hirsute, and have menstrual disorders, making this the most likely of the diagnoses. An androgen-secreting ovarian neoplasm (A) would cause infertility, but the associated hirsutism is rapidly progressing. Hypothyroidism (B) may cause amenorrhea and weight gain and contribute to infertility, but the overall picture is more suggestive of PCOS. The hyperprolactinemia associated with pituitary adenoma (C) might also produce mild hirsutism, but other symptoms such as galactorrhea would also likely be present. Premature ovarian failure (E) is the spontaneous cessation of menses prior to age 40, and does not apply to this woman.
Q 70.8: What is the treatment for a patient who has recurrent ventricular tachycardia with no reversible cause, and has failed oral medication therapy? A Biventricular pacemaker insertion B Ablation therapy C Heart transplantation D Implantable defibrillator E Single chamber pacemaker
The Correct Answer is: D Patients with recurrent symptoms benefit from the implantation of a defibrillator, which will reduce sudden death. Ablation therapy is usually not indicated. In rare cases of patients who do not have any other underlying disease, cardiac transplantation is an option. Pacemakers are options if the underlying rhythm is in need of pacing. (Bashore et al., Current Medical Diagnosis and Treatment, Chapter 10)
Q 97.81: A 24-year-old female, with a history of type 2 diabetes, presents with the inability to conceive after 14 months of unprotected sexual intercourse with her husband. Vital signs are as follows: Temperature 37˚ C, BP 115/85, P 65, RR 20. Her BMI is 31. Which physical examination finding would you expect to observe with your suspected diagnosis? A Undeveloped breasts B Palpable uterus on abdominal exam C Acromegaly D Coarse dark hair on the face, chest, and back E Egophony on lung exam
The Correct Answer is: D Polycystic ovarian syndrome (PCOS) affects 5-10% of reproductive age women. It is associated with hirsutism (50% of cases) (D), obesity (80% of cases), and virilization (20%), and those affected have an increased risk of diabetes mellitus, cardiovascular disease, and metabolic syndrome. PCOS patients are often infertile. A palpable ovary may be noted on thin individuals with enlarged ovaries, not the uterus (B). Undeveloped breasts are not a diagnostic indicator for PCOS (A) and acromegaly (C) is seen with Marfan's Syndrome. Egophony (E) is when there is consolidation of the lung on exam.
Q 32.14: A 72-year-old female presents with a 4-month history of pain and stiffness in her shoulders and hips. She identifies the pain being worse in the morning and aggravated with getting in and out of the car along with difficulty brushing her hair. She also reports malaise and a 10-pound weight loss over the past few months. Her blood work shows an erythrocyte sedimentation rate (ESR) of 74 mm/h. What is the most likely diagnosis? A Gouty arthritis B Bursitis C Fibromyalgia D Polymyalgia rheumatica E Septic arthritis
The Correct Answer is: D Polymyalgia rheumatica (D) is a clinical diagnosis based on pain and stiffness of the shoulder and pelvic girdle areas, frequently in association with fever, malaise, and weight loss. Because of the stiffness and pain in the shoulders, hips, and lower back, patients have trouble combing their hair, putting on a coat, or rising from a chair. Elevated ESRs are present in most cases being > 50 mm/h. The age group most affected is over 50 years old. Fibromyalgia (C) is not specific for only the shoulders and hips. Bursitis (B) would usually be specific to one location. Gouty arthritis (A) is usually monoarticular and is not commonly seen in the hips and shoulders on presentation. Septic arthritis (E) would be joint specific and more acute.
A 42-year-old woman with a history of migraine cephalgia and Raynaud's phenomenon comes to the emergency department with complaints of severe chest discomfort that occurs at rest every morning (at approximately 10 AM). An EKG performed during an episode of chest discomfort demonstrates transient ST segment elevation, which is relieved with sublingual nitroglycerin. There is no troponin elevation. Cardiac catheterization is performed, and reveals coronary artery spasm, which corresponds with ST segment elevation, and no significant coronary artery stenosis. Which of the following choices is the most likely diagnosis? A Pericarditis B Acute myocardial infarction C Costochondritis D Prinzmetal angina E Myocarditis
The Correct Answer is: D Prinzmetal angina, or variant angina pectoris, is defined as coronary artery spasm associated with ST-segment elevation, and usually occurs at rest and at the same time of the day. Patients with a history of migraine cephalgia and Raynaud's phenomenon demonstrate Prinzmetal angina more frequently than the rest of the patient population. This can occur in patients with normal coronary arteries and with coronary artery stenosis. Choice A, pericarditis, would present with chest discomfort that is worse while supine and improves with sitting up, as well as a pericardial friction rub. Choice B, acute myocardial infarction, would present with troponin elevation, and is unlikely in the setting of a patient with normal coronary arteries on cardiac catheterization. Choices C and E would not be relieved with sublingual nitroglycerin or demonstrate transient ST-segment elevation.
Q 118.7: A 42-year-old woman with a history of migraine cephalgia and Raynaud's phenomenon comes to the emergency department with complaints of severe chest discomfort that occurs at rest every morning (at approximately 10 AM). An EKG performed during an episode of chest discomfort demonstrates transient ST segment elevation, which is relieved with sublingual nitroglycerin. There is no troponin elevation. Cardiac catheterization is performed, and reveals coronary artery spasm, which corresponds with ST segment elevation, and no significant coronary artery stenosis. Which of the following choices is the most likely diagnosis? A Pericarditis B Acute myocardial infarction C Costochondritis D Prinzmetal angina E Myocarditis
The Correct Answer is: D Prinzmetal angina, or variant angina pectoris, is defined as coronary artery spasm associated with ST-segment elevation, and usually occurs at rest and at the same time of the day. Patients with a history of migraine cephalgia and Raynaud's phenomenon demonstrate Prinzmetal angina more frequently than the rest of the patient population. This can occur in patients with normal coronary arteries and with coronary artery stenosis. Choice A, pericarditis, would present with chest discomfort that is worse while supine and improves with sitting up, as well as a pericardial friction rub. Choice B, acute myocardial infarction, would present with troponin elevation, and is unlikely in the setting of a patient with normal coronary arteries on cardiac catheterization. Choices C and E would not be relieved with sublingual nitroglycerin or demonstrate transient ST-segment elevation. (LeBlond et al., 2009, Chapter 8)
Q 106.20: Which of the following laboratory findings will be observed in a patient with noninflammatory nonbacterial prostatitis? A positive bacterial culture from postmassage urine (chronic) B positive bacterial culture with expressed prostatic secretions (chronic) C negative bacterial culture with elevated leukocytes D negative bacterial culture with normal leukocytes
The Correct Answer is: D Prostatitis includes a continuum of prostate characteristics ranging from acute episodes to prostatodynia, a noninflammatory disorder. Patients with acute bacterial prostatitis will have an exquisitely tender prostate gland, and prostatic massage is contraindicated in these patients. Chronic prostatitis patients may have no evidence of an acute infection but will have increased leukocytes. Nonbacterial prostatitis is broken into two subcategories with the noninflammatory classification having neither bacteria nor leukocytes. (Nguyen, 2008, pp. 208-210; Stoller et al., 2009, pp. 830-832) Nguyen HT. Bacterial infections of the genitourinary tract. In: Tanagho EA , McAninch JW, eds. Smith's General Urology. 17th ed. New York, NY: McGraw-Hill; 2008:193-218. Stoller ML , Kane CJ , Meng MV. Urologic disorders. In: McPhee SJ , Papadakis MA, eds. Current Medical Diagnosis and Treatment. 48th ed. New York, NY: McGraw-Hill; 2009:827-847.
A 65-year-old male presents with pain and swelling to his right knee without any history of injury. He has had this type of pain and swelling before, and does recall that he had fluid drained out of the knee several years ago. On examination the patient has a swollen, tender knee with a palpable effusion. There is decreased range of motion to the joint secondary to the effusion. An arthocentesis is performed, and the analysis of the fluid reveals calcium pyrophosphonate crystals. Based on these findings, what is the most likely diagnosis? A Septic arthritis B Acute synovitis C Hemarthoma D Pseudogout E Occult fracture
The Correct Answer is: D Pseudogout is also known as Calcium Pyrophosphate Deposition Disease (CPDD) and most commonly affects patients over the age of 65. The knee joint is most commonly affected with the wrist joint second most common. An examination of synovial fluid aspirated from the affect joints, will reveal calcium pyrophosphate crystal. Ankle joints are rarely affected by pseudogout but are fairly common locations for gout caused by urate crystal deposition. The first metatarsal phalangeal joint is the classic location for gout (sometimes referred to as podagra), but it is not a typical pseudogout location. The glenohumeral joint of the shoulder is affected by pseudogout much more than gout. Distal interphalangeal joints of the hand are rare locations for either gout or pseudogout, but are classic locations to see signs and symptoms of osteoarthritis. When pseudogout does affect the hands it is generally seen in the metacarpophalangeal joints.
Q 32.10: A 65-year-old male presents with pain and swelling to his right knee without any history of injury. He has had this type of pain and swelling before, and does recall that he had fluid drained out of the knee several years ago. On examination the patient has a swollen, tender knee with a palpable effusion. There is decreased range of motion to the joint secondary to the effusion. An arthocentesis is performed, and the analysis of the fluid reveals calcium pyrophosphonate crystals. Based on these findings, what is the most likely diagnosis? A Septic arthritis B Acute synovitis C Hemarthoma D Pseudogout E Occult fracture
The Correct Answer is: D Pseudogout is also known as Calcium Pyrophosphate Deposition Disease (CPDD) and most commonly affects patients over the age of 65. The knee joint is most commonly affected with the wrist joint second most common. An examination of synovial fluid aspirated from the affect joints, will reveal calcium pyrophosphate crystal. Ankle joints are rarely affected by pseudogout but are fairly common locations for gout caused by urate crystal deposition. The first metatarsal phalangeal joint is the classic location for gout (sometimes referred to as podagra), but it is not a typical pseudogout location. The glenohumeral joint of the shoulder is affected by pseudogout much more than gout. Distal interphalangeal joints of the hand are rare locations for either gout or pseudogout, but are classic locations to see signs and symptoms of osteoarthritis. When pseudogout does affect the hands it is generally seen in the metacarpophalangeal joints.
Q 105.14: Which of the following is the bacteria that is most concerning regarding a plantar puncture wound to the sole of the foot through a rubber soled shoe and virtually unique to that specific skin injury? A bartonella henselae B methicillin-resistant Staphylococcus aureus C methicillin-sensitive Staphylococcus aureus D pseudomonas aeruginosa E streptococcal species
The Correct Answer is: D Pseudomonas aeruginosa is a bacteria frequently cultured from the inside of rubber-soled foot wear, and these bacteria are more frequently pushed deep into plantar puncture wounds when made through a rubber-soled shoe. Treatment of plantar puncture wounds through rubber-soled shoes, especially deep puncture wounds, should factor this into antibiotic consideration. (Skinner, 2006, Chapter 11)
Q 32.16: A 5-week-old male infant presents with a 1-week history of vomiting which occurs shortly after feeding. The mother describes the vomiting as forceful and the vomitus is occasionally blood streaked; the infant has not had diarrhea. You note that the infant appears slightly dehydrated and has lost weight since a routine check at 2 weeks. Which of the following is the most likely diagnosis? A peptic ulcer disease B viral gastroenteritis C Hirschsprung disease D pyloric stenosis E intussusception
The Correct Answer is: D Pyloric stenosis usually presents with forceful/projectile vomiting between 2 and 4 weeks of age. There is a 4:1 male predominance; dehydration and failure to thrive may develop. Peptic ulcer disease can occur at any age and commonly affects more men but is more common from 12 to 18 years of age. Intussusception is more common in men but presents with colicky abdominal pain with subsequent development of vomiting and bloody diarrhea.
Q 9.5: A 56-year-old male who works in construction climbing ladders has developed pain to the right foot for several days. You have seen and examined the patient a few days after the patient started complaining of pain to the foot. Your initial x-rays of the foot are negative for fracture. He continues to have pain, and decreased ability to bear weight. Based on this clinical scenario, how many days after the initial examination should another x-ray be ordered to look for a stress fracture? A 1-2 days B 5-7 days C 10-12 days D 14-30 days E 180 days
The Correct Answer is: D Radiographic evidence of stress fractures is not immediately apparent after the initial onset of symptoms. Estimates of anywhere from 14 to 42 days before visible signs of a stress fracture can be detected on X-ray have been put forth with a commonly utilized range of 14 to 30 days. Any answer choice above that included numbers less than 14 days would simply be inaccurate due to the gradual appearance of the classic x-ray findings of a stress fracture. After 180 days from the time of stress fracture onset, the fracture should be nearly healed if managed properly. If a stress fracture is highly suspected in light of negative x-rays, imaging with a bone scan should help make the diagnosis. MRI scans may help confirm the diagnosis, but they are rarely utilized in the work-up of suspected stress fractures.
Q 82.3: You are treating a healthy 50-year-old man with no cardiac risk factors. This patient has no other medical conditions and takes no medications. His fasting lipid panel from last week demonstrates the following: What is the LDL goal for this patient? A less than 100 mg/dL B less than 120 mg/dL C less than 130 mg/dL D less than 160 mg/dL E less than 190 mg/dL
The Correct Answer is: D Recommendations of the National Cholesterol Education Program (NCEP) Adult Treatment Panel III Report states the LDL goal for patients with 0 to 1 risk factor to be less than 160 mg/dL. (Baron, 2009, p. 1101) Baron RB. Lipid disorders. In: McPhee SJ , Papadakis MA, eds. Current Medical Diagnosis and Treatment. 48th ed. New York, NY: McGraw-Hill; 2009.
Q 14.1: A 72-year-old female presents with a several month history of symmetric pain to the hands, wrists, and feet. She also is noticing a change to the appearance of the joints of the hands and feet, with a more "nobby" appearance than before. When the patient wakes up in the morning, the joints are particularly stiff and red at times. Out of all the areas that are painful, it is the hands that are the most painful. Based on this history, what physical exam findings would you expect to find on the hands that are classic findings of this disease? A Radial deviation and subluxation of the DIP joints B Radial deviation and subluxation of the MCP joints C Radial deviation and subluxation at the PIP joints D Ulnar deviation and subluxation at the MCP joints E Ulnar deviation and subluxation at the PIP joints
The Correct Answer is: D Rheumatoid arthritis is a systemic autoimmune disease that is incurable and affects woman more than men by a 3 to one ratio. It can damage any joint, but it most often affects joints of the hands, wrists, knees, feet and ankles. The classic appearance of hands that have been afflicted with progressive rheumatoid arthritis is that they demonstrate ulnar deviation and subluxation at the MCP joints. Involvement of the PIP joints is not uncommon, but DIP joints are generally not as significantly affected. Radial deviation is not a typical characteristic of rheumatoid arthritis of the hands.
Q 119.11: A 73-year-old male presents to clinic with a history of blood in his urine for the past month. He has mild irritation with voiding but denies any other symptoms except an unintentional weight loss of 20 pounds in the past 6 months. His past medical history includes hyperlipidemia, seasonal allergies, and fibromyalgia. He denies alcohol use and has a 22 pack-year history of tobacco use. Which risk factor supports your diagnosis? A Fibromyalgia B Hyperlipidemia C Seasonal allergies D Cigarette smoking E Age
The Correct Answer is: D Risk factors for bladder cancer include cigarette smoking (D) and exposure to industrial dyes and solvents. Age (E) is not a risk factor but the mean age of diagnosis is 73 years old and is more common in men than women (3.1:1). Fibromyalgia (A), hyperlipidemia (B), and seasonal allergies (C) have not shown any correlation to predisposing a person to bladder carcinoma. Papadakis MA, McPhee SJ, "Bladder Cancer." Quick Medical Diagnosis & Treatment: http://www.accessmedicine.com/quickam.aspx
Q 64.9: An otherwise healthy 20-year-old man has been given the accurate diagnosis of migraine. His frequency of attack is about one per month. He has never experienced adequate relief with any over-the-counter analgesics. Which of the following would be appropriate to try next? A verapamil 120 mg once daily B amitriptyline 25 mg once nightly C codeine/acetaminophen 15 mg prn D rizatriptan 10 mg at onset, may repeat once in 2 hours E oxygen at 7 L/min inhaled via nonrebreather mask
The Correct Answer is: D Rizatriptan, like the other 5-HT receptor agonists, is an extremely effective medication for the acute treatment of migraine. This patient has tried over-the-counter analgesics, which can work for mild forms of migraine, and so the use of a migraine-specific abortive agent is appropriate. Narcotic analgesics should be avoided not only because of the possibility of dependence but more importantly because they are not as effective as other analgesics for targeting the neurochemical causes of migraine. With a frequency of about 1 attack per month, a preventive agent such as verapamil or amitriptyline is not needed. Oxygen can be useful as an acute treatment for cluster headache but has not been shown to be useful for migraine.
Q 97.18: You are caring for a 33-year-old G3P2 women. Her PMH is significant for obesity and allergies. She receives routine obstetrical care. When is she due for her glucose challenge testing to rule out gestational diabetes? A Between 12 and 16 weeks gestation B Between 16 and 20 weeks gestation C Between 20 and 24 weeks gestation D Between 24 and 28 weeks gestation E Between 28 and 32 weeks gestation
The Correct Answer is: D Routine surveillance for gestational diabetes of a pregnant women is between 24 and 28 weeks; it is not modified due to obesity or other risk factors for diabetes. Gestational diabetes is a hormone-mediated intolerance. Surveillance is modified in the presence of pre-existing diabetes.
Q 97.37: A patient with no history of treatment for primary syphilis presents with symptoms and signs consistent with secondary syphilis. The most common sign of secondary syphilis is A generalized lymphadenopathy B aseptic meningitis C alopecia D generalized maculopapular rash E superficial painless gummas
The Correct Answer is: D Secondary syphilis generally manifests itself a month or two after appearance of the primary chancre. Patients will complain of headache, fever, sore throat, and malaise and will exhibit generalized lymphadenopathy along with a maculopapular rash that begins at the sides of the trunk and later spreads over the rest of the body. The skin lesions may coalesce in warm moist areas, such as the perineum, and form large, flat-topped, pale papules termed condyloma lata. Skin and mucosal lesions are the most common signs of secondary syphilis. Aseptic meningitis and alopecia may also occur in secondary syphilis. Formation of granulomatous nodules (gummas) is not a feature of secondary disease, but rather is the hallmark of tertiary syphilis.
Q 104.8: A patient with no history of treatment for primary syphilis presents with symptoms and signs consistent with secondary syphilis. The most common sign of secondary syphilis is A generalized lymphadenopathy B aseptic meningitis C alopecia D generalized maculopapular rash E superficial painless gummas
The Correct Answer is: D Secondary syphilis generally manifests itself a month or two after appearance of the primary chancre. Patients will complain of headache, fever, sore throat, and malaise and will exhibit generalized lymphadenopathy along with a maculopapular rash that begins at the sides of the trunk and later spreads over the rest of the body. The skin lesions may coalesce in warm moist areas, such as the perineum, and form large, flat-topped, pale papules termed condyloma lata. Skin and mucosal lesions are the most common signs of secondary syphilis. Aseptic meningitis and alopecia may also occur in secondary syphilis. Formation of granulomatous nodules (gummas) is not a feature of secondary disease, but rather is the hallmark of tertiary syphilis. (Philip and Jacobs, 2009, pp. 1301-1302) Philip SS , Jacobs RA. Spirochetal infections. In: McPhee SJ , Papadakis MA, eds. Current Medical Diagnosis and Treatment. 48th ed. New York, NY: McGraw-Hill, 2009.
Q 32.5: A 76-year-old male presents to the hospital with a hot, swollen, and painful right knee that appears to have an effusion. After your examination, you decide to perform an arthrocentesis. The fluid aspirate is turbid, cloudy, and most definitely looks like it is infectious material. Based on this clinical scenario, what would be most likely bacterial organism present in an elderly septic arthritis? A Hemophilus influenzae B Neisseria gonorrhoeae C Pseudomonas aeruginosa D Staphylococcus aureus E Streptococcus pneumonia
The Correct Answer is: D Septic arthritis is an infection within a joint space. The infection can be caused by multiple pathogens including bacteria, viruses, fungi and mycobacteria, but in older adults the most common organism implicated is Staphylococcus aureus. These bacteria can get into the joint space through multiple mechanisms including direct inoculation after a penetrating injury, hematogenous spread from an infection elsewhere in the body, or from extension from a contiguous bone infection. All of the bacteria mentioned as possible answers are possible causes of septic arthritis, but there prevalence varies within different patient populations. Hemophilus influenza and Streptococcus pneumonia are commonly seen in children less than five years of age that develop septic arthritis. Neisseria gonorrhoeae is most common in sexually active adolescents and young adults. Pseudomonas aeruginosa is a common cause of septic arthritis in IV drug abusers.
Q 91.10: A 42-year-old man who is HIV positive develops fever of 38.8°C, mild nonproductive cough, and shortness of breath. He takes no medications other than a multivitamin tablet, does not smoke cigarettes, or use alcohol or illicit drugs. Of the following findings on diagnostic studies, which is most consistent with a diagnosis of Pneumocystic jiroveci pneumonia in this man? A apical infiltrates on chest radiography B bronchiolar consolidation on computed tomographic (CT) scan C CD4 count of 300 cells/mL D PO2 of 54 mm Hg E serum lactate dehydrogenase (LDH) level of 54 units/L
The Correct Answer is: D Severe hypoxemia is a common finding in Pneumocystis pneumonia even when symptoms are not severe. The characteristic chest radiograph findings are diffuse or perihilar infiltrates. Apical infiltrates are more likely to be seen in patients who have been receiving aerosolized pentamidine prophylaxis. High-resolution chest CT scanning would most likely demonstrate interstitial lung disease. This pneumonia is rare unless the CD4 count is less than 250. An elevated LDH is found in about 95% of patients, but this is not specific. (Katz and Zolopa, 2009, pp. 1183, 1361) Katz MH , Zolopa AR. HIV infection. In: McPhee SJ , Papadakis MA, eds. Current Medical Diagnosis and Treatment. 48th ed. New York, NY: McGraw-Hill; 2009.
Q 79.5: Which of the following is a tumor of bronchial origin that is known to grow rapidly and have diffuse metastases at the time of diagnosis? A Adenocarcinoma B Carcinoid C Large cell D Small cell E Squamous cell
The Correct Answer is: D Small cell lung cancer is a fast-growing, rapidly spreading form of lung cancer. Although the cells are small, they grow very quickly, metastasize to many parts of the body, and form large tumors. At the time of diagnosis, tumor spread is presumed. The growth and spread is considered much faster than that of non-small cell lung cancers. Staging is also different, utilizing a two-stage system based on the extent of spread. (McPhee SJ, Papadakis MA. Current Medical Diagnosis & Treatment 2011, Chapter 39, Cancer)
Q 97.3: A 56-year-old male is diagnosed with a duodenal ulcer through endoscopy. He is placed on an appropriate regimen of medications to heal the ulcer. Which of the following lifestyle choices is known to retard the rate of ulcer healing and should be discouraged? A Moderate alcohol intake B Ingestion of spicy foods C Eating at regular intervals D Smoking cigarettes E Ingestion of fruit juices
The Correct Answer is: D Smoking cigarettes does retard the rate of ulcer healing and increases the frequency of recurrence. Patients should be encouraged to eat balanced meals at regular intervals. There is no justification for bland or restrictive diets. Moderate alcohol intake is not harmful. (McPhee SJ, Papadakis MA. Current Medical Diagnosis & Treatment, 2010, p. 550)
Q 114.2: A 25-year-old asymptomatic, non-smoking male presents with a 9mm solitary pulmonary nodule, with dense central calcification on routine chest x-ray. The nodule's appearance is smooth and calcified with a well-defined edge. This same lesion was present on a work physical x-ray two years ago. Which of the following is the most appropriate management step for this patient? A Positron emission tomograph to rule out malignancy B Resection of the pulmonary nodule C Thoracic magnetic resonance imaging (MRI) D Watchful waiting with serial imaging E Pulmonary function tests
The Correct Answer is: D Solitary pulmonary nodules are categorized based upon the probability of malignancy. This patient is under the age of 30, with previous documentation of the lesion and without evidence of growth or change, which decreases the probability of malignancy. In this case, watchful waiting is indicated. Risk factors, such as advanced age, smoking, lesion growth or appearance change, and history of prior malignancy increase malignancy probability, and indicate a need for more defined evaluation. (McPhee SJ, Papadakis MA. Current Medical Diagnosis & Treatment 2011, Chapter 9, Pulmonary Disorders)
Q 107.2: A 32-year-old male presents to your office with the complaint of low back pain for 7 months. The patient states he was initially injured on the job while trying to lift a 50-pound barrel off a truck. He denies any paresthesias or bowel/bladder problems associated with the low back pain. The patient states that he had been given NSAIDs and a muscle relaxer, followed by physical therapy treatments. X-rays that were taken 5 months ago were reported as normal. He was placed on light duty at that time. The patient has seen many practitioners who have "not helped him." Another person who works with this patient was at the clinic and stated the patient has had problems with one of his other coworkers. You consider trying the patient on an antidepressant first and then possibly sending him to a pain clinic if there is no success. What is the most likely diagnosis? A conversion disorder B hypochondriasis C drug addiction D somatoform pain disorder E schizophreniform
The Correct Answer is: D Somatoform pain disorder is a focus on pain for greater than 6 months. The subjective findings outweigh the objective findings. Pain in the neck, pelvic, or low back areas are frequent sites, as well as headaches. The disorder may be precipitated by an injury. The patient will have a history of seeing multiple providers and possibly many medical and surgical treatments. The patient is unresponsive to treatment. Stressors can aggravate or precipitate the pain. There may be an expectation of secondary gains. Age of onset is around 30s and 40s. Treatment consists of placing the patient on an antidepressant and sending the patient to a pain clinic. (Ford, 2008, pp. 407-408; Sadock and Sadock, 2008, pp. 284-285) Ford, CV. Somatoform disorders. In: Ebert MH , Loosen PT , Nurcombe B , Leckman JF, eds. Current Diagnosis and Treatment in Psychiatry. New York: McGraw-Hill; 2008. Sadock BJ , Sadock VA. Concise Textbook of Clinical Psychiatry, 3rd ed. Philadelphia, PA: Lippincott, Williams & Wilkins; 2008.
Q 97.25: A 27-year-old female is 8 weeks postpartum with her first child and has been exclusively nursing since discharge at the hospital. She has a 5-day history of engorgement in her right breast, which is red, tender, and feels warm to the touch. She states she is feverish but has not taken her temperature. She reports no known drug allergies. On physical examination you see the breast as shown below. Which of the following is the most appropriate therapy in the management of this patient? A Cephalexin 500 mg every 6 hours for 3 days; discontinue breastfeeding B Cephalexin 500 mg every 6 hours for 3 days; continue breastfeeding C Discontinue breastfeeding and see if symptoms resolve D Dicloxacillin 500 mg orally every 6 hours for 10 days; continue breastfeeding E Dicloxacillin 500 mg orally every 6 hours for 10 days; discontinue breastfeeding
The Correct Answer is: D Staphylococcus aureus is usually the causing agent in puerperal mastitis. Treatment includes antibiotics that are effective against penicillin-resistant staphylococci and nursing of the infant with the affected breast is safe. Alternatively, a cephalosporin can be given orally for 10-14 days. Cephalexin 500 mg every 6 hours (A and B) can be used but should be continued for longer than 3 days. Failure of the patient to respond to antibiotics after three days becomes concerning for an abscess or resistant organism. Further evaluation would be needed including surgical consultation, ultrasound, and hospital admission for IV antibiotics. Dicloxacillin is commonly used in uncomplicated cases without any known allergies. To preserve milk supply, it is encouraged to continue breastfeeding with mastitis; do not recommend discontinuing breastfeeding (A, C, and E).
Q 14.7: A 42-year-old woman presents complaining of shortness of breath, three days of fever as high as 103F, and a cough productive of green sputum. On physical examination, you hear crackles in her lungs. A chest radiograph reveals a consolidation in the left lower lobe. What tactile fremitus findings do you expect to observe in the left lower lobe region? A Absent B Decreased C Hyperresonant D Increased E Normal
The Correct Answer is: D Tactile fremitus is increased (D) by processes that cause consolidation within the lung parenchyma and decreased (B) by processes that insulate the lung parenchyma from transmitting vibration (e.g., pneumothorax, hemothorax) or airway obstruction.
Q 39.1: A mother presents with a 10-year-old female with a complaint of thinning hair. On physical exam there are no areas of alopecia, broken hairs, scaling, or erythema. A hair pull produces 12 to 14 hairs. Examination under microscopy indicates that the follicles are intact and normal in appearance. What is the most likely diagnosis? A Tinea capitus B Anagen dysplasia C Alopecia areata D Telogen effluvium
The Correct Answer is: D Telogen effluvium is a condition of hair thinning that can occur at any age. It is a reaction to a physical or mental stressor. A normal scalp has 10 percent to 15 percent of hairs in the telogen phase at any time, resulting in a normal shedding of 50 to 100 hairs per day. Increased shedding of hairs occurs in telogen effluvium. This results in more hairs being shifted into the telogen phase more quickly. A hair pull with microscopy is the diagnostic test that will indicate an increase in hairs in the telogen phase.
Q 41.1: The mother of a four-month-old brings her son in for evaluation of cyanosis. The mother noted the cyanosis in the last two days, and it is most evident when he is feeding or crying. He was previously healthy with no medical problems. On physical examination, a grade III/VI systolic ejection murmur is present at the left sternal border in the third intercostal space, and radiates to the back. What is the most likely diagnosis? A Atrial septal defect B Patent ductus arteriosus C Mitral valve prolapse D Tetralogy of fallot E Transposition of the great arteries
The Correct Answer is: D Tetralogy of fallot is the correct answer. The epsiodes of cyanosis described by the mother represent "tet spells." Atrial septal defect is incorrect, as it is not associated with cyanosis; while the murmur may be located at the left sternal border, it is most often heard in the second intercostal space and is associated with a widely split S2. Patent ductus arteriosus (PDA) is incorrect, as this does not lead to cyanosis, and the description of this murmur is classically described as a rough machinery systolic murmur. PDA murmur may radiate to the anterior lung fields, but does not radiate well to the posterior lung fields. Mitral valve prolapse is incorrect, as it is not associated with cyanosis and the murmur is described as a systolic click. Transposition of the great arteries is incorrect, as there is typically no significant murmur present and they present as neonates with profound cyanosis.
Q 9.9: A 7-year-old girl is brought in by her mom for evaluation of a rash. She has had a fever for a few days and woke up this morning looking like she had been slapped on both cheeks. Other than supportive care, which instruction below represents the best patient education for this patient? A She should remain out of school because she is contagious until the rash resolves. B She may return to school but stay out of physical education class to avoid splenic injury. C It spreads by the fecal-oral route, so she should wash her hands after using the bathroom. D She may resume normal activities as her energy level improves.
The Correct Answer is: D The "slapped cheek" appearance to the rash is consistent with a Parvovirus B19 etiology for erythema infectiosum. It is a droplet infection that is no longer contagious once the rash breaks out. It generally has a benign course and patients recover fully with supportive care. Splenic involvement is not typically a part of the course, so she may resume activities as she feels able.
Q 89.8: You evaluate a 72-year-old male for a pre-operative physical examination prior to total left-knee replacement. He has a 20-year history of hypertension and is currently taking lisinopril 10 mg QD and HCTZ 25 mg QD. He denies any complaints. You obtain the EKG below. What finding on this patient's EKG is a potential complication of sustained hypertension? Source: (Knoop et al., 2010, Chapter 23) A acute STEMI B atrial flutter C atrial fibrillation D left ventricular hypertrophy E congestive heart failure
The Correct Answer is: D The EKG represents left ventricular hypertrophy (LVH) with strain pattern. A finding of LVH on an EKG in a patient with hypertension suggests an increased cardiovascular risk. The EKG does not show ST elevations consistent with a STEMI and the patient is asymptomatic. Congestive heart failure (CHF) cannot be diagnosed by this EKG alone, but patients with CHF may have LVH. The patient has a sinus rhythm with distinguishable P waves, which are not consistent with atrial fibrillation or flutter. (McPhee and Papadakis, 2011, Chapter 11)
Q 105.15: A 56-year-old woman is being seen for regular assessment and monitoring of her type 2 diabetes mellitus. She has been following a strict diet and exercise plan for 2 years with the addition of metformin 6 months ago for an increased HgA 1c level. Her HgA 1c at today's visit is 7.1. What is the appropriate management for this patient? A add exenatide to her current therapy B change her oral therapy to rosiglitazone C add insulin to her current therapy D maintain her current therapy and recheck in 6 months
The Correct Answer is: D The HgA 1c goal for this patient is less than 6.5, with action at a level of greater than 8.0. The appropriate action at this time is to continue her current therapy and reassess in 6 months. (Barnett et al, pp. 683-694) Barnett PS , Braunstein GD. Diabetes mellitus. In: Andreoli TA , Carpenter CC , Griggs RC , et al. , Cecil Essentials of Medicine. 7th ed. Philadelphia, PA: WB Saunders; 2007.
Q 97.10: A 53-year-old woman presents to clinic complaining of amenorrhea, irritability, and hot flashes for 6 months. She would like to try hormone replacement therapy. You are counseling her about possible risks versus benefits. Which of the following is a benefit of combined estrogen progestin therapy? A A decreased risk of breast cancer B A decreased risk of myocardial infarction C A decreased risk of stroke D A decrease in somatic symptoms E A decreased rate of cognitive decline
The Correct Answer is: D The WHI study showed increase chance of cardiovascular risks and breast cancer, and showed no improvement in prevention of cognitive decline.
Q 47.9: A 3 year-old boy is brought to the emergency department due to acute onset of cough and wheezing. Physical exam reveals focal wheezing in the right lower lobe. Which of the following is the most likely diagnosis? A Asthma B Bronchiectasis C Epiglottitis D Foreign body aspiration E Vocal cord dysfunction
The Correct Answer is: D The acute onset of symptoms and their localization to the right lower lobe are consistent with foreign body aspiration (D). Asthma (A) presents with diffuse wheezing. Bronchiectasis is less common in this age group and typically presents with a copious amount of sputum production. Epiglottis (C) and vocal cord dysfunction (E) present with upper airway symptoms.
Q 32.7: A 23-year-old woman presents to the outpatient clinic complaining of chronic congestion and obstruction of her right nasal passage. Upon physical exam you visualize the following. Which of the following is the most likely diagnosis? A Chronic sinusitis B Inverted papilloma C Juvenile angiofibroma D Nasal polyp E Squamous cell carcinoma
The Correct Answer is: D The appearance of this pale, edematous, mucosally covered mass is consistent with a nasal polyp (D). Juvenile angiofibromas (C) typically present as epistaxis in adolescent males. The symptoms and appearance of this polyp are inconsistent with the presentation of inverted papilloma (B), chronic sinusitis (A), and squamous cell carcinoma (E).
Q 82.2: A 30-year-old female presents with a five day history of a sore throat. She denies cough or nasal congestion. She also denies vomiting or diarrhea. On physical exam, her temperature is 101˚F, the pharynx is red with tonsillar exudates, and she has tender anterior cervical lymphadenopathy. What is your next step? A Perform culture and sensitivity B Perform rapid strep C Treat symptomatically with antipyretics D Begin oral penicillin E Begin oral ciprofloxacin
The Correct Answer is: D The centor criteria include fever, tonsillar exudates, tender anterior lymphadenopathy and lack of cough. These signs and symptoms highly suggest group A beta hemolytic strep. Treatment with penicillin would be the most appropriate step, especially if cost is a concern to the patient. Penicillin v potassium, 250 mg three times per day, or 500 mg twice daily for 10 days, is highly effective. Some studies show a five-day regime to be as effective. (McPhee et al., 2011, Chapter 8)
A 23-year-old patient presents to the Emergency Department complaining about a bloody left eye that occurred after being struck with a tennis ball. The patient denies any decreased vision or photophobia. A photo of the left eye reveals the following (see below). Which of the following is the most likely diagnosis? A Episcleritis B Hyphema C Pinguecula D Subconjunctival hemorrhage E Uveitis
The Correct Answer is: D The classic appearance of a bright red, flat collection of blood is consistent with a subconjunctival hemorrhage (D). A hyphema (B) is an internal hemorrhage within the anterior chamber of the eye. The patient lacks conjunctival injection seen in episcleritis (A) and unveitis (E). A pinguecula (C) is a small nodular conjunctival growth.
Q 24.5: A 23-year-old patient presents to the Emergency Department complaining about a bloody left eye that occurred after being struck with a tennis ball. The patient denies any decreased vision or photophobia. A photo of the left eye reveals the following (see below). Which of the following is the most likely diagnosis? A Episcleritis B Hyphema C Pinguecula D Subconjunctival hemorrhage E Uveitis
The Correct Answer is: D The classic appearance of a bright red, flat collection of blood is consistent with a subconjunctival hemorrhage (D). A hyphema (B) is an internal hemorrhage within the anterior chamber of the eye. The patient lacks conjunctival injection seen in episcleritis (A) and unveitis (E). A pinguecula (C) is a small nodular conjunctival growth.
Q 18.6: A 5-year-old boy presents to urgent care complaining of painful lesions in his mouth that have made eating difficult the past 2 days. The mother confirms he has been unable to eat for 48 hours, but has been able to sip water. On physical exam he has a temperature of 102.6 F; numerous small vesicles and ulcers on the buccal mucosa and tongue, inflamed gingiva; and tender anterior cervical adenopathy. Which of the following is the most likely diagnosis? A Aphthous ulcers B Hand, foot, and mouth disease C Herpangina D Herpes simplex gingivostomatitis E Viral pharyngitis
The Correct Answer is: D The classic presentation of initial herpes simplex infection (D) includes multiple small, painful vesicles or ulcers on the mucousa with gingival involvement, fever, and adenopathy. (A), (B), and (C) all present with ulcers, but typically involve an isolated area (A), or the tonsils and posterior pharynx (B and C).
Q 109.11: A 25-year-old man presents for evaluation of diarrhea. He is generally healthy and reports he finished a 4-day hike about 2 weeks ago. He does mention that he ran out of water on day 3 and did not have a filter with him. Today he reports that he does not feel too badly but he has had 24 hours of abdominal bloating, increased flatulence, and loose stools. He denies melena or hematochezia. His physical examination is unremarkable but stool ova and parasite examination reveal ova and trophozoites. The most appropriate antimicrobial agent for this patient is A no medicine needed; this is self-limiting and will resolve in 24 hours B ciprofloxacin C amphotericin D metronidazole
The Correct Answer is: D The clinical picture is consistent with infection with Giardia lamblia, a parasite that can be picked up from contaminated water and infects the small intestine. It can be difficult to diagnose because it may be clinically asymptomatic. Nevertheless, an infected person passes the cysts in the stool and they can survive for weeks in cold water. Infection can also be transmitted by direct fecal-oral route, especially among small children and their caregivers. Clinical symptoms vary but often include bloating, loose stools or diarrhea, belching, and possibly weight loss. The course can be episodic. Diagnosis is made with stool specimen examined for ova and parasites or by stool antigen immunoassay. First-line therapy is metronidazole 250 mg po tid for 5 days. Tinidazole is an effective alternative. (Weller, 2008, pp. 1311-1313) Weller PF. Protozoal intestinal infections and trichomoniasis. In: Fauci AS , Braunwald E , Kasper DL, et al., eds. Harrison's Principles of Internal Medicine. 17th ed. New York, NY: McGraw-Hill; 2008.
An 18 month old female is brought to the pediatricians office with a history of cough, fever of 102, and decreased fluid intake. Her immunizations are not up to date as the family just moved to the United States from out of the country. On physical exam she is drooling and sitting up in a "tripod position" with mild stridor. What is the most appropriate treatment indicated for this condition? A Humidified air B Albuterol nebulizer C Budesonide nebulizer D Recemic epinephrine nebulizer E Ipratropium nebulizer
The Correct Answer is: D The clinical presentation suggests epiglotitis. This is an emergent airway condition. The anesthesiologist , or the pediatric otolaryngologist must be called to stand by to intubate or insert a tracheostomy if the patients airway closes. Racemic epinephrine via nebulizer relieves much of the edema to the upper airway in a patient with epiglotitis. It is a stabilizing measure until definitive care can be arranged. Oxygen and antibiotics should administered emergently also. No x-rays are indicated when the presentation is classic. Albuterol is a beta-agonist used for treatment of asthma. Budesonide,a steroid and ipratropium, an anticholinergic agent are most often used in combination with albuterol for treatment emphysema and asthma.
Q 39.21: An 18 month old female is brought to the pediatricians office with a history of cough, fever of 102, and decreased fluid intake. Her immunizations are not up to date as the family just moved to the United States from out of the country. On physical exam she is drooling and sitting up in a "tripod position" with mild stridor. What is the most appropriate treatment indicated for this condition? A Humidified air B Albuterol nebulizer C Budesonide nebulizer D Recemic epinephrine nebulizer E Ipratropium nebulizer
The Correct Answer is: D The clinical presentation suggests epiglotitis. This is an emergent airway condition. The anesthesiologist , or the pediatric otolaryngologist must be called to stand by to intubate or insert a tracheostomy if the patients airway closes. Racemic epinephrine via nebulizer relieves much of the edema to the upper airway in a patient with epiglotitis. It is a stabilizing measure until definitive care can be arranged. Oxygen and antibiotics should administered emergently also. No x-rays are indicated when the presentation is classic. Albuterol is a beta-agonist used for treatment of asthma. Budesonide,a steroid and ipratropium, an anticholinergic agent are most often used in combination with albuterol for treatment emphysema and asthma.
A 28-year-old male presents with hypotension, marked tachypnea, and severe dyspnea following a fistfight. A physical exam reveals ecchymoses over the lateral left rib cage, hyperresonance on the left, and tracheal shift to the right. Which of the following is the most appropriate next step for management of this patient? A Anesthesia consultation B Chest CT scan C Chest x-ray D Needle thoracostomy to the second intercostal space, midclavicular line E Tube thoracostomy to the fifth intercostal space, midclavicular line
The Correct Answer is: D The diagnosis for this patient is a traumatic tension pneumothorax, a true medical emergency. Diagnosis can be made based on physical exam findings. Although definitive treatment with a properly placed tube thoracostomy is preferred, choice E has the position incorrect. Needle decompression in the second intercostal space, midclavicular line is the correct choice, with this remaining in place until a chest tube is properly in place.
Q 111.3: A 28-year-old male presents with hypotension, marked tachypnea, and severe dyspnea following a fistfight. A physical exam reveals ecchymoses over the lateral left rib cage, hyperresonance on the left, and tracheal shift to the right. Which of the following is the most appropriate next step for management of this patient? A Anesthesia consultation B Chest CT scan C Chest x-ray D Needle thoracostomy to the second intercostal space, midclavicular line E Tube thoracostomy to the fifth intercostal space, midclavicular line
The Correct Answer is: D The diagnosis for this patient is a traumatic tension pneumothorax, a true medical emergency. Diagnosis can be made based on physical exam findings. Although definitive treatment with a properly placed tube thoracostomy is preferred, choice E has the position incorrect. Needle decompression in the second intercostal space, midclavicular line is the correct choice, with this remaining in place until a chest tube is properly in place. (Fauci et al., Harrison's Principles of Internal Medicine, 17e, Chapter 257, Disorders of the Pleura and Mediastinum)
Q 27.1: Your patient is a 4-year-old boy whose father brings him to your primary care office concerned about behavioral problems. He cannot seem to sit still in school. He seems to have a great deal of difficulty focusing for any length of time. Which of the following is required in order to make the diagnosis of attention deficit/hyperactivity disorder (ADHD) according to the DSM-IV-TR? A He exhibit more symptoms of inattention than hyperactivity B He must be at least 10-years-old C He behaves aggressively towards others on at least some occasions D Some impairment from the symptoms is present in two or more settings E Symptoms of inattention be present on a daily basis for 3 months
The Correct Answer is: D The diagnosis of ADHD requires symptoms of both hyperactivity/impulsivity and of inattention (D). It does not require more or more prominent symptoms (A) of one or the other. Aggression (C) is seldom involved, although the impulsivity may manifest as lack of concern for others. The symptoms must be present before the age of seven (B) and must have been present for at least 6 months (E).
Q 117.13: In a 73-year-old male, laboratory and radiologic findings that demonstrate marrow plasmacytosis with >30% plasma cells, a monoclonal globulin spike present, and lytic bone lesions is most consistent with which of the following? A Acute myelogenous leukemia B Chronic myelogenous leukemia C Hodgkin lymphoma D Multiple myeloma E Non-Hodgkin lymphoma
The Correct Answer is: D The diagnosis of plasma cell myeloma is confirmed when at least one major and one minor criterion, or three minor criteria, are documented in a patient. This patient has one major and two minor criteria. AML and CML are not associated with lytic lesions. Hodgkin and Non-Hodgkin lymphoma typically do not have lytic lesions or monoclonal globulin spikes. (Lichtman et al., Williams Hematology 8e, Chapter 109, Myeloma)
Q 39.12: A 12-month-old female presents with bilateral lichenification, scaling, and excoriations in the antecubital fossae and popliteal fossae. Which of the following should treatment include? A clotrimazole/betamethasone diproprionate cream bid to affected areas B topical ketoconazole cream bid-tid to affected areas and po hydroxyzine C petroleum jelly bid-tid to affected areas and po augmentin D triamcinolone 0.1% ointment bid to affected areas and po hydroxyzine
The Correct Answer is: D The first line treatment for atopic dermatitis is a topical steroid, such as triamcinolone ointment. Oral hydroxyzine is an antihistamine that is used to control pruritis in atopic dermatitis. Clotrimazole/betamethasone diproprionate cream is a combination antifungal and topical steroid. There are no clinical indications for this medication. An antifungal such as ketoconazole is not indicated. The oral hydroxyzine is an antihistamine that is used to help control pruritis. Petroleum jelly is an excellent emollient; however, augmentin is not indicated unless there is a secondary infection. No infection is present in this patient.
Q 20.3: You are evaluating a 32-year-old male who has possibly injured his Achilles tendon while planting his foot into the soft ground while playing touch football. When evaluating a patient with this suspected injury, you have the patient plantar flex his ankle as part of the exam. Based on your knowledge of anatomy, what two plantar flexing muscles attach into the Achilles tendon? A Anterior tibialis and soleus B Flexor hallucis longus and flexor digitorum C Flexor hallucis longus and gastrocnemius D Gastrocnemius and soleus E Peroneus longus and peroneus brevis
The Correct Answer is: D The gastrocnemius and soleus muscles form a common tendon at their distal insertions known as the Achilles tendon. This the strongest and thickest tendon in the body and it attaches at its distal end onto the calcaneus bone of the foot. Contraction of the gastrocnemius and soleus muscles are responsible for plantar flexion of the foot and ankle. The anterior tibialis muscle is one of the important dorsiflexors of the foot and ankle. Flexor hallucis longus and flexor digitorum are both involved in plantar flexion, but are not as strong as the gastrocnemius and soleus muscles and do not attach to the Achilles tendon. The flexor hallucis longus is primarily responsible for plantar flexion of the great toe and flexor digitorum produces plantar flexion of the remaining toes. The peroneus longus and peroneus brevis muscles do contribute to plantar flexion, but are not connected to the Achilles tendon. They are also the primary evertor muscles of the foot and ankle.
You are evaluating a 32-year-old male who has possibly injured his Achilles tendon while planting his foot into the soft ground while playing touch football. When evaluating a patient with this suspected injury, you have the patient plantar flex his ankle as part of the exam. Based on your knowledge of anatomy, what two plantar flexing muscles attach into the Achilles tendon? A Anterior tibialis and soleus B Flexor hallucis longus and flexor digitorum C Flexor hallucis longus and gastrocnemius D Gastrocnemius and soleus E Peroneus longus and peroneus brevis
The Correct Answer is: D The gastrocnemius and soleus muscles form a common tendon at their distal insertions known as the Achilles tendon. This the strongest and thickest tendon in the body and it attaches at its distal end onto the calcaneus bone of the foot. Contraction of the gastrocnemius and soleus muscles are responsible for plantar flexion of the foot and ankle. The anterior tibialis muscle is one of the important dorsiflexors of the foot and ankle. Flexor hallucis longus and flexor digitorum are both involved in plantar flexion, but are not as strong as the gastrocnemius and soleus muscles and do not attach to the Achilles tendon. The flexor hallucis longus is primarily responsible for plantar flexion of the great toe and flexor digitorum produces plantar flexion of the remaining toes. The peroneus longus and peroneus brevis muscles do contribute to plantar flexion, but are not connected to the Achilles tendon. They are also the primary evertor muscles of the foot and ankle.
Q 46.2: A 12-year-old female presents with a cough, fever, and severe right ear pain for two days. Her mother states that she was crying last evening with severe pain, but then she seemed to rapidly improve. There was some bloody drainage in the ear canal. The child is currently swimming for her middle school swim team. Other children on the team have been ill also. Her temperature is 101˚F. During physical exam, you notice the following finding. What is the diagnosis? A Otitis externa B Suppurative otitis media C Bullous myringitis D Otitis media with perforation E Otorrhea
The Correct Answer is: D The image shows an erythemic tympanic membrane with a perforation. Otitis externa won't have tympanic membrane involvement, and will have tragal tenderness upon palpation. Bullous myringitis is often caused by mycoplasma, and shows multiple bubbles or blisters on the TM.
Q 40.5: A 5-year-old male child presents to the office for his kindergarten physical examination. Assuming that the patient's immunizations have been up to date, which of the following are the immunizations that the patient should receive at the end of today's visit? A hepatitis B, inactivated poliovirus (IPV), diphtheria, tetanus, acellular pertussis (DTaP), measles, mumps, rubella (MMR), varicella B IPV, DTaP, MMR, pneumococcal (PCV) C IPV, DTaP, MMR, Haemophilus influenzae type B (Hib) D DTaP, IPV, MMR, varicella
The Correct Answer is: D The immunization schedule is developed biannually by the Centers for Disease Control and Prevention. Assuming that the child has had the appropriate immunizations at the regularly scheduled examinations, the recommended immunizations at the 4- to 6-year-old range are the DTaP (diphtheria, tetanus, acellular pertussis), IPV (inactivated polio), and the MMR (measles, mumps, and rubella). The hepatitis series should have been completed by the age of 6 months and the Haemophilus influenzae type B (Hib) should be completed by the age of 12 to 15 months. Varicella is given from 12 to 18 months and again from 4 to 6 years; the PCV (pneumococcal) should be finished by 12 to 15 months.
Q 31.8: A 25 year-old HIV positive white male presents with dyspnea and a nonproductive cough. The patient's temperature is 103.5 degrees F, respirations 40, and pulse 140. Physical exam reveals a thin, dyspneic male, height 5'10" and weight 150 pounds, in moderate respiratory distress with minimal bibasilar crackles and peripheral cyanosis. CXR reveals a diffuse interstitial infiltrate and his LDH is elevated. Your most likely diagnosis is: A Streptococcal pneumoniae B Klebsiella pneumoniae C Mycoplasma pneumoniae D Pneumocystis jiroveci E Staphlococal aureus
The Correct Answer is: D The insidious onset of atypical pneumonia symptoms in a patient with HIV is consistent with pneumocystis jiroveci (D). Streptococcal pneumoniae (A), Klebsiella pneumonia (B), and Staphlococal aureus (E) present as "typical" pneumonia with the acute onset of fever and productive cough.
Q 28.3: A 22-year-old male presents to the Emergency Department with severe pain in his right arm and is in distress without relief after being given narcotic pain medication. He had a right short arm cast placed for a fractured distal radius 12 hours ago. His pain began 4 hours ago, without any associated trauma. On removal of the cast his arm is noticeably swollen, firm, and tender to any palpation or movement. Based on high clinical suspicion of the suspected diagnosis what is the most appropriate treatment option? A Apply a short arm splint B Emergent MRI for evaluation C IV antibiotics D Emergent surgical consult for a fasciotomy E IV fluids and narcotics with observation
The Correct Answer is: D The key to early detection of compartment syndrome is a high index of clinical suspicion. Initiate medical management and an emergent surgical consult while the surgical fasciotomy is being arranged. This includes supplemental oxygen and supporting the blood pressure in the hypotensive patient. Place the affected limb at the level of the heart; elevation higher than the heart reduces the arteriovenous pressure gradient. Associated open or closed fractures are common. The patient with compartment syndrome will initially complain of severe pain, often difficult to control even with narcotic pain medications. The pain typically starts within a few hours of the injury, but may develop up to 48 hours after the event. Clinically, the compartment is commonly swollen, firm, and tender to squeezing by the examiner. Removing constrictive casts or dressings, you would not want to reapply a cast (A). An MRI (B) would take too long when time is critical. IV antibiotics would not correct compartment syndrome (C). IV fluids and pain medication (E) can be given while waiting for surgery but are not used to treat compartment syndrome.
Q 22.2: The two major risk factors for human immunodeficiency virus (HIV) infection in American women are intravenous drug use and which of the following? A history of blood transfusion B needle stick injuries C pelvic inflammatory disease D sexual contact with an infected male E use of oral contraceptives
The Correct Answer is: D The major risk factors for HIV infection in American women are intravenous drug use (33%) and heterosexual contact with an infected partner (65%). Thanks to universal blood donor screening using the HIV ELISA, antigen, and viral load testing, the risk for any person contracting HIV from a screened unit of blood is only 1:1,000,000. The risk for any person following a needle-stick injury is about 1:300 with deeper sticks, hollow bore needles, visible blood on the needle, and advanced stage of disease in the source increasing the risk. HIV infection puts a woman at increased risk for gynecologic complications such as pelvic inflammatory disease. Unlike the use of latex condoms, the use of oral contraceptives does not protect against HIV transmission, but is not, per se, a risk factor for HIV infection.
Q 97.103: The two major risk factors for human immunodeficiency virus (HIV) infection in American women are intravenous drug use and which of the following? A history of blood transfusion B needle stick injuries C pelvic inflammatory disease D sexual contact with an infected male E use of oral contraceptives
The Correct Answer is: D The major risk factors for HIV infection in American women are intravenous drug use (33%) and heterosexual contact with an infected partner (65%). Thanks to universal blood donor screening using the HIV ELISA, antigen, and viral load testing, the risk for any person contracting HIV from a screened unit of blood is only 1:1,000,000. The risk for any person following a needle-stick injury is about 1:300 with deeper sticks, hollow bore needles, visible blood on the needle, and advanced stage of disease in the source increasing the risk. HIV infection puts a woman at increased risk for gynecologic complications such as pelvic inflammatory disease. Unlike the use of latex condoms, the use of oral contraceptives does not protect against HIV transmission, but is not, per se, a risk factor for HIV infection.
Q 106.5: Of the following diseases, which would you consider screening for first in a 55-year-old diabetic with no other comorbidities? A abdominal aortic aneurysm with a sonogram B carotid arterial disease with a sonogram C coronary artery disease with an angiogram D peripheral arterial disease with an ABI
The Correct Answer is: D The majority of patients with peripheral artery disease (PAD) will be asymptomatic. In fact, clinicians will miss 90% of patients with PAD if they wait for classic symptoms to appear. According to the American Heart Association, diabetics over the age of 50 are at "extremely high risk" of PAD and should be considered for screening. Generally speaking, screening for PAD should be performed in patients 50 years of age and older who smoke and/or have diabetes, all patients 70 years of age and older, as well as any patient with symptoms of PAD. (Fauci et al., 2008, Chapter 243)
Q 98.7: What is the most common site of an acute arterial occlusion due to embolic disease? A Iliac artery B Aortic bifurcation C Mesenteric arteries D Femoral artery E Popliteal artery
The Correct Answer is: D The most common site for an acute embolic occlusion is the femoral artery. Other common sites include the axillary, popliteal, and iliac arteries as well as the aortic bifurcation and mesenteric vessels. The majority (80%) of arterial embolic originate in the heart in patients with atrial fibrillation or from mural thrombi in the left ventricle from an akinetic or dyskinetic portion of the myocardium following a myocardial infarction.
Q 1.4: A 26 year-old man presents to the clinic after developing a severe productive cough, fever, chills, night sweats and weight loss. Which is the most definitive diagnostic test for determining the presence of Mycobacterium tuberculosis as the causative agent of his active pulmonary tuberculosis and yields results in less than 24 hours? A Acid-fast bacilli smear of sputum B Blood culture C Mycobacterial culture of sputum D Nucleic acid amplification of sputum E Tuberculin skin test
The Correct Answer is: D The most definitive diagnostic tests for confirmation of tuberculosis are mycobacterial culture (C) which requires at least 2 weeks to grow mycobacterium, and nucleic acid amplification (D) of a respiratory specimen that can be completed in a matter of hours. AFB smear (A) has a sensitivity of only 40-60%, but positive results help establish a presumptive diagnosis. Tuberculin skin testing (D) is effective in screening patients for latent TB. Routine blood cultures (B) are ineffective in diagnosing TB.
Q 47.7: A 2-month-old female presents for a well child check. The mother has no concerns and feels that the child is doing well. On exam, there is no evidence of cyanosis and the peripheral pulses are normal and equal. However, there is a fixed and widely split S2, a right ventricular heave, and a systolic ejection murmur present. The murmur is heard best at the left sternal border second intercostal space. What is the most common abnormality present on an ECG? A Atrioventricular heart block B Atrial fibrillation C Bifasicular block D Right axis deviation E Supraventricular tachycardia
The Correct Answer is: D The most likely diagnosis is an atrial septal defect, which usually shows right axis deviation on ECG. The other ECG abnormalities listed do not commonly occur with an atrial septal defect.
Q 115.19: A 31-year-old male presents due to excessive daytime somnolence affecting his ability to work. He states that he often sleeps at inappropriate times and alters his schedule to avoid being caught. He describes an episode yesterday where he felt almost paralyzed upon awakening from sleep. He denies any other neurologic symptoms, recent illness, or previous sleep testing. He denies depression symptoms. Which of the following is the most appropriate diagnostic test to confirm the suspected diagnosis? A Ambulatory pulse oximetry monitoring B Brain MRI C Electroencephalogram D Multiple sleep latency test E Thyroid stimulating hormone
The Correct Answer is: D The most likely diagnosis is narcolepsy, based upon the patient's symptoms. Evaluation should include a multiple sleep latency test. Consideration should also be given to a sleep study with polysomnography, to evaluate for underlying sleep apnea. An in-depth sleep history should be obtained to rule out chronic sleep deprivation. Additional differentials should be considered (and are suggested by the other answer choices), but will be based upon the patient's presentation, symptoms, past medical history, and likelihood of additional diagnoses. (Fauci et al., Harrison's Principles of Internal Medicine, 17e, Chapter 28, Sleep Disorders)
Q 67.5: A 67-year-old man presents with pain and stiffness in his shoulders and hips lasting for several weeks with no history of trauma. He also has complaints of headache, throat pain, and jaw claudication. It is imperative to diagnose this patient promptly in order to prevent which of the following complications? A anemia B cerebral aneurysms C mononeuritis multiplex D ischemic optic neuropathy E respiratory tract complications
The Correct Answer is: D The most urgent need for diagnosis of a patient with symptoms of polymyalgia rheumatica (PMR) and giant cell arteritis is to prevent blindness caused by ischemic optic neuropathy as a result of occlusive arteritis of the ophthalmic artery. Early diagnosis is imperative as the neurological damage to the optic nerve is not reversible. Most patients with this diagnosis will have a normochromic-normocytic anemia, but this does not create urgency in treatment. Cerebral aneurysms are not common findings with PMR; large vessels such as the subclavian and aorta may be involved in giant cell arthritis in 15% of patients. Mononeuritis multiplex commonly presents with painful paralysis of a shoulder, and respiratory tract complications are more nonclassic findings with the presentation of PMR. (Hellmann and Imboden, 2008, pp. 739-740) Hellmann DB , Imboden JB. Arthritis and musculoskeletal disorders. In: Tierney LM , McPhee SJ , Papadakis MA, eds. 2008 Current Medical Diagnosis and Treatment. New York, NY: McGraw-Hill; 2008:703-756.
Q 39.4: A 16-year-old boy is seen for a sports physical prior to starting football. On auscultation, a grade II/IV holosystolic murmur is appreciated at the apex. Utilizing isometric hand grip exercises, the murmur increases in intensity and can be heard radiating to the axilla. With the Valsalva maneuver, the murmur decreases in intensity. Given the patient's physical exam findings, which of the following is the most likely diagnosis? A Aortic stenosis B Aortic regurgitation/insufficiency C Mitral stenosis D Mitral regurgitation/insufficency E Tricuspid regurgitation
The Correct Answer is: D The murmur of mitral regurgitation can be described as a holosystolic murmur, usually heard best at the apex, so choice D is the most appropriate answer. Isometric hand grip exercises increase the intensity of the murmur of mitral regurgitation by increasing arterial and left ventricular pressure, which increases the flow across the mitral valve, thereby increasing the murmur's intensity. The murmur of mitral regurgitation is heard best at the apex. Radiation, if it occurs, is frequently to the axilla. Choice A is incorrect, as the murmur of aortic stenosis is usually described as a crescendo-decrescendo or systolic ejection murmur that is heard best at the right upper sternal border. Choice B is incorrect as the murmur of aortic regurgitation is usually described as a high-frequency decrescendo early diastolic murmur that is heard best at the left upper sternal border or at the right upper sternal border. Choice C is incorrect, as the murmur of mitral stenosis is described as a low-frequency rumbling diastolic murmur that is decrescendo in early diastole, but may become crescendo up to the first heart sound with moderately severe mitral stenosis and sinus rhythm. Choice E is incorrect, as the murmur of tricuspid regurgitation is described as a holosystolic descrescendo murmur.
Q 116.5: A 45-year-old male with asthma and diabetes is diagnosed with influenza B by nasal swab. He has been ill for one and a half days. Which of the following is indicated for treating this patient? A Acyclovir B Amantadine C Nevirapine D Oseltamivir E Zanamivir
The Correct Answer is: D The neuraminidase inhibitors, including oseltamivir and zanamivir, are associated with a reduction in duration of illness, and secondary complications for both influenza A and B viral strains. However, zanamivir, due to the oral inhalation delivery route, is relatively contraindicated in this patient, due to his history of asthma and an associated increased risk of bronchospasm. Amantadine is inactive against influenza B, as well as certain influenza A strains. Nevirapine is an antiretroviral agent used in the treatment regimen for HIV. Acyclovir is an antiviral agent, but is not indicated for influenza. (Fauci et al., Harrison's Principles of Internal Medicine, 17e, Chapter 180, Influenza)
Q 24.9: A 56-year-old right hand dominant male presents with swelling to the right elbow. He explains to you that he has had this type of swelling before and was diagnosed with bursitis. Based on this description, where would the most likely site for the elbow bursitis be located? A Capitellum B Lateral epicondyle C Medial epicondyle D Olecranon E Radial head
The Correct Answer is: D The olecranon bursa is very vulnerable to injury and chronic inflammation due to its superficial location on the extensor side of the elbow. Acute falls, chronic pressure, and other inflammatory processes such as rheumatoid arthritis or gout may contribute to olecranon bursitis. The medial and lateral epicondyles are known to be associated with chronic tendonopathies. The radial head is a common location for subluxation or fracture, while the capitellum at the distal end of the humerus is sometimes associated with fractures and articular cartilage damage.
Q 21.1: A 32-year-old woman comes to your office for contraception. She has divorced her husband two weeks ago and is now dating several new boyfriends. She states she has started a new company just over a week ago and has sunk all of her money into it. Although she knows that businesses like hers have not done well lately, she is certain the market is about to shift in her favor. She states she must hurry and get back to work, that she hasn't even had time to sleep for over a week. She speaks very rapidly and is pacing back and forth, seemingly unable to sit still. Conversation is difficult as she is easily distractible. She denies use of drugs or alcohol. She also denies any history of depression. What is her most likely diagnosis? A Bipolar disorder B Delusional disorder C Hypomanic episode D Manic episode E Neurosis
The Correct Answer is: D The patient described is exhibiting behaviors consistent with a manic episode (D). Her symptoms have been present for over a week and are impairing her ability to function, so it is not hypomania (C). As she has no clear history of depression, it is likely that she will have a depressive episode in the future, which will allow for a diagnosis of bipolar disorder (A). She does exhibit grandiosity, but there are no clear delusions (B). Neurosis (E) is a more general term for cognitive distress including some form of anxiety.
Q 14.4: A 48-year-old man presents to the clinic for a routine employment physical. The patient is asymptomatic. A pre-employment CBC reveals hemoglobin of 13.2g/dl, hematocrit of 39.5%, and MCV of 60.6 fL. Subsequent iron studies, hemoglobin electrophoresis, and sickle cell screening are "normal." After the labs are reviewed, the patient states "they are always normal." Which of the following is the most appropriate management for this patient? A Bone marrow aspirate B Ferrous sulfate daily C Folic acid supplements D Reassurance E Refer to an oncologist
The Correct Answer is: D The patient has a mild anemia with pronounced microcytosis consistent with alpha thalassemia minor and requires reassurance. Alpha thalassemia minor hemoglobin electrophoresis reveals normal results. Patients with alpha thalassemia are often inappropriately advised to take iron supplements (B) and don't require iron, folic acid (C) or vitamin B 12 supplements unless a coexisting deficiency is confirmed. Referral to an oncologist (E) and a bone marrow aspirate (A) aren't necessary for this patient.
Q 9.4: A 48-year-old female presents with a gradual progression of fatigue, pallor, and dyspnea on exertion over the last few months. Initial CBC results show a hemoglobin of 10.2 mg/dL, hematocrit of 31%, an MCV of 74 fL, and a reticulocyte count of 0.1%. Which of the following is the most likely pathophysiologic mechanism responsible for her anemia? A Acute blood loss B Defective bone marrow/stem cell function C Defective DNA production D Defective hemoglobin production E Increased destruction of red blood cells
The Correct Answer is: D The patient most likely has an underlying iron deficiency anemia (IDA), which requires confirmation and then evaluation for sources of chronic blood loss. IDA results from defective hemoglobin production. Acute blood loss (A) would present more acutely and with normal hemoglobin, hematocrit, and MCV until hemodilution occurs and lowers the hemoglobin concentration and hematocrit. Defective bone marrow/stem cell function (B) tends to produce normocytic red blood cells. Defective DNA production (C) results in failure of RBC maturation and macrocytosis (elevated MCV). Hemolysis (E) is less likely than IDA, but can be confirmed or ruled out through the assessment of iron studies, the reticulocyte count and further hematologic labs as necessary.
Q 15.9: A 23-year-old man presents to the outpatient clinic for follow-up from a recent urgent care visit. He complains of sore throat, fever, fatigue, myalgias, and a rash that started 5 days ago, and have worsened since he was seen in the urgent care 3 days ago. The patient appears non-toxic with a temperature of 39.4 degrees Celsius. Physical exam reveals pharyngeal and tonsillar erythema without exudates, generalized lymphadenopathy, a morbilliform rash on his trunk, and no hepatosplenomegaly. A rapid strep screen and Monospot performed at the local urgent care were reportedly negative. Which of the following laboratory tests is most likely to confirm the expected diagnosis? A Complete blood count B Cytomegalovirus titer C Group A beta-hemolytic strep culture D HIV viral load E Epstein-Barr virus titer
The Correct Answer is: D The patient presentation is consistent with acute retroviral syndrome, which is best confirmed during this initial presentation phase through direct testing for the HIV virus, such as an HIV viral load (D). The lack of tonsillar exudates, a negative monospot, and presence of generalized adenopathy make infectious mononucleosis (B and E) less likely. A CBC (A) may show lymphopenia and support the diagnosis, but it doesn't confirm the diagnosis.
Q 7.7: A 45-year-old man presents to the clinic complaining of a lesion in the middle of his right lower eyelid margin that is obscuring his vision when he works on fly-tying or other detailed projects. On physical exam there is a 5 mm round lesion that lacks erythema or tenderness. The patient's visual acuity is 20/20. Which of the following interventions is most appropriate for this patient? A Apply warm compresses four times daily until resolution B Apply bacitracin ophthalmic ointment 500 units/gram three times a day C Aspirate the lesion D Excise the lesion E Take amoxicillin-clavulanate 875/125 mg by mouth two times a day for 10 days
The Correct Answer is: D The patient's presentation is consistent with a chalazion that is best treated by surgical excision of the granulomatous tissue (D). Warm compresses (A), bacitracin ointment (B), and amoxicillin-clavulanate (E) are effective treatment options for hordeolum, or hordeoulum with associated preseptal cellulitis.
Q 121.12: You are conducting a physical exam on a female, who was referred to you from an optometrist. She sought a visual screening due to progressive loss of visual acuity. She has not been seen by a physician in 10 years due to lack of insurance. She admits to a 15 lb weight gain in the past three years, and also complains of parasthesias in her feet. During an ophthalmoscopic exam you notice deep retinal microvascular hemorrhages, and cotton wool spots. What is the most likely cause of her visual disorder? A Macular degeneration B Retinal detachment C Central retinal artery occlusion D Diabetic retinopathy E Central retinal vein occlusion
The Correct Answer is: D The patient's symptoms suggest a likelihood of diabetes. Retinal findings can include microaneurysms, deep hemorrhages, a flame-shaped hemorrhage, exudates, and cotton wool spots. (Riordan et al., 2008, Chapter 6)
Q 108.7: A 58-year-old man presents with the acute onset of abdominal pain associated with fever and shaking chills. The patient is hypotensive and febrile with a temperature of 102.2°F. Although he is confused and disoriented, he complains of right upper quadrant pain during palpation of the abdomen. His sclerae are icteric and the skin is jaundiced. Which of the following is the most likely diagnosis? A Acute cholecystitis B Choledocholithiasis C Acute pancreatitis D Ascending cholangitis
The Correct Answer is: D The presenting symptoms associated with ascending cholangitis include fever, chills, right upper quadrant pain, and jaundice (Charcot's triad); the symptoms are secondary to an infected obstruction of the common bile duct. With spread of the infection, the patient may also develop hypotension and mental status changes; these additional symptoms in conjunction with Charcot's triad are known as Reynolds' pentad. Additional symptoms of common bile duct obstruction include light-colored stools and dark, tea-colored urine. (Danziger, 2006, pp. 344-345) Danzinger RG , Nauta R , Park J. Biliary tract. In: Lawrence PF, ed. Essentials of General Surgery. 4th ed. Philadelphia, PA: Lippincott Williams & Wilkins; 2006.
Q 68.3: A 67-year-old male with a history of dilated cardiomyopathy, 3-vessel coronary artery disease, and an ejection fraction of 20% has a history of palpitations for the last two weeks. A 24-hour Holter monitor is ordered and reveals >1000 episodes of the attached rhythm strip. His blood pressure on examination is 102/62, and his resting pulse is 52. Based on these findings, what is the best therapy to treat the patient's symptoms? A Medical management B Coronary bypass surgery C Ablation therapy D Biventricular pacing E No therapy needed
The Correct Answer is: D The rhythm strip reveals episodes of non-sustained ventricular tachycardia. Due to the patient having a low ejection fraction with the dilated cardiomyopathy, primary prevention for this patient at a minimum would be to provide him with a biventricular pacer. (Bashore et al., Current Medical Diagnosis and Treatment, Chapter 10)
A 39-year-old male patient presents with low back pain with radiation to the right leg. On examination you place the right hip in a flexed position, and as you palpate between the iscial tuberosity and the greater tronchanter of the femur the patient complains of radiation of pain down his right leg. Based on this history and exam, which nerve was affected by this part of the examination maneuver? A Femoral B Peroneal C Saphenous D Sciatic E Sural
The Correct Answer is: D The sciatic nerve does lie midway between the ischial tuberosity and greater trochanter and it can be palpated when the patient is in a hip flexed position. The gluteus maximus obscures the nerve from being effectively palpated when the leg is in an extended position. Tenderness of the sciatic nerve can be caused by a lumbar disk herniation, direct trauma, or spasm of the nearby pyriformis muscle. The femoral nerve is a deep structure that lies lateral to the femoral artery and is not considered to be palpable. The femoral nerve is responsible for the L1-3 dermatomes and for supplying motor function to the iliopsoas muscle. The peroneal nerve originates from the sciatic nerve and splits into the superficial and deep peroneal nerves, which are responsible for much of the sensory and motor nerve function in the lower leg. The saphenous nerve originates from the femoral nerve in the femoral triangle and runs down the medial aspect of the leg. The sural nerve has medial and lateral components that are found in the lower leg. The medial cutaneous sural nerve arises from the tibial nerve just below the knee and eventually connects with peroneal nerve to form the sural nerve. On the lateral side of the lower leg, the sural nerve arises from the common peroneal nerve just above the knee and eventual connects with the previously discussed medial branch to form the sural nerve.
Q 49.5: A two-week-old female is being evaluated in the clinic, and on examination she is noted to have bounding pulses with a widened pulse pressure. There is a rough, machinery sounding murmur present at the second left intercostal space. Cyanosis is not present. Which of the following diagnostic tests would be the most useful in confirming the suspected diagnosis in this patient? A Cardiac catheterization B Chest x-ray C ECG D Echocardiography E Nuclear stress test
The Correct Answer is: D The suspected diagnosis is a patent ductus arteriosus. The most useful test in confirming the diagnosis is echocardiography. This test provides direct visualization, and confirms the direction and degree of shunting. Cardiac catheterization is not required for diagnosis, but the catheterization lab is used to perform the surgical procedure to close the patent ductus arteriosus. A chest x-ray may be normal in cases of a small shunt. ECG may also be normal if the size of the shunt is small. A nuclear stress test is not used in evaluation of a patent ductus arteriosus.
Q 115.10: A 54 year old male presents with dark thickened skin and soft pedunculated papules around his neck. He states that the lesions are asymptomatic. What disease process are the findings commonly associated with? A Hypertension B Malignant melanoma C Fungal infection D Metabolic syndrome
The Correct Answer is: D The thick dark plaque around the patient's neck is acanthosis nigricans. The papules are acrochordons (or skin tags). Both conditions are associated with metabolic syndrome. (Wolff & Johnson, p231)
A 67-year-old female with a history of oxygen dependent emphysema presents with a 4-hour history of increasing shortness of breath and pleuritic chest pain on the right side. Her resting oxygen saturation rate is 90%, and she is having pain on inspiration. On examination, the patient has decreased lung sounds with wheeze on the left and absent sounds on the right. There is also tympany to percussion on the right. Based on these findings, what is the best therapy for this patient? A Needle insertion to right chest wall B Supportive care C Increased oxygen delivery D Chest tube insertion E Nebulized albuterol
The Correct Answer is: D The treatment for this patient, who has a pneumothorax, is chest tube insertion and reinflation of the lung. Once the air leak has been eliminated and the lung appears reinflated on serial chest x-rays, the chest tube may be removed.
Q 98.18: A 67-year-old female with a history of oxygen dependent emphysema presents with a 4-hour history of increasing shortness of breath and pleuritic chest pain on the right side. Her resting oxygen saturation rate is 90%, and she is having pain on inspiration. On examination, the patient has decreased lung sounds with wheeze on the left and absent sounds on the right. There is also tympany to percussion on the right. Based on these findings, what is the best therapy for this patient? A Needle insertion to right chest wall B Supportive care C Increased oxygen delivery D Chest tube insertion E Nebulized albuterol
The Correct Answer is: D The treatment for this patient, who has a pneumothorax, is chest tube insertion and reinflation of the lung. Once the air leak has been eliminated and the lung appears reinflated on serial chest x-rays, the chest tube may be removed.
Q 71.4: A 67-year-old female with a history of oxygen dependent emphysema presents with a 4-hour history of increasing shortness of breath and pleuritic chest pain on the right side. Her resting oxygen saturation rate is 90%, and she is having pain on inspiration. On examination, the patient has decreased lung sounds with wheeze on the left and absent sounds on the right. There is also tympany to percussion on the right. Based on these findings, what is the best therapy for this patient? A Needle insertion to right chest wall B Supportive care C Increased oxygen delivery D Chest tube insertion E Nebulized albuterol
The Correct Answer is: D The treatment for this patient, who has a pneumothorax, is chest tube insertion and reinflation of the lung. Once the air leak has been eliminated and the lung appears reinflated on serial chest x-rays, the chest tube may be removed. (Chesnutt MS, Prendergast TJ. Current Medical Diagnosis and Treatment, 2011, Chapter 9, Pulmonary Disorders)
Q 86.4: Your patient with type 2 diabetes mellitus (T2DM) who is already on metformin and pioglitazone but continues to have elevated fasting and postprandial blood sugars. Together, you elect to begin insulin glargine at 10 U QHS initially and have the patient self manage their upwards titration adding 1 IU SQ in the evening every day that their morning fasting glucose is above 100 mg/dL. You caution the patient that she should expect to take approximately how long to achieve well-controlled sugars? A immediately B about one week C about two weeks D about one month E about three months
The Correct Answer is: D There are many regimens to begin and titrate basal insulins. Regardless, most patients require approximately 45 units of glargine to reach therapeutic goals. If they began glargine at 10 IU and add 1 IU daily until they reach 45 IU, this should take approximately 35 more days, or approximately one month. (Fauci et al., 2008, Chapter 338)
Q 39.3: A 2-year-old girl has developed a "barking" cough and a low-grade fever. She has some runny nose and her voice is somewhat raspy and hoarse. When approached by the PA, she becomes somewhat upset and exhibits mild inspiratory stridor. She appears otherwise well and has no drooling or dyspnea. What is the most likely etiology of this child's illness? A Adenovirus B Coxsackievirus C Cytomegalovirus D Parainfluenza virus E Respiratory syncytial virus
The Correct Answer is: D This child has the classic presentation for croup. Sixty-five percent of croup is caused by a parinfluenza virus infection. Adenoviruses tend to cause severe pharyngitis/tonsillitis with tender, enlarged cervical nodes (A), while Coxsackievirus is commonly associated with hand-foot-and-mouth disease. (B) Cytomegalovirus causes an infectious-mononucleosis-like picture. (C) RSV is more typically associated with bronchiolitis. (E)
Q 120.17: A new patient to the practice reports that his paternal grandfather had a tumor of the pituitary gland, his father had hyperparathyroidism, one uncle had a pancreatic tumor and another a thyroid cancer. His first cousin has "some kind of facial tumors" and another relative had treatment for "producing too much stomach acid." Given this family history, he should be screened for which of the following? A Carney complex B Cowden disease C McCune-Albright syndrome D MEN1 E Sipple syndrome
The Correct Answer is: D This family history strongly suggests Multiple Endocrine Neoplasia 1, which are inherited in an autosomal dominant manner and involve mutations on the long arm of chromosome 11 (11a13). Carney complex (A) is another syndrome of multiple endocrine neoplasia (MEN) with tumors of the adrenal cortex, pituitary gland, thyroid, and gonads and with hyperpigmentation and cardiac myxomas. Cowden disease (B) is also a syndrome of MEN including thyroid abnormalities, breast cancer and hamartomas. Another MEN, McCune-Albright syndrome (C) is associated with precocious puberty, Cushing syndrome, hyperthyroidism, and acromegaly. Sipple syndrome (E), also known as MEN2a. may develop medullary thyroid carcinoma, pheochromocytomas, or Hirschsprung disease. Fitzgerald PA, Endocrine Disorders, in Current Medical Diagnosis and Treatment, 52 nd ed. 2013.
Q 88.4: A two-month-old bottle fed infant female presents with a sharply demarcated scaling red rash on the face and in the diaper area. The mother reports that her child has been irritable and has had diarrhea. The primary care provider has treated with hydrocortisone 2.5% ointment bid for two weeks with no improvement. What should be the next step in confirming the diagnosis? A punch biopsy at border of genital rash B bacterial culture and viral culture C KOH and bacterial culture D cbc with diff and Zn level
The Correct Answer is: D This infant is displaying classic signs of zinc deficiency. This disorder can occur in infants who are bottle fed. Treatment with topical steroids will not improve the condition until the zinc level is corrected. A punch biopsy would not help in diagnosing the zinc deficiency that this patient is exhibiting. Bacterial and viral cultures will not be helpful in establishing the diagnosis. A KOH is used to diagnose fungal infections. The KOH and bacterial culture will not be helpful in establishing the diagnosis of zinc deficiency. (Wolff et al., 2009, Pages 442-443)
Q 46.8: An 8-year-old female presents with numerous discrete, skin colored papules with a central umbilication. The lesions are mildly pruritic. What is the most likely diagnosis? A herpes simplex virus B varicella zoster virus C comedonal acne D molluscum contagiosum
The Correct Answer is: D This is classic presentation of Molluscum contagiosum. Molluscum primarily affects younger children, from infancy through elementary age. Lesions appear as pearly, skin colored papules with a central umbilication. They can be pruritic. Herpes simplex virus is described as a vesicle that crusts after a few days. Neurogenic pain is associated with the outbreak. The varicella zoster virus is chicken pox. These lesions are described as "a dew drop on a rose petal," due to the characteristic vesicle on a red base. They occur in crops, with some being vesicular and some being crusted. There is no umbilication. Comedonal acne primarily affects teens. These lesions are located on the face, back, and chest.
Q 110.4: A 22-year-old complains of a spreading rash. He states it began about a week ago, with one large spot on his abdomen. Very shortly thereafter, the rash rapidly spread on his torso. The lesions are small oval red plaques with a collarette of scale. He states they are only mildly pruritic. What is the most likely diagnosis? A tinea corporis B guttate psoriasis C atopic dermatitis D pityriasis rosea
The Correct Answer is: D This is the classic presentation of pityriasis rosea. The large initial lesion is the herald patch. The ensuing lesions are small, oval, and have a collarette of scale. The lesions are distributed along Blaschkoe's lines, which is the "Christmas tree" distribution. Presentation of tinea corporis is pruritic, annular scaling patches that enlarge with central clearing. Guttate psoriasis presents as salmon-pink papules or small plaques with overlying fine silvery scales. Atopic dermatitis is usually manifested prior to the age of six. The classic distribution is the flexural surfaces of the extremities. (Wolff et al., 2009, Page 122)
Q 24.7: A 56-year-old male was admitted to the regular medical floor with a diagnosis of sepsis due to urinary tract infection. While on the floor he became acutely unstable and had a blood pressure of 70/40. A medical emergency code was called, which revealed the rhythm shown. There was no palpable pulse on exam, and no evidence of heart sounds on auscultation. The patient was not responding to any stimulus during this time, and there did not appear to be any spontaneous respirations noted. A single, unsynchronized cardioversion shock is given, and the patient has restored blood pressure, and his mental status and responsiveness returns to baseline. Given this clinical scenario, what long-term medical therapy is indicated for this patient? A Observation B Permanent pacemaker insertion C Long-term anticoagulation D Implantable cardiac defibrillator E Long-term antibiotics
The Correct Answer is: D This patient developed ventricular fibrillation with hypotension. When a patient has an event of this magnitude, prevention of further events will be a goal, to reduce sudden cardiac death. In this particular situation, the patient would benefit from an implantable cardiac defibrillator (D). This would be accomplished once the acute sepsis is resolved and the patient has been stabilized. The patient would not survive with observation alone (A), and once a rhythm is reestablished a permanent pacemaker (B) is not indicated. Long-term anticoagulation (C) or long-term antibiotic do not address the underlying rhythm problem.
Q 69.3: Your 48-year-old male diabetic patient presents for a new-patient visit and a complete physical examination. You elicit a history approximately one month ago of some vague complaints of left shoulder discomfort that lasted about 30 minutes and occurred during strenuous exercise. At the time, he broke out in a sweat. The patient has had no symptoms whatsoever since. The physical examination is normal, as are his vital signs, and his ECG is below. His ECG below is unchanged from previous ECGs. What is the most appropriate plan of action for this patient? A Give the patient a full-dose aspirin and call 911. B Draw labs including CBC, CMP, a fasting lipid panel, hs-CRP, HbA1C, TSH, and treat accordingly when the labs return with a follow-up visit in two weeks. C Draw labs including a CK-MB and troponin, and treat accordingly when the labs return with a follow-up visit in two weeks. D Draw labs including a CBC, CMP, fasting lipid panel, HS-CRP, HbA1C, TSH, and treat accordingly when the labs return with a follow-up visit in two weeks; until that time start the patient on an 81 mg enteric-coated aspirin, metoprolol ER 25 mg once daily and schedule an urgent appointment within the next 72 hours with a cardiologist for a nuclear stress test insuring that the patient knows to call 911 if the symptoms re-occur. E Treat the patient with a proton-pump inhibitor and begin life style changes.
The Correct Answer is: D This patient has a normal ECG and is asymptomatic for many days, but may have had a precursor episode of an acute coronary syndrome. Acute coronary syndromes (ACS) comprise the spectrum of unstable cardiac ischemia from unstable angina to acute myocardial infarction. Rather than the traditional nomenclature of unstable angina, non-Q wave and Q wave myocardial infarction, acute coronary syndromes are now classified based on the presenting ECG as either "ST elevation" or "non-ST elevation." This allows for immediate classification and guides determination of whether patients should be considered for acute reperfusion therapy. ACS represent a dynamic state in which patients frequently shift from one category to another. This warrants protecting the patient with both beta-blockers and aspirin (unless contra-indicated), as well as a cardiology consultation including a stress test, but does not escalate to the point of sending the patient to the emergency department. (McPhee and Papadakis, 2011, Chapter 10)
Q 104.2: A 54-year-old male complains of easy fatigability, loss of sense of wellbeing, anorexia, weight loss, and excessive sweating that has been going for approximately three months. On physical exam he has pallor and splenomegaly. White count is 98,000 (4 to 12,000). The marrow is hypercellular and contains the Ph chromosome. What is the most likely diagnosis? A Acute lymphocytic leukemia B Acute myelogenous leukemia C Chronic lymphoblastic leukemia D Chronic myelogenous leukemia E Multiple Myeloma
The Correct Answer is: D This patient has chronic myelogenous leukemia. 90% of patients presenting with CML are Ph chromosomes positive, and have vague symptoms and complaints that have been going on for weeks to months. Acute lymphocytic leukemia on CBC tends to have an eosinophilia. Acute myelogenous leukemia is not associated with the Ph chromosome. Chronic lymphoblastic leukemia marrow examination notes lymphocytic infiltrates. Multiple myeloma is noted for light chains and not ph chromosome. (Lichtman et al., Williams Hematology 8e, Chapter 90, Chronic Myelogenous Leukemia and Related Disorders)
Q 106.16: A 31-year-old female presents with headache, fever, and petechiae to her lower extremities. A microangiopathic hemolytic anemia is noted. She denies diarrhea, recent infectious episodes, cough, shortness of breath, or urinary symptoms. What other finding would you expect to find in this woman to confirm the diagnosis? A Blasts on blood smear B Edema C Hepatomegaly D Thrombocytopenia E Sickle Cell
The Correct Answer is: D This patient has classic signs of thrombotic thrombocytopenic purpura. Microangiopathic hemolytic anemia and thrombocytopenia are seen in all cases of TTP in the absence of another plausible explanation. Edema is not seen in TTP. Sickle cell is found in sickle cell anemia. Hepatomegaly is seen in less than 20% of cases of TTP and does not confirm the diagnosis. Blasts are seen in leukemia and do not confirm the diagnosis. (Thrombotic Microangiopathy, Quick Answers to Medical Diagnosis and Therapy)
Q 64.1: A 45-year-old male presents to your office complaining of severe unilateral eye pain with some photophobia for one day. He denies any history of trauma. On examination and with staining, you notice a dendritic lesion to the cornea, and an otherwise normal examination. Which of the following medications would be contraindicated in this patient? A atropine ophthalmic drops B azelastine ophthalmic drops C levofloxacin ophthalmic drops D prednisolone ophthalmic drops
The Correct Answer is: D This patient has herpes simplex keratitis is an important cause of ocular morbidity. The ability of the virus to colonize the trigeminal ganglion leads to recurrences precipitated by fever, excessive exposure to sunlight, or immunodeficiency. The dendritic (branching) ulcer is the most characteristic manifestation. More extensive ("geographic") ulcers also occur, particularly if topical corticosteroids have been used. Ophthalmic corticosteroids in cases of suspected herpes simplex keratitis are contraindicated.
Q 110.18: A 65-year-old woman presents with severe mid-back pain of 2 weeks duration. She has no history of trauma. Radiographic evaluation reveals compression fractures of T11 and T12. A complete blood count, erythrocyte sedimentation rate, serum protein, serum calcium, phosphate, and parathyroid hormone levels are all within normal ranges. In addition to ordering a dual-energy x-ray absorptiometry (DEXA) scan, which of the following laboratory evaluations is most helpful in evaluating this patient for secondary causes of this presentation? A bone biopsy B rheumatoid factor C serum magnesium D 25-hydroxyvitamin D E antinuclear antibodies test
The Correct Answer is: D This patient has typical findings associated with osteoporosis. Most patients with osteoporosis are asymptomatic until fractures present. Fractures occur spontaneously and are associated with back pain of varied degrees. Serum calcium, phosphate, and parathyroid hormone levels are often normal. Since vitamin D deficiency state is common in osteoporosis, a 25-hydroxyvitamin D should be ordered. Bone biopsy is not indicated with this patient; this would be reserved for evaluating for osteomalacia. Rheumatoid factor and antinuclear antibodies would not be of importance with this patient presentation. Serum magnesium is associated more with evaluating for parathyroid or thyroid disorder and has no value with osteoporosis. (Fitzgerald, 2008, pp. 994-995) Fitzgerald PA. Endocrine disorders. In: Tierney LM , McPhee SJ , Papadakis MA, eds. 2008 Current Medical Diagnosis and Treatment. New York, NY: McGraw-Hill; 2008:949-1031.
Q 70.1: A 70-year-old man with a history of pulmonary hypertension and obstructive sleep apnea presents with complaints of increasing dyspnea while walking his dog. He has also recently noted increased lower extremity edema. On physical examination, jugular venous distension is noted. Auscultation of the chest demonstrates a high-pitched blowing diastolic murmur. With inspiration, the murmur increases in intensity and is heard over the second and third left intercostal spaces. An S 3 is appreciated. Palpation of the precordium reveals a hyperdynamic right ventricle, and both a systolic and diastolic thrill. The abdominal exam reveals hepatomegaly and splenomegaly. Based on this patient's history and physical exam findings, which of the following is the most likely finding on echocardiogram? A Tricuspid regurgitation B Aortic stenosis C Atrial septal defect D Pulmonic regurgitation/insufficiency E Mitral stenosis
The Correct Answer is: D This patient is demonstrating signs and symptoms of right heart failure, and with a history of pulmonary hypertension and a high-pitched diastolic blowing murmur (Graham Steell murmur), Choice D is the most likely of the choices offered. A blowing holosystolic murmur at the left lower sternal border is characteristic for tricuspid regurgitation, Choice A. Choice B, aortic stenosis, presents with a systolic ejection murmur. An atrial septal defect, Choice C, if large, could present with similar symptoms of exertional dyspnea secondary to a large shunt, but auscultation would reveal a moderately loud systolic ejection murmur that is heard best in the second and third interspaces. This is secondary to increased pulmonary arterial flow. Choice E, mitral stenosis, presents with a diastolic murmur heard best in the left lateral decubitus position, with the bell of the stethoscope at the apex. (McPhee et al., 2011, Chapter 10)
A 22-year-old African American male presents to the emergency department with shortness of breath, which started 2 hours prior to arrival. He does not have a history of pulmonary disease that he is aware of, and he states that in the past at random events he has had similar episodes. He does nothing to get the episodes to stop, and he also states that he feels his chest pounding at the same time of the shortness of breath. He has no medical history that he is aware of, and he takes no medications or any illicit drugs. On examination he is alert, awake, and oriented. His vital signs show T 99.0, P 142, R 18, and BP 132/82. His chest x-ray is negative for any acute cardiopulmonary disease, and his electrocardiogram has an irregularly irregular rate of 142 with visible delta waves. Based on the information provided, what anatomical pathway is responsible for the above findings of the delta waves? A Intranodal pathway B Bachmann's bundle C Perkinje fibers D Bundle of Kent E Bundle of His
The Correct Answer is: D This patient is exhibiting Wolff-Parkinson-White (WPW) syndrome by evidence of the delta waves on the electrocardiogram. The anatomic pathway that is known to be the causative abnormality in WPW is the Bundle of Kent (D). Intranodal pathway (A), Bachmann's bundle (B), Perkinje (C), and His bundles (E) are not the anatomic pathways found with WPW.
Q 67.10: A 22-year-old African American male presents to the emergency department with shortness of breath, which started 2 hours prior to arrival. He does not have a history of pulmonary disease that he is aware of, and he states that in the past at random events he has had similar episodes. He does nothing to get the episodes to stop, and he also states that he feels his chest pounding at the same time of the shortness of breath. He has no medical history that he is aware of, and he takes no medications or any illicit drugs. On examination he is alert, awake, and oriented. His vital signs show T 99.0, P 142, R 18, and BP 132/82. His chest x-ray is negative for any acute cardiopulmonary disease, and his electrocardiogram has an irregularly irregular rate of 142 with visible delta waves. Based on the information provided, what anatomical pathway is responsible for the above findings of the delta waves? A Intranodal pathway B Bachmann's bundle C Perkinje fibers D Bundle of Kent E Bundle of His
The Correct Answer is: D This patient is exhibiting Wolff-Parkinson-White (WPW) syndrome by evidence of the delta waves on the electrocardiogram. The anatomic pathway that is known to be the causative abnormality in WPW is the Bundle of Kent (D). Intranodal pathway (A), Bachmann's bundle (B), Perkinje (C), and His bundles (E) are not the anatomic pathways found with WPW. Source: http://www.accessmedicine.com/content.aspx?aID=6357250&searchStr=kent+bundles#6357250
Q 26.5: A 44-year-old female presents to the emergency department with a right-sided headache. She states the headache is located on the right temple region, is non-radiating, and does not cause photophobia. She is otherwise healthy and has no reported medical problems. She only takes acetaminophen for the pain, which has minimal relief. On physical exam she is alert, awake, and oriented. Her vitals are T 98.8, P 78, R 18, and BP 128/76. Her head is normocephalic, atraumatic, and pupils are equal and reactive. She has tenderness to the right temporal area of the temporal artery, and there is no noted swelling, redness, or abnormalities noted. There is no tenderness to the cervical muscles. She exhibits a non-focal neurological exam. Based on the description, what is the most likely diagnosis of this patient? A Migraine headache B Tension headache C Cluster headache D Temporal arteritis E Takayasu arteritis
The Correct Answer is: D This patient is exhibiting a case of temporal arteritis (D). She has the signs and symptoms that are classic in nature and do not fit into the realm of the other headaches (A, B, and C), all of which usually have different characteristics on history and physical exam. Takayasu arteritis (E) will typically not present in the temporal artery.
A 44-year-old female presents to the emergency department with a right-sided headache. She states the headache is located on the right temple region, is non-radiating, and does not cause photophobia. She is otherwise healthy and has no reported medical problems and has only taken acetaminophen for the pain, which has minimal relief. On physical exam she is alert, awake, and oriented. Her vitals are T 98.8, P 78, R 18, and BP 128/76. Her head is normocephalic, atraumatic, and pupils are equal and reactive. She has tenderness to the right temporal area of the temporal artery, and there is no noted swelling, redness, or abnormalities noted. There is no tenderness to the cervical muscles. She exhibits a non-focal neurological exam. Based on the clinical presentation, what test is the best test in confirming an accurate diagnosis? A C-reactive protein B Erythrocyte sedimentation rate (ESR) C Computed tomography of head D Temporal artery biopsy E Complete blood count (CBC)
The Correct Answer is: D This patient is exhibiting a case of temporal arteritis. She has the signs and symptoms that are classic in nature and do not fit into the realm of the other headaches, all of which usually have different characteristics on history and physical exam. The best test to confirm the diagnosis is to biopsy the temporal artery (D) for pathologic confirmation. The three blood tests (A, B, and E) would only suggest the presence of the disease, but they are not specific enough to confirm. Computed tomography of the head (C) would not be indicated as temporal arteritis is an extracranial condition.
Q 23.4: A 44-year-old female presents to the emergency department with a right-sided headache. She states the headache is located on the right temple region, is non-radiating, and does not cause photophobia. She is otherwise healthy and has no reported medical problems and has only taken acetaminophen for the pain, which has minimal relief. On physical exam she is alert, awake, and oriented. Her vitals are T 98.8, P 78, R 18, and BP 128/76. Her head is normocephalic, atraumatic, and pupils are equal and reactive. She has tenderness to the right temporal area of the temporal artery, and there is no noted swelling, redness, or abnormalities noted. There is no tenderness to the cervical muscles. She exhibits a non-focal neurological exam. Based on the clinical presentation, what test is the best test in confirming an accurate diagnosis? A C-reactive protein B Erythrocyte sedimentation rate (ESR) C Computed tomography of head D Temporal artery biopsy E Complete blood count (CBC)
The Correct Answer is: D This patient is exhibiting a case of temporal arteritis. She has the signs and symptoms that are classic in nature and do not fit into the realm of the other headaches, all of which usually have different characteristics on history and physical exam. The best test to confirm the diagnosis is to biopsy the temporal artery (D) for pathologic confirmation. The three blood tests (A, B, and E) would only suggest the presence of the disease, but they are not specific enough to confirm. Computed tomography of the head (C) would not be indicated as temporal arteritis is an extracranial condition.
A 66-year-old male with a history of hypertension, diabetes mellitus, and hypercholesterolemia presents by emergency medical services (EMS) to the emergency department complaining of severe chest pain with radiation into his back. The patient states that he was feeling well in the morning, but while performing some light activity he felt a "ripping" sensation in his back, which he initially thought was a pulled muscle. The pain continued and the patient started to have chest pain, shortness of breath, and lightheadedness. On initial examination the patient is still in pain, pale, diaphoretic, and has a blood pressure of 85/40. His chest is clear to auscultation, and he has a 3/6 diastolic murmur best appreciated at the base of the heart. Given this clinical scenario, what would be the expected finding on chest x-ray? A Normal B Pleural effusion C Decreased lung volume D Widened mediastinum E Cardiomegaly
The Correct Answer is: D This patient is exhibiting a history and physical examination that is consistent with a thoracic aneurysm. The patient's history of hypertension, along with the "ripping" sensation in his back and hypotension give a clinical presentation that is suggestive of a thoracic aneurysm dissection. In this instance it would be expected that the patient would have evidence of a widened mediastinum (D) on chest radiography. While pleural effusions (B), decreased lung volumes (C), and cardiomegaly (E) can all be present on a chest film with an aneurysm, the widened mediastinum would be the expected finding if you suspect an aneurysm. It is very unlikely that a thoracic aneurysm with hypotension would have a normal chest radiograph finding (A).
Q 120.9: A 66-year-old male with a history of hypertension, diabetes mellitus, and hypercholesterolemia presents by emergency medical services (EMS) to the emergency department complaining of severe chest pain with radiation into his back. The patient states that he was feeling well in the morning, but while performing some light activity he felt a "ripping" sensation in his back, which he initially thought was a pulled muscle. The pain continued and the patient started to have chest pain, shortness of breath, and lightheadedness. On initial examination the patient is still in pain, pale, diaphoretic, and has a blood pressure of 85/40. His chest is clear to auscultation, and he has a 3/6 diastolic murmur best appreciated at the base of the heart. Given this clinical scenario, what would be the expected finding on chest x-ray? A Normal B Pleural effusion C Decreased lung volume D Widened mediastinum E Cardiomegaly
The Correct Answer is: D This patient is exhibiting a history and physical examination that is consistent with a thoracic aneurysm. The patient's history of hypertension, along with the "ripping" sensation in his back and hypotension give a clinical presentation that is suggestive of a thoracic aneurysm dissection. In this instance it would be expected that the patient would have evidence of a widened mediastinum (D) on chest radiography. While pleural effusions (B), decreased lung volumes (C), and cardiomegaly (E) can all be present on a chest film with an aneurysm, the widened mediastinum would be the expected finding if you suspect an aneurysm. It is very unlikely that a thoracic aneurysm with hypotension would have a normal chest radiograph finding (A). Source: http://www.accessmedicine.com/content.aspx?aID=3651494
Q 99.7: A 22-year-old male presents with his girlfriend. She is concerned because of her boyfriend's behavior and she feels that he needs help. During the visit, you learn that the behaviors of concern include excessive demands for attention and grandiosity. He is also preoccupied with power and shows little interest in others. What is the most likely classification of this personality disorder? A Antisocial B Avoidant C Histrionic D Narcissistic E Schizoid
The Correct Answer is: D This patient is exhibiting all of the characteristics of narcissistic personality disorder. Antisocial personality disorder is characterized by selfishness, callousness, promiscuousness, and impulsive behavior, and an inability to learn from experience and legal problems. Avoidant personality disorder presents clinically as someone who fears rejection, overreacts to rejection and failure, and has poor social endeavors and low self-esteem. Clinical findings of histrionic personality disorder include being dependent, immature, seductive, egocentric, vain, and emotionally labile. Schizoid personality disorder is characterized by shyness, introversion, being withdrawn, and avoiding close relationships. (McPhee SJ, Papadakis MA. Current Medical Diagnosis & Treatment, 2010, p. 951)
Q 32.31: An 84-year-old male is admitted to the hospital for the chief complaint of syncope. The history provided states that the patient was in his normal state of health and feeling well when he had fallen, with an apparent loss of consciousness for 25-30 seconds. He has a history of hypertension and arthritis, and is well managed on medications that include lisinopril and acetaminophen. During his time on the telemetry unit it's noted that the patient has periods of sinus bradycardia in the 30s, followed by normal sinus rhythms that fluctuate in the 60 to 120 range. During several of the bradycardia episodes the patient becomes symptomatic with shortness of breath, lightheadedness, and dizziness. His blood pressure during these episodes is measured at 88/56. Given this clinical scenario, what is the most appropriate treatment for this patient? A Observation B Increase in lisinopril C Cardiac catheterization D Permanent pacemaker insertion E Beta blocker therapy
The Correct Answer is: D This patient is having sick sinus syndrome, in which the patient has developed an aberrancy wherein the heart is not able to regulate a steady rate and maintain an adequate blood pressure. Because of the patient having syncope, observation only (A) is not advised. Increasing the ACE inhibitor (B) is contraindicated given the patient experiencing hypotension. This is also true with beta-blocker therapy (E). The best treatment for this clinical situation would be permanent pacemaker insertion (D). Cardiac catheterization (C) is not indicated in this situation, and does not address the electrophysiological issue.
Q 107.8: A 28-year-old male presents with complaints of pain along the cervical chain of lymph nodes after ingesting alcohol. In further discussion with him, he mentions having occasional drenching night sweats that have been occurring more frequently over the last month. He has also had a low-grade temperature of 38C on and off over the last several months, but attributed both to the summer weather. He also has lost weight over the last six months without trying. Recently, he has noted a generalized pruritus as well. He has palpable cervical chain lymph nodes. Histologic examination of a lymph node will most likely show which of the following? A Auer rods B IgG lyte chains C Philadelphia chromosome D Reed-Sternberg cells E VZV inclusion bodies
The Correct Answer is: D This patient is presenting with findings consistent with Hodgkin Lymphoma. Pain in involved lymph nodes immediately after the ingestion of alcohol is nearly specific to this disease. On histologic examination of lymph tissue, Reed-Sternberg cells can be seen. Auer rods are seen in AML. Philadelphia chromosome is typically seen in CML. IgG lyte chains are seen in multiple myeloma. VZV inclusion bodies would not be seen in this setting. (Lichtman et al., Williams Hematology 8e, Chapter 99, Hodgkin Lymphoma)
Q 109.10: A 29-year-old male who appears healthy was injured playing basketball and received an x-ray of his spinal column. A mass was noted on the film and referred to the hematologist. He denies shortness of breath or chest pain. He does note drenching night sweats that he has had for several weeks, but assumed it had to do with the hot weather. On exam he has painless lymphadenopathy and is underweight for his height. A CBC is normal and LDH is elevated. What is the most likely diagnosis? A Acute myelogenous leukemia B Cat scratch disease C Infectious mononucleosis D Non-Hodgkin lymphoma E Sarcoidosis
The Correct Answer is: D This patient presents with classic signs of Non-Hodgkin lymphoma, consisting of mediastinal mass, "B" symptoms, and painless lymphadenopathy. AML would not have a normal CBC. Cat scratch disease would have painful lymphadenopathy if present. Infectious mononucleosis would have lymphadenopathy and fever, but not drenching night sweats or mediastinal mass. Sarcoidosis would have granulomatous disease noted on a chest x-ray, not a large mediastinal mass. (McPhee SJ, Papadakis MA, Tierney LM. Current Medical Diagnosis and Treatment, 2010, Chapter 13, Blood Disorders)
Q 65.5: A 49-year-old female with a known history of hypertension presents to the emergency department with a generalized headache that is throbbing. She states she had run out of her normal blood pressure medication about a week ago and since then she has noticed that her headache came about and has been getting worse. She denies any nausea, vomiting, visual changes, chest pain, or other symptoms. On exam the patient has a BP 227/120, P 78, R 18. Her HEENT exam is essentially normal, lungs are clear to auscultation, and heart is a regular rhythm without murmur or gallop. Given this clinical situation, which medication would be the most appropriate to address the patients condition? A IV nitroprusside B Oral furosemide C Spironolactone D IV labetalol E Oral hydralazine
The Correct Answer is: D This patient's clinical situation is one of a hypertensive emergency. In this situation the goal is to bring down the systolic pressure to prevent end organ damage. Given the possible choices, the best choice would be intravenous labetalol (D) due to its effective quick onset, and its ability to be tolerated with most patients. While oral furosemide (B) and hydralazine (E) can both be effective in managing hypertension, the IV dosing of labetalol would be the better choice. Nitroprusside (A) is no longer a treatment option. Spironolactone (C) would not have strong enough effects to appropriately lower the blood pressure in an efficient manner.
Q 97.101: A 31-year-old woman is being evaluated for irregular, infrequent menstrual periods. On further questioning, she complains of headaches, fatigue, and breast discharge. She takes ibuprofen only occasionally. Which of the following labs would most likely be elevated in this patient? A BUN and creatinine B luteinizing hormone (LH) and follicle-stimulating hormone (FSH) C oxytocin D prolactin E TSH
The Correct Answer is: D This patient's symptoms are consistent with a pituitary adenoma. Prolactinomas account for about half of all functioning pituitary tumors and may secrete PRL, GH, and ACTH.
Q 120.8: A 31-year-old woman is being evaluated for irregular, infrequent menstrual periods. On further questioning, she complains of headaches, fatigue, and breast discharge. She takes ibuprofen only occasionally. Which of the following labs would most likely be elevated in this patient? A BUN and creatinine B luteinizing hormone (LH) and follicle-stimulating hormone (FSH) C oxytocin D prolactin E TSH
The Correct Answer is: D This patient's symptoms are consistent with a pituitary adenoma. Prolactinomas account for about half of all functioning pituitary tumors and may secrete PRL, GH, and ACTH. (Herman-Bonert, 2007, p. 640) Herman-Bonert VS. Hypothalamic-pituitary axis. In: Andreoli TA , Carpenter CC , Griggs RC, et al., eds. Cecil Essentials of Medicine. 7th ed. Philadelphia, PA: WB Saunders; 2007.
Q 22.3: An 82-year-old male presents to the emergency department with a 2-day history of weakness, fatigue, and mild shortness of breath. His past medical history includes hypertension, arthritis, diabetes, and hypercholesterolemia. His medications include metoprolol, glyburide, simvastatin, and acetaminophen. On physical examination, the vital signs reveal T 97.7, P 40, R 15, and BP 84/60. His neck exam reveals no jugular venous distention, lungs are clear to auscultation, and cardiac has a regularly irregular bradycardia rhythm. There is no evidence of edema to the extremities. The patient is placed on a telemetry monitor, which reveals the image shown. Based on the information provided, what is the most appropriate therapy for this patient? A Observation B Start cardiopulmonary resuscitation C Unsynchronized cardioversion D Permanent pacemaker insertion E Cardiac catheterization
The Correct Answer is: D This patient's telemetry strip is revealing complete heart block. Given this scenario it is unlikely that a rhythm such as this will convert back to a normal sinus rhythm. Therefore, a permanent pacemaker (D) is warranted. In this case, the patient would require immediate intervention due to his symptoms as well as his unstable hemodynamics. Patients left unattended in this setting may go into cardiac arrest due to the slow ventricular rate the patient has developed. The patient would not survive with observation alone (A). The patient has valid vital signs with some hemodynamics, so cardiopulmonary resuscitation (B) is not indicated and harmful. Unsynchronized cardioversion (C) and cardiac catheterization (E) are both not indicated.
Q 37.8: A six-month-old male has recurrent diaper rashes, which are not responding to ketoconazole cream or zinc oxide diaper creams. Physical exam shows well demarcated perianal erythema, with scattered red papules on the buttocks. A KOH is negative. What would appropriate management include? A hydrocortisone 2.5% ointment bid x 2 weeks B tretinoin 0.25% cream bid x 2 weeks C lotrisone cream bid until resolved D topical mupirocin bid until resolved
The Correct Answer is: D This rash is consistent with a perianal staph or strep infection. This bacterial infection is easily treated with topical mupirocin. If the rash does not resolve after two weeks of topical treatment, treatment with an oral antibiotic such as Keflex is usually successful. Treatment with a topical steroid may worsen the infection. Tretinoin is a topical retinoid indicated to treat acne, and is not appropriate for this patient. Lotrisone is a combination medication that includes an antifungal and a topical steroid, neither of which is indicated in this patient.
Q 97.30: A 30-year-old woman comes in for evaluation of infertility. She and her husband have been having unprotected intercourse for the past year. Her menstrual cramps have become increasingly painful, and she has a severe low backache for several days before and during her menses. She complains that intercourse is painful when her husband "goes deep." Physical examination reveals multiple tender nodules of various sizes in the posterior vaginal fornix. Definitive diagnosis is best accomplished using what methodology? A Abdominal radiography B CA-125 measurement C CT scanning of the abdomen D Laparoscopy E Pelvic ultrasonography
The Correct Answer is: D This woman has a classic presentation for endometriosis. Definitive or final diagnosis can only be made at laparoscopy or laparotomy, allowing direct visualization of the endometrial implants. Abdominal radiography (A), computed tomography scanning (C), and ultrasonography (E) are not usually helpful in making a definitive diagnosis. CA-125 (B) may be elevated in endometriosis, but lacks specificity.
Q 119.16: A 55-year-old right-hand dominant man presents with a 4-hour history of weakness and tingling of his right hand and numbness of the right side of his mouth. Mild difficulty was noted with word finding. His symptoms have improved since onset but have not fully resolved. There is no significant medical history. Physical examination revealed flat right nasolabial fold, subjective numbness of the right hand, right pronator drift, clumsiness of finger tapping on the right hand, increased deep tendon reflexes on the right, as well as a present Babinski. What is the most likely etiology for this patient's problem? A migraine headache B peripheral neuropathy C syncope D transient ischemic attack E seizure
The Correct Answer is: D Three key features of a transient ischemic attack include sudden onset and complete reversal of symptoms within 24 hours, usually within 15 minutes. The symptoms are usually in the anatomical distribution of a single blood vessel. This patient's history is not suggestive of migraine or syncope. His physical examination findings do not correlate with peripheral neuropathy or seizure. (Aminoff et al., 2005, p. 286) Aminoff MJ , Greenberg DA , Simon RP. Clinical Neurology. 6th ed. New York, NY: McGraw-Hill; 2005.
Q 105.11: A 63-year-old woman presents with shortness of breath, cough, and proximal muscle weakness of 1-month duration. On clinical exam, she is noted to have a blood pressure of 156/102 mm Hg, facial flushing, mild hirsutism, truncal obesity, marked proximal muscle weakness of both the upper and lower extremity, and hyperpigmentation over the palms and back of the neck. Laboratory exam reveals hypercortisolism and increased ACTH. Which of the following would be the most likely primary diagnosis in this patient? A lymphoma B ovarian cancer C renal cell carcinoma D small cell lung carcinoma
The Correct Answer is: D Tumor cells may secrete hormones that have the same biologic actions as the normal hormone. This patient's symptoms are consistent with adrenocorticoid hyperfunction. The most common cause of ectopic ACTH syndrome is small cell lung carcinoma. This should be suspected in any patient with risk factors for lung cancer. (Friedman, 2007, pp. 663, 591-592) Friedman TC. Adrenal Gland. In: Andreoli TA , Carpenter CC , Griggs RC, et al., eds. Cecil Essentials of Medicine. 7th ed. Philadelphia, PA: WB Saunders; 2007.
Q 98.71: A 62-year-old man presents to the office concerned about an abdominal aortic aneurysm (AAA). He has had no symptoms but states that his father died from an aortic dissection at the age of 50 and his brother was diagnosed last week with an AAA. What is the most appropriate screening tool in this situation? A Abdominal radiograph B Computed tomographic angiography (CTA) C Palpation D Ultrasound E Aortography
The Correct Answer is: D Ultrasonography is cost-effective and is the most commonly utilized screening modality for AAAs. It can be utilized for initial detection of a nonruptured AAA and for monitoring of progression. Anteroposterior and lateral abdominal radiographs may reveal calcification of an AAA as an incidental finding but are nonsensitive/nonspecific. CTA scans are used in monitoring for progression of AAA size and preoperatively to assess aortic anatomy. Abdominal palpation on physical examination may be reliable in thin patients but cannot accurately provide information about the presence or absence of aneurysms in all patients. Aortography is an expensive and invasive procedure traditionally used in preoperatively planning of AAA repair; however, it has largely been supplanted by multiplanar CTA scans.
Q 88.10: A 62-year-old man presents to the office concerned about an abdominal aortic aneurysm (AAA). He has had no symptoms but states that his father died from an aortic dissection at the age of 50 and his brother was diagnosed last week with an AAA. What is the most appropriate screening tool in this situation? A Abdominal radiograph B Computed tomographic angiography (CTA) C Palpation D Ultrasound E Aortography
The Correct Answer is: D Ultrasonography is cost-effective and is the most commonly utilized screening modality for AAAs. It can be utilized for initial detection of a nonruptured AAA and for monitoring of progression. Anteroposterior and lateral abdominal radiographs may reveal calcification of an AAA as an incidental finding but are nonsensitive/nonspecific. CTA scans are used in monitoring for progression of AAA size and preoperatively to assess aortic anatomy. Abdominal palpation on physical examination may be reliable in thin patients but cannot accurately provide information about the presence or absence of aneurysms in all patients. Aortography is an expensive and invasive procedure traditionally used in preoperatively planning of AAA repair; however, it has largely been supplanted by multiplanar CTA scans. (Rapp, 2006, pp. 810-811) Rapp JH , MacTaggart J. Arteries. In: Doherty GM , Way LM, eds. Current Surgical Diagnosis & Treatment. 12th ed. New York, NY: Lange Medical Books/McGraw-Hill; 2006.
Q 28.1: Which of the following is the most common cause of sudden death in young athletes under 35 years of age? A sthelosclerotic heart disease B Brugada syndrome C commotio cordis D idiopathic hypertrophic cardiomyopathy E Marfan syndrome
The Correct Answer is: D Undetected idiopathic hypertrophic cardiomyopathy is the most common cause of sudden death in otherwise healthy young athletes. A heart murmur, chest pain, or shortness of breath in this patient population should trigger an escalating level of investigation including consideration of at least both an EKG and an echocardiogram. Commotio cordis, while most common in adolescent athletes, remains extremely rare.
Q 76.5: A 38-year-old male presents with red-brown urine in the mornings. The discoloration has come and gone several times over the last several months. He has also noted hard, painful areas on his skin, which seem to be located over veins, which have occurred intermittently. Laboratories reveal nonspherocytic red cells and a Coombs negative intravascular hemolysis. What is the most likely diagnosis? A Acute Myelogenous Leukemia B Aplastic Anemia C Iron Deficiency Anemia D Paroxysmal Nocturnal Hemoglobinuria E Von Willebrand's Disease
The Correct Answer is: D Urine that appears to be bloody, primarily in the morning and episodic in nature, along with thrombosis of dermal veins, nonspherocytic red cells and Coombs negative intravascular hemolysis are classic signs of PNH. PNH may be a symptom of aplastic anemia, but this patient doesn't have a pancytopenia. He also does not have a microcytic anemia, which rules out iron deficiency. Von Willebrand's is a coagulation disorder. (Lichtman et al., Williams Hematology 8e, Chapter 40, Paroxysmal Nocturnal Hemoglobinuria)
Q 47.11: A 6-year-old child with leukemia presents with a very painful vesicular rash as seen in Figure 8-2. This is the only dermatologic manifestation but is accompanied by fever and malaise. His medical history is significant for the leukemia, tonsillectomy, and adenoidectomy at age 3 and chicken pox 4 months later. He has had all his other childhood immunizations. The most likely diagnosis in this case is A measles B roseola C recurrent chicken pox D zoster E molluscum contagiosum
The Correct Answer is: D Varicella zoster outbreak (shingles) most often occurs in an elderly patient years after experiencing chicken pox as a child or who have received the immunization. The virus lies dormant in the spinal nerve root and later manifests as a very painful, skin-sensitive vesicular rash in a dermatomal distribution. Unlike chicken pox, a postherpetic neuralgia pain can persist for years and is at times, debilitating. Although most common in the elderly, zoster can occur in children in immunocompromised states such as those with malignancy or human immunodeficiency virus infection. Molluscum contagiosum presents with flesh colored, umbilicated papules and is typically not painful. Measles and roseola are maculopapular rashes.
Q 27.6: An 18-year-old female with no significant past medical history presents with a gradual progression of fatigue and pallor over the last three months. Initial CBC results show a hemoglobin of 9.8 mg/dL, hematocrit of 30%, an MCV of 116 fL, and a reticulocyte count of 0.1%.Which of the following dietary practices is most likely to contribute to her anemia? A High protein B Low carbohydrate C Low fat D Vegan E Ovo-lacto vegetarian
The Correct Answer is: D Vitamin B 12 is derived from animal products and thus vegans are at risk for deficiency that results in macrocytic anemia. Patients who pursue a low carbohydrate (B), high protein (A) diet likely rely on animal sources of protein and/or fats to meet their caloric needs. Ovo-lacto vegetarians (E) obtain vitamin B 12 from eggs and dairy products.
Q 40.1: A 12-year-old presents with sharply demarcated depigmented macules on bilateral knees. The parents report that this began about one year ago, after the child fell and skinned his knees. After healing, he was left with these markings. What is the most likely diagnosis? A post inflammatory hypopigmentation B hypertrophic scar C pityriasis alba D vitiligo
The Correct Answer is: D Vitiligo is the most likely diagnosis. Vitiligo is an autoimmune disorder that affects the melanocytes. There is often a history of trauma that can precede an occurrence of vitiligo. The course for vitiligo is variable. Sometimes it will resolve spontaneously. Other times, it will continue to progress despite treatment. Post-inflammatory hypopigmentation are areas of lighter pigment, not complete depigmentation, which result from a resolved inflammatory process. The pigment will return over time. A hypertrophic scar is one which is enlarged but stays within the borders of the original injury. No pigment changes are associated with these scars. Pityriasis alba is a yeast infection of the skin, and usually appears on the face. It usually occurs in elementary school aged children, and is more apparent in late summer or early fall.
Q 59.4: Which of the following findings is consistent with a lower motor neuron deficit? A aphasia B dysdiadochokinesia C sensory loss D weakness E hyperreflexia
The Correct Answer is: D Weakness is one potential finding of a lower motor neuron process. Aphasia results from injury to the speech pathways within the brain. Sensory loss arises from many causes, but it is not a motor issue. Dysdiadochokinesia is consistent with cerebellar pathology. Hyperreflexia is typically a signal of upper motor neuron disease.
Q 98.21: A 88-year-old female is found lying on the ground in a supine position after sustaining a fall in her house from tripping on the rug.. Her leg is shortened, abducted and externally rotated, and she is complaining of pain to the right leg with radiation to the knee. Based on the history of the patient, what type of injury has this patient sustained? A Non-displaced pelvic fracture B Hip dislocation C Femoral shaft fracture D Femoral neck fracture E Distal femur fracture
The Correct Answer is: D When a fracture occurs in the femoral neck that includes displacement, the leg appears shortened, abducted and externally rotated. The tension that exists from muscles attached above and below the fracture site results in shortening of leg length after the fracture as the muscles contract. If there is no displacement of the bone, no leg length changes should occur. Generally, hip fractures would not result in a lengthening of the leg, so all answers that include this option would be wrong. When the leg appears shortened, flexed, adducted, and internally rotated, a posterior hip dislocation should be suspected. A leg that is flexed, abducted and externally rotated suggests an anterior hip dislocation. Posterior hip dislocations are far more common than anterior dislocations, accounting for more than 90% of all hip dislocations.
Q 86.5: You are examining a 65-year-old male who complains of partial vision loss in his right eye. Besides obtaining visual acuity, what is the most important initial part of the physical exam to perform in order to evaluate his condition? A Cover uncover test B Consensual papillary response C Extraocular movements D Visual fields by confrontation E Intraocular pressure
The Correct Answer is: D When a patient complains of vision loss, all of the choices are important parts of the eye examination; the visual field by confrontation exam is a screen to detect visual field defects. (LeBlond et al., 2009, Chapter 20)
Q 97.76: A 47-year-old female presents to the office complaining of excessive menstrual flow for the past two months. She complains of mid-cycle bleeding, as well as periods that last for nine days. What is the mostly likely cause of her bleeding? A Chlamydia trachomatis infection B Adenocarcinoma of the uterus C Uterine leiomyoma D Anovulatory uterine bleeding E Pregnancy
The Correct Answer is: D While all are causes for abnormal bleeding, anovulotory bleeding occurs in perimenopausal women due to the unopposed estrogen. STD and pregnancy would be more likely during reproductive years. Neoplasms accounts for approximately 20% of abnormal uterine bleeding in the post reproductive years.
Q 97.20: A 48-year-old woman presents complaining of vaginal fullness and difficulty passing stool. Upon exam she is found to have a stage 3 rectocele by the Baden Walker System. What is the most common cause for this? A Increasing age B Genetic disposition C Obesity D Pelvic floor injury E Constipation
The Correct Answer is: D While all of the answer choices are risk factors, the most common cause of pelvic organ prolapse remains pelvic floor injury, usually related to child birth or trauma.
Q 25.8: A 17-year-old female patient presents to your family practice office complaining of a persistent sore throat for the past 2 weeks duration. She is extremely fatigued, and she tells you that her boyfriend has had similar symptoms for about a week. Her vital signs are a blood pressure of 116/72 mm Hg, pulse 88 beats/min, respirations of 16 beats/min, and temperature of 101.1˚F taken orally. Her white blood cell count (WBC) reveals 14,800 mcL with more than 10% atypical lymphocytes. On physical examination you find significant yellowish-grey pharyngeal exudate, cervical lymphadenopathy as well as an enlarged spleen. Her rapid strep is negative. What should be your treatment of this patient? A Begin treatment with cephalexin. B Send out a back-up throat culture and begin treatment with cephalexin pending those results. C Send out a back-up throat culture and hold treatment pending those results. D Draw a Monospot test and treat the patient supportively pending the results, insuring that they avoid contact sports until a definitive diagnosis has been made. E Draw an Epstein-Barr virus (EBV) IgG level and treat the patient supportively if positive.
The Correct Answer is: D With this patient's presentation and a negative rapid strep test, other etiologies beyond group A beta-hemolytic strep should be seriously considered. In light of her symptoms including fatigue, her WBC, and that her significant other is having similar symptoms, infectious mononucleosis is highest on the differential diagnosis. A Monospot test would rule this possibility in or out. She should refrain from contact sports until her infection and any associated splenomegaly has resolved.
Q 44.11: A 12-year-old male exhibits prominent brown scaling on his neck, trunk, and extremities with involvement of flexural regions. The palms and soles are spared. What is the most likely diagnosis? A Epidermolysis bullosa simplex B Extensive keratosis pilaris C Lamellar ichthyosis D X-linked ichthyosis
The Correct Answer is: D X-linked ichthyosis is a recessive condition affecting males. The patient will have large scales that appear brown in color. This condition spares the palms and soles, and begins between the ages of two to six weeks.
Q 77.7: A 50-year-old woman presents with constipation and crampy abdominal pain for the past 3 months. She is also undergoing a divorce and has had a 15-lb weight loss in the past 3 months. You note mild tenderness to palpation in the left lower quadrant; no masses are noted. Rectal exam result is negative, but her stool tests positive for fecal occult blood. Which of the following is the most appropriate next step to evaluate her symptoms? A keep a food diary for the next 2 weeks B flexible sigmoidoscopy C increase dietary fiber and increase daily water intake D refer for psychologic evaluation to help with stress of her divorce E colonoscopy
The Correct Answer is: E Any symptomatic adult with a positive fecal occult blood test should undergo colonoscopy to rule out colorectal cancer. A flexible sigmoidoscopy will allow for only partial visualization of the colon. (McQuaid, 2009, p. 507) McQuaid KR. Gastrointestinal disorders. In: McPhee SJ , Papadakis MA, eds. Current Medical Diagnosis and Treatment. 48th ed. New York, NY: McGraw-Hill; 2009.
Q 77.9: A 13-year-old girl is brought to the office by her mother who is having difficulty managing the child's recent angry outbursts. Her grades in schools have gone from A's and B's to C's and D's; she has been sent to the office for "behavior problems" several times in the last two months despite having been an exemplary student in the past. The girl says she "feels nervous" all the time, "just can't sit still" in class, and "sweats like a pig" even when she isn't exercising. Her weight today is 10# less than at her previous well-child visit 6 months ago, but the child denies dieting and her mother reports that she eats "constantly." Vital signs include temperature of 99.0F, pulse of 96, respirations of 26. Her blood pressure is 148/82. Her skin is warm and moist. Which of the following is the most likely diagnosis? A amphetamine abuse B anorexia nervosa C bipolar disorder D bulimia nervosa E hyperthyroidism
The Correct Answer is: E Hyperthyroidism occurs more common in girls than in boys, most frequently in adolescence. Deterioration in school performance is a common finding, along with emotion lability, nervousness, personality changes, warm and moist skin, polyphagia, and weight loss. Systolic hypertension and a widened pulse pressure are often found. Amphetamine abuse (A) is certainly a possibility, but it tends to depress the appetite. Anorexia nervosa (B) is unlikely given the child's appetite. Bipolar disorder (C) in adolescents more typically presents with depressive symptoms, and does not explain the physical findings. Patients with bulimia nervosa (D) are typically normal or overweight. Zeitler PS, et al. Endocrine Disorders, in Current Diagnosis and Treatment, Pediatrics, 21 st ed. 2012.
A 24-year-old female HIV-positive patient, who is not currently on medication, presents to the emergency department with acute dyspnea, tachycardia, fever, nonproductive cough, and a room air oxygen saturation of 92%. She admits feeling poorly for the past five days. A physical exam reveals bilateral basilar crackles. An x-ray reveals Pneumocystis jiroveci pneumonia. The patient has no drug allergies. Which of the following is the first-line treatment of choice? A Amphotericin B B Clarithromycin C Clindamycin D Pentamidine E Trimethoprim-sulfamethoxazole
The Correct Answer is: E *Based upon current clinical data, the preferred agent for treatment of Pneumocystis jiroveci infections is oral trimethoprim-sulfamethoxazole.* Second-line medications may include single agent or combination therapy, utilizing clindamycin, primaquine, dapsone, pentamadine, and/or atovaquone. Clarithromycin is the agent of choice for Mycobacterium avium complex infection, and amphotericin B would be indicated for Cryptococcal meningitis, as well as other fungal infections.
Q 41.3: A 3-year-old male is brought to your office with a red, tearing right eye. The mother stated that the child was playing in another room with his 4-year-old brother. All she heard was the child beginning to cry. Upon physical exam, the child is intermittently crying, and his right eye is red and tearing. The child is continually rubbing the eye. The anterior chamber is clear and the pupil is equal and reactive. What is your next step in evaluating this patient? A Contact child protective services B Order a head CT scan C Perform a tonometry D Obtain a visual acuity E Perform a fluorescein stain
The Correct Answer is: E A corneal abrasion must be ruled out in a child with a red eye.
Q 22.8: A 36-year-old female presents with increasing fatigue, appears pale, and states that her hands and feet feel numb. She is a strict vegan and avoids all animal products. What diagnostic study would be most useful in diagnosing this patient? A Direct bilirubin B Folate C Thyroid stimulating hormone D Total iron binding capacity E Vitamin B12
The Correct Answer is: E A diagnosis of vitamin B12 deficiency is confirmed by an abnormally low vitamin B12 serum level. Early deficiencies are noted with a serum level of <170, and patients with symptoms usually have levels <100 pg/mL. Other abnormalities may include elevated serum lactate dehydrogenase and a modest increase in indirect bilirubin.
A 9-year-old female presents to the emergency department after falling out of a second story window. Imaging of the left upper extremity shows the following fracture pattern: How would this fracture be described? A Transverse B Oblique C Spiral D Segmental E Greenstick
The Correct Answer is: E A greenstick fracture occurs almost exclusively in young children as a result of their bones being more pliable than those of adults. On imaging, a greenstick fracture has cortical disruption on the convex side of the bone with an intact periosteum on the concave side of the fracture. Spiral fractures (C) are often seen in non-accidental or child abuse cases. Transverse (A), segmental (D), and oblique (B) fractures involve both cortices.
Q 39.16: A 4-year-old girl is brought to urgent care complaining of left ear fullness and difficulty hearing. The parents express concern that she put an elbow macaroni noodle into her ear. This concern is confirmed on otoscopic evaluation of the ear. Which of the following steps is most appropriate for removing this foreign body? A Administer general anesthesia B Anesthetize the ear canal with lidocaine solution C Irrigate the ear with warm water D Irrigate the ear with a hydrogen peroxide solution E Remove the noodle with a loop
The Correct Answer is: E A loop (E) or alternative instrument should be used to pull the noodle outward and away from the tympanic membrane. Organic material may swell when wet, and irrigation (C and D) or the insertion of anesthetic solution (B) is not recommended.
Q 49.15: A 3-week-old male infant is brought in by his mother due to his vomiting. The mother notes that a few days ago her son started vomiting after feeding, and it has become projectile in nature. The vomitus is non-bilious and contains no blood. The child seems hungry and nurses regularly, but the vomiting has become more frequent and is occurring with every feeding now. On physical examination, an oval mass is palpated in the right upper quadrant. Appropriate imaging is obtained and confirms the suspected diagnosis. What is the treatment of choice in this patient? A Acid supression B Dilatation of the lower esophageal sphincter C Diverting colostomy D Ladd procedure E Pyloromyotomy
The Correct Answer is: E A pyloromyotomy involves an incision along the length of the pylorus, down to the mucosa, and is the treatment of choice in pyloric stenosis. Acid suppression is the treatment of choice in cases of peptic ulcer disease. Dilatation of the LES is performed in cases of achalasia of the esophagus. A diverting colostomy may be used in cases of Hirschsprung disease, after removal of the aganglionic section of colon. The Ladd procedure is used in surgical treatment of intestinal malrotation.
Q 117.5: What type of disorder develops within 3 months of an identified stressor such as finances, going to school, divorce, or illness in their life. The stressor causes impairment in their job and relationships, but the symptoms resolve within 6 months. What is the most likely diagnosis? A depression B bereavement C post-traumatic stress disorder D personality disorder E adjustment disorder
The Correct Answer is: E A response to a stressor that disturbs the mood of the patient causes impairment in function. The symptoms occur within 3 months of the stressor and last no longer than 6 months. Anxiety, depression, or combination is associated with adjustment disorders. (Eisendrath and Lichtmacher, 2009, p. 912; Sadock and Sadock, 2008, p. 371) Eisendrath SJ , Lichtmacher JE. Psychiatric disorders. In: McPhee SJ , Papdakis MA, eds. Current Medical Diagnosis and Treatment, 48th ed. New York: McGraw-Hill; 2009. Sadock BJ , Sadock VA. Concise Textbook of Clinical Psychiatry, 3rd ed. Philadelphia, PA: Lippincott, Williams & Wilkins; 2008.
Q 97.71: A 35-year-old female presents to discuss non-permanent methods of birth control. She is a G3P2, PMH negative. She is a smoker. Which method of contraception would have the lowest risk profile for her? A Combination low dose pill daily B Injectable progestin monthly C Transdermal combination patch D Levonorgestrel intrauterine device E Copper intrauterine device
The Correct Answer is: E A women who is over 35 and smokes is at high risk for cardiovascular complications. All choices except the copper IUD contain hormones, which may increase the risk of complications. The copper IUD is long term but non-permanent.
Q 91.7: A 78-year-old male presents to the office due to increasing exertional dyspnea and cough for the past week. Physical exam reveals an S3 gallop, mild JVD, and 2+ pitting edema of the bilateral lower extremities. The patient has had mild congestive heart failure in the past. A chest x-ray reveals prominent pulmonary vasculature without any additional complications. The patient's current medications are aspirin and simvastatin daily. He has no drug allergies. Which of the following is the most appropriate initial management for this patient? A Angiotensin II receptor blocker B β blocker C Calcium channel blocker D Nitrate E Thiazide diuretic plus ACE inhibitor
The Correct Answer is: E ACE inhibitors are recommended, as first line treatment for symptomatic congestive heart failure, based upon clinical trials, reveal an approximately 20% reduction in CHF mortality in symptomatic heart failure patients. Diuretics provide CHF symptom improvement and promote water and sodium excretion to decrease intravascular volume. blockers and Angiotensin II receptor blockers are also beneficial for CHF patients. Nitrates are usually reserved for acute or decompensating patients. Calcium channel blockers may accelerate CHF progression and should be used with caution. (McPhee SJ, Papadakis MA. Current Medical Diagnosis & Treatment 2011, Chapter 10, Heart Disease)
Q 101.5: You are asked to see a diabetic patient with retinopathy and hypertension. On examination, the patient's blood pressure is noted to be 180/90 mm Hg. Urinalysis shows microalbumin of 300 mg/dL. Labs: blood urea nitrogen 22 mg/dL, creatinine 1.5 mg/dL. Which of the following classes of antihypertensive medications would be best to prescribe in this setting? A calcium channel blocker B loop diuretic C alpha blocker D thiazide diuretic E ACE inhibitor
The Correct Answer is: E ACE inhibitors are the drug of choice in this setting. Control of systemic blood pressure can reduce renal vascular damage. In diabetic patients, ACE inhibitors are especially beneficial because of the added effect of reducing intraglomerular pressure and decreasing proteinuria. Current target blood pressure in patients with diabetic nephropathy is <130/80 mm Hg. Calcium channel blockers and diuretics do not offer renoprotective benefits but may be used to control hypertension. (American Diabetes Association, 2009, p. S28; JNC 7 Report, 2003; NKF-K/DOQI Guidelines, 2002, pp. 79-80; Watnick and Morrison, 2009, p. 819) American Diabetes Association . Diabetes care . Diabetes Care. 2009;32:S13-S61. The seventh report of the Joint National Committee on Prevention, Detection, Evaluation, and Treatment of High Blood Pressure: the JNC 7 Report . JAMA. 2003;289:2560-2572. NKF-K/DOQI Clinical Practice Guidelines for Chronic Kidney Disease: Executive Summary. New York, NY: National Kidney Foundation; 2002. Watnick S , Morrison G. Kidney. In: Tierney LM , McPhee SJ , Papadakis MA, eds. Current Medical Diagnosis and Treatment. 48th ed. New York, NY: McGraw-Hill; 2009.
Q 97.74: During her active phase of labor, a provider has placed an external fetal monitor on a patient. The fetal heart rate (FHR) is noted to have a base line rate of 109, there are no late or variable decelerations, and the baseline variability is 0 beats per minute. What is this monitor tracing indicative of? A A normal FHR pattern B An indeterminate FHR pattern C An unreadable FHR pattern D Poor contact with monitor E A distress FHR pattern
The Correct Answer is: E According to the Three-Tier Fetal Heart Interpretation System, recommended by the 2008 NICHD workshop on electronic fetal monitoring, the definition of fetal distress includes absence of baseline variability, and either bradycardia (FHR<110) or recurrent variable or late decelerations.
Q 32.18: A 64-year-old female presents with acute onset of severe right knee pain. She denies any trauma and her history is only significant for diabetes mellitus. On physical examination, her knee has significant edema and erythema, with warmth on palpation. After an aspiration of the fluid the results show: Color: opaque yellow fluid WBC: 90,000/mcL PMN: 90% What is the most likely diagnosis? A Osteoarthritis B Knee dislocation C ACL injury D Fibromyalgia E Septic arthritis
The Correct Answer is: E Acute bacterial septic arthritis (E) is found most commonly in a weight-bearing joint with acute onset of inflammatory monoarticular arthritis. Joint effusions are usually large with the fluid being opaque and white blood cell counts commonly > 50,000/mcl with 90% or more polymorphonuclear cells. Compromised immunity, such as diabetes mellitus, increases the risk of septic arthritis. Osteoarthritis (A), knee dislocation (B), ACL injury (C), and fibromyalgia (D) would not show an increased WBC count with opaque yellow fluid and 90% PMNs.
Q 65.7: Which of the following lab results reflect acute anxiety neurosis hyperventilation? A Acidotic pH, PCO2 increased, HCO3 decreased B Acidotic pH, PCO2 decreased, HCO3 decreased C Acidotic pH, PCO2 increased, HCO3 normal D Alkalotic pH, PCO2 increased, HCO3 increased E Alkalotic pH, PCO2 decreased, HCO3 normal
The Correct Answer is: E Acute hyperventilation, associated with anxiety and other disorders, results in decreased PCO 2 and leads to acute respiratory alkalosis. Acute respiratory alkalosis is associated with an increased pH and a decreased PCO 2 . Chronic respiratory alkalosis will result in compensatory bicarbonate (HCO 3 ) level changes; in this scenario, the bicarbonate is expected to be normal initially. Respiratory alkalosis has other causes (as shown in Table 21-26). The other answers represent a variety of acid-base disorders including, mixed acid-base disorder (choice A), metabolic acidosis with compensation (choice B), uncompensated respiratory acidosis (choice C), and metabolic alkalosis with compensation (choice D). These disorders, and the respective expected compensation, are reviewed in Table 21-12. Table 21-16. Causes of respiratory alkalosis. Hypoxia Decreased inspired oxygen tension High altitude Ventilation/perfusion inequality Hypotension Severe anemia CNS-mediated disorders Voluntary hyperventilation Anxiety-hyperventilation syndrome Neurologic disease Cerebrovascular accident (infarction, hemorrhage) Infection Trauma Tumor Pharmacologic and hormonal stimulation Salicylates Nicotine Xanthines Pregnancy (progesterone) Hepatic failure Gram-negative septicemia Recovery from metabolic acidosis Heat exposure Pulmonary disease Interstitial lung disease Pneumonia Pulmonary embolism Pulmonary edema Mechanical overventilation (Current Medical Dx & Tx, Chapter 21, Electrolyte & Acid-Base Disorders, Acid-Base Disorders, Respiratory Alkalosis (Hypocapnia)) (McPhee SJ, Papadakis MA. Current Medical Diagnosis & Treatment 2011, Chapter 21, Electrolyte & Acid-Base Disorders)
Q 83.9: A 57-year-old male is being monitored for Binet Stage A CLL. He is emergently seen in the clinic with rapid lymph node enlargement, fever, weight loss, and hepatosplenomegaly. On laboratory examination, he is found to have an elevated serum lactate dehydrogenase and a monoclonal gammopathy on serum protein electrophoresis. A retroperitoneal ultrasound reveals bulky adenopathy. What is the most likely diagnosis? A Acute lymphadenitis secondary to HSV B Epstein-Barr viral infection C Hodgkin lymphoma D Rapidly advancing CLL E Richter transformation
The Correct Answer is: E All findings are classic for Richter transformation of CLL to an aggressive large B-cell, high-grade lymphoma. While HSV cannot be totally ruled out without a node biopsy, given the patient history and findings the most likely diagnosis is Richter transformation. Rapidly advancing CLL does not usually develop retroperitoneal adenopathy. Occasionally, Richter syndrome with Hodgkin lymphoma features is seen, but accounts for less than one-fifth of all cases of Richter transformation. EBV may play a role in this later syndrome. (Lichtman et al., Williams Hematology 8e, Chapter 94, Chronic Lymphocytic Leukemia and Related Diseases)
Q 2.4: A 5 year-old girl presents to the clinic with her mother who is concerned about her daughter's energy level since starting full day kindergarten. CBC reveals a hemoglobin of 12.3 g/dl, hematocrit of 36%, MCV of 62 fL, and an MCHC of 34 g/dL. Serum ferritin levels are normal. What is the most likely hemoglobin electrophoresis results for this patient? A Decreased HbA2 and increased HbF levels B Decreased HbA2 and HbF levels C Increased HbA2 and normal HbF levels D Increased HbA2 and HbF levels E Normal HbA2 and HbF levels
The Correct Answer is: E Alpha thalassemia presents with a normal hemoglobin electrophoresis, since all adult hemoglobin are alpha-containing and affected equally. Beta thalassemia major may present with increased HbA 2 and/or HbF levels (C and D). Beta thalassemia minor typically presents with increased HbA 2 (C).
Q 85.3: A patient that was diagnosed with severe hemophilia B in 1974 and requiring regular factor IX replacement therapy will have a 1:2 chance of also having which of the following? A Hemarthroses B Hemophilia A C Hepatitis B D Hepatitis C E HIV
The Correct Answer is: E Approximately 50% of older and severely affected patients with either hemophilia A or hemophilia B are now HIV positive, due to the pooled nature of therapeutic products prior to 1985. These patients are also at increased risk for Hepatitis B and C, but not at the same ratio. Hemarthroses is a complication of not treating factor IX bleeding. (Lichtman et al., Williams Hematology 8e, Chapter 124, Hemophilia A and Hemophilia B)
Q 70.3: Your patient is a 66-year-old female who has been dropping her coffee cup and concurrently slurring her speech. The episodes last for approximately 15 minutes. Her blood work, carotid dopplers, and MRI of the brain are normal and you suspect recurrent transient ischemic attacks (TIAs). Which of the following is NOT approved or recommended for the prevention of stroke in this patient? A aspirin B extended-release dipyridamole plus aspirin C clopidogrel D prasugrel E warfarin
The Correct Answer is: E Aspirin, aspirin plus extended-release dipyridamole, and clopidogrel are all antiplatelet agents and approved for use to reduce recurrent TIAs and ischemic cerebrovascular accidents (CVAs). Prasugrel is not FDA-approved for this indication. Warfarin is an anticoagulant and has no role in prevention of either recurrent TIA or ischemic CVA. (McPhee and Papadakis, 2011, Chapter 24)
Q 82.1: A 24-year-old female HIV-positive patient, who is not currently on medication, presents to the emergency department with acute dyspnea, tachycardia, fever, nonproductive cough, and a room air oxygen saturation of 92%. She admits feeling poorly for the past five days. A physical exam reveals bilateral basilar crackles. An x-ray reveals Pneumocystis jiroveci pneumonia. The patient has no drug allergies. Which of the following is the first-line treatment of choice? A Amphotericin B B Clarithromycin C Clindamycin D Pentamidine E Trimethoprim-sulfamethoxazole
The Correct Answer is: E Based upon current clinical data, the preferred agent for treatment of Pneumocystis jiroveci infections is oral trimethoprim-sulfamethoxazole. Second-line medications may include single agent or combination therapy, utilizing clindamycin, primaquine, dapsone, pentamadine, and/or atovaquone. Clarithromycin is the agent of choice for Mycobacterium avium complex infection, and amphotericin B would be indicated for Cryptococcal meningitis, as well as other fungal infections. (McPhee SJ, Papadakis MA. Current Medical Diagnosis & Treatment 2011, Chapter 36, Mycotic Infections)
Q 21.5: A fracture involving the medial epicondyle will most likely cause damage to which nerve? A Axillary B Median C Peroneal D Radial E Ulnar
The Correct Answer is: E Because the ulnar nerve passes through the cubital tunnel, which is a groove on the posterior aspect of the medial epicondyle, any fractures involving the medial epicondyle can also cause damage to the ulnar nerve. The median nerve is most susceptible to injury at the carpal tunnel. Branches of the radial nerve can become entrapped on the lateral side of the elbow and the associated symptoms are often confused with lateral epicondylitis. Radial nerve injuries are more commonly associated with humeral shaft fractures. The axillary nerve is significantly proximal to the medial epicondyle and the peroneal nerve is in the leg.
Q 98.46: A fracture involving the medial epicondyle will most likely cause damage to which nerve? A Axillary B Median C Peroneal D Radial E Ulnar
The Correct Answer is: E Because the ulnar nerve passes through the cubital tunnel, which is a groove on the posterior aspect of the medial epicondyle, any fractures involving the medial epicondyle can also cause damage to the ulnar nerve. The median nerve is most susceptible to injury at the carpal tunnel. Branches of the radial nerve can become entrapped on the lateral side of the elbow and the associated symptoms are often confused with lateral epicondylitis. Radial nerve injuries are more commonly associated with humeral shaft fractures. The axillary nerve is significantly proximal to the medial epicondyle and the peroneal nerve is in the leg.
Q 17.9: Mr. Addley is a 69-year-old, recently retired accountant who comes to you in your family practice, accompanied by his wife, for a routine follow-up of his blood pressure. During his visit he comments that he is concerned about his memory. He notes that he is having some difficulty remembering where he left his car keys, the day of the week, and names of people immediately after being introduced to them. When his wife is asked, she has noticed nothing regarding any change in his mental status or his ability to perform activities of daily living. You perform a mini-mental state exam (MMSE) and he scores 25 out of 30. What is the most appropriate working diagnosis for Mr. Addley? A mild to moderate depression B mild Alzheimer's disease C moderate Alzheimer's disease D Lewy body dementia E mild cognitive impairment
The Correct Answer is: E Because your patient notices symptoms but family members, friends, and coworkers do not, and his MMSE is slightly below normal, your patient meets the diagnostic criteria for Mild Cognitive Impairment, or MCI. Metabolic causes should be ruled out before making a definitive diagnosis. This may or may not progress to Alzheimer's and there are no FDA-approved medications for MCI, although some clinicians do utilize acetylcholinesterase-inhibitors after discussions with patients and their families.
Q 105.20: Which of the following is the primary mechanism by which benzodiazepines exert their sedative and anxiolytic effects? A acting as dopamine receptor agonists B acting as NMDA receptor antagonists C acting as serotonin receptor antagonists D decreasing reuptake of serotonin and norepinephrine E increasing GABAA receptor-mediated chloride conductance
The Correct Answer is: E Benzodiazepines bind to GABA A receptors, which consist of many peripheral subunits that form chloride channels at their core. GABA is one of the major inhibitory neurotransmitters in the brain; hence, benzodiazepines enhance this inhibitory influence to produce sedation and calm. (Kirkwood and Melton, 2008, p. 1167) Kirkwood CK , Melton ST. Anxiety disorders I: generalized anxiety, panic and social anxiety disorders. In: DiPiro JT , Talbert RL , Yee GC, et al., eds. Pharmacotherapy: A Pathophysiologic Approach. 7th ed. New York: McGraw-Hill; 2008.
Q 10.5: A 2 year-old girl presents to the clinic due to fever, and right ear pain. Physical exam reveals a temperature of 103.1 F and an erythematous and immobile right tympanic membrane. The patient is subsequently prescribed amoxicllin. What is the mechanism of amoxicillin? A Binds to the 30S ribosomal subunit to inhibit protein synthesis B Binds to the 50S ribosomal subunit to inhibit protein synthesis C Inhibits dihydropteroate synthase and folate production D Inhibits DNA replication by binding to DNA gyrase and topoisomerase IV E Inhibits the transpeptidation reaction
The Correct Answer is: E Beta lactam antibiotics inhibit the transpeptidation reaction leading to cell wall destruction (E). Tetracycline binds to the 30S ribosomal subunit to inhibit protein synthesis (A). Macrolide antibiotics bind to the 50S ribosomal subunit to inhibit protein synthesis (B). Sulfamethoxazole inhibits dihydropteroate synthase and folate production (C). Ciprofloxacin inhibits DNA replication by binding to DNA gyrase and topoisomerase IV (D).
Q 116.20: A 73-year-old male presents to clinic with a history of blood in his urine for the past month. He denies irritation with voiding or any other symptoms. He does state an unintentional weight loss of 20 pounds in the past 6 months. His past medical history includes hyperlipidemia, seasonal allergies, and fibromyalgia. He denies alcohol use and has a 22 pack-year history of tobacco use. On physical examination you note bilateral inguinal lymphadenopathy. Which of the following is the most likely diagnosis? A Benign prostate hypertrophy B Urinary tract infection C Prostatitis D Syphilis E Bladder carcinoma
The Correct Answer is: E Bladder cancer's presenting symptom is hematuria in 85-90% of patients (E). Clinical findings include masses detected on bimanual exam, lymphedema of the lower extremities, or palpable lymphadenopathy with advanced cancers. Urinary tract infections (B) and prostatitis (C) commonly cause discomfort with voiding. BPH (A) does not correlate with a 20-pound weight loss, smoking history, or lymphadenopathy. This history and physical examination do not show any signs of syphilis (D). Papadakis MA, McPhee SJ, "Bladder Cancer." Quick Medical Diagnosis & Treatment: http://www.accessmedicine.com/quickam.aspx
A 21-year-old female patient presents with a large area of swelling localized to the front of the knee, between the patella and the skin after a fall directly on her knee. She has been stating that her knee is becoming more difficult to move due to the swelling and pain. She is otherwise healthy with no other medical problems. Her exam reveals a tender, fluctuant area just anterior to the patella without warmth. She does not appear to have any effusion, and her ligament stability testing is normal. Based on these findings, what is the most likely diagnosis? A Anterior cruciate ligament tear B Medial meniscal tear C Pes anserine bursitis D Posterior cruciate ligament tear E Prepatellar bursitis
The Correct Answer is: E Bursae are synovial fluid filled sacs that facilitate the reduction of friction between adjacent structures. They may be found between the skin and various bony prominences or between tendons, ligaments and bone. They can become injured by an acute direct impact or gradual repetitive stress such as what might occur if someone was required to do extensive kneeling on the job. They may also be sites of infection with staphylococcus aureus and streptococcus species being the most common pathogens. The knee contains two primary bursae. The prepatellar bursa lies directly below the skin and above the patella. It is often acutely injured with a fall or other traumatic impact. A localized swelling can occur quickly and this fluid accumulation is not intracapsular as occurs with a joint effusion after an anterior cruciate ligament tear, meniscal tear or posterior cruciate tear. Meniscal injuries tend to produce a smaller effusion than cruciate ligament tears. The mechanism of falling on a bent knee can be consistent with a posterior cruciate ligament injury, but given the lack of an effusion and the presence of a well contained anterior knee swelling, this becomes less likely. Pes anserine bursa are located on the anterior medial aspect of the tibia, just below the tibial plateau. This location is the insertion of the gracilis, sartorius, and semitendinosus muscles. Pes anserine bursitis is commonly associated with early osteoarthritis in the medial compartment of the knee.
Q 22.6: A 21-year-old female patient presents with a large area of swelling localized to the front of the knee, between the patella and the skin after a fall directly on her knee. She has been stating that her knee is becoming more difficult to move due to the swelling and pain. She is otherwise healthy with no other medical problems. Her exam reveals a tender, fluctuant area just anterior to the patella without warmth. She does not appear to have any effusion, and her ligament stability testing is normal. Based on these findings, what is the most likely diagnosis? A Anterior cruciate ligament tear B Medial meniscal tear C Pes anserine bursitis D Posterior cruciate ligament tear E Prepatellar bursitis
The Correct Answer is: E Bursae are synovial fluid filled sacs that facilitate the reduction of friction between adjacent structures. They may be found between the skin and various bony prominences or between tendons, ligaments and bone. They can become injured by an acute direct impact or gradual repetitive stress such as what might occur if someone was required to do extensive kneeling on the job. They may also be sites of infection with staphylococcus aureus and streptococcus species being the most common pathogens. The knee contains two primary bursae. The prepatellar bursa lies directly below the skin and above the patella. It is often acutely injured with a fall or other traumatic impact. A localized swelling can occur quickly and this fluid accumulation is not intracapsular as occurs with a joint effusion after an anterior cruciate ligament tear, meniscal tear or posterior cruciate tear. Meniscal injuries tend to produce a smaller effusion than cruciate ligament tears. The mechanism of falling on a bent knee can be consistent with a posterior cruciate ligament injury, but given the lack of an effusion and the presence of a well contained anterior knee swelling, this becomes less likely. Pes anserine bursa are located on the anterior medial aspect of the tibia, just below the tibial plateau. This location is the insertion of the gracilis, sartorius, and semitendinosus muscles. Pes anserine bursitis is commonly associated with early osteoarthritis in the medial compartment of the knee.
A 33-year-old IV drug user presents to the emergency department with chills, diaphoresis, anorexia, and malaise. On physical exam, her temperature is 40C, BP 98/55, P 115 bpm, and RR 22. Two separate blood cultures are positive for S.aureus. Which of the following diagnostic studies would be most useful in establishing this patient's diagnosis? A EKG B CXR C Rheumatoid factor D ESR E TEE
The Correct Answer is: E Choice E, TEE (or transesophageal echocardiogram), would be most useful in establishing a diagnosis of infective endocarditis, as a positive echocardiogram demonstrating presence of a vegetation would satisfy one of the Duke criteria's major criteria. TEE is more sensitive than TTE (transthoracic echocardiogram) for detecting vegetations. EKG and CXR should be performed as part of this patient's evaluation, but would be less useful than TEE in establishing a diagnosis of infective endocarditis. ESR and rheumatoid factor are frequently elevated in patients with endocarditis, but are not specific to the diagnosis of endocarditis.
Q 72.6: A 70-year-old man, with a history of HTN and aortic valve replacement 3 months ago, presents with complaints of arthralgia, myalgia, anorexia, fatigue, and weight loss over the last month, with recent dyspnea on exertion and lower extremity edema. Vital signs are as follows: Temperature 38°C, BP 102/64, P 98, RR 20. On physical exam, a new high-pitched, blowing, decrescendo diastolic murmur is noted along the left lower sternal border. Two separate blood cultures are positive for S. aureus. Which of the following is the most appropriate next diagnostic study? A EKG B CXR C TTE D ESR E TEE
The Correct Answer is: E Choice E, TEE or transesophageal echocardiogram, would be most useful in establishing a diagnosis of infective endocarditis, as a positive echocardiogram demonstrating presence of a vegetation would satisfy one of the Duke criteria's major criteria, as well as determine the extent of the prosthetic valvular dysfunction, if present. TEE is more sensitive than TTE, transthoracic echocardiogram, for detecting vegetations, so choice C is incorrect. Choices A and B, EKG and CXR, should be performed as part of this patient's evaluation, but would be less useful than TEE in establishing a diagnosis of infective endocarditis. Choice D, erythrocyte sedimentation rates, are frequently elevated in patients with endocarditis, but are not specific to the diagnosis of endocarditis. (Fauci et al., 2001, pp. 809-811)
Q 65.6: A 24-year-old man with a recent history of a viral illness comes to the emergency department complaining of severe left-sided chest discomfort, which radiates through to the left trapezius region. On coming into the room, you note that he is sitting up and hunched forward. On physical examination, the patient's temperature is 39°C, blood pressure is 135/78, with a pulse of 85 bpm, and a pericardial friction rub is noted. Laboratory findings demonstrate elevated serum creatine kinase levels and normal serial troponin levels. His EKG demonstrates peaked T waves. His CXR demonstrates no acute process. Which of the following is the most appropriate treatment for this patient? A Morphine B Enoxaparin C Nitroglycerin D Penicillin V E Indomethacin
The Correct Answer is: E Choice E, indomethacin 25-75 mg QID, and bed rest would be the most appropriate treatment in a patient with acute viral pericarditis, as a nonsteroidal anti-inflammatory agent will ameliorate the inflammatory process. Choices A and C are appropriate in a patient suspected of acute coronary syndrome. Choice B, enoxaparin, is contraindicated in patients with pericarditis, as anticoagulants could lead to worsening of pericardial effusion and cardiac tamponade, especially if it is secondary to bleeding into the pericardial space, such as with trauma or postoperatively. (Fauci et al., 2001, p. 1369)
Q 119.18: A 57-year-old woman with a history of rheumatic fever is seen complaining of dyspnea while vacuuming her apartment, which has been worsening over the last few months. On physical exam, a possible opening snap, loud S 1 , and a very soft diastolic rumbling murmur is auscultated. When the patient is placed in the left lateral decubitus position, the murmur is accentuated, and heard best at the apex. With inspiration, the murmur does not increase in amplitude. Which of the following is the most likely finding on echocardiogram? A Tricuspid regurgitation B Tricuspid stenosis C Atrial septal defect D Aortic regurgitation/insufficiency The Correct Answer is: E Choice E, mitral stenosis, is the most likely finding in a patient with a history of rheumatic fever presenting with a possible opening snap, loud S 1 , and a very soft diastolic rumbling murmur which is heard best at the cardiac apex and accentuated by placing the patient in the left lateral decubitus position. Choices A and B, tricuspid regurgitation and tricuspid stenosis, are also linked with patients with rheumatic heart disease. The murmur of tricuspid regurgitation, however, is a systolic murmur, which increases with inspiration and is heard best at the left lower sternal border. Tricuspid stenosis presents with a diastolic murmur, and with inspiration, the murmur increases. It is also heard best at the left lower sternal border. Choice C, an atrial septal defect, if large, could present with similar symptoms of exertional dyspnea secondary to a large shunt, but auscultation would reveal a moderately loud systolic ejection murmur heard best in the second and third interspaces. This is secondary to increased pulmonary arterial flow. Choice D, aortic regurgitation/insufficiency, is also a diastolic murmur; however, it is usually a diastolic decrescendo murmur heard best at the left sternal border. (LeBlond et al., 2009, Chapter 8) E Mitral stenosis
The Correct Answer is: E Choice E, mitral stenosis, is the most likely finding in a patient with a history of rheumatic fever presenting with a possible opening snap, loud S 1 , and a very soft diastolic rumbling murmur which is heard best at the cardiac apex and accentuated by placing the patient in the left lateral decubitus position. Choices A and B, tricuspid regurgitation and tricuspid stenosis, are also linked with patients with rheumatic heart disease. The murmur of tricuspid regurgitation, however, is a systolic murmur, which increases with inspiration and is heard best at the left lower sternal border. Tricuspid stenosis presents with a diastolic murmur, and with inspiration, the murmur increases. It is also heard best at the left lower sternal border. Choice C, an atrial septal defect, if large, could present with similar symptoms of exertional dyspnea secondary to a large shunt, but auscultation would reveal a moderately loud systolic ejection murmur heard best in the second and third interspaces. This is secondary to increased pulmonary arterial flow. Choice D, aortic regurgitation/insufficiency, is also a diastolic murmur; however, it is usually a diastolic decrescendo murmur heard best at the left sternal border. (LeBlond et al., 2009, Chapter 8)
Q 117.1: A 22-year-old recent immigrant from Vietnam, who is 28 weeks pregnant with her first child, presents to the emergency department with complaints of worsening dyspnea and lower extremity edema. She is unable to answer definitively whether or not she has a history of rheumatic fever. On physical examination, a possible opening snap, loud S 1 , and a very soft diastolic rumbling murmur is auscultated. When the patient is placed in the left lateral decubitus position, the murmur is accentuated, and heard best at the apex. With inspiration, the murmur does not increase in amplitude. Which of the following is the most likely finding on echocardiogram? A Tricuspid regurgitation B Tricuspid stenosis C Atrial septal defect D Aortic regurgitation/insufficiency E Mitral stenosis
The Correct Answer is: E Choice E, mitral stenosis, is the most likely finding in this patient, who presents with physical exam findings including a possible opening snap, loud S 1 , and a very soft diastolic rumbling murmur which is heard best at the cardiac apex and accentuated by placing the patient in the left lateral decubitus position. Although rheumatic fever was not positively confirmed, the patient likely did have a history, given that the majority of cases of mitral stenosis are secondary to rheumatic heart disease. Patients from Asia, Central America, and South America are exposed more frequently than their counterparts in more developed countries, where antibiotic use is more common. Choices A and B, tricuspid regurgitation and tricuspid stenosis, are also linked with patients with rheumatic heart disease. The murmur of tricuspid regurgitation, however, is a systolic murmur, which increases with inspiration and is heard best at the left lower sternal border. Tricuspid stenosis presents with a diastolic murmur, and with inspiration the murmur increases. It, too, is heard best at the left lower sternal border. Choice C, an atrial septal defect, if large, could present with similar symptoms of exertional dyspnea secondary to a large shunt, but auscultation would reveal a moderately loud systolic ejection murmur that is heard best in the second and third interspaces. This is secondary to increased pulmonary arterial flow. Choice D, aortic regurgitation/insufficiency, is also a diastolic murmur; however, it is usually a diastolic decrescendo murmur that is heard best at the left sternal border. (Fauci et al., 2008, Chapter 7; LeBlond et al., 2009, Chapter 8)
Q 69.9: A 67-year-old woman with a history of gastric ulcers is admitted with complaints of recent onset of dyspnea with exertion, 3 pillow orthopnea, lower extremity edema, and palpitations, in which she describes her heart as racing. Which of the following is likely to be the cause of her high-output congestive heart failure? A Mitral regurgitation B Aortic stenosis C Uncontrolled hypertension D Ruptured chordae tendinae E Severe anemia
The Correct Answer is: E Choice E, severe anemia, may result with progression of gastric ulcers, and is the only high-output cause of congestive heart failure among the choices offered. Other causes include severe anemia, thyrotoxicosis, and arteriovenouis shunting (for example, in hemodialysis patients). Choice A, mitral regurgitation, is a cause of excessive preload leading to heart failure. Choice D, ruptured chordate tendinae, associated with mitral regurgitation, would also be a cause of excessive preload leading to heart failure. Choices B and C, aortic stenosis and uncontrolled hypertension, are causes in which too much afterload leads to heart failure. (McPhee et al., 2011, Chapter 10)
Q 98.62: A 70-year-old man with a history of hypertension, DM Type 2, and hyperlipidemia is seen for preoperative evaluation prior to left total knee replacement. On auscultation, a very soft high-frequency decrescendo early diastolic murmur is heard at the upper left sternal border. Utilizing isometric hand grip exercises, the murmur increases in intensity and can be heard radiating to the left sternal border and apex. Given the patient's physical exam findings, which of the following is the most appropriate next step in the management of this patient? A Chest x-ray B Transesophageal echocardiogram C Holter monitor D Treadmill exercise stress test E Transthoracic echocardiogram
The Correct Answer is: E Choice E, transthoracic echocardiogram, is a simple, sensitive and non-invasive diagnostic tool which can evaluate for the presence of valvulopathy in this patient. Choice A might be able to give evidence of cardiomegaly, but would not be sensitive enough to detect valvulopathy. Choice B, transesophageal echocardiogram, would give information regarding valvulopathy, but is a more invasive test; therefore, choice E is more appropriate. Choice C is a useful diagnostic tool for evaluation of patients complaining of palpitations, but incorrect for this patient who has no symptoms. Choice D, although a useful diagnostic tool for the evaluation of exercise tolerance and in patients complaining of chest pain, does not allow direct visualization of the heart valves to evaluate for valvulopathy. In addition, a patient scheduled for left total knee replacement is unlikely to perform very well on a treadmill, thus the more appropriate test for preoperative evaluation, if necessary after transthoracic echocardiogram, would be a nuclear stress test.
Q 108.9: A 70-year-old man with a history of hypertension, DM Type 2, and hyperlipidemia is seen for preoperative evaluation prior to left total knee replacement. On auscultation, a very soft high-frequency decrescendo early diastolic murmur is heard at the upper left sternal border. Utilizing isometric hand grip exercises, the murmur increases in intensity and can be heard radiating to the left sternal border and apex. Given the patient's physical exam findings, which of the following is the most appropriate next step in the management of this patient? A Chest x-ray B Transesophageal echocardiogram C Holter monitor D Treadmill exercise stress test E Transthoracic echocardiogram
The Correct Answer is: E Choice E, transthoracic echocardiogram, is a simple, sensitive and non-invasive diagnostic tool which can evaluate for the presence of valvulopathy in this patient. Choice A might be able to give evidence of cardiomegaly, but would not be sensitive enough to detect valvulopathy. Choice B, transesophageal echocardiogram, would give information regarding valvulopathy, but is a more invasive test; therefore, choice E is more appropriate. Choice C is a useful diagnostic tool for evaluation of patients complaining of palpitations, but incorrect for this patient who has no symptoms. Choice D, although a useful diagnostic tool for the evaluation of exercise tolerance and in patients complaining of chest pain, does not allow direct visualization of the heart valves to evaluate for valvulopathy. In addition, a patient scheduled for left total knee replacement is unlikely to perform very well on a treadmill, thus the more appropriate test for preoperative evaluation, if necessary after transthoracic echocardiogram, would be a nuclear stress test. (Crawford et al., 2009, Chapter 1)
Q 19.2: A 57-year-old woman with a history of rheumatic fever is seen complaining of dyspnea while vacuuming her apartment, which has been worsening over the last few months. On physical exam, a possible opening snap, loud S 1 , and a very soft diastolic rumbling murmur is auscultated. When the patient is placed in the left lateral decubitus position, the murmur is accentuated, and heard best at the apex. With inspiration, the murmur does not increase in amplitude. Which of the following is the most appropriate next diagnostic study? A Chest x-ray B Transesophageal echocardiogram C Holter monitor D Treadmill exercise stress test E Transthoracic echocardiogram
The Correct Answer is: E Choice E, transthoracic echocardiogram, is a simple, sensitive and non-invasive diagnostic tool, which can evaluate for the presence of valvulopathy in the setting of a patient with a diastolic murmur and a history of rheumatic fever. Choice B is also useful for an evaluation for the presence of valvulopathy, but is more invasive than choice E; therefore, choice E is the more appropriate study. Choice A might be able to give evidence of cardiomegaly, but would not be sensitive enough to detect valvulopathy. Choice C is a useful diagnostic tool for the evaluation of patients complaining of palpitations, but incorrect for this patient who has no symptoms. Choice D, although a useful diagnostic tool for the evaluation of exercise tolerance and in patients complaining of chest pain, does not allow direct visualization of the heart valves to evaluate for valvulopathy.
Q 49.2: A 16-year-old boy is seen for a sports physical prior to starting football. He denies any symptoms. His physical examination is normal, except for a grade II/VI holosystolic murmur auscultated at the cardiac apex. Utilizing isometric hand grip exercises, the murmur increases in intensity and can be heard radiating to the axilla. With the Valsalva maneuver, the murmur decreases in intensity. Given the patient's physical exam findings, which of the following is the most appropriate next diagnostic study? A Chest x-ray B Transesophageal echocardiogram C Holter monitor D Treadmill exercise stress test E Transthoracic echocardiogram
The Correct Answer is: E Choice E, transthoracic echocardiogram, is a simple, sensitive, and non-invasive diagnostic tool that can evaluate for the presence of valvulopathy or congenital heart disease in this young patient. Choice A might be able to give evidence of cardiomegaly, but would not be sensitive enough to detect valvulopathy. Choice C is a useful diagnostic tool for evaluation of patients complaining of palpitations, but incorrect for this patient who has no symptoms. Choice D, although a useful diagnostic tool for the evaluation of exercise tolerance and in patients complaining of chest pain, does not allow direct visualization of the heart valves to evaluate for valvulopathy.
Q 114.5: A 22-year-old woman with a history of scoliosis presents to the office with complaints of a retrosternal chest discomfort, occurring frequently at rest and lasting for several hours at a time. She is currently experiencing this chest discomfort during the office visit. On physical exam, a mid-systolic click is noted. With standing, the click moves closer to S1. An EKG demonstrates normal sinus rhythm at 76 bpm, with no acute ST or T wave changes. Which of the following diagnostic studies would be the most appropriate next step given this patient's physical exam findings? A Chest x-ray B Transesophageal echocardiogram C Holter monitor D Treadmill exercise stress test E Transthoracic echocardiogram
The Correct Answer is: E Choice E, transthoracic echocardiogram, is a simple, sensitive, and non-invasive diagnostic tool, which can evaluate for the presence of valvulopathy or congenital heart disease in this young patient. Choice A might be able to give evidence of cardiomegaly, but would not be sensitive enough to detect valvulopathy. Choice C is a useful diagnostic tool for evaluation of patients complaining of palpitations, but incorrect for this patient who has no symptoms of palpitations. Choice D, although a useful diagnostic tool for the evaluation of exercise tolerance and in patients complaining of chest pain, does not allow direct visualization of the heart valves to evaluate for valvulopathy. Given her relative youth and lack of other cardiac risk factors, transthoracic echocardiogram should be performed before stress testing. (Fauci et al., 2008, Chapter 230)
Q 121.10: A 57-year-old man with a history of HTN, hyperlipidemia, and chronic tobacco use presents to the office with complaints of worsening chest tightness over the last 2 months. He initially noticed that every time he raked leaves he had a few minutes of chest tightness, which was relieved within 5 minutes if he rested. He now notices that raking will precipitate severe chest discomfort, diaphoresis, and dyspnea, which lasts for 20 minutes even if he rests. Last night, while watching football, he again noticed chest tightness, which began suddenly and slowly dissipated over 15 minutes. His physical examination is normal. Which of the following is the most likely diagnosis? A Pericarditis B Non-ST-segment myocardial infarction C Stable angina pectoris D Prinzmetal angina E Unstable angina pectoris
The Correct Answer is: E Choice E, unstable angina pectoris, is based on clinical presentation, and requires chest or arm discomfort or an anginal equivalent, that either occurs at rest or with minimal activity lasting for at least 10 minutes, recent onset of severe chest discomfort, or anginal equivalent, and/or chest discomfort or anginal equivalent that has progressively been increasing in either severity, frequency, or duration. Choice A, pericarditis, would present with chest discomfort that is worse while supine and improves while sitting forward, as well as a pericardial friction rub. Choice B, acute myocardial infarction, requires troponin elevation to establish the diagnosis. Choice C, stable angina pectoris, is chest or arm discomfort that is reliably precipitated by activity and/or emotional distress, and relieved with rest or sublingual nitroglycerin. Choice D, prinzmetal angina, or variant angina pectoris, is defined as a coronary artery spasm associated with ST-segment elevation, usually occurring at rest and frequently at the same time of the day. (Fauci et al., 2008, Chapter 238)
Q 40.3: A 3-year-old girl presents to the otolaryngologist for evaluation of a persistent left ear infection and drainage that have failed to respond to multiple antibiotic regimens. Which of the following is the most likely causative organism for this patient's condition? A Aspergillus B Chlamydia pneumoniae C E. coli D Streptococcus pneumoniae E Staphylococcus aureus
The Correct Answer is: E Chronic otitis media is typically caused by P. aeruginosa, H. influenzae, S. aureus (D), Proteus species, Klebsiella pneumoniae, or Moraxella catarrhalis. Aspergillus (A) and E. coli (C) are associated with otitis externa and streptococcus pneumoniae (D) is the most common bacterial cause of otitis media.
Q 120.5: You are asked to see a patient who was admitted to the hospital. Upon attempts to obtain a history, you notice the patient states words that sound similar, but do not have the same meaning. He also does some rhyming of his words. What type of thought process would this be? A flight of ideas B circumstantiality C looseness of association D word salad E clanging
The Correct Answer is: E Clanging is a disturbance in thought in which the person selects words that are similar by sound, but do not mean the same. Sometimes the person will rhyme the words. Flight of ideas is rapid transitioning between subjects, but tends to be connected. Looseness of association is when a person changes subjects, but there is no connection between the subjects. Circumstantiality is where the person has a point and eventually gets to that point, but with delay in the thought process. Word salad is a mixture of words that have no sense. (Sadock and Sadock, 2008, pp. 23-32) Sadock BJ , Sadock VA. Concise Textbook of Clinical Psychiatry, 3rd ed. Philadelphia, PA: Lippincott, Williams & Wilkins; 2008.
Q 88.5: A 33-year-old female with history of non-insulin dependent diabetes mellitus (NIDDM), hypertension, and obesity comes to see you for treatment of her significantly elevated cholesterol. She intends to have children in the next year and has been off birth control. Which of the following medications is considered to be safest for treatment of hyperlipidemia in pregnancy? A lovastatin B ezetimibe C rosuvastatin D fenofibrate E colesevelam
The Correct Answer is: E Colesevelam (WelChol) is considered a Category B pregnancy risk. Both lovastatin and rosuvastatin (statins) are considered Pregnancy Category X and should be avoided in pregnancy. Ezetimibea and fenofibrate are both Pregnancy Category C. (McPhee and Papadakis, 2011, Chapter 28)
What is the most common radiologic finding in a patient with a tension pneumothorax? A Pleural effusion B Infiltrate C Enlarged cardiac silhouette D Elevated diaphragm E Mediastinal shift
The Correct Answer is: E Collapse of the lung, followed by an increase in intrapleural pressure, will lead to a tension pneumothorax. In most cases, the air leaks through the subcutaneous spaces. But in a tension pneumothorax, the air is locked in and creates a life-threatening emergency.
Q 2.1: A 24-year-old male patient will be undergoing cisplatin-based chemotherapy for treatment of his germ cell cancer. Which of the following antiemetic regimens is most effective in this situation? A Dexamethasone B Dronabinol (Marinol®) C Palonosetron (Aloxi®) D Prochlorperazine E Dexamethasone and palonosetron
The Correct Answer is: E Combining 5-HT3 antagonist with glucocorticoids is the most effective therapy for prevention of acute chemotherapy-induced nausea and vomiting (CINV). Dronabinol (B) is ineffective in the treatment of CINV, while prochlorperazine (D) plays a limited role in pregnant patients.
Q 113.6: A 52-year-old man recently underwent surgery for a hip replacement. Upon discharge, he is prescribed oxycodone 7.5 mg and acetaminophen 325 mg and told to take 1 to 2 tablets every 6 hours prn to help manage the pain he is expected to encounter as he recovers at home. Which of the following medications would you also recommend for the patient to help minimize potential side effects associated with his pain medication? A esomeprazole B diphenhydramine C guaifenesin D hydrocortisone 1% cream E senna & docusate
The Correct Answer is: E Constipation is a common adverse effect of opioid therapy. In order to minimize or prevent constipation, the use of stool softeners (docusate) and a stimulant laxative (senna) can be initiated when opioid therapy is begun. (Rabow and Pantilat, 2008, pp. 69, 78) Rabow MW , Pantilat SZ. Palliative care & pain management. In: Tierney LM Jr , McPhee SJ , Papadakis MA, eds. Current Medical Diagnosis & Treatment. 47th ed. New York: McGraw-Hill; 2008.
Q 105.5: Which of the following medications, if started within five days of symptoms, has been shown to increase the rate of full recovery in an acute Bell's palsy patient? A Acyclovir B Aspirin C Midazolam D Nonsteroidal anti-inflammatory E Oral corticosteroid
The Correct Answer is: E Corticosteroids, used for their systemic anti-inflammatory and immune properties, have been shown to increase the proportion of Bell's palsy patients who recover completely, particularly over time. These should be considered as beneficial for the clinical outcome of the patient. Acyclovir, an antiviral medication, has not shown any additional benefit. Nonsteroidal anti-inflammatory medications may aid patient comfort, but have not been shown to impact outcome. Midazolam is not considered a first-line therapy for patients with Bell's palsy. (McPhee SJ, Papadakis MA. Current Medical Diagnosis & Treatment 2011, Chapter 24, Nervous System Disorders)
Q 98.41: During a pre-surgical workup, a patient reports that she had received Desmopressin following the birth of her son. Her CBC is within normal range, as well as her PT and aPTT. What is the most likely cause of her needing this medication? A Allo immunization following vaginal delivery B Hemolytic anemia C Iron deficiency anemia D Thalassemia E Von Willebrand's disease
The Correct Answer is: E Desmopressin is the mainstay of therapy for people with von Willebrand's disease. Hemolytic anemia, iron deficiency anemia, and thalassemia would all have abnormal findings on the CBC. Allo immunization does not require desmopressin therapy.
Q 65.9: During a pre-surgical workup, a patient reports that she had received Desmopressin following the birth of her son. Her CBC is within normal range, as well as her PT and aPTT. What is the most likely cause of her needing this medication? A Allo immunization following vaginal delivery B Hemolytic anemia C Iron deficiency anemia D Thalassemia E Von Willebrand's disease
The Correct Answer is: E Desmopressin is the mainstay of therapy for people with von Willebrand's disease. Hemolytic anemia, iron deficiency anemia, and thalassemia would all have abnormal findings on the CBC. Allo immunization does not require desmopressin therapy. (Lichtman et al., Williams Hematology 8e, Chapter 127, von Willebrand Disease)
A 46-year-old female with a history of poorly-controlled diabetes and grand mal seizures presents for evaluation of bilateral foot pain. She describes the pain as burning and has noticed it is worse at night. She occasionally has this pain in her feet. She denies other medical conditions, and her medications are metformin and dilantin. Which of the following medications, used for the suspected diagnosis, should be avoided in this patient? A Capsaicin B Desipramine C Gabapentin D Lidocaine patch E Tramadol
The Correct Answer is: E Diabetic peripheral neuropathy is the diagnosis, and describes any neuropathy in the diabetic patient. This patient is exhibiting a distal symmetric polyneuropathy with the classic associated symptoms, commonly called "pins and needles" by patients. When associated with pain and functional impact, pharmacologic therapy is warranted. There are many agents to choose from, with each of the answer choices being options. *However, in a patient with a known seizure disorder, tramadol should be avoided, as it decreases the seizure threshold.* Gabapentin, also a seizure medication, may be used, but close monitoring is suggested.
Q 83.1: A 46-year-old female with a history of poorly-controlled diabetes and grand mal seizures presents for evaluation of bilateral foot pain. She describes the pain as burning and has noticed it is worse at night. She occasionally has this pain in her feet. She denies other medical conditions, and her medications are metformin and dilantin. Which of the following medications, used for the suspected diagnosis, should be avoided in this patient? A Capsaicin B Desipramine C Gabapentin D Lidocaine patch E Tramadol
The Correct Answer is: E Diabetic peripheral neuropathy is the diagnosis, and describes any neuropathy in the diabetic patient. This patient is exhibiting a distal symmetric polyneuropathy with the classic associated symptoms, commonly called "pins and needles" by patients. When associated with pain and functional impact, pharmacologic therapy is warranted. There are many agents to choose from, with each of the answer choices being options. However, in a patient with a known seizure disorder, tramadol should be avoided, as it decreases the seizure threshold. Gabapentin, also a seizure medication, may be used, but close monitoring is suggested. (Tintinalli et al., Tintinalli's Emergency Medicine: A Comprehensive Study Guide, 7e, Chapter 166, Acute Peripheral Neurologic Lesions)
Q 71.1: What class of medications are the antibiotics of choice for adults with acute pertussis? A Cephalosporins B Penicillins C Fluroquinolones D Aminoglycosides E Macrolides
The Correct Answer is: E Erythromycin, azithromycin, and clarythromycin are all acceptable choices for the acute treatment of pertussis. Trimethoprim-sulfamethoxazole is an acceptable alternative to the macrolides for allergy or macrolide intolerant patients. (Schwartz BS. Current Medical Diagnosis and Treatment, 2011, Chapter 33, Bacterial and Chlymdial Infections)
Q 2.2: A 32 year-old obese woman presents to the clinic complaining of swelling, warmth, and redness of her left lower extremity. Doppler ultrasound confirms a deep venous thrombosis is the greater saphenous vein. Which of the following best describes the most likely pathophysiology underlying her thrombophilia? A Antithrombin deficiency B Increased plasma prothrombin C Protein C deficiency D Protein S deficiency E Resistance to activated protein C
The Correct Answer is: E Factor V Leiden is a common genetic mutation that results in resistance to activated protein C. Increased plasma prothrombin concentration results from a prothrombin gene mutation (B), while deficiencies of antithrombin (A), protein C (C) and protein S (D) are rarer causes of prothrombotic states and are not directly associated with factor V Leiden mutation which is the most common cause of thrombophilia in this clinical scenario
Q 7.6: Approximately what percentage of patients with a solid primary tumor elsewhere will end up with metastatic disease of the vertebrae during the clinical course of their cancer? A 10% B 20% C 30% D 40% E 50%
The Correct Answer is: E Fifty percent of cancer patients will develop metastatic disease of the vertebrae at some point during the course of their illness. The highest percentages of cancers that lead to such spinal lesions are carcinomas of the breast, lung, prostate, colon, thyroid and kidney. This likely occurs through hematogenous spread. Pain is a common presenting symptom, but it may be found while still asymptomatic if routine screenings are done with bone scans, MRI or CT in patients with a known primary tumor elsewhere in the body. The pain is usually worse with weight bearing activities and better when lying down, but pain that persists through the night and prevents sleep needs to be evaluated for possible neoplasm. Some metastatic disease of the spine may present with neurological symptoms such as sensory or motor deficits following a spinal nerve root distribution or more generalized neurological deficits due to spinal cord compression or cauda equina syndrome.
Q 98.8: Approximately what percentage of patients with a solid primary tumor elsewhere will end up with metastatic disease of the vertebrae during the clinical course of their cancer? A 10% B 20% C 30% D 40% E 50%
The Correct Answer is: E Fifty percent of cancer patients will develop metastatic disease of the vertebrae at some point during the course of their illness. The highest percentages of cancers that lead to such spinal lesions are carcinomas of the breast, lung, prostate, colon, thyroid and kidney. This likely occurs through hematogenous spread. Pain is a common presenting symptom, but it may be found while still asymptomatic if routine screenings are done with bone scans, MRI or CT in patients with a known primary tumor elsewhere in the body. The pain is usually worse with weight bearing activities and better when lying down, but pain that persists through the night and prevents sleep needs to be evaluated for possible neoplasm. Some metastatic disease of the spine may present with neurological symptoms such as sensory or motor deficits following a spinal nerve root distribution or more generalized neurological deficits due to spinal cord compression or cauda equina syndrome.
Q 97.16: A 37-year-old female presents to the office complaining of increasing heavy vaginal bleeding with her menses. CBC reveals an iron deficient anemia. Her pelvic exam is notable for a moderately enlarged uterus. A transvaginal ultrasound reveals a 3-cm leiomyoma and a 15-cm leiomyoma. The patient wishes to preserve her fertility. Appropriate management would include which of the following? A Expectant management aimed at pain reduction B Bilateral uterine artery embolization C Reduction of blood loss with combined oral contraceptives D Immediate hysterectomy E Gonadotropin-releasing hormone analog followed by myomectomy
The Correct Answer is: E GRH causes a reversible hypogonadism, which reduces tumor size, makes surgical intervention safer, and reduces bleeding. The patient wishes to preserve her fertility, making embolization and hysterectomy non-viable options. COC do not significantly reduce bleeding, and do not regress the tumor for optimal surgical removal.
Q 6.10: A 64-year-old female who has a history of injectable drug use presents with blood work that reveals leukocytosis with a left shift, and there is suspicion of osteomyelitis based on the patient's prior history. Based on this history, what bone would be most affected by hematogenous osteomyelitis in adults? A Feet B Long bones C Pelvis D Sternoclavicular bones E Vertebrae
The Correct Answer is: E Hematogenous osteomyelitis accounts for about 20% of all cases of osteomyelitis in adults. It is more common in males and the prevalence is higher amongst those who are IV drug abusers, patients being treated with dialysis or who have sickle cell disease. Other conditions which may lead to sepsis (i.e. patients with central lines, urinary infections, and urethral catheterization) increase the risk of hematogenous osteomyelitis. Unlike children, the long bones are rarely affected in adults with the vertebrae being the most likely location for the bone infection to occur. Lumbar vertebrae are most often affected, followed by thoracic and cervical vertebrae. Osteomyelitis of the sternoclavicular bones and pelvic bones are not uncommon sites, but these tend to be most frequent amongst IV drug abusers. The feet do not tend to be significantly affected by hematogenous osteomyelitis as frequently as osteomyelitis caused by infected foot ulcers as are often seen in those with diabetes or peripheral vascular disease.
Q 83.8: A 23-year-old male presents with uncontrolled mucocutaneous bleeding following a wisdom tooth extraction. Traditional methods of controlling the bleeding have not significantly reduced the bleeding. On history, he reports epistaxis that takes longer than usual to stop bleeding. He also states that he seems to bruise more frequently than his friends. What laboratory test would be the most beneficial given his history? A Bleeding time and platelet count B Bleeding time and VWF antigen C Factor VII assay and factor VIII assay D Factor VII assay and platelet count E Factor VIII assay and VWF antigen
The Correct Answer is: E In a patient suspected of having von Willebrand's disease, routinely performed tests include the assay of factor VIII activity, VWF antigen, and VEW Ristocetin cofactor activity. Bleeding time was previously used as a screening test, but is inconsistent. Factor VII assay would be used for someone suspected of having VII deficiency. Platelet counts are usually normal in a patient with von Willebrand's disease. (Lichtman et al., Williams Hematology 8e, Chapter 127, von Willebrand Disease)
Q 97.21: A 30-year-old woman presents to the office complaining of an inability to achieve pregnancy after over a year of trying. What is the couple's least likely cause for infertility? A Azoospermia B Ovarian failure C Endometrial disease D Tubal disease E Pituitary disease
The Correct Answer is: E In the vast majority of women who are infertile or subfertile, the issue is due to either ovarian dysfunction or structural abnormalities, with hormonal abnormalities accounting for only 10%.
Q 113.8: A 36-year-old man with a history of tobacco use notices palpitations after attending a Super Bowl party. He admits to drinking at least six bottles of beer and several mixed drinks, which is much more than he usually drinks. He denies chest discomfort or dyspnea. On physical exam, his BP is 139/82, P 136, RR 22, and Temp 37°C. On auscultation, his heart rate is tachycardic and irregular, without S3 or murmur appreciated. His echocardiogram demonstrates normal LV systolic function, normal left atrial dimensions, and normal valvular function. Based on his history, what is the most likely diagnosis regarding his palpitations? A Sinus tachycardia B Wolff-Parkinson-White syndrome C Normal sinus rhythm with PACs D Alcohol withdrawal E Holiday heart syndrome
The Correct Answer is: E In young patients without left ventricular systolic dysfunction or valvular dysfunction, binge drinking can lead to episodes of atrial fibrillation, also known as holiday heart syndrome. This is a syndrome in which atrial fibrillation is linked with excessive alcohol intake frequently higher than usual; however, in some patients, moderate alcohol intake can also lead to episodes of atrial fibrillation. Choices A, B, and C are not necessarily associated with a history of alcohol intake. Choice D is not described in this patient scenario. (Fauci et al., 2008, Chapter 387)
Q 16.2: Which of the following statement or statements is/are true regarding influenza vaccination? A The Advisory Committee on Immunization Practices (ACIP) recommends annual influenza vaccination for all persons six months of age and older without specific contraindications. B In spite of the fact that the influenza vaccine is FDA category C, ACIP still recommends immunizing pregnant women or women who are planning to get pregnant during the upcoming influenza season. C In spite of the manufacturing method of the current influenza vaccine using chicken embryos, ACIP recommends vaccination persons with known allergies to eggs if they can eat baked products manufactured in the US without having an allergic reaction. D If an unvaccinated person develops a flu-like illness early in the influenza season, ACIP recommends that they receive an influenza vaccine once their febrile illness has resolved. E All of the above is true.
The Correct Answer is: E Influenza (the flu) is a contagious respiratory illness caused by influenza viruses. It can cause mild to severe illness, and at times can lead to death, especially in certain populations. Some people, such as older people, young children, and people with certain health conditions including pregnancy, are at the highest risk for serious flu complications. The best way to prevent the flu is by getting vaccinated each year. The vaccine contains three strains (two of influenza A and one of influenza B) and is derived from chicken embryos and people with severe egg allergies should not be vaccinated. However, if a person is able to eat backed goods manufactured in the United States, according to the CDC, their potential level of allergy is so low that the benefit of vaccination outweighs any reaction. Once a vaccine-eligible patient has a viral-like illness and their fever resolves, he or she should receive the annual influenza vaccine. (Hay, et al., 2011, Chapter 9)
Q 64.5: A 63-year-old male with a longstanding history of chronic obstructive pulmonary disease and a recent exacerbation, is also found to have new onset rapid atrial fibrillation. Which of the following medications, used for atrial fibrillation, should be used cautiously as it is associated with bronchospasm and pulmonary function changes? A Amiodarone B Digoxin C Diltiazem D Ibutilide E Metoprolol
The Correct Answer is: E Initial atrial fibrillation management will depend on the stability of the patient, and may involve emergent electrocardioversion if the patient is unstable. Several types of medications are used for medication cardioversion and subsequent rate management in atrial fibrillation patients, including each of the medications listed in the answer choices. Caution should be used when administering a beta-blocker, even a cardioselective choice, to a patient with longstanding lung disease and recent exacerbation, as this may promote bronchospasm and associated dyspnea.
Q 72.5: In a patient with chronic hepatitis C infection, which of the following medical conditions would be considered a contraindication to starting the patient on interferon? A hypertension B hyperlipidemia C diabetes D migraine headaches E systemic lupus erythematosus
The Correct Answer is: E Interferon is contraindicated in patients with autoimmune disease. Interferon is also contraindicated in patients with severe liver disease and history of cardiac arrhythmia. It should be used with caution in patients with major depressive disorders, cytopenia, hyperthyroidism, and severe renal insufficiency. (Safrin, 2009, pp. 868) Safrin S. Antiviral agents. In: Katzung BG , Masters SB , Trevor AJ, eds. Basic and Clinical Pharmacology. 11th ed. New York, NY: McGraw-Hill; 2009.
Q 68.1: In patients with dilated cardiomyopathy who have multiple runs of symptomatic non-sustained ventricular tachycardia, what is the best intervention to treat this condition? A Ablation therapy B Heart transplantation C Medical therapy alone D Diet modification E Implantable defibrillator
The Correct Answer is: E It is recommended for patients with a history of dilated cardiomyopathy who have documented non-sustained ventricular tachycardia to have implantable defibrillators inserted for primary prevention of sudden cardiac death. Medical therapy alone does not protect the patient from arrhythmias, and ablation therapy is not indicated for this type of arrhythmia. (Bashore et al., Current Medical Diagnosis and Treatment, Chapter 10)
Q 13.10: A 23 year-old male with cystic fibrosis inquires about the availability of treatments that can help improve his lung function. Which of the following treatments is most effective at reversing the pulmonary effects of cystic fibrosis? A Albuterol B Azithromycin C Inhaled hypertonic saline D Inhaled levofloxacin E Ivacaftor
The Correct Answer is: E Ivacaftor (E) is the only treatment that restores function of the CFTR protein in cystic fibrosis patients with a G551D mutation thereby reversing the effects of the disease, approximately 5% of all cystic fibrosis patients have the G551D mutation. Albuterol (A) and hypertonic saline (C) are indicated to improve lung function and mucous clearance. Azithromycin (B) and inhaled levofloxacin (D) are used to treat chronic infection/colonization with pseudomonous.
Q 92.8: A 61-year-old male presents with acute onset dyspnea, cough with thick, blood tinged sputum, and fever. He is a known, longstanding alcoholic and has Type II Diabetes, for which he takes no medication. A chest x-ray reveals a right upper lobe infiltrate with early fibrotic changes. Which of the following is an appropriate first-line medication(s) for the most likely etiologic organism? A Macrolide B Doxycycline C Second generation cephalosporin D Beta-lactam plus fluoroquinolone E Fluoroquinolone
The Correct Answer is: E Klebsiella pneumoniae, a gram negative bacteria, is associated with pneumonia in immunocompromised patients, especially alcoholics. Community-acquired pneumonia due to Klebsiellae is associated with a broad range of presentations. This may range from mild acute bronchitis or bronchopneumonia to acute onset of high fevers, chills, and cough with blood tinged, thick sputum (often referred to as "currant jelly sputum"), and aggressive chest x-ray findings, such as cavitation and empyema. Community-acquired pneumonia empiric treatment is based upon the likely causative organisms within patient subsets. This includes appropriate selections for individuals able to be handled as outpatients, but with significant past medical history and risk factors for drug-resistant bacteria, such as alcoholism. The guidelines currently indicate that these patients should be treated with a beta-lactam plus a macrolide or doxycycline, or a fluoroquinolone, or amoxillin-clavulanate. The guidelines also include treatment recommendations for other patient subsets, and alternative medications to consider (which can be viewed in Table 251-4). (Harrison's Online, Chapter 251, Part Ten: Disorders of the Respiratory System, Section 2: Diseases of the Respiratory System)
Q 116.15: An elderly patient is brought in to the emergency department (ED) complaining of incontinence of liquid "like tea water" stool. He is complaining of rectal pressure and lower abdominal pain. The pain is cramping in quality and the patient's abdomen is "bloated." Digital rectal exam reveals hard stool in the rectum. Which of the following should be selected as the initial treatment for this patient? A passing a nasogastric tube B milk of magnesia C opiate analgesics for pain D oral sodium phosphate E manual disimpaction
The Correct Answer is: E Mechanical bowel obstruction in the rectum does not usually respond to oral laxatives. A nasogastric tube would not be used for an obstruction in the distal colon/rectum. One would avoid opiates in fecal impactions and other constipation problems because they tend to be more constipating. This patient needs to be disimpacted. Oral agents are unlikely to be effective against the fecal impaction and may cause complications. (McQuaid, 2008, p. 481) McQuaid KR. Gastrointestinal disorders. In: McPhee SJ , Papadakis MA, eds. Current Medical Diagnosis and Treatment. 47th ed. New York, NY: McGraw-Hill; 2008.
Q 86.9: A patient that has exhibited excessive bleeding tendencies is scheduled for a cholecystectomy. By history, he is noted to be of Ashkenazi Jewish decent. Which coagulation disorder is most likely the cause of his bleeding tendency? A Factor V B Factor VII C Factor VIII D Factor IX E Factor XI
The Correct Answer is: E Most patients with factor XI deficiency are Jewish, with most in the Ashkenazi population. While other factors may affect individual patients, the most common of these coagulation disorders within this population base is factor XI. (Lichtman et al., Williams Hematology 8e, Chapter 125, Inherited Deficiencies of Coagulation Factors II, V, VII, X, XI, and XIII and Combined Deficiencies of Factors V and VIII and of the Vitamin K-Dependent Factors)
Q 80.4: A 34-year-old female with a definitive diagnosis of multiple sclerosis presents with an acute symptomatic exacerbation for two days, causing difficulty ambulating and markedly painful paresthesias, which is affecting her ability to sleep. She is currently on Interferon-β-1A, although she admits to missing some recent doses. Which of the following medications is most appropriate for treating her acute exacerbation? A Glatiramer acetate B Interferon-β-1B C Mitoxantrone D Natalizumab E Prednisone
The Correct Answer is: E Multiple sclerosis (MS) therapy is often discussed by the goal of the treatment. Medications, such as the glucocorticoids, are utilized for acute exacerbations or initial episodes of MS, to decrease exacerbation severity. Additionally, they are used for associated conditions, such as optic neuritis. Steroids have not been shown to decrease MS progression or impact the relapse rate. Other medications have shown to be beneficial regarding altering disease progression and/or the relapse rate. These medications include Interferon-β agents, glatiramer acetate, and Natalizumab. Mitoxantrone, an antineoplastic agent, is also indicated as an MS disease-altering agent, but due to a high risk of cardiotoxicity with prolonged use, it is a second-line agent. (McPhee SJ, Papadakis MA. Current Medical Diagnosis & Treatment 2011, Chapter 24, Nervous System Disorders)
Q 6.6: A 15-year-old male presents complaining of a sore throat, headache, and mild cough that started 8 days ago and has progressed to include a worsening cough and increasing fatigue. His chest x-ray reveals bilateral hilar infiltrates, CBC is normal and a nasal secretions test positive for mycoplasma pneumoniae by PCR. What is the most appropriate therapy? A Amoxicillin B Cefuroxime C Clindamycin D Ciprofloxacin E Doxycycline
The Correct Answer is: E Mycoplasma pneumoniae is commonly treated with macrolides, doxycycline (E), or respiratory fluoroquinolones. Mycoplasma pneumonia doesn't respond to beta-lactam antibiotics (A, B) or non-respiratory fluoroquinolones (D).
Q 72.4: A 24-year-old male has an eight-month history of loose thought associations, social withdrawal, auditory hallucinations, and deterioration in his personal appearance and hygiene. Upon examination, he is noted to have a flat affect, perceptual distortions, and behaves like he is detached from his own actions. An atypical antipsychotic medication is chosen for this patient. Which of the following medications is classified as an atypical antipsychotic? A Chlorpromazine (Thorazine) B Haloperidol (Haldol) C Loxapine (Loxitane) D Molindone (Moban) E Olanzapine (Zyprexa)
The Correct Answer is: E Olanzapine is the only atypical antipsychotic listed in the given answers. The remainder of the medications listed are typical antipsychotic medications. (McPhee SJ, Papadakis MA. Current Medical Diagnosis & Treatment, 2010, p. 954)
Q 24.2: A 78-year-old woman with chronic, open-angle glaucoma (COAG) presents to the outpatient clinic complaining of fatigue, and near syncope. She just began eye drops for her glaucoma 3 days ago. Which of the following ophthalmic drop medications is likely causing her side effects? A Bimatoprost B Brimonidine C Brinzolamide D Dipivefrin E Timolol
The Correct Answer is: E Open angle glaucoma is typically treated first-line with beta adrenergic blocking agents (E) or prostaglandin analogs (A). A common side effect of beta adrenergic agents is their systemic absorption and subsequent beta blockade sides effects (i.e. decreased pulse, blood pressure) leading to symptoms such as fatigue, pre-syncope, or syncope. Treatments (B), (C), and (D) are not typically first-line agents and lack the beta blockade side effects this patient is experiencing.
Q 15.4: A 56-year-old insulin dependent diabetic has been under your evaluation for his diabetes for several years. The patient has a 3-year history of diabetic neuropathy to the right foot, and may have suffered an injury to the foot without knowing due to loss of sensation. The patient now presents with a tender, reddened, and swollen right foot for the last 10 days that is also warm to the touch. You suspect that this patient may have an acute case of osteomyelitis. Based on this history, what bacterial organism is most commonly the cause of osteomyelitis? A Group A beta-hemolytic streptococci B Hemophilus influenzae C Mycoplasma D Pseudomonas aeruginosa E Staphylococcus aureus
The Correct Answer is: E Osteomyelitis is an infection in a bone and can occur in patients of all ages. The most common organism implicated in osteomyelitis across all age groups is Staphylococcus aureus. It can enter the bone through multiple mechanisms including by direct inoculation during an open fracture or during surgical intervention following a fracture (most common mechanisms for adults) or by hematogenous spread from another source (the usual cause in children). Hemophilus influenzae was a much more common organism in pediatric osteomyelitis in the past, but its prevalence is decreasing due to routine immunizations. Group A beta-hemolytic streptococci is the second most common organism found in osteomyelitis in children, while Pseudomonas aeruginosa is the second most common pathogen in adults. Mycoplasma induced osteomyelitis is relatively rare and usually confined to immunocompromised patients.
Q 98.14: A 56-year-old insulin dependent diabetic has been under your evaluation for his diabetes for several years. The patient has a 3-year history of diabetic neuropathy to the right foot, and may have suffered an injury to the foot without knowing due to loss of sensation. The patient now presents with a tender, reddened, and swollen right foot for the last 10 days that is also warm to the touch. You suspect that this patient may have an acute case of osteomyelitis. Based on this history, what bacterial organism is most commonly the cause of osteomyelitis? A Group A beta-hemolytic streptococci B Hemophilus influenzae C Mycoplasma D Pseudomonas aeruginosa E Staphylococcus aureus
The Correct Answer is: E Osteomyelitis is an infection in a bone and can occur in patients of all ages. The most common organism implicated in osteomyelitis across all age groups is Staphylococcus aureus. It can enter the bone through multiple mechanisms including by direct inoculation during an open fracture or during surgical intervention following a fracture (most common mechanisms for adults) or by hematogenous spread from another source (the usual cause in children). Hemophilus influenzae was a much more common organism in pediatric osteomyelitis in the past, but its prevalence is decreasing due to routine immunizations. Group A beta-hemolytic streptococci is the second most common organism found in osteomyelitis in children, while Pseudomonas aeruginosa is the second most common pathogen in adults. Mycoplasma induced osteomyelitis is relatively rare and usually confined to immunocompromised patients.
Q 29.9: Which of the following is a type of primary malignant tumor of the bone? A Enchondroma B Leiomyoma C Lipoma D Melanoma E Osteosarcoma
The Correct Answer is: E Osteosarcoma is a type of primary bone cancer. Fortunately this is a rare cancer, accounting for only 1% of the cancers diagnosed each year in the USA for patients of all ages. Osteosarcomas are more common in the pediatric population, accounting for 5% of all childhood cancers and 56% of all cancers of the bone in patients less than 20 years old. Osteosarcomas have a bimodal occurrence rate with the highest prevalence between ages 11-13 and over the age of 65. Enchondromas are common benign cartilaginous tumors that develop in the medulla (marrow cavity) of bone. A leiomyoma is another name for a benign uterine fibroid. A lipoma is a benign, soft, freely movable, generally nontender mass in the soft tissue sometimes referred to as a fatty tumor. These are generally inconsequential, but may be a marker for spina bifida if found in the lumbar region. Melanoma can be found in bone, but it would be considered a metastatic lesion and not a primary malignant bone tumor.
Q 98.55: Which of the following is a type of primary malignant tumor of the bone? A Enchondroma B Leiomyoma C Lipoma D Melanoma E Osteosarcoma
The Correct Answer is: E Osteosarcoma is a type of primary bone cancer. Fortunately this is a rare cancer, accounting for only 1% of the cancers diagnosed each year in the USA for patients of all ages. Osteosarcomas are more common in the pediatric population, accounting for 5% of all childhood cancers and 56% of all cancers of the bone in patients less than 20 years old. Osteosarcomas have a bimodal occurrence rate with the highest prevalence between ages 11-13 and over the age of 65. Enchondromas are common benign cartilaginous tumors that develop in the medulla (marrow cavity) of bone. A leiomyoma is another name for a benign uterine fibroid. A lipoma is a benign, soft, freely movable, generally nontender mass in the soft tissue sometimes referred to as a fatty tumor. These are generally inconsequential, but may be a marker for spina bifida if found in the lumbar region. Melanoma can be found in bone, but it would be considered a metastatic lesion and not a primary malignant bone tumor.
Which of the following is a type of primary malignant tumor of the bone? A Enchondroma B Leiomyoma C Lipoma D Melanoma E Osteosarcoma
The Correct Answer is: E Osteosarcoma is a type of primary bone cancer. Fortunately this is a rare cancer, accounting for only 1% of the cancers diagnosed each year in the USA for patients of all ages. Osteosarcomas are more common in the pediatric population, accounting for 5% of all childhood cancers and 56% of all cancers of the bone in patients less than 20 years old. Osteosarcomas have a bimodal occurrence rate with the highest prevalence between ages 11-13 and over the age of 65. Enchondromas are common benign cartilaginous tumors that develop in the medulla (marrow cavity) of bone. A leiomyoma is another name for a benign uterine fibroid. A lipoma is a benign, soft, freely movable, generally nontender mass in the soft tissue sometimes referred to as a fatty tumor. These are generally inconsequential, but may be a marker for spina bifida if found in the lumbar region. Melanoma can be found in bone, but it would be considered a metastatic lesion and not a primary malignant bone tumor.
Q 97.119: A 29-year-old woman comes to the emergency department with abrupt onset of left lower quadrant pain approximately 1.5 hours ago after she and her roommate moved multiple heavy pieces of furniture into their new apartment. The pain is now excruciating and she rates it "100 out of 10." She vomits on admission to the department. She is not currently sexually active and has no other significant past medical history. On exam she is found to have fullness and extreme tenderness in the left adnexa. Of the following, what is the most likely diagnosis? A Diverticulitis B Pelvic inflammatory disease C Renal calculus D Ruptured ovarian cyst E Torsion of the left ovary
The Correct Answer is: E Ovarian torsion is often extremely painful and is often accompanied by vomiting, with sudden onset and may be associated with episodes of heavy exertion. Diverticulitis (A) typically causes LLQ pain, but would be unusual in a 26-year-old woman because it occurs more often in older adults. The symptoms of PID (B) do not begin abruptly and vomiting is not typical. The pain of renal calculi (C) is colicky and calculi are not associated with a palpable adnexal mass. The pain of a ruptured ovarian cyst (D) would be sudden in onset; however, rupture would not be associated with a palpable mass.
Q 75.2: What is the laboratory test that has the best predictability for diagnosing pneumonia caused by adenovirus? A Nasal washings B Viral DNA test C Complete blood count with differential D Sputum gram stain E Polymerase chain reaction test (PCR)
The Correct Answer is: E PCR testing is the best method for determining adenovirus pneumonia. It has a sensitivity of 90 to 100% and the specificity is >95%. (Lee et al., Current Diagnosis and Treatment in Pulmonary Medicine, Section X, Pulmonary Lung Disease, Chapter 37, Viral and Atypical Pneumonia)
A 59-year-old woman presents to the emergency department with an acute upper gastrointestinal hemorrhage. Her medical history is pertinent for peptic ulcer disease for the past 5 years and hypertension. A nasogastric tube is inserted and bright red blood is seen. Her vital signs are BP: 110/70 mm Hg, P: 94/min, R: 14/min, oxygen saturation: 97%, T: 99°F. Which of the following diagnostic studies would be the most appropriate next step to determine the site of bleeding? A Abdominal and pelvic computed tomography B Abdominal ultrasound C Upper gastrointestinal series with barium D Bleeding scan E Upper endoscopy
The Correct Answer is: E Patients who present with upper gastrointestinal bleeding and shock requiring multiple transfusions in 24 hours are at high risk for mortality from gastrointestinal bleeding. The upper gastrointestinal bleeding in this patient warrants further investigation with upper gastrointestinal endoscopy to both determine the site of bleeding and provide potential therapy by endoscopic electrocautery or injection.
Q 11.2: A 22-year-old patient with sickle cell disease presents to the emergency department complaining of chest pain, fever, and non-productive cough. On physical exam his temperature is 100.6˚F, BP is 144/88, pulse is 110, respiratory rate is 24, and pulse oximetry is 84%. CBC shows a WBC of 11,500, hemoglobin of 8.3%, and hematocrit of 28%. What is the most critical/emergent treatment for this patient? A Acetaminophen B Bone marrow transplant C Furosemide D Hydroxyurea E Transfusion
The Correct Answer is: E Patients with sickle cell disease are prone to acute chest syndrome resulting from sickling of cells within the lung that typically presents with chest pain, tachypnea, cough, fever, and oxygen desaturation. Transfusion is indicated when the hematocrit drops below 30% or oxygen saturation drops below 90%. Bone marrow transplant is only an option in children (B) and not indicated for emergent therapy. Furosemide (C) would promote dehydration and further exacerbation of cell sickling. Hydroxyurea (D) is used prophylactically to reduce the frequency of sickle cell crises and/or acute chest syndrome. Acetaminophen (A) may be used for symptomatic treatment, but is less critical than transfusion and may not provide adequate pain management.
Q 106.9: A 35-year-old male presents complaining of increasingly constant headaches, double vision centrally, and a progressive loss of peripheral vision for two weeks. He has no previous headache history and denies any other medical conditions. Physical examination reveals bitemporal hemianopsia without additional neurologic findings. What is the most likely diagnosis? A Acute ischemic stroke B Circle of Willis ruptured aneurysm C Migraine headache D Multiple sclerosis E Pituitary adenoma
The Correct Answer is: E Pituitary adenomas, benign neoplasms associated with pituitary hormone secretory changes, may enlarge and become symptomatic. Symptoms are based upon the location and size of the tumor, and may include bitemporal hemianopsia, double vision, color desaturation, and visual acuity loss. Headaches may occur, due to associated pressure changes within the intrasellar space. Additional evaluation should include a T1-weighted MRI, screening laboratory tests, and a full ophthalmologic evaluation. These tests will also help evaluate for potential differential diagnoses, such as those listed. The patient's history is not consistent with an acute ischemic stroke or migraine headache. Although an unruptured aneurysm may have very similar findings to a pituitary tumor, ruptured aneurysms present with acute headache, nausea, vomiting, and potential changes in consciousness. Multiple sclerosis (MS) should remain on the differential for this patient and will also be evaluated through MRI (although the current findings are more consistent with a pituitary adenoma), and additional neurologic findings would be likely with MS. (Fauci et al., Harrison's Principles of Internal Medicine, 17e, Chapter 333, Disorders of the Anterior Pituitary and Hypothalamus)
Q 92.3: A 68-year-old female has been having increasing low back pain that radiates into her right hip and down her right leg. She has failed physical therapy and anti-inflammatories. She feels like the bones "are rubbing on each other." Radiographic imaging of the spinal cord, hip, and leg reveal several lytic lesions. The most helpful finding to confirm the diagnosis would include which of the following? A Reed-Sternberg cells in the marrow B Granulocytosis in the marrow C Macrocytosis in the peripheral smear D Monocytosis in the peripheral smear E Plasmacytosis in the marrow
The Correct Answer is: E Plasmacytosis in the marrow is the most prominent and diagnostically important finding. Reed-Sternberg cells are seen in Hodgkin's lymphoma. Granulocytosis is an increased white cell count seen in leukemias. Macrocytosis is enlargement of red cells, and can be seen in vitamin B12 deficiency. Monocytosis is an increase in monocytes, and can be seen in infectious mononucleosis or some leukemias. (Lichtman et al., Williams Hematology 8e, Chapter 109, Myeloma)
Q 29.7: A 64-year-old, nonsmoking male presents to the clinic with complaints of headache and dizziness. Initial labs reveal hemoglobin of 20.1 g/dl, hematocrit of 60%, and platelet count of 567,000. Which of the following is the most likely diagnosis in this patient? A Acute myeloid leukemia B Chronic myeloid leukemia C Chronic obstructive pulmonary disease (COPD) D Multiple myeloma E Polycythemia vera
The Correct Answer is: E Polycythemia vera often presents with complaints related to increased red blood cell mass, such as, headache, fatigue, and dizziness with elevated red blood cell mass with leukocytosis and/or thrombocytosis noted on CBC. Chronic hypoxia from conditions such as COPD (C) can also cause elevated RBC mass, but is unlikely in this non-smoker. The patient lacks white blood cell abnormality symptoms (e.g., recurrent infections, elevated white count) making acute myeloid leukemia (A), chronic myeloid leukemia (B), or multiple myeloma (D) unlikely diagnoses.
Q 97.121: A 17-year-old female presents to the emergency department complaining of watery vaginal discharge for 6 hours. She is found to be at 35 weeks gestation. An external fetal monitor reveals fetal heart tones in the 130s, good variability, and no contractions. What is the most likely diagnosis? A Preterm labor B Preterm rupture of membranes C Premature rupture of membranes D Preterm labor and premature rupture of membranes E Preterm and premature rupture of membranes
The Correct Answer is: E Preterm rupture of membranes is defined as rupture before 37 weeks gestation; premature is defined as before the onset of labor. The absence of pain or contractions decreases the likelihood of labor.
Q 45.1: A 10-year-old male patient has been seen in the clinic on several occasions, with complaints of fatigue and fevers of unknown origin. On prior CBC, he was noted to have a hypereosinophilia. Today's smear reveals multiple small lymphoblasts, with scanty, light-blue cytoplasm after being stained with Wright-Giemsa, and inconspicuous nucleoli. Initial therapy for the patient should include which of the following? A Bone marrow biopsy and repeat CBC B Hematologic support for thrombocytopenia C Immediate referral for radiation therapy D Placement of an indwelling catheter and IV fluids E Prevention of metabolic and infectious complications
The Correct Answer is: E Prevention of metabolic and infectious complications are the immediate therapies of choice in this patient with ALL. This is done to prevent complications of hyperuricemia, hyperphosphatemia, and infection. Bone marrow biopsy, while important, is not a therapy. Platelet transfusions are only used if there are signs of bleeding and a low platelet count (usually less than 20,000). There is no place for radiation therapy at this point in the patient's initial treatment. Placement of an indwelling catheter is important early in treatment, but is not necessary as an initial treatment.
Q 101.6: A 55-year-old woman with a history of emphysema, who is undergoing chemotherapy for lung cancer, is sent to see you by her oncologist regarding a sudden increase in dyspnea, with exertion and fatigue. On physical exam, a drop in systolic blood pressure of 20 mm Hg is noted upon inspiration. What is this physical exam finding known as? A Pulsus parvus B Pulsus alternans C Bisferiens pulses D Pulsus bigeminus E Pulsus paradoxus
The Correct Answer is: E Pulsus paradoxus is defined as a decrease in systolic arterial pressure of greater than 10 mmHg. It is an accentuation of the normal decrease in systolic arterial pressure of less then 10mm Hg that normally accompanies inspiration. It is frequently noted in patients with pericardial tamponade. Pulsus parvus means a small weak pulse. Pulsus alternans is noted in patients who despite a regular rhythm, demonstrate a regular alteration of the pressure pulse amplitude. This is frequently found in patients with severe left ventricular systolic dysfunction. A bisferiens pulse is a pulse with two systolic peaks, commonly seen in patients with aortic regurgitation or hypertrophic cardiomyopathy. (Fauci et al., 2001, p. 1256)
Q 49.16: A 3-week-old male infant is brought in by his mother due to his vomiting. The mother notes that a few days ago, her son started vomiting after feeding and it has become projectile in nature. The vomitus is non-bilious and contains no blood. The child seems hungry and nurses regularly, but the vomiting has become more frequent and is occurring with every feeding now. On physical examination, an oval mass is palpated in the right upper quadrant. What is the most likely diagnosis? A Duodenal atresia B Intestinal malrotation C Intussusception D Peptic ulcer disease E Pyloric stenosis
The Correct Answer is: E Pyloric stenosis is the result of muscular hypertrophy with gastric outlet obstruction. The patient's symptoms and signs are most consistent with this diagnosis. Children with duodenal atresia present with bilious vomiting and epigastric distention within a few hours of life. Intestinal malrotation presents with bilious vomiting, usually within the first three weeks of life. Intussusception presents in a thriving three- to 12-month-old child with recurring bouts of abdominal pain. Vomiting and diarrhea usually follow, and bowel movements may appear bloody and contain mucus. In a young child, peptic ulcer disease usually presents with vomiting and upper GI bleeding.
Q 27.5: A 23 year-old patient being treated for tuberculosis presents to the clinic complaining of red discoloration of his contact lenses and staining of his eye. Which of the following medications can change the color of tears leading to this complication? A Amoxicillin B Clindamycin C Doxycycline D Metronidazole E Rifampin
The Correct Answer is: E Rifampin (E) is an anti-tuberculosis medicine that changes urine, tears, feces, and sweat to a harmless orange color. Patients should be made aware of this effect prior to starting the medication and should be advised to discontinue contact lens usage while taking rifampin.
Q 98.56: A 57-year-old woman underwent a hemicolectomy for adenocarcinoma of the colon. Which of the following recommendations is part of postoperative monitoring for a potential recurrence? A Annual fecal occult blood testing B Annual chest radiograph C Annual CA 19-9 testing D Annual flexible sigmoidoscopy E Annual colonoscopy
The Correct Answer is: E Routine follow-up after surgical resection of a colon cancer includes annual colonoscopy not sigmoidoscopy, which only assesses the distal colon. The tumor marker for colon cancer is carcinoembryonic antigen (CEA) not carbohydrate antigen 19-9 (CA 19-9), which is used for pancreatic cancer. There is no role for annual chest films or fecal occult blood testing to monitor for a recurrence.
Q 114.6: A 68-year-old man is recently diagnosed with depression associated with the loss of his close sister to an automobile accident. He is currently taking oxybutynin for overactive bladder disease and lisinopril for hypertension. He has no known drug allergies. Which of the following medications would be most appropriate to prescribe for this patient? A alprazolam B amitriptyline C buspirone D desipramine E fluoxetine
The Correct Answer is: E Selective serotonin reuptake inhibitors (SSRIs) such as fluoxetine are usually considered first-line antidepressants due to their relative safeness in overdose and their minimal affinity for muscarinic, α-adrenergic, and histamine receptors, thereby causing fewer side effects. Tricyclic antidepressants such as amitriptyline and desipramine produce several adverse effects associated with their antimuscarinic properties (eg, dry mouth, constipation, blurred vision, urinary retention, etc). The patient is already taking the antimuscarinic agent oxybutynin, so a tricyclic antidepressant could attenuate these adverse effects. Orthostatic hypotension is also common with tricyclic antidepressants, and because the patient is taking lisinopril for hypertension, the risk for a significant drop in blood pressure is high. Buspirone and alprazolam are not indicated for depression. (Teter et al., 2008, pp. 1128-1129) Teter CJ , Kando JC , Wells BG , et al. , Depressive disorders. In: DiPiro JT , Talbert RL , Yee GC, et al., eds. Pharmacotherapy: A Pathophysiologic Approach. 7th ed. New York: McGraw-Hill; 2008.
Q 97.102: A 25-year-old woman comes in for her annual physical examination and renewal of her oral contraceptive prescription. She has no problems today and appears fit and well. On palpation of the thyroid, the right lobe is small, smooth, and free of nodules, but appears to be slightly larger than the left lobe. She has no other unexpected physical findings. The enlargement most likely represents which of the following? A inflammation of the thyroid B goiter C hypothyroidism D malignancy E normal finding
The Correct Answer is: E Slight enlargement of one lobe of the thyroid gland in a healthy woman with no complaints is most likely a normal finding. In patients with an inflammation of the gland (A), i.e., thyroiditis, and hypothyroidism (C) the thyroid is usually diffusely enlarged and may be markedly asymmetric. Other findings depend upon the cause. Goiters (B) are typically nodular and may be quite enlarged. Thyroid malignancy (D) usually presents as a firm, non-tender nodule.
Q 32.8: Your patient is a 42-year-old male who comes to you because, after a recent move to the country, he finds he has an excessive fear of snakes leading to extreme anxiety. He realizes that this fear has been present since childhood, but was not much of a problem before. What is your diagnosis? A Conversion disorder B Factitious disorder C Generalized anxiety disorder D Social phobia E Specific phobia
The Correct Answer is: E Specific phobia (E) is the fear of a very specific object or situation that the individual knows to be excessive. Social phobia (D) involves performance, while generalized anxiety (C) involves anxiety without a known stimulus. Conversion disorder (A) and factitious disorder (B) are both somatoform disorders, involving somatic symptoms with a psychogenic cause.
Q 9.3: A 64-year-old, nonsmoking male presents to the clinic with complaints of headache and dizziness. Initial labs reveal hemoglobin of 20.1 g/dl, hematocrit of 60%, and platelet count of 567,000. Which of the following is the most likely physical exam finding in this patient? A Erythromelalgia B Hepatomegaly C Loss of vibratory sense D Petechiae E Splenomegaly
The Correct Answer is: E Splenomegaly is the most common physical exam finding in patients with polycythemia vera (PCV) and may be accompanied by hepatomegaly (B). Erythromelalgia (A) also occurs in patients with PCV due to increased red blood cell mass, but occurs less commonly than splenomegaly. Patients with PCV often have thrombocytosis, which most commonly contributes to thrombotic events, but less commonly may result in abnormal platelet function and petechiae (D) and increased bleeding risk. PCV is associated with elevated B 12 levels and not likely to result in the classic neurologic findings of B 12 deficiency including peripheral neuropathy assessed by vibratory sensation (C), ataxia and dementia.
Q 88.3: A 62-year-old male diabetic patient with a past medical history of percutaneous transluminal coronary angioplasty (PTCA) with drug-eluting stent presents to review his blood work. He currently takes rosuvastatin 10 mg daily, metformin 1,000 mg BID, an 81 mg aspirin, and ramapril 5 mg, along with metoprolol ER 50 mg daily. His total cholesterol is 212 mg/dL, LDL-C is 126 mg/dL, HDL-C is 46 mg/dL, and triglycerides at 145 mg/dL. Of the following, what is the best choice for him to reach ATP-III revised goals? A Add a fenofibrate 145 mg daily. B Add garlic to his diet. C Add an omega-3 ethyl acid ester daily. D Increase his rosuvastatin to 20 mg daily. E Increase his rosuvastatin to 20 mg daily and add ezetimibe 10 mg daily.
The Correct Answer is: E Statins lower LDL-C by 20% to 55%, depending on the dose and statin used. Regardless, analysis of dose-response relationships for all statins demonstrates that the efficacy of LDL-C lowering is log-linear; LDL-C is reduced by 6% (from baseline) with each doubling of the dose. In doubling his rosuvastatin, it would be reasonable to expect his LDL-C to drop from 126 to 119 mg/dl, and this would not be a goal for the patient. While there is no outcome data, laboratory data would indicate that if ezetimibe were then added, LDL-C would drop approximately another 25% to approximately 89 mg/dl, closer to his ATP-III goal of 70 mg/dl. Intensifying his life style changes might then allow him to reach goal. (Brunton, et al., 2006, Chapter 35)
Q 97.19: A 21-year-old female presents to clinic complaining of mild, low abdominal ache and intermittent dysuria. She denies N/V/D, and she is sexually active and uses condoms some of the time. Her LMP was 10 days ago, and she is a G0P0. Physical exam reveals a healthy female in no acute distress. Vitals are as follows: P 70, BP 120/80, T 99.9°F. Lungs are clear, CV RRR, abd soft non-tender, + BS. Pelvic exam reveals normal external genitalia, scant discharge, moderate cervical motion tenderness, and no adnexal masses. What is her most likely diagnosis? A Tubo-ovarian abscess B Gastroenteritis C Ectopic pregnancy D Cervicitis E Pelvic inflammatory disease
The Correct Answer is: E Suspicion for PID should be very high in a young, healthy, and sexually active woman with cervical motion tenderness. She is not spotting and just menstruated, making ectopic pregnancy much less likely.
Q 102.1: What is the most appropriate age to administer the pneumococcal vaccine in healthy individuals with no other comorbid diseases? A Under age 16 B After age 16-34 C > Age 35 D > Age 50 E > Age 65
The Correct Answer is: E The Advisory Committee on Immunization Practices of the Centers for Disease Control and Prevention recommend that the most appropriate age to administer the pneumococcal vaccine in healthy individuals with no other comorbid diseases is over the age of 65. (Musher D. Harrisons Online, Part 6, Infectious Disease, Section 6, Diseases Caused by Gram-Positive Bacteria)
Q 110.15: You are asked to counsel a patient concerning prognosis of treatment for their non-Hodgkin lymphoma. The patient is 70 years old, has a prior history of smoking and a myocardial infarction 15 years ago, is overweight and has a sedentary lifestyle, and his disease is stage IV. What is the most likely prognosis? A 100% five year survival B 80% five year survival C 60% five year survival D 40% five year survival E Less than 20% five year survival
The Correct Answer is: E The International Prognostic Index is widely used to categorize patients into prognostic groups. Patients with greater than two risk factors have the lowest response rates to treatment. Worse prognosis is for patients over 60 years of age with elevated serum LDH, stage III or stage IV disease, and/or poor performance status. This patient has all but the elevated serum LDH. (Non-Hodgkin Lymphomas, Quick Answers to Medical Diagnosis and Therapy)
Q 17.1: A 16-year-old male high school wrestler presents to your family practice clinic with a fluctuant 3 cm by 3 cm abscess to his back superior and lateral to his scapula. Which of the following is the most appropriate treatment? A amoxicillin-clavulanate orally for 10 days B cephalexin orally for 10 days C trimethoprim sulfamethoxazole orally for 10 days D vancomycin intravenously for 10 days E incision and drainage is likely to resolve the abscess without the need for medications
The Correct Answer is: E The causative agent of this abscess is most likely caused by community-acquired methicillin-resistant S aureus (caMRSA). Infectious Diseases Society of America (IDSA) guidelines issued in January of 2011 generally recommend incision and drainage alone for fluctuant abscesses in an otherwise immunocompetent patient.
Q 27.9: A 23-year-old male with a recent diagnosis of Non-Hodgkin's lymphoma presents complaining of swelling of the neck and face, cough, and dyspnea on exertion. On exam you note dilated neck veins. Which of the following is the most likely diagnosis? A Angioedema B Carotid artery dissection C Cushing's syndrome D Myxedema E Superior vena cava syndrome
The Correct Answer is: E The classic presentation signs and symptoms of superior vena cava syndrome (SVCS) are present in this patient. Although the most common cancer associated with SVCS is lung cancer, it also occurs in Non-Hodgkin's lymphoma. Angioedema (A), carotid artery dissection (B), Cushing's syndrome (C) and myxedema (D) can result in facial swelling, but should not lead to the presence of dilated neck veins.
Q 11.7: A 64-year-old, nonsmoking male presents to the clinic with complaints of headache and dizziness. Initial labs reveal hemoglobin of 20.1 g/dl, hematocrit of 60%, and platelet count of 567,000. Which of the following laboratory finding confirms the suspected diagnosis? A Bone marrow hypocellularity B Elevated ferritin C Elevated serum erythropoietin D Presence of Philadelphia chromosome E Presence of JAK2 mutation
The Correct Answer is: E The clinical presentation is most consistent with polycythemia vera (PCV), which, in the presence of a markedly elevated red blood cell mass, is confirmed by the presence of the JAK2 mutation. The patient's erythropoietin (C) and ferritin (B) are most likely decreased as a result of bone marrow overproduction of RBCs and use of iron stores to produce these RBCs. The bone marrow of patients with PCV is typically hypercellular (A) across all cell lines.
Q 120.4: What is the hallmark finding on an EKG that is consistent with Wolff-Parkinson-White syndrome? A Prolonged PR interval B Long QT interval C Widened QRS complex D Sinus arrhythmia E Delta wave formation
The Correct Answer is: E The delta wave appears as an up sloping curvature that begins the QRS complex (as seen on the ECG shown). (Calkins H. Hurst's the Heart, Chapter 38, Supraventricular Tachycardia: AV Nodal Reentry and Wolff-Parkinson-White Syndrome)
Q 12.10: A 24-year-old woman comes to your office complaining of episodic chest pains that have been occurring over the past several months. She experiences shortness of breath with each occurrence and feels like "I'm going to die." She denies cough, fever, or sputum production and has noticed the episodes occur when she is in a crowd. Her current examination is normal and she feels well between episodes. You suspect panic disorder, but want to rule out the most likely organic causes. Which of the following tests will you include in your initial work-up? A Arterial blood gases B CBC C Chest x-ray D Sputum culture E Urine toxicology screen
The Correct Answer is: E The differential diagnosis of panic disorder includes many cardiac, pulmonary, and endocrine conditions, but in a young woman without any symptoms or signs between episodes, drug use, as detected on a urine toxicology screen (E), is the most likely cause. An EKG might also be considered to rule out a cardiac cause. CBC (B) and sputum culture (D) might be helpful if infection is a consideration, but she has no fever or other signs of infection. Chest x-ray (C) and blood gases (A) would not be helpful without symptoms or signs suggesting particular etiologies.
Q 7.1: What is the first type of motion that is lost as a person develops progressively worsening osteoarthritis of the hip joint? A Abduction B Adduction C External rotation D Flexion E Internal rotation
The Correct Answer is: E The earliest sign of the development of osteoarthritis of the hip is often the loss of internal rotation. As the condition worsens, muscular contractures may develop which hold the affected limb in a flexed and externally rotated position, which has great consequence on the individual's gait and functioning level. Typically a person experiencing this type of osteoarthritis will eventually develop an antalgic gait where the time spent bearing weight on the affected limb is brief due to pain. Also, the gluteus medius (which is a hip abductor and helps stabilize the pelvis) may become weakened as the condition worsens, resulting in an abductor lurch as the trunk of the body sways out over the affected limb when attempting to walk.
Q 32.23: A 73-year-old, male with a 30 pack-year smoking history presents to the clinic with complaints of headache and dizziness. Labs reveal a hemoglobin of 21.3 g/dl, hematocrit of 63%, and platelet count of 498,000. He tests positive for the JAK2 mutation. Which of the following is the most appropriate treatment for the patient's suspected diagnosis? A Allopurinol B Anagrelide C Diphenhydramine D Hydroxyurea E Phlebotomy
The Correct Answer is: E The first-line, and primary treatment for polycythemia vera is chronic phlebotomy. Patients who fail to improve or don't tolerate chronic phlebotomy can be treated with hydroxyurea (D), with anagrelide (B) being a third-line treatment. Allopurinol (A) may be necessary to treat hyperuricemia and gout. Diphenhydramine (C) may be used as symptomatic treatment for pruritus that doesn't improve with phlebotomy.
Q 46.1: What is the treatment of choice for rheumatic fever? A Macrolides B Cephalosporin C Fluoroquinolone D Aminoglycosides E Penicillin
The Correct Answer is: E The goal is to eradicate the Streptococcus bacteria. Penicillins are the drug of choice, with the dose being benzathine penicillin G, 1.2 million units intramuscularly every four weeks as the ideal regimen. Oral penicillin does not absorb as well and has a weaker effect. The alternative to those with penicillin allergies is erythromycin 250 mg BID. Other alternatives are azithromycin or sulfadiazine.
Q 4.8: A 14-year-old girl presents 1 week after the neighbor's cat bit her hand. In the first 3 days after the bite she developed a shallow ulcer at the bite site. Because her parents knew the cat was up to date on shots, they treated the ulcer with topical antibiotics and did not seek medical care. Now, the patient has low-grade fever and headache and feels tired. Axillary nodes on the affected side are swollen. The ulcer on the hand is nearly healed. The best treatment option is A doxycycline 100 mg bid × 21 days B Augmentin 500 mg po bid × 10 days C azithromycin 500 mg po qd × 7 days D acyclovir 400 mg po bid × 10 days E no therapy required
The Correct Answer is: E The history and course of illness are consistent with cat-scratch fever. It is caused by infection with Bartonella henselae. Cat scratch or bite transmits it to humans. Clinical course usually begins with papule or ulcer at the site within a few days of the bite. Fever, headache, and malaise develop 7 to 21 days later. Lymph drainage of the site may result in swollen, tender, and/or suppurative nodes. Clinical diagnosis is the norm but special cultures or biopsy is possible. The symptoms usually resolve spontaneously with no specific therapy required. Complications may include encephalitis or disseminated disease in immunocompromised patients.
Q 97.60: A G4P4 woman delivers a viable infant at 38 weeks gestation by normal spontaneous vaginal delivery. The infant has apgars of 7 and 8 (at 1 and 5 minutes respectively). What is the most crucial time for maternal and fetal physiologic changes to occur? A 30 minutes prior to birth B 60 minutes prior to birth C Birth D 30 minutes after birth E 60 minutes after birth
The Correct Answer is: E The hour after birth is the most critical time for physiologic changes to occur, including maternal fluid shifts, hemorrhage, retained placenta, and fetal lung cardiovascular transitions.
Q 4.4: A 17 year-old male is training for an Ironman triathlon and notes excessive coughing, chest tightness and wheezing when running. Which of the following is the most appropriate treatments for this patient? A Albuterol B Cromolyn C Fluticasone D Ipratropium bromide E Salmeterol
The Correct Answer is: E The mainstay of treatment for exercise-induced asthma are beta-2 agonists. Due to the duration of physical activity this patient should be treated with a long-acting beta-2 agonist, salmeterol (E) instead of albuterol (A). If his symptoms are persistent then addition of a daily inhaled corticosteroid (C) is required, and daily ICS should be added if his use of salmeterol is frequent to avoid asthma-related death associated with the lone use of long-acting beta-agonists. Cromolyn (B) is an additional treatment options for exercise-induced bronchospasm, but is less commonly used compared to beta-2 agonists.
Q 40.2: An infant is born to an HIV-positive mother who received three-drug treatment during pregnancy. Which of the following, if positive, indicates HIV infection in the infant? A HIV ELISA and Western Blot on cord blood B HIV ELISA and Western Blot at 1 month of age C HIV ELISA and Western Blot at 6 months of age D HIV ELISA and Western Blot at 12 months of age E HIV ELISA and Western Blot at 24 months of age
The Correct Answer is: E The median age at which infants no longer show the maternal antibody for HIV is 10 months; by 18 months, they all do not. HIV ELISA and Western blot are not appropriate for testing pediatric patients until after that age. HIV nucleic acid, RNA in plasma, or DNA in blood cells can be detected earlier. Tests for these include polymerase chain reaction (PCR), branched DNA chain assay (bDNA), and nucleic acid sequence-based amplification (NASBA). (
Q 98.26: A 39-year-old man presents to the emergency department with massive hematemesis. His physical examination reveals slight jaundice, palmar erythema, spider angiomas, and marked ascites. Vitals at the time of presentation are as follows: BP: 85/44 mm Hg, P: 122/min, R: 16/min, oxygen saturation: 96%, and T: 99.8°F. Which of the following is the most likely cause of the massive hematemesis? A Peptic ulcer disease B Mallory-Weiss tear C Gastric carcinoma D Arteriovenous malformation E Esophageal varices
The Correct Answer is: E The most common cause of massive upper gastrointestinal bleeding in patients with cirrhosis is esophageal varices. Although 20% of patients with portal hypertension will have bleeding from other causes (peptic ulcer disease, Mallory-Weiss tears, or gastritis), endoscopic evaluation in patients with portal hypertension is necessary for diagnosis and initial therapy.
Q 88.9: A 42-year-old woman has a developed a solid and quite firm thyroid mass that is approximately 1 cm diameter by palpation. She does not have any hoarseness, difficulty breathing or swallowing, or symptoms of thyroid disease. She has never been exposed to radiation and has no history of cancer, nor does she have a family history of thyroid or other cancers. A thyroid panel is within normal limits. What is the next step in the care of this patient? A fine-needle aspiration biopsy of the nodule B observation to see if the nodule increases in size C radionuclide thyroid scan D resection of the nodule E ultrasonography of the neck
The Correct Answer is: E The next step in the care of this woman is neck ultrasonography to determine its size and characteristics; this technique is preferred to CT or MRI scanning because it is accurate, noninvasive, far less costly. Should the ultrasound results be suspicious for malignancy, a fine-needle aspiration biopsy (A) should follow. A thyroid mass 1 cm in diameter or larger must be evaluated rather than observed (B). Radionuclide scanning (C) would be indicated if the patient's TSH were low but is otherwise limited in determining whether a thyroid mass is malignant. Resection (D) would be inappropriate until further testing is done. Fitzgerald PA, Endocrine Disorders, in Current Medical Diagnosis and Treatment, 52 nd ed. 2013.
Q 38.6: A 4-month-old male presents for a well child check. He is healthy, and the mother feels that the child is eating and growing well. On examination, there is no evidence of cyanosis. The peripheral pulses are normal and equal. There is a medium-pitched harsh pansystolic murmur heard best at the left sternal border at the fourth intercostal space. There is no heave or thrill present. The murmur radiates over the entire precordium and the S2 is physiologically split. What is the most likely diagnosis? A Atrial septal defect B Coarctation of the aorta C Patent ductus arteriosus D Tetralogy of fallot E Ventricular septal defect
The Correct Answer is: E The patient in this scenario is exhibiting the classic signs of a ventricular septal defect. An atrial septal defect has a fixed, widely split S2, with a right ventricular heave as well as a systolic ejection murmur, which is best heard at the left sternal border second intercostal space. Coarctation of the aorta has absent or diminished femoral pulses and a blowing systolic murmur. A patent ductus arteriosus murmur is classically described as a rough machinery systolic murmur, and there are bounding pulses with a widened pulse pressure. Tetralogy of fallot can have associated cyanosis, with hypoxemic spells during infancy, easy fatigability, and dyspnea on exertion. Tetralogy of fallot has the presence of a right ventricular lift and a rough, systolic ejection murmur that is present along the left sternal border in the third intercostal space that radiates to the back. Aortic stenosis has a harsh systolic ejection murmur present at the right sternal border, and associated thrill in the carotid arteries.
Q 97.43: You are caring for a 29-year-old G3P2 at 39 weeks gestation, who has been laboring for 6 hours. She is a diet-controlled diabetic. Her last child was 9 pounds 8 ounces. She has been completely dilated for 2 hours, and the fetal head is at a plus 2 station, which is unchanged. What is the next most appropriate course of action? A Begin oxytocin B Vacuum extraction C High forcep extraction D High forcep rotation E Cesarean section
The Correct Answer is: E The patient is a diabetic with a history of a macrosomic infant; the likelihood of macrosomia in this infant is significant. Instrument delivery is not recommended if macrosomia is suspected. By definition, she has had an arrest of descent of the fetal head and one should be highly suspicious for macrosomia, in which case a c-section is the preferred method of delivery.
Q 11.6: A 48-year-old female presents with a gradual progression of fatigue, pallor, and dyspnea on exertion over the last few months. Initial CBC results show a hemoglobin of 10.2 mg/dL, hematocrit of 31%, an MCV of 74 fL, and a reticulocyte count of 0.1%. After treatment for her anemia begins, which of the following diagnostic studies can be ordered first (earliest) to demonstrate a response to therapy? A Bone marrow aspirate B Ferritin C Hemoglobin/hematocrit D Iron binding capacity E Reticulocyte count
The Correct Answer is: E The patient most likely has an underlying iron deficiency anemia (IDA) which requires confirmation and then evaluation for sources of chronic blood loss. IDA is treated with iron replacement (e.g., ferrous sulfate) and response to therapy can be assessed in 7-10 days via a reticulocyte count. The elevation of the reticulocyte count (after initial reticulocyte count was relatively low) at this time demonstrates the return of normal hemoglobin production (E). Bone marrow aspirate (A) would be an inappropriately invasive test to perform in this scenario. The patient's hemoglobin, hematocrit (C), MCV, and iron studies (B and D) will take a couple to a few months to return to normal.
Q 23.6: A 65-year-old man presents to the clinic complaining of fatigue, weakness, paresthesias, and an unsteady gait. Physical exam reveals decreased vibratory sensation and lack of proprioceptive feedback, noted on Romberg testing. CBC reveals hemoglobin of 10.7 g/dl, hematocrit of 32.1% and MCV 117.6 fL. Which of the following is the most appropriate treatment? A Erythropoietin B Ferrous sulfate C Folic acid D Metformin E Vitamin B12
The Correct Answer is: E The patient most likely has vitamin B 12 deficiency, as evidenced by the macrocytic anemia and neurological signs and symptoms. The patient should be treated with vitamin B 12 . Anemia secondary to chronic kidney disease (A) typically presents as a normocytic, normochromic anemia. Iron deficiency anemia (B) presents as a microcytic, hypochromic anemia. Folate deficiency (C) typically presents as a macrocytic anemia, but without the classic neurological symptoms consistent with vitamin B 12 deficiency.
Q 32.17: An 11-month-old African-American male presents to the pediatric office with lethargy, jaundice and splenomegaly. A CBC reveals hemoglobin of 8.0 mg/dl and a hematocrit of 25%. Peripheral smear appearance is available below. Which of the following is the most likely diagnosis? A Acute lymphocytic leukemia B Alpha thalassemia minor C Beta thalassemia minor D G6PD deficiency E Sickle cell disease
The Correct Answer is: E The patient's clinical presentation is most consistent with sickle cell disease and is confirmed by the presence of sickle cells and target cells on the peripheral smear. The patient lacks white blood cell abnormalities consistent with acute lymphocytic leukemia (A). Alpha thalassemia minor (B) is typically an asymptomatic, mild microcytic anemia with target cells present of peripheral smear. Beta thalassemia minor (C) typically presents with hematocrits between 28 and 40% and target cells on peripheral smear; however, the presence of sickle cells only occurs in the presence of combined sickle cell-beta thalassemia. G6PD (D) deficiency results in a hemolytic anemia with marked anemia and the presence of "bite" cells on peripheral smear.
Q 102.2: A 42-year-old female returns for a follow up of her hypothyroidism and admits to increased hair loss and dry skin. Her thyroid-stimulating hormone (TSH) is 20 mU/L. She is currently on levothyroxine 25 mcg daily which she has taken as directed for the past 6 months. What treatment do you recommend for this patient at this time? A change to methimazole B radioactive iodine therapy (131I therapy) C decrease the dose of levothyroxine to 12.5 mcg QD D add propylthiouracil E increase the dose of levothyroxine to 50 mcg QD
The Correct Answer is: E The patient's symptoms and elevated TSH suggests that she requires more thyroid hormone replacement. Increasing the dose to 50 mcg daily (by 25 mcg) is a reasonable treatment plan. Decreasing the dose (already very low) would likely cause worsening symptoms of hypothyroidism. Choices (A), (B), and (D) are treatments for hyperthyroidism. (McPhee and Papadakis, 2011, Chapter 26)
Q 5.4: A 36 year-old woman with no significant past medical history presents with gradual onset of dyspnea and fatigue leading to an episode of "fainting" this morning. Physical exam reveals increased jugular venous pressure, weak carotid pulses, clear lungs, and a loud S2. What is the most likely diagnosis? A Aortic stenosis B Cardiac tamponade C Mitral Regurgitation D Pulmonary fibrosis E Pulmonary hypertension
The Correct Answer is: E The patients symptoms are due to decreased cardiac output resulting from decreased preload associated with pulmonary hypertension (E). Aortic stenosis (A) presents more commonly in geriatric patients who present with a murmur. Cardiac tamponade (B) can decrease cardiac output, but would lead to decreased heart sounds. Mitral regurgitation (C) would cause pulmonary edema and rales in conjunction with increased jugular venous pressure. Pulmonary fibrosis (E) is unlikely in this patient with normal lung sounds.
Q 23.2: A 58-year-old female presents to the outpatient clinic complaining of 1 week of rhinorrhea, nonproductive cough, and hoarseness. On physical exam she is noted to have erythematous nasal mucosa and decreased phonation without significant nasal discharge, sinus tenderness, pharyngeal erythema, or lymphadenopathy. Which of the following is the most likely causative organism for this patient's condition? A Clostridium diphtheriae B Group A streptococcus C Moraxella catarrhalis D Mycobacterium tuberculosis E Rhinovirus
The Correct Answer is: E The presence of acute hoarseness associated with an upper respiratory infection is consistent with laryngitis, which may be caused by all of the organisms above (A-E), but most likely has a viral (E) etiology.
Q 118.3: A 58-year-old male is diagnosed with a gastrin-secreting gut neuroendocrine tumor. What is the medical term for this condition? A Ascending cholangitis B Celiac disease C Clostridium difficile colitis D Irritable bowel syndrome E Zollinger-Ellison syndrome
The Correct Answer is: E The question is referring to a gastrinoma, which is also called Zollinger-Ellison syndrome. Ascending cholangitis results from blockage of the common bile duct, which leads to inflammation and fibrosis. Celiac disease is a small bowel enteropathy due to the ingestion of gluten, which leads to malabsorption of nutrients. C. difficile colitis is an overgrowth of this bacteria, which results in diarrhea. The cause of irritable bowel syndrome may be due to abnormal motility, visceral hypersensitivity, or enteric infection, but is not due to a gastrin secreting tumor. (McPhee SJ, Papadakis MA. Current Medical Diagnosis & Treatment, 2010, p. 556)
Q 14.9: You are evaluating a patient who is having decreased sensation to his arm after he sustained a head on injury while playing football. He is stating that his right arm has decreased sensation that goes into the hand, but he is slightly vague on being descriptive. As you perform the physical examination, what part of the body would exhibit sensation for the C7 component of the brachial plexus? A Lateral forearm B Lateral upper arm C Medial forearm D Medial upper arm E Third Finger
The Correct Answer is: E The sensory nerve emanating from the C7 level is responsible for sensation of the third finger. The lateral forearm is supplied by C6. The lateral upper arm is supplied by C5. The medial upper arm is supplied T1 and the medial forearm is supplied by C8.
Q 98.44: You are evaluating a patient who is having decreased sensation to his arm after he sustained a head on injury while playing football. He is stating that his right arm has decreased sensation that goes into the hand, but he is slightly vague on being descriptive. As you perform the physical examination, what part of the body would exhibit sensation for the C7 component of the brachial plexus? A Lateral forearm B Lateral upper arm C Medial forearm D Medial upper arm E Third Finger
The Correct Answer is: E The sensory nerve emanating from the C7 level is responsible for sensation of the third finger. The lateral forearm is supplied by C6. The lateral upper arm is supplied by C5. The medial upper arm is supplied T1 and the medial forearm is supplied by C8. (
Q 98.59: A 46-year-old male is evaluated for a fall from approximately 15 feet, landing on his legs. There is intense pain and swelling to the lower extremity, with decreased sensation. What bone is fractured and is most commonly associated with acute compartment syndrome? A Femur B Patella C Navicular D Talus E Tibia
The Correct Answer is: E The tibia is the most common bone fracture that leads to acute compartment syndrome. It can also occur after other long bone fractures of the arms and legs. While it does occur in the hands and feet, it is unlikely that the fracture of a small carpal bone (scaphoid) or tarsal bone (talus) would result in compartment syndrome. Damage to the hyoid bone in the neck and the patella in the knee are not associated with compartment syndrome.
Q 8.8: The most commonly fractured long bone in both adults and children is which of the following? A Femur B Fibula C Humerus D Radius E Tibia
The Correct Answer is: E The tibia is the most commonly fractured long bone in the body for both adults and children. The fractures are often the result of sporting activities in the young and may occur from a simple fall in the elderly - especially those with osteoporosis. Motor vehicle accidents are another common cause of tibial fractures. Open or complex tibial fractures are sometimes associated with compartment syndromes, infection and neurovascular compromise. The femur is the strongest of the long bones and generally only sustains fractures when exposed to extreme stress, such as that experienced in a motor vehicle collision or industrial accident. Fibular fractures commonly occur with a direct below to the lateral lower leg or with extreme ankle rotational forces or excessive inversion. A high percentage of ankle fractures involve the fibula, especially in older women. Humerus fractures are relatively rare in adults, but are the second most common fractures to occur at birth - behind only the clavicle in frequency. The radius is the most commonly fractured bone in the upper extremity, but still less common in frequency than the tibia. Falls on an outstretched arm are a common mechanism for the injury.
Q 91.2: A 45-year-old woman with recent diagnosis of rheumatoid arthritis has begun treatment with celecoxib. She has been on this medication for 3 months and notes that her pain continues. Early signs of joint involvement are present in the patient's hands. Which of the following medications is the most appropriate to add to her treatment? A aspirin B rituximab C etanercept D leflunomide E methotrexate
The Correct Answer is: E The treatment of rheumatoid arthritis (RA) is aimed at reduction of pain, preservation of function, and prevention of deformity. Although non-steroidal anti-inflammatory drugs (NSAIDs) provide symptomatic relief, they do not alter progression or prevent erosion of the joint. Consequently, in addition to NSAID therapy, disease-modifying anti-rheumatological drugs (DMARDs) should also be initiated as soon as the diagnosis is confirmed. The most common initial DMARD used as treatment of choice in RA is methotrexate. Aspirin should not be added because of the increased risk of gastrointestinal side effects as well as having no effect on altering RA disease progression. Rituximab is a biological DMARD and is indicated to be added in patients with RA refractive to treatment with combination therapy of methotrexate and a tumor necrosis factor inhibitor (TNF). Etanercept is a TNF inhibitor. This class of medication is often added in patients with RA who are not responding to methotrexate therapy alone. Leflunomide is a pyrimidine synthesis inhibitor that is approved for the treatment of RA; however, it is contraindicated for use in premenopausal women secondary to its carcinogenic and teratogenic potential. (Hellmann and Imboden, 2008, pp. 722-725) Hellmann DB , Imboden JB. Arthritis and musculoskeletal disorders. In: Tierney LM , McPhee SJ , Papadakis MA, eds. 2008 Current Medical Diagnosis and Treatment. New York, NY: McGraw-Hill; 2008:703-756.
Active elbow extension is primarily controlled by which muscle(s)? A Anconeus B Biceps C Brachialis D Brachioradialis E Triceps
The Correct Answer is: E The triceps are the primary muscles that produce active elbow extension. The anconeus muscle is known as a secondary elbow extensor. The biceps and brachialis muscles are primary muscles of elbow flexion and the brachioradialis is a secondary elbow flexor.
Q 25.4: Active elbow extension is primarily controlled by which muscle(s)? A Anconeus B Biceps C Brachialis D Brachioradialis E Triceps
The Correct Answer is: E The triceps are the primary muscles that produce active elbow extension. The anconeus muscle is known as a secondary elbow extensor. The biceps and brachialis muscles are primary muscles of elbow flexion and the brachioradialis is a secondary elbow flexor. (
Q 97.22: A 30-year-old G1P0 woman who is 15 weeks pregnant undergoes "triple screening." The maternal alphafetoprotein (AFP), human chorionic gonadotropin (hCG), and unconjugated estriol (uE3) levels are all lower than normal. This suggests which of the following possible problems with the fetus? A cystic fibrosis B Down syndrome C homocystinuria D G6PD deficiency E trisomy 18
The Correct Answer is: E The triple screen detects possible Down syndrome and trisomy 18. When the fetus has Down syndrome (B), the AFP and uE3 are low and the hCG is high, while trisomy 18 is suggested by low values in all three. A positive screen must be followed up by fetal karyotyping. Cystic fibrosis (A), homocystinuria (C), and G6PD deficiency (D) are not detected by the triple screen.
Q 29.5: A 64-year-old male presents with right knee pain and stiffness while walking and going up and down stairs for the past six months. He states the pain persists all day long and is relieved with rest. There is no history of trauma and his past medical history is unremarkable. An x-ray of the right knee is performed and the following is observed: What is being identified by the white arrow? A Osteophyte B Salter-Harris Type I fracture C Torn anterior cruciate ligament D Osteosarcoma E Narrowed joint space
The Correct Answer is: E The white arrow is identifying a narrowed joint space (E) on the medial side of the knee joint seen in osteoarthritis. The black line depicts sclerosis of the bone in the medial compartment, which is evidence of cortical thickening. The white wedge is showing the formation of osteophytes (A) in the medial femur as a result of the osteoarthritis. Salter-Harris fractures (B) are identified in adolescents with open growth plates. Osteosarcoma (D) would show a boney lesion. Soft tissue injuries (C) would not usually be identifiable on x-ray and would need further imaging, such as a CT or MRI.
Q 66.10: A 49-year-old man presents to the office complaining of general malaise with muscle aches, anorexia, fever, and severe pain over his anterior neck radiating to his ears. He states that he was ill about 2 weeks ago with a sore throat, but it resolved within a few days. On palpation, the thyroid gland is enlarged and tender. His laboratory workup shows a high T4 level and increased erythrocyte sedimentation rate (ESR). What is the most appropriate therapy for this patient's disease? A levothyroxine sodium B PTU therapy C radioiodine ablation D surgery E supportive therapy only
The Correct Answer is: E This is subacute, painful thyroiditis. This is a self-limiting disorder that at most requires symptomatic therapy. In mild cases, analgesics (ASA) are sufficient for pain relief and to decrease the inflammation. Prednisone may bring more relief if needed. Transient hypothyroidism should be treated as well. (Ladenson and Kim, 2008, p. 1708) Ladenson P , Kim M. Thyroid. In: Goldman L , Ausiello D, eds. Cecil Textbook of Medicine. 23rd ed. Philadelphia, PA: WB Saunders; 2008.
Q 110.14: A 68-year-old male presents to the clinic stating that his wife has noticed a resting tremor in his right hand, and that his gait is more shuffling over the past six months. Which of these findings on physical exam is most likely for the suspected diagnosis? A Aphasia B Atonia C Chorea D Hyperreflexia E Masked facies
The Correct Answer is: E This patient exhibits classic findings of Parkinson's disease. Parkinson's is a nervous system disorder due to decreased dopamine, resulting from a degeneration of the dopaminergic nigrostriatal system. Symptoms may include a combination of tremor, rigidity, bradykinesia, progressive postural instability, slowing of automatic movements, gait changes, decreased facial expression, speech changes, and cognition deficits. Muscle strength and reflexes are typically preserved. Chorea, an irregular, rapid, and involuntary movement, is typically seen with Huntington's and a variety of other disorders. Parkinson's must be differentiated from other nervous system disorders. (McPhee SJ, Papadakis MA. Current Medical Diagnosis & Treatment 2011, Chapter 24, Nervous System Disorders)
Q 117.3: A 52-year-old male with chronic alcoholism is brought to the emergency department by his family, due to his acting differently for several days. A physical exam reveals nystagmus, eye muscle weakness, global confusion, retrograde amnesia, and ataxia. Which of the following is the most likely etiology of the diagnosis? A Cerebrovascular accident B Hypoxemia C Uremia D Vitamin B12 deficiency E Vitamin B1 (thiamine) deficiency
The Correct Answer is: E This patient exhibits classic symptoms associated with Wernicke encephalopathy. Wernicke encephalopathy is due to vitamin B1 (thiamine) deficiency. In the United States, this condition occurs most frequently in chronic alcoholics, but it may be seen in any condition affecting thiamine levels. Each of the other etiologies listed may also cause neurologic findings, and should be considered in the differential diagnosis and evaluation of this patient. (Simon RP, Greenberg DA, Aminoff MJ. Clinical Neurology, 7e, Chapter 1. Disorders of Cognitive Function)
Q 95.2: A patient presents with clinical features of thrombotic thrombocytopenic purpura. They also have an associated diarrhea positive hemolytic uremic syndrome. Which is the best treatment option for this patient? A Aspirin and dipyridamole B Glucocorticoids C Plasma exchange D Splenectomy and prostacyclin E Supportive therapy and dialysis
The Correct Answer is: E This patient has a diarrhea-associated hemolytic uremic syndrome with clinical features of TTP. D+HUS is associated with infectious etiologies, usually through food borne illnesses. Treatment consists of supportive therapy, treating the underlying infection, and dialysis if renal function warrants. All other therapies listed are used in the treatment of TTP or diarrhea negative HUS. (Lichtman et al., Williams Hematology 8e, Chapter 133, Antibody-Mediated Thrombotic Disorders: Thrombotic Thrombocytopenic Purpura and Heparin-Induced Thrombocytopenia)
Q 82.4: A 50-year-old male with an elevated WBC of 143,000, hematocrit 47% (45 to 52%), platelet count of 247,000 (150,000 to 4000,000), and BCR/ABL found in peripheral blood. He complains of fatigue, night sweats, and abdominal fullness. The best initial course of therapy would include which of the following? A Bone marrow transplant B Cytarabine C Dasatinib D Hydroxyurea E Imatinib mesylate
The Correct Answer is: E This patient has classic signs of chronic myelogenous leukemia. The first line of therapy for patients in the chronic phase of this disease is Imatinib mesylate, starting at 400 mg/day. For patients that demonstrate to Imatinib alone, Dasatinib may be an adjunct therapy. Hydroxyurea and Cytarabine were used prior to formulation of Imatinib, and may play a role for resistant CML. Bone marrow transplant would be reserved for patients that are resistant to current therapy or are in relapse. (Lichtman et al., Williams Hematology 8e, Chapter 90, Chronic Myelogenous Leukemia and Related Disorders)
Q 80.12: A patient being treated for Burkitt's lymphoma is found to have hyperkalemia, hyperphosphatemia, hypocalcemia, a metabolic acidosis, hyperuricemia, and appears to be going into renal failure after the first dose of chemotherapy. To prevent or minimize this finding, the best therapy is to start which of the following? A Allopurinol and rasburicase B Colchicine and monitored hydration of at least 3 L/day C Continuous venovenous hemofiltration and colchicine D Continuous venovenous hemofiltration and hydration E Monitored hydration of at least 3 L/day and allopurinol
The Correct Answer is: E This patient has developed a tumor lysis syndrome. The usual prophylactic therapy is carefully monitored hydration of at least 3 L of saline per day, and either allopurinol or rasburicase to decrease serum uric acid concentration and hyperuricosuria. Allopurinol and rasburicase alone are not enough to prevent TLS. Colchicine is a gout medication and not appropriate in this setting. Continuous venovenous hemofiltration is useful in allowing concomitant full-dose chemotherapy and preventing tumor lysis and renal failure, but is not the preventative treatment prior to starting chemotherapy. (Lichtman et al., Williams Hematology 8e, Chapter 104, Burkitt Lymphoma)
Q 32.13: An 82-year-old male presents to the emergency department with a 2-day history of weakness, fatigue, and mild shortness of breath. His past medical history includes hypertension, arthritis, diabetes, and hypercholesterolemia. His medications include metoprolol, glyburide, simvastatin, and acetaminophen. On physical examination, the vital signs reveal T 97.7, P 40, R 15, BP 84/60. His neck exam reveals no jugular venous distention, lungs are clear to auscultation, and cardiac has a regularly irregular bradycardia rhythm. There is no evidence of edema to the extremities. The patient is placed on a telemetry monitor, which reveals the image shown. Based on the information provided, what is the most appropriate medical therapy for this patient? A Digoxin B Diltiazem C Amiodarone D Increase metoprolol E Stop metoprolol
The Correct Answer is: E This patient has developed complete heart block as evidenced by the telemetry rhythm strip. While the patient will require a pacemaker as the definitive treatment, the pharmacologic intervention that must be done is to halt the use of beta blockers (E), which will contribute to the patient's slower rate. The other choices (A, B, C, and D) would all exacerbate the condition the patient is already in, and make the clinical situation worse and potentially fatal.
Q 39.26: A 1-year-old boy is brought to the emergency department by his parents, who state that the child refuses to walk or crawl and begins crying when they stand him. He seems calm while lying on the examination table. Vitals are as follows: Temp: 38°C, HR: 70, RR: 15. Bruising is noted in several places. His parents deny trauma, but have noticed that he bruises easily. What other physical finding would you expect? A Conjunctival hemorrhages secondary to shaken baby syndrome B Pain response over the wrists secondary to passive range of motion C Pain response with passive range of motion to the hip secondary to slipped epiphysis D Pharyngitis and sand paper rash secondary to a staph infection E Swelling and warmth over the knee secondary to hemarthroses
The Correct Answer is: E This patient has hemophilia A. Hemarthroses usually occur when an affected child begins to walk. Due to his hemophilia, easy bruising can occur. Hemarthroses can cause low-grade fevers without infection being present, so choice D is incorrect. Wrist joints are less involved then knees, ankles, and elbows.
Q 109.13: You are evaluating a generally healthy 75-year-old male patient with a history of hypertension and persistent elevated blood pressure, despite atenolol 25 mg QD. He also complains of some urinary frequency, urgency, and nocturia over the past year and is bothered by these symptoms. His blood pressure today is 146/90 P = 56. His urinalysis is negative. His prostatic-specific antigen (PSA) from his last visit 4 months ago was 1. His rectal examination reveals a smooth, enlarged prostate without masses. What medication, in addition to atenolol, would be the best choice to treat this patient at this time? A carvedilol B lisinopril C amlodipine D hydrochlorothiazide E terazosin
The Correct Answer is: E This patient has uncontrolled hypertension as well as symptomatic benign prostatic hypertrophy (BPH). Although the addition of lisinopril and amlodipine are great options for blood pressure control they will not improve his symptoms of BPH. The addition of carvedilol, a beta blocker, to atenolol (also a beta blocker) is potentially dangerous, especially considering the bradycardic pulse. The addition of HCTZ is an option for further control of his blood pressure, but has the potential of increasing urinary frequency. Due to effects on blood pressure and BPH, the best choice for this patient would be a trial of terazosin, an alpha blocker. (McPhee and Papadakis, 2011, Chapter 11)
Q 112.6: A 23-year-old male presents with uncontrolled mucocutaneous bleeding following a wisdom tooth extraction. Traditional methods of controlling the bleeding have not significantly reduced the bleeding. On history, he reports epistaxis that takes longer than usual to stop bleeding. He also states that he seems to bruise more frequently than his friends. What is the most likely diagnosis? A Acute myelogenous leukemia B Aplastic anemia C Idiopathic thrombocytopenic purpura D Thalassemia E Von Willebrand's disease
The Correct Answer is: E This patient has von Willebrand's disease. Common clinical symptoms include mucocutaneous bleeding, epistaxis, bruising, and hematomas (and menorrhagia in women). Idiopathic thrombocytopenic purpura is the only other diagnosis that is a coagulation disorder, and typical presentation includes purpura and petechiae to the lower extremities. (Lichtman et al., Williams Hematology 8e, Chapter 127, von Willebrand Disease)
A 44-year-old female presents to the emergency department with a right sided headache. She states the headache is located on the right temple region, is non-radiating, and does not cause photophobia. She is otherwise healthy and has no reported medical problems and has only taken acetaminophen for the pain, which has minimal relief. On physical exam she is alert, awake, and oriented. Her vitals are T 98.8, P 78, R 18, and BP 128/76. Her head is normocephalic, atraumatic, and pupils are equal and reactive. She has tenderness to the right temporal area of the temporal artery, and there is no noted swelling, redness, or abnormalities noted. There is no tenderness to the cervical muscles. She exhibits a non-focal neurological exam. A biopsy of the temporal artery reveals inflammatory cells of the artery wall. Based on the history and biopsy results, what is the best initial treatment for this patient? A Cyclosporine B Azathioprine C Methotrexate D Ibuprofen E Prednisone
The Correct Answer is: E This patient is exhibiting a case of temporal arteritis. She has the signs and symptoms that are classic in nature and do not fit into the realm of the other headaches, all of which usually have different characteristics on history and physical exam. In this instance, the use of oral steroids will be the best choice in management of the condition. Anti-inflammatories (C, D) and immunosuppressant agents (A, B) will not be as effective as oral prednisone (E), and the long-term outcomes are improved with steroids.
Q 67.4: A 66-year-old male with a history of hypertension, diabetes mellitus, and hypercholesterolemia presents by emergency medical services (EMS) to the emergency department complaining of severe chest pain with radiation into his back. The patient states that he was feeling well in the morning, but while performing some light activity he felt a "ripping" sensation in his back, which he initially thought was a pulled muscle. The pain continued and the patient started to have chest pain, shortness of breath, and lightheadedness. On initial examination the patient is still in pain, pale, diaphoretic, and has a blood pressure of 85/40. His chest is clear to auscultation, and he has a 3/6 diastolic murmur best appreciated at the base of the heart. Given this clinical scenario, what is the best test to definitively diagnose this medical problem? A Chest x-ray B Transthoracic echocardiography C Transesophageal echocardiography D Cardiac catheterization E Computed tomography
The Correct Answer is: E This patient is exhibiting a history and physical examination that is consistent with a thoracic aneurysm. The patient's history of hypertension, along with the "ripping" sensation in his back and hypotension give a clinical presentation that is suggestive of a thoracic aneurysm dissection. Given this clinical situation, the best test to evaluate for a potential dissection is by computed tomography (E). This test is sensitive enough to determine if there is a luminal irregularity. While echocardiography (B and C) may be able to show evidence of an aneurysm, it is not specific enough to show all the areas of an aneurysm. In this situation a chest x-ray (A) would not give enough specificity to appropriately diagnose an aneurysm, as well as a cardiac catheterization (D). Source: http://www.accessmedicine.com/content.aspx?aID=3651494
Q 32.21: A 39-year-old male presents to the clinic with a concern for a sexually transmitted infection. He suspects his wife has been having an affair. He denies any penile pain or discharge. This is the third time he has been into the office with this concern in the past four months. All of the STD tests have been negative thus far. Upon further questioning, he is noted to be defensive, extremely sensitive, and quite secretive. He also has suspicions that his neighbor is throwing fallen leaves from a tree into his yard. This patient is exhibiting symptoms most consistent with which of the following personality disorders? A Antisocial B Borderline C Narcissistic D Obsessive compulsive E Paranoid
The Correct Answer is: E This patient is exhibiting symptoms consistent with paranoid personality disorder. Antisocial personality disorder is characterized by selfishness, callousness, promiscuity and impulsive behavior, inability to learn from experience and legal problems. The clinical findings of borderline personality disorder include impulsiveness, unstable and intense interpersonal relationships, lack of self control, suicidal, aggressive behavior and a high drug abuse rate. Clinical findings of obsessive-compulsive disorder include being a perfectionist, egocentric, indecisive, with rigid thought patterns and a need for control. Narcissistic personality disorder presents with the clinical findings of grandiosity, a preoccupation with power, lacking interest in others, and excessive demands for attention.
A 68-year-old male presents with the complaint of palpitations in the center of his chest over the last few hours. The symptoms come and go, and last anywhere from 30 seconds to a few minutes. There is some associated lightheadedness with these episodes, and occasionally there is mild shortness of breath. While examining the patient, he has another episode. During this time, the patient's vital signs reveal a pulse of 170 and a blood pressure of 118/69. Based on this history and the findings on the exam and rhythm strip shown, what is the best treatment for this patient? A Diltiazem B Digoxin C Lisinopril D Metoprolol E Amiodarone
The Correct Answer is: E This patient is having non-sustained ventricular tachycardia (NSVT). Based on the clinical presentation, this patient can benefit from an infusion of amiodarone. The dose is normally a 150mg bolus, followed by a drip of 1mg/min for the first 6 hours, then followed by 0.5mg/min for the next 18 to 24 hours. The other agents would not be able to maintain a rhythm for this type of abnormality.
Q 79.2: A 43-year-old male farmer is seen in the oncology clinic for his next round of chemotherapy for acute myelogenous leukemia. He complains of a persistent cough over the last three weeks, but has not run a fever at home. Today, his temperature is 37.4°C, HR is 67, RR is 18, and BP is 120/72. Lung examination is clear to auscultation bilaterally, no lymphadenopathy is noted, color is pale, there is good capillary refill, and the rest of the examination is negative. A CT is obtained, and a RUL wedge-shaped infiltrate at the periphery and a 1 cm cavitary lesion is noted. The best course of treatment would include which of the following? A Acyclovir B Gentamicin C Cefepime D Valacyclovir E Voriconazole
The Correct Answer is: E This patient is neutropenic following chemotherapy induction for the acute myelogenous leukemia. Likely infectious etiologies include gram negative and positive organisms, viral and fungal etiologies. A classic finding on CT is a cavitating lesion consistent with a fungal infection. The best treatment is voriconazole, an antifungal treatment. Acyclovir and valacyclovir are used for antiviral treatments. The others are antibiotics for gram negative organisms. (Lichtman et al., Williams Hematology 8e, Chapter 89, Acute Myelogenous Leukemia) (Lichtman et al., Williams Hematology 8e, Chapter 22, Treatment of Infections in the Immunocompromised Host)
Q 70.10: A 29-year-old woman presents in July to your office with symptoms of palpitations, sore neck, and excessive sweating, despite using her air conditioner all the time. No surgical or trauma history is noted. She is currently not taking any medications. Vitals include the following: BP = 124/68, pulse = 110 beats per minute, respirations = 18 per minute, and temperature = 101 o F orally. Upon exam, her thyroid is mildly enlarged without nodules, and severely tender. No local erythema or heat is noted. What is the most likely diagnosis in this patient? A Thyroid abscess B Papillary thyroid cancer C Hashimoto's thyroiditis D Graves' disease E Subacute thyroiditis
The Correct Answer is: E This patient is presenting with signs and symptoms of hyperthyroidism, most likely caused by subacute thyroiditis. The enlarged and tender thyroid, along with the fever, are classic findings in this condition. Choice A, thyroid abscess, is a localized infection of the thyroid. It would present with heat, swelling, and erythema in the area of tenderness. Choice B, papillary thyroid cancer, commonly presents with either a local non-tender thyroid nodule or as lymph node metastasis. Choice C, Hashimoto's thyroiditis, is a cause of chronic thyroiditis and is associated with a clinical presentation of hypothyroidism. Choice D, Graves' disease, is the most common cause of hyperthyroidism, but does not present with thyroid tenderness and fever. (Cooper et al., 2007, Chapter 8)
Q 81.9: A 52-year-old man with hypertension associated with recent unexplained weight loss presents with fever, malaise, and gradual onset of pain and weakness of his leg muscles for the past month. Physical examination reveals a mottled reticular pattern overlying portions of both calves and an area of ulceration with surrounding induration on the left lateral malleolus. Initial laboratory results reveal mild normochromic anemia, leukocytosis, and elevation of C-reactive protein, BUN, and creatinine. Which of the following is the most appropriate diagnostic evaluation to confirm the suspected diagnosis? A HLA-B27 typing B rheumatoid factor C MRI of sacroiliac joints D antinuclear antibodies test E tissue biopsy of area of induration
The Correct Answer is: E This patient most likely has polyarteritis nodosa (PN). A major obstacle in making the diagnosis is the absence of a disease-specific serological test. The diagnosis requires confirmation with either a tissue biopsy or angiogram. HLA-B27 antigens are not associated with the suspected diagnosis. While classic PN will have low titers of rheumatoid factor and antinuclear antibodies, both are nonspecific findings and will not confirm the diagnosis. An MRI of the sacroiliac joints is indicated in evaluation of the early stages of suspected ankylosing spondylitis and plays no role in the evaluation of PN. (Hellmann and Imboden, 2008, pp. 738-739) Hellmann DB , Imboden JB. Arthritis and musculoskeletal disorders. In: Tierney LM , McPhee SJ , Papadakis MA, eds. 2008 Current Medical Diagnosis and Treatment. New York, NY: McGraw-Hill; 2008:703-756.
Q 82.5: A 65-year-old woman presents to the office with decreased hearing, and pain over her sternum, pelvis, and her right tibial tubercle. On x-ray, the involved bones are noted to be expanded and denser than normal. Her serum calcium and phosphorus levels are normal, but serum alkaline phosphatase level is markedly elevated. Which of the following would be the appropriate initial treatment for this patient? A ibuprofen 600 mg po every 6 hours B indomethacin 25 mg po tid C meclizine 25 mg po tid D methotrexate 7.5 mg po qd E tiludronate 400 mg po qd
The Correct Answer is: E This patient's signs and symptoms are consistent with Paget disease of bone. Biphosphates have become the treatment of choice for this disease. Tiludronate, taken orally for 3 months, is very effective in treatment of this disease. (Fitzgerald, 2009, pp. 1018-1020) Fitzgerald PA. Endocrine diseases. In: McPhee SJ , Papadakis MA, eds. Current Medical Diagnosis and Treatment. 48th ed. New York, NY: McGraw-Hill; 2009.
Q 44.2: A 14-year-old boy is evaluated for headaches. When he greets the PA, his hands are sweaty and have a soft, doughy consistency and he has a remarkable low, husky voice for his age. He is extremely tall, much more so than either of his parents. His father says no one in either family has ever been this tall, and his mother says they can't find shoes wide enough for his feet locally and have to order online. His teeth have wide spaces between them. He has previously been healthy, although he has not had medical care since his pre-K physical examination. What is the most likely underlying cause of this condition? A Carney syndrome B hypothalamic tumor C lymphoma D McCune-Albright syndrome E pituitary adenoma
The Correct Answer is: E This presentation suggests acromegaly/gigantism, which is almost always caused by a pituitary adenoma. It rarely occurs due to ectopic secretion of GHRH or GH that is being secreted by a hypothalamic tumor (B) or lymphoma (C), or may be part of Carney (A) or McCune-Albright (D) syndromes.
Q 98.10: A 42-year-old woman comes in with a "lump in her neck." When she was a teenager, she underwent radiation treatment for "some kind of tumor" in her neck. Ultrasound reveals a 1.5 cm lesion in the left lobe of the thyroid; biopsy confirms papillary carcinoma. Thorough evaluation reveals no evidence of metastasis. Which of the following is the initial treatment of choice for her? A chemotherapy B lobectomy with isthmectomy C radiation therapy D radioactive iodine therapy E total thyroidectomy
The Correct Answer is: E Total or near-total thyroidectomy is indicated for this woman. No chemotherapy (A) is available for thyroid cancer. Lobectomy with isthmectomy is reserved for papillary carcinoma that is less than 1 cm in size in persons with no history of radiation exposure and no evidence of metastasis. Radiation therapy (C) is used to treat bone metastasis and anaplastic carcinoma. Radioactive iodine therapy (D) may be used following thyroidectomy to ablate any remnant of the gland and to treat cancer that has metastasized or is otherwise high risk.
Q 119.8: Which of the following is the greatest risk associated with a patient suffering a transient ischemic attack (TIA)? A a substantial decreased risk of returning to baseline mentation. B a substantial increased risk of subsequent Alzheimer's disease. C a substantial increased risk of pulmonary embolism. D a substantial increased risk of a subarachnoid hemorrhage. E a substantial increased risk of a subsequent stroke.
The Correct Answer is: E Transient ischemic attacks (TIAs) are a very strong predictor of subsequent and escalating neurovascular events. A TIA that resolves fully is something that can be taken advantage of—this is an indicator that the clinician needs to treat the patient aggressively to prevent a subsequent event. (McPhee and Papadakis, 2011, Chapter 24)
Q 70.6: An adult male, not previously vaccinated for rabies, presents to the emergency department after being bitten by an aggressive stray dog. The dog was captured, and declared "probably rabid" by a local veterinarian. Which of the following treatment options should you select for this patient? A administer human rabies immune globulin only B administer equine rabies antiserum only C administer human rabies immune globulin and equine rabies antiserum D administer human rabies immune globulin and equine rabies antiserum and human diploid cell rabies vaccine E administer human rabies immune globulin and human diploid cell rabies vaccine
The Correct Answer is: E Transmission of rabies to this patient must be seriously considered, and postexposure immunization should begin immediately by the administration of human rabies immune globulin (HRIG; 20 IU/kg). About half the HRIG should be infiltrated around the bite wound, and the remainder injected intramuscularly. Human diploid cell rabies vaccine (HDCV) should also be given (1 mL IM in the deltoid), and again on days 3, 7, 14, and 28. HDCV should be delivered in a different syringe and administered at a different site than HRIG. (Shandera and Corrales-Medina, 2009, p. 1225) Shandera WX , Corrales-Medina VF. Viral & rickettsial infections. In: McPhee SJ , Papadakis MA, eds. Current Medical Diagnosis and Treatment. 48th ed. New York, NY: McGraw-Hill; 2009.
Q 43.5: A 12-year-old boy who is 60" tall and weighs 190# is found on routine physical examination to have 2+ glucose and trace ketones in his urine. His fasting glucose is 140 mg/dL and hemoglobin A1C is 6.0%. What is the next step in this child's management? A Initiation of insulin treatment B Prescription of oral metformin C Referral for a glucose tolerance test D Watchful waiting E Weight loss and exercise
The Correct Answer is: E Treatment of type 2 diabetes and pre-diabetes in children varies with the severity of the disease. If the HbA1C is near normal and ketones are not significantly increased, the first line of treatment is lifestyle modifications, including nutrition counseling for the entire family, weight loss, and exercise. Insulin (A) is not indicated in early type 2 disease. If life style changes are not successful, addition of metformin (B) is the next step. Glucose tolerance tests (C) are rarely needed in children. Watchful waiting (D) is inappropriate as continued elevated glucose levels put the child at risk for micro- and macrovascular damage.
Q 67.8: A 52-year-old male, who is a known alcoholic, is admitted to the hospital for GI bleed. Librium is ordered, but he must be monitored for early signs of alcohol withdrawal as his total daily intake of alcohol prior to admission is unknown. Which of the following symptoms might occur as an early sign of withdrawal rather than a later sign? A Delusions B Hallucinations C Nausea and vomiting D Seizures E Tremulousness
The Correct Answer is: E Tremulousness (E) and agitation are early signs of alcohol withdrawal, occurring within the first few hours. These symptoms can progress to nausea and vomiting (C) along with hallucinations (B), delusions (A), and seizures (D) after several hours. Seizures generally occur 12-48 hours after the last drink and full delirium tremens occurs after 48-72 hours. Martin, PR. Chapter 15. Substance-Related Disorders. In: Ebert MH, Loosen PT, Nurcombe B, Leckman JF, eds. CURRENT Diagnosis & Treatment: Psychiatry. 2 nd ed. New York: McGraw-Hill; 2008. http://accessmedicine.com/content.aspx?aid=3283393 .
Q 97.54: An 18-year-old woman comes in for evaluation of a bad smelling vaginal discharge. She says that it is so heavy that she must wear a pad to absorb it and that it is "yellowy green and bubbly." She also has a great deal of itching of vulva. Evaluation confirms the presence of a discharge and a vaginal pH of 5.5. Her vagina and cervix are diffusely erythematous with scattered petechiae. What is the most likely diagnosis? A Atrophic vaginitis B Bacterial vaginosis C Candidiasis D Gonorrhea E Trichomoniasis
The Correct Answer is: E Trichomoniasis vaginitis is the most common STD in the United States that is not caused by a virus. It causes a profuse and frothy discharge that is greenish and can be foul smelling. Pruritus may occur. The vaginal pH is usually more than 5.0 and the vagina and cervix are red and may demonstrate small petechiae. Atrophic vaginitis (A) occurs primarily in postmenopausal women and is characterized by a thin watery or serosanguinous discharge, while bacterial vaginosis (B) has a greyish homogeneous discharge with a characteristic "fishy" odor released when potassium hydroxide is mixed with vaginal secretions. The discharge of candidiasis (C) is typically white and clumpy and is accompanied by intense itching in the vulva. More than three quarters of women who have gonorrhea (D) have no symptoms, but they may have a mucopurulent discharge.
Q 76.6: A patient being treated for Burkitt's lymphoma is found to have hyperkalemia, hyperphosphatemia, hypocalcemia, a metabolic acidosis, hyperuricemia, and appears to be going into renal failure after the first dose of chemotherapy. These are all classic signs of which of the following? A Chemo-reactive disease B Chemotherapeutic anaphylaxis C Pseudohyperkalemia syndrome D Refeeding syndrome E Tumor lysis syndrome
The Correct Answer is: E Tumor lysis syndrome is a serious metabolic complication of rapidly growing tumors. Burkitt's is a classic example of a rapidly growing tumor that can cause this syndrome, as the tumor is broken down by the chemotherapeutic treatment. Chemo-reactive disease means that the tumor responds to chemotherapy. Anaphylaxis is an immune mediated response to any allergen producing substance. There is no diagnosis called Pseudohyperkalemia syndrome. Refeeding syndrome is caused by patients that haven't eaten for an extended period and are then provided parentral nutrition, displacing electrolytes and causing other metabolic complications. (Lichtman et al., Williams Hematology 8e, Chapter 104, Burkitt Lymphoma)
Q 71.9: A 38-year-old woman diagnosed with pancreatic cancer 2 months ago develops jaundice and steatorrhea. Which of the following is most likely to be diminished in the blood? A calcium B iron C vitamin B12 D vitamin C E vitamin K
The Correct Answer is: E Tumors in the pancreatic head region can often block the flow of bile from the gall bladder and liver to the duodenum, resulting in jaundice and steatorrhea. The bile salts are important for micelle formation within the lumen of the small intestine. Micelles provide a mechanism whereby the hydrophobic products of lipid digestion as well as fat-soluble vitamins (eg, A, D, E, and K) can be absorbed in the small intestine. (Costanzo, 2006, pp. 368-369; Rugo, 2008, pp. 1420-1421) Costanzo LS. Physiology. 3rd ed. Philadelphia, PA: Saunders Elsevier; 2006. Rugo HS. Cancer. In: McPhee SJ , Papadakis MA, eds. Current Medical Diagnosis & Treatment. New York, NY: McGraw-Hill Medical; 2008.
Q 92.2: The risk of extrapyramidal side effects (pseudoparkinsonism) and tardive dyskinesia is associated with which class of medications? A amphetamines B benzodiazepines C monoamine oxidase inhibitors (MAOIs) D tricyclic antidepressants (TCAs) E typical (first-generation) antipsychotics
The Correct Answer is: E Typical antipsychotics (eg, haloperidol, chlorpromazine, fluphenazine) can produce pseudoparkinsonism via blockade of dopamine (D 2 ) receptors in the nigrostriatum. Symptoms can include akinesia, bradykinesia, mask-like facial expression, tremor, cogwheel rigidity, and postural abnormalities. Tardive dyskinesia may also occur, as the reported incidence with first generation antipsychotics ranges from 0.5% to 62%. (Carson et al., 2008, pp. 1111-1113) Carson ML , Argo TR , Buckley PF. Schizophrenia. In: DiPiro JT , Talbert RL , Yee GC, et al., eds. Pharmacotherapy: A Pathophysiologic Approach. 7th ed. New York: McGraw-Hill; 2008.
Q 97.124: A 25-year-old gravida 1 woman who is HIV positive arrives at the hospital in early labor. Membranes are intact and the cervix is 50% effaced and 3 to 4 cm dilated. Fetal heart rate is 150 beats/min. Which of the following procedures is contraindicated during labor? A amniotomy B augmentation of labor with oxytocin C external monitoring D operative delivery E use of fetal scalp electrodes
The Correct Answer is: E Use of fetal scalp electrodes and scalp sampling is contraindicated in the HIV-positive woman because it increases the risk of vertical transmission of the human immunodeficiency virus to the infant. While ruptured membranes for more than 4 hours is associated with an increased risk of vertical transmission, amniotomy per se is not contraindicated. Augmentation of labor, external monitoring, and operative delivery are not contraindicated and, in fact, may be indicated in specific instances for the well-being of the infant and/or mother.
Q 9.2: A 63 year-old woman presents to the clinic seeking counseling and medication to help quit smoking. Which of the following tobacco cessation agents binds with high affinity and selectivity at a4β2 neuronal nicotinic acetylcholine receptors? A buproprion B clonidine C nicotine replacement D nortriptyline E varenicline
The Correct Answer is: E Varenicline (E) is a selective partial agonist of the a4β2 neuronal nicotinic acetylcholine receptors and relieves withdrawal symptoms while also blocking the binding of nicotine (from continued tobacco use) to the receptor. Clonidine (B) is a centrally acting alpha-adrenergic agonist. Bupropion (A) and nortriptyline (D) are antidepressants that have shown efficacy in supporting smoking cessation efforts. Bupropion is FDA approved for smoking cessation, while clonidine and nortriptyline are not FDA approved for this indication.
Q 7.3: A 17 year-old male notes acute onset of dyspnea, excessive non-productive coughing, "wheezing" and upper chest tightness when running. Symptoms usually resolve in a few minutes and he is able to resume running. He is prescribed albuterol with no improvement. Which of the following is the most likely diagnosis in the patient? A Angioedema B Bronchiectasis C Cystic fibrosis D GERD E Vocal Cord Dysfunction
The Correct Answer is: E Vocal cord dysfunction (E) is commonly misdiagnosed as asthma or is a comorbidity in patients with asthma. The history of acute onset and rapid disappearance of symptoms is consistent with vocal cord dysfunction, and not typical of cystic fibrosis (C), or GERD (D). The presence of urticarial or swelling of the face or lips would support the life-threatening diagnosis of angioedema (A). Bronchiectasis (B) typically presents with a chronic productive cough and additional pulmonary symptoms based on the underlying cause and severity.
Q 113.3: A central component of hemostasis, it also functions as a carrier factor and an adhesive link between platelets and injured blood vessel walls. Dysfunction of this factor is the most common inherited bleeding disorder in humans. Which factor has these characteristics? A Factor VII B Factor VIII C G6PD deficiency D Tumor necrosis factor E Von Willebrand's factor
The Correct Answer is: E Von Willebrand's factor is a central component in hemostasis, and is a carrier for factor VIII as well as an adhesive molecule. It is the most common bleeding disorder in the general population. G6PD deficiency causes anemia. Tissue necrosis factor plays a role in the regulation of immune cells. Factor VII and VIII deficiencies cause coagulation disorders, but are not the most common. (Lichtman et al., Williams Hematology 8e, Chapter 127, von Willebrand Disease)
Q 25.6: Which of the following would be the best type of exercise to do to strengthen bones and prevent osteoporosis? A Ballistic stretching B Recumbent bike C Static stretching D Swimming E Weight training
The Correct Answer is: E Weight bearing and resistance exercises are the most effective at maintaining bone density. Bones respond favorably to regular impact loading exercises such as walking and strength training with either machines or free weights. Adding balance training exercises like those done in tai chi, yoga, or Pilates would be beneficial as well as part of a fracture prevention plan. Ballistic stretching is a type of flexibility exercise that uses bouncing movements to try and increase range of motion. Static stretching is a more gentle approach to stretching where the person stretches as far as they comfortably can and hold that position for several seconds. There are various schools of thought on the benefits and safety of each stretching method. While both likely have a role in maintaining flexibility and function, neither is likely to play a significant role in the prevention of osteoporosis. Exercising on a recumbent bike likely offers some strength and cardiovascular benefits, especially if utilized as part of a strenuous workout, but as a non weight bearing exercise (seeing as the person using the bike is sitting and leaning back with their weight fully supported), this is not the ideal exercise for osteoporosis prevention. Swimming is a wonderful exercise choice since it affords a challenging cardiovascular workout, uses all of the muscles of the body, improves range of motion and is well tolerated by people with various joint ailments, but because it is not a weight bearing exercise, it should be combined with weight training for optimal bone health.
Q 97.66: Which of the following would be the best type of exercise to do to strengthen bones and prevent osteoporosis? A Ballistic stretching B Recumbent bike C Static stretching D Swimming E Weight training
The Correct Answer is: E Weight bearing and resistance exercises are the most effective at maintaining bone density. Bones respond favorably to regular impact loading exercises such as walking and strength training with either machines or free weights. Adding balance training exercises like those done in tai chi, yoga, or Pilates would be beneficial as well as part of a fracture prevention plan. Ballistic stretching is a type of flexibility exercise that uses bouncing movements to try and increase range of motion. Static stretching is a more gentle approach to stretching where the person stretches as far as they comfortably can and hold that position for several seconds. There are various schools of thought on the benefits and safety of each stretching method. While both likely have a role in maintaining flexibility and function, neither is likely to play a significant role in the prevention of osteoporosis. Exercising on a recumbent bike likely offers some strength and cardiovascular benefits, especially if utilized as part of a strenuous workout, but as a non weight bearing exercise (seeing as the person using the bike is sitting and leaning back with their weight fully supported), this is not the ideal exercise for osteoporosis prevention. Swimming is a wonderful exercise choice since it affords a challenging cardiovascular workout, uses all of the muscles of the body, improves range of motion and is well tolerated by people with various joint ailments, but because it is not a weight bearing exercise, it should be combined with weight training for optimal bone health.
Q 98.69: In a boxer's fracture, the presence of how much angulation of the fifth metacarpal neck would require referral for reduction as opposed to treatment with a simple ulnar gutter splint? A Less than 10 degrees B 10-19 degrees C 20-29 degrees D 30-39 degrees E Greater than 40 degrees
The Correct Answer is: E When there is greater than 40 degrees of angulation or an ex.tensor lag (the patient cannot fully extend the affected finger) then a referral is required. Lesser degrees of angulation without an extensor lag can generally be handled conservatively with an ulnar gutter
Q 14.6: In a boxer's fracture, the presence of how much angulation of the fifth metacarpal neck would require referral for reduction as opposed to treatment with a simple ulnar gutter splint? A Less than 10 degrees B 10-19 degrees C 20-29 degrees D 30-39 degrees E Greater than 40 degrees
The Correct Answer is: E When there is greater than 40 degrees of angulation or an extensor lag (the patient cannot fully extend the affected finger) then a referral is required. Lesser degrees of angulation without an extensor lag can generally be handled conservatively with an ulnar gutter.
Q 119.20: A 64-year-old woman with a medical history of rheumatoid arthritis and deteriorating vision presents to the clinic with complaints of painful bilateral swelling of her ankles and hands, morning stiffness, loss of appetite, and fatigue. She is currently taking naproxen sodium 500 mg twice per day. Which medication(s) would be most appropriate for this patient? A acetaminophen B azathioprine C cyclosporine D hydroxychloroquine E methotrexate
The Correct Answer is: E Whereas NSAIDs such as naproxen provide some symptomatic relief in rheumatoid arthritis, they do not alter disease progression like DMARDs (disease-modifying antirheumatic drugs). NSAIDs are best used in conjunction with DMARDs. Methotrexate is usually the DMARD of choice because it is well tolerated by the majority of patients and can produce beneficial effects in 2 to 6 weeks. Hydroxychloroquine is another DMARD that can be used for rheumatoid arthritis but can produce ocular toxicity. Cyclosporine and azathioprine are used less frequently today due to toxicity and lack of long-term benefits. Similar to naproxen, acetaminophen would also fail to alter disease progression. (Schuna, 2008 pp. 1512-1513; Rabow and Pantilat, 2008, pp. 69, 78) Schuna AA. Rheumatoid arthritis. In: DiPiro JT , Talbert RL , Yee GC, et al., eds. Pharmacotherapy: A Pathophysiologic Approach. 7th ed. New York: McGraw-Hill; 2008. Rabow MW , Pantilat SZ. Palliative care & pain management. In: Tierney LM Jr , McPhee SJ , Papadakis MA, eds. Current Medical Diagnosis & Treatment. 47th ed. New York: McGraw-Hill; 2008.
Q 12.1: A 16-year-old girl presents to the office complaining of a very sore throat, swollen lymph nodes, fever, and general malaise. Her examination reveals a temperature of 102.2°F, enlarged exudative tonsils, tender cervical lymphadenopathy, and borderline enlarged spleen. Rapid strep screen is negative. Which of the following laboratory findings best supports the most likely diagnosis? A decreased white blood cell count B increased monocytes on white cell differential C thrombocytosis D decreased levels of antibody to Epstein-Barr viral capsid antigen E increased atypical lymphocytes on white blood cell differential
The Correct Answer is: E With a negative rapid strep screen, the most likely explanation for this presentation is acute infectious mononucleosis. The fever, fatigue, tonsillar hypertrophy, and splenomegaly are all classic symptoms and signs. Laboratory evaluation often includes an elevated total white blood cell count with increased atypical lymphocytes on differential. Platelets may be decreased. Initially, IgM antibodies for the Epstein-Barr virus, and viral capsid antigen (VCA) levels will be elevated. Later, the IgG levels increase and IgM normalizes.
Q 68.6: A 32-year-old male presents with odynophagia, dysphagia, and chest pain. His past medical history consists of him being HIV positive. He is currently not taking any medications, as he cannot afford to pay for them. An endoscopy is ordered, and the results show several shallow, superficial ulcerations. What is the most likely diagnosis? A Cytomegalovirus esophagitis B Gastroesophageal reflux disease C Candidal esophagitis D Barrett's esophagus E Herpetic esophagitis
The correct answer is (A). Cytomegalovirus esophagitis can occur in patients with HIV. Gastroesophageal reflux disease typically presents with heartburn and regurgitation, and an endoscopy may be normal or show erosions in the distal esophagus at the squamocolumnar junction. Candidal esophagitis appears as diffuse, linear, yellow-white plaques adherent to the mucosa on endoscopy. Barrett's esophagus appears as orange, gastric-type epithelium that extends upward from the stomach, into the distal tubular esophagus, in a tongue-like or circumferential fashion on endoscopy. Herpetic esophagitis appears as multiple small, deep ulcerations on endoscopy.
Q 18.4: A 64-year-old male presents with right knee pain and stiffness while walking and going up and down stairs for the past six months. He states the pain persists all day long and is relieved with rest. There is no history of trauma and his past medical history is unremarkable. An x-ray of the right knee is performed and the following is observed: What is the suspected diagnosis based on the imaging above? A Bursitis B Osteoarthritis C Torn posterior cruciate ligament D Oblique tibia fracture E Spiral femur fracture
The correct answer is (B). Osteoarthritis (OA) is the most common cause of chronic knee pain over 45 years of age and commonly seen in weight-bearing joints when walking or climbing stairs. As disease progresses, it becomes continuous and present at night. OA in the x-ray shows a narrowed joint space (white arrow) on the medial side of the knee (B); sclerosis of the bone in the medial compartment (black arrow), which is evidence of cortical thickening; and formation of osteophytes in the medial femur (white wedge). Soft tissue injuries including a PCL tear (C) and bursitis (A), would not usually be identifiable on x-ray and would need further imaging, such as a CT or MRI. No fractures, (D) and (E), are identified on this image.
Q 66.8: A 66-year-old male patient complains of pain and swelling in his left foot intermittently over the past year. He denies any current symptoms. On examination you note the following findings. His left foot is unremarkable. Laboratory finding include an elevated uric acid. What is the most likely diagnosis? A rheumatoid arthritis B gout C osteoarthritis D septic arthritis E systemic lupus
The correct answer is (B). The patient has gouty tophi. His uric acid level is elevated, which further supports a diagnosis of gout. It is suspected that the intermittent left-foot swelling is related to acute flares of gout, which usually affects the first MCP joint. Choices (A), (C), (D), and (E) are not associated with elevated uric acid level or tophi. The patient may have osteoarthritis, but this does not explain the high uric acid level and the tophi on exam.
A 45-year-old male who presents to the emergency department with sudden onset of lip swelling, which began shortly after awakening this morning. He denies any history of allergies and denies any new medications. His current medications include hydrochlorothiazide (HCTZ), captopril, atenolol, atorvastatin, and fexofenadine. What is the most likely cause of this gentleman's symptoms? Source: (Knoop et al., 2010, Chapter 5) A HCTZ B captopril C atenolol D atorvastatin E fexofenadine
The correct answer is (B). ACE inhibitor angioedema is a potentially life-threatening known side effect of captopril. The patient may need emergency intubation to ensure that his airway remains patent. Patients with a history of ACE inhibitor angioedema should not be placed on any ACE inhibitor or an ARB due to the possibility of similar reaction. HCTZ (a diuretic) and atenolol (a beta blocker) are antihypertensives that are generally unlikely to be associated with angioedema. Atorvastatin (a statin) and Fexofenadine (an antihistamine) are unlikely to present with angioedema.
Q 16.4: A 78-year-old male returns to the FP office for a follow up of non-insulin-dependent diabetes mellitus (NIDDM) as a new patient to you, although he has been an office patient for the past year. He denies any problems this visit and says his blood sugars are in the 90-120 mg/dl range. He is currently taking the medications listed in the following choices. You receive his labs and note that his creatinine is 2.0 mg/dl and on the previous few labs this creatinine was also in the 1.8-2.0 mg/dl range. What medication should be discontinued? A glipizide B metformin C omeprazole D sitagliptin E atenolol
The correct answer is (B). Metformin is contraindicated in this diabetic patient with chronic renal failure due to an increased risk of lactic acidosis and should be discontinued. Sitagliptin requires a dosing adjustment in renal failure but is not contraindicated in this patient. Glipizide (sulfonylurea), omeprazole (a PPI for GERD), and atenolol (a beta blocker for hypertension) are not contraindicated in this patient. Beta blockers should be used with caution in diabetics due to the potential of masking symptoms of hypoglycemia, but are not contraindicated.
Q 2.3: A parent brings her child into the office with the concern of the way he stands. To the parent the child looks like they are "bowlegged." If the child is indeed bowlegged, what would be the best description of this orthopedic abnormality? A Angulation of an extremity at a joint with the more distal part angled anteriorly. B Angulation of an extremity at a joint with the more distal part angled away from the midline. C Angulation of an extremity at a joint with the more distal part angled toward the midline. D Angulation of an extremity at a joint with the more distal part angled posteriorly. E Angulation of an extremity at a joint with the more proximal part angled away from the midline.
The correct answer is (C). A varus deformity involves angulation of an extremity at a joint with the more distal part angled toward the midline. This is the type of deformity found in someone who is described as "bow-legged."
A patient with severe hemophilia A is brought into the emergency department following an automobile accident. Concern is for hemorrhage. What is the best intervention to raise factor VIII levels to hemostatic levels? A Fresh frozen plasma B Cryoprecipitate C Recombinate D DDAVP (desmopressin) E EACA (aminocaproic acid)
The correct answer is (C). While fresh frozen plasma and cryoprecipitate have been used in the past for hemophilia A hemostatis, volumes may be too large or unable to reach levels to achieve hemostasis in severe hemophilic A patients. *Recombinate is a commercial lyophilized factor VIII concentrate that can case factor VIII levels to reach hemostasis in smaller volumes, and do not have the disadvantages of plasma or cryopreciptate.* DDAVP is not used in severe hemophilics. EACA is used to enhance hemostasis, but is not able to achieve initial hemostasis seen with recombinate therapy.
Q 30.1: You are a physician assistant working in family practice. A 54-year-old male with a history of hypertension comes to your office complaining of fatigue. Which of the following medications is the most likely cause of the complaint and the EKG findings below? Source: (Gomella and Haist, 2007, Chapter 19) A hydralazine B losartan C metoprolol D captopril E terazosin
The correct answer is (C). This patient has a sinus bradycardia with a heat rate of about 38 bpm. Metoprolol is a beta blocker known to cause bradycardia. ARBs (such as losartan) and ACE inhibitors (such as captopril) are unlikely to cause bradycardia. Hydralazine an alpha blocker that may cause symptomatic hypotension, but it is unlikely to cause bradycardia. Hydralazine, a direct vasodilator, is more likely to cause tachycardia. Terazosin does not have an effect on rate.
Q 89.1: A 24-year-old female HIV-positive patient, who is not currently on medication, presents to the emergency department with acute dyspnea, tachycardia, fever, nonproductive cough, and a room air oxygen saturation of 92%. She admits feeling poorly for the past five days. A physical exam reveals bilateral basilar crackles. An x-ray reveals the image shown. What is the most likely causative organism of this clinical picture? A Chlamydia psittaci B Histoplasmosis C Klebsiella pneumoniae D Pneumocystis jiroveci E Streptoccocus pneumoniae
The correct answer is (D). Pneumocystis is an opportunistic fungal infection of immunocompromised patients, most frequently seen in patients with untreated HIV/AIDS. Patients often present with fever, dyspnea, a nonproductive cough, decreased arterial oxygen pressure, and tachycardia. Lung auscultation may reveal adventitious sounds, but may also be without abnormality. Chest x-ray results classically include bilateral diffuse infiltrates with perihilar involvement. Although the other etiologies are associated with pneumonia, and may be seen in immunocompromised patients, pneumocystis pneumonia occurs in up to 80% of untreated HIV patients and is a leading cause of death.
Q 37.10: A 9-year-old female child presents with tachycardia, tachypnea, shortness of breath, bibasilar rales, and distended jugular veins. Which of the following is the most likely cause for her signs and symptoms? A rheumatic heart disease B sickle cell anemia C viral myocarditis D patent ductus arteriosus
The correct answer is (D). This patient is presenting with signs of congestive heart failure. The most common causes of heart failure in children/adolescents are due to acquired heart disease. Congenital heart diseases, such as malformations of the heart—patent ductus arteriosus and ventricular septal defects, are the most common causes of heart failure in infants-toddlers, and are second to fluid overload in neonates.
Q 2.6: Consideration should be given to screening patients with type 1 diabetes mellitus should also be screened for which of the following: A sarcoidosis B Sheehan's syndrome C Sjögren's Syndrome D thyroid disease
The correct answer is (D). Type 1 diabetes mellitus (T1DM) is an autoimmune disease. As such, patients have a significantly higher risk of other autoimmune diseases, including celiac and thyroid disease. Most recommendations include screening for both diseases in patients with T1DM.
A 24-year-old female HIV-positive patient, who is not currently on medication, presents to the emergency department with acute dyspnea, tachycardia, fever, nonproductive cough, and a room air oxygen saturation of 92%. She admits feeling poorly for the past five days. A physical exam reveals bilateral basilar crackles. An x-ray reveals the image shown. What is the most likely causative organism of this clinical picture? A Chlamydia psittaci B Histoplasmosis C Klebsiella pneumoniae D Pneumocystis jiroveci E Streptoccocus pneumoniae
The correct answer is (D). Pneumocystis is an opportunistic fungal infection of immunocompromised patients, most frequently seen in patients with untreated HIV/AIDS. Patients often present with fever, dyspnea, a nonproductive cough, decreased arterial oxygen pressure, and tachycardia. Lung auscultation may reveal adventitious sounds, but may also be without abnormality. Chest x-ray results classically include bilateral diffuse infiltrates with perihilar involvement. Although the other etiologies are associated with pneumonia, and may be seen in immunocompromised patients, pneumocystis pneumonia occurs in up to 80% of untreated HIV patients and is a leading cause of death.
You are evaluating a 72-year-old male who is referred to the emergency department from his primary care office. On examination you note a lethargic obese male with the following vitals: BP of 225/135 a P of 88, and T of 98.4˚F. He reports discontinuing all his medications for blood pressure 6 months ago due to financial reasons. Which of the following physical examination findings would be most consistent with a diagnosis of a hypertensive emergency? A normal exam B xanthelasma C varicose veins D papilledema E enlarged nodular prostate
The correct answer is (D). This patient has significantly elevated blood pressure, lethargy, and evidence of hypertensive encephalopathy, which is supported by a physical examination finding of papilledema. Additional workup may be indicated to rule out other causes of his papilledema. Target organ damage is apparent in anyone with hypertensive emergencies and may be neurological, renovascular, cardiovascular, or a combination thereof. Prompt treatment is necessary to prevent complications. Although the physical examination may be normal, evidence of end-organ damage is likely on examination and/or laboratory/diagnostic studies. Choices (B), (C), (E) are not representative of end-organ damage seen in hypertensive emergencies.
Q 10.6: A 28-year-old female presents with a gradual progression of fatigue and pallor over the last few months. Initial CBC results show a hemoglobin of 10.4 mg/dL, hematocrit of 32%, an MCV of 112 fL, and a reticulocyte count of 3%. Which of the following is the most likely pathophysiologic mechanism responsible for her anemia? A Chronic blood loss B Defective bone marrow/stem cell function C Defective DNA production D Defective hemoglobin production E Increased destruction of red blood cells
The correct answer is (E). The time course of the patient's presentation is consistent with multiple episodes of acute hemolysis. Defects in bone marrow (B) or red blood cell precursors (C and D) are refuted by the elevated reticulocyte count. Chronic blood loss (A) would have a more insidious, gradual onset and likely result in a decreased MCV.
Q 102.9: A patient with severe hemophilia A is brought into the emergency department following an automobile accident. Concern is for hemorrhage. What is the best intervention to raise factor VIII levels to hemostatic levels? A Fresh frozen plasma B Cryoprecipitate C Recombinate D DDAVP (desmopressin) E EACA (aminocaproic acid)
The correct answer is (E). While fresh frozen plasma and cryoprecipitate have been used in the past for hemophilia A hemostatis, volumes may be too large or unable to reach levels to achieve hemostasis in severe hemophilic A patients. Recombinate is a commercial lyophilized factor VIII concentrate that can case factor VIII levels to reach hemostasis in smaller volumes, and do not have the disadvantages of plasma or cryopreciptate. DDAVP is not used in severe hemophilics. EACA is used to enhance hemostasis, but is not able to achieve initial hemostasis seen with recombinate therapy.
Q 32.9: A 30-year-old healthy female's labs return showing an HDL of 28 mg/dl, LDL of 132 mg/dl, and total cholesterol of 185mg/dl. She is concerned due to a family history of coronary artery disease (CAD) on her father's side in his 60s. She denies tobacco use. Her blood pressure is 108/50, P-64. She requests advice on what she can due to increase her HDL. What do you advise her? A atorvastatin B cholestyramine C ezetimibe D gemfibrozil E exercise
The correct answer is (E). This patient is at very low risk of CAD based on the 10-year Framingham projections. Based on her < 1%, 10-year risk drug therapy is not indicated. Exercise has been shown to increase HDL. HDL is lower in patients who have a sedentary lifestyle or are obese. Her LDL may also be reduced with exercise, weight loss, and dietary modifications.
Q 18.2: Your patient is a 26-year-old male in whom you've diagnosed major depression. You wish to start him on pharmacologic therapy, but he expresses significant concern regarding sexual side effects. Which of the following depression medications is an option that will reduce the risk of sexual dysfunction? A Bupropion B Citalopram C Fluoxetine D Paroxetine E Venlafaxine
The correct answer is bupropion (A). All the SSRIs, including citalopram (B), fluoxetine (C), and paroxetine (D), as well as venlafaxine (E), which is a combination serotonin and norepinephrine reuptake inhibitor (SNRI), have high rates of sexual side effects for men and women. Bupropion is a norepinephrine and dopamine reuptake inhibitor and can be helpful in averting or reducing both sexual side effects and weight gain. It is also indicated for smoking cessation.
Q 18.5: Your patient is a 26-year-old mother of two young children whose second child was just delivered two weeks ago and she is breast-feeding. She complains of insomnia and depression. She denies the use of any drugs or alcohol. You would like to hold off on pharmacologic therapy unless her symptoms progress. Which of the following will be your first line of treatment? A Encourage more time with the baby B Encourage her to begin planning to get back to work C Promote adequate sleep D Suggest she get out of the house more often E Suggest her husband help more around the house
The correct answer is to promote adequate sleep (C). Postpartum "blues" are very common. Depression can occur in some cases, but most resolve without therapy. When required, SSRIs may be used even when a woman is breast feeding, though no studies have been done.
Q 17.5: A 34-year-old male presents to the primary care office with a complaint of heartburn. He has symptoms two to three times a week, and it occurs about 30 minutes after eating. He has tried over-the-counter antacids and they were helping to relieve his symptoms for a few months, but they are not working well now. He denies dysphagia, odynophagia, or weight loss. What is the next, most appropriate step in managing this patient? A Upper endoscopy B Increase the dose of the antacids C Barium esophagography D Treat empirically with a proton pump inhibitor E Esophageal pH monitoring
The correct answer is to treat empirically with a proton pump inhibitor. This is the treatment of choice in a patient with typical symptoms of heartburn and regurgitation. If the patient fails therapy with a proton pump inhibitor, or had any alarm symptoms present, then it would be reasonable to start with an upper endoscopy, barium esophagography, or esophageal pH monitoring. Increasing the dose of the antacids is not appropriate management for this patient.
Q 39.25: A 1-year-old adopted Chinese infant is brought into the pediatrician's office for a routine visit. The parents are concerned because they think the child is short for her age. They do not have very much information relating to the birth of the child, other than that she was considered slightly short at birth and she had a seizure at one month of age. The foster parents of the child were not concerned, since the infant was somewhat chubby and ate well. She has no history of hypotonia. Upon exam you note normal shaped eyes with mild nystagmus. The infant is less than the third percentile for weight and her limbs are in normal proportion to her height. Her lung and heart exam are normal. CBC is normal. What is the most likely cause of her short stature? A Congenital growth hormone deficiency B Sickle cell anemia C Prader-Willi syndrome D Osteochondrodysplasia E Cystic fibrosis
The correct choice is A, congenital growth hormone deficiency. Parents of children with this disorder typically become concerned when the child is between one and two years of age. This patient fits the characteristic picture of this disorder, with short stature, increased fat mass, and hypoglycemia due to relatively unopposed insulin action. This may have been the cause of her seizure. She may also have optic hypodysplasia as a cause of her nystagmus. Choice B, sickle cell anemia, can be associated with short stature. However, the CBC from this patient was normal. Choice C, Prader-Willi syndrome, is also associated with short stature. Children with this disorder commonly have almond shaped eyes and hypotonia at birth. Choice D, ostrochondrodysplasia, is a type of inherited skeletal abnormality that presents with short stature and abnormal body proportions. This patient doesn't have abnormal body proportions. Choice E, cystic fibrosis, can be associated with growth abnormalities, but this patient has no evidence of lung disease.
Q 70.5: A patient presents with a several year history of developing coarse facial features, prognathism, widely spaced teeth, and sleep apnea. Upon exam, you also note oily skin and high blood pressure. What is the most likely cause of this patient's symptoms? A Pituitary adenoma B Pheochromocytoma C GHRH secreting hypothalamic tumor D Adrenal carcinoma E Ectopic ACTH secretion
The correct choice is A, pituitary adenoma. The patient is presenting with classic signs and symptoms of acromegaly. This disorder is caused most commonly by a growth hormone secreting pituitary adenoma. On rare occasions, it has been caused by choice C, a GHRH secreting hypothalamic tumor. Choice B, pheochromocytoma, is a tumor of the adrenal glands that causes high blood pressure, but does not cause the classic body features of acromegaly. Choice D, adrenal carcinoma, and choice E, ectopic ACTH secretion, may cause Cushing's syndrome, but not acromegaly. (Aron et al., 2007, Chapter 5)
Q 39.6: A 5-year-old girl is seen in your office with a several week history of increased thirst, weight loss, and blurred vision. She has a positive family history for diabetes mellitus, hypertension, and stroke. Her urine dipstick chemical testing reveals positive glucose and negative ketones, protein, blood, and nitrites. Which of the following laboratory test results would support a diagnosis of diabetes mellitus in this patient? A random plasma glucose > 200 mg/dL B random urine glucose dipstick > 1+ C plasma hemoglobin A1c < 7% D fasting plasma glucose > 110 mg/dL E 2-hour postprandial plasma glucose > 135 mg/dL
The correct choice is A, random plasma glucose > 200 mg/dL. The most recent recommendations from the international committee of diabetes experts list the following as diagnostic criteria, which must be confirmed before use: 1) Symptoms of diabetes and a random plasma glucose > 200 mg/dL 2) Fasting plasma glucose > 126 mg/dL 3) Two-hour plasma glucose > 200 mg/dL during a standard 75 gram oral glucose tolerance test More recently, the American Diabetes Association has also included a hemoglobin A1c level equal to or greater than 6.5% as a diagnostic criteria as well. None of the other choices fit into this list of diagnostic criteria.
Q 12.5: A 60-year-old female presents to her primary care practitioner for her yearly check up. Her past medical history is significant only for a history of repeat kidney stones. She enjoys gardening outside, has no significant family history, and is not taking any medications. Routine mammograms have been normal. During the review of her systems, the patient describes feeling tired lately and has noticed muscle aches over the last few weeks. Initial lab values are shown below: Hemoglobin= 12.0 gm/dL Hematocrit = 36% BUN= 18 mg/dL Creatinine= 1.0 mg/dL Calcium= 12 mg/dL Phosphate= 2.0 mg/dL Intact PTH= 80 pg/mL TSH= 3.0 uU/L What is the most likely pathology associated with the findings in this patient? A Solitary parathyroid adenoma B Excessive Ingestion of calcium C Parathyroid carcinoma D Medullary thyroid cancer E Chronic renal disease
The correct choice is A, solitary parathyroid adenoma. Primary hyperparathyroidism is caused by a solitary adenoma in 80% of patient cases. Choices B, D, and E do not cause primary hyperparathyroidism. Choice C can cause primary hyperparathyroidism, but with an incidence of less than 1%.
Q 15.7: A 29-year-old woman presents in July to your office with symptoms of palpitations, sore neck, and excessive sweating, despite using her air conditioner all the time. No surgical or trauma history is noted. She is currently not taking any medications. Vitals include the following: BP = 124/68, pulse = 110 beats per minute, respirations = 18 per minute, and temperature = 101 o F orally. Upon exam, her thyroid is mildly enlarged without nodules, and severely tender. No local erythema or heat is noted. Which of the following lab results would you expect in this patient? A Serum total T4 level = 5.0 ug/dL B Serum TSH level = 0.25 uIU/mL C Sedimentation rate = 15 mm/hr D Free thyroxine index = 8.0 E Positive thyroid stimulating antibodies
The correct choice is B, Serum TSH level=0.25 uIU/mL. The reference range for TSH is 0.34 to 4.25 uIU/mL, and therefore the level in this patient is low. This patient is presenting with signs and symptoms of hyperthyroidism, most likely due to subacute thyroiditis. The leaking of thyroid hormone into the circulation causes anterior pituitary suppression and reduced TSH secretion. Choice A is seen in patients with hypothyroidism. Choice C is within the reference range for woman. Since subacute thyroiditis is an acute inflammatory disorder, patients with this disorder will commonly present with an elevated sedimentation rate. Choice D corresponds to a euthyroid situation. It is an estimate of the free thyroid hormone level in the plasma. This result is within the reference range. Choice E is not seen in subacute thyroiditis. They are commonly found in patients with Graves' disease.
A teenage girl presents to the emergency department with her parents. She has had symptoms of a urinary tract infection for the last two days, but did not tell her parents until today. She is not sexually active. Today, the girl also has diffuse abdominal pain with vomiting, general malaise, and difficulty breathing. She has no significant past medical history. Her physical exam reveals sinus tachycardia and deep fast respirations with no localization of abdominal pain or rebound tenderness. Initial lab test results reveal a plasma glucose = 378 mg/dL and serum bicarbonate = 14 mEq/L. What is the most likely diagnosis? A Hyperosmolar nonketotic hyperglycemia B Diabetic ketoacidosis C Chronic corticosteroid use D Gestational diabetes E Schmidt syndrome
The correct choice is B, diabetic ketoacidosis (DKA). Classic signs and symptoms of this disorder include polyuria, polydipsia, marked fatigue, nausea, vomiting, signs of dehydration, fruity breath odor, postural hypotension, Kussmaul respirations, and possibly mental stupor or coma. Patients with type 1 diabetes mellitus may present for the first time in DKA. DKA is commonly precipitated by a recent infection. Choice A, hyperosmolar nonketotic hyperglycemia, is seen in patients with type 2 diabetes and presents with extremely high plasma glucose levels without acidosis and ketosis. Choice C, chronic corticosteroid use, can cause hyperglycemia and possible glucose intolerance or diabetes mellitus. This patient has no history of oral corticosteroid use. Choice D, gestational diabetes, occurs in women who are pregnant. Choice E, Schmidt syndrome, is an autoimmune polyglandular syndrome which includes diabetes mellitus in greater than 50% of patients.
Q 28.8: A 50-year-old woman presents with a history of polyuria, polydipsia, muscle aches, bone pain, nausea, and constipation for the past few months. Her past medical history is significant for a pituitary adenoma, peptic ulcer disease, and kidney stones. You are considering a diagnosis of primary hyperparathyroidism. Which of the following lab values would you expect in this patient? A Low serum calcium and elevated serum PTH B Elevated serum calcium and PTH levels C Elevated serum calcium and low serum PTH levels D Low serum phosphate and PTH levels E Elevated serum phosphate and calcium levels
The correct choice is B, elevated serum calcium and PTH levels. In primary hyperparathyroidism, excess PTH is secreted and stimulates a rise in serum calcium by increasing calcium release from bone, thereby reducing renal clearance of calcium and increasing calcium absorption through the intestine. In choice A, low serum calcium is not seen in hyperparathyroidism. In choice C and D, low serum PTH levels aren't seen in hyperparathyroidism. In choice E, elevated serum phosphate levels aren't seen in hyperparathyroidism. It typically presents with low serum phosphate levels.
Q 5.1: A 35-year-old woman comes to see you with symptoms of anxiety, sweating, and tremors. She has no history of diabetes, liver or kidney failure, hormone deficiencies, or past surgeries. She states that she is not currently prescribed any medications and does not drink alcohol. She is concerned that she will not be able to continue to care for her husband, who has a long history of diabetes mellitus. She is asking you to give her some medication to stop her symptoms. Initial lab results are as follows: Plasma glucose = 54 mg/dL (70-110 mg/dL) TSH = 2.0 mIU/L (0.34-4.25 uIU/mL) Insulin = 35 uU/mL (2.0-20 uU/mL) C-peptide= 0.4 ng/mL (0.5-2.0 ng/mL) Her symptoms are relieved with the drinking of orange juice. What is the most likely cause of her hypoglycemia? A Alimentary hypoglycemia B Factitious hypoglycemia C Beta cell insulinoma D Congenital hyperinsulinism E Reactive hypoglycemia
The correct choice is B, factitious hypoglycemia. This occurs when patients accidentally or on purpose self-administer insulin or an insulin secretagogue. This occurs most commonly among health care personnel, patients with diabetes or family members of those with diabetes, and people with a history of other factitious diseases. It can also happen secondary to a pharmacy error. Patients with this disorder will have increased measured insulin without the physiologic corresponding increase in C-peptide. Choice A, alimentary hypoglycemia, is a cause of hypoglycemia in patients with a history of gastrectomy. Choice C, beta cell insulinoma would present with elevated levels of both insulin and C-peptide. Choice D, congenital hyperinsulinism would have presented itself earlier than in a patient who is 35 years old. Choice E, reactive hypoglycemia, occurs after eating a meal and must be documented in this fashion.
Q 32.12: Until now, an elderly patient with type 2 diabetes mellitus has been able to keep her plasma glucose and hemoglobin A1c under control with a diabetic diet and regular exercise. Her recent check-ups revealed that her hemoglobin A1c has been rising up. Her health care provider suggested that she would benefit from treatment with medication. Which of the following medications acts by reducing hepatic gluconeogenesis? A Pioglitazone B Metformin C Acarbose D Miglitol E Saxagliptin
The correct choice is B, metformin. This medication is used as an adjunct to diet, to control hyperglycemia in patients with type 2 diabetes mellitus. It works well in patients who are obese and who may not be responding to sulfonylurea medications. It tends to improve both fasting and postprandial hyperglycemia. Choice A, pioglitazone, is a thiazolidinedione and acts to reduce plasma glucose by sensitizing peripheral tissues to insulin. Choice C, acarbose, and choice D, miglitol, are alpha-glucosidase inhibitors. They act to treat hyperglycemia by acting as competitive inhibitors of alpha-glucosidases in the intestinal brush border. This results in a delay in the absorption of carbohydrates and reduced postprandial glucose excursion. Choice E, saxagliptin, is a DPP-4 inhibitor that acts to prolong the action of endogenously released glucagon like peptide 1 (GLP-1) and glucose-dependent insulinotropic polypeptide (GIP), which increase the glucose-induced insulin release.
Q 10.10: A 45-year-old patient presents to your office with a magazine ad for lipitor. She is wondering if she should be taking this medication. She heard on television that it may help to reduce her risk of death. Most of her friends are taking it, and she wants the best health care possible. She has no past medical history or family history of obesity, diabetes, hypertension, stroke, or heart attacks. She doesn't smoke and she exercises regularly. Her diet includes a wide variety of vegetables, fish, meats, and fruit. Her fasting lipid panel reveals the following results: Total cholesterol = 180 mg/dL Total triglycerides = 100 mg/dL HDL = 60mg/dL LDL = 140 mg/dL According to the 2004 revised NCEP ATP III guidelines, what plan should you recommend for this patient? A Initiate strict dietary changes to lower total fat intake to between 10% and 20% daily B Suggest preparing a food diary to look for patterns and choices C Increase the amount of exercise to five days per week D Start lipid lowering agent such as niacin E Suggest starting a low carbohydrate diet
The correct choice is B, suggest preparing a food diary to look for patterns and choices. According to the NCEP ATP III guidelines, this patient does not have any risk factors for coronary heart disease. Her LDL level should be less than 160 mg/dL. All patients can benefit from observing their dietary habits and looking for opportunities to make changes to lower their fat intake. Dietary cholesterol should be less than 200 mg/day, and soluble fiber should be included in the diet. Patients can look at their eating patterns at home, look at the methods used for cooking and baking, and determine the frequency of eating out. Strict changes in her diet, choice A, are not necessary at this time. She already exercises regularly and her HDL is at an adequate level. Therefore, choice C, increasing the amount of exercise to five days per week, is not necessary. Choice D, start a lipid lowering agent such as niacin, is not recommended. Niacin is used to increase serum HDL and lower serum LDL. Her lipid levels are within the reference ranges. Choice E, suggest starting a low carbohydrate diet, may be associated with short-term weight loss, an increase in HDL, and a decrease in triglycerides. This is not something that this patient requires.
Q 27.3: Which of the following is appropriate in the pharmacologic management of patients with hypoparathyroid tetany? A Aggressive IV hydration B Cinacalcet hydrochloride PO C Calcium gluconate IV D Pamidronate IV E Calcitonin IV
The correct choice is C, calcium gluconate IV. In severe hypocalcemia, replacement calcium must be started promptly, as well as airway maintenance and magnesium and vitamin D replacement, as necessary. Choices A, aggressive hydration, B, cinacalcet hydrochloride, D, pamidromate, and E, calcitonin, are all possible treatment options for patients with hypercalcemia.
Q 13.5: A patient presents with symptoms of palpitations, tremor, confusion, and sweating. She is also hungry. Her plasma glucose level is currently 54 mg/dL. Which of the following drugs would be particularly important to investigate during the patient history? A Vancomycin B Acetaminophen C Ethanol D Lisinopril E Prednisone
The correct choice is C, ethanol. Ethanol can suppress gluconeogenesis and stimulate glucose utilization. Alcohol induced hypoglycemia occurs most commonly after a several day drinking binge with little food intake. Other medications that can be associated with hypoglycemia include insulin, insulin secretagogues, salicylates, sulfonamides, pentamidine, quinine, quinolone antibiotics, and beta 1 -adrenergic antagonists. The other choices are not associated with inducing hypoglycemia. Choice E, prednisone use, has been associated with hyperglycemia.
Q 16.10: A 60-year-old female presents to her primary care practitioner for her yearly check up. Her past medical history is significant only for a history of repeat kidney stones. She enjoys gardening outside, has no significant family history, and is not taking any medications. Routine mammograms have been normal. During the review of her systems, the patient describes feeling tired lately and has noticed muscle aches over the last few weeks. Initial lab values are shown below: Hemoglobin = 12.0 gm/dL Hematocrit = 36% BUN = 10 mg/dL Creatinine = 0.7 ng/mL Calcium = 12 mg/dL Phosphate = 2.0 mg/dL PTH = 100 pg/mL TSH = 2.0 UIU/mL What is the most likely cause of her symptoms? A Vitamin D deficiency B Hypercalcemia of malignancy C Primary hyperparathyroidism D Factitious hypercalcemia E Hyperthyroidism
The correct choice is C, primary hyperparathyroidism. This patient has the characteristic signs and symptoms of hypercalcemia, along with the lab findings of primary hyperparathyroidism with elevated calcium, low phosphate, and elevated PTH. Other common presenting features include bone pain, paresthesias, polyuria, and gastrointestinal symptoms. Females are affected three times as much as males, and it is typically found accidentally. Choice A, vitamin D deficiency, is unlikely since she enjoys gardening outside in the sun. Vitamin D deficiency can be seen is association with primary hyperparathyroidism, but is not the cause of this disorder. Choice B, hypercalcemia of malignancy, is unlikely with her history of normal mammograms. Serum PTH is typically normal in this disorder as well. Choice D, factitious hypercalcemia, is unlikely since she doesn't take any calcium supplements or other medications. Choice E, hyperthyroidism, can be a cause of hypercalcemia if the patient is suffering from thyrotoxicosis, but this patient's TSH is within the reference range.
Q 10.9: A 50-year-old male is seen with a routine check-up. He is concerned about the possibility of developing diabetes mellitus. He has a negative family history of diabetes. He has no signs or symptoms of diabetes and he is not overweight. Without any risk factors for diabetes, what is the recommended screening protocol for this patient according to the American Diabetes Association (ADA)? A screen all men over 25 years of age every five years B screen all men over 35 years of age every two years C screen everyone over 45 years of age every three years D no screening is necessary without risk factors E no screening is necessary without a family history of diabetes
The correct choice is C, screen everyone older than 45 years of age every three years. In addition, the ADA recommends screening for younger people if they are overweight and have at least one additional risk factor, such as positive family history, hypertension, and/or vascular disease. The other choices are not recommended by the ADA for screening the general population for diabetes mellitus.
Q 21.3: Which of the following sets of disorders is commonly found in multiple endocrine neoplasia (MEN) 2A? A Islet cell tumor; renal cell carcinoma; pheochromocytoma; B Pheochromocytoma; medullary thyroid carcinoma; mucosal neuromas C Medullary thyroid carcinoma; parathyroid hyperplasia; pheochromocytoma D Parathyroid adenoma; islet cell hyperplasia; pituitary adenoma E Visceral lipomas; Marfanoid features; retinal angiomas
The correct choice is C. The three primary features of MEN type 2A include medullary thyroid carcinoma, parathyroid hyperplasia or adenoma, and pheochromocytoma. In choice A, islet cell tumor and renal cell carcinoma are disorders found in various other MEN syndromes, but not in MEN type 2A. Choice B, pheochromocytoma, medullary thyroid carcinoma, and mucosal neuroma are disorders found in MEN 2B. Choice D, parathyroid adenoma, islet cell hyperplasia, and pituitary adenoma are found in MEN I. Choice E, visceral lipomas, Marfanoid features, and retinal angiomas are features found in various MEN syndromes, but not in MEN 2A.
Q 12.4: Patients diagnosed with type 2 diabetes are encouraged to maintain their plasma glucose, to prevent or slow the initiation of chronic complications. What is the maximum level that is within ADA guidelines for a one to two hour postprandial plasma glucose in these patients? A 70 mg/dL B 100 mg/dL C 120 mg/dL D 180 mg/dL E 220 mg/dL
The correct choice is D, 180 mg/dL. Patients are encouraged to adhere to lifestyle and medication treatment plans in order to help achieve this goal. Other ADA guidelines include maintaining the hemoglobin A1c < 7% or as close to normal (<6%) as possible, blood pressure < 130/80, and preprandial glucose between 90 and 130 mg/dL. (Powers et al., 2008, Chapter 338)
A 14-year-old boy presents to the emergency department with his parents. He has a history of type 1 diabetes, and has had bronchitis for the last few days. He is now presenting with difficulty breathing, worsening fatigue, polydipsia, and polyuria. His last fingerstick glucose at home this morning was 350 mg/dL. Which of the following patterns of breathing are characteristic of this complication of diabetes? A Cheyne stokes respiration B Bradypnea C Biot breathing D Kussmaul breathing E Painful respiration
The correct choice is D, Kussmaul breathing, which is deep regular breathing or hyperpnea. It can be seen as a compensatory action of metabolic acidosis and hypoxia. Choice A, Cheyne-Stokes respiration, is a waxing and waning pattern of rate and volume that includes periods of apnea. This can be seen in patients at high altitudes, and with severe left sided heart failure or neurologic disease. Choice B, bradypnea, is noted with a slower than usual respiratory rate and can be seen with use of CNS depressant drugs, uremia, or structural intracranial lesion. Choice C, Biot breathing, is an uncommon variant of Cheyne-Stokes respiration, with periods of apnea alternating with a series of equal breaths that end abruptly. It can be seen in patients with meningitis. Choice E, painful respiration, is relatively normal in pattern, but interrupted by pain during breathing from such disorders as pleurisy, fractured ribs, or subphrenic inflammation.
Q 25.10: A patient presents with polydipsia and polyuria. Diabetes mellitus is ruled out with a normal plasma glucose and hemoglobin A1c. You are concerned that he may have hypothalamic diabetes insipidus. If you are correct, what treatment would you recommend to this patient? A Lithium B Indomethacin C Metformin D Desmopressin E Fluid restriction
The correct choice is D, desmopression. Patients with hypothalamic diabetes insipidus can't secrete vasopression (or antidiuretic hormone), since the vasopressin producing neurons are dead. Vasopressin analog desmopressin is available in tablets, nasal solution, and parenteral solution for patients with this disorder. Choice A, lithium, can cause nephrotoxicity, and can be a cause of acquired nephrogenic diabetes insipidus. Choice B, indomethacin, is an anti-inflammatory medication used to treat conditions such as gout. Choice C, metformin, is an oral diabetic medication that acts by suppressing hepatic glucose production. Patients must drink sufficient fluids to maintain serum sodium levels since without ADH, they can't conserve water. Therefore, choice E, fluid restriction, is not correct.
Q 36.5: A 12-month-old infant is being worked up for congenital growth hormone deficiency. Her length at birth was short and she has been consistently less than the third percentile on her height chart. Her past medical history is only significant for three seizures since birth. Upon exam you note that she has a full face and her body proportions are normal. The remainder of the exam is normal. What is the most likely cause of her seizures? A Abusive head trauma B Brain tumor C Arteriovenous malformation D Hypoglycemia E Cerebrovascular accident
The correct choice is D, hypoglycemia. Patients with growth hormone deficiency will not have the counter regulatory action of growth hormone against insulin. This will allow unregulated insulin action causing hypoglycemia and possible associated seizures. Choices A, abusive head trauma, B, brain tumor, C, arteriovenous malformation, and E, cerebrovascular accident, can all cause seizures. This patient does not show any signs or symptoms of any of these disorders.
Q 1.1: A patient was recently diagnosed with type 1 diabetes mellitus. A treatment plan was initiated, with a combination regimen of insulin. Which of the following types of insulin works well with a rapidly acting insulin, such as insulin lispro, to provide 24-hour coverage for the patient? A NPH insulin B Regular insulin C Insulin aspart D Insulin glargine E Humalog 75/25
The correct choice is D, insulin glargine. This is the only long acting insulin listed. The combination of a long acting insulin with a rapidly acting insulin provides physiologic insulin replacement to the patient. This regimen provides postprandial control after meals and basal coverage throughout the day and night. Choice A NPH insulin, can be used by itself in two or more injections throughout the day. Choice B, regular insulin, can be used instead of rapid acting insulin, and not in combination with it. Choice C, insulin aspart, is a type of rapidly acting insulin and would not be used in combination with another rapidly acting insulin. Choice E, Humalog 75/25, is a combination insulin preparation with 75% intermediate acting insulin and 25% insulin lispro.
Q 23.8: An elderly women presents with a history of atrial fibrillation and falls. Her current review of symptoms reveals increased sweating and weight loss. A physical exam revealed multiple non-tender nodules on thyroid exam. Which of the following serum lab values is most consistent with this diagnosis? A High TSH, high free T3, normal total T4 B Normal TSH, low free T3, low total T4 C Normal TSH, high free T3, high total T4 D Low TSH, high free T3, high total T4 E Low TSH, high free T3, low total T4
The correct choice is D, low TSH, high free T 3 , and high total T 4 . A diagnosis of toxic multinodular goiter must be made first. This disorder presents with symptoms of hyperthyroidism. In the elderly, patients present more frequently with symptoms of the complications of the disease including cardiovascular and cerebrovascular presentations. These may include atrial fibrillation, congestive heart failure, angina, apathy, and confusion. Other general symptoms of hypermetabolism may include weight loss, sweating, and tremor. Laboratory findings in various causes of hyperthyroidism include a reduced serum TSH, as the pituitary is responding to the increased serum levels of thyroid hormone in the circulation. These thyroid hormone levels feed back on the anterior pituitary, causing a suppression of TSH secretion. In choice A, high TSH, high free T 3 ,and normal total T 4 , high TSH is only seen in secondary causes of hyperthyroidism (e.g. anterior pituitary origin). This patient's history of multiple thyroid nodules indicates a thyroid cause for her symptoms. In choices B and C, the TSH is normal. This is seen in euthyroid patients. Choice E does include suppression of TSH, but the total T 4 is typically elevated in hyperthyroid states.
Q 21.10: A mother expresses concern for her teen son after feeling a lump in his neck. He has no history of trauma to his neck. Surgical history is negative, and the patient does not take any medications. The mother tells you that thyroid problems run in the family. The patient has not been ill recently. Upon exam you feel a nontender, firm nodule on the right side of his thyroid with associated cervical lymphadenopathy. His serum TSH level is within the reference range. Radionuclide thyroid scanning demonstrates a "cold" nodule in the right side of the thyroid. What is the most appropriate next step in the work up of this patient? A MRI of the anterior pituitary B CT of the thyroid C MRI of the thyroid D Thyroid nodule fine needle aspiration E Emergent thyroidectomy
The correct choice is D, thyroid nodule fine needle aspiration. With the advent of fine needle aspiration, it has become much easier, safer, and more reliable to obtain a specimen for biopsy. This patient has several characteristics that increase his risk of malignancy including his gender, young age, firmness of the nodule, and related lymphadenopathy. These, along with the ease of biopsy, suggest this path for diagnostic work up. Not enough information is known to warrant an emergent thyroidectomy, choice E. Choice A, MRI of the anterior pituitary, would be warranted if there was a suspicion of a pituitary cause of the thyroid nodules. Since the TSH is normal and the patient is not presenting with headaches or other pituitary related symptoms, this is not suggested. Choice B, CT of the thyroid, and choice C, MRI of the thyroid, would not provide any additional information after the thyroid scan. They may be helpful prior to any surgery if needed.
Q 25.1: A female patient presents with a history of hypertension and low plasma HDL. She is asking you if she has metabolic syndrome. She does not have diabetes and she is not obese. Which of the following NCEP ATP III criteria would be needed to confirm that diagnosis? A LDL > 70 mg/dL B LDL > 90 mg/dL C Triglycerides > 100 mg/dL D Triglycerides ≥ 150 mg/dL E Triglycerides > 300 mg/dL
The correct choice is D, triglycerides ≥ 150 mg/dL. The NCEP ATP III criteria for metabolic syndrome include three or more of the following: Central obesity with waist circumference > 102 cm in men and > 88 cm in women Hypertriglyceridemia with serum triglycerides ≥ 150 mg/dL, or the patient is taking medication for hypertriglyceridemia Low HDL level < 40 mg/dL in men and < 50 mg/dL in women, or the patient is taking medication for low HDL Hypertension with blood pressure ≥ 130 mm systolic or ≥ 85 mm diastolic, or the patient is taking medication for high blood pressure Fasting plasma glucose ≥ 100 mg/dL, or a positive diagnosis for diabetes mellitus, or the patient is taking medication for hyperglycemia
Q 10.4: A combination of which of the following medications may increase the risk of muscle and liver disease more than either drug used alone? A Ezetimibe and HMG-CoA reductase inhibitor B Low dose niacin and HMG-CoA reductase inhibitor C Colestipol and gemfibrozil D Niacin and cholestyramine E Gemfibrozil and HMG-CoA reductase inhibitor
The correct choice is E, gemfibrozil and HMG-CoA reductase inhibitor. In combination, these can potentiate the risk of developing hepatic disease or myopathy. Choice A, Ezetimibe and HMG-CoA reductase inhibitor, is a synergistic treatment plan for patients with primary hypercholesterolemia. Choice B, low dose niacin and HMG-CoA reductase inhibitor, is a practical and effective treatment plan for patients with familial combined hyperproteinemia. Choice C, colestipol and gemfibrozil, is sometimes useful in patients with familial combined hyperlipidemia who are intolerant of niacin or HMG-CoA reductase inhibitors. Unfortunately, this combination may increase the risk of cholelithiasis. Choice D, niacin and cholestyramine, is useful in disorders with elevated VLDL and LDL, and useful in treating heterozygous familial hypercholesterolemia.
Q 29.3: A 45-year-old patient came in to see his health care provider today, to discuss the results of his last annual assessment. He was told that he had developed type 2 diabetes mellitus. One of the recommendations from the physician assistant included a visit to an ophthalmologist. The physician assistant was concerned after seeing new capillaries, macular edema, and fibrous tissue within the retina during his funduscopic exam. What type of ocular complication does this patient most likely have at this time? A Background retinopathy B Closed angle glaucoma C Macular degeneration D Diabetic cataracts E Proliferative retinopathy
The correct choice is E, proliferative retinopathy. The distinguishing factor in the patient's presentation, which signals this disorder, is the development of newly formed vessels. Proliferative retinopathy is the leading cause of blindness in the United States. Up to 20% of patients with type 2 diabetes have retinopathy at the time of diagnosis. Choice A, background retinopathy, or simple retinopathy includes retinal microaneurysms, hemorrhages, exudates, and edema, without new vessel formation. Choice B, closed angle glaucoma, is relatively uncommon in patients with diabetes, except after cataract extraction. Choice C, macular degeneration, is not associated with diabetes mellitus specifically. Choice D, diabetic cataracts, tends to occur in patients with diabetes earlier than the general population, and may correlate with the severity of the disease.
Q 39.2: A 10-year-old child is seen with his parents for a routine check up. During the review of symptoms, his parents mention that their son has been extremely thirsty and is going to the bathroom to urinate frequently. The patient agrees. The parents are concerned that their son has developed diabetes mellitus. The family history is negative for diabetes mellitus, but the mother has a history of familial hypothalamic diabetes insipidus. Screening blood work includes a CBC, hemoglobin A1c, and renal function tests, all of which are within the reference range. Which of the following serum analytes would you expect to be deficient? A Sodium B Glucose C Thyroxine D Prolactin E Vasopressin
The correct choice is E, vasopressin. The reader must first understand that the patient's symptoms are classic for diabetes insipidus, with the increased thirst, frequency, and polyuria. These symptoms are also seen in diabetes mellitus, but this would be less likely since the patient doesn't have a family history of diabetes mellitus, and his hemoglobin A1c is not elevated. Choice A, sodium, should be within range, as long as the boy is able to drink when thirsty; otherwise, he would become hypernatremic. Choice B, glucose, is incorrect. Although the signs and symptoms also fit the pattern of diabetes mellitus, the glucose would most likely be elevated in this disorder, and not low. Choice C, thyroxine, is under control of TSH from the anterior pituitary gland, and choice D, prolactin, is secreted from the anterior pituitary gland. Vasopression is secreted from the posterior pituitary gland.
Q 72.8: A 40-year-old patient presents to your walk-in clinic with symptoms of hyperhydrosis, oily skin, daytime sleepiness, and snoring. Upon exam, you note large fleshy heel pads and hands with sweaty palms. The patient also has coarse facial features. When asked, the patient isn't aware of any major changes in her face or body. She has not seen another health care provider in many years and has not kept up with any health care maintenance schedule. The patient lives with her ill mother and is not currently employed. Which of the following screening tests would best aid in the diagnosis of this patient? A CT of the chest and abdomen B Thyroid scan C 24-hour urine for catecholamines D Serum calcitonin level E Serum IGF-I level
he correct choice is E, serum IGF-I level. Age and gender matched levels of IGF-I are elevated in patients with acromegaly. IGF-I is the mediator of most of the effects of GH on the body, and lead to the proliferation of bone, cartilage, and soft tissue. Although GH levels may be elevated in patients with acromegaly, they are secreted in a pulsatile fashion and are not consistently elevated. Serum GH levels are not the best screening test for acromegaly. Choices A, a CT of the chest and abdomen, and B, a thyroid scan, are expensive imaging studies that are not usually used as screening tests. They also have no role in the routine workup of patients with suspected acromegaly. Choice C, 24-hour urine for catecholamines, is a test that can be used in the work up of patients with suspected pheochromocytoma. Choice D, serum calcitonin levels, are associated with medullary thyroid cancer and other thyroid disease. (Melmed et al., 2008, Chapter 333) (Greenspan et al., 2007, Chapter 5)
Q 56.6: A 42-year-old female has an endoscopy performed, which shows a smooth, sessile nodule with normal overlying mucosa present in the esophagus. A biopsy is performed and the results show a benign esophageal neoplasm. What is the most likely diagnosis? A Leiomyoma B Zenker diverticulum C Esophageal web D Leiomyosarcoma E Barrett's esophagus
A The correct answer is leiomyoma, which is the most common benign esophageal neoplasm. Zenker diverticulum is a protrusion of pharyngeal mucosa, which develops at the pharyngoesophageal junction, between the inferior pharyngeal constrictor and the cricopharyngeus. An esophageal web would appear as a thin, diaphragm-like membrane of squamous mucosa. Leiomyosarcoma is a malignant tumor. Barrett's esophagus would appear as orange, gastric-type epithelium that extends upward from the stomach, into the distal tubular esophagus, in a tongue-like or circumferential fashion.
Q 62.4: A 62-year-old man with a history of hypertension, diabetes mellitus type 2, hyperlipidemia, and chronic tobacco use presents to the office with complaints of a retrosternal chest pressure radiating down his left arm, associated with diaphoresis, nausea, and dyspnea, for the last 45 minutes after mowing his lawn. The patient's vital signs are stable, and on physical examination a new systolic murmur is appreciated. According to the most recent American College of Cardiology/American Heart Association Guidelines for the Management of Patients with Unstable Angina/Non-ST-Elevation MI recommendations, an EKG should be performed on patients with a clinical suspicion for acute coronary syndrome within how many minutes of their arrival to the emergency department? A 1 minute B 5 minutes C 10 minutes D 15 minutes E 20 minutes.
C Choice C is correct, as the 2007 American College of Cardiology/American Heart Association Guidelines for Management of Patients with Unstable Angina/Non-ST-Elevation MI recommend that an ECG "be performed and shown to an experienced emergency physician as soon as possible after ED arrival, with a goal of within 10 minutes of ED arrival for all patients with chest discomfort or other symptoms suggestive of ACS." Although choices A and B would be optimal, the question asks for the goal time for which busy emergency departments should aim for in obtaining an EKG in at-risk patients. Choices D and E are less desirable, as times greater than 10 minutes increase both morbidity and mortality rates.
Q 57.8: A young man presents with difficulty breathing at times. Upon exam you note evidence of a firm, fixed thyroid nodule with extension toward the trachea and surrounding muscles. The patient has a family history of thyroid cancer. You are concerned that the patient may have medullary thyroid cancer. After you confirm your suspicion, which of the following genetic tests would you suggest to his family members? A BRCA 1 B APC C CFTR D ret proto-oncogene E CAG triplet expansion
D *The correct choice is D, ret proto-oncogene. Genetic testing to identify this oncogene is available for patients whom you suspect have medullary thyroid cancer and/or pheochromocytoma, and for their asymptomatic families who may be at risk for the same disorders. A mutation in this gene leads to uncontrolled growth of the C cells of the thyroid.* Choice A, BRCA 1, is an oncogene that is associated with risk for breast, ovarian, colon, and prostate cancers. Choice B, APC, is an oncogene noted in the familial colon cancer syndrome known as familial adenomatous polyposis. DNA analysis for CFTR mutations, choice C, can identify the gene mutations in patients with cystic fibrosis in the vast majority of instances. Choice E, CAG triplet expansion, is the result of a gene mutation in Huntington's disease.
Q 57.7: You are evaluating a urinalysis on a 44-year-old female due to complaints of recurrent urinary tract infections (UTIs). Which of the following microscopic findings would be most suggestive of chronic kidney disease? A red blood cell (RBC) casts B hyaline casts C white blood cell (WBC) casts D broad waxy casts E granular casts
D The correct answer is (D). Broad waxy casts are suggestive of chronic kidney disease. RBC casts suggest glomerulonephritis. Hyaline casts may occur with heavy exercise or febrile illness. WBC casts suggest infection or inflammation such as in pyelonephritis. Granular casts, although nonspecific, may suggest acute tubular necrosis.
Q 58.8: A 38-year-old male complains of increasing fatigue, weakness, weight loss, and intermittent nausea, vomiting, and diarrhea over the past few months. He has noted some agitation at times. When this first started he thought that this was related to a virus but the GI symptoms have reoccurred on multiple occasions. Labs show a complete blood count (CBC) within the normal reference range. He is noted to have hyponatremia. On examination you note the multiple areas of hyperpigmentation as seen below. His blood pressure in the office is 100/50, P = 66, T = 97.1˚F. What test would you order to confirm your suspected diagnosis? Source: (Wolff and Johnson, 2009, Section 15) A dexamethasone suppression test B vasopressin challenge test C radioactive iodine uptake scan D cosyntropin stimulation test E follicular stimulation test
D The correct answer is (D). The patient's symptoms and examination findings are consistent with a diagnosis of Addison's disease, which is most likely due to an autoimmune process that destroys the adrenal glands resulting in a chronic adrenal insufficiency. The cosyntropin (ACTH) stimulation test should reveal a low am cortisol level and an elevated ACTH level if he has Addison's disease. The dexamethasone suppression test, choice (A), is a laboratory test for Cushing's syndrome. The vasopressin challenge test, choice (B), is a laboratory test for diagnosis of diabetes insipidus. A radioactive iodine uptake scan, choice (C), is used in the diagnosis of thyroid disease (hyperthyroidism and thyroid nodules). A follicular stimulation test, choice (E), is a factitious test.
Q 62.1: Which of the following medication classes has been shown to improve the short-term rate of cognitive preservation in mild to moderate Alzheimer's dementia patients? A Acetylcholinesterase inhibitors B Decarboxylase inhibitors C Dopamine receptor agonists D Monoamine oxidase inhibitor E N-methyl-D-aspartate receptor antagonist
The Correct Answer is: A Alzheimer's dementia is a chronic, progressive, neurodegenerative disorder. Acetylcholinesterase inhibitors have been associated with a modest decrease of cognitive decline and increased functioning. These medications increase the levels of acetylcholine, a neurotransmitter, and increase cholinergic activity within the affected brain regions. They have been approved for use in mild to moderate Alzheimer's disease. Memantine, an N-methyl-D-aspartate receptor antagonist, is indicated for use in moderate to severe disease, has been associated with decreased destruction of cholinergic neurons, and may slow cognitive decline. It is often added to acetylcholinesterase inhibitor therapy for progressed disease, but may be first line in a patient unable to tolerate the acetylcholinesterase inhibitor medications. Decarboxylase inhibitor medications inhibit the metabolism of dopamine; one example, carbidopa, is used in conjunction with L-dopa in the treatment of Parkinson's disease. Dopamine receptor agonists activate dopamine receptors in the absence of dopamine, and are useful in conditions with low dopamine levels, such as Parkinson's, prolactinomas, and restless leg syndrome. Selegiline, a monoamine oxidase inhibitor, is selective for monoamine oxidase-B and inhibits catabolic dopamine breakdown, with a potential neuroprotective effect. Use in moderately advanced Alzheimer's disease may slow progression, but remains controversial.
Q 54.13: According to the Centers for Disease Control and Prevention (CDC), which of the following cannot be treated with a course of doxycycline 100 mg PO BID for 10 to 14 days? A babesiosis B human monocytic ehrlichiosis (HME) C Lyme disease D Rocky Mountain spotted fever E All can be treated with doxycycline.
The Correct Answer is: A Human monocytic ehrlichiosis, Lyme disease and Rocky Mountain spotted fever are all tick-borne diseases causes by spirochetes and treated with doxycycline for 10 to 14 days. Babesiosis is an intracellular parasite and may not necessarily require treatment but, when warranted, the CDC recommends atovaquone plus azithromycin.
Q 61.1: A 38-year-old thin, HIV+ male is found to have a distended abdomen and is complaining of a feeling of fullness. A CT scan of the abdomen revealed a large abdominal mass with ascites in the retroperitoneal region. Laboratories found an elevated EBV of 5400 copies (normal=0), LDH 554 (100 to 250), and Ig-positive MYC translocation. What is the most likely diagnosis? A Burkitt lymphoma B Cat scratch disease C Hodgkin's lymphoma D Infectious mononucleosis E Sarcoidosis
The Correct Answer is: A In immunosuppressed individuals, including those who are HIV+ and in the presence of highly reactive EBV, the development of an abdominal mass with elevated LDH is most likely Burkitt's lymphoma. Of the masses that are found, 30 to 40% will be positive for EBV, and it appears that this virus drives the conversion of cells to a cancerous state. Infectious mononucleosis would be positive for EBV, but does not develop large abdominal masses.
Q 56.1: A 70-year-old man with a history of hypertension, DM Type 2, and hyperlipidemia is seen for preoperative evaluation prior to left knee replacement. On auscultation, a very soft high-frequency decrescendo early diastolic murmur is heard at the upper left sternal border. Which of the following maneuvers would be the most appropriate to choose to increase the intensity of the murmur for better identification? A Isometric hand grip exercise B Listening with the bell at the apex with the patient in the left lateral decubitus position C Inspiration, followed by the patient holding his/her breath D The Valsalva maneuver E Having the patient lie flat with the knees bent
The Correct Answer is: A Isometric hand grip exercises will increase the intensity of the murmur of aortic regurgitation, which is usually described as a high-frequency decrescendo early diastolic murmur heard best at the left upper sternal border or at the right upper sternal border. Radiation, if it occurs, is frequently to the lower left sternal border and the apex. Isometric hand exercises increase arterial and left ventricular pressure, which increases the flow across the aortic valve, thereby increasing the murmur's intensity. Choice B is best used when listening to the murmur of mitral stenosis. Choice C will increase the AP diameter, making it more difficult to hear the murmur. Choice D, the Valsalva maneuver, will decrease the intensity of the murmur. Choice E is the best position for the abdominal exam, especially in males.
Q 59.8: A 36-year-old auto mechanic presents to the emergency department after hurting his back on the job. While lifting an object, he experienced sudden pain in his lower back with radiation to the right buttock. He was initially treated for muscle strain with a nonsteroidal anti-inflammatory drug (NSAID) after x-rays of his lumbosacral spine demonstrated no pathology. He continued to complain of this low back pain now radiating posteriorly down his left leg to the mid-thigh. Physical examination is unremarkable. The most likely diagnosis is A lumbosacral strain B left S1 radiculopathy C cauda equina syndrome D L5-S1 disc herniation E lateral femoral cutaneous neuropathy
The Correct Answer is: A Low back pain is one of the more common presenting neurologic complaints to a primary care provider. Most acute pain syndromes are benign, self-limiting conditions, with pain arising from myofascial sources. Patients with back pain and normal neurologic examinations are unlikely to have any serious underlying pathology and further diagnostic testing is usually unrevealing.
Q 62.7: A 49-year-old male presents with night sweats, weight loss, coughing, and shortness of breath. A urinalysis demonstrates hematuria. The patient has had a positive PPD skin test six months ago, but did not receive treatment. Which of the following suggests reactivation tuberculosis on chest x-ray? A Fibrocavitary apical disease B Diffuse small nodular densities C Ghon complex D Kerley B lines E Pleural scarring
The Correct Answer is: A Primary tuberculosis, caused by inhalation and infection with M tuberculosis, is often without marked systemic symptoms or lung changes. Radiologic findings may include atelectasis, small infiltrates, and lympadenopathy (greatest in the hilar region), but initial x-rays may be negative. The immune system often contains the infection, and it remains latent until reactivated or appropriately treated. In some cases of primary tuberculosis, but more often in reactivated latent infection, systemic symptoms and lung infection with x-ray findings will occur. X-ray findings may include infiltrates, cavitary lesions, and nodules, with the apices and upper lobes commonly being involved. A Ghon complex represents a calcified focus of previous infection, typically containing viable bacteria. A pleural effusion may be present with active infection, and may lead to pleural scarring. Kerley B lines are associated with congestive heart failure.
Q 57.10: What is the most common blood gas abnormality in patients with a pulmonary embolism? A Respiratory alkalosis B Metabolic acidosis C Metabolic alkalosis D Respiratory acidosis E Compensated respiratory acidosis
The Correct Answer is: A Respiratory alkalosis occurs as a result of the hyperventilation.
Q 57.6: A 64-year-old male presents with fatigue, pallor, and dyspnea on exertion. He has noted this gradual onset over the last several months. His initial reason for coming in was due to new papules and nodules that he noted on his skin, with some violaceous in color. On exam you also note petechiae to the lower extremities and splenomegaly. On questioning him further, he notes feeling full sooner than he had in the past. On laboratory you note a pancytopenia. What is the most likely diagnosis? A Acute myelogenous leukemia with leukemia cutis B Hereditary Spherocytosis with drug eruption C Lymphoma with erythema nodosum D Pyoderma Gangrenosum E Sarcoidosis
The Correct Answer is: A The most likely diagnosis is AML with leukemia cutis. Leukemia cutis is seen in a subset of AML patients that are nontender, and are infiltrates of leukemic cells into the dermis. Hereditary spherocytosis is a hemolytic anemia, and does not present with papules and nodules or pancytopenia. Lymphoma does not typically present with a pancytopenia or skin rash. Pyoderma gangrenosum is an uncommon ulcerative cutaneous condition of uncertain etiology. Sarcoidosis is a chronic noncaseating granulomatous disease of unknown etiology, which affects many organs and tissues, most commonly the lungs.
Q 62.2: A 45-year-old female presents with a sudden onset of vertigo, nausea, and vomiting. Upon physical exam, you note that she is holding on to the rails of the bed, and her pain gets worse when you attempt any movement of her head. Neurologic exam is grossly normal. Which combination of the following medications is indicated to treat the patient's symptoms? A Hydrochlorothiazide, lorezapam, and gentamycin B Lorezapam, prochlorperazine, and diphenhydramine C Scopolamine, aspirin, and cisplatin D Metoclopromide, hydrochlorothiazide, and cyclobenzaprine E Diazepam, hydrocodone, and hydrochlorothiazide
The Correct Answer is: B *All other combinations include an ototoxic medication: furosemide, gentamycin, aspirin, and cisplatin.* Treatment of acute vertigo is more effective using a combination of vestibular suppressants (benzodiazepines), anti-emetics (prochlorperazine), and anticholinergics (diphenhydramine or scopolamine).
Q 59.6: A 62-year-old male presents with complaints of numbness in his hands and feet, with occasional foot drop, memory disturbance, fatigue, paleness, anorexia, nausea, and weight loss. He has a known history of diabetes and hypertension. Which of the following conditions is most likely responsible for these symptoms? A Autonomic neuropathy B Chronic renal failure C Diabetic polyneuropathy D Guillain Barre syndrome E Middle cerebral artery occlusion
The Correct Answer is: B *Chronic renal disease is associated with functional disturbances in all organ systems, including the central nervous system. Renal disease promotes CNS complications including neuropathies and neuromuscular irritability, along with systemic symptoms. The symptoms are typically progressive if the underlying renal disease is not addressed. Although other conditions promote similar neuropathies, such as diabetes, they are differentiated by the level of involvement, progression, and associated symptoms.* With Guillain Barre, an acute polyradiculoneuropathy would be expected to progress, and have associated weakness. Cerebrovascular accidents are not typically accompanied by generalized systemic symptoms, and a middle cerebral artery occlusion would be expected to have contralateral hemiparesis and hemisensory deficit.
Q 54.6: A 66-year-old man with a history of a cardiac murmur since childhood presents with complaints of increasing dyspnea while walking up one flight of stairs and increased lower extremity edema. On physical examination, a late-peaking crescendo-decrescendo murmur, preceded by a systolic ejection click, is noted. An S4 gallop is appreciated. Hepatomegaly and splenomegaly are appreciated. Which of the following is the most likely finding on echocardiogram? A Mitral stenosis B Pulmonic stenosis C Tricuspid stenosis D Atrial septal defect E Aortic regurgitation
The Correct Answer is: B Choice B, pulmonic stenosis, is the most likely finding on echocardiogram, given the patient's physical exam findings. Pulmonic stenosis can present with symptoms of right heart failure in the later stages. Patients may exhibit symptoms similar to aortic stenosis, including dyspnea with exertion, angina, fatigue and syncope, and evaluation through echocardiogram is recommended. Choice A, mitral stenosis, presents with a systolic murmur that is heard best in the left lateral decubitus position, with the bell of the stethoscope at the apex. Choice C, tricuspid stenosis, presents with a diastolic murmur that increases with inspiration. It is heard best at the left lower sternal border. Choice D, atrial septal defect, if large, could present with similar symptoms of exertional dyspnea secondary to a large shunt, but auscultation would reveal a moderately loud systolic ejection murmur that is heard best in the second and third interspaces. This is secondary to increased pulmonary arterial flow. Choice E, aortic regurgitation, presents with a soft diastolic murmur that is heard best at the left sternal border.
Q 58.9: A 22-year old female presents to your office complaining of itchy red welts all over her body now fading. She is vacationing from Florida. She has no past medical history and her only medication is an oral contraceptive, but she did take an over-the-counter dipenhydramine four days ago on the flight from Florida to calm her nerves. The welts began after swimming in the ocean in New England three days ago, lasted a few hours, then disappeared spontaneously. They reoccurred Saturday morning again shortly after swimming, lasted a little longer, and again resolved. She relates that they were intensely itchy, red, and raised. She ate out at a restaurant and had seafood Saturday night, and thought that she might be allergic to the seafood, although she ate nothing new or unusual. From the history, which of the following is the likely cause of her urticaria? A atopic dermatitis B cold urticaria C contact dermatitis from jellyfish stings D food allergy E medication allergy
The Correct Answer is: B Cold urticaria is a hypersensitivity to cold exposure (ie, wind, freezer compartments, water) resulting in histamine release. The hypersensitivity usually presents as localized redness, burning, pruritus, and urticaria in the exposed areas, or the response may progress to generalized systemic reaction, shock, and death. This condition may be familial or acquired. Familial cold urticaria is an autosomal dominant inflammatory disorder (including the Muckle-Wells syndrome), manifested as a burning sensation of the skin occurring about 30 minutes after exposure to cold. Acquired cold urticaria may be associated with medication (ie, griseofulvin) or with infection. Cold urticaria may occur secondarily to cryoglobulinemia or as a complication of syphilis. Most cases of acquired cold urticaria are idiopathic. For diagnosis, an ice cube is usually applied to the skin of the forearm for 4 to 5 minutes, then removed, and the area is observed for 10 minutes. As the skin rewarms, an urticarial wheal appears at the site that may be accompanied by itching. Second-generation antihistamines have been used as first-line treatment. Ebastine is also reported to safely and effectively prevent symptoms from acquired cold urticaria. Use of antileukotrienes in cold urticaria is anecdotal.
Q 56.8: Which of the following do the majority of patients with dissociative identity disorder also meet diagnostic criteria for? A schizophrenia B post-traumatic stress disorder C bipolar II disorder D major depressive disorder
The Correct Answer is: B Dissociative identity disorder (DID), formerly known as multiple personality disorder, is classified as a trauma spectrum disorder due to the strong link with early childhood trauma and/or maltreatment. As such, approximately 70% of DID patients also meet criteria for PTSD.
Q 58.10: A 48-year-old woman has been taking gentamicin for the last several weeks for a serious Pseudomonas aeruginosa infection. She presents today with a complaint of tinnitus. After further evaluation, it is determined that, in addition, she is also suffering from a bilateral high-frequency sensorineural hearing loss. The patient denies any dizziness or balance disturbances. Which of the following structures is most likely affected? A ear ossicles B organ of Corti C saccule D semicircular canals E tympanic membrane
The Correct Answer is: B Gentamicin belongs to the class of antibiotics known as "aminoglycosides," which are capable of producing ototoxicity as an adverse effect. Ototoxic agents target the hair (receptor) cells of the inner ear. Hair cells are found in organs involved in hearing (eg, cochlea) and balance (eg, semicircular canals, utricle, and saccule). This patient complains only of tinnitus and has sensorineural hearing loss. She denies any symptoms associated with a balance disturbance. The structure within the cochlea that contains the hair cells is the organ of Corti.
Q 55.1: A 41-year-old male presents due to concerns regarding headaches. He states that for the past month and a half he has had daily headaches, described as aching, which he notes are worse first thing in the morning and do not seem to change with activity or position. He has noted that when he drives, his left hand seems weaker on the steering wheel, and he drops things more often. He denies any other neurologic symptoms or recent illnesses. A physical exam reveals weakness of the left hand, forearm, and upper arm, with mildly decreased reflexes. There are no other significant findings. Which of the following is the most likely diagnosis? A Amyotrophic lateral sclerosis B Intracerebral neoplasm C Lyme encephalitis D Simple partial seizure E Transient ischemic attack
The Correct Answer is: B Headaches and neurological changes may be seen with many conditions. However, *intracerebral neoplasms are often associated with persistent headaches and described as worse in the morning, and may involve neurologic disturbances of many forms, based on the location of the lesion. Coordination deficits, sensory deficits, ataxia, and limb involvement are often seen with brainstem lesions.* Amyotrophic lateral sclerosis, a progressive, degenerative nerve disorder with associated weakness, may have similar symptoms, but is not classically associated with headaches. With Lyme encephalitis, signs and symptoms such as fever, vomiting, meningeal signs, and photophobia would be expected. Simple partial seizures are not associated with headaches, although focal neurologic findings are possible. Transient ischemic attacks may have associated headache and neurologic symptoms, but should have improving symptoms and resolution within 24 hours, and are not likely to recur consistently.
Q 56.7: A 30-year-old man presents to the office for follow-up on an endoscopically diagnosed gastric ulcer. At endoscopy, he was found to have a Helicobacter pylori infection and now he has completed appropriate therapy. He has another refill available on the proton-pump inhibitor. He is currently asymptomatic. What is the most appropriate follow-up on the infection? A Because he is asymptomatic, no further testing is required. B Check urea breath test or fecal antigen today. C Repeat endoscopy with histologic testing for H. pylori. D Check H. pylori serology today. E Collect stool specimen for culture.
The Correct Answer is: B Helicobacter pylori is a spiral, Gram-negative rod that resides in the gastric mucosa, where it causes PUD. It may be diagnosed by rapid urease test or by histology when endoscopy is performed. Noninvasive H pylori testing options include the urease breath test, fecal antigen testing, and serology. Serological and fecal antigen tests are the most cost-effective methods. All three noninvasive tests have sensitivities and specificities greater than 90%. Proton-pump inhibitor therapy should be discontinued 1 to 2 weeks prior to the fecal antigen or breath tests because PPIs may increase the number of false negatives. In this case, serology is the least invasive, most cost-effective, and least likely to be invalidated by the proton-pump inhibitor therapy.
Q 59.10: A 37-year-old female presents to the labor and delivery department complaining of intermittent pain and contractions. Upon arrival, she also complains of vaginal bleeding. She is a G3P2 at 39 weeks gestation; no other prenatal complications are noted. She is a non-smoker. A physical exam reveals the following: P 90, BP 130/80, T 98.7°F, abdomen gravid, positive bowel sounds, and left lower quadrant tenderness noted. A sterile speculum exam reveals the cervix to be dilated 8, fetus is cephalic, and membranes are intact. The fetal monitor reveals heart tones in the 140s with mild, decreased variability and good quality contractions noted. Delivery is felt to be imminent, and vaginal delivery has been determined to be the best course of action. What will likely decrease bleeding and shorten time to delivery? A Increased activity level B Amniotomy C Oxytocin therapy D Epidural placement E IV sedation
The Correct Answer is: B If the fetus is mature and vaginal delivery (versus c-section) has been determined to be the best course of action, then amniotomy may diminished amnionic fluid volume. This might also allow for better spiral artery compression, and serve to both decrease bleeding from the implantation site and reduce entry of thromboplastin into the maternal circulation.
Q 54.2: A 72-year-old man is transported via ambulance to the emergency department with severe chest pain and shortness of breath. Electrocardiogram (ECG) reveals ST-segment elevation in leads II, III, and aVF. While in the emergency department, he loses consciousness and is found to be in ventricular fibrillation. Resuscitation is successful, and a pulse is restored within 3 minutes. He is taken to the cardiac catheterization laboratory, where he undergoes two-vessel stenting. Two days later, his creatinine has increased from a baseline of 1.1 to 2.2 mg/dL. The next day, the creatinine is 3.9 mg/dL. Fractional excretion of sodium is ordered. You would expect this to be A <1 B >1 C unchanged from baseline D undetectable E equal to the serum creatinine level
The Correct Answer is: B Intrinsic ARF results in alterations in the kidneys' ability to respond to changes in hemostasis. When the integrity of the kidneys remains intact, sodium is conserved when GFR declines in an attempt to reestablish volume and perfusion, resulting in a fractional excretion of sodium (FENa) of <1. However, when the glomeruli are injured, the kidneys lose the ability to reabsorb sodium as the GFR decreases, and the FENa will be >1. The etiology of this patient's renal failure is most likely contrast-induced acute tubular necrosis following an ischemic episode, which is intrinsic ARF.
Q 54.16: A patient exhibits air hunger and labored, deep respirations due to increased stimulation of the respiratory center in the brain. Which of the following is the most likely cause? A Congestive heart failure B Metabolic acidosis C Obstructive sleep apnea D Respiratory acidosis E Traumatic brain injury
The Correct Answer is: B Kussmaul's respiration is a form of respiratory compensation, and is most commonly associated with metabolic acidosis. During early acidosis, breathing may be rapid, but when advanced the breaths become deep, slow and labored with an urge to breathe described as "air hunger." The other etiologies suggest other causes of breathing variation, such as tachypnea, apnea, and Cheyne-Stokes respiration.
Q 58.3: A 66-year-old man with a medical history of aortic stenosis is admitted to the hospital with increasing shortness of breath. Physical examination reveals a regular pulse of 120 beats/min, blood pressure of 95/50 mm Hg, and a respiratory rate of 32 breaths/min. The estimated jugular venous pressure (JVP) is greater than 15 cm, rales are heard halfway up the lung fields bilaterally, and a holosystolic murmur is heard at the apex. There is a tender enlarged liver with hepatojugular reflux and 2+ pretibial and pedal edema. Plain film of the chest reveals cardiomegaly and pulmonary edema. ECG is suggestive of left ventricular hypertrophy. Admission laboratory studies include the following: What type of hyponatremia does this patient most likely have? A hypovolemic hypotonic B hypervolemic hypotonic C hypovolemic isotonic D hypervolemic hypertonic E hypovolemic hypertonic
The Correct Answer is: B Most often, hyponatremia is due to excessive water retention rather than a true sodium deficiency. The first step in evaluating hyponatremia is to determine serum osmolality. Knowing whether the serum is isotonic (normal osmolality), hypotonic (low osmolality), or hypertonic (high osmolality) can help determine the etiology of the hyponatremia, and therefore, treatment. The most common causes of isotonic hyponatremia are hyperproteinemia and hyperlipidemia. The most common causes of hypertonic hyponatremia are hyperglycemia, presence of radiocontrast agents, and the presence of inactive metabolites, that is, mannitol, sorbitol, glycerol, and maltose. Treatment is aimed at correcting the underlying disorder. Most commonly, hyponatremia occurs in the setting of low osmolality (hypotonic). To further evaluate the etiology of the hyponatremia, it must be determined if the patient is hypovolemic, euvolemic, or hypervolemic. Hypovolemic hyponatremia is usually due either to extrarenal or intrarenal sodium losses. Extrarenal losses occur from dehydration, diarrhea, and vomiting. Urinary sodium measures <10 mEq/L (normal, >20 mEq/L), as the kidneys are avidly retaining sodium in an attempt to restore volume. Treatment is directed at restoring volume. Intrarenal sodium losses occur from the use of diuretics and ACE inhibitors, nephropathies, and mineralocorticoid deficiency. Urinary sodium measures >20 mEq/L. Treatment is directed at reversing the underlying cause. The most common causes of euvolemic hyponatremia are SIADH, postoperative hyponatremia, hypothyroidism, psychogenic polydipsia, and endurance exercise. In these cases, electrolyte-free water is retained, which results in a true physiologic hyponatremia. Treatment is directed at correcting the underlying abnormality and replacing sodium losses. Hypervolemic hyponatremia is caused by congestive heart failure, liver disease, nephrotic syndrome, and advanced CKD in general, anything that causes fluid retention. Treatment is directed at treating the underlying disease, restricting water intake, and facilitating excretion of water. The first step in characterizing this patient's hyponatremia is to determine the serum osmolality, which is 276 mOsm/kg. This is low, so we know that this is hypotonic. Second, we have to determine the volume status. Urinary sodium is 12 mEq, which does not indicate either intrarenal or extrarenal losses of sodium and is not consistent with hypovolemia. Given the elevated JVP, extensive edema, and hepatojugular reflux, this patient is presenting with a clinical picture of fluid overload, or hypervolemia. This patient has hypervolemic hypotonic hyponatremia due to CHF.
Q 62.10: A patient presents to your office with a sudden onset of headache, right eye pain, decreased visual acuity, nausea and vomiting. His intraocular pressure is 47. Which of the following classes of medications are indicated for treatment of this condition? A Alpha agonists and antihistamines B Alpha agonists and Beta blockers C Mydriatics D Cycloplegics E Angiotensin converting enzyme inhibitors
The Correct Answer is: B Ophthalmic alpha-agonists (brimonidine) and beta blockers (timolol) decrease aqueous humor production, and decrease intraocular pressure. They facilitate aqueous flow through outflow tract and the canal of Schlemm. Other acute treatments include prostaglandin analogs(latanoprost) and carbonic anhydrase inhibitors(acetazolamide). Mydriatics, cycloplegics(tropicamide) and antihistamines( diphenhydramine) can precipitate angle closure glaucoma in patients at risk. Angiotensin converting enzyme inhibitors are used for treating systemic hypertension.
Q 60.5: A 19-year-old woman presents to the emergency department complaining of headache. The headaches are generalized and increasing in intensity. They have not responded to over-the-counter (OTC) medications. She complains of approximately 1 week of blurred vision, intermittent diplopia, and vague dizziness. Her medical history includes obesity and acne. She takes Accutane and oral contraceptives. She is found to have bilateral papilledema, visual acuity of 20/30 on physical examination, and a normal MRI of the brain. The next most appropriate step would be A CT scan of the head B lumbar puncture C therapy with high-dose prednisone D stat cerebral arteriogram E reassurance and follow-up in the office in 6 months
The Correct Answer is: B The presence of headache associated with papilledema raises the concern for a brain tumor. The MRI excluded a mass lesion, raising a strong suspicion of pseudotumor cerebri. This is also known as benign intracranial hypertension. It is not a benign condition, however, since it causes severe headache and may result in visual loss. It is particularly frequent in obese adolescent girls and young women. The etiology is unknown but may be associated with the use of oral contraceptives, vitamin A, and tetracycline. The presentation consists of headaches caused by an increase in intracranial pressure and blurring of vision. There may be diplopia, but the remainder of the neurologic examination is unremarkable. Papilledema is virtually always part of the presentation. The mental status is normal. The differential diagnosis includes venous sinus thrombosis, sarcoidosis, and tuberculosis or carcinomatous meningitis. The last two are excluded by lumbar puncture. An abnormal cerebrospinal fluid is not consistent with pseudotumor cerebri. The diagnosis is made by excluding mass lesions with CT scan or MRI and demonstrating markedly increased intracranial pressure by lumbar puncture. The treatment involves weight loss, diuretics, and steroids. Repeat lumbar punctures to remove cerebrospinal fluid and decrease intracranial pressure are effective. In cases that are unresponsive to these measures, lumbar-peritoneal shunting is effective, as is unilateral optic nerve sheath fenestration. Effective treatment can improve headaches and prevent vision loss.
Q 60.3: You are monitoring a 30-year-old G2P1 at 40 weeks gestation, who is in an active stage of labor and is 6-cm dilated. The fetal heart tracing has a baseline heart rate of 140, with 7 to 10 beats of variability. With the last five contractions you have noted late decelerations. What would be the next most appropriate course of action? A Close observation of FHR tracing B Assessment of dilatation C Augment contractions with oxytocin D Intravenous analgesic E Surgical intervention
The Correct Answer is: B The presence of recurrent late decelerations should raise the suspicion for fetal distress. Vaginal evaluation for change in dilatation or cord prolapse, and to assess the fetal response to stimulation, are the first steps in evaluating the need for intervention.
Q 60.6: Which of the following is an absolute contraindication to acute thrombolysis, with a recombinant tissue plasminogen activator (tPA) in an ischemic stroke patient? A Platelet count of 120,000/mm3 B Ischemic stroke two months ago C Symptoms for three hours D Current daily low dose aspirin regimen E Systolic blood pressure of 170mmHg
The Correct Answer is: B The role of thrombolysis with recombinant tPA (tissue plasminogen activator) during acute ischemic stroke has been clearly defined. In select patients without contraindications, the use of tPA has been associated with decreased mortality, decreased disability, and improved clinical outcome. However, tPA has defined contraindications to use
Q 60.8: A 41-year-old woman presents with complaints of weight gain, infrequent menses, and mood changes. You observe her to have moon facies, centripetal fat distribution, and purple striae on her abdomen (see Figure 4-3). Her blood pressure is 152/98 mm Hg. What is the first step in confirming this diagnosis? (Reproduced, with permission, from Fauci AS, Braunwald E, Kasper DL, et al. Harrison's Principles of Internal Medicine, 17th edition. New York: McGraw-Hill, 2008: 2255.) A random cortisol level B overnight dexamethasone suppression test C thyroid studies D MRI
The Correct Answer is: B This patient's signs and symptoms indicate possible Cushing syndrome. Overnight dexamethasone testing is the most widely used test, with normal results excluding Cushing syndrome. Cortisol levels are not useful because of diurnal variations.
Q 57.1: In a person with normal marrow function what is the mean life span of platelets? A 3 to 6 days B 7 to 10 days C 11 to 14 days D 15 to 18 days E 19 to 22 days
The Correct Answer is: B Under normal conditions, human platelets have a mean life span in circulation of 7 to 10 days. In order to confirm that the thrombocytopenia is only related to the surgery, waiting at least a week for full platelet conversion is needed.
Q 61.7: Your 24-year-old female patient has a longstanding history of hypercoaguability disorders and is on lifelong anticoagulation due to prior pulmonary emboli. She advises you that she is attempting to get pregnant. Which of the following medications should be utilized during her pregnancy? A aspirin B enoxaparen C heparin D warfarin E none of the above
The Correct Answer is: B Warfarin therapy is contraindicated in the first trimester due to its association with fetal chondrodysplasia punctata. In the second and third trimesters, warfarin may cause fetal optic atrophy and mental retardation. In general, all diagnostic and therapeutic modalities afforded the nonpregnant patient should be utilized in pregnancy. Outpatient anticoagulant therapy, with low-molecular-weight heparin (LMWH) such as enoxaparen, is indicated in pregnant women at high risk for deep venous thrombosis (DVT). LMWH may be associated with an increased risk of epidural hematoma in women receiving an epidural anesthetic in labor. One approach to this problem is to switch from LMWH to unfractionated heparin before the anticipated delivery date.
Q 55.3: You have just diagnosed Mr. Jones, a 31-year old male, with community-acquired pneumonia. He has no comorbidities and was treated for a strep throat with antibiotics within the last three months. According to the Infectious Disease Society of America (ISDA) and American Thoracic Society (ATS) guidelines, the first-line antibiotic choice would be? A a cephalosporin B a respiratory fluoroquinolone C macrolide D trimethoprim-sulfamethoxazole (TMP-SMZ) E an augmented semi-synthetic penicillin
The Correct Answer is: B With the emergence of other pathogens causing pneumonia and the development of resistance to penicillin and other drugs in S pneumoniae, treatment decisions have become more complex. The 2003 update to ISDA and ATS guidelines for the treatment of community-acquired pneumonia differ depending on the health and age of patients (ie, 65 years or older), whether they have recently been treated with an antibiotic, and whether they are at risk for an aspiration pneumonia or influenza superinfection. For patients with no serious comorbidities, the ISDA/ATS recommends a respiratory quinolone or an advanced macrolide plus high-dose amoxicillin (or amoxicillin-clavulanic acid) as first-line therapy. If an antibiotic has been used recently, a respiratory quinolone or an advanced macrolide, plus a second- or third-generation cephalosporin are recommended options by the ISDA/ATS.
Q 58.6: Which of the following best describes the mechanism of action of angiotensin-converting enzyme (ACE) inhibitors in controlling blood pressure and preventing or slowing kidney damage? A They result in systemic vasodilation. B They increase renal tubular excretion of sodium. C They result in dilation of the efferent arteriole, reducing glomerular pressure. D They block the angiotensin II receptor on the cell membrane. E They reduce production of angiotensinogen, the precursor to angiotensin I.
The Correct Answer is: C *ACE inhibitors prevent the conversion of angiotensin I to angiotensin II, thereby interrupting the renin-angiotensin-aldosterone system, which regulates blood pressure. The glomerular efferent arteriole dilates, given the decreased stimulus from angiotensin II to constrict. This lowers pressure in the glomerulus by lowering resistance to outflow. This effectively results in a decrease in GFR, resulting in increased serum creatinine and potassium levels. However, these changes are not necessarily indications to discontinue the ACE inhibitor. Usually, the creatinine increases 0.2 to 0.4 mg/dL and then levels out.* Monitoring serum creatinine and potassium levels is indicated. If only mild increases occur and stabilize, or if there are no changes, the ACE inhibitor can, and should, be continued so that the patient derives the beneficial effect of the decline in pressure within the glomerulus, which will slow down the progression of CKD.
Q 57.4: A 42-year-old man is prescribed a drug classified as a "bile acid sequestrant." Which of the following will most likely be diminished as a result of administering this drug? A activity of pancreatic amylase B bile flow from the gall bladder to the duodenum C emulsification of triglycerides D secretion of gastric acid E synthesis of plasma proteins
The Correct Answer is: C Bile acids are constituents of the bile that are synthesized into bile salts by liver hepatocytes or by intestinal bacteria. One of the major roles of bile salts is to emulsify dietary lipids within the lumen of the small intestine. Emulsification is an important first step in lipid digestion, as it increases the surface area for lipases to function. Bile salts also help to form micelles, which transport the products of lipid digestion from the lumen and into the cells (enterocytes) lining the small intestine. Without bile salts, digestion and absorption of dietary lipids is incomplete and therefore decreased.
Q 54.11: Which of the following foodborne illnesses is most likely to be acquired through eating raw oysters? A salmonellosis B shigellosis C cholera D giardia E hookworms
The Correct Answer is: C Cholera infection is most often caused by Vibrio cholerae, Vibrio vulnificus, or Vibrio parahaemolyticus. Although cholera infection can be transmitted through wounds, the most often reported cause is eating undercooked shellfish. The clinical picture most often includes a watery diarrhea that can lead to dehydration. In the immune compromised host, overwhelming sepsis is possible. It is most often treated with doxycycline plus a third-generation cephalosporin or by a fluoroquinolone alone. Salmonella infection is associated with consumption of raw eggs and undercooked chicken or beef. Shigella is transmitted by the fecal-oral route, often because of poor hygiene. Giardia is waterborne and hookworms are found in the soil.
Q 61.2: A 36-year-old male with a history of acute lymphoblastic leukemia (ALL) is treated with high dose methotrexate and glucocorticoids, vincristine, and L-asparaginase at age 10. He has been in remission since induction therapy was completed. He presents with pain to his right hip/back area, which radiates down his leg. He is also having some back pain that keeps him up at night. What is the most likely diagnosis? A Bladder cancer with referred pain B Lymphoma with metastasis to the bone C Osteonecrosis of the right hip D Osteoporosis E Relapsed ALL
The Correct Answer is: C Common treatment sequelae that can present years out from therapy include osteonecrosis, especially following the use of high dose methotrexate and glucocorticoids. Lymphoma can arise as a post transplant complication, but is not typically seen following anti-leukemic therapy. Osteoporosis is also a complication of anti-leukemic therapy, but is more associated with the use of mercaptopurine. Relapsed ALL is typically seen in the first two years following therapy, but can arise more than 10 years out. Typically, this is seen with fever, pancytopenia, and bony pain. The pain is not typically isolated to one side of the body.
Q 55.9: A 55-year-old man with a history of chronic renal failure, 6 months status post renal transplant, presents with chest pain, productive cough, and low-grade fever. He reports generalized malaise as well. Current medications include only those related to the transplant. He has no known allergies. Examination reveals a temperature of 102°F, unremarkable HEENT (head, ears, eyes, nose, throat), and few crackles anteriorly in the upper right lung field. Chest X-ray reveals a solitary nodule in the right upper lobe. The most likely etiology for his symptoms is A Streptococcus pneumoniae B Pneumocystis jiroveci C cryptococcosis D Candida E influenza A
The Correct Answer is: C Cryptococcal species are opportunistic organisms responsible for infections in immune-compromised hosts. With the rise of HIV infections in the past few decades in the United States, cryptococcosis is becoming increasingly prevalent. It is also a common infection in those who have undergone solid organ transplantation. The two most common areas for infection are the lungs and the central nervous system. Pulmonary involvement includes fever, productive cough, chest discomfort, and weight loss. Pleural effusions, lymphadenopathy, and solitary or multiple nodules can all be seen on chest x-ray. Central nervous system manifestations include meningitis and meningoencephalitis. Diagnosis is confirmed with India ink prep of cerebrospinal fluid showing yeast or histologic stains of tissue from the involved organs. Treatment is with oral or parenteral antifungal agents.
Q 57.9: Which disorder is characterized by episodes of hypomania and depression for greater than 2 years? A dysthymia B major depressive disorder C cyclothymia D bipolar E mood disorder
The Correct Answer is: C Cyclothymia is characterized by symptoms of depression and hypomania for at least 2 years. Symptoms are milder than a regular depressive or manic episode. Occasionally, patients will have regular depressive or manic symptoms at which time they need to be reclassified as bipolar.
Q 60.2: Which of the following medications, used for the treatment of Tourette syndrome, has been associated with long-term extrapyramidal side effects? A Clonazepam B Clonidine C Haloperidal D Metoclopramide E Risperidone
The Correct Answer is: C Haldol, a typical antipsychotic agent, has been shown to be effective for symptomatic treatment of patients with Tourette syndrome. Haldol blocks dopaminergic action and decreases psychomotor agitation. It is linked to a high frequency of extrapyramidal side effects, likely due to this action. It also has sedative properties within the limbic system. Other medications, such as atypical psychotics, benzodiazepines, alpha-2 agonists, and dopamine-blockers have been used for Tourette management. Each medication should be monitored for effectiveness and side effects, as other medications can cause extrapyramidal side effects, such as metoclopramide.
Q 54.7: A 29-year-old man who is HIV positive has developed dark purple papular nonblanching lesions between the toes of his right foot. He has no other symptoms. Careful examination shows that this is the only area of involvement. His CD4 count is 150 cells/mL. Of the following, what is the most appropriate treatment? A alpha interferon B chemotherapy with daunorubicin, bleomycin, and vinblastine C intralesional vinblastine D liposomal doxorubicin E radiation
The Correct Answer is: C Kaposi sarcoma that is in a limited area of the skin may be treated with intralesional vinblastine or by simply observing it over time. Liposomal doxorubicin and alpha interferon are used for extensive or aggressive skin disease, while combination chemotherapy is used for visceral disease.
Q 54.8: An 18-year-old female presents to your office with the complaint of palpitations for the last 2 months. The episodes are frequent and are accompanied with lightheadedness and shortness of breath. The patient's mother has taken her pulse when some of the episodes occur and states that the rate gets as high as 170 beats per minute. On exam, she is alert, awake, and oriented. Her resting pulse is 80 and her blood pressure is 122/65. Her lungs are clear throughout, and her cardiac exam revealed a regular rate and rhythm, without murmurs, rubs, or gallops. The patient has been treated with flecanide for several months, and has done well until she started experiencing more episodes of tachycardia. Her blood pressure remains stable in the 125 to 135 systolic range, and her symptoms are mild when the tachycardia occurs. What therapy should be given next for this patient? A Stop flecanide B Ablation therapy C Beta blocker with flecanide D Beta blocker without flecanide E Stop flecanide, add diltiazem
The Correct Answer is: C Patients currently on flecanide with ongoing symptoms can benefit from the addition of a beta-blocker, to help control rate and symptoms. This would be started prior to ablation therapy, provided that the patient's vital signs can tolerate the added medication. A calcium channel blocker would potentially worsen the patients arrhythmia.
Q 54.17: A 24-year-old male has an eight-month history of loose thought associations, social withdrawal, auditory hallucinations and deterioration in personal appearance and hygiene. Upon examination, he is noted to have a flat affect and perceptual distortions, and he behaves like he is detached from his own actions. Which of the following medications, if chosen for treatment, is known to be associated with lens changes and would require an eye examination to detect cataracts every six months? A Chlorpromazine (Thorazine) B Haloperidol (Haldol) C Quetiapine (Seroquel) D Risperidone (Risperdal) E Ziprasidone (Geodon)
The Correct Answer is: C Quetiapine is the only medication listed that is associated with lens changes, and an eye examination every six months, starting at the initiation of treatment, is recommended.
Q 59.2: A 20-year-old woman was just told by her new sexual partner that she needed to be checked for a sexually transmitted infection because he has developed dysuria and a profuse urethral discharge. She herself has had a subjective fever for the past two days, some nausea but no vomiting, diffuse lower abdominal pain, and a severe backache. On examination, she has a temperature of 100.5˚F, hypoactive bowel sounds, bilateral lower abdominal quadrant tenderness, a profuse mucopurulent cervical discharge and pronounced cervical motion tenderness. Serum pregnancy testing is negative. She is given an injection of ceftriaxone and a prescription for doxycycline for 14 days and an appointment for follow up the next day. Under which of the following conditions should metronidazole be added to her regimen? A If she does not appear improved by the following morning B If she develops vomiting or diarrhea C If she has a probable tubo-ovarian abscess D If she has an allergic reaction to the doxycycline E If she has had more than one partner in the past month
The Correct Answer is: C Recommended regimens for treatment of pelvic inflammatory disease include ceftriaxone or another parenteral third-generation cephalosporin and doxycycline or cefoxitin, probenecid, and doxycycline. Metronidazole (or clindamycin) should be added to either regimen if a tubo-ovarian abscess is present. If being treated as an outpatient and she has not improved in 3 days (A), or if she develops vomiting (B), she should be hospitalized rather than given metronidazole. Metronidazole is not a satisfactory substitute for doxycycline (D). The patient's condition rather than the number of sexual partners (E) determines the appropriate therapy.
Q 63.1: A 27-year-old African American with sickle cell anemia presents to the emergency department with acute onset intractable pain. She is taking quick, shallow breaths and her oxygen saturation is 84% on room air. She appears desiccated, states she hasn't eaten in the last 24 hours, and says that she "just doesn't feel well." She is also afebrile. What should your next course of action be? A Start morphine, hydrate, and start antibiotics B Start morphine, oxygen, and start antibiotics C Start oxygen, hydrate, and exchange transfusion D Start oxygen, hydrate, and give pneumococcal vaccination E Start oxygen, hydrate, and start antibiotics
The Correct Answer is: C Start oxygen, hydrate, and exchange transfusion Exchange transfusions are primarily indicated for the treatment of intractable pain crises, priapism, and stroke. Patients should be kept well hydrated, and oxygen should be given if the patient is hypoxic. Antibiotics would be used if there was an infection identified, but are not part of initial treatment in a sickle crises.
Q 56.10: Which of the following is a major contraindication for surgical resection of a lung carcinoma? A Chest wall invasion B Non-malignant pleural effusion C Superior vena cava syndrome D Unilateral endobronchial tumor E Vagus nerve involvement
The Correct Answer is: C Surgical resection of lung carcinoma is contraindicated in cases of superior vena cava syndrome, extrathoracic metastases, heart, pericardial or great vessel involvement, recurrent laryngeal or phrenic nerve involvement, esophageal or carina involvement, malignant effusion, or contralateral mediastinal lymph nodes. Other contraindications are patient and staging dependent.
Q 60.1: A 54-year-old woman has suffered a stroke that has resulted in dramatic changes to her personality, left leg and foot weakness, loss of sensation in the left leg, and apathy. Which of the following arteries was most likely affected by the stroke? A left anterior cerebral B left middle cerebral C right anterior cerebral D right middle cerebral E right posterior cerebral
The Correct Answer is: C The anterior cerebral arteries supply the frontal lobes as well as the medial aspects of the parietal and occipital lobes rostral to the parietooccipital sulcus. The prefrontal cortex of the frontal lobe is concerned with a person's personality, depth of feeling, and initiative. Hence, occlusion of an anterior cerebral artery can cause neuronal injury to this area, leading to feelings of apathy and personality changes. The paracentral lobule represents the medial aspects of the precentral gyrus (frontal lobe) and postcentral gyrus (parietal lobe), which are responsible for motor control and somatosensory perception, respectively, of the leg and foot. Hence, occlusion of an anterior cerebral artery can produce contralateral hemiparesis and hemisensory loss involving the leg and foot. With the 54-year-old patient, the symptoms were occurring on the left side, which points to a right anterior cerebral artery occlusion.
Q 60.7: A 74-year-old woman presented with the new onset of seizures. An MRI with gadolinium showed a parasagittal mass with homogenous enhancement and a "dural tail." What is the appropriate management of this patient? A Radiation therapy B Chemotherapy C Surgical resection D Surgical resection with chemotherapy
The Correct Answer is: C The clinical presentation is consistent with a meningioma. Meningiomas are commonly located in the parasagittal region, the convexity of the brain, sphenoid ridge, or posterior fossa. Radiographic features on MRI include homogenous enhancement and evidence of a "dural tail" indicating the origin of the tumor. Since meningiomas are a benign tumor, the primary treatment is surgical removal. In the event of a subtotal resection or if the meningioma is found to be malignant, surgical resection is followed by radiation therapy.
Q 57.5: A 36-year-old female patient is being treated for thyroid disease and returns to your family practice office for a six month exam. She is on 100 ugm of levothyroxine once daily. Her vital signs are a blood pressure of 128/82 mm Hg, pulse 62 bpm, respirations 12 bpm, and she is afebrile. Her physical examination is normal. Her thyroid-stimulating hormone (TSH) level is 11.5 uIU/mL (normal 0.4 to 5.0 uIU/mL). What is your next step in her treatment? A Draw a free T3 and free T4. B Lower her levothyroxine to 88 ugm daily and recheck her TSH in 4 to 6 weeks to adjust if necessary. C Increase her levothyroxine to 112 ugm daily and recheck her TSH in 4 to 6 weeks to adjust if necessary. D She is asymptomatic so do not adjust her levothyroxine, but recheck her level at her next visit in six months.
The Correct Answer is: C The thyroid gland operates on a reverse feedback mechanism. When TSH is high, free T 3 /T 4 is low and need repletion, and vice versa. In this case, the patient has an elevated TSH, indicating that her serum T 3 /T 4 is likely low and she should have her dose of levothyroxine adjusted upwards and levels can be rechecked starting in approximately four weeks and adjusted subsequently as necessary.
Q 54.14: A 42-year-old woman, with a history of struvite renal calculus, calls the office with a complaint of a urinary tract infection. As part of the interview, she reports intermittent, mild right flank pain for 4 days. Her urine dipstick is positive for microscopic hematuria, and the urine pH is 7.5. The KUB film is positive with two visible stones in the right kidney. Which of the following organisms is most likely to be cultured from the urine specimen? A Escherichia coli B Klebsiella C Proteus D Chlamydia trachomatis
The Correct Answer is: C This patient has struvite stones. They are frequently associated with recurrent urinary tract infections, visible stones, and high urine pH. These stones are formed by urease-producing organisms including Proteus and Pseudomonas while being caused less commonly by Klebsiella. Struvite stones are not typically caused by E. coli and C. trachomatis.
Q 59.1: A 40-year-old female is status post a dilatation and curettage for hydatidiform mole. On week 3 post surgery, her follow-up quantitative hCG level has elevated slightly. What is the most likely diagnosis? A Adenocarcinoma of the ovary B Adenocarcinoma of the uterus C Retained hydatidiform mole D Choriocarcinoma E Corpus luteum cyst
The Correct Answer is: D 5% of hydatidiform mole progress to choriocarcinoma; the longer the mole in intrauterine the higher the risk. Pre-surgical evaluation for mole removal includes a chest x-ray to rule out distant metastasis. hCg that either plateuas or elevates is choriocarcinoma until proven otherwise, and requires prompt evaluation.
Q 62.8: A 35-year-old homeless male presents with a painful red right eye and decreased visual acuity, which occured over the past 48 hours. He doesn't recall any trauma, and prior to this his vision was good. His past history includes alcoholism and liver disease. On physical exam you notice a white opacity in the center of his right cornea. You are unable to do an ophthalmoscopic exam due to the opacity, and a fluorescein staining is strongly positive. What is the likely etiology of the white opacity? A Hypopyon B Hyphema C Corneal Infiltrate D Corneal Ulcer E Corneal Abrasion
The Correct Answer is: D A rapidly progressing central corneal ulcer must be considered first and treated aggressively. An ulcer will show fluorescein staining, due to a break in the corneal epithelium. Pseudomonas, strep pneumonia, herpes, and fungus must be considered as possible causes. An emergent ophthalmology consult can be sight saving.
Q 58.5: Which of the following diagnostic studies is indicated for a patient with amaurosis fugax? A CT of the head B Intraocular pressure C Temporal artery biopsy D Carotid ultrasound E Ocular fluorescein angiogram
The Correct Answer is: D Amaurosis fugax is a monocular vision loss that appears like a curtain passing over the eye, and comes from carotid artery disease. A CT of the head is indicated for lateralizing stroke symptoms. Intraocular pressure is taken for evaluation of chronic or acute glaucoma. A temporal artery biopsy is taken if giant cell arteritis is suspected. An ocular fluorescein angiogram is done to evaluate retinal disorders.
Q 54.1: A 70-year-old man, with a history of pulmonary hypertension and obstructive sleep apnea, presents with complaints of increasing dyspnea while walking his dog. He has also recently noted increased lower extremity edema. On physical examination, jugular venous distension is noted. Auscultation of the chest demonstrates a high-pitched blowing diastolic murmur. The murmur is heard over the second and third left intercostal spaces. An S 3 is appreciated. Abdominal exam reveals hepatomegaly and splenomegaly. Which of the following maneuvers would be the most appropriate to choose for better identification of the murmur? A Left lateral decubitus position listening with the bell of the stethoscope B Standing C Seated leaning forward D Inspiration E Expiration
The Correct Answer is: D Choice D, inspiration, will increase the intensity of the murmur of pulmonic regurgitation/insufficiency. The Valsava maneuver will diminish the intensity of the murmur. Choice B, standing, will cause the mid-systolic click associated with mitral valve prolapsed to move toward S1 or become more audible. Choice A, rolling the patient to the left lateral decubitus position, is most useful in identification of the murmur of mitral stenosis. Choices B, C, and E, have no effect on the Graham Steell murmur of pulmonic regurgitation.
Q 58.1: A 62-year-old man with a history of hypertension, diabetes mellitus type 2, hyperlipidemia, and chronic tobacco use presents to the office with complaints of retrosternal chest pressure radiating down his left arm, associated with diaphoresis, nausea, and dyspnea for the last 45 minutes after mowing his lawn. The patient's vital signs are stable, and on physical examination a new systolic murmur is appreciated. His EKG demonstrates normal sinus rhythm with minor nonspecific changes. Which of the following would be the most appropriate next step in management of this patient? A Discharge patient with anxiolytic therapy B Discharge patient with NSAID therapy C Transesophageal echocardiogram D Repeat EKG with addition of leads V7 through V9 E Transthoracic echocardiogram
The Correct Answer is: D Choice D, repeating the EKG with addition of the posterior leads (leads V 7 through V 9 ), can uncover hidden posterior ST-segment elevation in patients with symptoms suggestive of acute coronary syndrome (Figure 4). These patients should then be treated following acute STEMI guidelines, including acute reperfusion therapy. A new systolic murmur would not occur with anxiety or costochondritis, and in a patient with numerous cardiac risk factors, with symptoms suggestive of acute coronary syndrome, neither anxiolytics nor anti-inflammatory therapy would be appropriate. Choices C and E may be considered later in this patient's treatment plan, once acute myocardial infarction has been ruled out, for further investigation of this patient's new murmur. (Tintinalli et al., 2011, Chapter 52)
Q 55.8: A 25-year-old female presents to your office complaining of intermittent episodes of shortness of breath, especially at night and when she gets a cold. She has smoked one pack per day for 11 years and is asymptomatic on physical examination, with clear breath sounds and stable vital signs. She had "a touch of asthma" as a child, having been to the emergency department on numerous occasions. What would be the first step in your diagnosis and treatment plan for this patient? A Provide smoking-cessation counseling, prescribe a short-acting beta2-agonist inhaler, and give the patient education in utilizing the medication with a follow-up appointment in one month. B Provide smoking cessation counseling, prescribe an anti-cholinergic inhaler, and give the patient education in the use of the medication with a follow-up appointment in one month. C Perform a peak flow and compare it to the patient's predicted peak flow; if normal (or nearly so) reassure the patient and ask her to return if she develops the symptoms again. D Perform spirometry, administer a nebulizer treatment, and after waiting 20 minutes repeat the spirometry; after interpretation to differentiate between asthma and chronic obstructive pulmonary disease, treat accordingly. E Refer the patient to pulmonary medicine.
The Correct Answer is: D Clinical, imaging, and laboratory findings should enable the clinician to distinguish chronic obstructive pulmonary disease (COPD) from other obstructive pulmonary disorders such as bronchial asthma, bronchiectasis, cystic fibrosis, bronchopulmonary mycosis, and central airflow obstruction. Simple asthma is characterized by complete or near-complete reversibility of airflow obstruction. Bronchiectasis is distinguished from COPD by features such as recurrent pneumonia and hemoptysis, digital clubbing, and characteristic imaging abnormalities. Patients with severe α 1 -antiprotease (α 1 -antitrypsin) deficiency are recognized by family history and the appearance of panacinar bibasilar emphysema early in life, usually in the third or fourth decade; hepatic cirrhosis and hepatocellular carcinoma may occur. Cystic fibrosis occurs in children and younger adults. Rarely, mechanical obstruction of the central airways simulates COPD. Flow-volume loops may help separate patients with central airway obstruction from those with diffuse intrathoracic airway obstruction characteristic of COPD. Spirometry provides objective information about pulmonary function and assesses the results of therapy. Pulmonary function tests early in the course of COPD reveal only evidence of abnormal closing volume and reduced midexpiratory flow rate. Reductions in FEV 1 and in the ratio of forced expiratory volume to vital capacity (FEV 1 % or FEV 1 /FVC ratio) occur later. In severe disease, the FVC is markedly reduced. Lung volume measurements reveal a marked increase in residual volume (RV), an increase in total lung capacity (TLC), and an elevation of the RV/TLC ratio, indicative of air trapping, particularly in emphysema. Arterial blood gas measurements characteristically show no abnormalities early in COPD other than an increased A-a-Do 2 . Indeed, they are unnecessary unless (1) hypoxemia or hypercapnia is suspected, (2) the FEV 1 is < 40% of predicted, or (3) there are clinical signs of right heart failure. Hypoxemia occurs in advanced disease, particularly when chronic bronchitis predominates.
Q 55.10: A 46-year-old man with a history of EtOH abuse is brought to the emergency department in the morning by his wife. She has noted that he has developed tremors in both arms, and he seems mildly confused to her. He complains of feeling weak, with some cramping in the legs. On physical examination, his blood pressure is noted to be 162/95 mm Hg, and his heart rate is 108 beats/min. There is no asterixis. Which of the following electrolyte disorders are you likely to find in this patient? A hypercalcemia B hypocalcemia C hypermagnesemia D hypomagnesemia E hyperphosphatemia
The Correct Answer is: D Hypomagnesemia is a common finding in the patient who abuses alcohol. Other leading causes include diarrhea, diuretics, aminoglycosides, and amphotericin B. The etiology of hypomagnesemia in the patient with a history of alcohol abuse is thought to be a combination of malabsorption and inadequate dietary intake, possibly with alcohol exerting an antagonistic effect on absorption. Signs and symptoms are those of neuromuscular and central nervous system hyperirritability, including weakness and muscle cramps, tremors, nystagmus, a positive Babinski response, confusion, and disorientation. Hypertension, tachycardia, and ventricular arrhythmias may develop.
Q 61.5: A 70-year-old man with a history of hypertension, DM Type 2, and hyperlipidemia is seen for pre-operative evaluation prior to left knee replacement. On auscultation, a very soft high-frequency decrescendo early diastolic murmur is heard at the upper left sternal border. Utilizing isometric hand grip exercises, the murmur increases in intensity and can be heard radiating to the left sternal border and apex. Given the patient's physical exam findings, which of the following is the most likely diagnosis? A Aortic stenosis B Ventricular septal defect C Mitral stenosis D Aortic regurgitation/insufficiency E Mitral regurgitation/insufficiency
The Correct Answer is: D Isometric hand grip exercises will increase the intensity of the murmur of aortic regurgitation, which is usually described as a high-frequency decrescendo early diastolic murmur that is heard best at the left upper sternal border or at the right upper sternal border. Radiation, if it occurs, is frequently to the lower left sternal border and the apex. Isometric hand exercises increase arterial and left ventricular pressure, which increases the flow across the aortic valve, thereby increasing the murmur's intensity.
Q 58.2: The triad of "dermatitis, diarrhea, and dementia" (pellagra) results from a severe deficiency of which of the following vitamins? A thiamine B vitamin K C riboflavin D niacin E pyridoxine
The Correct Answer is: D Niacin deficiency is known as pellagra. It is rare in the United States and is most often a complication of alcoholism or malabsorption syndrome. Clinical signs of pellagra are known as the 3 Ds—dermatitis, diarrhea, and dementia.
Q 54.15: Tumor necrosis factor (TNF) inhibitors are most often considered for use in patients with rheumatoid arthritis (RA) that does not respond to initial therapy. Which of the following screenings should occur before a patient is placed on this class of medication? A chest x-ray B allergy testing C liver function tests D purified protein derivative (PPD) test E serum BUN (blood urea nitrogen) and creatinine test
The Correct Answer is: D Patients being treated for rheumatoid arthritis with tumor necrosis factor (TNF) inhibitors are at increased risk for developing an opportunistic infection, such as tuberculosis (TB). It is recommended that screening for the presence of latent TB occur before TNF inhibitors are started. There is no specific indication to order a chest x-ray, allergy testing, liver function tests, or serum BUN and creatinine prior to initiation of TNF inhibitors.
Q 62.3: A 37-year-old woman under your care is diagnosed with bipolar I disorder. As part of her drug regimen, you prescribe lithium carbonate as long-term maintenance therapy. Which of the following would be most appropriate to perform or order prior to the initiation of lithium carbonate? A electrocardiogram B fasting plasma glucose C liver function tests D serum creatinine E urine culture
The Correct Answer is: D Patients on chronic lithium carbonate therapy have an approximate 10% to 20% risk of developing renal problems such as glomerulosclerosis, tubular atrophy, or interstitial nephritis. Each of these conditions can lead to filtration problems and a subsequent rise in serum creatinine. Hence, it is advised to obtain a baseline serum creatinine prior to administering lithium carbonate to follow any changes that may occur in renal function during therapy. It is also advised that lithium carbonate be avoided in patients with pre-existing renal disease.
Q 56.5: Which of the following U.S. Preventive Services Task Force (USPSTF) recommendations for prostate cancer is correct? A All African-American men over 40 years old should have a digital rectal examination (DRE) and a prostate-specific antigen (PSA) screening annually. B All patients should receive a DRE and PSA screening at 50 years of age. C All high-risk patients should receive a PSA screening annually starting at 50 years of age. D DREs are not recommended and PSA screening is recommended based upon patient risk, but only after discussing the benefits and risks with the patient and the patient expressing the desire to have the examination performed.
The Correct Answer is: D Prostate cancer screening in older men is a particularly challenging topic because significantly more older men die with prostate cancer than from it. Nonetheless, 92% of prostate cancer deaths occur in men who are older than 65 years. Although prostate cancer can be detected earlier via PSA testing, mortality from prostate cancer has not been reduced as a result of screening. The USPSTF has concluded that evidence to recommend for or against screening is lacking. Other organizations such as the American Cancer Society and American Urological Society recommend screening among those with at least a ten-year life expectancy. The matter is complicated by the fact that techniques to treat prostate cancer often greatly affect quality of life. Furthermore, we are not able to reliably predict whether a particular prostate cancer is life-threatening. Given the uncertainties, PSA screening should be reserved for the most robust of older men and only after a thorough discussion of the arguments for and against screening.
Q 58.7: A 42-year-old male with a past medical history of renal failure and diabetes type II presents with facial swelling and pain. He states that it has been getting worse since it started five days ago. He also states that the side of his cheek became acutely swollen and painful five days ago when he was eating. His physical exam reveals a markedly swollen left submandibular space, with a firm and tender 1.5 cm nodule, palpable near the mandible, on the left side. When pressed, pus is seen coming out of the submandibular salivary duct. What is the most likely diagnosis? A Sialolithiasis B Parotitis C Ludwig's angina D Sialadenitis E Dental abcess
The Correct Answer is: D The history and physical exam is consistent with suppurative sialadenitis. The preceding episode of pain and swelling while eating indicates that the patient may have a salivary duct stone, which predisposed the patient to the salivary gland infection. Ludwig's angina is a bilateral submandibular and sublingual abcess, most often caused by infected mandibular molars. Parotitis is seen in mumps, a bilateral inflammation of the parotid glands caused by paramixovirus. Sialolithiasis are salivary duct stones without infection, are unilateral, and can affect the sublingual, submandibular, and parotid glands.
Q 54.5: What is the most common site of an acute arterial occlusion due to embolic disease? A Iliac artery B Aortic bifurcation C Mesenteric arteries D Femoral artery E Popliteal artery
The Correct Answer is: D The most common site for an acute embolic occlusion is the femoral artery. Other common sites include the axillary, popliteal, and iliac arteries as well as the aortic bifurcation and mesenteric vessels. The majority (80%) of arterial embolic originate in the heart in patients with atrial fibrillation or from mural thrombi in the left ventricle from an akinetic or dyskinetic portion of the myocardium following a myocardial infarction.
Q 56.3: A 22-year-old African American male presents to the emergency department with shortness of breath, which started 2 hours prior to arrival. He does not have a history of pulmonary disease that he is aware of, and he states that in the past at random events he has had similar episodes. He does nothing to get the episodes to stop, and he also states that he feels his chest pounding at the same time of the shortness of breath. He has no medical history that he is aware of, and he takes no medications or any illicit drugs. On examination he is alert, awake, and oriented. His vital signs show T 99.0, P 142, R 18, and BP 132/82. His chest x-ray is negative for any acute cardiopulmonary disease, and his electrocardiogram has an irregularly irregular rate of 142 with visible delta waves. Given the clinical situation above, what is the best medication for managing this patient's condition with a long-term approach? A Amiodarone B Atropine C Hydralazine D Flecainide E Digoxin
The Correct Answer is: D This patient has Wolff-Parkinson-White (WPW) syndrome along with atrial fibrillation and a rapid ventricular response. Of the choices given in managing this patient's tachycardia, oral flecainide (D) will serve to slow the process within the accessory pathway and prolong the refractory period. Amiodarone (A) has been shown to not be effective in managing the patient's tachycardia with respect to WPW, and the other medications (B, C, and E) would not have an effect on the condition and may actually worsen the patient's symptoms.
Q 57.3: A 29-year-old female who is only taking oral contraceptives presents to the emergency department with a 1-day history of worsening shortness of breath. On examination, the patient is afebrile, her pulse rate is 105, respiratory rate is 24, and blood pressure is 122/78. She has wheezing to all lung fields and appears to be in mild to moderate distress. There is no swelling or edema to the lower extremities. Based on these findings, what is the best test to order to determine the diagnosis in this patient? A CT chest B Chest x-ray C Ventilation to perfusion scan D Pulmonary arteriography E MRI chest
The Correct Answer is: D This patient presents with a history and physical exam that is consistent with an acute pulmonary embolus (PE). The prognosis for this type of illness can be serious, and in some cases death can result. The test that is still the gold standard for determination of a PE is the pulmonary arteriogram. CT of the chest with contrast tends to be the more utilized testing modality, but the arteriogram remains the test of choice.
Q 61.3: A 45-year-old man with a history of neck irradiation for Hodgkin lymphoma at the age of 15 is found to have a 1.5-cm, nontender, firm thyroid nodule. Upon laboratory evaluation, the patient is found to be euthyroid, and fine needle biopsy reveals malignancy. What histologic type is most likely? A anaplastic B follicular C medullary D papillary
The Correct Answer is: D Thyroid carcinoma often presents as an asymptomatic thyroid nodule. The most common histologic form is papillary carcinoma, representing more than 80% of cases.
Q 54.12: A patient carrying the diagnosis of asymptomatic multiple myeloma is asking what the best course of therapy and treatment is. Calcium levels are currently at high normal, he is slightly anemic with a hemoglobin of 11.8 (12 to 16), and no new lytic lesions are identified. What is the best treatment at this time? A Allogeneic transplant B Autologous stem cell transplant C Hydration and calcitonin D Observation E Reduced intensity allogeneic transplant
The Correct Answer is: D With patients that are not showing signs of advancing disease or minimal disease, and it is unclear whether paraproteniemia is benign, observation is the preferred therapy. All others are possible treatments with advancing disease, depending on the condition of the patient, advancement of disease, and response to prior therapies.
Q 61.9: A 65-year-old man presents to the emergency department with an acute ischemic stroke. His CT scan is normal. His blood pressure is 180/100 mm Hg. What is the most appropriate treatment for his hypertension? A labetalol (Normodyne) 20 mg IV B nifedipine (Procardia) 10 mg po C nitroprusside (Nipride) drip at 1 mg/kg/min D clonidine (Catapres) 0.1 mg po E no antihypertensive at this time
The Correct Answer is: E *Blood pressure is typically elevated at the time of presentation in acute ischemic stroke. It will decline without medication in the first few hours to days. Aggressively lowering blood pressure in an acute ischemic stroke may decrease the blood flow to the ischemic but salvageable brain tissue. This potentially salvageable brain tissue is referred to as the penumbra. Decreasing blood flow to the ischemic penumbra by acutely lowering blood pressure may result in eventual infarction of this brain tissue. Treatment of previously undiagnosed hypertension should be deferred for several days.* Blood pressure should be treated if there are other indications, such as angina or heart failure. Control of blood pressure is appropriate in patients who are receiving tissue plasminogen activator (t-Pa) for their stroke. Blood pressure should be lowered cautiously to a systolic of less than 185 mm Hg and a diastolic of less than 110 mm Hg. This is thought to decrease the incidence of intracerebral hemorrhage in these patients.
Q 59.7: A 20-year-old college student describes his headaches as unilateral, a 5 out of 10 for pain, and with throbbing, associated with photophobia and often accompanied by nausea and vomiting. He states that the headaches occur two to three times a month and are associated with decreased sleep. He has taken OTC medication with relief and is trying to stay on a sleep schedule. He denies any other neurologic symptoms. Which of the following is the most appropriate next step for managing this patient? A Brain MRI B Hydrocodone-acetaminophen tablets C Physical therapy D Topiramate E Sumatriptan nasal spray
The Correct Answer is: E *Classic migraines often present with episodic, unilateral, and throbbing headache pain, associated with photophobia and phonophobia. Nausea and vomiting may also be involved. Management of migraines includes nonpharmacologic therapies, such as healthy eating, sleep pattern stabilization, caffeine avoidance, and stress relief. If nonpharmacologic therapies are not sufficient, or if symptoms impact activities of daily living, pharmacologic management is indicated.* Medications are available for migraine sufferers and are typically catergorized as abortive or preventive. Abortive therapy for this patient may include nonsteroidal anti-inflammatory medications. Additionally, studies have shown that stimulation of the 5-HT receptors can successfully stop a migraine, and the 5-HT1 receptor agonists, with selective agents (the triptans) often used successfully for headache improvement. Nasal spray formulations are useful for nausea and vomiting. Topiramate, an anticonvulsant, has received FDA approval for migraine prevention therapy in patients with increasing migraine frequency or poor response to abortive therapies. Narcotics are typically avoided for migraine management, and MRI is not warranted without additional clinical signs and symptoms, and other differential diagnoses.
Q 61.8: A 66-year-old female has a chief complaint of vision loss in her left eye. She denies pain and states that this occurred over the past few hours. Her past medical history includes hypertension, high cholesterol, and peripheral vascular disease. Upon funduscopic exam, you note marked hemorrhages in all quadrants and disc edema. The contralateral eye shows only mild hypertensive vascular changes. What is your diagnosis? A Macular degeneration B Retinal detachment C Central retinal artery occlusion D Cerebrovascular accident E Central retinal vein occlusion
The Correct Answer is: E A central retinal vein occlusion is characterized by a "blood and thunder" fundus, with marked hemorrhages, tortuous vessels, and optic disc edema.
Q 62.5: A 35-year-old patient with AIDS has had unintended weight loss of nearly 30 lb over the past 6 months. This loss has been primarily in muscle mass. He has little appetite, but no nausea, diarrhea, or evidence of oral candidiasis. He reports interest in resuming his former weight-training regimen. Which of the following is the most appropriate pharmacologic agent to help him gain muscle mass? A dronabinol B megestrol acetate C odansetron D prochlorperazine E testosterone enanthate
The Correct Answer is: E Anabolic steroids, most commonly testosterone enanthate or cypionate, increase lean body mass in patients with AIDS, particularly those who do weight training. Dronabinol, an antiemetic, and megestrol acetate, a progestational agent, are used to increase appetite and assist in weight gain but have little effect on lean muscle mass. Odansetron and prochlorperazine are both used to treat weight loss caused by nausea of unclear origin in patients with AIDS and are given prior to meals.
Q 54.10: Which of the following statements regarding diabetic medications is/are most correct? A Incretin-mimetics like exenatide commonly result in weight loss. B Thiazolidinediones (TZDs) should be held prior to and for 48 hours after administration of ionidated contrast material. C Sulfonylureas have the highest risk of hypoglycemia of all oral diabetic agents. D Sulfonylureas help preserve beta-cell function. E All of the above.
The Correct Answer is: E Incretin-mimetics act upon the satiety center of the brain and as a result promote weight loss, averaging approximately six pounds. Metformin may cause fatal lactic acidosis when given to patients on concomitantly-administered iodinated contrast dye. Sulfanylureas, in part because of first-pass metabolism, have the highest risk of hypoglycemia of all the oral anti-diabetic agents. TZDs have been found to preserve beta-cell function more so than any other agent.
Q 54.18: A 37-year-old man with a history of Paget disease presents with a deviation of the tongue to the right side upon sticking it out. The right side of the tongue is also observed to have slight atrophy. These symptoms most likely point to a lesion of which of the following cranial nerves? A left hypoglossal nerve B left vagus nerve C left glossopharyngeal nerve D right glossopharyngeal nerve E right hypoglossal nerve
The Correct Answer is: E One of the many clinical manifestations of Paget disease is narrowing of cranial foramina. The 12th cranial nerves (hypoglossal nerves) pass through the hypoglossal canals of the occipital bones. The hypoglossal nerves innervate many of the extrinsic muscles that move the tongue, including the genioglossus, hyoglossus, and styloglossus. Each hypoglossal nerve supplies the ipsilateral extrinsic muscles. Hence, the tongue deviates to the paralyzed side during protrusion because of the actions of the unaffected extrinsic muscles on the other side.
Q 54.9: The organism responsible for most cases of peritonitis in patients on peritoneal dialysis is A Candida albicans B Escherichia coli C Streptococcus pneumoniae D Pseudomonas aeruginosa E Staphylococcus aureus
The Correct Answer is: E S. aureus is the organism responsible for most cases of peritonitis in patients on peritoneal dialysis. Overall, gram-positive organisms are responsible for 50% of cases, and gram-negative organisms cause 15% of cases. Four percent of cases are polymicrobial in nature. Improper technique by the patient in making catheter connections during dialysis exchanges is the reason for bacterial inoculation in most cases. Abdominal pain, fever, and cloudy dialysis fluid are the presenting symptoms and signs.
Q 60.4: A G4P4 woman delivers a viable infant at 38 weeks gestation by normal spontaneous vaginal delivery. The infant has apgars of 7 and 8 (at 1 and 5 minutes respectively). What is the most crucial time for maternal and fetal physiologic changes to occur? A 30 minutes prior to birth B 60 minutes prior to birth C Birth D 30 minutes after birth E 60 minutes after birth
The Correct Answer is: E The hour after birth is the most critical time for physiologic changes to occur, including maternal fluid shifts, hemorrhage, retained placenta, and fetal lung cardiovascular transitions.
Q 54.20: A 32-year-old man who is HIV positive has a seizure. On presentation to the emergency department (ED) he is confused and unsure of what happened. His partner reports that he had been complaining of headache in the days preceding the event. CT scanning of the head demonstrates five peripheral contrast-enhancing lesions. What is the most likely diagnosis? A AIDS dementia complex B central nervous system lymphoma C cryptococcal meningitis D progressive multifocal leukoencephalopathy E toxoplasmosis
The Correct Answer is: E The most common space-occupying CNS lesion in patients with HIV is toxoplasmosis. This condition may present with headache, focal neurologic deficits, seizures, and/or mental status changes. The typical appearance on brain imaging is that of multiple contrast-enhancing lesions in the periphery, particularly the basal ganglia. CNS lymphoma is more typically a single lesion. AIDS dementia complex presents a diagnosis of exclusion, without a characteristic appearance on imaging. The diagnosis of cryptococcal meningitis is made by examination of the spinal fluid, while PML imaging shows nonenhancing white matter lesions without mass effect.
Q 55.7: You have determined that a multiple sclerosis patient with relapsing-remitting disease has not returned to her baseline functional status with her most recent exacerbations, despite her returning to baseline with exacerbations for many years. Her exam also reveals residual neurologic dysfunction. Which of the following multiple sclerosis clinical types would you classify this patient within? A Clinically isolated syndrome B Primary progressive C Progressive/relapsing D Relapsing/remitting E Secondary progressive
The Correct Answer is: E This patient would be classified within the secondary progressive clinical type. Secondary progressive MS follows a relapsing/remitting course initially, during which the patient returns to baseline neurologically and functionally between exacerbations, then evolves into a course of the patient having deterioration in function and neurologic status, which is not associated with acute exacerbations. The patient is also noted to not return to baseline following acute exacerbations. Primary progressive MS is associated with a decline in functional and neurologic status from the onset of disease, and disability is common. Progressive/relapsing MS is associated with progressive neurologic and functional decline from the onset of disease, but with associated exacerbations during the clinical course as well. The designation of clinically isolated syndrome indicates that a patient has had a single clinical event suggestive of MS but does not meet MS diagnostic criteria, thus requiring a waiting period and additional studies.
Q 47.1: Which valve is the most commonly affected in a case of rheumatic heart disease? A Aortic B Mitral C Tricuspid D Pulmonic E Bicuspid aortic valve
The Correct Answer is: B The mitral valve is affected in about 75 to 80% of all cases of rheumatic heart disease. The aortic valve is second, and it is rare that the right-sided valves get diseased.
Q 61.6: What is the most common heart sound that is heard in patients with pulmonary hypertension? A S3 gallop B S2 Split C S4 D Normal S1S2 E Regularly irregular rhythm
The Correct Answer is: B The splitting on the second heart sound is also accompanied by the louder second sound of a split of the pulmonary component. There should not be a change in the rhythm with the patient due to pure pulmonary hypertension.
Q 71.5: What is the most common cause of pneumothorax in a healthy patient? A Traumatic B Infectious C Ruptured bleb D Surfactant abnormality E Malignancy
The Correct Answer is: C Rupture of a bleb is thought to be more relevant to young, thin men, and also patients who have a family history of them, and smoking. (Chesnutt MS, Prendergast TJ. Current Medical Diagnosis and Treatment, 2011, Chapter 9, Pulmonary Disorders)
What is the most common ECG abnormality in patients with a pulmonary embolism (PE)? A Atrial fibrillation B Sinus tachycardia C Ventricular ectopy D Sinus bradycardia E High grade AV block
*The Correct Answer is: B In most cases, sinus tachycardia is the only abnormality in patients with a PE. You may also find some ECGs that will have non-specific ST-T wave changes.* Sinus bradycardia and AV blocks are not common findings that are associated with PE.
An 18 month infant with congenital heart disease is diagnosed with acute bronchiolitis secondary to respiratory syncytial virus. Which of the following therapies should be initiated? A Albuterol B Amoxicillin C Azithromycin D Prednisone E Ribavirin
*The Correct Answer is: E High-risk infants (i.e., congenital heart disease) who develop RSV are eligible for treatment with ribavirin.* Antibiotics (B, C) are indicated if secondary bacterial infections develop. Albuterol (A) and prednisone (D) haven't been shown to improve RSV-related bronchiolitis.
A patient presents to the emergency department with signs and symptoms of acute pancreatitis. He does not drink alcohol and has no history of gallstones. He has a positive history of increased lipids. Very high levels of which of the following lipids is associated with an increased risk of pancreatitis? A High density lipoprotein B Low density lipoprotein C Apolipoprotein C D Total cholesterol E Total triglycerides
*The correct choice is E, total triglycerides. Very high levels of triglycerides increase the risk of acute pancreatitis, probably from local release of free fatty acids and lysolecithin from lipoprotein substrates in the pancreatic bloodstream.* If the albumin binding capacity is exhausted, these fatty acids can initiate a chemical pancreatitis and destroy parenchymal cells in the area.
A 76-year-old man with a history of three myocardial infarctions, CABG x 4, COPD, HTN, and hyperlipidemia presents to the emergency department with complaints of increasing dyspnea with exertion. He now has to sit in his recliner to sleep. He complains of lower extremity edema, now above the knees. On physical examination, he exhibits jugular venous distention to the angle of the jaw at 90 degrees of truncal elevation, diffuse bilateral rales on auscultation of the lung, and a blowing systolic murmur that is heard best at the left lower sternal border. Which abdominal physical exam finding would be most likely given this patient's history? A Periumbilical ecchymoses B Murphy's sign C Tenderness at McBurney's point D Hepatomegaly E Absent bowel sounds
Choice D is correct, as the patient is demonstrating signs and symptoms of congestive heart failure, which can lead to hepatic congestion and hepatomegaly. Choice A, periumbilical ecchymoses, or Cullen's sign, are representative of hemoperitoneum. Choice B, Murphy's sign, is indicative of cholecystitis. Choice C, tenderness at McBurney's point, is frequently seen with appendicitis. Choice E, absent bowel sounds, is seen in patients with ileus.
Q 45.3: In a 4-year-old female child who presents with "toeing in," which of the following is the likely etiology? A femoral anteversion B genu valgum C genu varum D tibial torsion
The Correct Answer is: A "Toeing in" in children before the age of 2 is typically due to tibial torsion; however, any "toeing in" after the age of 2 to 3, is usually due to femoral anteversion. The femur has more internal rotation that results in the presentation. Genu varum is known as bowleg and genu valgum is known as knock-kneed.
Q 69.6: What is the most common pulmonary complaint of a patient with tuberculosis? A Hemoptysis B Chronic cough C Wheezing D Dyspnea E Chest pain
The Correct Answer is: B Chronic cough is the most common complaint. Hemoptysis can occur but is rare, and a productive cough becomes more prevalent after the chronic cough has been established for a while. (Chesnutt MS, Prendergast TJ. Current Medical Diagnosis and Treatment, 2011, Chapter 9, Pulmonary Disorders)
Q 68.5: Which of the following conditions is a cause for central vertigo? A Meniere syndrome B Labyrinthitis C Vestibular neuronitis D Acoutic neuroma E Perilymphatic fistula
The Correct Answer is: D Meniere syndrome, labyrinthitis, vestibular neuronitis, and perilymphatic fistula are causes of peripheral vertigo. Acoustic neuroma, or eight cranial nerve schwannomas, are among the most common intracranial tumors, and a cause for central vertigo. (McPhee et al., 2011, Chapter 8)
Q 91.8: A 41-year-old male appears pale and mildly icteric. He complains of increasing fatigue lasting several weeks. Laboratory findings note an elevated MCV of 133. What other physical finding would most support a diagnosis of megaloblastic anemia? A Anorexia B Dementia C Diarrhea D Glossitis E Paresthesias
The Correct Answer is: E While all choices may be clinical findings in a vitamin B12 deficiency, paresthesias most supports the diagnosis. Peripheral nerves are usually affected first and patients initially complain of paresthesias. The other findings are later in the disease process or appear with very advanced anemia. (McPhee SJ, Papadakis MA, Tierney LM. Current Medical Diagnosis and Treatment, 2010, Chapter 13, Blood Disorders)
Q 55.4: A 63-year-old female 3-day's postoperative thyroidectomy complains of tingling around her mouth and feet, and muscle spasms. You note the following finding on exam. What is the most likely cause of her symptoms and physical examination findings? Source: (McPhee and Papadakis, 2011, Chapter 26) A hypothyroidism B hypoparathyroidism C hyperkalemia D hyperparathyroidism E hypocalcemia
B The correct answer is (B). The patient's recent history of thyroidectomy, together with her symptoms, are suggestive of hypocalcemia due to hypoparathyroidism. Hypoparathyroidism is most commonly caused by removal of the parathyroids during thyroidectomy. The picture above is known as Chvostek's sign, which is suggestive of hypocalcemia. The other choices listed in this question would not cause Chvostek's sign. Hyperparathyroidism causes hypercalcemia.
Q 59.9: A 33-year-old man with HIV infection is brought in by his partner for evaluation of altered mental status. The partner has noticed waxing and waning periods of confusion throughout the day, difficulty in performing tasks such as balancing a checkbook, and deterioration of handwriting. The patient reports no fever or headache. What is the most likely diagnosis? A AIDS dementia complex B central nervous system lymphoma C cryptococcal meningitis D progressive multifocal leukoencephalopathy (PML) E toxoplasmosis
The Correct Answer is: A AIDS dementia complex is the most common cause of mental status changes in patients with HIV infection. The deterioration of handwriting is often an early manifestation. Difficulty in performing cognitive tasks and diminished motor speed are typical, as is the waxing and waning of manifestations of dementia. Patients with central nervous system lymphoma and toxoplasmosis present with headache, focal neurologic deficits, seizures, and/or altered mental status. Patients with cryptococcal meningitis have, most typically, headache and fever, but fewer than 20% have meningismus. They also usually have normal mental status. Patients with PML have primarily focal neurologic deficits such as aphasia, hemiparesis, and cortical blindness.
Q 56.4: Which of the following etiologies is the most common cause of nontraumatic subarachnoid hemorrhage (SAH)? A Cerebral aneurysm B Poorly controlled hypertension C Anticoagulant use D Arteriovenous malformation
The Correct Answer is: A Congenital cerebral aneurysms or Berry aneurysms account for 75% to 80% of nontraumatic subarachnoid hemorrhages (SAHs). Poorly controlled hypertension and anticoagulant use are more commonly associated with intracerebral hemorrhages (ICH). AVMs can cause either SAH or ICH.
Q 62.9: A 68-year-old male presents with the complaint of palpitations in the center of his chest over the last few hours. The symptoms come and go, and last anywhere from 30 seconds to a few minutes. There is some associated lightheadedness with these episodes, and occasionally there is mild shortness of breath. While examining the patient, he has another episode. During this time, the patient's vital signs reveal a pulse of 170 and a blood pressure of 118/69. Based on this history and the findings on the exam and rhythm strip shown, what is the best treatment for this patient? A Diltiazem B Digoxin C Lisinopril D Metoprolol E Amiodarone
The Correct Answer is: E This patient is having non-sustained ventricular tachycardia (NSVT). Based on the clinical presentation, this patient can benefit from an infusion of amiodarone. The dose is normally a 150mg bolus, followed by a drip of 1mg/min for the first 6 hours, then followed by 0.5mg/min for the next 18 to 24 hours. The other agents would not be able to maintain a rhythm for this type of abnormality.